You are on page 1of 1158

Bertyn yo naTodi3zionorii

Among the following choose the direction in etiology according to which disease has only one
reason and its influence causes a disease obligatory:
{=Monocausalism
~Psychosomatics
~Conditionalism
~Polyetiologism
~Constitutionalism}

It is diagnosed trisomy of 13 chromosome during investigation of karyotype of newborn boy with


defects “harelip” and “cleft palate”, defects of the nervous and cardiovascular systems, organ of
vision. Which from the following syndromes is present in a boy?
{=Patau syndrome
~Kleinfelter’s syndrome
~Tumer’s syndrome
~Edvards syndrome
~Down’s syndrome}

It is known that hemeralopia is inherited on X-linked recessive inheritance type. What is the
probability of hemeralopia appearance in newborn boy when it is known that the father of pregnant
woman suffers with the hemeralopia?
{=50 %
~0 %
~15 %
~100 %
~25 %}

The sickle-cell sharp of erythrocytes is dominant sign. The normal HBA is synthesized in
heterozygote as well as HBS. Hemolysis of erythrocytes begins only in the conditions of hypoxia.
The expressed sickle-cell anemia arises in homozygote. What type of inheritance is typical for this
illness?
{=Incomplete dominant
~Autosomal-recessive
~X-linked
~Y-linked
~Autosomal-dominant}

The inherited defect of lysosomes formation in phagocytes is typical for


{=Chediak-Higasy’s syndrome
~Down’s syndrome
~Alder’s syndrome
~Chronic granulomatosis
~Congenital neutropenia}

What type of diathesis is characterised by the hypersensitivity of an organism to the biological


active substances?
{=Exudative-catarrhal
~Lympho-hypoplastic
~Asthenic
~Nervous-arthritic
~Hypoergic }
A child was born with numerous defects of external and internal organs - heart, kidney, digestive
system in a maternity hospital. Down’s syndrome was diagnosed. What method of investigation
may confirm this diagnosis?
{=Cytogenetic method
~Biochemical method
~Genealogical method
~Population-statistical method
~Twins’s method}

A father has a phenylketonuria, mother is healthy. A type of inheritance is autosomal-recessive. The


probability of appearance of phenylketonuria in their children is:
{=0 %
~50 %
~15 %
~100 %
~25 %}

A father has congenital cataract, mother is healthy. The type of inheritance is autosomal-dominant.
What is the probability of congenital cataract appearance in their children?
{=50 %
~33 %
~15 %
~25 %
~0 %}

Prolongate using of the glucocorticoides represses mononuclear phagocytes system activity mainly
as a result of
{=Deprerssion of lysosomes activity
~Oppressing of glycolisis
~Blocking of mytochondrias
~Activation of catabolism
~Oppressing of Krebs cycle}

AJIEPI TA
Patient with immunodeficit was treated by medicine, which stimulates the humoral link of
immunity. What changes in the patient’s organism will reflect the adequate reaction of the immune
system on medical influence?
{=Increase of antibodies level
~Activation of NK-cells
~Change of leucocyte formula
~Complement system activation
~Increase of leucocytes amount}

In 2 minutes after the bite of bee the edema developed in a boy. Concentration of antibodies in
blood did not change. What is the name of such organism reaction?
{=Pseudoallergy
~Autoallergy
~Anaphylaxy
~Toxemia
~Sensitization}

What primary immunodeficit is characterized by eczema and thrombocytopenia?


{=Viskott-Oldrich’s syndrome
~Lui-Barr’s syndrome
~Bruton’s disease
~Di Georgy’s syndrome
~Nezelof’s syndrome}

Turosi NaToOriaHi Hpowecn


The process of blood clot dissolution due to activation of fibrinolytic blood system and proteolytic
enzymes of leucocytes is named:
{=Aseptic autolysis
~Petrifaction
~Organization
~Recanalization
~Thromboembolism}

To the main factors of thrombformation does not belong


{=Decrease of blood oxygenation
~Decrease of blood stream speed
~Local angiospasm
~Adhesion of thrombocytes
~Aggregation of thrombocytes}

What is the trigger factor of thrombosis origin at the infectious disease?


{=Damage of vascular wall by endotoxin of microorganisms
~Decrease of anticoagulants activity
~Hemolysis of red corpuscles as a result of damage of their membranes by toxins
~Increase of procoagulants activity at infection
~Depression of fibrinolytic systems by the microorganisms toxins}

What type of embolism does arise in a patient with the thrombosis of lower extremities veins more
frequently?
{=Embolism of pulmonary artery
~Embolism of coronary artery
~Embolism of pulmonary vein
~Embolism of renal artery
~Embolism of portal vein}

What types of local blood circulation violation in an abdominal cavity does take place during
quickly removal of ascitic fluid?
{=Arterial hyperemia
~Embolism
~Ischemia
~Venous hyperemia
~Thrombosis }

When does the embolus pass from the right part of heart to the left part of heart?
{=Ventricular septal defect
~Pulmonary artery stenosis
~Aortic stenosis
~Bacterial endocarditis
~Mitral valvular insufficiency}

Why does the rapid redistribution of blood as a result of ascitic fluid removal is dangerous for a
brain?
{=Development of brain hypoxia
~Development of brain vessels thrombosis
~Development of stasis in brain vessels
~It is high risk of cerebral hemorrhage
~Development of liquors hypooncia}

That does testify to development of tumour progression during transplantation the animal of Erlich
karcinoma?
{=Insensibility to cytostatics
~Increasing weight of tumour
~Anaplasia
~Unhampered growth
~Infiltrative growth}

The major biochemical feature of tumour cell is


{=Activating of synthesis of nucleic acids
~Activating of synthesis of lipoprotein
~Oppressing of synthesis of nucleic acids
~Activating of synthesis of phosphotides
~Activating of synthesis of tricarboxylic acids}

It doesn’t concern to lysosomal factors which take part in inflammation development


{=Phospholipase A2
~Unenzymic cationic proteins
~Free radicals
~Acidic hydrolase
~Peroxidase}

Neoangiogenesis in termination of inflammation is stipulated by migration of


{=Endotheliocytes
~Fibroblastes
~Monocytes
~Limphocytes
~Granulocytes }

From the pleural cavity of the patient was received exudate the next content: protein — 34 g/l, cells -
3600/mcl, prevail neutrophils, pH — 6,8. What is kind of exudate in the patient?
{=Purulent
~Fibrinous
~Hemorrhagic
~Serous
~Putrefactive}

In the patient 28-years-old after opening of furuncle on the hand there was rapid healing of wound
without formation of scar. Specify, what cells play the main role in the process of proliferation?
{=Fibroblasts
~Neutrophils
~Eosinophils
~Lymphocytes
~Monocytes }

In the patient is revealed redness of gingive and edema due to mechanical irritant by acryl set of
removable false teeth prosthesis in oral cavity was found. The patient complains on pain and
impossibility of chewing. What type of inflammation did arise in this case?
{=Exudative
~Alternative
~Proliferative
~Catarrhal
~Purulent}

In the patient through few hour after burn the in the area of hyperemia and edema of skin was
appeared focus of necrosis. What main mechanism provide strengthening of destructive phenomena
in the area of inflammation?
{=Secondary alteration
~Primary alteration
~Diapedesis of erythrocytes
~Emigration of lymphocytes
~Proliferation of fibroblastes}

In the patient with acute pulpite appeared painful of teeth and edema of lower part of face in side of
illness tooth. What is leading mechanism of edema development in that case?
{=Microcirculation disorders in injury area
~Increase production of aldosterone
~Trophical function of neurus system disorder
~Nervous regulation of water-electrolyte disorder
~Hypoproteinemia}

In the patient with gout is often determined increase and deform joints as a result of inflammation.
What type of inflammation laies in basic of these changes?
{=Proliferative
~Alternative
~Exudative
~Fibrinous
~Croupous }

In the patient with phlegmon of the arm is sowed streptococci. What cells will be prevail in the
exudate?
{=Neutrophils
~Eosinophils
~Lymphocytes
~Basophils
~Monocytes }

Mechnikov LI. studying an inflammatory process, described certain lawfullness of leucocytes


emigration to the area of inflammation. Cells emigrate in such sequence:
{=Neutrophils , monocytes, lymphocytes
~Monocytes, lymphocytes, neutrophils
~Neutrophils, lymphocytes, monocytes
~Monocytes, neutrophils, lymphocytes
~Lymphocytes, monocytes, neutrophils}

The girl 7-years-old ill by diphtheria and died for asphyxia on the third days. On autopsy the
mucous membrane of trachea and bronchus is thickened, edematous, tarnished is covered, greyish
film which are not easily taken of. What type of inflammation morphological changes testify about?
{=Croupous
~Diphtherous
~Hemorrhagic
~Serous
~Catarrhal}
The patient with arthritis complains to pain in joints, edema and redness their. Activity of what
enzyme necessary to reseach for determine of disease nature?
{=Hyaluronidase
~Creatinkinase
~Alkalic phosphatase
~Acidic phosphatase
~Urease}

Defects in neutrophil NADPH oxidase system produce:


{=Chronic granulomatous disease.
~Chediak-Higashi disease.
~Leukocyte adhesion deficiency.
~Hashimoto's disease.
~Streptococcal infection. }

Paroxysmal nocturnal hemoglobulinuria results from deficiency in:


{=M yeloperoxidase.
~Decay accelerating factor. (DAF).
~Classical pathway C components.
~C1 inhibitor.
~C8}

X-linked agammaglobulinemia results from a mutation in:


{=A tyrosine kinase gene
~The CHTA promoter protein.
~An HLA gene.
~CD40L (CD154)
~IFNU receptor}

Mutations in the gamma, chain of the receptors for IL-2, 4, 7, 9 and 15 lead to:
{=Severe combined immunodeficiency (SCID)
~Bare lymphocyte syndrome.
~Hyper-IgM syndrome.
~Reticular dysgenesis
~Build-up of toxic nucleotide metabolites.}

Poor skin tests to a range of microbial antigens such as tuberculin and mumps indicate a deficiency
of:
{=NK cells.
~T-cells.
~B-cells.
~Phagocytosis.
~Opsonization. }

A coreceptor for HIV is:


{=CXCR4
~CD8
~CD54
~CR5
~RANTES}

Which of the following is not helpful in the diagnosis of AIDS:


{=CD8 numbers.
~Skin tests to bacterial antigens.
~Lymph node biopsy.
~Serum p24 antigen.}

Which of the following HIV antigens provides a potential target for neutralizing antibody:
{=gp120
~U3
~Reverse transcriptase
~Protease
~Fat}

Which of the following have not provided examples of secondary immunodeficiency:


{=High fat diet
~Lymphoproliferative disorders.
Cytotoxic drugs.
~Viral infection
~Low iron diet.}

Which one of the following mast cell products is not preformed and therefore has to be newly
synthesized?:
{=Prostaglandin D2
~Histamine.
~Heparin.
~Neutral protease.
~Eosinophil chemotactic factor (ECF).}

The term reactive lysis usually refers to a sequence of events involving:


{=Complement.
~Cytotoxic T-lymphocytes (CTL).
~Antibody-dependent cellular cytotoxicity (ADCC).
~Th! cells.
~Natural killer (NK) cell}

Maple bark stripper’s disease is a hypersensitivity largely affecting the


{=Lung
~Kidneys.
~Nervous system.
~Skin
~Platelets. }

Cronic granuloma represents in the body


{=Wall off a site of chronic infection.
~Make a site of chronic infection accessible.
~Digest antibody-antigen complexes.
~Initiate an immune response.
~Change from a Th! to a Th2 type of response.}

In thyroid autoimmunity, an antibody causing type V hypersensitivity may be present and is


directed against:
{=Thyroid stimulating hormone (TSH) receptor
~Thyroid peroxidase.
~Thyroglobulin
~Acetylcholine receptor.
~Thyroxine. }
Which type of hypersensitivity cannot be transferred with serum antibody?:
{=Type IV
~Type II
~Type III
~Type I
~Type V}
Rhesus hemolytic disease of the newborn involves:
{=Antibody to cell surfaces
~IgE
~Soluble immune complexes
~Cytokine release from T-cells
~Stimulatory antibodies}

The Arthus reaction is characterized by an intense infiltration by:


{=Neutrophils
~Mast cells
~Eosinophils
~Macrophages
~Langerhans' cells}

The major effector molecules involved in type IV hypersensitivity reactions are:


{=Cytokines
~Complement components.
~Antibodies
~Prostaglandins.
~5-hydroxytryptamine (5-HT).}

The immunosuppressive drug which probably attacks DNA by alkylation and cross-linking is:
{=Cyclophosphamide.
~Azathioprine.
~Cyclosporine.
~Rapamycin.
~Prednisone. }

Hyperacute graft rejection is caused by:


{=Preformed antibody.
~CD4 lymphocytes.
~CD8 lymphocytes.
~Platelets.
~Circulating immune complexes.}

An example of a known oncogenic virus is:


{=Epstein-Barr virus.
~HIV-2.
~Herpes zoster
~Vesicular stomatitis virus.
~Proteus mirabilis.

In pancreatic carcinoma the ras gene:


{=Contains a single point mutation, but not always at the same position.
~Is normal but is overexpressed.
~Has a large deletion.
~Contains a single point mutation, always at the same position.
~Is absent.}

Chronic lymphocytic leukemia:


{=Is usually found in people over 50 years of age.
~Has a good prognosis only in those patients with circulating monoclonal immunoglobulin.
~Is mostly a disease of childhood.
~Usually has a very poor prognosis.
~Is a leukemia where both kappa and lambda light chains are found on the surface of the malignant
cell.}
KPOB, CEPIE
A patient applied to the doctor with complaints of hypodermic hemorrhage at insignificant
mechanical traumas. What from enumerated below can be reason of such phenomenon?
{=Thrombocytopenia
~Leukopenia
~Decrease of hemoglobin concentration
~Erythropenia
~Lymphocytosis }

A patient applied to the policlinic with complaints of general weakness, dyspnea. Shortly before it
he accepted levomycetin for the prophylaxis of intestinal infection. In blood: erythrocytes —
1,9x10! /l, Hb — 58 g/l, colour index— 0,9, leukocytes — 2,2x10°/1. What type of anemia does it
testify about?
{=Hypoplastic
~Tron deficiency
~Hemolytic
~Metaplastic
~Aplastic}

A patient with paradontosis takes antimicrobial drugs during the long time. Results of blood test:
erythrocytes — 2,.x107/1, hemoglobin — 60 g/l, colour index — 0,85, leukocytes — 2,5x10°/1. What
type of anemia has this patient?
{=Hypoplastic
~Metaplastic
~Tron deficiency
~Hemolytic
~Protein deficiency anemia}

A woman 55 years old suffers from thyrotoxicosis. Results of blood test: amount of erythrocytes —
5,9x10!” /l, content of hemoglobin — 171 g/l, colour index — 0,9, amount of leukocytes — 4.9x10°/1.
How these changes of blood are named?
{=Absolute erythrocytosis
~Hypoplastic anemia
~Absolute leukocytosis
~Leukemoid reaction
~Relative leukopenia}

In a patient tumor of head of pancreas was revealed which is accompanied with steatorrhea and
hemorrhage in skin. Concentration of prothrombin and other factors of coagulation is diminished.
What Is the mechanism of hemorrhage origin in this case?
{=Avitaminosis K
~Avitaminosis C
~Deficit of trypsin
~Cholemia
~Deficit of pepsin}

A man appealed to the doctor with complaints of appearance of hypodermic hematomas. In the
blood test: erythrocytes — 2,5x10'/1; hemoglobin — 80,0 g/l; colour index — 0,9. Thrombocytes —
40,0x10°/1. Leucocytes — 5,8x10°/1. Leucocytic formula: basophils — 5 %, eosinophils — 15 %,
myeloblasts — 6 %, myelocytes — 10 %, metamyelocytes — 18 %, stab neutrophils — 26 %,
segmentonucleonic neutrophils — 10 %, lymphocytes — 8 %, monocytes — 2 %. What pathology is in
the patient?
{=Chronic myelocytic leucosis
~Leukemoid reaction
~Acute myeloblastic leucosis
~Basophilic leucocytosis
~Eosinophilic leucocytosis}

A patient 33 years old was admitted to the hospital with an abscess of femur. Leukogram:
leucocytes — 20,0x107/1, basophils — 0 %, eosinophils — 1 %; neutrophils: myelocytes — 5 %,
metamyelocytes — 10 %, stab neutrophils — 12 %, segmentonuclear neutrophils — 60 %,
lymphocytes — 10 %, monocytes — 2 %. This leukogram testifies about
{=Leukemoid reaction
~Shift of leukocyte formula to the left
~Acute myeloblastic leucosis
~Degenerative shift to the left
~Chronic myelocytic leucosis}

A patient 40 years old was operated for acute phlegmonous appendicitis. Amount of leucocytes —
12,0x10°/1. In the smear of blood: basophils — 0 %, eosinophils — 2 %, myelocytes — 0 %,
metamyelocytes — | %, stab neutrophils — 35 %, segmentonucleonic neutrophils — 50 %,
lymphocytes — 10 %, monocytes — 2 %. Stab neutrophils ere with piknosis of nucleus, in some from
them there are toxogenic granulas. How such change of leucocytic formula is named?
{=Regenerative-degenerative shift to the left
~Hyperregenerative shift to the left
~Regenerative shift to the left
~Leukemoid reaction
~Nucleonic shift to the right}

A patient appealed to stomatologist with complaints of hemorrhagic from gingiva. Blood test:
erythrocytes — 3,6x10'7/1, hemoglobin — 120 g/l, colour index — 1,0; leucocytes — 56,0x10°/1,
myeloblasts — 3 %, promyelocytes — 2 %, myelocytes — 3 %, metamyelocytes — 1 %, stab
neutrophils — 12%, segmentonucleonic neutrophils — 4 %, eosinophils — 8 %, basophils — 8 %,
lymphocytes — 16 %, monocytes — 3 %, thrombocytes — 30x10°/1. What pathology of blood is in the
patient?
{=Chronic myelocytic leucosis
~Leukemoid reaction
~Chronic monocytic leucosis
~Chronic erythromyelosis
~Chronic eosinophilic leucosis}

At the analysis of leukogram of a patient was revealed: amount of leucocytes — 250,0x10°/1,


leucocytic formula: myeloblasts — 1 %, promyelocytes — 2 %, myelocytes — 16 %, metamyelocytes
— 18 %, stab neutrophils — 20 %, segmentonucleonic neutrophils — 15 %, basophils — 8 %,
eosinophils — 9 %, lymphocytes — 9 %, monocytes — 2 %. What type of chromosomal anomaly may
leads to illness in this patient ?
{=Translocation of segment of 22th chromosome on 9th
~Translocation segment of 8th chromosome on 14th
~Inversion segment of 7th chromosome
~Inversion segment of 15th chromosome
~Trisomia on a 8th pair}

Blood test of child: amount of leucocytes — 20,0x10°/; basophils — 1 %, eosinophils — 16 %, stab


neutrophils — 3 %, segmentonucleonic neutrophils — 60 %, lymphocytes — 18 %, monocytes — 2 %.
These results testify about presence
{=Eosinophilic leucocytosis
~Chronic myeloid leucosis
~Leukemoid reaction
~Chronic lymphoid leucosis
~Neutrophilic leucocytosis }

In a patient at examination of peripheral blood there are: hemoglobin — 80 g/l, erythrocytes —


3,2x10'7/1, leukocytes — 3,5x 10°/1. Leycocytic formula: basophils — 5 %, eosinophils — 9 %,
myeloblasts — 3 %, promyelocytes — 8 %, myelocytes — 11 %, metamyelocytes — 22 %, stab
neutrophils — 17 %, segmentonucleonic neutrophils — 19 %, lymphocytes — 3 %, monocytes — 3 %.
What pathology is in the patient the most likely?
{=Chronic myelocytic leucosis
~Acute promyelocytic leucosis
~Chronic erythroblastic leucosis
~Acute myeloblastic leucosis
~Chronic eosinophilic leucosis}

In a patient is increase of lymphatic nodes, hepatosplenomegalia. Hemoglobin — 80 g/l, leucocytes —


90,0x10" /, lymphocytes — 75 %, ESR — 35 mm/hour. In the smear of peripheral blood there are a
lot of Gumprecht’s shadow. What disease is in a patient?
{=Chronic lymphocytic leucosis
~Acute lymphoblastic leucosis
~Acute myeloblastic leucosis
~Pernicious anemia
~Tron deficiency anemia}

In a patient with the chronic abscess of lungs there are leucopenia, increase of segmentonucleonic
leucocytes, increase of stab neutrophils, metamyelocytes is absent. Among of stab there are
leucocytes with vacuolated cytoplasm, toxogenic granulas, Dele’s bodies. How shift of leucocytic
formula is named in this patient?
{=Degenerative
~Regenerative
~Hyperregenerative
~Hyporegenerative
~Regenerative-degenerative}

In a woman after days after abortion neutrophilic leukocytosis with regenerative shift to the left was
revealed. What is the mechanism of its development?
{=Increase of leukopoiesis
~Redistribution of leukocytes in vessels
~Decrease of leukocytes destruction
~Delay of leukocytes emigration
~Transport of leukocytes from tissues in vessels}

In a worker which worked with benzol the amount of leukocytes in blood is 1,0x10°/1. Leucocytic
formula: basophils — 0 %, eosinophils — 1 %, stab neutrophils — 0 %, segmentonucleonic neutrophils
— 15 %, lymphocytes — 60 %, monocytes — 24 %. What variant of pathology of leucocytes is in this
case?
{=Agranulocytosis
~Aleukia
~Monocytosis
~Neutrophilia
~Lymphocytosis }

In the blood of a patient there are leucocytosis, sharp increase of amount of stab granulocytes and
metamyelocytes, myelocytes and promyelocytes. What is type of nuclear shift in the patient?
{=Hyperregenerative shift to the left
~Regenerative shift to the left
~Nucleonic shift to the right
~Degenerative shift to the left
~Regenerative-degenerative shift to the left}

In the Itsenko-Cushing’s illness which is characterized by surplus formation of adrenocorticotropic


hormone, is arose
{=Eosinopenia
~Neutropenia
~Lymphocytopenia
~Basophilia
~Monocytosis}

In the purulent endometriris is arose leucopenia in base which lie


{=Secretion of leucocytes from an organism
~Inhibition of leucopoiesis
~Destruction of leucocytes in blood
~The lysis of leucocytes in spleen
~Redistribution of leucocytes to internal organs}

Leucosogenic viruses which get to the cell cause leucosis because they
{=Activate cellular oncogene
~Disorder process of transcription
~Disorder process of translation
~Stimulate formation of energy
~Activate lysosomal enzymes}

Nuclear Gumprecht’s shadow - it is:


{=Destroyed lymphocytes
~Rest of nucleus in megalocytes
~Basophilic including in leucocytes
~Rest of nuclear membrane
~Alkalized erythrocytes}

Simultaneous increase of basophils and eosinophils (basophilic-eosinophilic association) are


characterized for
{=Chronic myelocytic leucosis
~Chronic lymphocytic leucosis
~Chronic erythroblastic leucosis
~Acute myeloblastic leucosis
~Acute megacaryoblastic leucosis}

The decrease of thyroid function (hypothyroidism) is accompanied


{=Basophilia
~Eosinophilia
~Neutrophilia
~Lymphocytosis
~Monocytosis}

The hereditary neutropenia is a result


{=Defect of neutrophils differentiations
~Intravascular leucolysis
~Delay of leucocytes in depot
~Draw of leucocytes to mucous membranes
~Destruction of leucocytes in spleen}

The philadelphian chromosome formates as a result


{=Translocation of segment 22th chromosome on 9th
~Translocation of segment 9th chromosome on 22th
~Translocation of segment 21th chromosome on 22th
~Translocation of segment 21th chromosome on 14th
~Translocation of segment 21th chromosome on 15th}

CEPLE
Choose the correct sequence of organism reactions, which represents pathogenesis of edema
development at heart insufficiency.
{=Slow down of blood flow - exit of blood liquid out the vessels - activation of suprarenal glands -
releasing of aldosterone - retention of sodium
~Slow down of blood flow - increase of diuresis - activation of suprarenal glands - releasing of
vasopressin - retention of water
~Slow down of blood flow - activating of sympathetic-adrenal system - releasing of adrenalin -
retention of calcium
~Slow down of blood flow - activation of hypophysis - releasing of thyroxin retention of sodium
and calcium
~Slow down of blood flow - exit of blood liquid out the vessels - activation of thyroid glands -
retention of iodum}

Choose the indexes, which characterize the stage of exhaustion of heart hypertrophy
{=Diminishing of activity of cardiomyocytes genetical structures, diminishing of proteins synthesis,
accumulation of calcium
~Increase of activity of cardiomyocytes genetical structures, diminishing of proteins synthesis,
accumulation of sodium
~Increase of activity of cardiomyocytes genetical structures, activating of proteins synthesis,
accumulation of hydrogen
~Diminishing of activity of cardiomyocytes genetical structures, diminishing of nucleic acids
concentration, deficit of calcium
~Diminishing of activity of cardiomyocytes genetical structures, increase of nucleic acids
concentration, deficit of calcium}

Choose the sequence of reactions, which represent pathogeny of acid-base balance violation at heart
insufficiency in the stage of decompensation.
{=Hypoxia - glucolysis activation - surplus synthesis of lactic acid - acidosis
~Hypoxia - glucolysis activation - increase synthesis of adenosinthreephosphate acid - acidosis
~Hypoxia - activating of glucogenesis - increase synthesis of glucogen - alkalosis
~Hypoxia - activating of lipolysis - surplus synthesis of fat acids - alkalosis
~Hypoxia - activating of gluconeogenesis - disintegration of glucogen — acidosis}

Contractile damage of cardiomyocytes in zone of ischemia at development of reperfusion syndrome


is the result of
{=Accumulations of calcium in cardiomyocytes
~Accumulations of lactic acid in cardiomyocytes
~Deficit of magnesium in blood
~Deficit of glucose in blood
~Accumulation of sodium in cardiomyocytes}

Development of sinus tachycardia is the result of pace-makers activity change, namely


{=Increase of slow diastole depolarization speed, decrease of crutical threshold potential
~Decrease of slow diastole depolarization speed, increase of crutical threshold potential
~Decrease of slow diastole depolarization speed, decrease of crutical threshold potential
~Increase of slow diastole depolarization speed, increase of crutical threshold potential
~Increase of slow diastole depolarization speed, hyperpolarization}

Dispnoe, tachycardia, cyanosys of visible mucous membranes appeared in sick with myocardium
infarction that testified hypoxia in t he result of heart insufficiency. These signs are caused by
{=Reducing of blood flow speed
~Reducing of red blood cells number
~Insufficient concentration of haemoglobin
~Insufficient concentration of oxigene in a blood
~Violation of oxyhemoglobin dissociation}

For diagnose of sinus arrhythmia it is necessary to discover on an electrocardiogram


{=Difference of R-R duration = 0,2 sec, wave P is placed before QRS, complex QRS is not changed
~Difference of R-R duration = 0,1 sec, wave P is placed before QRS, complex QRS is not changed
~Difference of R-R duration = 0,25 sec, wave P is placed after QRS, complex QRS is not changed
~Difference of R-R duration = 0,45 sec, wave P is placed after QRS, complex QRS is deformed
~Difference of R-R duration = 0,15 sec, wave P is placed before QRS, complex QRS is deformed}

It is known that compensatory hyperfunction (it is essence of emergency phase of heart


insufficiency) conduces for cardiomyocites hypertrophy. It is happened because so-called ,,limited
mechanisms”, which cause depression of cardiomyocites contractility. A leading role in
pathogenesis of this phenomenon belongs to
{=Deficit of energy and accumulation of hydrogene ions
~Deficit of energy and accumulation of calcium ions
~Activation of nuclea and surplus of sodium ions
~Activation of glucolysis and accumulation of calcium ions
~Deficit of energy and accumulation of sodium ions}

It is proved that appearance of the state which has name ,,hibernal myocardium” can be
complication of myocardium infarction. This state is characterized by the changes of functional
descriptions of heart, namely by
{=Reducing of pumping function in condition of rest without cardiomyocytes cytolysis
~Reducing pumping function at the physical loading with cardiomyocytes cytolysis
~Increase of pumping function at the increase of adrenalin level without cardiomyocytes cytolysis
~Reducing of pumping function in the condition of stress with cardiomyocytes cytolysis}

Increase of pumping function in the conditions of rest with cardiomyocytes cytolysis


What changes of pace-makers activity do cause development of sinus bradycardia?
{=Decrease of slow diastole depolarization speed, increase of crutical threshold potential
~Decrease of slow diastole depolarization speed, decrease of crutical threshold potential
~Increase of slow diastole depolarization speed, increase of crutical threshold potential
~Increase of slow diastole depolarization speed, decrease of crutical threshold potential
~Increase of slow diastole depolarization speed, hyperpolarization}

What is the main in formation of ectopic areas of excitation in heart?


{=Damage of the conducting ways, electric unhomogenity of myocardium, depression of sinus node
~Anomaly of the conducting ways, hypoxia of myocardium, activation of sinus node
~Additional conducting ways, activation of sinus node, depression of atrium-ventricular node
~Activation of sinus node, activation of atrium-ventricular node, additional conducting ways
~Activation of sinus node, anomaly of the conducting system}
What properties of modified lipoproteins do conduce for development of atherosclerosis?
{=They increase of vessels permeability, activate macrophages; stimulate smooth musclle cels
proliferation
~They co-operate with apoB,E receptors, promote of hyperlipoproteinemia, activate of
endotheliocytes
~They penetrate easily through the vessel wall, promote formation of connective tissue, well co-
operate with apoC receptors
~They activate thrombocytes and phagocytes, activate the synthesis of apoproteins
~They oppress of endotheliocytes proliferation, cause damage of vascular wall, oppress activity of
phagocytes }

What is the main in pathogenesis of Dresler’s syndrome, which arises after carried myocardium
infarction?
{=Organism sensitization by the myocardium antigens
~Decrease of resistance to the infection
~Activating of saprophyte microflora
~Intoxication of organism by the products of necrosis
~Excretion in blood of myocardium enzymes}

What is the role of smooth muscle cells in pathogenesis of atherosclerosis?


{=They migrate in to intima, proliferate actively, absorb cholesterol, and stimulate the synthesis of
connective tissue
~They conduce for synthesis of connective tissue, absorb threeglicerides, and excrete lipolytic
enzymes
~They distroy a cholesterol, distroy a collagen, proliferate actively, synthesize the high density
lipoproteins
~They metabolize carbohydrates, synthesize a collagen, activate endotheliocytes, synthesize of low
density lipoproteins
~They conduce for synthesis of connective tissue, absorb cholesterol, and excrete lipolytic
enzymes }

TIMOKCIA, JMXAHHA

A patient, 64 years old, was hospitalized with complaints of a cough with sputum, expressed
dyspnea . During examiation the next signs were revealed: position is forced, breath rate — 32/min,
the intercostales muscles take part in the breathing. During X-ray examination the increased
transparency of lungs were determined. What is the most important in the pathogenesis of
respiratory failure in this patient?
{=Decrease of elastic properties of lungs
~Accumulation of sputum in the bronchial tubes
~Thinning of mucus shell of bronchial tubes
~Insufficiency of the surfactant system of lungs
~Fibrosis of lungs}

A patiet with the penetrating wound of pectoral wall was hospitalized to a clinic. What form
disorders of the external breathing may develop in this patient?
{=Ventilative-restrictive
~Obstructive
~Primary-diskinetic
~Difusion-restrictive
~Difusion-pneumonosis }
A young man having a cranial-cerebral trauma is in the serious condition. The breathing is
characterized with the convulsive attempts to breathe, which is not halted, that is sometimes broken
by exhalation. What type of breathing has this patient?
{=Apneic
~Gasping
~Kussmaul’s
~Chein-Stocks’
~Biot’s}

An young man with the signs of morphin poisoning was hospitalized to the emergency department.
His breathing is shallow and slow as a result of oppression of respiratory center. What type of
breathing disorders is present in this patient?
{=Disregulative disorders of alveolar ventilation
~Perfusive
~Ventilation restrictive
~Diffusive
~Ventilative obstructive}

In the laboratory animal has Gasping. It develops as a result of:


{=Stimulation cells caudal part of medulla oblongata
~Acceleration reflex Hering-Breyer’s
~Increase of haemoglobin in a blood
~Deceleration of the Hering-Dreyers reflex
~Irritation respiratory center by the surplus CO}

IKT, WEGIHKA, HUPKH


Put the scheme of pathogeny of hepatic icterus at protein starvation: decrease in the hepatocytes of
proteines Y and Z > ?> disorder of bilirubin transport > decreased transfer of it from blood through
cytoplasmatic membrane in the hepatocyte > icterus
{=Disorder of bilirubin cupture by ligandes
~Decrease of glucuroniltransferase activity
~Strong connection of bilirubin proteins
~Disorder of blood circulation in the liver
~Energetic insufficiency}

The sequence of development of acute pancreatitis includes next links: damage of cells >
?>bradykinin > edema, pain, inflammation. Define the absent link of pathogeny
{=Activating of kallikreinogen
~Activating of trypsin
~Development of autoimmune reaction>>
~Disorders of microcirculation
~Irritation of nervous ending}

In a patient which long time suffers of chronical enterocolitic after use meat food arise meteorism,
diarrhea, colics. With deficiency of what enzyme in intestine this associated?
{=Peptidase
~Saccharase
~Maltase
~Amylase
~Glycogensynthetase}

A boy 16-year-old complains of weakness, nausee, pain into right hypochondrium. On examenation
was revealed jaundiced sclera and skin. In the blood is large conjugated bilirubin, in the feces-
stercobilin in litlle amount, in the urine-urobilinogen and bilirubin. What type jaundice it is
characterized for?
{=Parenchymatous
~Mechanical
~Functional
~Hemolytic
~Hereditary}

A patient complains of dyspeptic disorders, melena, hemorrhoidal bleeding. Belly and liver
increased, sclera slightly jaundice What is pathology may be that symptoms in?
{=Portal hypertension
~Enteritis
~Intestine autoinxication
~Colitis
~Ulcer disease}

A patient complains of irritability, sleeplessness, rapit weakness, itch skin, yellowing skin and
mucous membrane, acholic feces. AP 100/75 mm Hg, cardial rate — 56/min. What jaundice it is
characterized for?
{=Mechanical
~Hemolytic
~Parenchymatous
~Hereditary
~Hepatic }

A patient complains of weakness, subfebrile temperature, jaundiced of sclera, dark urine, slightly
stained feces. Analysis of blood: conjucated bilirubin — 27,4 mmol/l, unconjucated bilirubin — 51,3
mmol/l. In the urine — urobilin. What is pathology take place in the patient?
{=Parenchymatous jaundice
~Mechanical jaundice
~Hemolytic jaundice
~Cholemic syndrome
~Portal hypertension syndrome}

A patient complaints on bad appetites weight loss, pain in epigastric department. For analysis of
gastric juice was revealed achylia. What does term meaning?
{=Lack of free HCl] and pepsin
~Lack of free HCl
~Lack of acidity
~Lack of free and binded HCl
~Lack of gastromucoprotein}

A woman 33-year-old suffers of hepato-cerebral dystrophy (Wilson’s disease). In the blood


decreased ceruloplasmin contents, in urine — sharply increased aminoacid contents. Effort of what
process was stipulated that changes?
{=Complex formation of aminoacid with copper
~Desintegration of tissuel proteins
~Synthesis of urea
~Glyconeogenesis
~Deamination}

A woman 33-year-old suffers with hepato-cerebral dystrophy (Wilson’s disease)/ In the blood —
decreased ceruloplasmin contents. In urine — sharply increased aminoacid contents. Disorder of
what process was caused these changes?
{=Copper metabolism
~Glyconeogenesis
~Deamination
~Synthesis of urea
~Desintegration of tissuel proteins}

After carried plural traumas and blood loss in a victim stomach dyskinesia developed on
hypokinetic type. Motore disorders of stomach in this case arose as a result of
{=Weakening of general tone of organism
~Use of fatty food
~Appearances of the unpleasant taste feelings
~Permanent phobia for the passed
~Appetite loss}

After removal of duodenum in a patient syndrom of duodenal insufficiency developted due to


disorder its endocrine function with phenomenon of cells deficiency of APUD-system. What from
called hormones is produced by ?-cells this part of intestine
{=Glucagon
~Secretin
~Serotonin
~Insulin
~Histamine}

After the carried hepatitis A in a patient maintenance of free corticosteroids was increased in blood.
Likely reason of disorders?
{=Deficit of transcortin
~Disorder of metabolism in the liver
~Surplus production of adrenals
~Activation of blood complement
~Liberation of peptidases from cells}

After the carried hepatitis in a sick child the signs of hypovitaminosis B, appeared as a result of
{=Disorder of formation of active forms it
~Decrease of absorption it in bowels
~Compete use by microflora
~Insufficient receipt with meal
~Forsing out by ascorbic acid}

After the carried rheumatism in a sick 33 years during a few years was formated insufficiency of
mitral valve of heart and superhepatic portal hypertension reason of which is
{=Rightventriculary insufficiency of heart
~Compression of portal vein by transsudate
~Embolism of portal vein
~Thrombosis of lower cavity vein
~Congestion of blood in lower cavity vein}

After the resection of stomach in a patient in connection with the duodenum ulcer the malabsorption
syndrome developed, reason of which became
{=Diminishing of microvillus amount of thin bowel
~Deficit of villicrinin hormone
~Strengthening of bowel peristalsis
~Activation of fermentative processes
~Loss of electrolytes and water}
After transfered infection with diarrhea and developed malabsorption syndrome with decrease of
aminoacids and glucose adaption. The main role in pathogenesis of activity transport their belong to
ions deficit?
{=Na

~CI}
As a result of liver insufficiency in a patient the signs of hyperaldosteronism developed due to
{=Hormone destruction disorder in a liver
~Disorder of hormone excretionis with urine
~Surplus formation in adrenal glands
~Insufficient binding of hormone with proteins
~Increase of sensitiveness of receptors to the hormone}

At fiberoscopic research of a patient 56 years was revealed cancer of stomach in lated stage and
intrahepatic portal hypertension reason of which is probably
{=Metastases of tumor in the liver
~Compression of portal vein by the tumor
~Metastases of tumor in the mesenteric nodes
~Compression of stomach veins by the tumor
~Decline of tone of portal vein}

At research of urine of a patient with hepatitis was revealed increase maintenance of amino acid.
Likely reason of this disorder?
{=Disorder of oxidative deamination
~Use of meat meal
~Increase of secretion in the kidneys
~Disintegration of cells in the liver
~Deficit in the meal of nicotine acid}

At the height of hepatitis in a patient was icreased maintenance of histamine in the blood that testify
about
{=Strengthening of decarboxylation of histidin in the liver
~Development of allergic reaction in the liver
~Decreasing of histaminase activity
~Decrease of binding of histamine by red corpuscles
~Strengthening of liberation of histamine from cells}

For differentiation of functional disorders of stomach secretion from organic lesion to patient is
proposed tests with straight stimulation of secretion. The most strong stimulating action of secretory
cells mucous membrane cause:
{=Gastrin
~Cholecystokinin
~Acetylcholine
~Insulin
~Histamine}

Heavy ecephalopathy at the Crigler-Najjar syndrome develops in the pathogenic sequence: inherited
defect of enzyme of UDPh-glucuroniltransferase > ? > ecephalopathy > nuclear icterus. Complete
the scheme of pathogeny
{=High level of free bilirubin
~Lesion of hepatocytes
~Decrease of permeability of hepatocytes membrane
~Disorder of transport of conjucated birubin in blood
~Decrease of bilirubin secretion in the bile}

In a child 8 months was revealed disorder of digestion in the bowels, signs of pellagra with
disorders of neuro-psychic development. Likely reason of disorders?
{=Congenital disorder of tryptophan absorption
~Decrease of pepsin secretion by mucus stomach
~Insufficient secretion of pancreatic juice
~Deficit in the meal of nicotine acid
~Disorder of metabolic function of liver}

In a child with hemolytic disease of the new born developed encephalopathy. Increase of what
matter in blood caused lesion of CNS?
{=Unconjucated bilirubin
~Complex bilirubin-albumin
~Conjucated bilirubin
~Verdoglobin
~Biliverdin}

In a man 38-year-old is observed jaundiced skin, anemia, spleen increased, hyperbilirubinemia at


the expense of unconjugated bilirubin., stercobilirubinuris, hypercholic dark feces. What state the
most characterized that changes for?
{=Hemolytic jaundice
~Mechanical jaundice
~Hepatic jaundice
~Gilber’s syndrome
~Hepatic insufficiency syndrome}

In a men 25-year-old which got closed craniocerebral trauma was observed long unrestrained
vomiting. What mechanism stipulated vomiting act
{=Straight irritance of vomiting centre by increased intracranial pressure
~Straight excitement of vomiting centre by toxic substances of blood
~Straight excitement of vomiting centre by impulses from stomach and intestine
~Activation of vomiting centre as a result of movement and secretory stomach function increase
~Excitemeut of vomiting centre through branches of sympathic nerves}

In a men 48-year-old which suffers of stomach ulcer was determined secretion increase and increase
of gastric juice acidity . What is mechanism stipulate this phenomenon?
{=Increase histamine output
~Increase secretin output
~Increase motilin output
~Increase catecholamines output
~Increase cholecystokinin output}

In a men was revealed hypersecretion of gastric juice at the time examination. Physician
recommended to except from diet saturated broth and vegetable-water because they contets matters
which stimulate gastric secretion, and namely:
{=Extractive substancies
~Hydrochloric acid
~Large of carbohydrates
~Large of fat
~Gastrin}
In a patient 27-year-old was revealed patholodical changes in liver and brain. In the plasma — sharp
slowing, and urine increase copper contents. Activity of what enzyme in serum blood necessary to
defermine in that case?
{=Ceruloplasmin
~Carbonic anhydrase
~Xanthine oxidase
~Alcoholdehydrogenase
~Leucinaminopeptidase}

In a patient 45 years duodenitis developed, but for that secretory function of pancreas was
disordered as a result of decrease of secretion of
{=Secretin
~Gastrin
~Gastroinhibiting of peptide
~Cholecystokinin
~Somatostatin}

In a patient was disordered digestion of proteins in the intestine. Deficit of what enzymes due to that
process with?
{=Peptidase
~Lipase
~Synthetase
~Amylase
~Transferase}

In a patient with acute pancreatitis was revealed sharply increased trypsin activity in blood and
sharp decrease of arterial pressure — 80/60 mm Hg. What may explain influence of trypsin onto
arterial pressure with?
{=Activation of kinin synthesis in blood
~Inhibition of kinin synthesis in blood
~Activation of angiotensin IT synthesis
~Inhibition of angiotensin I synthesis
~Activation of aldosterone synthesis}

In a patient with hepatic jaundice was revealed disorder of bilirubin conjugation with glucuronic
acid due to deficit UDP-glucuroniltransferase, which catalysed this process. What hereditary
disease characterized that disorder for?
{=Crigler-Najjar’s syndrome
~Gilbert’s syndrome
~Dubin-Johnson’s syndrome
~Rotor’s syndrome
~Down’s syndrome}

In a patient with stable hypoglycemia analysis of blood after injection of adrenalin essentialy non
changed. Physician supposed about liver function disorder. About changes of what liver function
may be question?
{=Glycogensynthesis
~Excretory
~Cholesterol formating
~Deintoxicatic
~Ketogenic}

In a woman 45-year-old was revealed in gallbloodder biliary stones as manifestation of dyscholia in


a result of excess take of fat food. By main mechanism of arising of lithogenic bile is in that case?
{=Decrease of biliary acids and lecithin to bile cholesterol relation
~Inhibition of cholic acid synthesis in liver
~Acceleration of biliary acid absorption by gallbladder mucous
~Decrease of ?-hydroxilase activity
~Decrease of hepato-intensinal circulation of cholic acids}

In blood of a patient was revealed low level of albumin and fibrinogen. Increase activity of what
hepatocyte organelle stipulate this phenomenon?
{=Granular endoplasmatic reticulum
~Mitochondrium
~Agranular endoplasmatic reticulum
~Lysosome
~Golgi complex}

In examination of a patient with stomach ulcer was determined gastric secretion rise maximum for 5
minutes after eat and fall down it arise through lhour 25 minutes. What type of gastric secretion in
that patient?
{=Excitable
~Normal
~Braking
~Asthenic
~Inert}

To a patient with hypersecretion of gastric juice physician was recomended to exclude from diet
concentrated broth and vegetable decoctions because they stimulate gastric secretion. What is
dominating mechanism of gastric secretion stimulation in this patient?
{=Stimulation of gastrin output by G-cells
~Irritation of gastric mechanoreceptors
~Irritation of taste receptors
~Irritation of gastric chemoreceptors
~Stimulation of secretin output in duodenum}

A 23-year-old patient who had frequent strep infections of the throat complained of periodic
headache, rapid tiredness, periorbital edema. Arterial hypertension, proteinuria, hypoproteinemia,
hyperlipidemia were revealed. What disease was most likely in this case?
{=Chronic glomerulonephritis
~Acute pyelonephritis
~Chronic pyelonephritis
~Lipoid nephrosis
~Diabetic nephropathy}

A 23-year-old woman has complaints of flank pain, loos of appetite, increase of temperature to
39°C. Positive symptom Pastepnatsky was found. There is leukocytosis , ESR is raised. In the urine:
albumen 0,9 g/l, leucocytes 250-300 in area of view, red blood cells 8-10 in area of view, mucus
+++, bacteria +++. What disease has the patient?
{=Acute pyelonephritis
~Acute glomerulonephritis
~Chronic glomerulonephritis
~Nephrolithiasis
~Acute renal failure}

A 24-year-old pregnant woman has frequent exsessive vomiting. What disorder of acid-basic
balance can be in her?
{=Metabolic alkalosis
~Respiratory acidosis
~Not respiratory acidosis
~Respiratory alkalosis
~Metabolic acidosis}

A 35-year-old patient after cranial trauma complains on permanent thirst, increased urine output (to
10 I/per day). Specific gravity of urine is 1010, pathological components are absent. What is the
cause of these symptoms in the patient?
{=Antidiuretic hormone deficiency
~Increased antidiuretic hormone production
~Decrease of insulin synthesis
~Hyperproduction of aldosterone
~Decrease of aldosterone synthesis}

A 42-year-old man has chronic glomerulonephritis. Hypertension, edema, vomiting, diarrhea, itch
of skin, arthritis are observed. Contents of residual nitrogen of blood - 43 mmol/l. The reason of
these signs is
{=Reduced renal excretion
~Increased glomerular filter permeability
~Ischemia of kidneys
~Disorder of reabsorption of bicarbonates
~Disorder of concentrating mechanism}

A 45-year-old patient with hypovolemic shock has acute renal failure. The main mechanism of this
disorder is
{=Decrease of blood flow to the kidneys
~Increase of reabsorption of sodium
~Violation of urine passage
~Obstruction of urine outflow
~Increase of reabsorption of water}

A 50-year-old patient with chronic pyelonephritis has arterial hypertension. What is the reason of
this symptom?
{=Increased formation of kidney prostaglandins
~Activation of central cholinergic mechanism
~Decreased partial pressure of oxygen in kidney
~Activation of angiotensinase synthesis in kidney
~Activation of acidogenesis and ammoniagenesis in kidneys}

A 75-year-old male has anuria. The arterial pressure is 55/20 mm Hg. What kidney function is
altered?
{=Glomerular filtration
~Obligate reabsorption
~Facultative reabsorption
~Tubular secretion
~All enumerated processes}

A patient has arterial hypertension for 10 years. After using of furosemyd loss of appetite, general
weakness, decrease of blood pressure and intestine peristaltic appeared. The cause of such changes
is
{=Hypokalemia
~Hyponatremia
~Hyperuricemia
~Hypercalcemia
~Hyperkalemia}

A patient has diabetes mellitus for 25 years. This disorder led to impairment of renal function.
Glomerular filtration rate is 9 ml/min. What is main mechanism of deminished glomerular filtration
rate?
{=Gradual tissue destruction
~Decrease of systemic arterial pressure
~Occlusion of tubules
~Development of acidosis in tissues
~Spasm of afferent glomerular arteriole}

A patient has isoosmolar dehydration as a result of prolonged diarrhea. What complication may
develop, if it to compensate by hypotensive solution?
{=Swelling of cells
~Dehydration of cells
~Edema
~Ascites
~Arterial hypertension}

A patient has unlateral loin pain, fever, malaise, anorexia. Full blood count shows elevated white
cell count with neutrophilia. Microscopy of urine shows pyuria, proteinuria. What is disease in this
patient?
{=Acute pyelonephritis
~Acute glomerulonephritis
~Chronic glomerulonephritis
~Nephrolithiasis
~Chronic kidney insufficiency}

A patient with polycystic kidney disease has chronic renal failure. Glomerular filtration rate is
reduced to 15 % from normal. What is the main cause of the decline of glomerular filtration in this
case?
{=Decreased amounts of functioning nephrons
~Tubulopathy
~Obstruction of urinary tract
~Ischemia of kidneys
~Thrombosis of kidney arteries}

EHAOKPHHHA
A 31-year-old woman, who has two healthy children, notes that she has had no menstrual periods
for the past 6 months, but she is not pregnant and takes no medications. Within the past week, she
has noted some milk production from her breasts. She has been bothered by headaches for the past 3
months. After nearly hitting a bus while changing lanes driving her vehicle, she is concerned with
her vision. An optometrist finds her lateral vision to be reduced. On physical examination she is
afebrile and normotensive. Which of the following laboratory test findings is most likely to be
present in this woman?
{=Hyperprolactinemia
~Lack of growth hormone suppression
~Increased serum alkaline phosphatase
~Increased serum cortisol
~Hyponatremia}

A 28-year-old woman has had difficulty concentrating at work for the past month. She is constantly
getting up and walking around to visit co-workers. She complains that the work area is too hot. She
seems nervous and often spills her coffee. She has been eating more but has lost 5 kg in the past 2
months. On physical examination her temperature is 37.5°C, pulse 101/minute, respiratory rate
22/minute, and blood pressure 145/85 mm Hg. Which of the following laboratory findings is most
likely to be present in this woman?
{=Decreased TSH
~Decreased iodine uptake
~ Decreased plasma insulin
~ Increased calcitonin
~Increased ACTH}

A 19-year-old previously healthy woman has had a mild pharyngitis followed by a high fever over
the past 24 hours. When seen in the emergency room, her skin now shows extensive areas of
purpura. Vital signs include temperature 39°C, pulse rate 102/minute, respiratory rate 21/minute,
and blood pressure 80/55 mm Hg. Laboratory studies show a serum sodium of 115 mmol/L,
potassium 5.3 mmol/L, chloride 92 mmol/L, CO2 22 mmol/L, glucose 42 mg/dL, and creatinine 1.1
mg/dL. Which of the following is the most likely diagnosis?
{=Meningococcemia
~Disseminated tuberculosis
~Reactive systemic amyloidosis
~Sheehan syndrome
~Idiopathic adrenalitis}

A 37-year-old man experiences abdominal pain, nausea, and constipation for the past 3 days. On
physical examination he has no palpable abdominal masses and bowel sounds are present. His lungs
are clear to auscultation. He has a heart rate of 80/min with an irregular rhythm. An
electrocardiogram demonstrates a shortened QT(corrected) interval and a prolonged PR interval. He
has a stool positive for occult blood. Upper GI endoscopy reveals multiple 1 cm diameter shallow
ulcerations of the gastric antrum. Which of the following laboratory test findings is most likely to
be present in this man?
{=Serum calcium of 12.4 mg/dL
~Thyroid peroxidase antibody of 4 TU/mL
~Blood glucose of 225 mg/dL
~Total serum thyroxine of 21 ng/mL
~ Plasma cortisol of 45 microgm/dL at 8 am}

A 58-year-old man with a history of diabetes mellitus has noted the presence of bone pain,
especially of his hands, for the past 6 months. On physical examination there is no swelling or
redness of his hands, no joint deformity, but the range of motion is slightly decreased. Laboratory
studies show sodium 139 mmol/L, potassium 4.0 mmol/L, chloride 98 mmol/L, CO2 22 mmol/L,
glucose 153 mg/dL, creatinine 7.8 mg/dL, calctum 7.8 mg/dL, phosphorus 5.7 mg/dL, total protein
6.2 g/dL, and albumin 4.0 g/dL. Which of the following conditions is this man most likely to have?
{=Adrenal adenoma
~Medullary thyroid carcinoma
~Extra-adrenal pheochromocytoma
~Parathyroid hyperplasia
~Pituitary adenoma}

A 49-year-old woman has had increasing cold intolerance, weight gain of 4 kg, and sluggishness
over the past two years. A physical examination reveals dry, coarse skin and alopecia of the scalp.
Her thyroid is not palpably enlarged. Her serum TSH is 11.7 mU/L with thyroxine of 2.1
micrograms/dL. A year ago, anti-thyroglobulin and anti-microsomal autoantibodies were detected at
high titer. Which of the following thyroid diseases is she most likely to have?
{=Hashimoto thyroiditis
~Papillary carcinoma
~DeQuervain disease
~Multinodular goiter
~ Graves disease}

A 48-year-old woman has experienced constant back pain exacerbated by movement over the past
month. She reports increasing weakness over the past 3 months. On physical examination her blood
pressure is 165/110 mm Hg. She is overweight, with a BMI of 28. A radiograph of the spine reveals
a compressed fracture at T10. Laboratory findings include a serum glucose of 155 mg/dL. Which of
the following pathologic lesions is most likely to explain her findings?
{=Adrenal cortical carcinoma
~Anaplastic thyroid carcinoma
~Empty sella syndrome
~Pheochromocytoma
~Multinodular goiter}

An 35-year-old woman has had insomnia for the past 4 months, as well as episodes of diarrhea with
up to 4 loose stools per day. On physical examination, she exhibits bilateral proptosis. Her
outstretched hands have a fine tremor. On palpation of her neck, the thyroid gland does not appear
to be enlarged and no masses are palpable. Laboratory studies show a serum TSH of 8.8
microU/mL in association with a serum total thyroxine of 15.1 microgram/dL. Which of the
following is the most likely diagnosis?
{=Pituitary adenoma
~Graves disease
~Chronic thyroiditis
~Prior thyroidectomy
~Multinodular goiter}

A 50-year-old man has episodic headaches for 3 months. On physical examination his blood
pressure is 185/110 mm Hg, with no other remarkable findings. Laboratory studies show sodium
145 mmol/L, potassium 4.3 mmol/L, chloride 103 mmol/L, C02 26 mmol/L, glucose 91 mg/dL, and
creatinine 1.3 mg/dL. An abdominal CT scan shows a 7 cm left adrenal mass. During surgery, as
the left adrenal gland is removed, there a marked rise in blood pressure. Which of the following
laboratory test findings most likely explains his findings?
{= Increased urinary free catecholamines
~Decreased urinary homovanillic acid
~Increased serum ACTH
~Decreased serum cortisol
~Elevated serum ANCA}

A 56-year-old woman has had diffuse, dull, constant abdominal pain for the past 2 months. On
physical examination no abnormal findings are noted. An abdominal CT scan shows a 3 cm right
adrenal mass. The right adrenal is excised and on microscopic examination the mass is composed of
cells resembling adrenal cortex. Which of the following features is the most reliable indicator that
this mass is malignant?
{=Invasion
~Presence of mitoses
~Cellular atypia
~Size of the mass
~Cellular necrosis}

A 33-year-old previously healthy man has lateral visual field deficits, but his residual vision is
20/20. His facial features have changed over the past year. His shoe size has increased. A head CT
scan reveals enlargement of the sella turcica. Which of the following hormones is most likely being
secreted in excessive amounts in this man?
{=Growth hormone
~Prolactin
~ACTH
~Antidiuretic hormone
~Luteinizing hormone}

A 45-year-old man has a 4 month history of nonfocal, generalized headaches. On physical


examination {=he is found to have a blood pressure of 170/110 mm Hg. Laboratory studies show a
serum sodium of 146 mmol/L, potassium 2.3 mmol/L, chloride 103 mmol/L, CO2 27 mmol/L,
glucose 82 mg/dL, and creatinine 1.2 mg/dL. His plasma renin activity is 0.1 ng/mL/hr and his
serum aldosterone 65 ng/mL. Which of the following is the most likely cause for his findings?
{=Adrenal cortical adenoma
~21-hydroxylase enzyme deficiency
~Pituitary adenoma
~Exogenous corticosteroid administration
~Renal cell carcinoma}

A 45-year-old woman with severe rheumatoid arthritis has been on chronic corticosteroid therapy
for the past 15 years. She is admitted for an orthopedic procedure to correct joint deformity from
her disease. She continues to receive her regular dosage of 5 mg of prednisone per day. Three days
postoperatively, she develops an aspiration pneumonia with Klebsiella pneumoniae cultured from
sputum. Five days following the surgery, she becomes obtunded. Laboratory findings at that time
include: sodium 105 mmol/L, potassium 5.4 mmol/L, chloride 87 mmol/L, CO2 16 mmol/L,
glucose 40 mg/dL, and creatinine 1.1 mg/dL. Which of the following complications is most likely to
have occurred in this patient?
{=Acute Addisonian crisis
~Waterhouse-Friderichsen syndrome
~Anterior pituitary necrosis
~Conn syndrome
~21-hydroxylase deficiency}

A 41-year-old man has been drinking large quantities of water--up to 20 liters per day--for the past
week. On physical examination he has diminished skin turgor and dry mucous membranes.
Laboratory studies show sodium 162 mmol/L, potassium 4.1 mmol/L, chloride 121 mmol/L, and
bicarbonate 27 mmol/L. His serum glucose is 75 mg/dL and creatinine 1.0 mg/dL. His serum
osmolality is 343 mOsm/kg. A deficiency of which of the following hormones is most likely present
in this man?
{=Vasopressin
~Oxytocin
~Insulin
~Growth hormone
~Prolactin}

A 50-year-old man has a lump on his neck found at physical examination. A fine needle aspirate of
this lump shows a follicular neoplasm of the thyroid. He undergoes a total thyroidectomy. Within a
day following surgery, he is noted to have tingling sensations and neuromuscular irritability. Which
of the following serum laboratory tests should be ordered immediately to determine further therapy
for this man?
{=Calcium
~Parathormone
~Thyroxine
~TSH
~Calcitonin}
A 57-year-old man is found comatose. On physical examination he has decreased skin turgor.
Laboratory studies show a blood glucose of 780 mg/dL. Urinalysis reveals no ketosis or proteinuria,
though there is 4+ glucosuria. Which of the following is the most likely diagnosis?
{=Type II diabetes mellitus
~Type I diabetes mellitus
~Cushing syndrome
~Ingestion of a large quantity of sugar
~Neuroendocrine tumor secreting glucagon}

Acromegaly is most frequently diagnosed in:


{=Middle-aged adults
~Newborns
~Children ages 2 to 5
~Adults age 65 and older}

A patient presents with weight loss, sweating, palpitations, tachycardia, tremors, lid lag,
exopthalmosis, and a goiter. Upon taking a thyroid scan, there appears to be an increased amount of
iodine. What is the pathogenesis of his disease?
{=Autoimmune mediated
~Immune complex mediated
~Unknown
~Infection
~None of the above}

A patient presents with lethargy, tiredness, cold intolerance, dryness of skin and hair, hoarsness of
voice and weight gain. He also has psychosis and his heart rate appears to be lower than normal.
Which of the following is not associated with the presenting symptoms?
{=Hashimoto's disease
~Cretinism
~Protruding tongue, potbelly appearance, dwarflike features
~Iodine deficiency
~Increased iodine levels}

A girl presents with delayed puberty, absent secondary sexual characteristics, and primary
amenorrhea. She also appears to be hypertensive and hypokalemia. Which of the following
enzymes is increased if there is virilization of a person?
{=11bHydroxylase
~17ahydroxylase
~21 b-hydroxylase
~Pyruvate kinase
~Alcohol degidrohenase}

A patient presents with a bump on his neck. Upon taking a tissue sample, there appears to be
parafollicular cells. The tumor is also associated with MEN II, although most of the time it is
sporadic. What is the best way to moniter treatment?
{= Measure calcitonin levels
~Measure iodine levels
~Measure TSH levels
~Measure thyroid hormone levels
~Measure iron levels}

A patient with Cushing syndrome might present with any of the following EXCEPT:
{=Bronze or hyperpigmented skin
~Obesity
~Buffalo hump
~Moon facies
~Glucose intolerance}

Patients with diabetes have an increased risk of all of the following EXCEPT:
{=Pancreatic carcinoma
~Infections
~Increased atherosclerosis
~Peripheral neuropathy
~Cataracts }

Which of the following is true regarding multiple endocrine neoplasia (MEN) I syndrome? .
{=Many patients develop parathyroid hyperplasia
~The genetic abnormality involves the RET gene
~The genetic abnormality involves a proto-oncogene
~Virtually all patients develop medullary thyroid carcinoma
~Some patients may have a Marfanoid habitus}

Which of the following is true regarding pheochromocytoma?


{=It can cause hypertension
~It is usually an aggressive, malignant tumor
~It only occurs in the adrenal gland
~It is derived from neural crest cells
~Many tumors have a |p deletion}

Which of the following is true regarding Addison’s disease?


{=It is also called primary chronic adrenal insufficiency
~Most cases are due to infection
~It is characterized by an overproduction of catecholamines
~The onset is sudden
~Patients are often hypertensive actin level.}

Which sysndrom belongs to hypothalamical?


{=Hyperhonadotropic hypogonadism
~Syndrom of persistent lactation
~Cushing syndrom
~Frelich syndrom
~DIC syndrom}

What can be provoked with Shuchan syndrom?


{=DIC syndrom
~Afterdeliverical hypochinadism
~Meningitis
~Pernicious anemia
~Hemolisis of erythrocytes}

Poin the state which leads to blood pressure changes in Addison disease
{=Hypovolemia
~Increase of vessels tonus
~Hypervolemia
~Tachicardia
~Decrease of n.vagus tonus}

Which clinical state is not caracterised for Kone syndrom?


{=Tachicardia
~Hypervolemia
~Decrease of n.vagus tonus
~Decrease of muscular tonus
~Thirst and poliuria}

In which type of tissures break down of protein is intensified during Cushing syndrome?
{=Muscular tissures
~Adrenal glands
~Skin
~Limphoid tissure
~Bones }

What is the reason of peripheral resistancy to hormons?


{=Block of receptors with autoantibodies
~Disorders of realising hormons syntesis
~Atrophy of target cells
~Low concentration of hormon un bloodstream
~Hereditary deficit of cellular programs}

Which mechanism of lipid metabolisms takes place in cushing disease?


{=Decrease of fat mobilisation from depot under action of glucocorticoids
~Low rateof fat synthesis
~Intensification of lipolysis
~Intensification of gluconeogenesis
~Intensification of fat mobilisation from depot under action of catecholamines}

Poin the basic change in Kone syndrom


{=Hypertension
~Disorders of heart functions
~Megacolon
~Lethargy
~Hypovolemia}

Which syndrom is characterised for hypopituitarism?


{=Simmonds syndrom
~Alimentary obisity
~Lactorrea-amenorrea
~Chican syndrom
~Frelich syndrom}

Characteristic of Simmonds disease:


{=Coused with adenohypophysis destruction
~Primary hypocorticism is manifestation
~Coused due to hypotalamus destruction
~Clinically manifests like ACT deficiency
~Clinically manifests like deficiency of somatotropin}

What is the manifestation of primary hyperaldosteronism?


{=Increasing of vasoconstrictive effect of catecholamines
~Arterial hypotension
~Muscular hypertonus
~Alopecia
~Poliphagia}
Point what is not characterised for total adrenal insufficiency?
{=Increase of mucular tonus
~Arterial hypotension
~Hyponatriemia
~Hyperkaliemia}

Gigantism and acromegaly are due to


{=Hyperpituitarism
~Hyperthyroidism
~Hypopituitarism
~Hypothyroidism}

What is Sheeshan’s syndrome?


{=The pituitary adenoma is hemorrhagic and necrotic
~Pituitary gland is infected with virus
~Pituitary gland is infected with Bacteria
~It is the malignancy of the pituitary}

Uniform painless enlargement of the thyroid gland is noted in:


{=long standing goiter
~Riedel’s thyroiditis
~Subacute granulomatous thyroiditis
~Grave’s disease}

A 50 -year -old male presented with a history of recurrent gastric ulcers and multiple renal stones.
The most likely histologic finding is:
{=Parathyroid adenoma
~Adrenal cortical adenoma
~Pituitary adenoma
~Pheochromocytom
~Thyroid adenoma}

A parathyroid adenoma may cause all of the following except:


{=Tetanus
~Renal calculi
~Elevated levels of serum calcium
~Osteomalacia}

A man 52, years old, complains on a general weakness and obesity. Objectively: height 168 cm,
weight 111 kg, obesity with extreme deposits of fat on the face, neck, stomach. BP — 195/100
mmHg, level of glucose in blood 8,9 mmol/l. What function of what endocrine gland is changed?
{=Hyperfunction of neurohypophysis
~Hyperfunction of thyroid gland
~Hypofunction of adrenal cortex
~Hypofunction of pancreas}

A man, 58 years old (growth 170 sm, mass 103 kg), has an obesity, filling out person, apathy,
hypothermia, bradycardia. The dysfunction of what gland leads to that state of the patient?
{=Thyroid gland
~Parathyroid gland
~Adrenal gland
~Pancreas
~Hypophysis}
A patient complains on loss of weight, agitation, fatigue, muscular weakness, skin
hyperpigmentation, hypotension. It is known from anamnesis, that patient suffers on pulmonary
tuberculosis and is addicted to alcohol for a long time. Name the most possible reason of disease at
this patient.
{=Tubercular impairment of adrenal glands
~Alcoholic illness of liver
~Tumour of adenohypophysis
~Impairment of parathyroid glands
~Chronic intoxication by alcohol}

A patient complains on polydypsia and poliuria (8 liters per day). Objectively no changes in
carbohydrates metabolism were found. What endocrine gland dysfunction is a cause of such
changes?
{=Neurohypophysis
~Adenohypophysis
~Pancreas
~Adrenal cortex
~Thyroid gland}

A patient complains on rapid fatigue, dyspepsia. It is discovered at the inspection: BP — 95/65


mmHg, glucopenia, hyponatremia, skin hyperpigmentation. What disease are these symptoms
typical for?
{=Addison disease
~Syndrome of Itsenko-Cushing
~Conn’s Syndrome
~Acute insufficiency of adrenal cortex
~Pheochromocytoma}

A patient for two months used glucocorticoids occasionally for the treatment of joint syndrome. He
abolished abruptly the reception of preparation by his own decision. He complains on dizziness,
decline of arterial pressure, general weakness, apathy, nausea, vomiting and hypothermia. How can
you explain these symptoms?
{=Development of insufficiency of adrenal glands
~Resistance to preparation
~Development of hepatic insufficiency
~Development of kidney insufficiency
~Hyperproduction of ACTH by hypophysis}

A very tall patient with long fingers of hands, large lower jaw, enlarged superciliary arcs and nose
appealed to the doctor. The increases secretion of what hormone can be suspected?
{=Somatotropic
~Thyrotropic
~Gonadotropic
~Antidiuretic
~Adrenocorticotropic }

A woman appealed to the doctor with complains on poliuria, thirst, weakness. At the inspection the
hypercalciemia and overactivity of acid phosphatase of blood was revealed. What pathology can be
suspected?
{=Hyperparathyroidism
~Hypothyroidism
~Hypoparathyroidism
~Hyperthyroidism
~Insulinoma}

A woman complains on violation of menstrual cycle, milkish discharge from mammal glands.
Hyperproduction of what hormone leads to the syndrome of galactorrhea-amenorrhea?
{=Prolactin
~Gonadotropic hormone
~Parathormone
~Progesteron
~Cortisone}

After massive hemorrhage in labor the woman has rapid loss of weight, hypothermia, atrophy of
muscles, hypoglycemia, hypotension. What pathology of hypophysis can be suspected?
{=Illness of Shihan
~Syndrome of Parhon
~Non mellitus diabetes
~Ilness of Itsenko-Cushing
~Panhypopituitarism}

At a patient while laboratory examination it is found out diminishing of daily diuresis,


hypernatremia, kaliopenia. The hypersecretion of what hormone can be the reason of these
changes?
{=Aldosterone
~Cortisone
~Thyroxine
~Atrial natiiuretic factor
~Vasopressine}

At a patient with hypothyroidism it is found while inspection a specific pale and yellowish color of
skin. It is present because of:
{=Violation of transformation of carotin into the vitamin A in liver
~Violation of bilirubine conjugation
~Increase of level of cholesterol in blood
~Increase of acidic glicosaminoglicans in skin
~Increase of concentration of ions of sodium in skin}

At a patient with obesity and signs of hirsutism the enlargement of hypophysis and hyperplasia of
adrenal gland’s cortex is found. BP — 200/100 mmHg. Concentration of glucose in blood — 18
mmol/l. What pathology is present?
{=Ilness of Itsenko-Cushing
~TlIness of Addison
~Syndrome of Itsenko-Cushing
~Adiposogenital dystrophy
~Hyperthyroidism}

At congenital insufficiency of cellular receptors to androgens it develops


{=Syndrome of testicular feminization
~Syndrome of Itsenko-Cushing
~Adiposogenital dystrophy
~Adiposogenital syndrome
~TIIness of Addison}

At experimental animal supraoptic and paraventricular nuclei of hypothalamus are impaired. What
disease may develop?
{=Diabetes insipidus
~Pituitary nanism
~Tilnesses of Itsenko-Cushing
~Acromegaly
~Pituitary cachexia}

At the inspection of boy of 5 years it is found out the presence of the secondary sexual signs (haired
pubis and arm-pits, changed voice), hypertrophy of gonads and testicles. The increase of secretion
of what hormones can be suspected?
{=Androgenes
~Estrogenes
~Glucocorticoids
~Mineralocorticoids
~Thyroid hormones}

At the patients there is hirsuitism, change of voice, diminishing of mammal glands, amenorrhea.
What tumorous process can lead to development of such changes?
{=Androsteroma of adrenal cortex
~Adenoma of glomerular zone of adrenal cortex
~Adenocarcinoma of thyroid gland
~Pheochromocytoma
~Eosinophylic adenoma of hypophysis}

During the examination of a boy of 13 years old you can observe the retardation in growth, the
disproportion of body, delay of mental development, obesity, hypothermia, bradycardia. This is a
result of the dysfunctions:
{=Thyroid gland
~Sexual glands
~Adrenal glands
~Hypophysis
~Thymus}

During the objective examination of the patient you can find: apathy, obese, edema, bradycardia,
low blood pressure, the decrease of exchange of matters and the hoarse of voice. What endocrine
disease can be suspected?
{=Hypothyriosis
~Acromegalia
~Thyrotoxicosis
~Itsenko- Kushing syndrome
~Diabetes mellitus}

Estimate the indexes of glucose tolerant test of patient: the content of glucose on an empty stomach
was 5,9 mmol/l, in 2 hours after the glucose loading — 7,2 mmol/L. Diagnosis:
{=Violated glycemia
~Thyrotoxicosis
~Insuline-dependent diabetes mellitus
~Changed tolerance to glucose
~Absent changes}

In patient with pheochromocytoma there is a periodical acute pain in epigastrium, tachycardia,


elevation of BP. These attacks are explained
{=By the massive liberation of catecholamines by adrenal glands
~By the massive liberation of glucocorticoids by adrenal glands
~By liberation of noradrenalin by sympathetic nerves
~By the massive liberation of thyroid hormones
~By the massive liberation of aldosterone}

In the blood of inspected patients the level of glucose on empty a stomach was 6,9 mmol/l, in 2
hours after glucose loading — 14,8 mmol/l. Such indexes are typical for
{=Patient with insuline-dependent diabetes mellitus
~Patient with thyrotoxicosis
~Patient with violated glycemia
~Patient with the changed tolerance to glucose
~Healthy person}

In the blood of patient level of glucose on empty stomach was 5,65 mmol/l, and 2 hours after
glucose loading — 4,95 mmol/l. Such indexes are typical for
{=Healthy person
~Patient with thyrotoxicosis
~Patient with violated glycemia
~Patient with the violated tolerance to glucose
~Patient with insuline-dependent diabetes mellitus}

In the blood of patient there is significant increase of catecholamins. He complains on a frequent


hypertensive crises, paroxysms of tachycardia, headache. Disfunction of what gland may cause such
symptoms?
{=Cerebral layer of adrenal glands
~Cortical layer of adrenal glands
~Thyroid gland
~Pancreas
~Parathyroid glands}

In two days after thyroidectomy because of the tumour of thyroid gland, a patient is drowsy, thirsty;
there is an increase of neuro-muscular excitability with development of hypoparathyroid tetanus.
What violation of electrolyte exchange takes place here:
{=Hypocalcemia
~Hypercalcemia
~Hyponatriemia
~Hypomagniemia
~Hypermagniemia}

In women after operation on thyroid gland the fascicular shaking of muscles of hands, feet and face
appeared. These violations can be removed by introduction of
{=Parathormone
~Triiodo-thyronine
~Thyrotropine
~Thyroxine
~Calcitonine}

In women of 35 years there are frequent uterine bleeding. The syndrome of persistant follicle is
diagnosed. The hypersecretion of what hormones is observed here:
{= Estrogenes
~Androgenes
~Progesteron
~Thyroid hormones
~Mineralocorticoids }
Patient of 20 years old complains on amenorrhea, thirst, feeling of hunger. At the inspection it is
found out obesity with lipopexia in the area of thighs, pelvis, stomach, shoulders, hypogenesis of
genitals, hypothermia. What endocrine pathology can be suspected?
{=Adiposogenital dystrophy
~Syndrome of Itsenko-Cushing
~Ilness of Itsenko-Cushing
~Syndrome of galactorrhea- amenorrhea
~Adrenogenital syndrome}

Patient with rheumatoid arthritis was on glucocorticoid hormone drug for a long time according to
the doctor’s prescription. The secretion of which hormone will be depressed?
{=Adrenocorticotropic hormone
~Aldosteron
~Somatotropic hormone
~Thyrotropic hormone
~Prolactin}

The patient had a protracted psychoemotional tension. After that she noticed an appearance of
poliuria, thirst, increased appetite. In a blood there is hyperglycemia, in urine there is glucosuria.
Violation of secretion of what hormone causes these disorders?
{=Insulin
~Somatotropin
~Glucagon
~Corticotropin
~Thyroxin}

The patient has nephrocalcinosis, nephrolithiasis, ossification of internal organs, osteoporosis.


Hyperfunction of what endocrine gland is the reason of this state?
{=Parathyroid glands
~Adrenal glands
~Thyroid gland
~Gonades
~Hypophysis}

The patient is hospitalized with complaints on the cramps of upper and lower extremities. BP —
170/105 mmHg, glucose of blood — 6,6 mmol/I, cholesterol — 6,1 mmol/1, calcium — 2,2 mmol/I,
phosphorus — 1 mmol/l, sodium — 162 mmol/l. Daily diuresis —- 750 ma. What pathology can be
suspected?
{=Illness of Conn
~Basedov illness
~Tllness of Itsenko-Cushing
~TIIness of Simmonds
~Tilness of Addison}

HEPBOBA

A baby with pupillar dilation, visual disturbances, difficulty breathing, and enlarged head is
diagnosed with Dandy-Walker syndrome. What is underlying possible mechanism of his disease?
{=B and D
~Problem with arachnoid granulations
~Tumor
~Alzheimers
~Hemorrhage/inflammations }
A pain is initially clear but later presents with headache, vomiting and seizures, later he has
hypertension and bradycardia. He received a blow to the side of his head. What kind of vessel was
involved?
{=Middle meningeal artery
~Bridging veins
~Anterior communicating artery
~Posterior communicating artery
~Middle cerebral artery}

A patient with headache, contralateral weakness and seizers and eventally congnitive impairment is
diagnosed to have Alzheimers. The type of condition most closely resembling his state would be?
{=Hematoma
~Tumor
~Respiratory illness
~Sexually tansmitted disease
~Liver cirrhosis}

The most common cause of intracerebral hemorrhage is associated with


{=B and C
~Tumors
~Cerebral amyloid angiopathy
~High cholesterol
~Liver failure}

A patient comes in with a sudden excruciating headache, or the worst headache of his life. His
syndrome worsens while doing activities that increase intracranial pressure like straining while
passing stool or having sex. The most common etiology of his condition is?
{=Aneurysm
~Hypertension
~Necrosis
~Pnemonia
~Transtentorial herniation}

A patient presents with symptoms indicative of increased intracranial pressure. Tests on his brain
indicates a lesion with a center that has undergone liquifactive necrosis with a surrounding fibrous
capsule. These does not seem to be any spread into the ventricles. What is the most common
etiological agent of his disease?
{=Streptococci
~Mycobacterium tuberculi.
~None of the above
~All of the above are equally common
~Bacteroides fragilli}

A patient presents with rapidly progressive dementia and myclonus. He reports to have had a
corneal transplantation recently. Upon taking a sample of the brain tissue, the pathologist would
most likely see:
{=PrPs plaques and spongiform degeneration
~Hemosiderin and increased neutrophilic infiltration
~Liquifactive necrosis and abscesses
~One of the above
~A and C}
A patient presents with unilateral vision loss, motor and sensory abnormalities, spasticity, improper
conjugate eye movements. Upon examination of the brain, it is detected that there are gray plaques
around the ventricles. Which of the following would most likely be elevated in his disease?
{=Y globulins
~A globulins
~Neutrophils
~Calcium levels
~None of the above}

A patient was recently placed on a therapy that overcorrected his hyponatremia. Which of the
followings diseases has a pathology most similar to his condition?
{=Multiple sclerosis
~Leukodystrophies
~Pneumonia
~Astrocytoma
~Meningioma}

A 68 year old male presents with loss of memory, cognitive impairment and demential. It is
discovered that there is an APP gene mutation associated with his condition. Which of the following
is not a key feature of his disease?
{=None of the above
~Ab deposition
~Tau proteins
~Senile plaques
~Hirano bodies}

A patient presents with slowness of movement and muscular rigidity, coarse tremor of the distal
extremities even at rest, expressionless face, reduced rate of swallowing, stooped posture, and
dementia. Which of the following is not a feature of his disease?
{=Neurofibrillary tangles
~Loss of pigmentations
~Lewy bodies
~Viral encephalopathy as an etiological factor
~Oxidative stress induction}

A patient presents with truncal ataxia, unsteady gate and nystagmus. He has been a drinker for a
while now. He distorted memory which he compensates for by confabulation. What histological
feature is seen in his disorder?
{=Degeneration of neurons in the medial dorsal nucleus of the thalamus
~Neurofibrillary tangles
~Excess accumulation of tau proteins
~Atrophy of the frontal caudate and putamen of the brain
~None of the above}

A 6 year old child presents with a tumor that is gray and friable within the vermis of the medulla.
Microscopically there appears to be small blue cells. His symptoms appear to be associated with
cerebellar dysfunction. If untreated, what would be the most likely prognosis of his tumor?
{=Poor
~Benign
~Undetermined
~Good because he is young
~None of the above}
A patient presents with weakness and wasting of his muscles. He drops objects easily and also
experiences spasticity and fasciculations .His lesion involves a loss of upper motor neurons. What is
the most common of death in these patients?
{=Pulmonary infections
~Fractures
~Dementia
~Visual impairment
~Wasting of the muscles}

Female neonate is noted to have pronounced enlargement of her head .she develops convulsions
MRI reveals excessive accumulation of cerebrospinal fluid. ventricular enlargement and atrophy of
the cerebral cortex. this development birth detect was most likely caused by ?
{=Atresia of the aqueduct of sylvus (Hydrocephalus)
~Hemorrhage
~Alzheimers
~Inflammations
~Neoplasia}

76 year old man admitted to hospital for evaluation of progressive memory loss and disorientation.
the pupils are small but react normally to light. muscle tone is normal a lumbar puncture returns
fleas, colorless, csf under normal pressure. an Electroencphalogram shows diffuse slowing a CT
scan of the brain reveals moderate atrophy. Which pathology have patient
{=Alzheimers
~Dementia
~Multiply sclerosis
~Inflammations
~Neoplasia}

Which of these definitions describe Multiple Sclerosis?


{=Disorder involving destruction of the myelin sheath predominantly in the in the white matter of
the CNS
~Resting tremor, muscle stiffness, akinesia, forward leaning posture
~Degenerative disease of the basal ganglia involving failure of the dopaminergic nigrostriatal
pathway
~Hereditary, degenerative disorder involving the basal ganglia and cerebral cortex featuring
involuntary movements (chorea)
~Weakness in all muscles of the body but with normal intellect and sensory functions}

Structural changes in the brain in Alzheimer's disease are due to:


{=Deposition of amyloid plaques and neurofibrillary tangles
~Degeneration of the basal ganglia
~Hypertrophy of the frontal lobe neurons
~Demyelination of the central nervous system
~Degeneration of dopamine receptor}

Bill has an injury of the cervical spinal cord. Cord swelling in this region may be life-threatening
because:
{=Diaphragmatic function may be impaired
~Increased intracranial pressure may occur
~Reflexes may be disrupted
~Bladder emptying will not occur
~Decreased intracranial pressure may occur}

Following head injury, this is a positive sign of transtentorial herniation.


{=Unequal pupillary reflex and decreasing level of consciousness
~Frontal headache
~Swelling of the head at the site of injury
~Increased pulse and respiratory rate
~Decreased blood pressure and increased pulse}

What motor function disorders have place in the absence of voluntary movements associated with
increased muscle tone of the right hand and foot after hypertensive crisis?
{=Central paralysis
~Peripheral paralysis
~Peripheral paresis
~Reflex paresis
~Central paresis}

Disorders of which of the following functions have place in hemiplegia?


{=Movement
~Equilibrium
~Vision
~Taste
~Hearing}

What is the designation of the state of the absence of motor function in half of the body after
cerebral insult?
{=Hemiplegia
~Ataxia
~Dyskinesia
~Myasthenia
~Paraplegia}

Which of the following disorders presents with muscular hypotonia, decreased reflexes, absence of
active movements in the left hand, transient pain and paresthesia in the area of the thoracic spine?
{=Peripheral paralysis
~Hyperkinesia
~Ataxia
~Central paralysis
~Brown-Sekar syndrome}

Which type of nervous system disorders do hypokinesia, decreased reflexes, muscular atrophy of
upper extremities in an infant after birth injury relate to?
{=Peripheral paralysis
~Neuritis
~Bulbar paralysis
~Myasthenia
~Central paralysis}

Point out the reason of Parkinson's syndrome with the development of progressive hand tremor,
muscle rigidity, and hypokinesia.
{=Black substance neurons degeneration
~Increased sensitivity of neuromuscular synapse
~Abnormal influence of the cerebellum on motor centres
~Increased motoneuron activity
~Reduced braking by the upper movement centres}
Point out a reason of spontaneous tremor of different parts of the body at rest and during voluntary
movements?
{=Disorders of basal ganglia function
~Disorders of the cerebellum functions
~Disorders of the mesencephalon functions
~Disorders of neuromuscular transmission of potential of action
~Disorders of motor cortex}

What CNS structure lesion presents with rapid, arrhythmic, involuntary movements of limbs and
trunk with decreased muscle tone, grimaces and smacking?
{=Striated body
~Medulla oblongata
~Cerebellum
~Corpus pallidum
~Black substance}

Which of the following statements characterising decerebral rigidity is correct?


{=Increased muscle tone of extremities extensors
~Arises after crossing the brainstem below the pons
~Is not associated with the loss of stimulation effects of higher motor centres on the brainstem
reticular system
~Is accompanied by braking y-motoneuron of the spinal cord
~Is accompanied by stimulation of o-motoneuron of the spinal cord}

Which of the following statements characterising decerebral rigidity is correct?


{=Is associated with the loss of braking influence of higher motor centres on the brainstem reticular
system
~Arises after crossing the brainstem below the pons
~Decreased muscle tone of extremities extensors
~Is accompanied by braking y-motoneuron of the spinal cord
~Is accompanied by stimulation of a-motoneuron of the spinal cord}

Damage to which brain stem levels causes decerebral rigidity?


{=Below the red nucleus
~Above black substance
~Below the Deiters nucleus
~Above the red nucleus
~Above the Yakubovich nucleus}

What are movement disorders associated with the cerebellum injury?


{=Asynergy
~Asymmetry
~Asthenia
~Athetosis
~Chorea}

What are movement disorders associated with the cerebellum injury?


{=Ataxia
~Athetosis
~Asymmetry
~Synergy
~Chorea}

What are movement disorders associated with the cerebellum injury?


{=Dysmetria
~Athetosis
~Asthenia
~Synergy
~Chorea}

Damage to what brain region is associated with precise movements impairments of upper
extremities while maintaining normal muscle strength, deep muscle feeling and coordination
mechanisms?
{=Supramarginal gyrus
~Precentral gyrus
~Broca's area
~Wernicke's center
~Calcarine fissure}

Name the brain structure, neurosecretory cells of which produce endorfines.


{=Hypothalamus
~Precentral gyrus
~Black substance
~Wall of the cerebral aqeduct of mesencephalon level
~Gelatinous substance of the spinal cord}

Which of the following neuropeptides is involved in pain responses?


{=Enkephalin
~Thyrotropin-releasing hormone
~Oxytocin
~Lipotropin
~Proopiocortyn}

Which of the following statements characterising the mechanism of pain emergence is correct?
{=Activation of gelatinous substance neurons weakens the pain sensations
~Pain is transmitted through the dorsal columns of the spinal cord
~Too strong stimulation of the skin thermoreceptors can cause pain
~Substance C is one of the mediators that is involved in transmitting pain sensation in the CNS
~Pain receptors are suppressed by bradykinin}

Which of the following statements characterising the mechanism of pain emergence is correct?
{=Substance P is one of the mediators that is involved in transmitting pain sensation in the CNS
~Inhibition of the gelatinous substance neurons weakens the pain
~Too strong stimulation of the skin thermoreceptors can cause pain
~Pain is transmitted through the dorsal columns of the spinal cord
~Pain receptors are suppressed by bradykinin}

Which of the following statements characterising the mechanism of pain emergence is correct?
{=Pain receptors are activated by bradykinin
~Inhibition of the gelatinous substance neurons weakens the pain
~Too strong stimulation of the skin thermoreceptors can cause pain
~Substance C is one of the mediators that is involved in transmitting pain sensation in the CNS
~Pain is transmitted through the dorsal columns of the spinal cord}

What is designation of the steady pain of the right half of the face augmented by slight irritation?
{=Neuralgia
~Causalgia
~Visceral pain
~Reflex pain
~Phantom pain}

What is designation of the severe pain that has for a long time irradiated to the missing limb after its
amputation?
{=Phantom pain
~Visceral pain
~Epicritical
~Protopathic
~Reflex pain}

What kind of pain occurs under the left scapula and left shoulder in case of acute myocardial
infarction?
{=Irradiating
~Visceral
~Causalgia
~Neuralgia of intercostal nerve
~Phantom pain}

Name the kind of pain in the left shoulder area in the patient with acute myocardial infarction?
{=Reflex pain
~Visceral pain
~Epicritical pain
~Protopathic pain
~Phantom pain}

What mechanism is responsible for the development of sever pain as a result of damage to
myelinated somatic nerves fibres?
{=Causalgia
~Visceral pain
~Reflex pain
~Phantom pain
~Nothing of listed above}

What pathological process develops in the eye cornea after impairment of the trigeminal nerve
integrity?
{=Ulcer
~Venous congestion
~Arterial hyperaemia
~Ischemia
~Edema}

What function of the nervous system is broken in case of impairment of the trigeminal nerve
integrity with the following development of the ulcer in the eye cornea?
{=Trophic
~Physical
~Of mediators
~Sensitive Autonomic
~Secretion}

What function of the nervous system is disordered in case of salivary gland denervation followed
by secretion of uniform, independent on the food nature saliva?
{=Trophic
~Autonomic
~Physical
~Secretion of mediators
~Sensitive}

What mechanism is responsible for pathogenesis of neurogenic dystrophy with peripheral nerve
damage?
{=All the given mechanisms
~Inadequate response to neurotransmitters
~Pathological impulsation from the damaged nerve
~Cessation of afferent information income from the denervated organ
~Cessation of the axoplasmic flow}

How are neurotrophic effects on tissues carried out?


{=Through all nerve types
~Through sympathetic nerves only
~Through parasympathetic nerves only
~Through the motor nerves only
~Through the sensitive nerves only}

What changes can be observed in limb myocytes after two weeks of crossing the sciatic nerve?
{=Increased sensitivity to acetylcholine
~Increased mitochondrial mass
~Increased activity of Krebs cycle enzymes
~Reduced number of cholinoreceptors
~Increased volume of lysosomes}

What changes in the organism results from damage to dorsal roots of the spinal cord?
{=Loss of muscle reflexes
~Development flaccid paralysis
~Increased sensitivity in denervated tissues
~Increase of skeletal muscle tone
~Increased skin reflexes}

What changes in the organism results from damage to dorsal roots of the spinal cord?
{=Loss of sensitivity in denervated tissues
~Development flaccid paralysis
~Increase of muscle reflexes
~Increase of skeletal muscle tone
~Increased skin reflexes}

What changes in the organism results from damage to dorsal roots of the spinal cord?
{=Loss of skin reflexes
~Development flaccid paralysis
~Increase of muscle reflexes
~Increased sensitivity in denervated tissues
~Increase of skeletal muscle tone}

What are the consequences of muscle denervation?


{=Atrophy
~Anaplasia
~Hyperplasia
~Hypertrophy
~Regeneration}
Specify the basic mechanism of nerve fibre degeneration following traumatic rupture?
{=Cessation of axoplasmic flow
~Loss of nerve excitability
~Damage to Ranvier nodes
~Amyelination
~Phagocytosis of nerve endings}

Which of the following neuronal functions will be disordered with abnormal synthesis of y-amino
butyric acid?
{=Braking
~Deafferentation
~Depolarization
~Desensitization
~Excitation}

Indicate the cause of the impaired ability of nerve fibres for excitation conduction.
{=Deficiency of cyanocobalamin
~Deficiency of ascorbic acid
~Effects of cholinesterase inhibitors on the nerve
~Reduction in blood catecholamine content
~The increase of Ca ions levels in the blood}

What effects may occur after the introduction of cholinesterase inhibitors in the organism?
{=Bronchioles spasm
~Intestinal atony
~Reduction of gastric juice secretion
~Increased pupils
~Tachycardia}

What effects may occur after the introduction of cholinesterase inhibitors in the organism?
{=Increased gastric juice secretion
~Intestinal atony
~Dilatation of bronchioles
~Dilatation of pupils
~Tachycardia}

What vitally dangerous state occurs in acute organic phosphorus poisoning?


{=Bronchospasm
~Coordination disorders
~Cerebral circulation disorders
~Blindness
~Tachycardia}

Which of the following reasons lead to increase of neurons excitability?


{=Inhibition of Na+/K+-pump function
~Increased membrane potential at rest
~Increase of K+ content in the cell
~Increase of Ca2+ levels in the blood
~Reduction of critical level of membrane depolarization}

What neurological disorder of central origin is observed in blood circulation disorders on the basis
of hypertensive crisis?
{=All is correct
~Hyperreflexia
~Pathologic reflexes
~Spastic paralysis
~Nothing is correct}

What disorder occurs as a result of the black substance destruction in Parkinson's disease?
{=Increase of muscle tone
~Increased formation of dopamine if the motor centres neurons
~Athetosis
~Hypokinesia of extremities
~Decrease of pain sensitivity}

What disorder results from the black substance destruction in Parkinson's disease?
{=Limbs tremor
~Amyotonia
~Increased formation of dopamine if the motor centres neurons
~Athetosis
~Decrease of pain sensitivity}

Which of the following clinical signs is not characteristic of the spinal shock with the trauma of the
cervical spine?
{=Aphasia
~Insufficiency of urination and defecation reflexes
~Loss of sensitivity
~Decrease of blood pressure
~Tetraparesis }

Disappearance of which kind of sense should be expected after the rupture of the right half of the
spinal cord only on the side of injury?
{=Proprioception [deep]
~Pain
~Tactile
~Temperature
~All the types of senses}

What mediators are responsible for transmission of excitation in the ends of preganglion
sympathetic fibres?
{=Acetylcholine, noradrenaline
~Noradrenaline, adrenaline
~Dopamine
~Noradrenaline
~Acetylcholine}

What is the mechanism of slowing heart rate during the intraoperative revision of guts?
{=Reflex increase of pneumogastric nerve tone
~Irritation of the frontal spinal cord roots
~Irritation of the rear spinal cord roots
~Reflex increase of sympathetic innervation
~Nothing is correct}

What urinary disorders arise with the transverse spinal cord injury at the level of L1-L3 segments?
{=Permanent incontinence
~Uroschesis (retention of urine)
~Periodic enuresis
~Urge for urination
~Nothing is correct}

A 25-year-old women during riding carousel developed nausea, vomiting, and increased sweating.
Activation of which receptor caused reflex development of these symptoms?
{=Vestibular receptors of semicircle channels
~Proprioreceptors of skeletal muscles
~Receptors of organ of Corti
~Otolith vestibular receptors
~Visual receptor}

A patient complains of rapid fatigue after a short time of usual work. In a standing position with his
eyes closed the patient swings and then loses balance. Skeletal muscle tone is decreased. Which of
the following brain structures is injured in this man?
{=Cerebellum
~Basal ganglia
~Hypothalamus
~Thalamus
~Cortex of precentral gyrus of large hemispheres}

A patient after trauma developed paralysis, disorders of pain sensitivity on the right, impaired pain
and temperature sensitivity on the left. What is the reason for this phenomenon?
{=Unilateral damage to the spinal cord on the right side
~Damage to the cerebellum
~Damage to the brain stem
~Damage to the mesencephalon
~Damage to motor areas of cerebral cortex}

A 43-year-old patient suffered a traumatic amputation of the left lower extremity four months ago.
At the time of examination the patient complains of feeling his amputees and constant, sever,
sometimes unbearable pain. What kind of pain has the patient?
{=Phantom
~Neuralgia
~Thalamic
~Causalgia
~Reflex }

A patient became unable to perform active movements with his left hand and left foot after acute
hemorrhagic stroke. Muscles tone of the extremities is increased, spinal reflexes are greatly
strengthened, reflexes zones are expanded. There is positive Babinsky reflex on the left. What kind
of movement disorder occurred in the patient?
{=Central paralysis
~Flaccid paralysis
~Reflex paralysis
~Flaccid paralysis
~Spinal shock}

A patient developed movements disorder that manifested as tremor, ataxia and movements asynergy
of, and dysarthria. What structure is the most likely affected?
{=Cerebellum
~Basal ganglia
~Limbic system
~Brain stem
~Medulla oblongata}
A patient with essential hypertension developed headache, tinnitus, and vomiting. The patient’s
arterial pressure elevated up to 220/160 mmHg. Examination revealed asymmetry on the right side
of the face, absence of random movements, increased tendon reflexes and muscle tone in the right
hands and feet. Which form of motor disorders is present in this case?
{=Hemiplegia
~Tetraplegia
~Paraplegia
~Hyperkinesis
~Monoplegia}

A 32-year-old patient came for medial attention complaining of general malaise, vomiting, pain in
the right lower part of the abdomen. After patient’s examination a doctor diagnosed acute
appendicitis. What kind of pain is present in the patient?
{=Visceral
~Somatic superficial late
~Somatic deep
~Phantom
~Somatic superficial early}

A 41-year-old man came for medial attention complaining of a rapid fatigue and muscle weakness.
Examination of the patient allowed to establish an autoimmune disease, as a result of which the
functional state of neuromuscular synapses receptors is impaired. The action of what mediators is
blocked with this disorder?
{=Acetylcholine
~Dopamine
~Serotonin
~Norepinephrine
~Glycine}

A 28-year-old man after gunshot wound of the shin developed ulcer on the damaged side. What is
the key factor in the pathogenesis of neurodystrophy in this case?
{=Trauma of the peripheral nerve
~Impairment of microcirculation
~Psychological stress
~Infection
~Damage to soft tissues}

In a victim of traffic accident the presence of rupture of a right half of spinal cord is diagnosed
(syndrome of Brown-Sekar). This conclusion is confirmed by such objectives:
{=Disappearance of proprioreceptive sensitivity on a right side
~Disappearance of and thermal sensitivity on a right side
~Disappearance of proprioreceptive sensitivity on left side
~Disappearance of proprioreceptive and pain sensitivity on left side
~Disappearance of proprioreceptive and thermal sensitivity on a right side}

What causes the epileptic attack?


{=Extra synchronization of impulses in the cortical neurons
~Slowing down the generation of impulses in the cortical neurons
~Absence of generation of impulses in the cortical neurons
~Slowing down the nuclei of extrapyramid system
~Imbalance between the black substance and globus pallidus}
Indicate the type of paralysis during which the basic mechanism of development is the oppression
of motoneurons of the proper muscle by impulses from the damaged sensible nerve?
{=Reflex paralysis
~Spastic paralysis
~Flabby paralysis
~Central paresis
~Peripheral paresis}

The most widespread form of motive violations is a paralysis. Paralyses are differed depending on
the defeat of different neurons. Name the type of paralysis at which the basic mechanism of
development is the oppression of motoneurons of the proper muscle by impulses from the damaged
sensible nerve.
{=Reflex paresis
~Peripheral paralysis
~Peripheral paresis
~Central paralysis
~Central paresis}

The passenger of airplane had dizziness, nausea, vomit, bradycardia, decline of arteriotony, general
weakness during flight in difficult terms. What is the main link of pathogenesis of pathological
process which appeared for a patient?
{=Increase of excitability of parasymphatical department of the vegetative nervous system
~Braking of vagus
~Activating of the system named hypothalamus-hypophysis-cortex of uperkidney gland
~Increasing of activity of the symphato-adrenalitic system
~Strengthening of adaptatiotrogical function of the sympatical nervous system}

EKCTPEMAJIHI CT
In the mechanisms of development of the comatose state at the alcoholic poisoning it is important
{=Blockade of respiratory enzymes
~Elevation of concentration of acidic products
~Violation of oxide phosphorylazing
~Violation of synthesis of neurotransmitters
~Slowing down of action of postsynaptic receptors}

What change from the ones listed below may become the reason of collapse?
{=Increase of tone of vagus
~Activating of the rennin-angiotensin-aldosteron system
~Activating of the sympatho-adrenal system
~Acceleration of cardiac rhythm
~Increased concentration of adrenalin in blood}

Specify violation of water-salt exchange, which arises up at development of hyposmotic comma?


{=Hyposmotic overhydratation
~Hypoosmolyarna overhydratation
~Hyperosmolic dehydratation
~Hyposmolic dehydratation
~Isoosmolic dehydratation}

Specify, what from the resulted examples belongs to the vascular form of shock:
{=Anaphylactic
~Primary hypovolemic
~Cardiogenic
~Tumiketical
~Traumatic }

The major link of pathogeny of traumatic shock is violation of:


{=Central regulation of circulation of blood
~Rheological properties of blood
~The volume of circulatory blood
~Arteriotony
~Retractive function of heart}

In the mechanisms of development of the comatose state at the alcoholic poisoning it is important
{=Blockade of respiratory enzymes
~Elevation of concentration of acidic products
~Violation of oxide phosphorylazing
~Violation of synthesis of neurotransmitters
~Slowing down of action of postsynaptic receptors}

What change from the ones listed below may become the reason of collapse?
{=Increase of tone of vagus
~Activating of the rennin-angiotensin-aldosteron system
~Activating of the sympatho-adrenal system
~Acceleration of cardiac rhythm
~Increased concentration of adrenalin in blood}

Specify violation of water-salt exchange, which arises up at development of hyposmotic comma?


{=Hyposmotic overhydratation
~Hypoosmolyarna overhydratation
~Hyperosmolic dehydratation
~Hyposmolic dehydratation
~Isoosmolic dehydratation}

Specify, what from the resulted examples belongs to the vascular form of shock:
{=Anaphylactic
~Primary hypovolemic
~Cardiogenic
~Turniketical
~Traumatic}

The major link of pathogeny of traumatic shock is violation of:


{=Central regulation of circulation of blood
~Rheological properties of blood
~The volume of circulatory blood
~Arteriotony
~Retractive function of heart}

A patient has been diagnosed withinfluenza. His condition became drasticallyworse after taking
antipyretic drugs.His consciousness is confused, AP is 80/50mm Hg, Ps is 140/m, body
temperaturedroped down to 35,80C. What complicationdeveloped in this patient?
{=Collapse
~Hyperthermia
~Hypovolemia
~Acidosis
~Alkalosis}
As a result of a trauma a patient has developed traumatic shock that led to the following disorders:
AP is 140/90 mm Hg, Ps is 120 bpm. The patient is fussy, talkative, pale. Such state relates to the
following shock phase:
{=Erectile
~Latent period
~Terminal
~Torpid
—}
After a road accident a victim has tachycardia, arterial blood pressure 130/90 mm Hg, tachypnoe,
the skin is pale and dry, excitation of central nervous system is observed. What shock stage is the
patient most likely in?
{=Erectile
~Terminal
~Torpid
~Preshock (compensation stage)
~Agony}

A 62-year-old patient has been hospitalizeddue to massive cerebral hemorrhage. Blood pressure is
70/30 mm Hg, heart rate is 120/min., respiratory rate is 4/min., unconscious, no response to external
stimuli. Such condition can be determined as:
{=Coma
~Shock
~Collapse
~Stress
~Agony}

A patient has been diagnosed withinfluenza. His condition became drastically worse after taking
antipyretic drugs. His consciousness is confused, AP is 80/50 mm Hg, Ps is 140/m, body
temperature droped down to 35, 80C. What complication developed in this patient?
{=Collapse
~Hyperthermia
~Hypovolemia
~Acidosis
~Alkalosis}

A patient with extensive burns of torso skin exhibits signs of severe intoxication. What stage of the
burn disease is this typical for?
{=Burn toxemia
~Burn shock
~Burn infection
~Burn emaciation
~Terminal}

An unconscious patient was deliveredby ambulance to the hospital. On objectiveexamination the


patient was found topresent no reflexes, periodical convulsions, Irregular breathing. After
laboratoryexamination the patient was diagnosed with hepatic coma. Disorders of the
centralnervous system develop due to accumulationof the following metabolite:
{=Ammonia
~Urea
~Glutamine
~Bilirubin
~Histamine}
A patient has been diagnosed with influenza. His condition drasticallyworsened after taking
antipyretic drugs. Heis unconscious, AP is 80/50 mm Hg, Ps is140/m, body temperature dropped
down to35,80C. What complication developed inthis patient?
{=Collapse
~Hyperthermia
~Hypovolemia
~Acidosis
~Alkalosis}

A 27-year-old patient with injury tothe neck has lost approximately 30% ofthe blood volume. The
patient’s condition issevere: blood pressure is 60/40 mm Hg, heartrate is 140/min., respiratory rate
is 30/min. conscious. Characterize the condition of thepatient’s circulatory system:
{=Hypovolemic shock
~Cardiogenic shock
~Collapse
~Coma
~Arterial hypertension}

A 30-year-old man with diabetes mellitus type I was hospitalised. The patient is comatose.
Laboratory tests revealed hyperglycemia and ketonemia. What metabolic disorder can be detectedin
this patient?
{=Metabolic acidosis
~Metabolic alkalosis
~Respiratory acidosis
~Respiratory alkalosis
~Normal acid-base balance}

After a road accident a victim has tachycardia, arterial blood pressure 130/90 mm Hg, tachypnoe,
the skin is pale and dry, excitation of central nervous system is observed. What shock stage is the
patient most likely in?
{=Erectile
~Terminal
~Torpid
~Preshock (compensation stage)
~Agony}

Shock (mean BP less than 60 mmHg) can be caused by:


{=Low cardiac output
~Low intravascular volume
~Pleural effusion
~Decreased peripheral vascular resistance}

A 30 year old man is admitted to the Emergency Ward with sudden onset of severe shortness of
breath, no chest pain, diffuse wheezing, cyanosis, and a BP of 70/30, Pulse = 100. This event
occurred soon after a meal. What is the likely diagnosis:
{=Anaphylaxis
~Acute pulmonary embolism
~Tension pneumothorax
~Myocardial infarction}

In a subject in shock with low body temperature, cold and clammy extremities, low urine output,
and elevated lactic acid level in the blood, the most likely cause of shock is:
{=Sepsis
~Pulmonary embolism
~Asthma
~CHF}

A patient who is in hypovolemic shock has the following clinical signs: Heart rate 120
beats/minute, blood pressure 80/55mmHg80, slash, urine output 20 mL/. After administering an TV
fluid bolus, which of these signs if noted by the healthcare provider is the best indication of
improved perfusion?
{=Urine output increases to 30 mL/hour30mL/hour30
~Right atrial pressure increases
~Systolic blood pressure increases to 85 mmH
~Heart rate drops to 100 beats/minute}

A patient is admitted to the emergency department after sustaining abdominal injuries and a broken
femur from a motor vehicle accident. The patient is pale, diaphoretic, and is not talking coherently.
Vital signs upon admission are temperature 98. 0 F (36., 3 C), heart rate 130 beats/minute,
respiratory rate 34 breaths/minute, blood pressure 50/40 mmHg, The healthcare provider suspects
which type of shock?
{=Hypovolemic
~Distributive
~Cardiogenic
~Neurogenic }

A patient is being treated for hemorrhagic shock secondary to multiple rib fractures and a lacerated
liver. Two units of packed red blood cells have been administered. Which of these measurements is
an indication the patient has received adequate volume replacement?
{=Decreased serum lactate
~Decreased right atrial pressure
~Increased serum creatinine
~Oxygen saturation 90%}

Identify the FALSE statement:


{=Septic shock is associated with cold clammy skin
~Hypovolemic shock is associated with low cardiac output
~Cardiogenic shock is associated with arrhythmia, MI, and myocarditis
~Extracardiac obstructive shock is associated with cardiac tamponade, pneumothorax, and
pulmonary emboli}

Which stage of shock is associated with the worsening of tissue hypoperfusion and onset of
worsening circulatory and metabolic imbalances, including acidosis?
{=Progressive stage
~Initial nonprogressive phase
~Developing phase
~Irreversible stage}

What are characteristics of the irreversible stage of shock?


{=Tissue and cell damage is too great tissue and necrosis of the tissue will occur even if the
underlying hemodynamic defect is corrected
~The worsening of tissue hypoperfusion and the onset of worsening circulatory and metabolic
imbalances, including acidosis
~The body tries to initiate compensatory mechanisms
~Nothing can correct the hemodynamic defect}
Your patient is in the progressive stage of shock. If the shock is not corrected and tissue hypoxia
occurs, what would happen with the patients metabolism?
{=The tissues will undergo anaerobic metabolism, creating lactic acid and lowering the tissue pH
~Vasomotor reflex of arteriolar constriction that reduces pooling of blood in the microcirculation
~Metabolism would be unaffected by the lower oxygen level for the 2-3 hours}

What characteristics distinguish septic shock from hypovolemic and cardiogenic shock?
{=Vasodilation will lead to warm flushed skin in septic shock patients
~Septic shock patients will present with cool skin due to vasoconstriction
~Patients with septic shock will have a bluish color to their skin
~Septic shock patients present with pulsus paradoxus}

True or False: Septic shock should present with a normal or high cardiac output.
{=True
~False}

True or False: Hypovolemic shock should typically present with a reduced cardiac output.
{=True
~False}

Anaphylactic schock is associated with which type of hypersensitivity?


{=Type I (immediate)
~Type II (cytotoxic)
~Type III (immunocomplex)
~Type IV (cell mediated)}

Which is true in regard to systemic anaphylaxis?


{=Systemic anaphylaxis occurs due to mast cell degranulation and release of mediators that causes
acute vasodilation which can ultimately lead to increased vascular permeability, bronchospasms,
and hypotension
~Anaphylaxis occurs due to the IgE reacting with the foreign object ultimately leading to release of
vasoconstictors. The vasoconstrictors lead to severe hypertension and bronchospasms
~Systemic anaphylaxis is associated with vomiting, abdominal cramps, diarrhea, and hypertension}
PATHOPHYSIO EXAM
(2ND PART) Complied
1. What diseases of sex-linked type of inheritance?
e Bruton’s agammaglobulinemia
e diabetes insipidus
G6PD deficiency
Haemophilia A&B
Ichthyosis
muscular dystrophy
colour blindness
juvenile glaucoma.

2.What diseases of autosomal-dominant type of


inheritance?
e Familial Hypercholesterolemia
e Achondroplasia
e Gilbert’s disease
e Huntington’s disease
e polycystic kidneys
e Colon polyposis
e Sickle cell anaemia
e Willebrand disease
e spherocytosis
e Thalassemia

1|Page
3.What diseases of autosomal-recessive type of
inheritance?
e Cystic fibrosis
e Sickle cell anaemia (SC)
e Tay Sachs disease
e Enzymopathy
e Hemoglobinopathy
e Erythrocytopathy
e Coagulopathy
e Agammaglobulinemia
e Glycogen storage disease
e Galactosemia
e Phenylketonuria
e Muscular dystrophy

4. What diseases of congenital (teratopathy) type of


inheritance?
Down syndrome (trisomy 21)
Patau syndrome (trisomy 13)
Edward syndrome (trisomy-18)
Turner syndrome 45X
Klinefelter syndrome-47 XXY

2|Page
5)What forms of endogenous mutagens do you know?
e Enzymatic process controlling DNA and repair
e Physio-chemical process
e Free radicals
e Genetic factor
e Diet

6)What diseases caused by chromosomal aberrations?


e Down syndrome
e Patau syndrome
e Edward syndrome
e Turner syndrome
e Klinefelter syndrome
e Di George syndrome
e Philadelphus chromosomal syndrome

7. The main methods of diagnosis of hereditary


diseases?
e Cytochemical
e Biochemical
e Molecular

8. The main methods of treatment of hereditary


diseases?

3|Page
e Gene therapy
e Protein/enzyme replacement
e Drug treatment
e Transplantation
e Tissue transplantation
e Stem cell transplantation

9. The main methods of prevention of hereditary


diseases?
e Check regulatory for disease
Follow a healthy diet
Get regular exercise
Avoid smoking tobacco & too much alcohol
Get specific genetic testing

10. Levels of lg was changed during examination of


immune status in 5-year-old boy. Reactions of T-
Lymphocytes are normal. This is inherited sex-linked
disease. Was diagnosed. Bruton’s disease. What
manifestations of this

pathology do you know?


e Absence of b- cells and plasma cells in blood
e Absence of b- cells and plasma cells in lymph node
e Decrease in IgG, IgA, IgM
e T-system of lymphocytes is not violated

4|Page
11. A 9-year-old girl was getting breast feeding during
the first year of life. She suffered prolonged severe

pneumonia in the end of the first year. She started


walking late. Was diagnosed Luis-Barr syndrome.
Which changes are defining this syndrome?
Cerebellum ataxia
Telangiectasia
IgA absent
T-lymphocytes reduce
Predispose to allergy

12. What symptoms are characterised for the Wiscott-


Aldrich’s syndrome?
e Small red rash consisting of “dots” under the skin (these
are called petechiae)chronic infections.
® eczema (atopic dermatitis)
e frequent and easy bleeding that can occur: from the
nose, from the mouth and gums also in bowel
movements.
e frequent and easy bruising.

13. What symptoms are characterized for the


DiGeorge syndrome?

5|Page
Congenital heart disease
Cleft palate
Recurrent infection
Hypoplasia or aplasia of thymus
Convulsions
Hypocalcaemia
GIT problems
Cyanosis
Hypo parathyroidism
Hypothyroidism

14. What can increase permeability of hemato


encephalic barriers?
The blood-brain barrier becomes more permeable
during inflammation
Allowing antibiotics and phagocytes to move across the
BBB
Virus and bacteria infection
Leukotrienes

15. Which factors can stimulate phagocyte activity?


Cytokines,
complement proteins
Antibodies
Immune complexes

6|Page
e Hormones (Adrenalin, thyroxine, mineralocorticoid,
sexhormone)
e Biological active substance

16. Which factors decreases phagocyte activity?


e Physical factors- ionising radiation
e Chemical factor -poison
e Biological factor-genetics
e Excessive glucocorticoid suppression
e Inhibitors of DNA
e Glycolytic poisons

17. Which clinical forms of allergic reactions belong to


the | type (anaphylactic) by Gell and Coombs?
e Spasm of smooth muscles of internal organs are
clinically manifested by Bronchospasm (cough,
breathlessness)
e Spasm of the gastrointestinal tract muscles (spasmodic
pain, nausea, vomiting)
e Hemorrhage syndrome
e Serum disease
e Pollinosis

7|Page
18. Which clinical forms of allergic reactions belong to
the Ill types — immunocomplex reactions by Gell and

Coombs?

e Hemorrhagic vasculitis develops


e Acute glomerulonephritis
Allergic dermatitis
Alveolitis
Rheumatoid arthritis
Systemic lupus erythematosus

19. A 18-year-old patient has insulin-dependent


diabetes mellitus | type. It’s cytotoxic type of allergy.
Check pathology of Cytotoxic components e.g.
antibodies to tissue.
e Cytotoxic antibodies: IgG & IgM
e Patho chemical state: BAS activation -phagocytosis- N-
killers
e Pathophysiological state: cell lysis due to activation of
complement and N- killers together with phagocytosis of
the cells covered with antibodies.

20. Which of the following cytokines are involved in


the reagin type of allergy:
e Induction of allergen specific IgE
e IL-4 upregulates IgE production

8|Page
Gamma — IFN down regulates IgE

21. What mediators of allergic reactions have a direct


cytotoxic effect:
activated complement
free radicals
peroxides and lysosomal enzymes
NK-cells

22. What are the diseases developing by the


immunocomplex type:
hemorrhagic vasculitis
dermal vasculitis
acute glomerulonephritis
rheumatoid arthritis
systemic lupus erythematosus
bronchial asthma

23. What effector mechanisms are involved in the


reagin type of allergy:

release of active substance from mast cells


synthesis of new mediators

9|Page
e activate kallikrein kinin system

24. Which biologically active substances of mast cells


are newly synthesized:
e histamine
e cytokines
e granulocyte macrophage colony-stimulating factor (GM-
CSF)
e leukotrienes
e heparin

25. What are the cytotoxic type allergy effector


mechanisms:
e Target cell
e phagocytosis
e complement-dependent cytotoxicity
e (CDC) and (3) ADCC

26. What are the delayed-type hypersensitivity


mechanisms:
e Type IV hypersensitivity
e antibody-mediated

10|Page
e T-cell, monocytes, macrophages
e inflammation or cytotoxicity

27. List the mechanisms of action of eicosanoids:

e classes of eicosanoids: prostaglandins, thromboxane’s,


and leukotrienes Role in:

1-Inflammation, fever, promotion, blood pressure regulation

2-blood clotting, prostacyclin, thromboxane and


leukotrienes-either omega 3 or omega 6 picosanoic pufas

28. Ig E are involved in:


e mast cells
e IgE dependent and independent, immunoglobulin
e plasma cell, Schistosoma manzoni, trichinella spirolis,
Fasciola hepatica

29. Indicate how the tone of the airways changes with


bronchial asthma and how it affects lung function:
e increase the tone of muscle
e bronchoconstriction by airway muscle

11|Page
e obstruction of airflow by intraluminal mucus and
inflammation and remodelling of the airway wall
e Second part:

1-high lung volumes


2- trapping air behind occluded or narrowed small airways
3-the airways are inflamed (swollen) and sensitive and

produce lots of thick mucus.

30. List the clinical signs of serum sickness:


e Fever, General ill feeling, hives, itching, joint pain, rash,
swollen lymph node
e Facial swelling, rash

31. In the implementation of delayed-type


hypersensitivity take part:
e Tuberculin reaction
e bacterial allergy
e contact dermatitis

32. Eicosanoids are:

e Prostaglandins
e Thromboxanes
® prostacyclin

12|Page
e Leukotrienes

33. What mediators are associated with an increase in


vascular permeability:
e products of degranulation of tissue (histamine,
serotonin)
e prostacyclin’s, elastase, collagenase, acidosis

34. To implement the damaging effects of the


immune complex, you need:
e violation of the complement system,
e functional defects of the mononuclear phagocyte
system

35. What are the main mediators involved in the


implementation of delayed-type hypersensitivity:
e Lymphokines (interleukin-2, macrophages activation
factor)

36. What are some examples of delayed-type


hypersensitivity:
e transplant rejection reaction
e contact dermatitis
e tuberculin reaction

13|Page
e bacterial allergy
e auto allergic diseases
e antitumor immune reactions

37. The implementation of the histamine type of


pseudo-allergy occurs as a result of:
e Aspirin drug
e NSAids
e Some food
e Benzoate food additive

38. Intravascular deposits of immune complexes


contribute to:
e Glomerulonephritis
e HepatitisB
e Serum sickness

39. What cytokines are involved in the


implementation of delayed-type hypersensitivity:
e Lymphokines (interleukin-2, macrophages activation
factor)
e contact dermatitis
e chemokines
e |IFN-Y

14|Page
e TNF-a and LT
e |L-3/GM-CSF

40. What is an example of delayed hypersensitivity:


e dermatitis
e tuberculin skin test reactions
e granulomatous inflammation (e.g., sarcoidosis, Crohn
disease)
e allograft rejection, graft versus host disease, and
autoimmune hypersensitivity reactions

41. Due to venous air embolism air may be sucked


into systemic veins under the following
circumstances?
e Air embolism occurs when large veins of the head and
neck are wounded. These veins, pressure in which is
close to zero or negative, being open suck air from the
outside. As a result, embolism in the vessels of
pulmonary circulation develops.

42. What manifestations of venous hyperaemia?


e enlargement of organs or tissues due to swelling
(tumor);

15|Page
e cyanosis as a result of hypoxia and accumulation of non-
oxygenated haemoglobin;
e edema caused by high hydrostatic pressure and
increasing permeability of vessel walls due to non-
oxygenated condition;
e local hypothermia;
e subjective unpleasant sensations of swelling and pain.
The microscopic picture is characterized by: dilatation of
small arteries, arterioles, veins, capillaries; blood flow
deceleration due to increased blood viscosity; transudate
formation; thrombosis; atrophic and dystrophic changes in
the tissue; of amino excessive growth of the connective
tissue sclerosis.

43. What is microscopic picture characterized by


arterial hyperemia?
dilatation of small arteries, arterioles, veins, capillaries;

increased number of functioning vessels;

¢blood flow acceleration;

¢ opening of inactive capillaries;

division of blood flow into two parts. central flow of the


blood cells and peripheral flow of the plasma.

44. Describe Virchow’s triad.

16|Page
e Injury of the vessel wall (endothelium).
e Imbalance between coagulative, anticoagulative and
fibrinolytic blood systems (hypercoagulability).
e Blood flow deceleration.
e It explains why thrombosis of veins occurs more often
than that of arteries. All the factors creating these
conditions are etiological factors of thrombosis. They
are exogenous and endogenous.
e The exogenous ones are physical (trauma, electric
current), chemical (tobacco smoking, side effects of
some drugs) and bio- logical (infectious, immune)
agents, which damage the blood vessels or change the
rheological properties of the blood.
e Atherosclerosis, arterial hypertension and allergy may
cause an injury of the vessel wall and predispose to
thrombosis.
e Diabetes mellitus and obesity change coagulation and
predispose to thrombosis.

45. What manifestations of ischemia.

e ECG changes such as deviations of the S-T segment,


changes of the ORS complex and T wave.
e Infarction develops in the area of the myocardium,
which is supplied by the blood through damaged
vessels.

17|Page
e Morphologically, at first structural impairment of
mitochondria is noted. Then nuclei swelling or pyknosis
occurs.
e The transverse striation of the muscle fibres appears.
Cardiomyocytes lose glycogen and potassium.
e H+ ions are accumulated. The main consequence of
infarction is local coagulative necrosis and myocyte lysis.
e Myocardial infarction causes acute cardiac insufficiency
that leads to hypoxia, acidosis, dysfunction of the brain
and other organs, and finally to death.
e Pain localization on the upper left part of the body
behind the breastbone.

46. What significances of arterial hyperemia.


e Inthe majority of cases arterial hyperemia is
accompanied by intensification of metabolism and
organ functioning. However, unfavourable outcome is
also possible. Thus, sclerotic vessel dilatation may result
in rapture and hemorrhage. It is especially dangerous in
the brain.
e Arterial hyperemia is of great significance in the course
of inflammation.

47. What manifestations of thrombosis do you know?


e Thrombosis is created in the process of evolution as a
defense reaction arresting bleeding and blood loss after
mechanical traumas and vessel ruptures.

18|Page
e thrombosis plays a negative role if it causes ischemia.
e Infarction is an area of ischemic tissue necrosis and is
the gravest outcome of thrombosis.
e Infarction of the heart, lung and brain collectively
account for more deaths than all forms of cancer and
infectious diseases taken together.
e It is usually caused by thrombotic (or thromboembolic)
occlusion of vessels.
e Thrombosis of veins has a chronic course causing venous
congestion and inflammation (thrombophlebitis).

48. What manifestations of embolism do you know?


e Embolism always has a negative effect.
Thromboembolism of the pulmonary artery (the clinical
picture: dyspnea, acute pain in the chest, cyanosis,
swelling of the cervical veins) is a frequent cause of
death in hospitalized patients.
e Heart disease (myocardial infarction, rheumatic heart
disease, arrhythmia) increase the risk of embolism.

49. Check reasons spastic type of ischemia.


e Functional disturbances of the vasoconstrictive and
vasodilative apparatus of the coronary vessels.

19|Page
50. What reasons of fat embolisms do you know?
e The most common cause of a fat embolism is a broken
long bone, when a bone breaks, fat from the bone
marrow can seep out into the bloodstream.
e Other causes include major surgery (like hip and knee
replacements), pancreatitis, burns, child birth, bone
marrow biopsies, soft tissue damage, and non-traumatic
procedures or conditions like liposuction, fatty liver, or
sickle cell disease. Anything that introduces fat into the
bloodstream can result in a fat embolism if the fat
reaches a blood vessel it can't fit through.

51. What reasons of gas embolisms do you know?


e Gas embolism is due to a sharp drop in atmospheric
pressure: (a) from elevated to normal (in workers of
caissons and divers) or

(b) from normal to low (with extremely rapid rise to a height


or during depressurization of the cabin in high altitude
aircraft).

e In these cases, the solubility of gases in tissues and


blood decreases and saturation occurs. The gases
(primarily nitrogen) go from the dissolved state to

20|Page
gaseous and become emboli, which occlude the
capillaries of the systemic circulation.

52. What reasons of air embolisms do you know?


Air embolism occurs when large veins of the head and neck
are wounded. These veins, pressure in which is close to zero
or negative, being open suck air from the outside. As a result,
embolism in the vessels of pulmonary circulation develops.

53. Oxygen-derived metabolites are released from


activated neutrophils and macrophages and include
e superoxide oxygen (O’2), H202, OH’ and toxic NO
products.
e These oxygen-derived free radicals
e have the following actions in inflammation...
e Endothelial cell damage and thereby increased vascular
permeability.
e Activation of protease and inactivation of antiproteases
causing tissue matrix damage.
e Damage to other cells like RBC.

54. Nitric oxide (NO) NO plays the following roles in

mediators of inflammation.
1. Vasodilatation (relaxation of vascular smooth muscle

21|Page
2. Antagonism of all stages of platelet activation (adhesion,
aggregation and degranulation)

3. Microbicidal or cytotoxic action in activated macro phages

4, Reduction of leukocyte recruitment at inflammatory sites.

55. In the development of pain with inflammation


involved:
e Recruited immune cells release further mediators
including cytokines and growth factors Pain mediators
included: adrenocorticotropic hormone (ACTH),
glucocorticoids, vasopressin, oxytocin, catecholamines,
brain opioids, angiotensin Il, endorphin/

encephalin, vasoactive intestinal peptide (VIP), substance P,


eicosanoids (e.g., prostaglandins, leukotrienes), tissue
kininogens (bradykinin), histamine.

56. The products of arachidonic acid metabolism via


the cyclooxygenase pathway are:
"classical" prostaglandins (PGD2, PGE2, PGF2a);

thromboxane (TXA2, TXB2,);

prostacyclin (PG 12 ).

57. An increase in vascular permeability during


22|Page
inflammation causes:

In acute inflammation there is striking increase in


permeability of the vessels to proteins.
The loss of protein from plasma reduces the
intravascular osmotic pressure and increase osmotic
pressure of interstitial fluid. This further cause marked
out flow of fluid and its accumulation in the interstitial/
Extravascular tissue.
The net increase of extravascular fluid is called
inflammatory oedema
Formation of endothelial gaps in venules:
Normally, the endothelial cells of blood vessels are
fused by tight intercellular Junctions.
In inflammation, these are loosened to permit outflow
of fluid and protein.

Most chemical mediators of inflammation cause an


increase in vascular permeability by opening inter-
endothelial junctions.
The inter-endothelial gaps are produced by contraction
of endothelial cells, which results in widening of the
junction.

58. From the phospholipids of cell membranes are


formed:

23|Page
59. From phospholipids of cell membranes are
formed:
e Arachidonic acid
e Leukotrienes
e Prostaglandins
e Thromboxanes

60. Statements characterizing histamine:


e Local Vasodilation
Increased vascular permeability
Contraction of smooth muscle
Dilation of blood vessels
Mediator in inflammation process

61. Following emigration from blood vessel,


leucocyte migration to the site of injury is mediated
by:
e -the capillary wall between endothelial cells
e the basal membrane

62. Which humoral mediators do you know?


e complement system
e kinins (bradykinin, kallidin)

24|Page
e Hageman factor

63. Local manifestations of inflammation?

tumor (swelling)
rubor (redness)
calor (heat)
dolor (pain)
funcito leasa (functional disorder)

64. Systemic manifestations of inflammation?


e Increase in body temperature (fever) is one of the most
prominent systemic manifestations, especially
when inflammation is associated with bacteraemia.

Endocrine system activation.


Hypothalamus involvement in fever development,
adenopituitary and adrenal cortex activation.
Mineralocorticoids stimulate inflammation, and
glucocorticoids inhibit it.
Hematological syndrome is characterized with
neutrophilic leukocytosis, which is a common feature of
inflammatory reactions, especially those induced by
bacterial infection. The leukocyte count usually climbs
to 10 G/1 and more.
Leukoformula shows a regenerative left-side nuclear
shift.

25|Page
e Increased number of neutrophils in leukoformula and
appearance of their im- mature forms are an evidence
of hemopoiesis activation in the bone marrow.
e Blood sedimentation rate (BSR) is accelerated.
e Immune system activation.
e Nervous disturbances, insomnia.
e So, inflammation, immunity and fever are tightly
connected.

65. What components of exudate do you know?


e plasma proteins (albumins, globulins, fibrinogen)
e serum, fibrin, wbc

66. What components of transudate do you know?


e Plasma
e Glucose
e mesothelial cell and cellular debris

67: Names factors which suppress proliferation?


e inhibition of FOXM1 transcription factor
e Tumor suppressor genes
e Chalones

26|Page
68: Acute increase of body temperature, dyspnea,
tachycardia, nausea, convulsions, and loss of
consciousness developed in a worker, working in the
thick uniform in summer. In a worker hyperthermia
develops. What reasons of hyperthermia do you
know?
e limitation of heat emission
e activation of heat production
e disconnection (uncoupling of oxidation and
phosphorylation)

69: A patient has flu. Which manifestations will


characterize 1 stage of fever?
hp>he (p production, e emission)
body temperature increase
Hairs become elevated
chills develop
Diuresis is increased
arterial BP rise
e Vasoconstriction

70: Manifestations second stage of fever.

27|Page
maintaince of high body temperature with fluctuation in
morning and evening.
Hp=He
diuresis is decrease
Retention of NA, CL and water is detected
Vasodilation

71. Extensive myocardium infarction was diagnosed in


a patient. By the end of day body temperature
increased to 38 Co. The number of leucocytes
increased to 11.6x109. Which internal pyrogens do
you know?
Interleukin 1-6
production of 1°2° alteration
Antigen antibody complex
PGE1,2
tumor necrosis factor(TNF)
Metabolites of steroid hormones
Leukocyte pyrogen that are product of neutrophils and
macrophages
Interferons -alpha
gp130

72. A sick man felt on work general weakness,


temperature increased to 38 C°. In evening the sick
28|Page
felt sever fever, temperature reached 38.8 C°. Family
doctor diagnosed influenza. What manifestations
third stage of fiver do you know?
e He>Hp
e Loss of water
e Na and cl excretion increase
Vasodilation
Intense sweating

73. Positive effects of fever?

e multiplications of many viruses are inhibited


e phagocytic activity of macrophages and neutrophiles
increase
e Antibody synthesis become more intense

74. Negative effects of fever?


e Disturbance of patient’s general condition - malaise,
headache
e metabolic disorders
e Increase heat load or decrease in bp
e Disorder of CNS
e Fetal development disorder

29|Page
75. What breathing is observed at the temperature of
body 40°C?
Tachypnoea (polypnea)
rapid or quick breathing
Hyperventilation

76. Which primary Pyrogens do you know?


Lipo polysaccharides
Virus and other microbes
Exotoxin, and erythrogenic toxin
Exogenous protein
Antigen breakdown products (etiocholanolone)

77. What substances will play role secondary


Pyrogens?
Interleukin 1-6
production price f 1°2° alteration
Antigen antibody complex
PGE1,2
Tumor necrosis factor (TNF)
Metabolites of steroid hormones
Leukocyte pyrogen that are product of neutrophils and
macrophages
Interferons -alpha
gp130

30|Page
78. Both benign and malignant tumours cause local
effects on the host due to their size or location.

Malignant tumours due to rapid and invasive growth


potential have more serious effects. Check of the local
effects of tumors.
Compression
Tissue destruction
Hemorrhage
Mechanical
Infraction
Bleeding
Obstruction
Ulceration
Abnormal biochemistry
Destruction of adrenal cortex by tumor leads to increase
aldosterone and cortisol secretion - Cause lactic
acidosis.
Serum lactate DH increase
Like increase serum rate concentration
Renal failure may occur

31|Page
79. A 56-years-old patient, who had contact with
diethyl nitrosamine at his work place, complains

of pain in right subcostal area, weakness, loss of


appetite, and decreased workability. At examination
of this patient: surface of his liver is rough;
splenomegaly and ascites are present in him; his body
temperature is 37.20C; in his hypochromic anemia
were found. Blood analysis ESR is 25 mm/hour,
besides neutrophilic leukocytosis. Was diagnosed
Cancer of liver, because this cancerogen has
organotropic effect irrespective of the way of entry.
Which manifestations are characterized malignant
cancer?

@ Monoclonality of origin
e Malignant tumors undergo a fast increase in their size.
e The cell mass that forms undergoes very little or no
differentiation at all, thereby forming a big lump of cells
that are totally useless.
e Malignant cells have a tendency to attack nearby cells
and either destroy them or make them malignant.
e These cells may also travel to other parts of the body via
lymph or blood vessels and affect other normal tissues,
thus causing secondary cancer at the other site.

80. Cardinal signs of benign tumor are:

32|Page
e Well different cell
e Retain typical structure of tissue
e Grow slowly may stop and sometimes reverse
development
e Have a capsule and grow expansily, here not lethal

81. Name endogenous cancerogens:

@ Steroid hormones (folliculin)


@ Bilic acids
e Cholesterols
@ Free radicals
@ Peroxides
© Derivatives of amino acid tryptophan (indole)

82. Check of the systemic effects of tumors:


e Neoplasm is local manifestation of systemic disease
e Interrelation between organism and malignant tumor
e Depressed reactivity
e weight loss
e poor appetite
e fatigue and cachexia
e excessive sweating
e Anemia
e Uncontrolled synthesis of hormones sometimes occurs
e specific paraneoplastic phenomena

33|Page
83. What cardinal signs of malignant tumor do you
know?
Independent of growth
Endless growth
Anaplasia
Metastatic expansion

84. Manifestations of biochemical anaplasia:


Quantity of mitochondria is decreased — activity of
oxidative enzyme depressed
Glycolysis is intensified
Lactic acid accumulated
Enzymatic decomposition
Cellular division increases
If RNA virus (etiology factor) — revertase appears
Synthesis of nucleic acid and protein intensified
Synthesis of histones decreased
Intensive synthesis of oncoprotein is characteristic
Embryonic alpha protein synthesised

85. Manifestations of immunological anaplasia:


Antigenic composition of malignant cell changed
Antigenic simplification
Tissue specific, individual antigens lost completely
Antigen complication of malignant cell takes place

34|Page
e Synthesis of embryonic antigen derepressed
Embryonic proteins produced
Accidental antigens appear

86. What are the of immune protective mechanisms


against neoplasia:

Mechanism of natural non-specific resistance -Nk cells,


macrophages
Mechanism of natural non-specific anti tumor
protection (tumor not exceed 10 power 3)
Reaction of specific antitumor immunity (T-killers to
eliminate mutant somatic cells -phagocytosis and
complement
Isolated malignant cell-mutant — eliminated by T-killers,
cytolysis & phagocytosis mechanism
since malignant cells — new antigen viral b -
lymphocytes can form antibodies, interferon

- 87. Which mechanisms of cancer cachexia do you know?

Retention of glucose, carbon, nitrogen, amino acid by a


tumor (all reserves depleted)
Loss of skeletal muscle (protein degradation)
Anorexia
decreased physical activity
decreased secretion of host anabolic hormones

35|Page
e and an altered host metabolic response with
abnormalities in protein, lipid, and carbohydrate
metabolism

88. What periods in pathogenesis of cancer do you


know?
e Neoplastic transformation
e Neoplastic promotions
e Neoplastic progression

89. A woman 45 years old has the state of excitation


with the feeling of starvation is the period of
starvation. Which manifestation is period of
starvation do you know?
Nervous excitement , sensation of hunger
All physiological system are activated
Stress for homeostasis regulation
e Motor activity promoted
ABP arises
Body weight decreases
3-4 days after hunger disappears

90. Peculiarities of absolute starvation?

e Duration —(2-3times) less compared to that of complete


starvation
36|Page
e Increase breakdown of fats -formation of oxidative
water-ketonemia, non-respiratory acidosis (large
quantity of water- fat oxidation-112g h20, protein and
carbohydrate oxidation provide half quantity)
e Intoxication-accumulation of large no of metabolic &
toxic products

91. Aman 55 years old has the state of excitation


with the feeling of starvation is the 1st

period of starvation. Which manifestation 2 period of


starvation do you know?
Maximal adaptation- period of economical utilisation of
substances and energy (rate of decreasing weight
loss disappears, feeling of hunger disappear)

92. A woman 35 years old has the state of excitation


with the feeling of starvation is the 1 period of
starvation. Which manifestation 3 period of
starvation do you know?
e Tissue decay
e intoxication, death (disorder of all vital functions-
arterial blood pressure decrease ,disorders of
respiration, body temperature reduces, heat production
collapses)

37|Page
93. Aman 48 years old with use fasting with
therapeutic aim. What diseases are treated by
medical fasting nowadays?
e Allergic,
Cardiovascular
Digestive
Dermal
diseases of the joints
obesity
and even mental ones.

94. Peculiarities of qualitative starvation of proteins?


Protein calorie deficiency
Alimentary dystrophy
Kwashiorkor — (depression syn of nucleoproteins,
proteins and decreased enzymal activity)
Atrophy in organs
Bone growth depressed
Avitaminosis
Anemia
Basal metabolism decreased

95. Peculiarities of qualitative incomplete starvation


of proteins?

38|Page
e Asa result of plastic material (protein) deficit synthetic
process suppressed
e Oncotic blood pressure decreased (content of protein in
blood decreased)
e Osmotic pressure in tissue rises (chloride accumulation)
e Water retention

96. Peculiarities of qualitative incomplete starvation


of vitamin PP?
e dermatitis
e damages of the mucous membranes of the digestive
tract with diarrhoea.

97. Peculiarities of qualitative incomplete starvation


of vitamin B2?
e (Impaired process of biological oxidation-damage of
highly aerobic mitotic tissues)
e Weakness, soreness, crackling, skin rash, dry/red
tongue (ariboflavinosis)
e Urinary excretion of riboflavin decreased

98.Peculiarities of qualitative incomplete starvation


of vitamin B6?
Peculiarities can be

e Exogenous (as a result of the absence or a low content


of vitamin in food) or

39|Page
Endogenous nervous system(neurological) problems
since Vit. B6 is important for it, so absence
(Avitaminosis) or insufficient(hypovitaminosis) amount
could cause neurological problems.
Endocrine system pathologies or diseases could be
observed if there's insufficiency(hypovitaminosis) or
Absence (Avitaminosis) of Vit. B6
seborrheic dermatitis
Cheilosis
Confusion and depression.
seizure, glossitis and peripheral neuropathy.

99.Peculiarities of qualitative incomplete starvation


of vitamin C?
Disorder of oxidative process, collagen

synthesis,

due to which vessels lose solidity and easily break


resulting in hemorrhage syndrome.
scurvy
edema
myalgia
hemolytic jaundice
weakness
anemia and Keratosis pilaris
Easy Bruising and petechia or small spots
Slowly Healing Wounds.

40|Page
100.Peculiarities of qualitative incomplete starvation
of vitamin D?
e Disorder of the absorption of calcium and phosphorus in
the small intestine and renal tubules with a disorder of
bone mineralization.
e Multiple sclerosis
e Rachitis(rickets),
e bone pain, broken bones and muscle pain.
e osteomalacia,
e osteoporosis,
e osteopenia and myopathy.

101.What hormones elevate the blood glucose level?

@ Epinephrine(adrenaline), cortisol, and growth


hormone(somatropin)
@ Glucagon, aspirin, ACTH and Thyroxine.
e@ glucocorticoids.

102. Describe changes in protein metabolism in


Diabetes 1 type?
e Increased permeability of cell membrane to amino acid.

41|Page
decreased catabolism of protein
increased synthesis of protein by influencing
transcription of DNA and increased translocations of
mRNA, impaired physical and mental development,
delayed healing of wounds
decreased antibody formation resulting in increased
sensitivity to infection and furunculosis.
Increase in circulating amino acid and ketones in liver,
the increased gluconeogenesis leads to hyperglycemia.
increased rate of proteolysis leads to elevated
concentration of plasma amino acids.
branched chain amino acid increase leucine oxidation
and protein synthesis and inhibits protein breakdown.
aminoacidemia, enhances the synthesis of urea leading
to azotemia and intoxication.
hyperglycemia and loss of weight

103. Describe peculiarities in Diabetes 1 type?


Autoimmune destruction of beta cells of pancreas
insulin dependent ketoacidosis
hyperosmolar coma
Hyperglycemia
polydipsia and frequent urination

42|Page
103.Describe peculiarities in Diabetes 2 type?

e Absolute or relative insulin deficiency.


Hyperglycemia
dyslipidemia.
Hyperlipacidemia
obesity
Low LDL levels and hypertension
endothelial dysfunction

104. Describe metabolic disorders in diabetes


ketoacidosis development:
e metabolic acidosis
e hyperglycemia-peripheral insulin resistance.
e hyperketonaemia(ketosis)
e non ketotic hyperosmolar coma

106. In what cases does relative insulin insufficiency is


occurred?
e Diabetes type 2
e polycystic ovary syndrome
e acromegaly
e Endothelial dysfunction
e Atherosclerotic CVD.
e Adiposity
Acanthosis
43|Page
Obesity
endothelial dysfunction and systemic inflammation

107. In what cases does absolute insulin insufficiency


is occurred?

Decreased Insulin biosynthesis


Impairment of glyco- receptor system which will cause
Decrease Insulin secretion in response to glucose
Disorder of Proinsulin and Transformation into Insulin
Disorder of insulin secretion from B-cells (improper
insulin secretion response)
Damage of B- cells by cytotropic chemicals or drugs
Damage of B-cells by B-cytotropic viruses eg. Coxsackie
virus
Autoimmune(immune) Damage i.e. formation of
antibodies against insulin or B- cells)
Destructive changes in the pancreas - (such as- Tumors,
cirrhotic and inflammatory processes, vessel sclerosis).
diabetes type 1
acute pancreatitis and hyperglycemia.

108. What causes can lead to hypoglycemia?


Medications [insulin and sulfonylureas]|
Adrenal or pituitary gland disorders, diabetes
pancreatic tumor
44|Page
kidney disorders which will lead to loss of glucose
renal glucosuria
severe infection and sepsis.
hereditary defect of enzymes and inhibition of
glycogenolysis.
sever debilitating physical work
disorders of gluconeogenesis
amylase deficiency
malignant tumors starvation {anorexia nervosa}
glycogenosis reaction.
hypothyroidism, intensified use of glucose
hyperinsulinism/insulin overdose. Inborn error of
metabolism.

109. In what cases does absolute insulin insufficiency


is occurred?
Decreased Insulin biosynthesis
Impairment of glyco-receptor system which will cause
Decrease Insulin secretion in response to glucose
Disorder of Proinsulin and Transformation into Insulin
Disorder of insulin secretion from B-cells(improper
insulin secretion response)
Damage of B- cells by cytotropic chemicals or drugs
Damage of B-cells by B-cytotropic viruses eg. Coxsackie
virus

45|Page
Autoimmune(immune) Damage i.e. formation of
antibodies against insulin or B- cells)
Destructive I’m changes in the pancreas - (such as-
Tumors, cirrhotic and inflammatory processes, vessels,
sclerosis).
diabetes type 1,
acute pancreatitis
hyperglycemia

110. Which peculiarities of Girke’s diseases do you


know?
short stature, delayed puberty, seizures secondary to
low glucose.
hepatomegaly leading to swollen belly and live adenoma
hypoglycemia — with lactic acidosis and constant hunger.
Neutropenia, hyperuricemia, hyperlipidemia
doll like or cherubic face and puffy cheeks, thin chest,
limbs and growth failure.
easy bruising and nosebleeds
anemia
oral ulcers.
xanthomas and renal enlargement
low muscle tone

111. What manifestation of hypoglycemic coma do


you know?

46|Page
Loss of consciousness,

loss of reflexes {brisk reflexes}to external and internal


stimuli.
brain damage
disorientation and confusion
neuroglycopenia
dementia
disorder of vital functions (circulation and respiration).
Coma
Agitation
Slurred speech
hunger
sweating
tachycardia.

112. What manifestation of hyperglycemic coma do


you know?
loss of consciousness, depression, tachycardia.
shortness of breath, stupor
dehydration due to increased urination.
glucosuria and Hallucinations
Warm, dry skin
sweet or fruity acetone odor on breath.
Ketosis

47|Page
113. What other pathological processes can be
accompanied by the hypo osmolar hyperhydration?

@ Edema (pulmonary and brain), water poisoning


cardiac failure, hypertension {intracranial}
acute renal failure
Anuria and polyuria
Hyperventilation
Hypoproteinemia.

114. What other pathological processes can be


accompanied by the hyperosmolar hyperhydration?
e Secondary aldosteronism
e Use of Salty Sea Water for Drinking

115. Myxedema is characterised by?

@ increase hydrophilicity in tissue colloids.


@ increase of vascular permeability.
e by thyroid gland hypofunction-hypothyroidism

48|Page
e By depressed mental status, progressive weakness,
cardiovascular vascular collapse
® coma
e death
e hypotension
e Hypothermia
e lactic acidosis
e Hypoglycemia.
e By thickening of the skin, blunting, of senses, and
intellect and labored speech associated with
hypothyroidism.
e By delayed sexual maturation.
e by firm inelastic edema, dry skin, loss of mental and
physical vigor.
e by decompensated hypothyroidism.
e by increased aldosterone
e by slowing of physical and mental activity
e by generalized edematous appearance demonstrating
baggies under the eye and swelling of the face.
e by low metabolic rate sluggishness sleepiness.

116. Lung edema was modelled in rat using solution


of ammonium chloride.

e This type of edema is characterised by accumulation


of fluid in the air spaces and interstitium of the lung.
e Airways obstruction or laryngotracheitis (not sure).
49|Page
117. Which clinical manifestations of hyponatriemia?
Stupor, primary neurologic
Spasm
Agitation
Depressed reflexes.
Confusion
Pulmonary edema
Respiratory failure
Cheyne-stroke respiration
Intracranial hypertension
Brainstem herniation
coma
death
lethargy
Cerebral edema
decreased body temperature
arterial hypotension
Loss of energy, drowsiness and fatigue.
Restlessness and irritability
anorexia
seizures
clouding of consciousness

50|Page
118. Reasons of hypernatremia are:

inadequate water intake


impaired thirst drive.
excessive sodium~>salt water ingestion or drowning
mineralocorticoid or glucocorticoid excess~> Cushing
syndrome
medication ~>colchicine, alcohol, lithium and
sulfonylureas.
Conn’s syndrome
Myeloma
Amyloidosis
Sjogren syndrome
hypercalcemia
diarrhoea, post-obstructive diuresis, diabetes insipidus,
cutaneous losses/excess sweating.
Intravenous hypertonic saline, ectopic ACTH production:
dementia.
renal failure
hyper aldosterone
diarrhoea
granulomatous diseases
osmotic diarrhoea.

119. Etiological factors of hyperkaliemia are:

intracellular proteolysis activation

51|Page
® excessive intake of potassium
® massive cell damage
@ increase in the intensity of catabolic processes
e acidosis.
@ violation of potassium excretion from the body
e {oliguria and Anura}
@ insufficiency of adrenal cortex, diabetes, cirrhosis of
liver, congestive heart failure, Gordon syndrome,
hemolytic anemia and pseudo hypoaldosteronism type 1
e kidney dysfunction {glomerulonephritis, lupus and
chronic renal failure}
e drugs such as ACE inhibitors
e NSAID
® potassium sparing diuretics.
@ Heparin
e dysfunction of the renin angiotensin.

120. ESG —features of hypokaliemia?


e Q-T interval prolongation
e ST depression
e PR prolongation
e visible U wave
e flattening and inversion ofT wave
e increased amplitude and width of P.

52|Page
121. Clinical manifestation of hypercalcemia:
e Hypertension
e renal failure
e hypotonia
® coma
e Chrondrocacinosis
e Short QT
e Osteopenia
e vitamin D toxicosis
e renal calculi
e peptic ulcer
e Bone pain
e altered mental status
e tachycardia
e Conjunctivitis
e Gout
e Vascular calcification
e CNS depression stupor
e Osteitis fibrosa cystica
e Arthritis
e Polyuria
e Nephrogenic diabetes insipidus
e Arrhythmia
e hyporeflexia
e Increased risk of bone fracture
e decreased neuromuscular excitability

53|Page
122. For hypocalciemia all symptoms are typical
except:
“The following are the symptoms”

e Confusion, memory loss, muscle spasm{tetany}


e tingling in the hands
e hallucination and fracturing of bones
e asphyxia
e death
e laryngospasm
e increased neuromuscular excitability
e skeletal muscle cramps.
e Rickets
e Stero distrophy
e Sterno cardia attacks

123. Which clinical symptoms don't correspond to


hyponatriemia?

The following are the clinical symptoms:}

e cerebral /peripheral edema, tachycardia, arterial


hypotension, polyuria, coma
e stupor
e absence of thirst

54|Page
Confusion
nausea, vomiting and vomiting, headache, short-term
memory loss, confusion
lethargy
fatigue
loss of appetite
irritability
muscle weakness
spasms or cramps
seizures
decreased consciousness or coma
decreased body temperature and low energy.

124. Etiological factors of hyperkaliemia don't include:


“The following are the etiological factors “

diabetes type 1
Adrenal insufficiency [Addison disease]
beta blockers
increased of potassium supplements
dehydration
use of angiotensin converting enzyme inhibitors
heart failure
kidney disease
liver cirrhosis

55|Page
e rhabdomyolysis and hypoaldosteronism.

125. Clinical manifestation of hypercalciemia don't


include some of next:
e@ Excessive thirst and frequent urination.

eStomach pain and digestive problems.


@Bone pain and muscle weakness.
eConfusion, lethargy, and fatigue.

e@Anxiety and depression.

@High blood pressure and abnormal heart rhythms.

e Muscle ache and twitches (tone)

126. Metabolic acidosis develops with when the body


produces too much acid.
e When the kidneys are not removing enough acid from
the body loss of bicarbonate.

127. Severe hypercapnia with gas acidosis leads to:


e Respiratory acidosis

56|Page
128. What are the mechanisms of compensation for
metabolic alkalosis:
e Hypoventilation
e metabolic alkalosis is hypoventilation leading to a pH
correction towards normal.
e Increases in arterial blood pH depress respiratory
centres. The resulting alveolar hypoventilation tends to
elevate PaCO2 and restore arterial pH toward normal.

129. List the pathological processes leading to the


development of excretory acidosis:
e Diarrhoea
e Renal dysfunction
e Adrenal gland disease

130. The causes of metabolic acidosis are:

e Complete starvation
e Shock
e Heart block
e respiratory failure
e DM
e Accumulation of acid metabolites
57|Page
e pH

e CO? HCO? are all reduced

131. The causes of respiratory alkalosis are:


e Hyperventilation
e CNS damage

132. What conditions lead to the development of


excretory acidosis:
e Kidney or lung insufficiency

133. What reasons can lead to the non-gas excretory


acidosis?

134. What reasons can lead to the metabolic acidosis?

e Complete starvation
e Shock
e heart block, respiratory failure
e DM

58|Page
e Accumulation of acid metabolites
e pH co2 HCO3 are all reduced

135. What characteristic parameters of acid-base


imbalance is observed in sailors who continue to work
during submarine catastrophe?
e Increased co2
e Decreased pH
e Respiratory acidosis

136. A rise in the blood pH occurring due to lowered


Pco2 consequent to hyperventilation of the lungs
(excess

removal of CO?) is called respiratory alkalosis. This


occurs in the following conditions.
e Hyperventilation
e CNS damage

59|Page
137. A fall in the blood pH occurring due to raised
Pco2 consequent to hypoventilation of lungs (CO?
retention) causes respiratory acidosis. This can occur
in the following circumstances.

e Chronic obstructive disease of lungs


e respiratory insufficiency
e cardiopulmonary insufficiency
e drug abuse
e submarine accident
e asphyxia
e cns trauma or damage
e accident in closed space
e trauma of breast

138. What reasons can lead to the respiratory


excretory alkalosis?
e hyperadrenocorticism (Cushing syndrome)
e aldosteronism
e wasting HCL under vomiting

60|Page
139. What reasons can lead to the non-gas excretory
acidosis?
e hyperventilation increased co2 excretion

140. What forms of buffer systems do you know?


e Protein buffer system
e Phosphate buffer system
e Carbonic acid bicarbonate buffer system
e Hemoglobin buffer system

141. A patient has chronic posthemorrhagic anemia


which is accompanied with reduce of concentration of
serum iron. What changes are the most likely in this
case?
lron deficiency anemia
Low hemoglobin count
Tissue starvation of oxygen
Erythrocytes can be decreased in size and amount
Hypo regenerative anemia

61|Page
Chronic Posthemorrhagic Anemia

Chronic posthemorrhagic anemia develops due to repeated blood loss caused


by a damage of the small blood vessels in some diseases (stomach ulcer, uterine
problems, hemorrhoids, etc.) as well as in hemostasis disorders (hemorrhage syn-
drome, p. 325).
Repeated blood loss is accompanied with iron deficiency in the organism. In
case of hemopoiesis inhibition, such anemia may become hypo- and aregenerative
with a low content of regenerative erythrocyte forms in the peripheral blood (/Ay-
poregenerative anemia).

Blood Picture

The quantity of erythrocytes and hemoglobin is decreased as well as the color


index (Aypochromic anemia).
Investigation of the blood smear shows the following changes:
* Appearance of regenerative forms of erythrocytes (reticulocytes), which is a de-
fense reaction, but regenerative forms are not as numerous as in case of acute
posthemorrhagic anemia. Single normoblasts may appear.
- Appearance of degenerative forms of erythrocytes — hypochromic erythrocytes,
microcytes with anisocytosis and poikilocytosis.
+ The ratio of regenerative and degenerative cells content moves in favor of degene-
rative ones. The reason lies in the fact that the bone marrow is also affected by
chronic hypoxia.

142. A patient was brought with massive blood loss


due to of road-transport trauma. Pulse — 110/min,
breathing rate — 11/min, arterial pressure is 90/50
mm Hg. What change of the blood will be most
characterized through 0,5 h after blood loss?

e Decrease of erythrocyte and hemoglobin content


e Hypovolemia (low circulating blood volume)
e Non-respiratory metabolic acidosis (acid base
imbalance)
e Thrombosis blood coagulation
e Redistribution of blood

62|Page
e Restoration of circulatory volume and erythrocyte
content due to blood reserve

143. What conditions lead to polycythemic


hypovolemia?
e Dehydration of organism
e Diarrhea
e Vomiting
e Hyperventilation
e Increased perspiration

144. What conditions lead to polycythemic


hypervolemia?
Hypobaria
Reduction of atmospheric pressure
Heart failure
Emphysema
Oxygen deficiency

145. What conditions lead to oligocythemic


hypovolemia?
e Acute blood loss

63|Page
146. What conditions lead to oligocythemic
hypervolemia?
e Water retention
e renal disorders
e rejected blood substitutes

147. What are the causes of oligocythemic


normovolemia?
e Anemias of different types

148. What are the factors of the negative outcome of


the bleeding?
e Size of damaged vessel
Volume of blood loss
Rate of blood escape
Localization of hemorrhage
Ability of organism to clot (coagulative system, quantity)

149. The reflex phase of compensation after acute


hemorrhage is characterized by:
e Spasm of vessels
e Decreased diuresis and thirst
e Reflex acceleration and intensification of cardiac
contractions

64|Page
e Reflex ventilation increase due to acceleration and
deepening to compensate for oxygen deficiency of
tissues
e Increased hemopoiesis
e Increased protein synthesis in liver
e Increased erythropoietin in kidney and gastric internal
castle factor
e Expense of blood reserve

150. What are the mechanisms of compensation


during reflex phase after hemorrhage?
e Increased hemopoiesis
e Increased protein synthesis in liver
e Increased erythropoietin in kidney and gastric internal
castle factor
e Expense of blood reserve
e Spasm of vessels
e Decreased diuresis and thirst
e Reflex acceleration and intensification of cardiac
contractions
e Reflex ventilation increase due to acceleration and
deepening to compensate for oxygen deficiency of
tissues.

65|Page
151. What are the mechanisms of blood volume
restoration after hemorrhage?
e Increased hemopoiesis
e Increased protein synthesis in liver
e Increased erythropoietin in kidney and gastric internal
castle factor
e Expense of blood reserve

152. What are the mechanisms of blood pressure


normalization in reflex phase after hemorrhage?
e Reflex acceleration and intensification of cardiac
contractions
e Tachycardia
e Increase renin and angiotensin 2
e Increased aldosterone
e Vasoconstriction
e Stimulation of volume receptors of aortic arch and
carotid artery

153.What are the reasons of normalization of blood


pressure at hydrous phase after haemorrhage?
e activation of renin-aldosterone-angiotensin system
e increased output of vasopressin
e redistribution of water from intersticum to the vessels

66|Page
154. Which reasons of Erythrocytosis do you know?
e chronic common hypoxia
e ischemia of the kidneys

155. A patient complains of dyspnea for the rapid


walking. A skin is pale, cold. Results of analysis:
erythrocytes — 3,2¢1012/I, hemoglobin — 90 g/I,
colour index — 0,6, contents of proteins — 72 g/I. In the
smear of blood there are much anulocytes and
microcytes, single oxyphilic normocytes. What may
leads to such disorders?

e lIron deficiency anemia


e Erythrocyte count is less, hemoglobin count is less, color
index is less(hypochromic).

156. Severe intravascular hemolysis is manifested by.


e methemalbuminemia
e hemoglobinuria

67|Page
e jaundice
e Patients may have clinical symptoms of increased
intravascular hemolysis, such as pallor, icterus, or dark
urine.
e Severe intravascular hemolysis leading to release of
hemoglobin in amounts sufficient to exceed the
absorptive capacity of circulating haptoglobin and the
normal renal metabolic reserve can lead to
e hemoglobinuria and acute renal injury.

157. Beta-thalassemia is characterised by:


e familial occurrence
e increased destruction of maturing erythroblasts in the
bone marrow
e anemia development
e reduction in the synthesis of beta-globin chains
e Common symptoms include pallor, jaundice, leg ulcers,
gallstones (cholelithiasis), and abnormal enlargement of
the liver and spleen.
e Moderate to severe skeletal malformations (as
described in beta thalassemia major) may also occur.
e The skin may show pallor from anemia and jaundice
from hyperbilirubinemia, and the skull and other bones
may be deformed secondary to erythroid hyperplasia
with intramedullary expansion and cortical bone

68|Page
thinning. Skin ulceration may be present on the
extremities.

158. Sickle cell anemia is characterised by:


it is accompanied by splenomegaly
it results from abnormal hemoglobin synthesis
Autosomal Recessive
RBC take shapes an sickle resulting anemia
They Begin in early childhood.
Anemia
RBC appear as crescent /sickle cell shape
Rig, sticky thus get stuck in small blood vessels & block
blood flow
Signs and symptoms appear at around at 5 months of age
Symptoms

Anemia
Episodes of Pain
Swelling of hands & feet
frequent infection
Delayed growth / Puberty
Vision problems
Complications
Stroke
Acute chest syndrome
Pulmonary Hypertension

69|Page
Organ damage
Blindness
Leg ulcer
Gall stone

159. What factors may cause iron deficiency anemia?


e depletion of intrinsic iron stores
e impairment of folic acid activation
e adecreased production of hydrochloric acid by gastric
mucosa
e an increased iron demands
|
The causes are the following:
eated or massive
‘ 4 ioss in bleeding is the most often cause. It may be a repdise ases,
single blood loss in the uterus, gastrointestinal, renal, pulmonary
occurs In:
+ Impaired entering of iron into the organism
* starvation;
+ iron deficit in food (if children are fed with cow or goat milk only),
Tog Impaired iron absorption in digestive tract pathology (hypoacid gastritis,
chronic enteritis) or resection.
+ Impaired iron transport (hypotransferrinemia in liver disturbances, genetic
atransferrinemia).
¢ Disorders of iron utilization from reserves (in infection, intoxication, helmin-
thic invasion).
* Disorders of iron deposition (in hepatitis, hepatic cirrhosis),
* Increased needs of the organism for iron and its increased consumptio
n dur-
ing:
* pregnancy;
* growth;
* lactation,

70|Page
160. Which changes in blood are characterized Iron
deficiency anemia?
colour index < 0.7
reticulocytes 1 %;
anisocytosis with a predominance of microcytes
Poikilocytosis

161. Which changes in blood are characterized


Addison -Birmer anemia?
neutropenia varying degrees
hyper chromia of erythrocytes
leukopenia, thrombocytopenia
atrophic glossitis
symptoms of funicular myelosis
It is dyserythropoietic anemia, characterized by shift of
Bence-Jones curve to the Right, Increased color index
Addison-Birmer
anemia/megaloblastic/pernicious/Vitamin B12
Hyperchromic

162. A patient has stomach resection. Later vit B12


deficiency anemia develops. What manifestations are
characterized this pathology?
increase in serum iron

71|Page
megaloblastic type of hemopoiesis
macrocytosis
presence of RBCs with Jolly bodies and Cabot rings
anisocytosis with a predominance of macrocytes
decrease of synthesis of nucleic acids

Blood picture is characterize by the following quantitative signs:


- a significant decrease of the amount of erythrocytes (2—3 T/I and less);
a decrease of hemoglobin content (70—80 g/l and less);
an increase of the color index to exceed | (1.2—1.4 and more);
leukopenia (decreased amount of leukocytes);
thrombocytopenia (decreased amount of thrombocytes).
The following morphological disorders of the blood cells (signs of degenera-
tion) are detected in a blood smear:
Erythrocytes are significantly enlarged (megalocytes with the diameter 10—
12 np and more, 20 pu has been recorded in some patients).
Erythrocytes (megalocytes) are hyperchromic, without a zone of central pal-
lor due to their large volume and changed form.
Poikilocytosis, anisocytosis. ’ ; '
Pathologic inclusions (nucleus remainders) in megalocytes — Jolly’s bodies.
Cabot’s rings and basophilic granules.
A large quantity of nuclear forms of erythrocytes (megaloblasts).
'
Physiologic regeneration cells (reticulocytes) are absent.
Leukocytes have a large size and a hypersegmented nucleus (right-side nu-
clear shift in leukoformula is described on p. 301).
So. vitamin B.. and folate deficiency anemia is megaloblastic and hyperchro-
) 12
mic,

163 . In an infant, who is under an artificial nutrition


with cow milk, severe anemia has developed. At the
blood count of the infant: number of erythrocytes is
4x1 012/L, content of hemoglobin is 68 g/L,
reti culocytes of 0%.

Wh at kind of anemia developed in the infant?


72|Page
e lron deficiency anemia

164. Megaloblastic anemia was characterised by.


e folic acid deficiency anemia
e intrinsic factor deficiency
e vitamin B12-deficiency
e Anemia
e increase in serum iron
e macrocytosis
e presence of RBCs with Jolly bodies and Cabot rings
e anisocytosis with a predominance of macrocytes

165. Patient S 36 years old has aplastic anemia. What


changes in blood will be?
e peripheral cytopenias
e deficiency of all blood cell types: red blood cells, white
blood cells
e platelets.

166. A patient with lympholeucosis complains of the


frequent bleeding from nose. Was diagnosed
Insufficiency of vascular-thrombocytic hemostasis
What change of the blood will be most characterized
this pathology?
73|Page
eThrombocytopenia

eSmudge cell

eAnemia

167. Results of blood tests of the patient:


erythrocytes — 1,2¢1012/I, concentration of
hemoglobin — 48 g/I, colour index — 1,2. Reticulocytes
is absent, anisocytosis, poikilocytosis, single
megalocytes, it was diagnosed: 12-(folate) deficiency
anemia. Name the type of anemia according to
regeneratory possibilities, and what change of the
blood will be most characterized this pathology?

eHypo regenerative anemia

74|Page
168. A man appealed to the doctor with complaints of
appearance of hypodermic hematomas. In the blood
test: erythrocytes — 2,5¢1012/I; hemoglobin — 80,0
g/|; colour index — 0,9. Leucocytes — 5,8¢109/I.
Leucocytic formula: myeloblasts — 6 %, myelocytes —
10 %, metamyelocytes — 18 %, stab neutrophils — 26
%, segment nucleonic neutrophils — 10 %,
lymphocytes — 10%, monocytes — 2 %. Was diagnostic
Chronic myelocytic leucosis What change of the blood
will characterized this pathology?

ein chronic myelocytic leucosis the tumor cells are


represented by the derivatives of leukemic

myeloblasts (promyelocytes, myelocytes,


metamyelocytes, stab and segmented cells)
eHyperregenarative shift of leucocyte formula to the
left

eHyperleukocytosis (increase in the content of WBC


above 20x10‘9)
¢Basophilic eosinophilic association (increased % of
eosinophils and basophils in the blood)

75|Page
169. A 20 years-old patient has transfusion reactions.
Its cytotoxic type of allergy. Check pathology of
Cytotoxic components of Cytotoxic antibodies to
blood cells e.g., TCRs have two parts, usually an alpha
and a beta chain. (Some TCRs have a gamma and a
delta chain.)
Hematopoietic stem cells in the bone marrow migrate into
the thymus, where they undergo V(D)J recombination

of their beta-chain TCR DNA to form a developmental form of


the TCR protein, known as pre-TCR.

170. Mechanisms and reasons of leukopenia.


e suppression of leucopoiesis
e increased leukocyte destruction in the vascular bed
e increased margination of leukocytes in the vascular bed

171. During medical examination of boy 4 years old


with weight 18 kg the considerable increase of
number of eosinophils in unit of blood volume was
revealed. What can be the reason of eosinophilia?
e Acquired -physical (ionizing radiation)
Chemical (benzene, cytostatics, drugs)

76|Page
Biological (viruses, infectious hepatitis, hepatitis)

Immune factors

e Hereditary- kostmann syndrome, hereditary


neutropenia, lazy leucocytes.

172. Leukopenia may be a result of.


insufficiency of vitamin B12
overwhelming bacterial, fungal or rickettsial infections
antimetabolites used in cancer treatment

Chronic myeloleukemia

Allergies

Hodgkin’s lymphoma

Ulcerative colitis

173. In a child 13 years old leukocytosis developed


after the protracted training (12,0¢109/1).
Development redistributive leucocytosis. Check all
possible mechanisms of redistributive leukocytosis.

77|Page
e The redistributive leukocytosis - redistribution of white
blood cells in the bloodstream.
e Characterized by the accumulation of a large number of
mature leukocytes into any body region, the absence of
signs of hyperplasia leukopoietic tissue preservation
total white blood cell count within the normal range.

- Can be observed after considerable physical exertion


("myogenic leukocytosis"), traumatic, blood
transfusion, anaphylactic shock (increased number of white
blood cells in the blood micro vessels of lung, liver, intestine
wall).

¢ The redistributive leukocytosis is temporary and not


accompanied increase in the number of young forms of
leukocytes. That is why leukocytosis with the
development of a mechanism called the false or relative.

174. The nuclear regenerative shift to the left is.


Shift that occurs after 6-8 days after acute blood loss of
moderate severity:

e It occurs in inflammation and activation of


immunological reactivity.

78|Page
175. The nuclear shift to the write is:

e Increase in hyper segmentation of neutrophils nuclei


e cytoplasmic vacuolation
e Predominance of mature neutrophils with a large
number of segments (5-6), disappearance of younger
cells, this change is of adaptive nature. It occurs in all
type of stress (and in the first stage of an acute radiant
disease).

176. What changes occur in peripheral blood at


chronic lymphoid leukemia?
e small count of lymphoblasts
e the presence of Botkin-Gumbrecht bodies
e Lymphocytes are mostly mature (60-70 percent). The
majority of the lymphoblasts is matured
e upto the end (B-lymphocytic kind of leukemia is more
common). In the leukocyte formula the quota of mature
lymphocytes reaches 80-90% (relative and absolute
lymphocytosis with relative neutropenia).

177. Which cells are increase in peripheral blood at


chronic myeloid leukemia?

79|Page
e Myeloblasts
e mature neutrophiles (segmented)
e promyelocyte
e neutrophilic myelocyte
e neutrophilic metamyelocyte
e baton-like stab neutrophile granulocyte
e segmentonuclear neutrophile granulocyte
e eosinophile myelocyte
eosinophile granulocyte
basophile granulocyte
lymphocyte

178. Chronic myeloid leukemia is associated with.


® asmall amount of myeloblasts
e increased amount of basophiles and eosinophiles
e absent hiatus leukemicus

179. Violation thrombocytovascular hemostasis.

and

180. Lack of coagulative hemostasis


1. Hemophilia a,b,c

2. Insufficiency of vitamin k

80|Page
181: Causes and mechanisms for violations of certain
stages of blood clotting
e ist phase VII deficiency, IX deficiency, XIII deficiency
platelet factor 3
e 2 Nd phase deficiency of factor Il, factor V
e 3rd. phase afibrinogenemia, hypofibrinogenemia
First phase:

1. Isolated disturbances of blood clotting activation by


external mehanism - f.vii deficiency(hypoproconvertinemia)

2. Internal mechanism- f.viii deficiency(hemophilia A), f ix


deficiency(hemophilia b), f xi deficiency (hemophilia c ), f. xii
deficiency, platelet factor 3 deficiency

3. Combined disorders - hereditary(autosomal recessive


type), acquired (hypovitaminosis, liver damage).

Second phase:
1. Deficiency f.ii- hypoprothrombinemia

2. Deficiency of f.v- parahemophilia.

third phase:

1. Deficiency of fibrinogen- afibronogemia,


hypofibrinogenemia

2. Dysfibrinogenemia

81|Page
3. Disturbances in polymerization of fibrin

4. Deficiency of factor xiii

182: Thrombotic resistance of the vascular wall due


to:

Production of prostaglandin, destruction of the agent which


stimulate thrombocyte aggregation such as ADP

Production of mucopolysaccharide which resembles heparin


plasminogen activation

1. Activation of platelet adhesion contribute to:

Slowing of blood flow

The reversal of electrical charge in the intima from — to +

Contact of platelet processes with collagen

2. Immune thrombocytopenic purpura is characterized by:


purple rash red or purple dots called petiache reduction
in circulating thrombocyte because of autoimmune
aggression
3. Hemophilia A is characterized by:
Deficiency of factor 8 Is x linked

Usually affects males Have prolonged bleeding


4, Primary anticoagulants:

82|Page
Heparin

Antithrombin 3

Alpha 1 antitrypsin, alpha 2 macroglobulin

Thrombocytopenia due to a violation of platelet production


in the bone marrow is observed:

e Anemia
e Leukopenia
e Damage to stem cells
e Abnormalities of normal cells

183. Activation of platelet adhesion contribute to:


e Slowing of blood flow
e Reversal of electric charge in the intima from
negative to positive
e Contact of platelet processes with collagen

184. Immune thrombocytopenic purpura is


characterized by:
e Purple rash, red or purple dot called petiache
e Reduction in circulating thrombocytes because of
autoimmune aggression

83|Page
185. Hemophilia A is characterized by:

Deficiency of factor 8
X linked recessive
Usually affect males
Have prolonged bleeding time

. Primary anticoagulant:
Heparin
Antithrombin 3
Alpha 1 anti-trypsin
Alpha 2 macroglobulin

187. Thrombocytopenia due to a violation of platelet


production in the bone marrow is observed:
Anemia
Leukopenia
Damage to stem cell
Stage of emergency hyper function
Stage of complete hypertrophy and relatively stable
hyper function
Stage of decompensation

84|Page
188.Hemorrhagic syndrome in violation of vascular-
platelet hemostasis may be due to:
e Lack of plasma procoagulants
e Surplus of anticoagulants
e Activation of fibrinolysis
e Deficit of platelets

189.The condition of coagulation hemostasis can be


estimated:
e Deficit of platelets
Reduced clotting factors
Spasm of blood vessels
Impaired fibrin clot formation
Increased anticoagulants

190.The state of vascular platelet

hemostasis can be assessed:

e Bleeding time

85|Page
191: Disseminated intravascular coagulation
syndrome of blood pathogenesis:
(Page: 332)

e imbalance between the coagulation, anticoagulation,


and fibrinolytic system and their stimulation with
further exhaustion of all of them.
e it occurs when there is a large quantity of procoagulants
and blood clotting activators enters the blood in
massive trauma which lead to micro circulatory clots
e hypo coagulation
e thrombocytopenia
e haemorrhage due to coagulation factor lack and
increased functional activity of anticoagulant system
and fibrinolysis.

2 stages:

* Stage of hyper coagulation

* Stage of hypo coagulation

192: Disseminated intravascular coagulation


syndrome of blood etiology:

86|Page
(Page 332)

never inherited but only acquired


massive trauma or tissue, crush syndrome, operative
trauma of parenchymatous organs.
massive burn
obstetrical pathology, premature placenta removal,
manual placenta removal, amniotic fluid embolism
acute intravascular erythrocytes hemolysis
leukemia
true erythrocytosis
uraemia
generalised infections, sepsis
all kinds of shock, terminal states
acute pancreanecrosis
peritonitis
side effects of the medicines, which influence
hemostasis

193: Primary outcome of myocardial injury:


Cardiac death
Heart failure
Thromboembolism
Atrial fibrillation, stroke, compared with those without
obstructive sleep apnea.

87|Page
188: Hemorrhagic syndrome in violation of vascular-
platelet hemostasis may be due to:
e Lack of plasma procoagulants
Surplus of anticoagulants
Activation of fibrinolysis
Deficit of platelets
Pathology

189: The condition of coagulation hemostasis can be


estimated:

Deficit of platelets
Reduced clotting factors
Spasm of blood vessels
Impaired fibrin clot formation
Increased anticoagulants

190: The state of vascular platelet hemostasis can be


assessed:
e bleeding time
88|Page
Mechanisms of immediate adaptation to cardiac overload
(pg303 new book) Heterometric chrono ionotropic inotropic
homotropic

e Mechanisms of long-term adaptation to cardiac


overload (pg343) heart enlargement
stage of emergency hyperfunction stage of complete
hypertrophy and relatively stable hyperfunction stage of
decompensation.

e Decompensation of heart insufficiency is characterised


by(pg344) Impaired myocardial trophicity Hypoxia
Cardiosclerosis Accumulation of Ca2*

191: Disseminated intravascular coagulation


syndrome of blood pathogenesis.
e Stage of hyper coagulation
e Stage of hypo coagulation

192: Disseminated intravascular coagulation


syndrome of blood etiology
e Massive burn
e Obstetrical pathology, premature placental removal
manual placenta removal
e Acute intravascular erythrocyte hemolysis
e Leukemia
e True erythrocytosis
e Uremia

89|Page
Generalized infections sepsis
Acute pancreas necrosis
Peritonitis
Side effects of the medicine

193: Study findings showed that at 30 days after


surgery, patients with obstructive sleep apnea had a

49% higher risk of.

194: Mechanisms of long-term adaptation to cardiac


overload (pg. 343)
e heart enlargement
e stage of emergency hyperfunction
e stage of complete hypertrophy and relatively stable
hyperfunction stage of decompensation

195: Mechanisms of immediate adaptation to cardiac


overload.

e Heterometric
e Chrono ionotropic homotropic

90|Page
196: Decompensation of heart insufficiency is
characterised by.

(pg344)
e Impaired myocardial trophicity
e Hypoxia
e Cardiosclerosis
e Accumulation of Ca2+

197: Heart insufficiency because myocardium damage


can arise at?

(pg346)
e Stenosis
e Occlusion of vessel
e Physical exertion or emotional stress

198: Manifestations of 4-th stages of atherosclerosis


are?

Sclerotization ( calcification)
Narrowing of lumen
Predisposition to thrombogenesis
ischemia ( infarction)

91|Page
199: Nomotopic violations of heart automatism
include such arrhythmia.

(pg. 348)
Bradycardia
Tachycardia
Respiratory

200: Which are the main risk factors of myocardium


infarction development?
High blood
Pressure
smoking, diabetes
lack of exercise
obesity
high blood cholesterol

201: The out of heart mechanisms of cardiac


insufficiency compensation include?
Caval and pulmonary veins increase pressure on inflow
pathways.

92|Page
e Aorta and carotid sinus decrease pressure on the
outflow pathways and subsequent decrease in the reflux
depressor effect Pericardium irritated.

202: What are the components of so-called “lipid


triad”, which is the one group of mechanisms of
myocardium necrosis?
e Ldl
e Hdl
e Triacyl glycerides

203: What arrhythmias do arise as a result of


conduction violation?
e Idioventricular rhythm

Due to blocks:

e intra atrial
e Atrioventricular
e Intraventricular

204: What clinical triad does characterise Dressler’s


syndrome, which arises after carried myocardium
93|Page
infarction?

e Pericarditis
e Pleurisy
e Pneumonia

205: Which manifestations of Left side cardiac


insufficiency do you know?
(Pg. 353)

e Blood congestion in veins of pulmonary circulation lung


edema
e Cardiac asthma syndrome

206: Which manifestations of Right-side cardiac


insufficiency do you know?
(Pg. 354)
e Veins of systemic liver enlargement edema on legs
ascites anasarca

207: Carcinogenic shock. Pathogenesis of its individual


species.

94|Page
(Pg. 334 new book)
e Reflex form: prolonged pain activation of sympatho
adrenal system
e Hypokinetic form: sharp increase in cardiac function
e Dyskinetic form: mismatch of cardiac contractions
e Arrhythmic forms: it is a consequence of severe
arrhythmias.

208: Hypertrophied heart of sick man is difference to


hypertrophied heart of sportsman by...
e Sportsman has Balanced hypertrophy in which all
components of the heart are uniformly increased.
e Functional reserves of the heart are significantly
increased.

209. Secondary dislipoproteinemia, as a reason of


atherosclerosis, is common.
e Violation of the relationship between individual classes
of plasma lipoproteins
e The appearance of qualitatively altered micelles.

95|Page
210. Which are the basic indexes of the first stage of
myocardium hypertrophy?
» Indice ISF (intensity of structural function) is used for
characteristics of a dynamic of myocardial hypertrophy
development. ISF=W/m (W is a work, m — mass of
myocardium) means a load per unit of myocardium and must

be constant for prolong and steady heart work

» Stage of Emergency (urgent) Hyperfunction. This stage is


characterized by a compensatory increase in the force of
the heart contractions according to the homeometric or
heterometric type in dependence on the type of the
load.
> This stage develops immediately after the beginning of
overload, however, it is short-term and energetically
uneconomical. The load on the unit of the muscular
mass and intensity of structure functioning (ISF) get
increased (ISF= increase W/m).

A combination of the pathologic changes in the myocardium


and mobilization of its reserves characterize this stage. The
pathologic changes include disappearance of glycogen,
reduction of creatine phosphate level, decrease of the
intracellular potassium and increase of sodium content,

mobilization of glycolysis, accumulation of lactate. The


disturbances of metabolism, which occur, start hypertrophy.
The heart becomes enlarged due to the intensified protein
synthesis and thickening of the muscular fibers.

96|Page
211. What functional indexes do characterize the
second stage of myocardium hypertrophy?
e The Stage of Completed Hypertrophy and Relatively
Stable Hyperfunction. In this stage, a hyperfunction of
myocardium obtains a plastic support. All elements of
heart undergo hypertrophy - myofibrils, capillaries and
nerve ends.
e Load per unit of myocardium mass (ISF) gets normalized
(ISF=increase W/ increase m). At this stage the

process of hypertrophy gets completed; the myocardium


mass may be increased by 100-120%.

e The uptake of oxygen, energy formation and the content


of the macroergic compounds are within the norm. The
hemodynamic indices are normal.
e The hypertrophic heart has adjusted to new load and
compensates it for a long time. Nevertheless, it is not
endless. The hypertrophied myocardium gets lost its
harmonial structural proportion. An increase in the mass
of myofibrils, capillaries, nuclear material and nerve
endings develops disproportionally. The myocardium
gets lost its reserves and a decompensation is inevitable.

97|Page
212. What indexes do characterize the emergency
phase of heart hypertrophy at development of heart
insufficiency?
e Stage of Emergency (urgent) Hyperfunction. This stage is
characterized by a compensatory increase in the force of
the heart contractions according to the homeometric or
heterometric type in dependence on the type of the
load. This stage develops immediately after the
beginning of overload; however, it is short-term and
energetically uneconomical. The load on the unit of the
muscular mass and intensity of structure functioning
(ISF) get increased (ISF= increase W/m).

e Combination of the pathologic changes in the


myocardium and mobilization of its reserves

characterize this stage. The pathologic changes include


disappearance of glycogen, reduction of creatin phosphate
level, decrease of the intracellular potassium and increase of
sodium content, mobilization of glycolysis, accumulation of
lactate. The disturbances of metabolism, which occur, start
hypertrophy.

e The heart becomes enlarged due to the intensified


protein synthesis and thickening of the muscular fibers.

98|Page
213. Among the mechanisms of cardiomyocytes
necrosis in pathogenesis myocardium infarction the
main ones are?
The following pathogenic variants of myocardial infarction
development are possible:

e Occlusion of the vessel promoting absolute decrease of


the coronary blood flow below the critical level (usually
three fourths of initial lumen)
e Stenosis, which is not manifested at rest but on little
exertion, physical or mental, resulting in ischemia of
the cardiac muscle.
e Considerable physical exertion or emotional stress,
which may cause inadequacy between the need in
oxygen and possibility of the blood inflow. In this case,
the intensified secretion of catecholamines and
hormones of the adrenal cortex plays an important role.
e In ischemia, the deficit of oxygen, substrates and ATP in
the myocardium takes place.

214. Atherosclerosis (according to HWO’s


determination) is the result of accumulation in
arteries of?
e Lipids
e Fibrin
e Calcium accompanied by elasticity impairment
e Vessel lumen narrowing

99|Page
215. The antiatherogenic role of high-density
lipoproteins is conditioned by their possibility.
e a-lipoproteins (lipoproteins of high density LPHD,
containing more quota of protein and few lipids) weakly

infiltrate the vascular wall and possess the antiatherogenous


influence (they are typical for the women).
This fact is an explanation of sexual differences in
predisposition to atherosclerosis (men are more predisposed

then woman).

216. Arterial hypertensions. Etiology and


pathogenesis and specific manifestations.
e Etiology
e Over tension of the higher nervous activity
e Negative emotions, “a disease of nonreacted emotions."
e Age
e Stress
e Table-salt abuse
e Nervous overstrain
e Hypodynamia
e Smoking, alcohol abuse, elderly age, hormonal
(climacteric, hyperthyroidism) disturbances,

100
| Page
atherosclerosis, the systemic disturbances of
metabolism, and the excessive use of common salt.

Genetic factors (predisposition)


Pathogenesis

-neurogenic hypertension
-salt (Na+, K+, ca2+)-dependent hypertension

-renal hypertension

- secondary (symptomatic) hypertension


-secondary neurogenic hypertension
-secondary renal hypertension
-secondary hormonal hypertension

Manifestation

e Increase in the arterial blood pressure above 140/90 mm


Hg
¢ Increase of a load on the heart (load by resistance)

¢ Myocardial hypertrophy
e Heart insufficiency

e Disorder of blood circulation in the kidneys

¢ Disorder of blood circulation in retina


e Edemas
e Hemorrhages

101|Page
¢ Disorder of trophicity of the tissues
e Risk factor of the atherosclerosis development

e Risk factor of myocardial infraction

e Risk factor of hemorrhagic stroke

217. What are causes of asphyxia?


e develops in a case when respiratory failure occurs
acutely or sub acutely and reaches such a degree when
oxygen stops to come into the blood and carbon dioxide
is not discharged out of the blood. The most frequently
asphyxia occurs in compression of the respiratory tract.
e Content of CO2 in the blood increases. The arterial
blood pressure increases. Experiments with inspiration
of gas mixtures containing 10-20% of CO2 showed that
respiratory intensification is connected with

e Reflex effect via hemoreceptors to H+ -ions on the vascular


motor center,
e Intensified secretion of adrenaline into the blood,

e Increased minute blood volume resulting from elevated


vein tension and increased blood inflow into the heart.

e Then, increase of CO2 concentration in the blood, its


narcotic effect begins to manifest, pH of the blood is
reduced to 6.8-6.5. Hypoxia Increases and so does
hypoxia of the brain. In its turn, it leads to inhibition of

102
| Page
respiration and decrease of arterial blood pressure.
Finally, respiratory paralysis and cardiac arrest occurs.

218. What kind of impaired content of gases in the


alveolar air and blood occurs in ventilatory
respiratory insufficiency?
e If ventilation gets slower then blood stream, pCO2 in
alveolar air gets increased and pO2 in the blood gets
decreased with hypoxemia and hypercapnia.

AF) Nenti\ ataepy Wc Ve vprraeny INvutFidency \

OO equlukve

@ Shy Muckive

Q) ~eytritive
\ blue brok ~ Daye N.3 92)

219. What kind of impaired content of gases in the alveolar


air and blood occurs in parenchymal diffusive

respiratory insufficiency?

e Decrease of pO2 in arterial blood pCO2 in arterial blood


doesn’t change or decreases when ventilation prevails

103 |Page
upon blood flow, an amount of CO2 is revealed from the
blood more than usually (hypocapnia).

(- Clewenute af CINCd Qk dlyestay copillany emda

e Tricketing of Capillayy Memb


© Ae renlte Oct Speed af NK hel!) Oy J Wh4

Oryyen Copulity ©F blard.


O deware af alveolyy venti hon,

220. Patient was diagnosed a considerable reduction


of the perfusive ability of the lungs. What are possible

causes of this disorder?

e Decrease of pulmonary blood circulation (hyperemia


venous congestion ischemia thrombosis embolism)
e Disorders of ventilation perfusion ratio

104|Page
pis ‘

Shock , Colla pre)


= Ar ord 2
SS aac Tg ht ;
vent Niche Con erage lid y).
Aira
ole (Gee —
ET loo dk co Agechan ia lungr
=Mdol IONS

— ‘4
Pulmonary Gniee

= Con
Pom .
eS 4 ACU Ved cardiac JI Worahe |

221. Patient was diagnosed a considerable reduction


of the diffusive ability of the lungs. What are possible

causes of this disorder?

e Alveolar wall thickening Capillary wall


e thickening Intra Alveolar edema capillary
e dilatation
e Interstitial edema

222. What else types of respiration belongs to


terminal (agonal) respiration?
e Kussmaul respiration
e Apnea
e Gasping

105|Page
223. Periods of respiration are substituted by periods
of apnea. What types of respiration belongs to this
group?
e Cheyne- Stokes
e Biot
e Apneustic breathing

224. What causes lead to restrictive respiratory


insufficiency?
e Atelectasis
e Emphysema
e Pneumosclerosis
5

ein SYM Ke Ir 9 {
St fee Lond
SU) J) FeTmEMA
Ny powa
\ S audcor
\
t ,
Ng 4

\nwé LOR
gy Kon
VORA 4+ Chay Diva \ On

\
ubex Lala \\p

© \
4

Cke ‘ 5
Ye

Clty o vcley
S> f
blaod
( \ if labo
. \ Ny
USSU Seo Ni\ gh Conch
8

ef Skyyen,

106|Page
225. What reasons of tachypnea do you know?
e Atelectasis
e Pneumonia
e Congestion

226. What reasons of hyperventilation do you know?

@ Circulatory
@ hemic hypoxia anemia

227. What reasons of terminal respirations do you


know?
e Cutting of vagus nerve extreme excitability of
respiratory center

228. Causes and mechanisms of pulmonary edema?


e lt happens when liquid gets filtrated via pulmonary
circulation more frequently Liquid accumulates in the
Interstitium.
e then goes to the interstitium under the pleura and near
the bronchi and prevents liquid from entering the alveoli
after the liquid enters the alveoli.

107
| Page
229. What non respiratory functions of the lungs do
you know?
Defence mechanism
Maintenance
Regulation of body temp
Regulation of acid base balance
Metabolism of biologically active substances

230. What are protective factors, which act in the


stomach?

Gastric mucus
sufficient blood supply
Intestinal hormones secretin
Pancreatic hormone glucagon
Somatostatin
Prostaglandins
Bicarbonates
Neutrophilic function of gastric nerves
High regenerative ability of epithelial cell

231. What are simulative factors, which act in the


stomach?

e Active pepsin
Hydrochloric acid

108| Page
e Vagal stimulation
e Acetylcholine
e Catecholamines
e Glucocorticoids
e Insulin
e Gastrin and cholecystokinin
e Bile acids
e Histamine
e H. Pylori
e Antigastric antibodies

232. Choose factors, which determine genetic


predisposition in ulcer disease.
e People with deficiency of alpha 1 antitrypsin and
glycoprotein of mucus
e People with parent of blood group O
e People with 1.5-2 times more parietal cells in gastric
mucosa.

233. A 67-year-old woman, who has cholecystitis for a


long time, suddenly developed sharp pain in the
upper part of abdomen, nausea, and vomiting after
food intake. Acute pancreatitis was diagnosed in this
patient. What are the peculiarities of acute pancreas
inflammation?
e Edema and hemorrhage into organ
109| Page
Systemic changes in the organism predominate over the
local ones
e Alteration because of necrosis of adipose tissue and
parenchyma of the gland
Links of pathogenesis rapidly develop like chain
branching reactions and vicious circles

234. Experimental animal was fixed on the fixing tool


and left without nutrition for day. After dissecting in

stomach the ulcers are found. What mechanisms of it


ulcerogenic acting ?
infectious mechanism
Neurogenic mechanism
Humoral mechanism
Chronic inflammation
Disorders of gastric motility
Disorders of blood circulation
Combination-due to stress
Genetic mechanism
Medicaments

235. A patient age 37 was admitted into a surgical


department with the signs of acute pancreatitis;
vomiting, diarrhea, bradycardia, hypotension,
dehydration of the organism. What are endogenic
etiological factors, which can lead to this disease?
110|Page
Disorders of blood circulation in pancreas (thrombosis,
ischemia)
Sclerosis of pancreatic vessel
Occlusion of the pancreatic duct by edema,
concrements polyps and tumors
Entry of bile into the pancreatic duct
Autoimmune aggression

236. Which negative complications of hyposalivation


do you know?
Tooth demineralization and carries
Oral infections (candidiasis)
Gingival changes
Oral malodour
Altered taste
Swallowing difficulties
Sjogren’s Syndromes- dryness of oral mucosa, eyes and
upper respiratory tract.
Atrophic-inflammatory processes of Oral mucosa, eyes
and upper respiratory tract.
Disturbs Mastication and Swallowing.
Demineralisation of teeth, oral candida infections and
mucositis.
Stomatitis
Caries

111|Page
237: Which negative complications of Hypersalivation
do you know?
e Physical complications include: perioral chapping and
maceration with secondary infection, dehydration, and
foul odour.
e The psychosocial complications include isolation,
barriers to education (such as an inability to share
books or computer keyboards), and increased
dependency and level of care.
Dehydration-large loss of saliva.
e|mpairment of stomach digestion by neutralisation of
gastric acid by saliva.
e Nat & Cl-, | k+in saliva

238. Etiology and pathogenesis of gastric ulcer and


duodenal ulcer.

ETIOLOGY: The most common causes of peptic ulcers are


infection with the bacterium Helicobacter pylori (H. pylori)
and long-term use of aspirin and nonsteroidal anti-
inflammatory drugs (NSAIDs) (Advil, Aleve, others). Stress
and spicy foods do not cause peptic ulcers

Pathogenesis:
-Gastric acid hypersecretion

-Impaired duodenal bicarbonate secretion

112|Page
e Endogenous Factor- due to gastric secretion regulations-
“acidity and pepsinogen
hypersecretion corrosive effect, Zollinger -Ellison
syndrome, Autoimmune aggression,

e Genetics predisposition.
Risk factors- drugs, alcohol, smoking eating patterns,
emotional stress.

239. Violation of cavernous digestion in the intestine,


causes, mechanisms and manifestations.

e Cavernous digestion takes place in the lumen of the


small intestine, violation occurs on impairment of bile
and pancreatic juice secretions...

(Blue book page:425)

e Bile salts emulsify fats, promote intestinal and colonic


motility, have bactericidal effect
e Acholia-absence pf bile
e Hypocholia- insufficient bile. (Due to obstruction by
stones)

113 |Page
Both accompanied by indigestion and malabsorption of fats,
decreased peristalsis and intensified putrefaction and
fermentation, steatorrhea

e Pancreatic juices contain proteolytic enzymes-inactive


form (trypsin, chemo trypsin, elastase,
carboxypeptidase, kallikrein, phospholipase A) others-
active form (lipase, amylase, RNase, DNase)
e Violation caused by

Cystic fibrosis

Occlusion of compression of pancreatic duct

e Acute or chronic pancreatitis


e Neurohumoral disturbance

e Secretin (secreted in duodenum)-activate secretion of


water and hydro carbonates
e cholecystokinin/pancreozymin -contraction of gall
bladder & production of pancreatic enzymes
® pancreatic insufficiency- steatorrhea, Impaired protein
digestion, impaired nucleic acid hydrolysis and starch
splitting

240: Disorders associated with secretary pancreatic


insufficiency.
eChronic pancreatitis

eCystic fibrosis

114|Page
eSchwachman—Diamond

syndrome (SDS), or Schwachman-Bodian—Diamond


syndrome

@ Celiac disease
e Diabetes (type | and type Illc)
@ Crohn’s disease
e Ulcerative colitis
e Acute pancreatitis

e Zollinger-Ellison syndrome
Changes in pancreatic secretion cause serious indigestion
because they produce main digestive enzymes.

* Overeating, abuse of fattening food can lead to increased


secretion of pancreatic juice.

Insufficiency is due to occlusion/compression of pancreatic


duct, cystic fibrosis of pancreas, acute/chronic pancreatitis or
duodenitis, disturbance of the neurohumoral mechanism of
pancreatic secretion regulation, Schwachman-diamond
syndrome.

* Insufficiency can lead to steatorrhea, weight loss, diarrhea,


edema due to low albumin from malabsorption, indigestion
of protein due to insufficient production of peptidases, also
impairs nucleic acid hydrolysis and starch splitting.

* Mutation in the Cystic fibrosis transmembrane


conductance regulator (CFTR) decreases bicarbonate
115|Page
secretion by pancreatic ductal cells lead to protein plugging,
duct obstruction and development of pancreatitis.

241: Pancreatitis: types, causes, pathogenesis of


acute pancreatitis.
eThere are two main types, acute pancreatitis and chronic
pancreatitis

Causes:

Acute pancreatitis causes include:

Autoimmune diseases
Drinking lots of alcohol
Infections
Gallstones
Medications
Metabolic disorders
Surgery
Trauma

Chronic pancreatitis causes include:

Cystic fibrosis
Family history of pancreas disorders
Gallstones
High triglycerides
Long-time alcohol use
Medications

116|Page
Pathogenesis. Acute pancreatitis occurs when there is
abnormal activation of digestive enzymes

within the pancreas. This occurs through inappropriate


activation of inactive enzyme precursors

called zymogens (or proenzymes) inside the pancreas, most


notably trypsinogen.

242: Pancreatic shock.

e Necrotizing pancreatitis may occur in severe acute


pancreatitis. In necrotizing pancreatitis, parts of

The pancreas may die and body fluid may escape into the
abdominal cavity, which decreases blood volume and

results in a large drop in blood pressure, possibly causing


shock and organ failure.

117 |Page
9 fomereake shock /hypovellmie
e Shocle
~) Aewte. ponevealils /aewvte necrohsing

Panresreg ke atlavenekea ;
Niecrves's 4
ponanentt uv. dene
— Ports 4

as ss els 4
as WwW AS

bomen
kesuea

pdb
CQ. do eke

Jang er hans y» Noasceator Wy all j

Aa u* wh ene
NEN erlt ~'
“Vato We. peneveske Pevend

canty ,
es “ We Avwinal
ac Strela

Paw is We ves
——} Nodowinal

uth decreasing pllaswre ‘


Sinica Syphm uy

Systemic heporie .
a hisonw. Caos er
bled
eo dneck 2
las can Led
=F

243. Malabsorption. Causes and mechanisms of


malabsorption, the basic pathogenesis of clinical
manifestations:
Possible causes, Factors that may cause malabsorption
syndrome include:

damage to the intestine from infection, inflammation,


trauma, or surgery
prolonged use of antibiotics
other conditions such as celiac disease, Crohn’s disease,
chronic pancreatitis, or cystic fibrosis

118|Page
e lactase deficiency, or lactose intolerance
e certain defects that are congenital, or present at birth,
such as biliary atresia, when the bile ducts don’t develop
normally and prevent the flow of bile from the liver
diseases of the gallbladder, liver, or pancreas
® parasitic diseases
e radiation therapy, which may injure the lining of the
intestine
e certain drugs that may injure the lining of the intestine,
such as tetracycline, colchicine, or cholestyramine

(Mechanism of malabsorption): The main purpose of the


gastrointestinal tract is to digest and absorb nutrients (fat,
carbohydrate, protein, micronutrients (vitamins and trace
minerals), water, and electrolytes. Digestion involves both
mechanical and enzymatic breakdown of food. Mechanical
processes include chewing, gastric churning, and the to-and-
fro mixing in the small intestine. Enzymatic hydrolysis is
initiated by intraluminal processes requiring gastric,
pancreatic, and biliary secretions. The final products of
digestion are absorbed through the intestinal epithelial cells

Clinical manifestation:

e bloating and stomach distention.


e diarrhea
e fatigue
e gas
e steatorrhea, or stool that is pale to white.
e stools that appear “greasy” in texture.
e stomach cramping.
119|Page
e Weakness
e bone pain
e bones that fracture easily
e iron-deficiency anemia, which can result in shortness of
breath
e muscle wasting
e asore tongue
e weight loss

Due to infective agents

HIV related malabsorption

Intestinal tuberculosis

Parasites e.g., Diphyllobothrium (fish tape worm) (B12


malabsorption), giardiasis (Giardia lamblia),
hookworm (Ancylostoma duodenale roundworm, and
Necator americanus)
Traveller’s diarrhea
Tropical sprue

Whipple's disease
Due to structural defects
Blind loops

Fistulae, diverticula and strictures

Infiltrative conditions such as amyloidosis, lymphoma,


eosinophilic gastroenteritis

120|Page
Inflammatory bowel diseases, as in Crohn's disease

Radiation enteritis

Short bowel syndrome

Systemic sclerosis and collagen vascular diseases


Due to surgical structural changes

Bariatric surgery (Weight loss surgery)

Gastrectomy; Vagotomy

Due to mucosal abnormality


Coeliac disease
Cows' milk intolerance

Fructose malabsorption
Soya milk intolerance

Due to enzyme deficiencies


Lactase deficiency inducing lactose intolerance
(constitutional, secondary or rarely congenital)
Intestinal disaccharidase deficiency

Intestinal enteropeptidase deficiency

Sucrose intolerance

Due to digestive failure

Bile acid/Bile salt malabsorption

Bacterial overgrowth

Obstructive jaundice

121|Page
Primary bile acid diarrhea

Terminal ileal disease such as Crohn's disease


Pancreatic insufficiencies:
Carcinoma of pancreas

Chronic pancreatitis

Cystic fibrosis

Zollinger-Ellison syndrome
Due to other systemic diseases affecting GI tract

Abetalipoproteinemia

Addison's disease
Carcinoid syndrome

Coeliac disease
Common variable immunodeficiency (CVID)

Fiber Deficiency

Hypothyroidism and hyperthyroidism

Diabetes mellitus
Hyperparathyroidism and Hypoparathyroidism

Malnutrition.

244: Which mechanisms belong to the pathogenesis


of hepatic coma.

122|Page
The first stage of hepatic encephalopathy is characterised by
an inverted sleep-wake pattern (sleeping by day, being
awake at night). The second stage is marked by lethargy and
personality changes. The third stage is marked by worsened
confusion. The fourth stage is marked by a progression to
coma.
1) Disturbances of synaptic transmission.

2)Disturbances of energy metabolism resulting in a deficiency


of ATP.

3) Disturbances in the function of cell membranes resulting


from the direct action of cerebrotoxic substances.
4) Development of metabolic acidosis and associated
metabolic disturbances of electrolytes.

245: What manifestations of acholic syndrome?


e Alcohol use disorder can be mild, moderate or severe,
based on the number of symptoms experience. Signs
and symptoms may include:
e Being unable to limit the amount of alcohol you drink
e Wanting to cut down on how much you drink or making
unsuccessful attempts to do so
e Spending a lot of time drinking, getting alcohol or
recovering from alcohol use
e Feeling a strong craving or urge to drink alcohol
e Failing to fulfil major obligations at work, school or
home due to repeated alcohol use

123|Page
e Continuing to drink alcohol even though you know it's
causing physical, social or interpersonal problems
e Giving up or reducing social and work activities and
hobbies
e Using alcohol in situations where it's not safe, such as
when driving or swimming
e Developing a tolerance to alcohol so you need more to
feel its effect or you have a reduced effect from the
same amount
e Experiencing withdrawal symptoms — such as nausea,
sweating and shaking — when you don't drink, or
drinking to avoid these symptoms.

246: What manifestations of cholemic syndrome?


Cholemia is a condition caused by the presence of excess bile
in the blood. Its symptoms can include somnolence
(drowsiness), yellow tinge to skin and whites of eyes, fatigue,
nausea and, in extreme cases, coma. It is often an early sign
of liver disease.

124|Page
247: Aman aged 38 with icteric skin has anemia,
enlarged spleen, hyperbilirubinemia, urobilinuria,
hypercholic stool. What is type of jaundice?
Suprahepatogenous jaundice

a) hemolytic anemia
b) manifestations: stercobilin forms excessively and
intensively colors the feces and urine.
INCREASED content of indirect bilirubin and direct bilirubin is
ABSENT.

(Page no.453)

125|Page
248: A boy 16-year-old complains of weakness,
nausea, pain into right hypochondrium. On
examination was revealed jaundiced sclera and skin.
In the blood is large conjugated bilirubin, in the faeces
stercobilin in little amount, in the urine-urobilinogen
and bilirubin. What type jaundice?
In the clinical picture of typical hemolytic jaundice the signs of hepatic insuffi-
ciency are absent. Cholemic syndrome is absent (bile acids do not enter the blood).
Acholic syndrome is absent, and bile acids enter the intestine. Digestion is not dis-
turbed, Stercobilin (which is not toxic) forms excessively and intensively colors the
feces and urine. Biochemical examination of the blood shows an increased content
of indirect bilirubin; direct bilirubin is absent. It is a typical variety of hemolytic
jaundice. Such a patient is called «more yellow than ill», (However, it should be
taken into account that such a patient suffers from anemia.)
A variant of hemolytic jaundice of newborns — nuclear icterus — was described
above,
Different types of jaundice may combine.
If hemolytic jaundice is associated with hepatocyte damage or biliary tract ob-
struction, clinical manifestations would be more complete. Parenchymatous jaun-
dice may join the hemolytic one, Although the liver possesses potent bilirubin
conjugation ability, hepatocellular insufficiency may occur in hemolytic anemia be-
cause many hemolytic poisons damage hepatocytes simultaneously with hemolysis.
Besides, hypoxia, which develops due to hemolytic anemia, may limit the activity
of hepatocyte enzymes participating in bilirubin conjugation.
Mechanical jaundice can be combined with the hemolytic one as a result of
occlusion of the biliary tracts by bile thrombi and stones of bilirubin, cholesterol
and calcium.

126|Page
249: Specific manifestations of portal hypertension do
you now?
The main symptoms and complications of portal
hypertension include:

e Gastrointestinal bleeding marked by black, tarry stools


or blood in the stools, or vomiting of blood due to the
spontaneous rupture and hemorrhage from varices
e Ascites (an accumulation of fluid in the abdomen)
e Encephalopathy or confusion and forgetfulness caused
by poor liver function
e Reduced levels of platelets, blood cells that help form
blood clots, or white blood cells, the cells that fight
infection.

Causes:

The most common cause of portal hypertension is cirrhosis of


the liver. Other causes of portal hypertension include blood
clots in the portal vein, blockages of the veins that carry

the blood from the liver to the heart, a parasitic infection


called schistosomiasis, and focal

Nodular hyperplasia, a disease seen in people infected with


HIV.

127|Page
250: Specific manifestations of portal hypertension do
you now?
e Gastrointestinal bleeding

eAscites>Inc.hydrodynamic blood pressure, Dec. Oncotic


blood pressure

e Na+ retention, Liver/Portal Cirrhosis

eSplenomegaly

e@Hypersplenism

e@Hepato-lienal syndrome
e@Hepato renal syndrome

251: Specific manifestations of Crigler — Najjar


syndrome do you now?

e Fatigue
@ Abdominal pain related to biliary stones
e Diarrhea
© Vomiting
e Fever
@ Confusion
e Slurred speech
e Difficulty swallowing

128|Page
e Change in gait, staggering, frequent falling
e Seizures
Affected infants develop severe, persistent yellowing of
the skin, mucous membranes and whites of the eyes
(jaundice)

252: Specific manifestations of Gilbert’s syndrome do


you now?
Occasionally a slight yellowish color of the skin or whites
of the eyes may occur. Other possible symptoms include
feeling tired, weakness, and abdominal pain.
tummy (abdominal) pai
feeling very tired (fatigue)
loss of appetite
feeling sick
dizziness
irritable bowel syndrome (IBS) — a common digestive
disorder that causes stomach cramps,
bloating, diarrhoea and constipation
problems concentrating and thinking clearly (brain fog)
a general sense of feeling unwell

253: Reasons of hemorrhagic syndrome during liver


insufficiency?
129|Page
Excessive dietary energy intake is believed to be the
cause of fatty liver hemorrhagic syndrome.
Hypo-proteinemia
Hypotonia
Ratio of Albumin and Globulin is |
Anemia
Leukopenia
Thrombocytopenia
Vascular-Platelet Hemostasis
Coagulopathy
Hypovitaminosis-K

254: Lack of hepatic excretory function: causes,


mechanisms and clinical implications.

Cause due to impairment of bile formation due to


Alcohol consumption, Viral infections, Copper/

lron accumulation, drugs, toxic substances and tumor


of pancreas, Fatty liver, bile tract

Obstruction and stones formation in kidney and


Gallbladder.
It causes syndromes: Cholemic, Acholic and Mechanical
jaundice.

130|Page
255: What reasons of hepatic coma do you know?
Hepatic coma may be triggered by:
@ infections such as pneumonia

ekidney problems

edehydration
ehypoxia, or low oxygen levels

@recent surgery or trauma


e Medications that suppress your immune system

e@eating too much protein

@medications that suppress your central nervous system,


such as barbiturates or benzodiazepine tranquilizers

e electrolyte imbalance, especially a decrease in potassium


after vomiting or taking diuretics

@ Toxins

256: What manifestations of ascites do you know?


@nauseated

eless hungry than usual

etired

ebreathless
@urinary urgency and constipation

131|Page
e@Abdominal pain and bloating

eLiver failure

257: Etiological factors contributing to the


development of polyuria are:
ePolydipsia

e Diabetes mellitus
@ Diabetes insipidus

Kidney disease
eLiver failure
Medications that include diuretics
eChronic diarrhoea
eCushing’s syndrome
ePregnancy

e Anxiety

@Hypercalcemia

e Alcohol

eCaffeine

132|Page
258: Etiological factors contributing to the
development of oliguria are:
e Dehydration

e Medications
elnfection or trauma

e Urinary tract obstruction


eSubstantial blood loss and shock

259: Pathogenetically, the development of anemia in


kidney pathology is associated with:
e anemia has been associated with more severe adverse
outcomes
e cardiovascular complications including left ventricular
hypertrophy
e congestive heart failure.

260: Pathogenetically, "urinary syndrome" is due to


the development of the patient:
e Urethral syndrome has various causes. Common causes
may include physical problems with the urethra, such as
abnormal narrowing or urethral irritation or injury.

The following can cause irritation to the urethra:

133|Page
e scented products, such as perfumes, soaps, bubble bath,
and sanitary napkins
e spermicidal jellies
e certain foods and drinks containing caffeine
e chemotherapy and radiation

261: The reasons for the development of prerenal


acute renal failure may be:
Causes of prerenal acute renal failure include:
e Severe blood loss and low blood pressure related to major
cardiac or abdominal surgery, severe infection

(sepsis), or injury.
@ Medicines that interfere with the blood supply to the
kidneys

e Severe dehydration.
e Severe burns.

e Pancreatitis.

262: The most characteristic manifestations of


urolithiasis are:

General signs and symptoms may include:


e@Renal or ureteral colic

134|Page
e@Blood in urine (hematuria)

e Urinary tract infection

e Abdominal pain

263: The development mechanism of acute renal


failure includes:
consists of 3 main mechanisms:
prerenal
Intrinsic
Obstructive
Violation of Blood Circulation in the Kidney-
Prerenal Factors.
Direct damage to the structure of the Glomeruli
And Tubules- Intrarenal Factors.
Violation of Urine Outflow- Postrenal Factor.

264: In the pathogenesis of renal edema, the leading


role is played by:
e Edema is one of the cardinal clinical features of
nephrotic syndrome (NS), and may range from localized
puffiness to massive anasarca.

135|Page
The pathophysiological mechanisms of edema formation
in NS were discussed over several decades.
Abnormal accumulation of interstitial fluid results from
the combination of: abnormal renal sodium retention
(primary and secondary) increased capillary wall
permeability (related to the release of vascular
permeability factor and other cytokines).
Nephritic Edema due to acute and Chronic
Glomerulonephritis and also 77% secretion of
renin, aldosterone & Vasopressin with hypo-
proteinemia and dys-proteinemia.

265: Typical cases of pyelonephritis, the following


clinical syndromes are:
INTOXICATION
ARTERIAL HYPERTENSION
EDEMA
ANEMIA
POLYURIA
OLIGURIA
HYPOSTENURIA
LEUKOCYTOURIA
HEMATURIA
Syndrome of acute renal insufficiency

136|Page
e Syndrome of chronic insufficiency
e Example of secondary chronic insufficiency-
hemorrhagic syndrome, anemia, GIT syndrome
e (Dyspeptic sign), Cardiovascular syndrome,
Neuromuscular syndrome, Respiratory insufficiency,
e Peripheral neuropathy, Encephalopathy, arthritis.
e Urinary syndrome - Polyuria, Oliguria, Hyposthenuria,
Leukocyturia, Hematuria, moderate
e proteinuria & cylinderuria.

266: The mechanism for the development of


nephrotic syndrome is:
e IMMUNE MECHANISM (DELAYED TYPE HYPERSENSITY)
BASED ON IMMUNOCOMIPLEX REACTIONS TYPE III
REACTION ACCORDING TO COOMBS CELLS
e METABOLIC MECHANISM AND ASSOCIATED PHYSICO-
CHEMICAL CHANGES

Glomeruli damage: - Amyloid formation


1.Fibrinogen & glycoproteins & lipoprotein on basement
membrane

2.Form IgM

3.Because of this permeability of plasma protein increases at


basement membrane

137|Page
eMiassive proteinuria- main symptoms: -

e Because of decrease electric charge of capillary wall.

elt is depletion of Sialoprotein & Polymorphonuclear


leukocytes are accumulated.

eLysosomal enzyme damage basement membrane. So that


higher molecular protein get filtrated.

267: The renal symptoms of acute glomerulonephritis

include:

OLIGURIA
PROTEINURIA
AZOTEMIA
HYPERTENTION
EDEMA
HEMATURIA
DISORDER OF CENTRAL NERVOUS SYSTEM

268: Uremic cardiopathy is characterized by:


CHARACTERIZED BY DIASTOLIC DYSFUNCTION AND
MARKED LEFT VENTRICULAR HYPERTROPHY WITH
PROFOUND VENTRICULAR FIBROSIS

138|Page
269: Violation of acid-base balance with uremia
manifests itself:

e VIOLATION OF AMMONIOGENESIS
e IMPAIRED ACIDOGENESIS

270: The renal factors of kidney stone formation are:


e CALCIUM OXALATE
e URIC ACID

139|Page
Clee fre totthidal

Te en ee eee ee
=. Heuclitany Ant Hihelor guvodes«
d ne
Te =
=
| byare: tmpoatrt d ame. Aue wv, ebeootypdio-.
fo Zs
: -
“CU ot leggpsn
=
e, ghyes)
7
_
d Go Sope

271: Which of the following indicators characterize


the renal tubule dysfunction?
e CHRONIC RENAL INSUFFICIENCY WITH SYPMTOMS AS
PROTENNURIA (ALBUMINURIA), - OLIGURIA (REDUCTION
IN NORMAL QUANTITY OF URINE PER DAY.

140|Page
1.Increase aldosterone- increase reabsorption of bicarbonate
(tubular alkalosis)

2.Decrease aldosterone- sodium, potassium reabsorption


disturbs (tubular acidosis),

hyperkalemia

3.Decrease ADH- polyuria (more than 10L) also called


Hyposthenuria

4.lsothenuria (density 1010)

5.Phosphaturia, Calciuria (present in urine)


6.Phosphotemia & hypocalcemia-decrease amount in blood

7.Aminoaciduria, Cylinduria.

1.Acidogenesis (excrete hydrogen)


2.Ammoniogenesis (excrete ammonium)

141|Page
e Together (1&2) called Excretory tubular acidosis.

3.Hyperurecimia- Utica acid accumulation & gout.

4.Tubulointestisial syndrome.

272: Indicate the main mechanisms contributing to


the development of renal hypertension:
NARROWED
RENAL ARTERY SUPPLY THE KIDNEY
CHRONIC HEART FAILURE

From JG apparatus- rennin formation


Rennin secretion depend on pressure in the afferent
vessels
Rennin is proteolytic & alpha-globulin of plasma
(angiotensinogen)
Angiotensinogen gives out AG1 & AG1 further gives out
AG2 (50 times more active than
AG1)
AG2:-
AG2 stimulates growth factors {1-vessel wall
hypertrophy}
Endothelial dysfunction (NO Deficiency) {2-
vasoconstriction}

142|Page
Activation of pressure factors (endothelial 1st) {2}
Activation of sympathetic effect {2- vasoconstriction}
Bradykinin destruction {2- vasoconstriction}
Increase aldosterone secretion {3-sodium & water
retention}

Depression of PG synthesis.

143|Page
273: Urinary syndrome includes:
e INTERSTITIAL CYSTITIS
e OVERACTIVE BLADDER OR PAINFUL BLADDER
° SYNDROME, URETHRAL SYNDROME

274: Which complications are characterised nephrotic


syndrome?
e BLOOD CLOTS

144|Page
e HIGH BLOOD
e CHOLESTEROL AND ELEVATED BLOOD SYNDROME
e POOR NUTRITION
e HIGH BLOOD PRESSURE
e ACUTE KIDNEY INJURY
e CHRONIC KIDNEY DISEASE
e INFECTION

275: Which complications are characterizing Nephritic


syndrome:
eOliguria

eProteinuria

eHematuria

eHypertension

Example-
1.post-streptococcal glomerulonephritis

2.Diffuse proliferative glomerulonephritis

3.Rapid progressive glomerulonephritis

4.Good Pasture syndrome

5.Berger’s disease.

145|Page
276- A patient aged 41 complains of weakness,
sweating, fever, tremor of hand, BP-160/90 mm Hg.
Diffuse toxic goitre was diagnosed (Basedow’s
disease). What are the mechanisms of impairment of
the functions in this disease?
Etiology- Thyrotoxicosis in men is mental trauma in men.

Infection & allergy are considered to be predisposing factors.

Pathogenesis-

elncrease thyroxin & T3

elncrease mitochondria in target cell & they swell up

elncrease activity of oxidising enzyme (Succinate


dehydrogenase)
eCytochrome oxidase

eGlycerophosphate dehydrogenase sodium

eATPase

eGlycolysis in liver & muscle- hyperglycaemia & utilisation of


glucose increase (increase hexokinase activity)

eThyroid hormone inhibits carbohydrate transformation into


fat.

eThyroid hormone increases cholesterol disintegration


elncrease fat oxidation in liver
elincrease sensitivity of fatty tissue to adrenaline

eShows lipolytic effect

146|Page
eDeport of lipid increase, so weight loss,
hypercholesterolemia & ketonemia occur.
e Negative nitrogen balance because of protein catabolism.

Manifestation-

elncrease thyroid gland size


e@Exophthalmus

elncrease BMR

e Tachycardia

eArrhythmia

e Trembling fingers

Psychic excitement increase

eVibration emaciation

eDystrophic change in myocardium

e Atrioventricular conductivity disturbs

eLeft ventricle overload


e@Heart rate inadequately to cholinergic & adrenergic
influences.

147|Page
278: A patient aged 23 complains of headache,
changes of his appearance (increase of size of feet,

fingers, features of the face), hoarseness of the voice,


worsening of the memory. The disease has begun 3
years ago without any causes. Increase of superciliary
arches, nose, and tongue were found during physical
examination of this patient; the blood sugar was
normal; the urinalysis is out of changes. Pituitary
gigantism was diagnosed. What is the pathogenesis in
this disease?

e TUMOR OF THE PITUITARY GLAND


e EXCESSIVE GROWTH HORMONE
e GH increase permeability of membrane to AA (increase
translation) inhibit proteolysis
e Increase lipolysis, inhibit lipid formation from
carbohydrates
e Increase deport of fat & increase oxidation of lipid
e Ketone body present
e Meta steroid diabetes
e Meta hypophyseal diabetes lipolysis increase. &
Increase acetylCoA gives out increase
e Gluconeogenesis
e Effect of GH on connective tissue, bone, cartilage
stimulates formation of oxyproline &

148|Page
e condrolethin sulphate
e lt happens only in young age
e And before epiphyseal cartilage fusion

279: Parathyroid gland are removed during the


operation on thyroid gland. What pathological
changes will be observed after this operation?
e OSTEOPOROSIS
e RHABDOMYOLYSIS

@ Reduce Ca+2 level

elncrease phosphate

eDecrease serum PTH

149|Page
eTetany

elncrease QT interval

eCalcification in basal ganglia


e Dental carries (enamel defect or cataract)

e Decrease calcium absorption from intestine

elncrease phosphate reabsorption in tubule


eNeuromuscular excitability increases

e Decrease conversion of ammonium ion into urea in liver

e Decrease nitrous metabolism

ePolyuria

eMemory defect

e@Parkinsonism

Metabolic acidosis
e Osteopathy

eTumour of bone
eChondrocalcinosis
e@Peptic ulcer

ePancreatitis

eVomiting

eConstipation

eLaryngospasm

eBronchospasm

150| Page
280: A woman with primary hyperparathyroidism has
periodically repeated attacks of renal colic.
Ultrasound examination shows presence of small
stones in her kidneys. What pathological changes
will be observe in this disease?

e NEPHROLITHIASIS
e HYPERCALCEMIA

elncrease PTH

eincrease serum calcium


eDecrease serum phosphate

eMost common in female

eRight inferior PTH gland infected


e@MAN 1 gene mutation; sporadic

1.In kidney- increase stone or Polyuria

2.GIT- nausea, constipation (peptic ulcer disease or


pancreatitis)
3.CNS- confusion, anxiety, psychosis

4.Bone- osteoporosis (o radial side of phalanges)

Skull- salt paper appear

151|Page
5.Cyst present- hemorrhage, brown tumour

6.Joint- Pseudogout
7.Accumulations of calcium, salt in kidney, lungs & heart. The
vessel wall became thick & BP increase

8.Generalised fibrous osteodystrophy


9.Nephrocalcinosis, nephrolithiasis, grave renal insufficiency.

281: Persistent hyperglycemia developed in a patient


with Cushing’s syndrome under the influence of

excessive amount of glucocorticoids. What are


manifestations of this syndrome?
DIABETES MELLITUS TYPE 2
HIGH BLOOD PRESSURE
MOON FACE
Increase arterial BP, hypertension
Dystrophic change in muscles, bone, joints
Immunodepression
Skin atrophy
Hyperglycaemia due to Gluconeogenesis activation.

282- After physic exertion a patient with


pheochromocytoma complains of tachycardia,

152|Page
increased arterial pressure, & sharp pain in epigastric
area. What changes are characterized this tumor?
e@ Headache (because of increase catecholamine)

@Episodic hypertension

ePostural hypotension (decrease plasma)


ePalpitation increases heart rate, sweating anxiety

elncrease metanephric

eVanillylmandelic acid increases in 24 hrs in urinary levels


eTumor of chromaffin cells (it is benign)

@MEN 2 AA/ 2B syndrome

283- The development of polydipsia in diabetes


insipidus can lead to:-
ePolyuria

eDiuresis increases up to 25 L per day

e Dehydration

ePolydipsia (It is thrust)

e@Hypovolemia

e Decrease arterial hypertension

@Hypoxia

153|Page
elt is because of low level of vasopressin or receptors at DCT
& collecting tubule of vasopressin.

284- A woman aged 44 complains of general malaise,


pain in the heart area, & considerable increase of
body weight. At examination of this patient following
symptoms are revealed: lunar face, hirsutism, BP
165/100 mmHg, height 164 cm, weight is 103 kg,

the adipose deposits mainly on the neck , shoulders,


abdomen, persistent hyperglycaemia. What more
manifestations of this endocrine disorders?

e The disease is Cushing’s syndrome.


Manifestations-

e|ncrease cortisol level

Central obesity
e@Moon face
elncrease arterial BP, hypertension

eDystrophic change in muscles, bone, joints


eimmunodepression

eSkin atrophy

e@Hyperglycaemia due to Gluconeogenesis activation.

154|Page
285- Which of the following characterizes Cohn’s
syndrome?

lt is primary hyperaldosteronism caused by adenoma


of glomerular zone of cortex.

Manifestations-
eSodium ion retentions
eLoss of potassium & chlorine
eArterial hypertension

eMyasthenia & Paresis spasm of muscles


eContractile disfunction of myocardium due to loss of ions

eNon-gaseous alkalosis
eWater retention but not Edema

286- The development of Addison’s disease causes a


decrease in the number of the following hormones:-
e Decrease aldosterone

elncrease ACTH

e@lncrease MSH

287. With pituitary nanism, it is noted

155|Page
e DECREASED GROWTH HORMONE
e UNUSUAL STRUCTURE
e SLOW GROWTH

288: The characteristic changes that occur in the


blood system under the influence of glucocorticoids
include:
INCREASED BLOOD PRESSURE
INCREASED CARDIAC CONTRACTILITY
INCREASED CARDIAC OUTPUT
ALTERED BAROREFLEX
CARDIOVASCULAR RESPOMSES TO ACUTE STRESS

289: Indicate the mechanism of change in blood


glucose in Addison's disease:

e THERE IS DISTURBED FEEDBACK REGULATION OF


GLUCOSE
e EXCESSIVE PRODUCTION OF CORTISOL WHICH INCREASE
THE INSULIN PRODUCTION AND FUNCTION

290. Trans hypophysial regulation is the basis for:

156|Page
REGULATION OF THE BODY FUNCTION BODY’S
FUNCTION
REGULATION OF GROWTH OF THE BODY

291: The production of which hormones decreases


with total hypofunction of the anterior pituitary
gland:
PROLACTIN
ESTROGEN
PROGESTERONE
CORTISONE
ALDOSTERONE
T3(TRIIDODOTHYRONINE)
T4 THYROXIN

292: Which of the following changes in women testify


in favour of congenital adrenogenital syndrome:
e pseudo hermaphrodism
e virilization
e early menstruation
e early growth of breasts

157|Page
293. What are the characteristic manifestations of
central paralysis in the affected limbs?
e Muscle tone of extremities increased
e Spinal reflexes are greatly strengthened
e Reflex zones are expanded
e Hyperreflexia

294. Indicate neurotropic viruses (TORCH infection)


e Herpex virus
e Rubella
e HIV
e Meningococcus
e Polio virus
e Measles
e Cytomegalovirus
e Encephalitis virus

295. What signs characterize peripheral paralysis?


e Muscular hypotonia /atrophia (loss of reflexes)
e Decreased reflexes
e Absence of active movements
e Transient pain
158|Page
e Paraesthesia- thoracic spine
e Hypokinesia
e Muscular Atrophy of upper extremity

296. List the exogenous causes of damage to the


nervous system:

Physical factors:
e mechanical trauma
e Barometric pressure
e lonizing radiation

Chemical factors:

e neurotropic (cerebrotoxic) poisons


Industrial poisons
Alcohol
Narcotics
Psychotropics

Biology:

e Neurotropic viral infection


e Meningococcus
Turbcle bacillus
HIV
TORCH infections
Immune factors

159|Page
297. Indicate neurotropic microbial toxins:
Cerebrotoxin (botulin toxin)
Industrial poison
Alcohol poison
Narcotics
Psychotropics

298. List the conditions that determine the strength of

damage to the nervous system:


e localization of the damage (nervous centre, conductor,
synapses, receptor)

299. Damage to the cerebellum may be accompanied

® movement disorder(asynergia) & tremor


e Dysmetria
e Inability to perform rapid alternating
® movements(adiadochokinesia)
e Hypotonia
e Ataxic gait
e Ataxic dysarthria
e Nystagmus

160|Page
300. What signs characterize peripheral paralysis?
e Muscular hypotonia /atrophia (loss of reflexes)
e Decreased reflexes
e Absence of active movements
e Transient pain
e Paraesthesia- thoracic spine
e Hypokinesia
e Muscular Atrophy of upper extremity

301. In the pathogenesis of neurogenic degeneration


of cells of a denervated organ, the following
processes are important:
e Nervous trophism -mediator mechanism
e Non- mediated mechanism
e Vascular mechanism

302. List the specific mechanisms of damage to


neurons:
Exogenous and endogenous
e Exogenous (physical- ionisation radiation, trauma,)

161|Page
e Chemical — cerebrotoxic, industrial poison, alcohol,
narcotics
e Biological factors- neurotropic viral infection, TORCH,
HIV
e Disturbances of neurochemical process
e Disturbances of motor function, vegetative function

303. What is characteristic of a neurodystrophic


process- severe disorders develop when afferent
fibres and nerves are damaged:
e Structural disorders- muscle atrophy, dystrophic
changes
e Functional changes- an increase in the sensitivity of
denervated structures
e Metabolic disorders- inhibition of certain enzymes

304. Life-threatening complications in shock are due


to hypoxic cell injury resulting in
immunoinflammatory responses and activation of
various cascades (clotting, complement, kinin). These
include the following:
162|Page
Acute respiratory distress syndrome
e Disseminated intravascular coagulation
Acute renal failure
Multiple organ dysfunction syndrome

305. The pathogenic factors of hemorrhagic shock


include:
e blood loss
e Low cardiac output
e High total peripheral vascular resistance
e Blood plasma loss in burns, lesions
e Loss of fluids (deficit of extracellular fluid)
e Spasm of vessels of microcirculatory bed
e Centralization of blood circulation cause damage to
organs
e Hypovolemic shock develops

306. Torpid phase of hemorrhagic shock is


characterized by.
e Weakening effects of sympathoadrenal and pituitary —
adrenal system
e Reduction of cardiac output

163 |Page
e Depositing of blood
e Arterial hypoxemia

307. The pathogenic factors of anaphylactic shock


include.
e an exit of fluid from blood vessels
e Vascular tonus is reduced
e Expanding microcirculatory vessels &increases its
permeability
e Hemodynamic option
e Reduction of venous return to heart
e Dilation of capillary and capacitor vessels

308. The pathogenic factors of pain shock include.


fall of neurogenic tone of arterioles
Decrease in blood pressure
Bradycardia
Decreased cardiac output
Peripheral vasodilation & venous pooling

164|Page
309. The pathogenic factors of pancreatic shock
include.
e reduction of general peripheral resistance in result of

generalized dilation of vessels

e syndrome of intravascular disseminated coagulation of


blood
e Reduction of basal tone of vessels under action of
biologically active substances

310. The pathogenic factors of hepatic coma include.


e hypoglycemia
e Disorders of Water electrolytes balance
e Hyponatremia
e Level of aldosterone increases
e Increase sensitivity to glutamine
e Loss of potassium

311. The pathogenic factors of cardiogenic shock


include.
e Increase of peripheral resistance of vessel
e Tachycardia

165|Page
decrease of cardiac output
Decreased contractility of the heart
Severe disorder of heart rhythm
The reduction of venous return of blood to heart
Disorders of intracardiac hemodynamics
The cardiac tamponade, massive thromboembolism of
pulmonary artery (increase of heart filling pressure)

312. What manifestations of ketonemic


hyperglycemic coma do you know?
Respiratory failure
Vascular collapse
Low blood pressure
Decreased muscle tonus
Oliguria/anuria
Low blood pH (6.8-7.3)
Ecg signs of hyper kalmia first and hypokalaemia

313. What specific manifestations of lactacidemic

hyperglycemic coma do you know?


e Hypoxia -excess of lactate more than 2mM/1
consciousness suppression
e Heart failure (tachycardia, hypotension)
e Anemia

166|Page
Disorders of lung, kidney liver and infectious diseases
Pain in the muscles, heartache dyspepsia,
stomach-ache
Kussumals respiration develops

314. What manifestations of hyperosmolar


hyperglycemic coma do you know?

e Secretion of consular hormones (of the stress, trauma,


or infection)
e Dehydration
e Polyuria (osmotic diuresis)
e Glucose level increases in blood
e Hypernatremia
e Hyperchloremia
e Blood pressure decreases
e Hemodynamic instability lead to thrombosis

315. The pathogenic factors of pancreatic shock


include.
e Reduction of general peripheral resistance in result of
generalized dilation of vessels.

167|Page
e syndrome of intravascular disseminated coagulation of
blood
e Reduction of basal tone of vessels under action of
biologically active substances

316. The pathogenic factors of collapse include.


e Cardiovascular failure
e Rapid and significant decrease blood pressure
e Fall of vascular tone
e Reduction of venous blood inflow to heart
e Decrease of heart output
e Decrease venous pressure
e Infringement of tissues perfusion and metabolism

317. Collapse: general manifestations


e Coronary insufficiency
e Decrease cardiac output
e Hypoperfusion of tissue
e Venous congestion
e Redistribution of circulations
e Capillarotrophic insufficiency
e Tremors of fingers
e Inhibitionness
e Apathy
e Decrease of nervo-muscle excitation
168|Page
e Oliguria
e Increase of blood viscosity (Irregular heartbeat, low
blood pressure, seizures)

169| Page
Ina patient viral hepatitis complicated acute hepatic insufficiency, which transferred to coma. By the main chain in
pathogenesis of hepatic coma is increase in blood neurotoxic substances, at first all: {=Ammonia ~Phenol ~Indole ~Skatole
~Acetoin}

In a child 12 years after the carried hepatitis the signs of anemia appeared as a result of {=Depositing of iron disorder
“Transport of iron disorder ~Enhance excretion of iron with bile ~Insufficient absorption of iron in bowels ~Formation
erythropoietin disorder}

Ina child with hemolytic disease of the new born developed encephalopathy. Increase of what matter in blood caused
lesion of CNS? {=Unconjucated bilirubin ~Complex bilirubin-albumin ~Conjucated bilirubin ~Verdoglobin ~Biliverdin}

Ina children 7 years on the base of the developed hepatitis B resistance to the staphylococcus infection. sharply decreased
as a result of disorder by Kupffer’s cells {=-Phagocytosis ~Pinocytosis ~Chemotaxis ~Apoptosis ~Production of cytokines} In a
dog was removed liver. After few hours it died from {=Hypoglycemic coma ~Hepatic coma ~Ketoacidotic coma
~Lactatacidotic coma}

In a man 38-year-old is observed jaundiced skin, anemia, spleen increased, hyperbilirubinemia at the expense of
unconjugated bilirubin., stercobilirubinuris, hypercholic dark feces. What state the most characterized that changes for?
{=Hemolytic jaundice ~Mechanical jaundice ~Hepatic jaundice ~Gilber’s syndrome ~Hepatic insufficiency syndrome}

A patient complains of general weakness, boring pain in the abdomen, bad appetite, suspicion on jaundice. Blood serum
contains 77.3 mcml/L of total bilirubin and 70.76 mcml/L of conjugative bilirubin. What is the most possible type of
jaundice? {=Mechanical jaundice ~Acute hepatitis ~Hepatic cirrhosis ~Parenchymatous jaundice ~Hemolytic jaundice}

Flabby contraction of gall bladder was revealed in a woman aged 55 after introducing some of vegetable oil into duodenum.
What hormone insufficiency with such state? {=Cholecystokinin ~Enterogastrin ~VIP ~Pancreozymin ~Gastrin}

A patient aged 25 has a diagnosis of chronic hepatitis. A patient has lost 10 kg of his body weight for 2 months. Objectively:
the skin is dry, desquamative, and pale with yellowish color, small punctate hemorrhages on the skin, stomatorrhagia. The
impairment of what hepatic function do petechial hemorrhage and stomatorrhagia prove? {=Albumin synthetic
~Ekromogenic ~Detoxicative ~Depositing ~Glycogen synthetic}

On examination bile congestion in the liver and cholelitiasis were revealed in a patient. Point out the main component of
cholelitiasis in this state: {=Cholesterol ~Triglycerides ~Protein ~Calcium bilirubinate ~Mineral salts}

What kind of jaundice is characterized by increased amount of direct bilirubin in the blood, appearance of bilirubin in urine,
acholic stool? {=Obstructive ~Hemolytic ~Parenchymatous ~-} Residual nitrogen and urea were determined in the patient’s
blood analysis.

The amount of urea in the residual nitrogen is considerably reduced. The disease of what organ is characterized by this
analysis? {=Liver ~Kidneys ~Stomach ~Heart ~Intestine}

In 70’s the scientists determined that the cause of severe jaundice in newborns was the impairment of connection of
bilirubin in hepatocytes. What substance is used for the formation of conjugate? {=Glucoronic acid ~Pyruvic acid ~Uric acid
~Sulphuric acid ~Lactic acid}

Aman aged 38 with ecteric skin has anemia, enlarged spleen, hyperbilirubinemia, urobilinuria, hypercholic stool. What
condition are these changes typical for? {=Suprahepatogenous jaundice ~Subhepatogenous jaundice ~Cellular-
hepatogenous jaundice ~Gilbert’s syndrome ~Syndrome of hepatic insufficiency}

It’s determined that a patient with jaundice has increased amount of total bilirubin instead of indirect one (free) in blood
plasma, high content of stercolbilin in stool and urine, the level of direct (connected) bilirubin in blood plasma is normal.
What kind of jaundice is it? {= Hemolytic ~lcterus of new borns ~Parenchymatous (hepatic) ~Mechanical ~Gilbert’s disease}
In inflammatory process colloidal properties of bile are impaired in gall bladder and this results in the formation of gall
stones. What substance crystallization is the main cause of the formation? {=Cholesterol ~Urate ~Chloride ~Oxalate
~Phosphate}

After an accident a completely crashed man’s liver was removed.what disorders do of hepatic absence may cause death
during the first hours after operation? {=Hypoglycemia ~Intoxication ~Fall of AP ~Sharp ascites ~Hemophilia and
hemorrhage}

Marked isoosmotic hyperhydration has developed in a patient with hepatocirrhosis. What is the leading mechanism of
dyshydria development? {=Secondary aldosteronism ~Growth of wall capillary permeability ~Rushyer-petrovsky reflex
~Cardio-vascular insufficiency ~Hypoproteinemia}

On the background of pain in the right hypochondrium and yellowness ‘a patient with hepatic cirrhosis has constant
dyspeptic disorders in a kind of bitter taste in the mouth, feeling of heaviness in epigatric area, nausea, unstable stool,
steatorrhea. What is the main cause of the described disorders? {=Hypocholia and intoxication ~Hypoglycemia ~Increase of
stercobilin ~Hyperbilirubinemia ~Hypoproteinemia}

There is increase of indole amount and decrease of indican 1 in the patient’s urine. This indicate the impairment of
{=Detoxication function of liver ~Filtering function of kidneys ~Reabsorptive funtion of kidney ~Albumin synthetic function
of liver ~Secretory function of pancreas}

Which of the factors plays the leading role in the development of encephalopathy in hepatic insufficiency? {=Increase of
concentration of toxic substances in the blood ~Hyperaldosteronism ~Hyperbilirubinemia ~Hypofibrinogenia
~Hypoproteinemia}

A patient admitted to the hospital has clearly marked widened subcutaneous veins in the area of umbilicus (“the head of
medusa”). Which of the large venous vessels has the impaired passage? {=V. porta ~V. renalis ~V. iliaca inferior ~V.
mesenterica superior ~V. mesenterica inferion}

In coprologic investigation it is determined that stool is colorless; there are drops of neutral fat in it. The most possible case
of this is impairment of: {=Entering the bile into intestine ~Secretion of intestine juice ~Acidity of gastric juice ~Processes of
absorption in the intestine ~Secretion of pancreatic juice}

A patient had nausea and malaise after taking fatty foodstuffs. Sings of steatorrhea developed in this patient some time
later. Content of cholesterol in patient’s blood is 9.2 mmol/L. This condition results from deficiency of: {=Bile acids
~Triglycerids ~Chylomicrones ~Phospholipids ~Lipase}

Low level of albumins and fibrinogen were revealed in the patient’s blood. The decreased activity of what hepatocyte
organells cause this phenomenon? {=Granular endoplasmic network ~Mitochondria ~Agranular endoplasmic network
~Goldgi’s complex ~Lysosoms}

A patient was admitted into a clinic with signs of acute alcohol poisoning. What changes of carbohydrate metabolism are
typical for this condition? {=The rate of gluconeogenesis decreases in the liver. “Glycogen decomposition increases in the
liver ~Aerobic decomposition of glucose increases in the muscles ~Anaerobic decomposition of glucose predominates in the
muscles. ~Gluconeogenesis increases in the liver}

A patient has a diagnosis of ascites. There are no edemas in the other parts of the body. There are large vessels of cyanotic
color on the abdomen. What pathology has this patient? {=Portal hypertension ~Hypertension of pulmonary circulation
~Essential hypertension ~Chronic circulatory insufficiency ~Hepatic hypertension}

The development of acute pancreatitis in a patient is accompanied by the impairment of permeability of common bile duct.
What pathologic process may this result in? {=Mechanical jaundice ~Parenchymatous jaundice ~Hepatic coma “Hemolytic
jaundice ~Portal hypertension}
A man aged 54 was admitted into the clinic with complaints of pains in the right hypochondrium vomiting with blood.
Objectively: enlargement of hepatic size, varicose of esophagus and stomach, bleeding from them. The functional disorder
of what vessel took place? {=Vena porta ~Vena cave superior ~Vena hepatica ~Aorta abdominalis ~Vena cava inferior}

Increase of direct and indirect bilirubin is determined in blood of a patient with marked yellowness of sclerae and skin.
There is great amount of bilirubin and urobilin in the urine, traces of stercobilin, decrease of stercobilin in stool. Define
pathogenic type of jaundice in a patient. {=Parenchymatous ~Hemolytic ~Mechanical ~By-pass ~Transmissible}

In a severe course of viral hepatitis, a patient has developed adynamia, sleepiness at day time and insomnia at night,
inadequate behavior (delirated ideas). There is fetor hepaticus, Kussmaul’s respiration. What kind of metabolism
impairment causes these symptoms? {=Nitrogenous metabolism ~Water and salt metabolism and hyperhydration ~Fat
metabolism ~Pigmental metabolism and hyperbilirubinemia ~Carbohydrate metabolism and hypoglycemia}

Arterial hypertension may develop in a case of hepatic insufficiency. Which of these factors causes the increase of arterial
pressure in such cases? {=Aldosteron ~Adrenaline ~Renin ~Noradrenalin ~Angiotensin II}

A patient was admitted to the hospital with complaints of dyspeptic disorders, melena, hemorrhoidal bleeding. Extensions
of the vessels on the anterior abdominal wall in combination with the enlargement of size of the abdomen were revealed
on examination of this patient. What pathology of gastrointestinal tract shows these symptoms? {=Portal hypertension
~Intestinal autointoxication ~Ulcerous disease ~Colitis ~Enteritis}

A patient complains of general weakness, breathlessness. Decrease of AP, ascites, widening of superficial veins of anterior
abdominal wall, splenomegaly were established in this patient. What impairment of hemodynamic has this patient?
{=Portal hypertension syndrome ~Left ventricular failure ~Right ventricular failure ~Collapse ~Arterial hypotension}

A patient aged 25 is ill with jaundice, his skin has got yellow and green color, there’s skin itch, hypocholic stool throbbing
gall bladder. What is the origin of jaundice? {=Obturative genesis ~Serum hepatitis ~Hepatic cirrhosis ~Leptospirosis ~Post-
transfusion}

A patient with jaundice complains of erythrism, headache, and insomnia. Objectively: pulse-54 beats/min. AP90/60 mmHg.
Coagulation of blood decreases. There are traces scratching on skin. The action of what component causes these
symptoms? {=Bile acid ~Cholesterol ~Bile pigment ~Bilirubin ~Fatty acid}

Yellow color of the skin and the sclera, dark urine appeared in a patient after fungus poisoning. What pigment causes the
color of urine in the patient with hemolytic jaundice? {=Stercobilin ~Verdoglobin ~Unconjugated bilirubin
~Monoglucoronide bilirubin ~Biliverdin}

A female patient with chronic hepatitis complains of increase sensitivity to barbiturates which she used previously without
any symptoms of intoxication. The disorder of what hepatic jaundice is responsible for this state? {=Metabolic
~Hemodynamic ~Bile formation ~Hemopoetic ~Phagocytotic}

Marked jaundice appeared in a patient 3 months later after the operation on his upper jaw. What pathological process may
this patient have? {=Hepatic jaundice ~Hereditary hemolytic jaundice ~Suprahepatic jaundice ~Subhapetic jaundice
~Cholecystitis}

A female patient aged 45 was admitted to a hospital with complains of sudden pain in the abdominal cavity, increase of
temperature, and leukocytosis. Which of factors caused these changes in the woman’s condition? {=Gall stones ~Bacteria
~Mechanical energy ~Acids ~Viruses}

In a 38-year-old patient, who endued viral hepatitis C and is abusing alcohol, symptoms of hepatic cirrhosis with ascites and
edemas on lower extremities developed. What changes in blood composition underlies edema development?
{=Hypoalbuminemia ~Hypoglycemia ~Hypoglobulinemia ~Hypokalemia ~Hypocholesterolemia}
A 67-year-old woman, who has cholecystitis for a long time, suddenly developed sharp pain in the upper part of abdomen,
nausea, and vomiting after food intake. Acute pancreatitis was diagnosed in this patient. What is the main link in
pathogenesis of this disease? {=Preliminary activation of pancreatic enzymes ~Decrease in enzyme levels in pancreatic
juice ~Intensification of enzyme activation in duodenum ~Reduction of pancreatic polypeptide secretion ~Increased level of
cholecystokinin}

A patient with obstructive jaundice presents with bradycardia, low arterial pressure, itching, irritability, asthenia. What is
the cause of these presentations? {=Cholemia ~Anacholia ~Hypercholesterolemia ~Hypocholesterolemia
~Hyperbilirubinemia}

A newborn born to an Rh-negative mother (3rd pregnancy) presents withprogressing jaundice, symptoms of CNS excitation,
anemia. What type of jaundice is it? {=Hemolytic ~Parenchymatous ~Obstructive ~Parasitic ~Toxic}

A patient has obstruction of the common bile duct. Which of these substances is usually found in urine in such cases?
{=Bilirubin ~Ketone bodies ~Uric acid ~Creatinine ~Glucose}

A patient with alcoholic cirrhosis complains of general weakness, dyspnea. He has been found to have decreased blood
pressure, ascites, enlargement of superficial veins of the anterior abdominal wall, esophageal varices, splenomegaly. What
hemodynamic disorder is observed in the patient? {=Portal hypertension ~Left ventricular failure ~Right ventricular failure
~Heart failure ~Collapse}

A 71-year-old woman developed mechanical jaundice due to obstruction of the bile duct with a chololith. Decrease of
blood pressure and bradycardia are detected. These changes in functioning of the patient’s cardiovascular system are
caused by increased blood content of the following substance: {=Bile acids ~Direct bilirubin ~Indirect bilirubin ~Urobilin
~Stercobilin}

During calculous cholecystitis attack the patient has developed the following symptoms: saponated feces and steatorrhea.
What stage of fats metabolism is disrupted according to those symptoms? {=Fat digestion, absorption and secretion ~Fat
absorption ~Intermediary metabolism of fats ~Fats metabolism in adipose tissue ~Depositing disruption}

A newborn infant has hemolytic jaundice caused by rhesus incompatibility. What bile pigment will be concentrated highest
in the blood of this infant? {=Unconjugated bilirubin ~Conjugated bilirubin ~Urobilinogen ~Stercobilinogen ~Bile acids}

The patient with alcoholic cirrhosis complains of general weakness and dyspnea. The following is revealed: decrease of
arterial pressure, ascites, dilation of stomach anterior wall superficial veins, esophageal varicose veins dilatation,
splenomegaly. What haemodynamics disorder does the patient uffer from? {=Portal hypertension ~Left ventricular failure
~Right ventricular failure ~Cardiac insufficiency ~Collapse}

The patient with mushroom poisoning has developed the following symptoms: yellow coloring of skin and sclera, dark-
colored urine. Hemolytic jaundice was diagnosed. What pigment causes such coloring of the patient’s urine? {=Stercobilin
~Conjugated bilirubin ~Biliverdin ~Unconjugated bilirubin ~Verdohemoglobin}

The patient has icteritous skin; unconjugated bilirubin content in blood is high; conjugated bilirubin in urine is not detected.
There is significant amount of urobilin in urine and stercobilin in feces. Name the pathology characterized by given
symptoms. {=Hemolytic jaundice ~Obstructive jaundice ~Jaundice of the newborn ~Hepatocellular jaundice
~Atherosclerosis}

A newborn child born from Rhnegative mother in the result of her third pregnancy presents with gradually worsening
jaundice, irritated central nervous system, anemia. What type of jaundice does the infant suffer from? {=Hemolytic
~Hepatocellular ~Obstructive ~Parasitic ~Toxic}

The patient with alcoholic cirrhosis complains of general weakness and dyspnea. The following is revealed: decrease of
arterial pressure, ascites, dilation of the superficial veins of the stomach anterior wall, esophageal varicose veins dilatation,
splenomegaly. What haemodynamics disorder does the patient suffer from? {=Portal hypertension ~Left ventricular failure
~Right ventricular failure ~Cardiac insufficiency ~Collapse}

Inhibition of the synthesis of bile acids from cholesterol in liver of an experimental animals has caused maldigestion of
lipids. What is the role of these acids in the enteral lipidic metabolism? {=They emulsify dietary lipids ~They keep balance of
alkaline environment in the intestines ~They participate in the synthesis of lipids ~They are part of LDL ~They activate the
formation of chylomicrons}

The patient has icteric skin; unconjugated bilirubin content in blood is high; conjugated bilirubin in urine is not detected.
There is significant amount of urobilin in urine and stercobilin in feces. Name the pathology characterized by the given
symptoms: {=Hemolytic jaundice ~Obstructive jaundice ~Jaundice of the newborn ~Hepatocellular jaundice
~Atherosclerosis}

A patient has icteric skin; unconjugated bilirubin content in blood is high; conjugated bilirubin in urine is not detected.
There is significant amount of urobilin in urine and stercobilin in feces. Name the pathology characterized by given
symptoms: {=Hemolytic jaundice ~Obstructive jaundice ~Jaundice of the newborn ~Hepatocellular jaundice
~Atherosclerosis}

A woman complains of nausea, vomiting, skin itch. She was diagnosed with mechanical jaundice. What is the possible cause
of skin itch in such a condition? {=Bile acids accumulating in the blood ~Increased blood content of indirect bilirubin
~Cholesterol accumulating in the blood ~Direct bilirubin appearing in the blood ~Erythrocyte disintegration products
accumulating in the blood}

A patient presents with icteric sclera and mucous tunics; urine is dark; feces are light-colored. Blood content of direct and
indirect bilirubin is increased, urine content of direct bilirubin is increased. What pathology can be characterized by these
signs? {=Obstructive jaundice ~Hemolytic jaundice ~Hepatocellular jaundice ~Jaundice of the newborn ~Atherosclerosis}

A patient has a gallstone lodged in the common bile duct, which blocks bile supply to the intestine. What digestive process
will be disturbed in this case? {=Fat digestion ~Protein absorption ~Carbohydrate digestion ~Carbohydrate absorption
~Protein digestion}

On examination the patient’s sclera and oral mucosa are icteric. What biochemical blood value can be expected to be
increased? {=Bilirubin ~Amylase ~Glucose ~Albumin}

On the seventh day after hemorrhage caused by a trauma, the following was revealed in a patient’s blood:
erythrocytes - 2.810!7/1, Hb - 3.9mmol/, reticulocytes - 15%, acidophilic and polychromatophilic normoblasts were
found in smear. What is the mechanism of appearance of regenerative forms of blood erythrocytes?
{=Intensification of regeneration of erythroid cells in bone marrow
~Increase permeability of erythrocyte sprout of bone marrow
~Going out of blood deposition
~Inhibition of maturation of erythroid cells in bone marrow ~Inhibition
of synthesis of erythroprotein inhibitor}

Singular oxyphilic normoblasts appeared in the blood of a patient after acute post-traumatic hemorrhage
composing 15% of blood volume. On supravital staining 25% of reticulocytes were found. What is the patient
anemia according to its ability of regeneration?
{=Hy perregenerative
~Regenerative
~Hyporegenerative
~Aregenerative
~Hypo- and aregenerative}

What index of blood analysis is the most typical for beta-thalassemia?


{=Increase of fetal hemoglobin
~Erythrocytes with basophilic stippling
~Considerable decrease of erythrocytes and hemoglobin
~Target-like erythrocytes
~Increase of met-hemoglobin}

While studying a blood smear different forms of erythrocytes were found. Which of them are regenerative?
{=Ploychromatophils
~Poikilocytes
~Anisocytes
~Ovalocytes
~Megalocytes}

A victim of a car accident has lost much blood. What impairment of general blood volume takes place?
{=Polycystehmic hypovolemia
~Polycythemic normovolemia
~Olygocysthemic normovolemia
~Simple hypovolemia
~Oligocysthemis hypovolemia}

During the examination of adolescents that reside in mounting region increase in level of erythrocytes and
hemoglobin in peripheral blood was found out. What is the reason for indicated erythrocytosis? {=Exogenous
hypoxia
~Diseases of lungs
~Congenital heart disease
~Condensation of blood due to large loss of water
~Vakes’s disease}

What kind of disorders of total blood volume appears in case of absolute erythrocytosis? {=Polycytemic
hypervolemia
~Oligocytemic hypervolemia
~Oligocytemic hypovolemia
~Simple hypervolemia
~Simple hypovolemia}
A patient had anemia due to profuse blood loss. What blood changes are typical at the beginning of development of
acute post-hemorrhagic anemia?
{=Normochromia
~Absence of reticulocytes
~Poikilocytosis, anisocytosis
~Hyperchromia
~Presence of megalocytes in the blood}
In a woman hypochromic anemia was diagnosed after delivery accompanied by marked bleeding. What pathologic
forms of erythrocytes are characteristic ones for this type of anemia?
{=Anisocytes
~Target-like erythrocytes
~Sickle-cell anemia
~Spherocytes
~Microcytes}

A patient aged 32 with massive hemorrhage due to car accident trauma was admitted to the hospital. Pluse-100
beats per min, respiratory rate-22 per 2 min, BP-100/60 mmHg. What blood change will be the most characteristic
in an hour after hemorrhage?
{=Hypovolemia
~Hypoproteinemia
~Erythropoenia
~Leucopoenia
~Erythrocyte hypochromia}

The experiment was carried out on a rabbit. The increase of the number of erythrocytes and hemoglobin in the
blood due to the stimulation of erythropoiesis by erythropoietin was determined 2 weeks later after the narrowing
of renal artery. What increases the formation of erythropoietin?
{=Hypoxemia
~Hypercapnia ~Hypoosmia.
~Hyperosmia
~Hypovolemia}

Hypochromic anemia was diagnosed in a patient on the 7th day after acute hemorrhage. What mechanism is the
leading one in its development?
{=Increased penetration of immature erythrocytes from bone marrow
~Impairment of globin synthesis
~Increased erythrocyte destruction in the spleen
~Impairment of iron absorption in the intestine
~Increase of iron excretion from the organism}

Posthemorrhagic anemia has developed in a patient with periodical bleeding due to uterus fribromyoma. What type
of chronic post hemorrhagic anemia takes place in this case?
{=Erythroblastic, hypochronic, hyporegenerative
~Megaloblastic, hypochromic, hyporegenerative
~Erythroblastic, hyperchromic, hyporegenerative
~Erythroblastic, hypochromic, hyperregenerative
~Megaloblastic, hyperchromic, hyperegenerative}

A victim lost about 20 % of blood during a car accident. For what time the volume of circulatory liquid will be
restored in the circulatory stream?
{=For | day
~For 3 days
~For 5 days
~For 7 days
~For 9 days}

In anemia the degenerative and regenerative forms of erythrocytes are determined in peripheral blood. Name the
regenerative forms of erythrocytes. {=Polychromatophilic erythrocytes
~Microcytes
~Spherocytes
~Poikilocytes
~Hyperchromic erythrocytes}

Indicate, what disorder of general blood volume does develop in the case of shock, when blood is deposited in the
dilated vessels of abdominal cavity.
{=Polycythemic hypovolemia
~Oligocythemic hypovolemia
~Polycythemic hypervolemia
~Oligocythemic hypervolemia
~Simple hypovolemia}

Physical and chemical properties of blood change in case of polycythemic hypervolemia (Vakez illness), namely:
{=Increase of blood viscosity
~Increase of ESR
~Decrease of blood coagulation
~Decrease of blood density
~Decrease of plasma density}

The compensatory reaction of the endocrine system at case of blood loss is appearence of hyperproduction
{=Vasopressin
~Thyroxine
~Endorphin
~Calcitonin
~Parathyroid hormone}

The irrepressible vomiting in pregnant leads to development of hypovolemia due to considerable dehydration. The
main endocrine mechanism of compensation at this state is
{=Increase of aldosterone production
~Increase of adrenaline secretion
~Increase of corticotropin synthesis
~Increase of prolactin secretion
~Increase of calcitonin production}

The mechanism of compensatory action of aldosterone after blood loss is


{=Increase of sodium reabsorption
~Decrease of sodium reabsorption
~Increase of potassium reabsorption
~Increase of calcium reabsorption
~Decrease of calcium reabsorption}
The mechanism of compensatory action of vasopressin after blood loss consists of
{=Increase of water reabsorption
~Decrease of water reabsorption
~Increase of sodium reabsorption
~Decrease of sodium reabsorption
~Increase of proteins reabsorption}

The spasm of peripheral vessels in reply to blood loss directs on


{=Increase of peripheral resistance
~Go out of blood from depot
~Increase of blood coagulation
~Renewal of circulatory blood volume
~Renewal of electrolyte blood composition}

To the remote mechanisms of compensation at case of blood loss belongs


{=Increase of blood formation
~Spasm of peripheral vessels
~Tachycardia
~Dyspnea
~Migration of erythrocytes from depot}

What condition may leads to appearance of absolute erythrocytosis?


{=Cardiac insufficiency
~Liver insufficiency
~Kidney insufficiency
~Removal of stomach
~Jonizing irradiation}

What does mean erythrocytosis in a sick with heart congenital defect?


{=Compensatory reaction
~Independent disease
~Complication
~Terminal condition
~Index of convalescence}

What from enumerated mechanisms of compensation at case of acute blood loss is delayed?
{=Activation of hematopoiesis in bone marrow
~Increase of blood coagulation
~Increase and strengthening of heart contraction
~Spasm of peripheral vessels
~Migration of intercellular liquid into vessels}

Blood count of an athlete is as follows: erythrocytes - 5,5 - 1012/1, Hb- 180 g/l, leukocytes - 7- 109/1, neutrophils -
64%, basophils - 0,5%, eosinophils - 0,5%, monocytes - 8%, lymphocytes - 27%. First of all, such results indicate
the stimulation of: {=Erythropoiesis
~Leukopoiesis
~Lymphopoiesis
~Granulocytopoiesis
~Immunogenesis}

What from enumerated mechanisms of compensation at case of acute blood loss is delayed? {=Increase
of proteins synthesis in liver
~Increase of blood coagulation
~Increase of arterial pressure
~Spasm of peripheral vessels
~Increase of breathing rate}

What is the reason of simple hypervolemia in case of increased physical work?


{=Draw of blood from depot
~Draw of tissue liquid into vessels
~Increase of stroke volume
~Increase of cardiac output
~Increase of blood circulation speed}

On the fifth day after the acute blood loss a patient has been diagnosed with hypochromic anemia. What is the main
mechanism of hypochromia development?
{=Release of immature red blood cells from the bone marrow
~Impaired iron absorption in the intestines
~Increased destruction of red blood cells in the spleen
~Impaired globin synthesis
~Increased excretion of body iron}

In acar accident a man got injured and lost a lot of blood. What changes in peripheral blood are most likely to occur
on the 2nd day after the injury?
{=Erythropenia
~Hypochromia
~Anisocytosis
~Microplania
~Significant reticulocytosis}

After the prolonged vomiting a pregnant 26-year-old woman was found to have the reduced volume of circulating
blood. What change in the total blood volume can be the case?
{=Polycythemic hypovolemia
~Simple hypovolemia
~Oligocythemic hypovolemia
~Polycythemic hypervolemia
~Oligocythemic hypervolemia}

A 30-year-old patient’s blood test revealed the following: erythrocyte count is 6 - 1012/, hemoglobin is
10,55mmonp/l. Vaquez’s disease was diagnosed. Name the leading part of pathogenesis:
{=Neoplastic erythroid hyperplasia
~Iron-deficiency
~B12-deficiency
~Hypoxia
~Acidosis}

10 minutes after the beginning of heavy physical work a person demonstrates increase of erythrocyte number in
blood from 4, 0 x 1012/1 to 4, 5 x 1012/1. What is the cause of this phenomenon? {=Suppression of erythrocyte
destruction
~Increase of cardiac output
~Water loss
~Erythrocytes exit from depot
~Erythropoiesis activation}
A 26 year old pregnant woman is under treatment at an in-patient hospital. After a continuous attack of vomiting
she was found to have reduced volume of circulating blood. What kind of change in general blood volume is the
case?
{=Polycythemic hypovolemia
~Simple hypovolemia
~Oligocythemic hypovolemia
~Polycythemic hypervolemia
~Oligocythemic hypervolemia}

A 27-year-old patient with injury to the neck has lost approximately 30% of the blood volume. The patient’s
condition is severe: blood pressure is 60/40 mm Hg, heart rate is 140/min., respiratory rate is 30/min., conscious.
Characterize the condition of the patient’s circulatory system:
{=Hypovolemic shock
~Cardiogenic shock
~Collapse
~Coma
~Arterial hypertension}
Anemia, leuko- and thrombocytopenia, color index-1.3, presence of megaloblasts and megalocytes were
determined in the laboratory analysis of blood of a patient a year later after he was operated on for subtotal
resection of the stomach for the ulcer of lesser curvature of the stomach. What factor deficiency results in these
changes?
{=Gastromucoprotein
~Gastrin
~Pepsin
~Chlorine hydrate
~Mucin}

Amino acids replacement in alpha and beta chains of hemoglobin takes place in a number of hemoglobimopathies.
Which of them is typical for HbS (sickle-cell anemia)?
{=Glutamate to valine
~Aspartate to lysine
~Methionine to histidine
~Glycine to serine
~Alanine to serine}

Hereditary microspherocytic hemolytic anemia (Mincovsky-Shoffar disease) was diagnosed in a woman aged 34.
What mechanism caused hemolysis of erythrocytes in the patient?
{=Membranopathy
~Hemoglobinopathy ~Autoimmune
impairment ~Enzymopathy.
~Hypoplasia of bone marrow}

Megaloblastic anemia was diagnosed in a patient. What substance deficiency may cause the development of this
disease?
{=Cyanocobalamin
~Cholecalciferol
~Magnesium
~Glycine
~Copper}

Funicular myelosis and hyperchromic anemia developed in a man 7 years later after the stomach resection due to
ulcer. What pathogenic mechanism of changes in spinal cord is the most possible one?
{=Accumulation of methylmalonic acid in cyanocobalamin deficiency
~Hypoxia impairment in anemia
~Impairment of DNA in cyanocobalamin deficiency
~Deficiency of folic acid
~Deficiency of iron containing enzyme}

A patient, carrier of hereditary sickle-cell erythrocytes anomaly, had pneumonia accompanied by hemolytic crisis
and development of anemia. What is the main cause of hemolytic crisis in this case?
{=Hypoxia caused by pneumonia
~Heterozygosis HbS ~Mutation
of structural gene ~Hyperoxia.
~Blood osmolarity change}
Three years ago a man aged 45 was operated on for stomach resection. After the operation the content of
erythrocytes in the blood is 2.0x10'*, Hb 85 g/l, color index-1.27. What vitamin absorption is impaired that causes
the change of erythropoiesis?
{=Bi2
~P
~A
~Bo
~C}

Examining the oral cavity of a patient, a dentist paid attention to the presence of inflammatory -dystrophy process
in the mucous membrane (Gunter’s glossitis, atrophic stomatitis). Blood analysis revealed hyperchromic anemia.
What factor is a cause of this disease?
{=Hypovitaminosis B12
~Hypovitaminosis A
~Increase of stomach juice acidity
~Hypovitaminosis By
~Hypovitaminosis Be}

A female patient complains of malaise, weakness, breathlessness, rapid fatigability, and dizziness. Her blood test
data: erythrocy tes-1.8x10!7/L, Hb-80 g/L, leukocytes-3.2x10°/L, color index-1.5. Anisocytosis, poikilocytosis,
megaloblasts, megalocytes were found in smear. What is the possible diagnosis?
{=B12-deficiency anemia
~Posthemorragic anemia
~Acute leukemia
~Iron deficiency anemia
~Immunohemolytic anemia}

A patient, aged 50, complains of a bad appetite, loss of weight, weakness, pain in the stomach area, and eructation.
At laboratory examination of him: Hb-2x10!7/L, stomach secretion 0.4 1, pH of stomach juice-7. Pernicious anemia
is diagnosed in this patient. What compound of gastric juice deficiency is the cause of the disease? {=Castle’s
factor
~Renin
~Secretin
~HCL
~Pepsin}

Two years ago a patient was operated on for resection of pyloric portion of the stomach. There is weakness,
periodical appearance of dark circles under eyes, breathlessness in this patient. In the blood analysis: HB-70g/L,
erythrocytes-3.3x10'7/L, color index-0.7. What erythrocyte changes in the blood smear are the most typical for
the given state? {=Microcytes
~Megalocytes
~Schizocytes
~Ovalocytes
~Macrocytes}

When treating a patient for malaria with primachine, hemolysis occurs in him. At examination deficiency of
Glucose-6-phosphate Dehydrogenase in patient’s erythrocytes was revealed. What metabolic process is impaired in
this patient?
{=Pentose-phosphate pathway
~Glycogenesis
~Glycolysis
~Gluconeogenesis
~Aerobic carbohydrate oxidation}

Bi2-folate deficient anemia was diagnosed in a patient aged 55. What hematological index will be the most
sensitive for pathogenetic treatment? {=Reduction of anisocytosis
~Increase of number of reticulocytes
~Increase of number of erythrocytes
~Decrease of ESC
~Decrease of amount of hemoglobin}

In a pregnant woman, in the 7" month of pregnancy, anemia begins rising fast. In her blood test: number of
erythrocytes is 2.7x10!7/L, amount of hemoglobin is 90 g/L, anisocy tosis, poicilocytosis, megaloblasts and
megalocytes were found, reticulocytes of 0%. What sort of anemia develops in this case?
{=B12-deficiency anemia
~Thalassemia
~Post-hemorrhagic anemia
~Iron-deficiency anemia
~Hemolytic anemia}

Biz folic deficiency anemia developed in a patient after stomach resection. What color index is typical for this
disease? {=1.30
~1.15
~1.0
~0.85
~0.70}

Predomination of erythroblasts, normoblasts and megaloblasts was revealed in blood analysis of a patient with
anemia. The same cells were found in bone marrow. What type of anemia do these changes characteristic for?
{=B)2-folate deficiency anemia
~Hemolytic
~Aplastic
~Post-hemorrhagic
~Iron-deficiency anemia}

Erythropenia, hyperchromia, normocytes, macrocytes, megalocytes, poikilocytosis were found out in a patient’s
blood at examination. What is the cause of this pathology?
{=Deficiency of gastromucoprotein
~Ascariasis
~Tron deficiency in food
~Trichocephaliasis
~Fequent loss of blood}
In an infant, who is under an artificial nutrition with cow milk, severe anemia has developed. At the blood count of
the infant: number of erythrocytes is 4x10!7/L, content of hemoglobin is 68 g/L, reticulocytes of 0%. What kind of
anemia developed in the infant?
{=Iron-deficiency anemia
~Inborm hemolytic anemia
~By2-deficiency anemia
~Hypopastic anemia
~Sickle-cell anemia}

Hemolytic anemia with decrease of osmotic erythrocyte resistance that averaged 0.6-0.5 was revealed in a smear of
venous blood during microscopic examination. What substance accumulation in the blood plasma may also
indicate the development oh hemolytic anemia?
{=Indirect bilirubin
~Urea
~Creatinine
~Lactic acid
~Inorganic phosphate}

Hypochromia of erythrocytes, micro-, anisocytosis, poikilocytosis, are determined in the blood in case of
development of iron deficiency and iron refractory anemia. What index must be determined to carry out differential
diagnosis of these anemias?
{=Serum iron
~Serum phosphorous
~Serum magnesium
~Serum calcium
~Serum chlorine}

Content of hemoglobin and number of erythrocytes significantly decreases in a patient’s blood from time to time. It
was found out that such attacks appear after taking some horse beans. What kind of anemia takes place in this
case? {=Enzymopathy
~Membranopathy
~Iron deficiency anemia
~Hemoglobinopathy
~Acquired hemolytic anemia}

Beta-thalassemia was revealed in a patient, who came from Tunis. The disease was accompanied by hemolysis and
jaundice. The disease was diagnosed on the base of presence in blood:
{=Target-like erythrocytes
~Grained erythrocytes
~Polychromatophil erythrocytes
~Normocytes
~Reticulocytes}

What is the cause of intracellular hemolysis?


{=Genetic infringements of erythrocytes
~Malaria
~Action of hemolytic poison
~Infection with hemolytic streptococcus
~Transfusion of incompatible blood}
Biz deficiency anemia was diagnosed in a man aged 57 after examination of him. Treatment was administered to
this patient. Control blood test in this patient was performed in 3 years. What is the most adequate criterion for
enhancement of erythropoiesis? {=Increase in number of reticulocytes
~Increase in hemoglobin content
~Decrease in color index
~Normoblastic hematopoiesis
~Increase in number of leukocytes}

Patient with chronic hypoacidic gastritis has hypochromic anemia. In the blood smear of this patient agranulocytes,
micro- and anisocytosis, and poikilocytosis are revealed. Name this anemia.
{=Iron deficiency anemia
~Acute post-hemorrhagic anemia
~Thalassemia
~Sickle-cell anemia
~Pernitious anemia}

In a patient aged 35 autoimmune hemolytic anemia developed. What index of blood serum is the most increased in
this case?
{=Mesobilinogen
~Stercobilinogen
~Direct bilirubin
~Indirect bilirubin
~Protoporphirin}

A patient aged 43 has stomatitis, glossitis, the tongue is crimson colour, smooth. Blood analysis revealed: Hb 100
g/l; erythrocytes 2.3x10'°/1; color index 1.30. What is the patient’s state due to?
{=Vitamin B12 deficiency
~Hypoplasia of red bone marrow
~Impairment of porphyrin synthesis
~Iron deficiency
~Erythrocytes haemolysis}

After resection of small intestine a patient complained of increased fatigability, infringement of taste, brittleness of
nails, quick decay of dental enamel, and appearance of breathlessness on physical exertion. Which of the below
given substances is the source of impairments in the patient’s organism?
{=Iron deficiency anemia
~Hypocalcaemia
~Vitamin D deficiency
~Hyponatriaemia
~B12-folic deficiency anemia}

A female patient with impairment of menstrual cycle accompanied by prolonged bleeding the blood analysis was
made which revealed hypochromia, decrease of reticulocytes, microcytosis. What group does this anemia belong to
according to pathogenesis? {=Iron deficiency anemia
~B12 folic deficiency anemia
~Hypoplastic anemia
~Hemolytic anemia}
In blood analysis of 37-years-old woman following data were revealed: content of hemoglobin is 60 g/L, number
of erythrocytes is 3.0x10'/L, and color index of 0.6; differential count of leukocytes without any changes; number
of platelets is 200x10°/L; reticulocytes count of 0.3%; ESR of 18 mm/hour,; microcytosis and poikilocytosis of
erythrocytes. Indicate the most probable type of anemia according to mechanisms of its development.
{=Hypoplastic anemia
~Hemolytic anemia
~Acute post-hemorrhagic anemia
~Iron deficiency anemia
~B12-folate deficiency anemia}

A patient aged 20, has periodically yellowness of sclera and skin that is accompanied by weakness.
MinkovskiShoffar disease is diagnosed. What is the most typical for blood picture in this disease? {=Micro
spherocytosis
~Agranulocytosis ~Macrocytosis
~Reticulositosis.
~Thrombocytosis}

Hypochromic anemia was found out in a patient aged 54, who had a prolonged contact with lead at his work.
Treatment with iron preparations for a month didn’t give any effect. The increased amount of iron was determined
in blood serum. What is this anemia due to?
{=Hypoplasia of red bone marrow
~Porphirin synthesis impairment
~Folic acid deficiency
~Vitamin B12 deficiency
~Erythrocyte hemolysis}

Evaluate the blood analysis: erythrocytes 3.10; Hb 90g\L reticulocytes 0.5%. In the smear there are poikilocytes
hypochromatic erythrocytes. Blood serum iron is 80 micromol/l. What pathology is this typical for? {=Iron
refractory anemia
~Iron deficiency anemia
~Bi2 deficiency
~Sickle cell anemia
~Minkovsky-Shoffer disease}

Excessive flow of estrogens into the blood due to follicle persistence (a state when follicle does not reach complete
maturation and ovulation does not take place) often cause uterine bleeding. What anemia may develop in this case?
{=Iron deficiency
~Sideroachrestic
~Sickle cell
~Hypoplastic
~Metaplastic}

Which of the below named anemias relevant to hemoglobinopathies?


{=Thalassemia
~Iron deficiency anemia
~Bi2 deficiency
~Minkovsky-Shoffar disease
~Iron refractory anemia}
A 37-year-old female patient complains of headache, vertigo, troubled sleep, numbness of limbs. For the last 6
years she has been working at the gas-discharge lamp-producing factory in the lead-processing shop. Blood test
findings: low hemoglobin and RBC level, serum iron concentration exceeds the norm by several times. Specify the
type of anemia:
{=Iron refractory anemia
~Iron-deficiency anemia
~Minkowsky-Shauffard disease
~Hypoplastic anemia
~Metaplastic anemia}

After an extended treatment with sulfanamides a patient has developed macrocytic anemia. Production of active
forms of the following vitamin is disrupted in such a condition:
{=Folic acid
~Riboflavin
~Cyanocobalamin
~Pyridoxine
~Thiamine}

Along with normal hemoglobin types there can be pathological ones in the organism of an adult. Name one of
them: {=HbS
~HbF
~HbAi
~HbA2
~HbO?}

A patient is diagnosed with chronic atrophic gastritis attended by deficiency of Castle’s intrinsic factor. What type
of anemia does the patient have?
{=B12-deficiency anemia
~Iron refractory anemia
~Hemolytic anemia
~Iron-deficiency anemia
~Protein-deficiency anemia}

Biochemical analysis of an infant’s erythrocytes revealed evident glutathione peroxidase deficiency and
low concentration of reduced glutathione. What pathological condition can develop in this infant?
{=Hemolytic anemia
~Pericious anemia
~Megaloblastic anemia
~Sicklemia
~Iron-deficiency anemia}

Along with normal hemoglobin types there can be pathological ones in the organism of an adult. Name one of
them: {=HbF
~HbA2
~HbO2
~HbS
~HbA1}
A patient is diagnosed with irondeficiency sideroachrestic anemia, progression of which is characterised by skin
hyperpigmentation, pigmentary cirrhosis, heart and pancreas affection. Iron level in the blood serum is increased.
What disorder of iron metabolism causes this disease?
{=Failure to assimilate iron leading to iron accumulation in tissues
~Excessive iron intake with food
~Disorder of iron absorption in bowels
~Increased iron assimilation by body
}
Examination of a 52-year-old woman has revealed a decrease in the amount of red blood cells and an increase in
free hemoglobin in the blood plasma (hemoglobinemia). Color index is 0,85. What type of anemia is being
observed in the patient? {=Acquired hemolytic
~Acute hemorrhagic
~Anemia due to diminished erythropoiesis
~Chronic hemorrhagic
~Hereditary hemolytic}

A 37-year-old woman complains of headache, vertigo, troubled sleep, numbness of limbs. For the last 6 years she
has been working at a gas-discharge lamp-producing factory in a lead-processing shop. Blood test findings: low
hemoglobin and RBC level, serum iron concentration exceeds the norm by several times. Specify the type of
anemia:
{=Iron refractory anemia
~Iron-deficiency anemia
~Minkowsky-Shauffard disease
~Hypoplastic anemia
~Metaplastic anemia}

A patient addressed a dentist with complaints of affections of mucous membrane of his mouth. During the
examination of the patient ulcerous stomatitis with necrosis in center was revealed at him in area of his palate. In the
history of disease of the patient recently endued pneumonia and taking of medicines (sulfonilamides) were present.
After administering treatment the doctor pointed the patient for blood analysis. What pathology from below
mentioned does doctor suppose?
{=Immune agranulocytosis
~Iron deficiency anemia
~Thrombocytopoenia
~Infectious mononucleosis
~Infectious lymphocytosis}

Myocardial infarction was diagnosed in 65-years-old man. Neutrophilic leukocytosis with left shift is present in the
blood of this patient. What factors underlie this phenomenon?
{=Products of tissue decay
~Elevation of mass of muscular fibers
~Disorders of alveolar ventilation
~Decrease in glycogen content in the myocardium
~Increase of arterial pressure}

Increased concentration of leukopoietins in blood was found in a patient with acute appendicitis. What kind of
lekocytosis occurs in these conditions?
{=Neutrophilic
~Basophilic
~Eosinophilic
~Lymphocytosis
~Monocytosis}

A patient has deficiency of cyancobalamine and folic acid that leads to disorder of leucopoiesis. What changes takes
place in these conditions?
{=Leucopoenia
~Eosinophilia
~Basophilia
~Hemophilia
~Hyperemia}

A patient, who was exposed to ionizing radiation, has panmyelophtisis and secondary infections. What changes occur
in blood analysis in this case?
{=Agranulocytosis
~Leucocytosis
~Eosinophilia
~Basophilia
~Hyperemia}

Patient with chronic leukemia has sharply increased temperature, breathlessness, marked muscular weakness at
insignificant physical exertion, increased sweating, cough. What mechanism of leukemia influence upon organism
underlies complications in this patient?
{=Immunodeficiency due to functional inability of leukocytes
~Internal bleeding because metastases into vessel wall
~Anemia
~Tumor progression
~Airway obstruction because of development of metastases}

A patient with leukemia has general number of leukocytes of 120.0x10°/L. What kind of leukemia does this patient
have? {=Leukemic
~Leucopenic
~Subleukemic
~Aleukemic
~Erythremia}

When examining a blood in a patient, who endued bleeding three days ago, following data was revealed: number of
leukocytes is 12x10°/L, basophils count is 0%, eosinophils count is 3%, myelocytes count 0%, juvenile neutrophils
count is 3%, stab neutrophils count is 12%, segmented neutrophils count is 62%, lymphocytes count 16%, and
monocytes count is 4%. What kind of changes of leukocyte differential count takes place in this case?
{=Neutrophilia with regenerative shift to the left
~Neutrophilia with degenerative shift to the left
~Neutrophilia with shift to the right
~Absolute lymphopoenia
~Absolute monocytopoenia}

A 40-years-old patient, who was admitted to the surgical department with diagnosis of phlegmona of thigh, had
high temperature, tachycardia, and breathlessness. On the blood test of this patient: number of leukocytes is
25x10°/L; eosinophils count is 1%, myelocytes count is 1%, juvenile neutrophils count is 15%, band neutrophils
count is 25%, segmented neutrophils count is 40%, lymphocytes count 14%, monocytes count is 4%. What kind of
shift in differential count is present in this case?
{=Hy perregenerative
~Regenerative
~Degenerative
~Regenerative-degenerative
~Leukemoid}

Acute inflammatory disease of upper airways and eyes appears in 45-years-old woman at the period of blossom of
grass. Symptoms of this disease are hyperemia, edema, and mucous secretions. What kind of leukocytosis is the
most characteristic one for this disease?
{=Eosinophilia
~Monocytosis
~Neutrophilia
~Basophilia
~Lymphocytosis}

A women fell ill with purulent stomatitis. What index of complete blood count is characteristic for this disease?
{=Leukocytosis
~Thrombocytosis
~Lymphocytosis
~Anemia
~Monocytosis}

During the development of acute pulpitis a patient complained of paroxysm of pain in the upper jaw, which is
increasing at night, fever. At examination leucocytosis was established in the blood. What kind of leucocytosis
is possible in this case? {=Neutrophilic leuccytosis ~Lymphocytosis
~Eosinophilic leucocytosis
~Monocytosis
~Basophilic leucocytosis}
Considerable increase of the number of eosinophils in a unit of blood volume was determined during the
examination of a 5 years old boy. What may cause eosinophilia in this patient? {=Helminthic invasion
~Obesity
~Hypodynamia
~Hypothermia
~Physical exertion}

In the patient’s blood analysis the number of leukocytes is 250*10°/L. What syndrome does this patient have?
{=Leukemia
~Leucocytosis
~Leucopoenia
~Leucomoid reaction
~Hyperleucocytosis}

Leucocytosis was found out in a person who didn’t complain of his health. The cause of this may be that fact that the
blood was taken for analysis after:
{=Physical load
~Mental work
~Rest at a health resort
~Considerable use of water
~Usage of alcohol}

Lymphocytosis was revealed in a patient. What diseases may be accompanied by lymphocytosis? {=Pertussis,
chicken pox
~Helminthic invasion
~Somatotropin insufficiency
~Sepsis
~Bronchial asthma}

What conditions are accompanied by neutrophilic leukocytosis with shift of differential count to the left? {=Purulent
inflammation
~Tuberculosis
~Infections mononucleiosis
~Agranulocytosis
~Alimentaray leukocytosis}

What blood pathology is the presence of Philadelphia chromosomes in the blood cells and bone marrow cells typical
for?
{=Chronic myelogenous leukemia
~Acute myelogenous leukemia
~Hodgkin’s disease ~Burkitt’s
lymphoma
~Chronic lymphocyte leukemia}

Neutropoenia is found out in a patient who has the manifestations of immunodeficiency. What diseases
may neutropoenia be determined in? {=Fuzarium fungus poisoning
~After profuse haemorrhage
~Myeloleukemia
~Insufficiency of sexual gland function
~Septic process}
Patient M, aged 20 was admitted to the hospital complaining of high temperature, pain in the bones, and
hemorrhage from his gums. Blood analysis of this patient shows: erythrocytes-2.5x10'7/L; Hb-80¢/L;
leucocytes2.0x10°/L; thrombocytes-6.0x10°/L; differential count: eosinophils-1%; stab neutrophils-1%; segmented
neutrophils-10%; lymphocytes-10%; monocytes-3%; blast cells-75%. What pathology is this blood analysis typical
for?
{=Acute leukemia
~Burkitt’s lymphoma
~Hodgkin’s disease
~Infections mononucleosis
~Chronic leukemia}

General amount of leucocytes is 90x10°/1. In differential count: eosinophils-1%; basophils-0%; juvenile


neutrophils-0%; stab neutrophils-2%; segmented neutrophils-20%; prolymphocytes-2%; lymphocytes-70%;
Botkin-Gumprecht cells. Cervical, submandibular lymph nodes are enlarged. What pathology is such blood picture
typical for?
{=Chronic lymphocytic leukemia
~Hodgkin’s disease
~Infectious mononucleosis
~Acute lymphoblastic leukemia
~Chronic myelogenous leukemia}

Increase of the number of eosinophils was determined in a patient with endocrine pathology during examination.
Which of the named diseases may be accompanied by cosinophilia?
{=Addison’s disease
~Conn’s disease ~Acromegaly
~Cushing disease
~Pheochromocytoma}

Functional insufficiency of monocytes is accompanied by immunodeficiency. What BAS produced by monocytes


stimulate specific response?
{=Interleukin-1
~Lysocime
~Interleukin-2
~Fibronectin
~Myeloperoxidase}

Relative neutropoenia with degenerative shift was revealed in the blood of a patient with TB during examination.
What change of differential blood count corresponds to this state?
{=Decrease of the number of segmented forms and increase of band forms of neutrophils
~Decrease of lymphocyte number
~Increase of monocyte number
~Increase of basophil number
~Decrease of eosinophil number}

Neutrophil leukocytosis was revealed in worker during examination. What pathologic condition may neutrophil
leukocytosis result from? {=Septic condition
~Chronic loss of blood
~Viral infection
~Radiation sickness
~Benzene poisoning}
Patient R. aged 12 was operated on for acute phlegmonous appendicitis. At the examination of his blood, the
amount of leukocytes is 12x10°/L. On the blood smear there are: basophils — 0%; eosinophils — 2%; monocytes —
2%; juvenile neutrophils — 0%; stab neutrophils — 30%; segmented neutrophils — 43%; lymphocytes — 23%;
monocytes — 0%, Stab (immature) neutrophils have pyknosis of nuclei. There is anisocytosis of neutrophils; some
of them have toxic granulation. What form of change of leukocyte blood composition takes place in this case?
{=Leukemoid reaction of neutrophil type
~Neutrophilia with regenerative shift to the left
~Neutrophilia with hyperegenerative shift to the left
~Neutrophilia with degenerative shift to the left
~Neutrophilia with the shift to the right}

Which of the below given hematological characteristics corresponds to chronic post-hemorrhagic anemia?
{=Hyporegenerative, hypochromic with erythroblasts type of bleeding
~Regenerative, hypochromic with erythroblasts type of bleeding
~Hyporegenerative, hypochromic with megaloblasts type of bleeding
~Hyporegenarative, hyperchromic with megaloblasts type of bleeding
~Hyperregenerative, hypochromic with erythroblasts type of bleeding}

Hemiparesis appeared in a patient with acute promyelocytic leukemia. What is the main mechanism of the
impairment of CNS in this case? {=Formation of leukemic infiltrates
~Intoxication by leukemic cells decay products
~Impairment of desintoxicative function of the liver
~Cachexia
~Increase of thrombogenesis}

Acute pain in the lower jaw, swelling of the cheek, temperature of 37.60C occurred in the patient who had had
dental caries for some years. What changes in the patient’s blood may be observed in this case?
{=Neutrophilic leukocytosis
~Leucopoenia
~Moncytosis
~Anemia
~Eosinophilia}

Acute inflammatory disease of upper airways and eyes appears in 45-years-old woman at the period of blossom of
grass. Symptoms of this disease are hyperemia, edema, and mucous secretions. What kind of leukocytosis is the
most characteristic one for this disease?
{=Eosinophilia
~Monocytosis
~Neutrophilia
~Basophilia
~Lymphocytosis}

Microscopical examination of an enlarged cervical lymph node revealed blurring of its structure, absence of
lymphoid follicles; all the microscopic fi- elds showed cells with roundish nuclei and thin limbus of basophil
cytoplasm. It is known from the clinical data that other groups of lymph nodes are also enlarged as well as spleen
and liver. What disease might be suspected?
{=Lymphoid leukosis
~Lymphogranulomatosis
~Lymphosarcoma
~Myeloid leukosis
~Multiple myeloma}
A 26-year-old man is in the torpid shock phase as a result of a car accident. In blood: 3, 2 - 109/1. What is the leading
mechanism of leukopenia development?
{=Redistribution of leukocytes in bloodstream
~Leikopoiesis inhibition
~Disturbed going out of mature leukocytes from the marrow into the blood
~Lysis of leukocytes in the blood-forming organs
~Intensified elimination of leukocytes from the organism}

Microscopic examination of the enlarged neck gland of a 14 year old girl revealed destruction of the tissue
structure of the node, absence of the lymph follicles, sclerotic areas and necrosis foci, cell constitution of the node
is polymorphous, lymphocites, eosinophiles, big atypical cells with multilobular nuclei (Beresovsky -Sternberg
cells) and mononuclear cells of the large size are present. What is the most likely diagnosis?
{=Lymphogranulomatosis
~Acute lympholeucosis
~Chronic lympholeucosis
~Berkitt’s lymphoma
~Fungoid mycosis}

A 62 year old woman complains of frequent pain attacks in the area of her chest and backbone, rib fractures. Her
doctor suspected myeloma (plasmocytoma). What of the following laboratory characteristics will be of the
greatest diagnostic importance? {=Paraproteinemia
~Hyperalbuminemia
~Proteinuria
~Hypoglobulinemia
~Hypoproteinemia}

Having helped to eliminate consequences of a failure at a nuclear power plant, a worker got an irradiation doze of
500 roentgen. He complains of headache, nausea, dizziness. What changes in leukocytes quantity can be expected
10 hours after irradiation?
{=Neutrophilic leukocytosis
~Lymphocytosis
~Leukopenia
~Agranulocytosis
~Leukemia}

After an attack of bronchial asthma a patient had his peripheral blood tested. What changes can be expected?
{=Eosinophilia
~Leukopenia
~Lymphocytosis
~Thrombocytopenia
~Erythrocytosis}

A patient suffering from chronic myeloleukemia has got the following symptoms of anemia: decreased number of
erythrocytes and low haemoglobin concentration, oxyphilic and polychromatophilic normocytes, microcytes. What
is the leading pathogenetic mechanism of anemia development?
{=Substitution of haemoblast
~Intravascular hemolysis of erythrocytes
~Deficiency of vitamin B12
~Reduced synthesis of erythropoietin
~Chronic haemorrhage}

As a result of a road accident a 26- year-old man is in the torpid phase of shock. Blood count: leukocytes - 3, 2 -
109/l. What is the leading mechanism of leukopenia development?
{=Leukocyte redistribution in the bloodstream
~Leukopoiesis inhibition
~Faulty release of mature leukocytes from the bone marrow into the blood
~Leukocyte destruction in the hematopietic organs
~Increased excretion of the leukocytes from the organism}

The cellular composition of exudate largely depends on the etiological factor of inflammation. What leukocytes are
the first to get into the focus of inflammation caused by pyogenic bacteria? {=Neutrophil granulocytes
~Monocytes
~Myelocytes
~Eosinophilic granulocytes
~Basophils}

A 3-year-old child had eaten some strawberries. Soon he developed a rash and itching. What was found in
the child’s leukogram? {=Eosinophilia
~Hypolymphemia
~Neutrophilic leukocytosis
~Monocytosis
~Lymphocytosis}

A patient visited a dentist to extract a tooth. After the tooth had been extracted, bleeding from the tooth socket
continued for 15 minutes. Anamnesis states that the patient suffers from active chronic hepatitis. What
phenomenon can extend the time of hemorrhage?
{=Decrease of fibrinogen content in blood
~Thrombocytopenia
~Hypocalcemia
~Increased activity of anticoagulation system
~Decrease of albumine content in blood}

A patient consulted a doctor with complaints of dyspnea occurring after physical exertion. Physical examination
revealed anemia, paraprotein was detected among gamma globulins. What value should be determined in the
patient’s urine to confirm the diagnosis of myeloma? {=Bence Jones protein
~Hemoglobin
~Antitrypsin
~Ceruloplasmin
~Bilirubin}

A 59-year-old woman has been hospialized in a surgical ward due to exacerbation of chronic osteomyelitis of the
left shin. Blood test: leukocytes - 15, 0 x 109/1. Leukogram: myelocytes - 0%, metamyelocytes - 8%, stab
neutrophils - 28%, segmented neutrophils - 32%, lymphocytes - 29%, monocytes - 3%. Such blood count would be
called:
{=Regenerative left shift
~Hyperregenerative left shift
~Regenerative-degenerative left shift
~Degenerative left shift
~Right shift}

Cellular composition of exudate largely depends on the etiological factor of inflammation. What leukocytes are the
first to be involved in the focus of inflammation caused by pyogenic bacteria?
{=Basophils
~Neutrophil granulocytes
~Myelocytes
~Monocytes
~Eosinophilic granulocytes}

A patient is 20 years old, an athlete.He addressed a doctor with complaints of fatigue, fever up to 38°C - 40°C.
Objectively: the liver and spleen are enlarged, lymph nodes on palpation are slightly enlarged, dense, painless.
Blood test: Hb- 100 g/l; erythrocytes - 2, 9 x 1012/1; leukocytes - 4, 4 x 109/1. Leukogram: 68% of blast cells.
Cytochemical investigation of blast cells revealed negative reactions to glycogen, peroxidase, non-specific
esterase, lipids. Name this disease:
{=Acute monoblastic leukemia
~Acute myeloid leukemia
~Acute undifferentiated leukemia
~Acute lymphoblastic leukemia
~Acute megakaryoblastic leukemia}

7-yeat-old boy died of acute posthemorrhagic anemia caused by profuse hemorrhage in the gastrointestinal tract.
Postmortem study revealed the following: macroscopically there were acutely enlarged various groups of the
lymph nodes, thymomegaly, hepatosplenomegaly, and bright red bone marrow; microscopically there was
hypercellular bone marrow with monomorphic infiltrations composed of blasts and diffuse-focal tumor infiltrations
in the liver, spleen, lymph nodes, brain substance and tunics. Make the diagnosis:
{=Acute myeloblastic leukemia
~Acute monoblastic leukemia
~Acute plasmablastic leukemia
~Acute lymphoblastic leukemia
~Acute undifferentiated leukemia}

32-years-old male patient has hemorrhage after injury of vessel that connected to formation of friable thrombi. What
coagulation factor deficiency has led to this disorder?
{=XI1I (fibrin stabilizing factor)
~III (thromboplastin)
~VII (proconvertin)
~XII (Hageman’s factor)
~II (prothrombin)}
A man, who has been to Arctic for a long time, has hemorrhage from gums, his teeth sway and pull out. What is the
initial mechanism in scurvy development?
{=Impairment of collagen synthesis
~Affection of alveolar process
~Infringement of elastin synthesis
~Fragility of capillaries
~Insufficient tightness of round ligament of tooth}

Preliminary exfoliation of placenta and hemorrhagic shock develop in a pregnant woman during the delivery.
While taking woman’s blood for analysis it coagulates in syringe. The acute DIC syndrome is diagnosed in this
woman. What pathogenic mechanism appears to be initial in disorder of hemostasis in this case? {=Activation of
tissue thromboplastin derived from destroyed tissues
~Elevation of platelet aggregation
~Activation of sympathetic-adrenal system
~Cascade activation of plasma coagulation factors
~Oppression of aticoagulative system of blood}

It was found out before operating on that the patient has bleeding time increased up to 9 min. What blood cells
deficiency do these changes result from?
{=Erythrocytes
~Platelets
~Monocytes
~Lymphocytes
~Leukocytes}

Breathlessness, acute pain in the chest, cyanosis, and enlargement of neck veins rapidly develop in a patient suffered
from thrombophlebitis. What is the most possible disorder of blood circulation in this patient? {=Cerebral
thromboembolism
~Pulmonary thromboembolism
~Coronary thromboembolism
~Mesenterial thromboembolism
~Portal thromboembolism}

A teeth was pulled out from the patient with chronic hepatitis. Bleeding, which developed after that, could not be
seized for 2 hours. Performed blood analysis establishes increase in content of several coagulation factors. What kind
of hemostasis is damaged in this case? {=Coagulation
~Platelet-vascular.
~Vascular stasis
~Platelet reaction
~Vascular reaction}
Numerous subcutaneous hemorrhages appear in a patient suffered from hepatic cirrhosis. What is a possible reason
for this?
{=Reduction in synthesis of factor II (prothrombin)
~Excessive decay of vitamin
~Disorder of vitamin K synthesis
~Hypocalemia
~Deficiency of factor II (thromboplastin) in blood plasma}
Hemorrhagic syndrome with disorders of the third stage of blood coagulation develops in a patient after she was
operated on uterus. What is the most probable mechanism of hemostasis disorder in this case?
{=Activation of fibrinolysis
~Qualitative abnormalities of fibrinogen
~Deficit of fibrin stabilizing factor
~Decrease in ptothrombin synthesis
~Decrease in fibrinogen synthesis}

Hemorrhagic diathesis has developed in a patient with streptococcus infection. What is the reason for hemorrhage
development?
{=Enhanced fibrinolysis
~Vitamin A deficiency
~Increase in callicrein content in blood
~Elevation of heparin content in the blood
~Vitamin C deficit}

Point hemorrhages appear on a forearm of a patient after putting a tourniquet on a patient’s arm. What functions of
blood cells do these changes connect to?
{=Platelets
~Neutrophils
~Erythrocytes
~Basophils
~Macrophages}

Numerous hemorrhages and bruises were found out on the patient’s body. At examination of this patient: his bleeding
time by Duke is 25 min, his number of platelets is 25x10°/L. What disease are these symptoms characteristic for?
{=Hemophilia B
~Von Willebrand’s disease
~Vitamin C deficiency
~Hereditary defect of platelet formation
~Hemophilia A}

A 65-years-old male patient suffered from atherosclerosis was admitted to the surgical department because of diffuse
purulent peritonitis. When the patient was operated on thrombosis of mesenterial vessels was diagnosed in him. What
is the most possible reason for peritonitis development?
Stasis
{=Agiospastic ischemia
~Ischemic infarction
~Hemorrhagic infarction
~Compressive ischemia}
Hemorrhagic syndrome connected to disorders of the third phase of coagulation developed in a patient after hi was
operated on pancreas. What is the possible mechanism of development of hemostasis disorder? {=Activation of
fibrinolysis
~Reduction in fibrinogen synthesis
~Elevation in content of heparin in patient’s blood
~Deficit of fibrin stabilizing factor
~Reduction in prothrombin synthesis}

A disaster fighter at a nuclear power plant developed hemorrhagic syndrome on the background of acute radiation
disease. What is the most important factor of syndrome pathogenesis?
{=Thrombocytopenia
~Vascular wall damage
~Increased activity of fibrinolysis factors
~Increased activity of anticoagulative system factors
~Decreased activity of coagulative factors}

A tooth extraction in a patient with chronic persistent hepatitis was complicated with prolonged hemorrhage. What
is the reason for the haemorrhagic syndrome?
{=Decrease in thrombin production
~Increase in thromboplastin production
~Decrease in fibrin production
~Increase in fibrinogen synthesis
~Fibrinolysis intensification}

After a tourniquet application a patient was found to have petechial haemorrhages. The reason for it is the dysfunction
of the following cells:
{=Platelets
~Eosinophils
~Monocytes
~Lymphocytes
~Neutrophils}

A disaster fighter at a nuclear power plant developed hemorrhagic syndrome on the background of acute radiation
disease. What is the most important factor of syndrome pathogenesis?
{=Thrombocytopenia
~Vascular wall damage
~Increased activity of fibrinolysis factors
~Increased activity of anticoagulative system factors
~Decreased activity of coagulative factors}

A boy has congenital disorder of hemostasis: he has prolonged hemorrhage even in case of insignificant injuries,
subcutaneous bruises, and bleedings in joint cavities, which restrict the movement activity. Patient’s blood does not
coagulate for a long time if it is taken out from the organism and it does not contain coagulation factor VIII. Blood
cells count including number of platelets is within norm. What underlies the congenital disease of this boy?
{=Hereditary gene defect linked with X chromosome
~Toxicosis of pregnancy in boy’s mother
~Intrauterine infection
~Intrauterine intoxication
~Intrauterine immune conflict}
A 3-year-old boy with pronounced hemorrhagic syndrome doesn’t have antihemophilic globulin A (factor VOI) in
the blood plasma. Hemostasis has been impaired at the following stage:
{=Internal mechanism of prothrombinase activation
~Extemal mechanism of prothrombinase activation
~Conversion of prothrombin to thrombin
~Conversion of fibrinogen to fibrin
~Blood clot retraction}

A patient is diagnosed with hereditary coagulopathy that is characterised by factor VIII deficiency. Specify the phase
of blood clotting during which coagulation will be disrupted in the given case:
{=Thromboplastin formation
~Thrombin formation
~Fibrin formation
~Clot retraction
~}
After implantation of a cardiac valve a young man systematically takes indirect anticoagulants. His state was
complicated by hemorrhage. What substance content has decreased in blood? {=Prothrombin
~Haptoglobin
~Heparin
~Creatin
~Ceruloplasmin}

A patient is diagnosed with hereditary coagulopathy that is characterised by factor VIII deficiency. Specify the phase
of blood clotting during which coagulation will be disrupted in the given case:
{=Thromboplastin formation
~Thrombin formation
~Fibrin formation
~Clot retraction}

A 3-year-old boy with pronounced hemorrhagic syndrome has no antihemophilic globulin A (factor VIID in the
blood plasma. Hemostasis has been impaired at the following stage:
{=External mechanism of prothrombinase activation
~Conversion of fibrinogen to fibrin
~Blood clot retraction
~Internal mechanism of prothrombinase activation
~Conversion of prothrombin to thrombin}

Thrombosis of coronary arteries occurs in a patient with atherosclerosis of cordial vessels. What is the reason for
formation of thrombi? {=Heparin deficiency
~Thrombocytopenia
~Acceleration of blood flow
~Activation of fibrinolysis
~Vitamin K deficiency}

Which of essential factors of thrombus formation the most often serves as the reason for thrombosis?
{=Injury of vessel wall
~Activation of systems of blood sedimentation
~Decrease in activity of anticoagulative system
~Acceleration of blood flow
~Turbulence of blood}
Anelderly patient was admitted to the hospital with thrombosis of veins of his calf. What is the reason for thrombosis
development?
{=Injury of vessel wall
~Increase in heparin concentration in blood
~Decrease in prothrombin concentration in blood
~Slowing-down of blood flow
~Activation of plasmin}

When examining a patient increased blood coagulation (thrombophilia) was found out in him. What reasons favor
such state development?
{=Intensification of prostacyclin synthesis in vessel wall
~Low concentration of thrombin in blood
~High concentration of heparin in blood
~Deficit of inhibitors of proteolytic enzymes
~High concentration of adrenalin in blood}

Thrombocytopenia was found out in blood of a patient with Werlgoff’s disease. Why the pathology of platelets leads
to hemorrhage?
{=Decrease in thromboplastin formation
~Decrease in concentration of prothrombin in blood
~Decrease in concentration of heparin in blood
~Activation of fibrinolytic system
~Decrease in concentration of fibrinogen in blood}

Hemophilia A was diagnosed in a patient with hemorrhagic syndrome. What is the reason for this disease
development?
{=Absence of factor VIII
~Absence of Christmas’s factor (factor IX)
~Absence of Stewart’s factor (factor X)
~Absence of Rosenthal’s factor (factor XT)
~Low concentration of Hageman’s factor (factor XII) in blood}

A patient was admitted to the hospital with abundant hemorrhoid bleeding. This patient has been suffering from
hepatic cirrhosis for a long time. What is the reason for hemorrhage development under hepatic cirrhosis?
{=Prothrombin deficiency
~Plasmin deficiency
~Activation of fibrinolysis
~Low concentration of thrombostenin in blood
~Excess of heparin}

Patient has hemorrhage from gums, subcutaneous hemorrhages, and frequent nasal bleedings. Significant
Thrombocytopenia was revealed in blood test of this patient. What is the reason for bleeding development in case of
thrombocytopenia?
{=Reduction of thromboplastin formation
~Thrombin deficiency
~Excessive heparin formation
~Activation of fibrinolysis
~Excess of prostacyclins}

Shonlein-Henoch disease was diagnosed in a patient. What changes in blood cells number are characteristic for this
disease? {=Eosinophilia
~Thrombocytopenia
~Erythropoenia
~Polycytemia
~Eosinopoenia}

While performing intravenous injections trauma of venous vessels leads to thrombosis. What favors thrombosis under
injury of vessel wall? {=Accumulation of thromboxan Az
~Activation of fibrinolysis
~Activation of phospholipase D
~Enhanced production of prostacyclin by endotheloicytes
~Activation of plasmin}

Changes of some indices of blood were revealed at examination of a patient suffered from hemophilia. What of
enumerated indices corresponds to this condition?
{=Time of coagulation takes longer
~Bleeding time by Duke takes longer
~Eosinophilia
~D Thrombocytopenia.
~Afibrinogenemia}

Hemophilia B was diagnosed in a child who has hemorrhagic syndrome. This type of hemophilia results from absence
of:
{=Coagulation factor [X (Christmas’s factor)
~Coagulation factor IT (prothrombin)
~Coagulation factor VII (antihemophilic globulin) ~Coagulation
factor XI (thromboplastin)
~Coagulation factor XII (Hageman’s factor)}

It is known that thrombus undergo same changes after its formation. What sort of finishing of thrombus formation is
the most dangerous for a patient?
{=Septic lysis
~Aseptic lysis
~Organization without recanalization
~Organization with recanalization
~Calsification of thrombus}

Such appearances as petechiae and ecchymoses develop in a boy aged 7, who often fall ill with acute respiratory
diseases. Pathology of internal organs is absent in this patient. What pathology is present in this case?
{=Thrombocytopenia
~Hypoplastic anemia
~Acute leukemia
~Hemophilia
~Chronic leukemia}
Course of atherosclerosis in 70-years-old patient is complicated by thrombosis of vessels of lower extremities and
development of gangrene of left foot toes. The beginning of thrombosis is connected to:
{=Adhesion, aggregation and agglutination of platelets
~Activation of prothrombinase
~Conversion of prothrombin to thrombin
~Conversion of fibrinogen to fibrin
~Decreased heparin synthesis}

Antihemophilic globulin A (factor VIID is absent in the blood plasma of a boy with significant hemorrhagic
syndrome. What phase of hemostasis is infringed primarily in this boy?
{=Intrinsic pathway of prothrombinase (thrombokinase) activation
~Conversion of fibrinogen to fibrin
~Conversion of prothrombin to thrombin
~Extrinsic pathway of prothrombinase (thrombokinase) activation ~Retraction
of blood clot}

A patient complains of hemorrhage from gums. What vitamin deficiency may lead to this phenomenon? {=K
~D
~B2
~A
~Bi}

Prolonged bleeding complicated pulling out of a tooth in a patient, who suffered from chronic leukemia. What may
serve a reason for hemorrhagic syndrome in this patient?
{=Thrombocytopenia
~Anemia
~Erythropenia
~Lymphocytosis
~Neutropenia}

Bleeding developed ina 15-years-old patient after her teeth was pulled out. It was found out that after endued hepatitis
A this patient has point hemorrhages (petechiae and ecchymoses) and bruises on her skin, and she had three accidents
of nasal bleeding. Before this she did not have hemorrhages even after pulling out of her tooth. It was established
under examination that the bleeding time is increased in this patient. At the patient’s blood test hypochromic anemia,
decrease in number of platelets, and markedly reduced adhesion of platelets are present. Prothrombin and thrombin
time are normal in this patient. Her differential blood count is normal too. What is the most possible diagnosis?
{=Acquired thrombocytopathy
~Acquired thrombocytopenia
~Hereditary thrombocytopenia
~Hemophilia
~Hereditary thrombocytopathy}

A patient complains of frequent hemorrhages from gums. When carrying out blood test in this patient deficiency of
factor of hemocoagulation (prothrombin) was revealed. What phase of blood coagulation is primarily impaired in
the patient?
{=Formation of thrombin
~Fibrinolysis
~Formation of fibrin
~Retraction of blood clot
~Formation of prothrombinase}

A 12-years-old patient was admitted to the hospital with hemarthrosis of knee joint (hemorrhage into joint cavity).
This patient has been suffering from hemorrhages since early childhood. What disease does this boy suffer from?
{=Hemophilia
~Hemorrhagic vasculitis
~Iron deficiency anemia
~Vitamin Bi2 deficiency anemia
~Thrombocytic purpura}

A 30-years-old female patient, who is suffering from megrim, often takes analgin. Hemorrhages on skin and frequent
nasal bleedings appear in her at the recent time. At her blood analysis: number of platelets is 30x10°/L; bleeding time
is increased. What do these changes result from?
{=Autoimmune thrombocytopenia
~Hemorrhagic vasculitis
~Hemolytic anemia
~Angiohemophilia
~Thrombocytopathy }

A worker at pharmaceutical plant addresses the doctor with complaints of general malaise, significant hemorrhages
from gums, nasal bleedings, and numerous subcutaneous hemorrhages. At blood analysis of this patient following
was revealed: number of erythrocytes is 2.2x10!7, content of hemoglobin is 48 g/L, presence of neutropenia with
relative lymphocytosis, and number of platelets is 35x10°/L. What is the possible pathogenesis of thrombocytopenia
in this patient? {=Decrease of platelet production
~Enhanced platelet destruction
~Increase platelet utilization
~Redistribution of platelets
~Increased loss of platelets}

Intravenous injection of mercury chloride to experimental animal evokes development of thrombosis in it. What is
the basic pathogenic factor in development of this pathologic process?
{=Injury of vessel wall
~Activation of coagulation system
~Activation of platelets adhesion
~Slowing-down of blood flow
~Decrease in anticoagulants activity}

A disaster fighter at a nuclear power plant developed hemorrhagic syndrome on the background of acute radiation
disease. What is the most important factor of syndrome pathogenesis?
{=Thrombocytopenia
~Vascular wall damage
~Increased activity of fibrinolysis factors
~Increased activity of anticoagulative system factors
~Decreased activity of coagulative factors}
A tooth extraction in a patient with chronic persistent hepatitis was complicated with prolonged hemorrhage. What
is the reason for the haemorrhagic syndrome?
{=Decrease in thrombin production
~Increase in thromboplastin production
~Decrease in fibrin production
~Increase in fibrinogen synthesis
~Fibrinolysis intensification}

What kind of pathologic processes underlie formation of thrombus?


{=Astringent metamorphose of thrombocytes
~Increase of osmotic pressure
~Decrease in number of thrombocytes
~Increase of oncotic pressure
~Decrease in number of erythrocytes}

After a tourniquet application a patient was found to have petechial haemorrhages. The reason for it is the dysfunction
of the following cells:
{=Platelets
~Eosinophils
~Monocytes
~Lymphocytes
~Neutrophils}

A disaster fighter at a nuclear power plant developed hemorrhagic syndrome on the background of acute radiation
disease. What is the most important factor of syndrome pathogenesis?
{=Thrombocytopenia
~Vascular wall damage
~Increased activity of fibrinolysis factors
~Increased activity of anticoagulative system factors
~Decreased activity of coagulative factors}

A 3-year-old boy with pronounced hemorrhagic syndrome doesn’t have antihemophilic globulin A (factor VIID in
the blood plasma. Hemostasis has been impaired at the following stage:
{=Internal mechanism of prothrombinase activation
~Extemal mechanism of prothrombinase activation
~Conversion of prothrombin to thrombin
~Conversion of fibrinogen to fibrin
~Blood clot retraction}

A patient is diagnosed with hereditary coagulopathy that is characterised by factor VIII deficiency. Specify the phase
of blood clotting during which coagulation will be disrupted in the given case:
{=Thromboplastin formation
~Thrombin formation
~Fibrin formation
~Clot retraction
~}
After implantation of a cardiac valve a young man systematically takes indirect anticoagulants. His state was
complicated by hemorrhage. What substance content has decreased in blood? {=Prothrombin
~Haptoglobin
~Heparin
~Creatin
~Ceruloplasmin}

A patient is diagnosed with hereditary coagulopathy that is characterised by factor VIII deficiency. Specify the phase
of blood clotting during which coagulation will be disrupted in the given case:
{=Thromboplastin formation
~Thrombin formation
~Fibrin formation
~Clot retraction}
A 3-year-old boy with pronounced hemorrhagic syndrome has no antihemophilic globulin A (factor VIID in the
blood plasma. Hemostasis has been impaired at the following stage:
{=External mechanism of prothrombinase activation
~Conversion of fibrinogen to fibrin
~Blood clot retraction
~Internal mechanism of prothrombinase activation
~Conversion of prothrombin to thrombin}

After pancreatic surgery the patient developed hemorrhagic syndrome with disturbed 3rd stage of blood clotting.
What will be the most likely mechanism of the hemostatic disorder?
{=Fibrinolysis activation
~Decrease of fibrinogen synthesis
~Fibrin-stabilizing factor deficiency
~Qualitative abnormalities of fibrinogenesis
~Decrease of prothrombin synthesis}
Choose the most exact definition of congenital diseases:
{=Diseases which appear with birth
~All inherited diseases
~Diseases are predefined by pathology of delivery
~Transplacental infectious diseases
~Diseases with the dominant type of inheritance}

Color blindness was diagnosed in a boy. The color vision of parents is normal. A grandfather and greatgrandfather
had the same anomaly. There were not sick persons among women of this family. What is the type of inheritance
of this illness?
{=X-linked recessive
~Incomplete dominant
~Autosomal-dominant
~Autosomal-recessive
~X-linked dominant}

Delay of physical and psychical development, upward slant of palpebral fissures, multiple anomalies of internal
organs were revealed in a boy 8 month. Trisomy of 21 chromosomes was revealed during investigation of the
karyotype. What is the most provisional diagnosis?
{=Down’s syndrome
~Kleinfelter’s syndrome
~Tumer’s syndrome
~Edvards syndrome
~Patau syndrome}

Disorders of higher nervous activity and coordination of movements in the conditions of increased atmospheric
pressure action associated to the toxic action on neurons of:
{=Nitrogen
~Oxygen
~Dioxide carbon
~Indole
~Ammonia}

Indicate a term from the beginning of clinical death in which the reanimation of organism is possible and
expedient
{=At the latest in 5 min
~At the latest in 25 min
~At the latest in 40 min
~At the latest in 60 min
~At the latest in 15 min}

Indicate the pathogenic principle of phenylketonuria treatment:


{=Limitation of phenylalanine in diet
~Addition of phenylalanine to diet
~Removal of galactose from diet
~Removal of fructose from diet
~Addition of galactose to diet}
It is diagnosed trisomy of 13 chromosome during investigation of karyotype of newborn boy with defects
“harelip” and “cleft palate”, defects of the nervous and cardiovascular systems, organ of vision. Which from the
following syndromes is present ina boy? {=Patau syndrome ~Kleinfelter’s syndrome
~Turner’s syndrome ~Edvards
syndrome
~Down’s syndrome}
It is known that biochemical methods are widely used in diagnostics of the inherited diseases. What express
method is possible in diagnose of phenylketonuria?
{=Test with trichloroacetic iron
~Test with methylene-blue
~Test with acetic acid
~Test with galactose
~Test with solution of iodine}

It is known that hemeralopia is inherited on X-linked recessive inheritance type. What is the probability of
hemeralopia appearance in newborn boy when it is known that the father of pregnant woman suffers with the
hemeralopia?
{=50 %
~0%
~15 %
~100 %
~25 %}

Itis known the diseases an environment and inherited susceptibility acts role in its development: diabetes mellitus,
arterial hypertension. These illnesses are named:
{=Multifactorial diseases
~Occupational diseases
~Chromosomal diseases
~Endocrine diseases
~Metabolic diseases}

It was discovered the somatic cells of three populations: normal cells, monosomy cells, thrisomy cells at analysis
of karyotype of patient with the Down’s illness. What mechanism underlie in the basis of this phenomenon?
{=Inversion
~Mosaicism
~Polyploidy
~Deletion
~One chromosome loss}

Three chromosomes of 21st pair were revealed in a baby by karyotyping. Mark the phenotypic manifestation of
this mutation. {=Down’s syndrome
~Edward’s syndrome
~Patau’s syndrome
~Tumer’s syndrome
~Polisomia Y-syndrome}

Test of amniotic fluid for determination of sexual chromatin showed that fetus’s cells include 2 bodies of sexual
chromatin (Bar’s bodies). Which disease was revealed in the fetus? {=Trisomy X-syndrome
~Patau’s syndrome
~Down’s syndrome
~Edward;s syndrome
~Klinefelter’s syndrome}

A 20-year-old man complains of headache, failing sight, muscular weakness. His height is 2.00 m, weight is 80
kg, he has long extremities, and kyphosis. On X-ray examination the skull film showed enlargement of Turkish
saddle. Sexual chromatin is absent. What pathway is the most possible in appearance of gigantism in the young
man?
{=Excess of growth hormone
~Somatoliberin insufficiency
~Gonadoliberin insufficiency
~Testosterone insufficiency
~Chromosomal mutation}

A woman addressed to the medical-genetic consultation. Short neck with wing-shaped skin folds (“sphinks’s
neck”), broad chest, and mammary hypoplasia were revealed in her on examination. What is the most possible
diagnosis?
{=Tumer’s synrome
~Syndrome of “cat’s cry”
~Patau’s syndrome
~Morris’s syndrome
~Klinefelter’s syndrome}

A child being born in late marriage has short stature, mental retardation, thick geographic tongue, narrow
palpebral fissures, and flat broad face. What disturbance underlies development of described syndrome?
{=Hereditary chromosomal pathology
~Birth injury
~Intrauterine intoxication
~Intrauterine infection
~Intrauterine immunoincompatible}

Mother’s karyotype has 45 chromosomes. There was revealed that it connects with translocation of the 21st
chromosome to the 15th one. Which disease more possible would develop in her child if the father’s karyotype
is normal?
{=Down’s syndrome
~Patau’s syndrome
~Klinefelter syndrome
~Morris’s syndrome
~Edward’s syndrome}

A child with hereditary defects manifested characteristic syndrome named “cat’s cry” just after the birth. At that
newborns have “meowing” timbre of their voice. In examination of the karyotype of this child following was
revealed:
{=Deletion of segment of 5th chromosome
~Additional 21st chromosome
~Additional Y-chromosome
~Additional X-chromosome
~Lack of X-chromosome}
A woman addressed to the hospital with complaints of suppurating incised wound. A doctor was cleaning the
wound surface with 3% solution of H202. But foam did not form. What does absence of preparation activity
connected with?
{=Hereditary insufficiency of katalase
~Low concentration of H2O2
~Superficial wound
~Hereditary insufficiency of erythrocyte phosphatdehydrogenese ~Presence
of pus in the wound}

Sexual chromatin was revealed at examination of the buccal epithelium of a man. Which chromosomal disease is
characterized by this symptom?
{=Klinefelter’s syndrome ~Trisomy
X-chromosome
~Tumer’s syndrome
~Hypophosphatemic rickets
~Down’s disease}

A 10-month old baby has fair hair, very white skin and blue eyes. His parents have dark hair. He had normal
appearance, but during last three months impairment of cerebral circulation and mental retardation developed in
him. The reason for such condition can be:
{=Phenylketonuria
~Acute porphyry
~Hystidinemia
~Lactosemia
~Glycogenose}

Daltonism (color blindness) was revealed in a 7-year-old boy at examination. His parents are healthy. But his
mother’s father had the same anomaly. Which is the type of inheritance of this anomaly? {=X-linked recessive
~Autosomal dominant
~X-linked dominant
~Autosomal recessive
~Partial dominance}

Two types of cells with chromosomal set 46XY and 47XXY in the equal quantity were revealed at examination
of karyotype of the patient. What is the most possible diagnosis?
{=Kleinefelter’s syndrome
~Down’s syndrome
~Normal karyotype
~X-syndrome monosomia
~Patau’s syndrome}

A 14-year old girl has short stature, absence of pubescence, very short neck, broad shoulders, and normal
intelligence. What disease does the girl have?
{=Turmer’s syndrome
~Patau’s syndrome
~Edward’s syndrome
~Down’s syndrome
~Klinefelter’s syndrome}
A 5-year old girl’s karyotype has 46 chromosomes. But one chromosome of the 15th pair is longer than normal
one, since it joins chromosome from 21st pair. What type of mutation is present in the girl? {=Translocation
~Deletion
~Inversion
~Shortage
~Duplication}

There is regurgitation of milk to the nose in an infant during a meal. Mark the possible reason for this disturbance?
{=Cleft palate
~Fracture of basis of skull

~Septal deformity
~Harelip}

The father of a pregnant woman suffers from gemeralopia. This is an X-linked recessive disorder. Her husband’s
relatives didn’t suffer from this disease. What is possibility that her child will suffer from gemeralopia, if it is a
boy?
{=50%
~0%
~75%
~100%
~25%}

Elementary discrete unit of inheritance in both sexual and asexual reproduction is:
{=One gene
~One chain of DNA molecule
~One pair of nucleotides
~Two chains of DNA molecule
~One nucleotide}

A 12-year-old boy has lower jaw carcinoma. What type of mutation underlies the tumor development? {=Non-
lethal mutation of somatic cell
~Lethal mutation of somatic cell
~Non-lethal mutation of sexual cell mother or father
~Non-lethal mutation of somatic cell in embryogenesis
~Lethal mutation of somatic cell in embryogenesis}

Child was born with hereditary pathology - cleft palate. What type of mutation underlies this pathology? {=Non-
lethal mutation of somatic cell in embryogenesis
~Non-lethal mutation of somatic cell
~Lethal mutation of somatic cell
~Non-lethal mutation of sexual cell mother or father
~Lethal mutation of somatic cell in embryogenesis}

A 10-year-old boy with mental retardation has typical appearance: small head with chamfered nape, oblique slant
of palpebral fissures, low-bridged nose, and half-open mouth. A doctor is keeping him under observation in
connection with congenital heart disease. What violation of karyotype causes the pathology in this boy?
{=Trisomy of 21 pare of chromosome
~Trisomy of 13 pare of chromosome
~Trisomy of 18 pare of chromosome
~Trisomy X
~Monosomy of X-chromosome}

A 25-year-old man complains of sterility. He is tall; he has asthenic body build and decrease in intelligence. Bar’s
bodies were revealed in biopsy of his buccal epithelium. What pathology it may be?
{=Klinefelter’s syndrome
~Adipopse-genital dystrophy
~Acromegaly
~Adrenogenital syndrome
~Eunohoidism}

Few drops of 5% solution of trichloroacetic iron were added to urine of a newborn. Color of urine did not change.
Phenylketonuria with irreversible brain injury was diagnosed in this child at the age of two. Why was not
phenylketonuria diagnosed in this newborn?
{=Wrong time was chosen
~Wrong substrate was chosen
~Analytic mistake was made
~Reagent was prepared incorrectly
~Wrong interpretation of results}

Discharge of milk from nose was observed in a newborn during first feeding. Which inborn defect of oral cavity
can cause such condition?
{=Cleft palate
~Absence of soft palate
~Splitting of alveolar bone
~Harelip
~This is normal occurrence}
A patient is short; he has mental retardation, epicanthic folds, and short fingers. The trisomy of 21st pare of
chromosome has been revealed in his karyotype. How is this chromosomal anomaly referred to?
{=Down’s syndrome
~Klinefelter’s syndrome
~Tumer’s syndrome
~Trisomy X-syndrome
~Specific fetopathy}

In the second half of pregnancy a woman has disorders of blood circulation in placenta with violation of all its
functions: respiratory, trophic, protective, excretory, and hormonogenic. This has led to development of
secondary placental insufficiency syndrome. Which pathologic process can develop in such conditions?
{=Fetopathy
~Blastopathy
~Embriopathy
~Gametopathy
~Galactosemia}

A patient was exposed to ionizing radiation by accident. It has led to development of deletion — a loss of
chromosome segment. How is this pathologic happening referred to?
{=Mutation
~Exudation
~Alteration
~Enzymopathy
~Reparation}

Mutation, inversion of one chromosome, has developed in a patient as a consequence of ionizing radiation
influence. How is pathogenic factor called, which evoked these pathologic changes? {=Physical mutagen
~Chemical mutagen
~Biological mutagen
~Carcinogen
~Virus}

A patient has mutation of gene which is responsible for hemoglobin synthesis. It has lead to development of
sickle-cell anemia. What kind of pathological hemoglobin is found out this case? {=Hb S
~Hb Ai ~Hb
F
~Hb A
~Bart-Hb}

A teenager has asthenic body build, gynecomastia, and testicular atrophy. For verification of diagnosis he was
sent to medical-genetic consulting room. What karyotype will be found out in this case?
{=47 XXY, one Bar’s body
~47 XXY, two Bar’s bodies
~46 XY, Bar’s bodies are absent
~45 XO, Bar’s bodies are absent ~47
XXX, two Bar’s body}

In which variant of karyotype in nucleus of somatic cells only one Bar’s body can be found? {=47
XX, 15+
~45 XO
~45 XY
~D.47 XY, 21+
~48 XXXY}

A patient has pathological processes resulted from X-linked gene mutation. This disease is accompanied by
deficiency of VIII factor of blood coagulation and prolongation of blood coagulation time to 25 min. What is this
disease? {=Hemophilia
~Glaucoma
~Hemeralopia
~Daltonism
~Galactosemia}

Healthy parents gave birth to a girl with blue eyes and fair hair. During the first months the girl developed
irritability, anxiety, disturbances of nutrition and sleep. Neurologist’s examination established development delay
in her. What genetic method has to be applied in this case?
{=Biochemical
~Cytological
~Gemellary
~Genealogical
~Population-statistic}

46 chromosomes were revealed at patient with signs of Down’s syndrome. What kind of chromosomal anomalies
have led to this disease?
{=Translocation
~Inversion
~Deletion
~Duplication
~Circling}

Father has astigmatism and mother is healthy. What is possibility that their children will suffer from astigmatism?
{=50% ~100%
~715%
~25%
~O%}

A 3-year-old child has edematous skin, frequent inflammations with marked exudations, and prolonged allergic
reactions. Which type of diathesis does this child have? {=Exudative
~Asthenic
~Lymphohypoplastic diathesis
~Gouty diathesis
~Hemorrhage}

Congenital diseases are diseases, which are:


{=Manifesting at birth of a child
~All hereditary diseases
~Hereditary dominant disorders
~Result from pathologic labor
~Transplacental infectious diseases}

Which disorder is X-linked? {=Hemophilia


A
~Klinefelter’s syndrome
~Trisomy X-syndrome
~Hypophosphatemic rickets
~Short fingers}

A 32-year-old man has tall stature, high-pitched voice, gynecomastia, adult woman pattern of hair distribution,
mental retardation, infertility. Klinefelter’s syndrome was diagnosed in him previously. For diagnosis verification
it is important to examine:
{=Karyotype
~Rhesus-factor
~Spermatogenesis
~Blood group
~Genealogy}

A percentage of concordancy among monozygotic and dizygotic twins at following diseases comprises relatively:
tuberculosis — 66 and 23, schizophrenia - 69 and 10, epilepsy — 56 and 10, harelip — 33 and 5, endemic goiter —
71 and 70. Taking into account this information, genotype is not important for formation of {=Endemic goiter
~Harelip
~Epilepsy
~Schizophrenia
~Tuberculosis}
A patient has hernias, aortic aneurysm, long thin extremities, lens ectopia. Which hereditary disease are these
symptoms typical for?
{=Ehlers-Danlos’ syndrome
~Marfan’s syndrome
~Shegren’s syndrome
~Mucopolysaccharidoses
~Incomplete osteodgenesys}

During a prophylactic medical examination a 7-year-old boy was diagnosed with daltonism. His parents are
healthy and have normal colour vision, but his grandfather on his mother’s side has the same abnormality. What
is the type of the abnormality inheritance?
{=Recessive, sex-linked
~Autosomal dominant
~Semidominance
~Dominant, sex-linked
~Autosomal recessive}

Mother and father are healthy. Mother underwent amniocentesis for fetal karyotyping. The fetal karyotype turned
out to be 45, XO. What syndrome can be expected in a newborn baby?
{=Turner’s
~Edwards’
~Patau’s
~Cri du chat
~"Superwoman"}

Genealogical study of a family with hereditary enamel hypoplasia has revealed that the disease occurs in every
generation. In women, the anomaly occurs more frequently than in men. Male patients only pass this trait to their
daughters. What type of inheritance takes place in this case?
{=X-linked dominant
~Autosomal dominant
~Autosomal recessive
~Y-linked
~X-linked recessive}

If a trait is determined mostly by genetic factors, the percentage of concordance between the twins is much higher
in monozygotic twins than in dizygotic ones. What is the percentage of blood group concordance in monozygotic
twins?
{=100%
~715%
~50%
~25%
~O%}

In Western Europe nearly half of all congenital malformations occur in the children conceived in the period when
pesticides were used extensively in the region. Those congenital conditions result from the following influence:
{=Teratogenic
~Carcinogenic
~Malignization
~Mutagenic
~Mechanical}
Phenylketonuria is a disease caused by a recessive gene that is localized in the autosome. The parents are
heterozygous for this gene. They already have two sons with phenylketonuria and one healthy daughter. What is
the probability that their fourth child will have the disease too?
{=25%
~0%
~50%
~715%
~100%}

X-chromatin test of somatic cells is used for quick diagnostics of hereditary diseases caused by variations of sex
chromosomes number. What is the karyotype of a male, whose cells mostly contain one X-chromatin body?
{=47, XXY ~45,
X0
~46, XY
~48, XXXY
~49, XXXXY}

A woman who had been consuming alcohol excessively during her pregnancy had a child with cleft palate and
upper lip. These presentationsm are indicative of some chromosomal anomalies. What process do they result
from?
{=Teratogenesis
~Carcinogenesis
~Mutagenesis
~Phylogenesis
~Ontogenesis}

A woman had been taking synthetic hormones during her pregnancy. Her newborn girl presents with excessive
hairiness which has formal resemblance to adrenogenital syndrome. This sign of variability iscalled:
{=Phenocopy
~Mutation
~Recombination
~Heterosis
~Replication}

When examining a female patient a doctor observed the following: misshapen auricles, elevated palate, teeth
growth disorder; mental retardation; no disruption of reproductive function. Provisional diagnosis is the"super
woman"syndrome. Point out the karyotype of this disease:
{=(47, XXX)
~(47, XXY)
~(47, YYY)
~(47, XYY)
~(45, X0)}
Heterozygous parents with A (II) and B (III) blood groups according to ABO system gave birth to a child. What
is the probability of the child having 0 (D blood group?
{=50% ~0%
~100% ~75%
~25%}
Cytogenetic analysis allowed to determine the patient’s karyotype — 47, XY, +21/46, XY. Name this
condition: {=Genocopy
~Phenocopy
~Deletion
~Translocation
~Mosaicism}

A girl is diagnosed with primary microcephaly that is a monogenic autosomal recessive disease. Her natural
brother develops normally. What genotypes do the parents of these children have?
{=aa x aa
~AAXxAA
~Aax Aa
~AA x aa
~AABB x AABB}

Autosomal nondisjunction had occurred in a woman during meiosis. An ovum with the third copy of the 18th
chromosome was formed. The ovum was impregnated by normal spermatozoon. The resulting child will suffer
from:
{=Turmer’s syndrome
~Edwards’ syndrome
~Patau’s syndrome
~Klinefelter’s syndrome
~Down’s syndrome}

A patient was diagnosed with a monogenic hereditary disease. Name this disease: {=Hemophilia
~Hypertension
~Peptic ulcer disease of the stomach
~Poliomyelitis
~Hymenolepiasis}

A patient was diagnosed with Klinefelter’s syndrome. The patient with this disease will have the karyotype (47,
XXY). How many sex chromosomes are in this complement? {=Three
~Zero
~One
~Two
~Forty four}

A woman is diagnosed with Turner’s syndrome (karyotype 45, X0). How many autosomal pairs would her
somatic cells contain?
{=22
~24
~23
~44
~45}

Examination of an 18-year-old girl revealed the following features: ovarian hypoplasia, broad shoulders, narrow
hips, shortening of the lower extremities, webbed neck. Mental development is normal. The patient has been
diagnosed with Turner’s syndrome. What chromosomal abnormality does this patient have? {=Monosomy X
~Trisomy X
~Trisomy 13
~Trisomy 18
~Nullisomy X}

Genealogical study of a family with a monogenic hereditary disease. Name this disease: {=Hemophilia
~Hypertension
~Peptic ulcer disease of the stomach
~Poliomyelitis
~Hymenolepiasis}

Autosomal nondisjunction had occurred in a woman during meiosis. An ovum with the third copy of the 18th
chromosome was formed. The ovum was impregnated by normal spermatozoon. The resulting child will suffer
from:
{=Down’s syndrome
~Edwards’ syndrome
~Patau’s syndrome
~Klinefelter’s syndrome
~Turmer’s syndrome}
In a patient the sense of periodic “heart fading” appeared. On ECG: complexes, which are preceded by decreased
interval TP, reduced voltage of R wave, shortened RQ interval, complex QRST - normal, that is followed by
extended interval TR. What form of cardiac arrhythmia has place in this case? {=Atrial extrasystole
~Atrial fibrillation
~Incomplete atrioventricular block
~Ventricular fibrillation
~Ventricular extrasystole}

An athlete-stayer (distance runner) during competition developed acute heart failure. What is the reason of this
pathology?
{=Volume overload of the heart
~Coronary circulation disorder
~Direct myocardial damage
~Pathology of pericardium
~Overload of the heart with increased resistance}

ECG of a women with ischemic heart disease showed heart rate 230 / minute, slightly deformed P wave,
ventricular complexes are intact. What heart rate disorder is observed in the patient? {=Paroxysmal
atrial tachycardia
~Fibrillation
~Atrial fibrillation
~Ventricular fibrillation
~Ventricular extrasystole}

A patient with essential hypertension developed myocardium hypertrophy. What is the primary mechanism of
myocardium hypertrophy? {=Hyperplasia of cardiomyocytes
~Increased number of mitochondria
~Increased myocardium capillary networks
~Increased endoplasmic reticulum
~Increased synthesis of retractive proteins}

A 25-year-old patient after acute endocarditis developed aortic valves insufficiency due to which left ventricular
hypertrophy emerged. After eight weeks, heart weight increased by 100% and then its growth ceased. What
stopped the growth of myocardium? {=Decreased load per unit of muscle mass
~Coronary circulation impairment
~Deterioration of cardiomiocytes energetic supply
~Disorder of the regulatory heart supply
~Decreased protein synthesis and impairment of cardiomyocytes supply with plastic material}

In the experiment conducted on animals, which have undergone removal of adrenals, a significant delay of
potassium in the organism that led to the development of hyperkaliemia was observed. What heart rate disorder is
most likely to occur in such animals?
{=Sinus bradycardia
~Sinus tachycardia
~Atrial extrasystole
~Ventricular extrasystole
~Atrioventricular block}
A 55-year-old man, who has for years suffered from mitral valve insufficiency, developed acute heart failure. Which
pathophysiological type of heart failure is observed in this case?
{=Due to heart overload
~Due to hypoxic damage of the heart
~As a result of coronary heart damage
~Due to neurogenic heart damage
~As a result of acute tamponade of the heart}

While recording a patient’s ECG a doctor easily pressed with his fingers on the patient’s eyes and continued to
record ECG. Slowing down of the heart rate — bradycardia emerged. Impairment of what heart conduction system
property underlies this phenomenon?
{=Automatism
~Nervous
~Contractile
~Conductivity
~Dissociation}

While taking electrocardiogram of a patient, the periodic appearance of ventricular extrasystoles was identified. It
was found that P-waves are missing before extrasystoles. What is the most likely reason of P-wave disappearance
in this clinical situation?
{=Failure of retrograde impulse conduction through AV-node
~Blockade of impulse conduction through the atriums
~Emergence of refractory period in the ventricles
~Blockade of impulses in the sinus node
~The emergence of refractory period in atriums}

A patients with extensive myocardial infarction developed heart failure (shortness of breath began to intensify,
crepitation in the lings appeared). What pathogenetic mechanism underlies the development of heart failure in the
patient?
{=Decrease of the mass of functioning cardiomyocytes
~Pressure overload
~Volume overload
~Acute heart tamponade
~Reperfusion myocardial damage}

In patients with coronary cardiosclerosis atriums and ventricles contract independently — cach by its own rhythm:
atrial contraction rate is about 70, ventricles contraction rate is about 35 per 1 min. What kind of rhythm is it?
{=Idioventricular
~Atrioventricular
~Slow atrial
~Alternation
~Heart rate driver migration}

A patient suffers from tachycardia, and dyspnea at rest. He acquires a sitting position. An attempt of the patient to
get a horizontal position leads to the development of cardiac asthma attack. What type of circulatory
decompensation developed?
{=Left ventricle
~Right ventricle
~Total
~Vascular
~Extracardial}

A patient suffers from mitral valve insufficiency resulting in heart volume overload. What mechanism of urgent
compensation is the main one in heart volume overload?
{=Heterometric
~Homeometric
~Chronotropic
~Inotropic effect of catecholamines
~Compensatory myocardial hypertrophy}

A 27-year-old patient periodically developed recurrent attacks of palpitations, which lasted for several minutes.
Cardiac work is rhythmic, heart contraction rate is up to 220 beats/min. What is the type of arrhythmia?
{=Paroxysmal tachycardia
~Sinus tachycardia
~Fibrillation
~Sinus arrhythmia
~Sinus extrasystole}

A patient with myocarditis developed clinical signs of cardiogenic shock. Which of the following pathogenetic
mechanisms is the leading one?
{=Impairment of the heart pump function
~Deposition blood in the organs
~Decrease of diastolic inflow to the heart
~Decrease of vascular tone
~Increase of peripheral vessels resistance}

Arrhythmia with generation of atrial contractions up to 400 / min was observed in a patient with cardiosclerosis.
This was accompanied by reduced pulse rate comparatively to the heart rate. Impairment of what heart muscle
function is observed in this case?
{=Generation of an impulse and its conduction
~Automatism
~Generation of an impulse
~Contractile
~Conduction}

A patient with heart failure developed arrhythmia in the form of generation of carly impulses in the His bunch.
Impairment of what heart muscle function is observed in this case?
{=Generation of an impulse
~Automatism
~Conduction
~Generation of an impulse and its conduction
~Contractile}
A patient with heart failure developed arrhythmia associated with decrease of the atrial contraction frequency to
70/min, and ventricles contraction frequency to 35 / min. Impairment of what heart muscle function is observed in
this case?
{=AV conduction
~Generation of an impulse
~Automatism
~Generation of an impulse and its conduction
~Contractile}

Exstrasystole was identified in a patient. On ECG: absent P-wave, deformed QRS-complex, complete compensatory
break is present. What kind of exstrasystole is it?
{=Ventricular
~Atrial
~Atrioventricular
~Sinus
~Extracardial}

The analysis of ECG revealed: missing P-waves, instead of them - numerous f-waves, QRST-complexes go in
different intervals, RR differ, R-waves are of different voltage. Name the type of arrhythmia. {=Atrial
~Sinus
~Paroxysmal tachycardia ~Sinus
tachycardia
~Idioventricular rhythm}

One of the most severe complications of myocardial infarction is cardiogenic shock, which is manifested by which
of the following signs? {=Oliguria
~Long-lasting pain
~Dyzuriya
~Tachycardia
~Acute heart failure}

A patient has congenital mitral stenosis. Which of the following processes will develop in the patient’s heart?
{=Hypertrophy
~Hypotrophy
~Anaplasia
~Atrophy
~Regeneration}

In myocardium hypertrophy the heart muscle volume is increased due to:


{=Thickening each muscle fiber
~Overgrowth of connective tissue
~Increased number of cardiomyocytes
~Buildup of adipose tissue
~Increased blood supply of the heart}

Resorption of necrotized areas of damaged heart cells in myocardial infarction is responsible for:
{=Appearance of creatinecinase in the blood
~Increased activity of alaninaminotransferase
~Slowing down of ESR
~Leukopenia
~Decrease of free fatty acids}

A 56-year-old male addressed a doctor with complaints of pain in heart area, shortness of breath. Objective
findings: edema of the lower extremities, cyanosis of the lips. From the patient’s history: long and steady increase
of arterial pressure. What is the mechanism of heart failure in this case?
{=Overload with increased peripheral resistance
~Overload with blood volume
~Immune destruction of the myocardium
~Electrolyte-steroid cardiopathy
~Hypoxic myocardium necrosis}
Changes on ECG manifested initially by extended PQ-interval, then — by occasional loss of QRS-complexes, later -
by increasing number of missing ventricular complexes and, ultimately, - by reducing atrial contraction rate to 70
beats/min, and the ventricles contraction rate to 35 beats/min. These changes are characteristic of:
{=Atrioventricular block
~Intraatrial blockade
~Intraventricular blockade
~Arrhythmia due to automatism disorder
~Arrhythmia due to rhythm accepting impairment}

Ventricles contract before they are filled with blood and their contractions do not illicit pulse waves. In this case,
pulse rate is less than heart contraction rate (pulse deficit). This disorder of heart rhythm is called: {=Fibrillation
~Sinus fibrillation
~Extrasystole
~Ventricular fibrillation
~Blockade}

In a patient with neurocirculatory dystonia heart rate increased up to 130 beats/min. Clinical symptoms of organic
heart lesions were not found. On pressing on the carotid sinus the heart rhythm slows down for a short time, and
then is restored. What is the cause of the dysrhythmia? {=Increased impact of sympathetic autonomic nervous
system
~Fluctuations in parasympathetic tone
~Inflammatory injury of myocardium
~Ischemic myocardial damage
~Toxic injury of myocardium}

The pulse rate of a patient who sustained cerebral haemorrhage slowed down to 50/min. Pulse is rhythmical. What
is the mechanism of such a heart rhythm disorder?
{=Irritation of nervus vagus
~Replacement of glutamic acid in the globin beta chain with serine
~Influence of sympathetic mediator
~Stretching sinoatrial node zone
~Increase of slow diastolic depolarization speed}
In a patient with arterial hypertension the pulse rate during crisis fell from 72 to 52 beats / min. and holds for 10
days. Intramuscular administration of 1 mg atropine led to the pulse deceleration to 16 beats / min. Which group of
arrhythmias does the described heart disorder belong to?
{=Automatism disorder
~AYV conduction
~Impairment rate adoption
~Atrial fibrillation
~Ventricular fibrillation}

What compensatory mechanism is triggered in conditions of heart failure caused by blood volume overload?
{=Heterometric (Frank - Starling)
~Tachycardia
~Homeometric
~Hypertrophy of myocardium
~Chronoinotropic}

What compensatory mechanism is triggered in conditions of heart failure caused by increased resistance of blood
outflow?
{=Homeometric
~Heterometric (Frank - Starling)
~Bradycardia
~Hypertrophy of myocardium
~Increased frequency of breathing rate}

What form of heart failure develops with rheumatism?


{=Mixed
~Overload
~Miocardial
~Right heart
~Left heart}

What kind of compensation in heart failure develops in conditions of hypertension?


{=Hypertrophy of myocardium
~Heterometric (Frank - Starling)
~Homeometric
~Tachycardia
~The increase of minute blood volume}

A 67 year old patient complains of periodic heart ache, dyspnea during light physical activities. ECG reveals
extraordinary contractions of heart ventricles. Such arrhythmia is called: {=Extrasystole
~Bradycardia
~Tachycardia
~Flutter
~Fibrillation}

Analysis of the ECG revealed the missing of several PQRST cycles. The remaining waves and complexes are not
changed. Specify the type of arrhythmia:
{=Sinoatrial block
~Atrial fibrillation
~Atrioventricular block
~Atrial premature beat
~Intra-atrial block}

Since a patient has had myocardial infarction, atria and ventricles contract independently from each other with a
frequency of 60-70 and 35-40 per minute. Specify the type of heart block in this case: {=Complete
atrioventricular
~Partial atrioventricular
~Sino-atrial
~Intra-atrial
~Intraventricular}

A patient with extensive myocardial infarction has developed heart failure. What pathogenetic mechanism
contributed to the development of heart failure in the patient?
{=Reduction in the mass of functioning myocardiocytes
~Pressure overload
~Volume overload
~Acute cardiac tamponade
~Myocardial reperfusion injury}
After a serious psychoemotional stress a 48 year old patient suddenly developed acute heart ache irradiating to the
left arm. Nitroglycerine relieved pain after 10 minutes. What is the leading pathogenetic mechanism of this process
development?
{=Spasm of coronary arteries
~Dilatation of peripheral vessels
~Obstruction of coronary vessels
~Compression of coronary vessels
~Increase in myocardial oxygen consumption}

The patient with acute miocardial infarction was given intravenously different solutions during 8 hours with
medical dropper 1500 ml and oxygen intranasally. He died because of pulmonary edema. What caused the
pulmonary edema?
{=Volume overload of the left ventricular
~Decreased oncotic pressure due to hemodilution
~Allergic reaction
~Neurogenic reaction
~Inhalation of the oxygen}

A patient complaining of pain in the left shoulder-blade region has been diagnosed with miocardial infarction. What
kind of pain does the patient have?
{=Radiating
~Visceral
~Phantom
~Protopathic
~Epicritic}
Since a patient has had myocardial infarction, his atria and ventricles contract independently from each other with a
frequency of 60-70 and 35- 40 per minute. Specify the type of heart block in this case: {=Complete
atrioventricular
~Partial atrioventricular
~Sino-atrial
~Intra-atrial
~Intraventricular}

In a dysentery patient undergoing treatment in the contagious isolation ward, a significant increase in packedcell
volume has been observed (60%). What other value will be affected by this change? A. Increasing blood viscosity
B. Increasing volume of blood circulation C. Leukopenia D. Thrombocytopenia E. Increasing erythrocyte
sedimentation rate (ESR) 191. A patient complains of palpitation after stress. The pulse is 104 bpm, P-Q = 0,12
seconds, there are no changes of QRS complex. What type of arrhythmia does the patient have? {=Sinus
tachycardia
~Sinus bradycardia
~Sinus arrhythmia
~Ciliary arrhythmia
~Extrasystole 1}

Electrocardiogram of a young man reveals deviation of his electrical axis of heart to the left. This phenomenon can
be caused by:
{=Hypersthenic body type
~Asthenic body type
~Dilation of the right atrium
~Dilation of the night ventricle
~Dilation of the left atrium}

A 50-year-old woman diagnosed with cardiac infarction has been delivered into an intensive care ward. What enzyme
will be the most active during the first two days?
{=Aspartate aminotransferase
~Alanine aminopeptidase
~LDHs
~LDHa4
~Alanine aminotransferase}

A patient has insufficient blood supply to the kidneys, which caused the development of pressor effect due to the
constriction of arterial resistance vessels. This is the result of the vessels being greately affected by the following
substance:
{=Renin
~Angiotensinogen
~Angiotensin II
~Catecholamines
~Norepinephrine}

Experimental stimulation of sympathetic nerve branches that innervate heart caused an increase in force of heart
contractions because membrane of typical cardiomyocytes permitted an increase in: {=Calcium and potassium
ion exit
~Calcium ion entry
~Potassium ion exit
~Calcium ion exit
~Potassium ion entry}

A 67-year-old patient complains of periodic heartache, dyspnea during light physical activities. ECG reveals
extraordinary contractions of heart ventricles. Such arrhythmia is called:
{=Bradycardia
~Flutter
~Fibrillation
~Extrasystole
~Tachycardia}

A patient complains of palpitations after stress. Pulse is 104 /min., P-Q = 0,12 seconds, there are no changes in QRS
complex. What type of arrhythmia does the patient have?
{=Ciliary arrhythmia
~Extrasystole
~Sinus bradycardia
~Sinus arrhythmia
~Sinus tachycardia}

A 15-year-old teenager complains of lack of air, general weakness, palpitations. Heart rate is 130/min., BP is
100/60 mm Hg. ECG: QRS complex has normal shape and duration. The number of P waves and ventricular
complexes is equal, T wave merges with P wave. What type of cardiac arrhythmia is observed in the teenager?
{=Atrial thrill
~Paroxysmal atrial tachycardia
~Sinus extrasystole
~Atrial fibrillation
~Sinus tachycardia}

Patient’s systolic blood pressure is 90 mm Hg, diastolic - 70 mm Hg. Such blood pressure is caused by decrease of
the following factor:
{=Aortic compliance
~Pumping ability of the nght heart
~Pumping ability of the left heart
~Total peripheral resistance
~Vascular tone}

An athlete (long-distance runner) during a contest developed a case of acute cardiac insufficiency. This pathology
resulted from:
{=Pericardium pathology
~Cardiac pressure overload
~Disrupted coronary circulation
~Direct damage to myocardium
~Cardiac volume overload}
Examination of the coronary arteries revealed atherosclerotic plaques with calcinosis that close the arterial opening
by 1/3. In the muscle there are numerous whitish layers of connective tissue. Name the process detected in the
myocardium: {=Tiger heart
~Myocarditis
~Myocardial infarction
~Diffuse cardiosclerosis
~Postinfarction cardiosclerosis}

Investigation of an isolated cardiac myocyte determined that it does not generate excitation impulses automatically,
which means this cardiac myocyte was obtained from the following cardiac structure: {=Ventricle
~Atrioventricular node
~Purkinje’s fibers
~His’ bundle
~Sinoatrial node}

A woman, who has been suffering from marked hypertension for 15 years, has lately developed dyspnea,
palpitations, slightly decreased systolic pressure, while diastolic pressure remains the same. What is the main
mechanism of heart failure development in this case? {=Disorder of impulse conduction in the myocardium
~Dysregulation of cardiac function
~Cardiac overload due to increased blood volume
~Damage to the myocardium
~Cardiac overload due to increased vascular resistance}
To prevent the transplant rejection after organ transplantation it is required to administer hormonotherapy
for the purpose of immunosuppression. What hormones are used for this purpose? {=Glucocorticoids
~Mineralocorticoids
~Catecholamines
~Thyroid
~Sexual hormones}

A patient consulted an immunologist about diarrhea, weight loss within several months, low-grade fever,
enlarged lymph nodes. The doctor suspected HIV infection. What immunocompetent cells must be studied in
the first place?
{=Helper T-lymphocytes
~Suppressor T-lymphocytes
~B-lymphocytes
~Monocytes
~Plasma cells}

A 30-year-old man addressed a doctor with complaints of enlarged submandibular lymph nodes. Anamnesis
states that the patient previously had been treated for acute apical periodontitis of the 36th and 46th teeth.
Histologically the following could be detected in the removed lymph node: hyperemia, edema, increased
amount of plasmocytes and plasmablasts in the medullary area and germinal centers of follicles. What can
cause such changes in lymph nodes?
{=Antigenic stimulation
~Immunodeficiency state
~Metastasis of a malignant tumor
~Granulomatous inflammation
~Lymphoma}

A patient with acute bronchitis has been prescribed sulfanilamide drugs for treatment. In an hour after
administration the patient developed itching and vesicles filled with light transparent liquid on the face, palms
and soles. Name the mechanism of immune response:
{=Reaginic reaction
~Cell cytotoxicity

~Immune complex-mediated hypersensitivity


~Antibody-mediated cytolysis}

Often the cause of secondary immunodeficiency is an infectious affection of an organism, when agents
reproduce directly in the cells of immune system and destroy them. Specify the diseases, during which the
described above occurs:
{=Infectious mononucleosis, AIDS
~Tuberculosis, mycobacteriosis
~Poliomyelitis, viral hepatitis type A
~Dysentery, cholera
~Q fever, typhus}

Throughout a year a 37-year-old woman periodically got infectious diseases of bacterial origin, their course was
extremely lingering, remissions were short. Examination revealed low level of major classes of
immunoglobulins. The direct cause of this phenomenon may be the following cell dysfunction: {=Plasmocytes
~Phagocytes
~Neutrophils
~Macrophages
~Lymphocytes}

Differentiation of B-lymphocytes into plasma cells leads to synthesis of immunoglobulins that ensure specific
immune response of the body. Differentiation of B-lymphocytes takes place in the following organ of immune
system: {=Tonsils
~Red bone marrow
~Liver
~Thymus

Often the cause of secondary immunodeficiency is an infectious affection of an organism, when agents
reproduce directly in the cells of immune system and destroy them. Specify the diseases, during which the
described above occurs:
{=Infectious mononucleosis, AIDS
~Tuberculosis, mycobacteriosis
~Poliomyelitis, viral hepatitis type A
~Dysentery, cholera
~Q fever, typhus}

There are no reactions of delayed type hypersensitivity in mice without hairs (nude mice). The most possible
reason for this pathology is: {=Absence of thymus
~Disorders of hematopoiesis
~Defect of phagocytosis
~Deficiency of components of complement system
~Absence of gamma globulins in blood}

Transplantation of skin was performed in a patient with wide spread burns. Graft swelled and changed its color
at 8th day and was rejected at 11th. What cells participate in this process?
{=T-lymphocytes
~Basophils
~Eosinophils
~B-lymphocytes
~Erythrocytes}

Deficient content of immunoglobulins was revealed in a patient. What cells of immune system produce
immmunoglobulins?
{=Plasma cells
~T-killers
~B-lymphocytes
~T-helpers
~T-suppressors}

Three times immunization of population was carried out by pertussis-diphtheria-tetanus vaccine when the
number of diphtheria cases rose. The levels of anybodies increased in the blood of immunized people as a
consequence of immunization. What cells produce these proteins?
{=Plasmosytes
~Neutrophilic granulocytes
~Macrophages
~Monocytes
~Acidophilic granulocytes}
It is known that plasma cells produce specific antibodies against the given antigen. Number of plasma cells
increases after introduction of antigen. What cells of peripheral blood serve as precursors of plasma cells? {=B-
lymphocytes
~Neutrophils
~Basophils
~T-lymphocytes
~Eosinophils}

Formation of T-helpers is held up in thymus. What processes of immunogenesis in connective tissue will be
violated at first?
{=Conversion of B-lymphocytes to plasma cells
~Opsonization
~Phagocytosis of antigens by macrophages
~Phagocytosis of foreign entities
~Formation of precursors of T-lymphocytes}

Transplantation of donor heart was performed in a patient. What conditions have to be observed to prevent
transplant rejection?
{=Selection of donor according to HLA
~Transplantation of bone marrow
~Transfusion of donor’s blood
~Removal of spleen
~Administration of immunomodulators}

A 3-year-old boy suffering from chronic pneumonia has low indices of B-lymphocyte system. Bruton’s
hypogammaglobulinemia was diagnosed in him. What is the consequence of B-lymphocyte system deficiency?
{=Decreased resistance of organism to pyogenic coccus flora
~Decreased resistance of organism to viruses
~Decreased resistance of organism to fungous and tuberculosis infection
~Absence of graft rejection reactions
~Increased risk of tumor development in organism}

Child has congenital heart disease, face defect, absence of thyroid gland and thymus, and T-lymphocytes in blood.
What hereditary pathology are these symptoms connected with?
{=Di George syndrome ~Luis-Barr
syndrome
~Tumer’s syndrome
~Down’s disease
~Bruton’s disease}

A 2-month-old girl with pneumonia was admitted to pediatric department. She bore otitis and pneumonia a
month ago. Dicrease in levels of gamma-globulins by 2 times was found at her examination. Which disorder of
immune system is it?
{=Physiological hypogammaglobulinemia
~Luis-Barr syndrome
~Late hypogammaglobulinemia
~Swiss type of immune deficiency
~Di George syndrome}
A 2-year-old boy has frequent bacterial infections since 2 months. Allergic reaction to tuberculin (IV type) is
positive. Which immunodeficiency is more possible at this patient?
{=Congenital B-lymphocytes insufficiency
~Congenital T-lymphocytes insufficiency
~Congenital T-suppressors defect
~Congenital total immunodeficiency
~Acquired immunodeficiency}

Association of staphylococcus aureus and staphylococcus epidermidis are found in numerous skin pustules at
25-year-old patient. Pneumocysta carinia is found in his sputum analysis. Cryptosporidia, proteus vulgaris and
fungi of candida genus are found in his stool. Which disease is accompanied by such multiple infections with
conditionally-pathogenic microorganisms?
{=AIDS
~Dysbacteriosis
~Sepsis
~Diabetes mellitus
~Drag agranulocytosis}

A 25-year-old man complains of frequent inflammation diseases of different localization. He is injection drug
addict. HIV-test was positive at him. What types of cells are damaged more substantially in case of HIV? {=T-
helpers
~Plasmocytes
~Macrofages
~Neutrophilic granulocytes
~NK-cells}

A 12-year-old boy with hereditary disease has eczema, 4 cases of pneumonia in his anamnesis, bleedings. Low
level of T-lymphocytes, decreased levels of IgM, normal levels of IgA was found in his blood. What disease
does the boy have?
{=Wiskott-Aldrich syndrome
~Luis-Barr syndrome
~Bruton’s disease
~Late hypogammaglobulinemia
~Early hypogammaglobulinemia}

A newborn has recurring respiratory tract diseases, pustule affections of the skin, severe clinical course of
herpes infection and recurring candidiasis (thrush). Inoculation against smallpox and BCG-vaccination was
accompanied by severe complication in him. All classes of immunoglobulins were decreased in his
immunological test. Delayed-type hypersensitivity reactions were absent in him. What cells insufficiency
underlies this immunodeficiency syndrome?
{=T- and B-lymphocytes
~Neutrophils
~Macrophages
~Fibroblasts
~T-suppressors}

A 1-year-old boy becomes very often ill with respiratory and pustular skin diseases. Even little damages
become complicated by long-time purulent inflammation. All classes of immunoglobulins are nearly
absent from the child’s blood. Contents of all leukocyte types and reactions of delayed-type
hypersensitivity are normal. What cells inherited decrease in functional activity underlies this syndrome?
{=B-lymphocytes
~T-lymphocytes
~Neutrophils
~Eosinophils
~Endoteliocytes}

Decrease in levels of IgG and particularly IgA, IgM was found during examination of immune status in 5-
yearold boy. B-lymphocytes and plasma cells are absent from his blood and lymph nodes. Reactions of
Tlymphocytes are normal. This is inherited sex-linked disease. What diagnosis is more possible?
{=Bruton’s disease
~Luis-Barr syndrome
~Wiskott-Aldrich syndrome
~Swiss type of immunodeficiency
~Early hypogammaglobulinemia}

A 9-year-old girl was getting breast feeding during the first year of life. She suffered prolonged severe
pneumonia in the end of the first year. She started walking late. Her gait was unsteady and her movements were
discoordinated. Telangiectasia appeared in her skin and conjunctivas. IgA is absent from and levels of
Tlymphocytes is decreased in her blood. What immunodeficiency disease does the girl have?
{=Luis-Barr syndrome
~Wiskott-Aldrich syndrome ~Chediak-Higashi
syndrome
~Swiss type of immunodeficiency
~Di George syndrome}

Immune system is depressed in a patient with HIV-infection. What cells damage contributes more to
immunodeficiency at this patient?
{=T-helpers
~Macrophages
~T-supressors
~B-lymphocytes
~T-killers}

A 2-year-old patient suffering from chronic pneumonia has decrease in quantity of B-lymphocytes, IgM and IgG.
What microorganisms can cause development of this disease?
{=Staphylococcus
~Mycobacterium tuberculosis
~Herpes virus
~Adenovirus
~Candidas}

Disorder of cellular immunity was revealed during investigation of immune system in the patient with chronic
fungus affection of the skin. What indexes decrease more typically in this case? {=T-lymphocytes
~IgE
~IgG
~B-lymphocytes
~Plasma cells}
Mantoux test was made to sick child on suspicion of tuberculosis. Swelling, hyperemia and painfulness occur in
the allergen injection place in 24 hours. What components determinate this reaction of organism?
{=Mononuclear cells, T-lymphocytes and lymphokines
~B-lymphocytes and IgM
~Plasma cells, T-lymphocytes and lymphokines ~Granulocytes,
T-lymphocytes and IgG
~Macrophages, B-lymphocytes and monocytes}

A patient was suffering from chronic renal insufficiency. He has renal transplantation from donor, who died in
motorcar accident. Donor had blood group same as recipient. Renal insufficiency occurs in patient again after a
time. Which system incompatibility takes part in transplant rejection?
{=HLA
~RH
~MNS
~ABO
~Kel-Chelano}

HIV-infection was suspected at a patient of dental clinics. What cells are damaged at first in patient with AIDS?
{=T-helpers
~T-suppressors
~T-killers
~B-lymphocytes
~Macrophages }

A 10-year-old girl suffers from viral and mycotic diseases very often. She has congenital heart disease and
insufficiency of thyroid gland. T-lymphocytes is absent from her immunological test. What disorder of
immune system takes place in this case? {=Hypoplasia of thymus
~Mixed immunodeficiency
~Terner’s syndrome
~Bruton's agammaglobulinemia
~Inherited defect of complement system}

Pneumonias and pyodermas very often occur in 5-year-old boy, as well as in his grandfather. B-lymphocytes are
absent from their immunological test. What disorder of immune system takes place in this case?
{=Bruton’s hypogammaglobulinemia
~Hypoplasia of thymus
~Mixed immunodeficiency
~Terner’s syndrome
~Inherited defect of complement system}

Viral and bacterial infections, eczema often occur in a 12-year-old boy. Decreased quantity of T-lymphocytes
and IgM and normal levels of IgG and IgA are found in him at examination. What kind of immune pathology is
observed in this case? {=Mixed immunodeficiency
~Hypoplasia of thymus
~Bruton’s hypogammaglobulinemia
~Terner’s syndrome
~Inherited defect of complement system}
A 20-year-old woman with intestinal polyposis has frequent mycotic and viral diseases in her anamnesis. What
link of immune system is insufficient more possibly in this case?
{=T-lymphocytes
~B-lymphocytes
~Natural killers ~Complement
~Phagocytosis}

Which of following is immunodeficiency disease? {=Inherited


disorder of immune system
~Decrease of immune reactions under avitaminosis
~Insufficiency of immune reactions after exposure to ionizing radiation
~Disorder of formation antibodies as consequences of protein starvation ~Decrease
in immune reactions under cytostatic drugs action}

A Patient needs renal transplantation. What kind of transplantation more expedient? {=Syngenic
~Xenogenic
~Autogenic
~Allogenic
~Explantation}

Heart transplantation is indicated for the patient. What kind of transplantation more expedient? {=Allogenic
~Xenogenic
~Autogenic
~Syngenic
~Explantation}

The main role in pathogenesis of diseases, which occur in case of B-lymphocytes immunodeficiency, plays:
{=Immunoglobulin synthesis disorders
~Cellular type immune reaction disorders
~Loss of ability to graft rejection
~Decrease in delayed-type hypersensitivity
~Decrease in antitumor immunity}

A newborn has convulsive syndrome and heart interventricular septum defect. Thymic hypoplasia was revealed
by X-ray examination of his chest. What immunodeficiency may be supposed in this patient? {=Di George
syndrome
~Bruton’s disease
~Wiskott-Aldrich syndrome
~Good’s syndrome
~Luis-Barr syndrome}

A 15-year-old patient was admitted to allergologic department with bronchial asthma. What
immunoglobulins excessive production can result in main clinical symptoms development in this patient?
{=IgE
~IgA
~IgG
~IgM
~IgD}
In crisis period a 14 year old child ill with diphtheria has AP- 70/50 mm Hg accompanied by abrupt fall in
temperature and tachycardia. What form of vascular tone disturbance is it?
{=Acute hypotension
~Chronic hypotension
~Vegetovascular dystonia
~Essential arterial hypotension
~}
A patient has the following diagnosis: renal hypertension. What is the initial pathogenetic factor of arterial
hypertension development in this case?
{=Renal ischemia
~Hypernatremia
~Intensified renin synthesis
~Intensified angiotensin synthesis
~Hyperaldosteronism}

Rabbits lived on food with addition of cholesterol. Five months later the atherosclerotic aorta changes were revealed.
Name the main cause of atherogenesis in this case:
{=Exogenous hypercholesterolemia
~Overeating
~Endogenous hypercholesterolemia

~Hypodynamia}

18-year-old patient complains of general weakness, fatigue, low spirits. The patient is of the asthenic constitution
type. Ps - 68/min., AP - 90/60 mm Hg. She has been found to have primary neurocirculatory hypotension. What is
the leading factor of the arterial pressure drop in this patient?
{=Decreased cardiac output
~Decreased tonus of resistive vessels
~Hypovolemia
~Decreased minute blood volume
~Deposition of blood in the veins of the systemic circulation}

18-year-old patient complains of general weakness, fatigue, low spirits. The patient is of the asthenic constitution
type. Ps - 68/min., AP - 90/60 mm Hg. She has been found to have primary neurocirculatory hypotension. What is
the leading factor of the arterial pressure drop in this patient?
{=Decreased tonus of resistive vessels
~Decreased cardiac output
~Hypovolemia
~Decreased minute blood volume
~Deposition of blood in the veins of the systemic circulation}

A 67-year-old patient has atherosclerosis of cardiac and cerebral vessels. Examination revealed hyperlipidemia.
What class of blood plasma lipoproteids is most important in atherosclerosis pathogenesis? {=Low-density
lipoproteids
~Chylomicrons
~a-lipoproteids
~High-density lipoproteids
~—}
A 5-year-old boy with congenital pulmonary artery stenosis while walking developed dyspnea, cyanosis appeared
on his face and finally he lost consciousness. What is the primary mechanism of development of this condition?
{=Acute hypoxia of the brain
~Expansion of peripheral vessels
~Decrease of blood pressure
~Disorder of pulmonary ventilation
~Disorder of gases diffusion in the lungs}

A 55-year-old woman suffers from renal failure and has blood pressure 170/100 mm Hg. Excessive activation of
which of the following systems determines constant increase of blood pressure in the woman? {=Renin-
angiotensin-aldosterone
~Simpatico-adrenal
~Hypothalamus-pituitary
~CNS
~Kalikrein-kinin system}

A 58-year-old patient appealed to a doctor with complaints of the steady increase of arterial blood pressure. On
clinical examination of the patient chronic kidney disease with impairment of renal blood circulation was found.
Activation of which of the mechanisms led to development of increased blood pressure in the man? {=Renin-
angiotensin-aldosterone
~Parasy mpathetic nervous system
~Sympathetic nervous system
~Cardiovascular
~Metasympathetic nervous system}

What mechanism is responsible for the development of hypertension in a patient with renal insufficiency? {=Increase
the renin secretion
~Reduction of water reabsorption
~Reduction of electrolytes reabsorption
~Increase of glucose reabsorption
~Proteinuria}

According to Pauzel’s formula: P = QR, where P it’s a fluid pressure in the pipe, Q - value of their filling and R -
resistance of fluid flow. What hemodynamic type of arterial hypertension the increase of as Q so R is characteristic
of?
{=Eukinetic
~Hyperkinetic
~Hipokinetic
~Mixed
~For all of these types}
A 24-year-old female was admitted to the hospital with complaints of headaches, pain in lower back and general
weakness. A month before the patient sustained tonsillitis. Objectively: blood pressure180/110 mm Hg, peripheral
blood analysis: Er - 3.1x10!7/1, Le - 12,6x10°/1, ESR - 28 mm/hr. Urine analysis demonstrates pronounced
proteinuria, hematuria, leukocyteuria. What is the mechanism of development of hypertension in this case?
{=Renal
~Mineralocorticoid
~Renovascular
~Reflexogenous
~Aldosterone}

In a patient with hypertension an increased concentration of angiotensin II in the blood has been detected. What is
the hypertensive effect of angiotensin associated with?
{=Muscle arterioles contraction
~Activation of biogenic amines synthesis
~Excessive prostaglandins synthesis
~Stimulation of vasopressin production
~Activation of kalikrein-kinin system}

In a patient with hypertension an increased concentration of vasopressin in the blood has been detected. Which organ
is the target of this hormone?
{=Kidneys
~Liver
~Heart
~Lungs
~Adrenals}

Arterial hypertension due to artificial narrowing of the renal arteries has been made in a dog. The increase of
reninangiotensin-aldosterone system activity has been observed. What component of this system has most
powerful hypertensive effect?
{=Angiotensin II
~Renin
~Angiotensin I
~Vasopressin
~Oxytocin}

A 65-year-old man has been suffering from marked arterial hypertension for 15 years. Recently his systolic
pressure began to decrease but diastolic remained unchanged. What is haemodynamic type of hypertension in the
patient?
{=Hypokinetic
~Normokinetic
~Hyperkinetic
~Eukinetic
~Akinetic}

A 47-year-old man has high cholesterol levels in the blood. Which vessels are the primary targets for
cholesterol deposition? {=Arteries
~Arteriols
~Capillaries
~Venules
~Veins}

A patient is experiencing high blood pressure, headaches. It was established that this pathological state is related to
a kidney disease. Which substance can cause this?
{=Renin
~Creatine
~NH4*
~Urea
~Histamine}

A patient complains of headache. He has high blood pressure and angiotensin levels in the blood. What exactly
causes this?
{=Renin
~Creatine
~NH4*
~Urea
~Histamine}

Routine medical examinations of a 36-year-old man, who is a driver by profession, determined blood pressure
150/90 mm Hg. The patient complains of tinnitus and weakness at the end of his working day, the symptoms
disappears after rest. Hypertension was diagnosed. What is the leading pathogenetic mechanism in this disease?
{=Nervous
~Renal
~Humoral
~Endocrine
~Reflexogenous}

In a patient, who suffers from hypertension, daily fluctuations in peripheral vascular resistance to blood flow were
determined. What type of vessels are responsible for the vascular resistance? {=Arterioles
~Aorta
~Capillaries
~Arteriolar bypass
~Veins}

A 82-year-old patient has been admitted to a hospital with systolic and diastolic blood pressure 195 mm Hg.
and 115 mm Hg respectively. What is the most important index for determining atherosclerosis? {=Pulse wave
velocity
~Arterial pressure
~Cardiac output
~Blood viscosity
~Volume of circulating blood}

A 39-year-old woman with a poor health condition preceding comma due to chronic renal failure was admitted to
the kidney center. Objective findings: edema of the legs and face; congestive and enlarged liver. Blood pressure
190/120 mm Hg. What is the primary pathogenic mechanisms that most likely led to the development of
hypertension in this patient?
{=Increased renin secretion
~Increased glucocorticoids secretion ~Increased
aldosterone secretion
~Increased secretion catecholamines
~Increased concentration of angiotensin in the blood serum}

A 54-year-old man suffering from atherosclerosis, ischemic heart disease, and angina has been hospitalized to
the cardiological department. Increased lipids levels were found on laboratory examination of the patient’s blood
serum. What class of plasma lipids performs a central role in the pathogenesis of atherosclerosis? {=Low
density lipoproteins
~Hilomicrons
~Alpha-lipoproteins
~High density lipoproteins
~Complexes of fatty acids with albumin}

In an experimental animal one kidney was removed, and a ligature narrowing conralateral kidney artery was put.
What type of secondary hypertension appeared?
{=Renovascular
~Metabolic
~Endocrine
~Neurogenic
~Anhiogenic}

A patient was diagnosed with renal hypertension. Name the initial pathogenetic factor in the development of
hypertension in this case?
{=Renal ischemia
~Hypernatriemia
~Hyperaldosteronism
~Increased renin synthesis
~Increased angiotensin synthesis}

A 65-year-old female suffers from atherosclerosis of brain vessels. Laboratory examination discovered
hyperlipidemia. Increase of what class of plasma lipoproteins has been detected in this biochemical study? {=Low
density lipoproteins
~Alpha-lipoproteins
~Hilomicrons
~High density lipoproteins
~Complexes of fatty acids with albumin}

Nowadays it has been proved that not native but modified lipoproteins play a central role in the development of
atherosclerosis. Capture and accumulation of modified complexes in the vascular wall is accomplished by:
{=Monocytes
~Fibroblast
~Endothelial cells
~Myocytes
~Neutrophils}

Rabbits were fed with foods supplemented with cholesterol. Atherosclerotic changes in aorta were revealed after 3
months. What is the main cause of atherosclerosis in this case?
{=Exogenous hypercholesterolemia
~Overeating
~Stress
~Hypodynamia
~Endogenous hypercholesterolemia}

A 35-year-old patient has been diagnosed with arterial hypertension. What is characteristic of primary pulmonary
hypertension?
{=Dyspnea and cyanosis
~Edema of lower extremities
~Paroxysmal tachycardia
~Cough
~Bradycardia}
-ycar-

A47_ old patient has been diagnosed with atherosclerosis. What is the initial moment in the pathogenesis of
atherosclerosis?
{=Excessive deposition of plasma lipoproteins in the artery’s intima
~Slowdown of blood flow
~Proliferative-degenerative changes of endarterium
~Formation on the inner shell artery fibrous plaques
~Impairment of the arterial walls integrity}

A 62-year-old patient has been treated for arterial hypertension for 20 years. Recently he began to feel dizziness
and twinkling in the eyes. What is the most likely complication of hypertensive disease one should beware of?
{=Stroke
~Myocardial infarction
~Renal insufficiency
~Pulmonary hypertension
~Shock}

An autopsy revealed atherosclerotic changes in aorta and arteries of flexible and muscular-elastic types. Which
functional type do these vessels belong to?
{=Compensatory vessels
~Resistant vessels
~Shunt vessels
~Vessels of exchange
~Reservoir vessels}

A patient suffers from hypertensive disease. What hemodynamic type of hypertension does essential
hypertension belong to? {=Hypokinetic
~Hyperkinetic
~Eukinetic
~Not to any of these types
~Not any of these types}

A 54-year-old man has been diagnosed with hypertension. Which type of hypertension can be this nosological unit
attributed to?
{=Essential
~Due to renovascular disorders
~Due to disorders in mineralocorticoids metabolism
~Due to salt disorders}

A 56-year-old man has been affected by atherosclerosis of aorta, coronary arteries, cerebral arteries, and arteries of
lower limbs. Due to decreased elasticity these vessels:
{=Can not adequately alter their diameter according to tissues’ needs
~Can be obstructed by an exogenous embolus
~Can be obstructed by an endogenous embolus
~Can undergo spasm
~May be replaced with connective tissue}

The first stage in the pathogenesis of atherosclerosis is:


{=Excessive deposition of plasma lipoproteins in the inner membrane of arteries
-ycar-

~Slowdown of blood flow


~Proliferative degenerative changes of the intima ~Formation
of fibrous plaques on the inner artery’s wall
~Disturbance of the arterial wall integrity}
A55 old man has developed impairment of functional state of cortical and subcortical centers regulating
vascular tone. Hypertension, resulting from bilateral cerebral ischemia can be attributed to: {=Central
ischemic
~Reflexogenic
~Renovascular
~Due to disorders in glucocorticoids metabolism
~Due to disorders in mineralocorticoids metabolism}

A patient due to impairment of renal circulation developed arterial hypertension. What has a direct impact on the
precapillary vascular wall?
{=Angiotensin II
~Angiotensin
~Renin
~Aldosterone
~Angiotensynase}

A 45-year-old patient developed circulatory failure by right heart type. What type of hypertension developed in this
case?
{=Pulmonary hypertension
~Central ischemic hypertension
~Essential hypertension
~Reflexogenic hypertension
~Salt hypertension}

Following a 38-year-old woman’s family history it has been revealed that the patient has an inherent predilection to
atherosclerosis due to high incidence of the disease in her family members. Atherosclerotic changes in the woman
began to appear 2 years ago when a routine hysterectomy and removal of ovaries were performed. Which of the
following etiological factors is responsible in the development of atherosclerosis in this woman? {=Insufficiency
of sex glands function
~Hereditary factor
~Female gender
~Age of the patient
~High content of fats on her diet}

A 56-year-old woman weighing 120 kg developed constant pain in the heat region, that is a consequence of
atherosclerotic changes in coronary vessels. Increased levels of which substances in the blood is important in the
pathogenesis of atherosclerosis?
{=Low density lipoproteins
~Alpha- lipoproteins
~High density lipoproteins
~Hilomicrons
~Fatty acids}
-ycar-

A 56-year-old patient suddenly felt a sever retrosternal pain irradiating to the left hand, accompanied with a fear of
death. Face became pale coated with cold sweat. Nitroglycerin did not alleviate the pain. The diagnosis of
myocardium infarction was established. What is the most frequent cause of myocardial ischemia?
{=Atherosclerosis of coronary arteries
~Spasm of coronary arteries
~Thrombosis of heart vessels
~Coronary embolism
~Aggregation of erythrocytes in coronary vessels}

A 42 old man with chronic heart failure developed edema of the lower extremities. The main role in the
development of edema is performed by: {=Increased hydrostatic pressure
~Increased plasma oncotic pressure
~Increased plasma osmotic pressure
~Reduction of hydrostatic pressure
~Urorrhagia (polyuria)}

A 52-year-old patient died of myocardial infarction, that was confirmed by clinical and electrocardiographic
examinations; nevertheless autopsy did not found any evidence of sclerotic changes in coronary vessels. What was
the probable cause of myocardial infarction in this patient?
{=Increased secretion of catecholamines
~Increased sympathetic tone of the autonomic nervous system
~Obturation of coronary vessels by an embolus
~Coronary thrombosis
~Rheumatoid coronaritis}

A 46-year-old patient with ischemic heart disease on the basis of arteriosclerotic lesions of coronary arteries
after considerable physical exertion developed acute myocardial infarction. What is the most likely cause of
coronary circulation failure in this case? {=Coronary artery stenosis
~Embolism of coronary arteries
~Coronary artery thromboembolism
~Rupture of coronary artery
~The blood redistribution}

A 54-year-old female experiences cardiac arrhythmia with generation of cardiac contractions up to 400 b/min.
Pulse rate is lower than the heart rate, suggesting disorders in excitability and conduction. What pathology
may cause such changes? {=Cardiosclerosis
~Coronarosclerosis
~Coronary embolism
~Coronary stenosis
~Coronary thrombosis}

What is the main cause of hypertension in diabetes mellitus?


{=Atherosclerosis
~Renal failure
~Heart failure
~Lack of insulin
~Hyperglycemia}
-ycar-

A 43-year-old patient with pheochromocytoma (adrenal medullar tumor producing epinephrine) developed
elevated blood pressure. What type of hypertension can this blood pressure change be attributed to? {=Secondary
~Primary
~Essential
~Compensated
~Tertiary }

A6l1_ old female developed general adynamy, fatigue, shortness of breath, dizziness, and headaches. Arterial
pressure is decreased. What is the cause of the symptoms described?
{=Hypoxemia
~Heart failure
~Vascular insufficiency
~Respiratory failure
~Intoxication}

After acute blood loss a patient developed hypotension. What is the cause of lowering blood pressure? {=Anemia
~Vasodilation
~Increased tone of parasympathetic nervous system
~Decreased tone of parasympathetic nervous system
~Loss of blood proteins}

A 34-year-old patients has blood pressure 140/100 mm Hg. What is the pulse pressure in the patient?
{=40
~100
~140
~240
~70}

In response to a change in body position from horizontal to vertical blood circulation system develops reflectory
pressor reaction. Which of the following is its compulsory component?
{=Systemic constriction of the venous vessels
~Systemic dilatation of the arterial resistive vessels
~Decrease in the circulating blood volume
~Increase in the heart rate
~Weakening of the pumbing ability of heart}

A month after surgical constriction of rabbit’s renal artery the considerable increase of systematic arterial pressure
was observed. What of the following regulation mechanisms caused the animal’s pressure change? {=Angiotensin-
II
~Vasopressin
~Adrenaline
~Noradrenaline
~Serotonin}

A patient with constant headaches, pain in the occipital region, tinnitus, dizziness has been admitted to the
cardiology department. Objectively: AP- 180/110 mm Hg, heart rate - 95/min. Radiographically, there is a
stenosis of one of the renal arteries. Hypertensive condition in this patient has been caused by the activation of the
following system: {=Renin-angiotensin
-y €ar-

~Hemostatic
~Sympathoadrenal
~Kinin
~Immune}
A patient underwent a course of treatment for atherosclerosis. Laboratory tests revealed an increase in the
antiatherogenic lipoprotein fraction in the blood plasma. The treatment efficacy is confirmed by the increase in:
{=HDL
~VLDL
~IDL
~LDL
~Chylomicrons}

A 12-year-old patient was found to have blood serum cholesterol at the rate of 25 mmol/l. The boy has a history of
hereditary familial hypercholesterolemia, which is caused by the impaired synthesis of the following protein
receptors:
{=Low density lipoproteins
~High density lipoproteins
~Chylomicrons
~Very low density lipoproteins
~Intermediate density lipoproteins}

A 43-year-old-patient has arterial hypertension caused by an increase in cardiac output and general peripheral
resistance. Specify the variant of hemodynamic development of arterial hypertension in the given case:
{=Eukinetic
~Hyperkinetic
~Hypokinetic
~Combined
}
A patient has insufficient blood supply to the kidneys, which has caused the development of pressor effect due
to the constriction of arterial resistance vessels. This is the result of the vessels being greately affected by the
following substance: {=Angiotensin II
~Angiotensinogen
~Renin
~Catecholamines
~}
A 43-year-old-patient has arterial hypertension caused by increase in cardiac output and general peripheral
resistance. Specify the variant of hemodynamic development of arterial hypertension in the given case:
{=Eukinetic
~Hyperkinetic
~Hypokinetic
~Combined
}
A patient has insufficient blood supply to the kidneys, which has caused the development of pressor effect due
to constriction of arterial resistance vessels. This condition results from the vessels being strongly affected by
the following substance: {=Angiotensin II
~Angiotensinogen
~Renin
~Catecholamines
~Norepinephrine}

A 46-year-old reports reduced exercise tolerance over the past 5 years. In the past year he has noted chest pain
after ascending a flight of stairs. He smokes 2 packs of cigarettes per day. On examination he has a blood pressure
of 155/95 mm Hg. His body mass index is 30. Laboratory findings include a total serum cholesterol of 245 mg/dL
with an HDL cholesterol of 22 mg/dL. Which of the following vascular abnormalities is he most likely to have?
{=Atherosclerosis
~Lymphedema
~Hyperplastic arteriolosclerosis
~Deep venous thrombosis
~Medial calcific sclerosis}

Autopsy of a 40-year-old woman, who died of cerebral hemorrhage during hypertensic crisis, revealed: upperbody
obesity, hypertrichosis, hirsutism, stretchmarks on the skin of thighs and abdomen. Pituitary basophil adenoma is
detected in the anterior lobe. What diagnosis is the most likely?
{=Alimentary obesity
~Essential hypertension
~Cushing’s disease
~Simmonds’ disease
~Hypothalamic obesity}

Cholesterol content in blood serum of a 12-year-old boy is 25 mmol/l. Anamnesis states hereditary familial
hypercholesterolemia caused by synthesis disruption of receptor-related proteins for. {=Low-density
lipoproteins
~Chylomicrons
~Middle-density lipoproteins
~Very low-density lipoproteins
~High-density lipoproteins}

Due to blood loss the circulating blood volume of a patient decreased. How will it affect the blood pressure in this
patient?
{=Only systolic pressure will decrease
~Systolic pressure will decrease, while diastolic will increase
~Diastolic pressure will decrease, while systolic will increase
~Systolic and diastolic pressure will decrease
~Only diastolic pressure will decrease}

A short-term physical load resulted in reflex increase of heart rate and systemic arterial pressure in a person. What
receptor activation was the most contributory to inducing the pressor reflex?
{=Vascular chemoreceptors
~Vascular baroreceptors
~Hypothalamic thermoreceptors
~Proprioceptors of the working muscles
~Vascular volume receptors}
Skin samples of a patient with bronchial asthma revealed allergen sensitization of poplar fuzz. What factor of
immune system plays the main part in development of this immunopathological state? {=IgE
~IgD
~IgM
~Sensitized T-lymphocytes
~}
A teenger had his tooth extracted under novocain anaesthesia. 10 minutes later he presented with skin pallor,
dyspnea, hypotension. When this reaction is developed and the allergen achieves tissue basophils, it reacts with:
{=IgE
~IgA
~IgM
~T-lymphocytes
~IgD}

Surgeon used novocaine as an anaesthetic during surgical manipulations. 10 minutes after it the patient became
pale, he got dyspnea and hypotension. What type of allergic reaction is it? {=Anaphylactic
~Immune complex
~Cytotoxic
~Stimulating
~Cell-mediated}

A 10-year-old child cut his leg with a piece of glass and was sent to a clinic for an anti-tetanus serum injection.
In order to prevent the development of anaphylactic shock, the Besredka desensitization method was applied.
What mechanism underlies this method?
{=Binding to IgE fixed to mast cells
~Inhibited synthesis of mast cells mediators
~Stimulation of the immunological antigen tolerance
~Stimulation of antigen-specific IgG2 synthesis
~Binding of IgE receptors on mast cells}

During anesthesia of the oral mucosa a 37-year-old patient has had anaphylactic reaction (widespread
vasodilation, increased vascular permeability with liquid exiting the blood vessels and penetrating in the
tissues). What type of hypersensitivity reaction occurred in the patient? {=Type I (anaphylactic)
~Type II (antibody-dependent)
~Type IIT Gmmune complex)
~Type IV (cell cytotoxicity)
~Type V (granulomatosis)}

Pain in the heart and joints and pneumonia appeared in a patient three weeks later acute myocardial infarction.
What is the main mechanism of development of post-infarction Dressler’s syndrome?
{=Autoimmune inflammation
~Resorption of enzymes from necrotized area of myocardium
~Secondary infection
~Thrombosis of vessels
~Ischemia of myocardium}
A patient addressed to a dentist with complaints of redness and edema of mucous membrane of his mouth a
month later dental prosthesis. Allergic stomatitis was diagnosed in this patient. What type of allergic reaction
by Gell and Cumbs underlies this disease?
{=Delayed-type hypersensitivity
~Cytotoxic
~Immune complex-mediated
~Anaphy lactic
~Stimulating}

Anaphylactic shock developed in a patient with botulism after second injection of antitoxic antibotulinus serum
mixture. What is the main mechanism of anaphylaxis?
{=Interaction of antigen with IgE
~Interaction of antigen with IgM
~Interaction of macrophages with antigens
~Interaction of T-lymphocytes with mediators
~Interaction of T-lymphocytes with tissue basophils}

In a 27-years-old man tuberculin test was carried out. Following was observed 24 hours later: infiltration with
size of 40x35 mm at the site of injection and hyperemia of skin above it. What group of biologic active
substances causes development of allergic inflammation in this patient?
{=Lymphokines
~Biogenic amines
~Prostaglandins
~Leukotriens
~Kinins}

Purulent endometritis developed in a woman after delivery. Treating with antibiotics - inhibitors of murein
synthesis was ineffective. Wide spectrum bactericidal antibiotic was administered to her. 6 hours later
temperature rapidly increased up to 40°C with shivering, pains in muscles appeared, BP dropped down to 70/40
mmHg, and oligura developed in this woman. What is the main reason for this condition development?
{=Endotoxic shock
~Toxic effect of preparation
~Internal bleeding
~Anaphylactic shock
~Bacteremia}

A 24-years-old patient has edema of face and increase in BP, which appeared 1.5 weeks later severe
streptococcus tonsillitis. The patient has hematuria and proteinuria of 1.2 g/L. Anti-streptococcus antibodies
and decrease in content of compliment system components were revealed in patient’s blood. Which
microvessels do deposits of immune complexes localize in and cause nephropathy?
{=Glomerule
~Proximal tubules
~Descendent tubules
~Loop of Henle
~Pyramids}

Nausea, fatigue, stomachache, palpitation, difficult respiration, and skin blisters developed in a patient 25
minutes later injection of antibiotics. What stage of allergic reaction is observed in this patient?
{=Pathophysiological
~Biochemical
~Pathocemical
~Immunological
~Sensibilization}

Skin tuberculin test was carried out in a patient with chronic lung tuberculosis. Local hyperemia and edema
appeared in the site of inracutaneous introduction of tuberculin preparation within 24-48 hours. What cells are
primary effectors in mechanism of this reaction?
{=T-lymphocytes
~Neutrophils
~B-lymphocytes
~Endotheliocytes
~Smooth muscle cells of microvessels}

Hyperemia, swelling and then necrosis of tissue, their rejection and ulcer (Arthus phenomenon) develop at the
rabbit in the place of secondary intracutaneus injection of a substance with strongly pronounced antigenic
properties (for example horse serum). What factors play the main role in pathogenesis of this phenomenon?
{=Antibodies presented by IgG and/or Ig M
~Antibodies presented by IgD
~Antibodies presented by IgA
~Antibodies presented by IgE
~Specific T-lymphocytes-effectors}

Skin rash, itching, swelling and pain in joints, increase in body temperature, and proteinuria appeared in a
patient in 5-8 days after use lots of medical serum. Serum sickness was diagnosed. What is the main factor in
pathogenesis of this syndrome?
{=Primary systemic accumulation of circulating immune complexes in the blood
~Primary systemic degranulation of mast cells in the organism
~Primary systemic activation of T-killers
~Primary systemic activation of endoteliocytes
~Primary systemic cytolysis of blood cells}

Allergic diagnostic tests are used for the diagnosis of many infectious diseases (tuberculosis, brucelosis,
tularemia etc). Diagnosis is confirmed if papula and redness appear in the place of the allergen injection.
Antigens interaction reaction is conditioned by:
{=T-lymphocytes and lymphokines
~IgM and macrophages
~IgE and lymphokines
~IgE and T-lymphocytes
~IgM and tissue basophiles}

Catarrhal inflammation of bulbar conjunctiva and nose mucous membrane develop in patient every year in
spring and early summer, when trees and flowers are in blossom. Production of specific antibodies to
pollen underlies this syndrome. What cells activate and develop exocytosis in this syndrome? {=Mast cells
~Macrophages
~Lymphocytes
~Neutrophils
~Throbocytes}
An 18-year-old man with shoulder phlegmon got intramuscular injection of penicillin. Tachycardia, thread-like
pulse; decrease in BP down to 80/60 mmHg occur after that. What kind of pharmacologic reaction develops?
{=Anaphy laxis
~Reflex action
~Central action
~Potentiation
~Peripheral action}

Patient has been wearing removable dental prosthesis for 7 days. Soreness, swelling and hyperemia of
prosthesis bed appear in the patient after that. Inflammation subsides after prosthesis taking off and
develops again after prosthesis putting on. What process underlies the inflammation development?
{=Resorption
~Hypersalivation
~Hyposalivation
~Mechanical irritation
~Secretion}

Thyrotoxicosis was diagnosed in a patient. Antithyroid antibodies were found in his blood. Which type of allergic
reaction is observed at development of this disease?
{=Stimulating
~Immune complex-mediated
~Anaphy lactic
~Cytotoxic
~Delayed type hypersensitivity}

Hives, itching of the skin, swelling of the skin and mucous membranes, swelling of lymphatic nodes develop in
the patient in 9 days after injection of medicinal serum. What disease develops? {=Pollinosis
~Serum sickness
~Shwartzman’s phenomenon
~Overy phenomenon
~Quincke’s edema}

Dressler’s syndrome was diagnosed at the patient 1.5 month later myocardium infarction. It is characterized by
pericarditis, pleurisy, and pneumonia. What is the reason for this syndrome?
{=Sensitization of the organism by myocardium antigens
~Decrease in resistance to microorganisms
~Activation of saprophytic microflora
~Intoxication of organism by products of necrosis
~Release of myocardial enzymes to the blood}

It is known that bronchial asthma develops by mechanism of immediate hypersensitivity, which includes 3
sequential stages:
{=Immunological, pathochemical, pathophisiological
~Pathochemical, pathophisiological, immunological
~Pathochemical, immunological, pathophisiological
~Pathophisiological, immunological, pathochemical
~Pathophisiological, pathochemical, immunological}

A 20-year-old man has injury of the right testicle. What danger does it brings for the left (healthy) testicle?
{=Mimicry of antigens and development of antibody -mediated damage
~Development of infectious process
~Development of atrophy
~Development of hypertrophy
~No danger}

Guinea-pig’s nephrocytotoxic serum was injected to the rabbit under the experiment. What human disease is
modeled in this case?
{=Acute diffuse glomerulonephritis
~Acute pyelonephritis
~Chronic renal insufficiency
~Nephrotic syndrome
~Chronic pyelonephritis}

Acute glomerulonephritis appeared in the patient 2 weeks later purulent tonsillitis. Antibodies against
microorganism antigens were found at the patient. Which microorganism are these antibodies against?
{=Hemolytic streptococcus
~Staphylococcus
~Pneumococcus
~Mycobacterium tuberculosis
~Meningococcus}

Severe edema of soft tissues of upper and lower jaws, rash on the skin of face, redness, and itching appear in
the patient in response to using anesthetic drag at tooth extraction. Which pathological process underlies the
reaction to anesthetic? {=Allergy
~Drag toxic action
~Inflammation
~Insufficiency of blood circulation
~Disorder of lymph outflow}

Novocain was injected by dentist for anesthesia at tooth extraction. Symptoms of anaphylactic shock appeared
at the patient few minutes later. Patient has drop of BP, tachypnea, loss of consciousness and convulsions. What
type of reaction is it?
{=Immediate type hypersensitivity
~Cytolytic or cytotoxic reactions
~Arthus phenomenon reactions
~Delayed type hypersensitivity
~Stimulating allergic reaction}

Man with the caries is subjected to constant sensitization by streptococcus antigen. What disease can appear due
to this etiological factor?
{=Glomerulonephritis
~Pancreatitis
~Myocarditis
~Pulpits
~Periodontitis}

A dentist injected ultracain to a patient before tooth extraction for the purpose of anesthesia. Sensitivity test was
not made. Anaphy lactic shook developed in the patient in few minutes after drag injection. What cells produce
reagins, which take part in development of anaphylactic reaction?
{=Plasma cells
~B-lymphocytes
~T-lymphocytes
~Mast cells
~Eozinophiles}

Tooth was extracted in a teenager under Novocain anesthesia. Paleness of skin, dyspnea and
hypotension occurred in the patient 10 minutes later. What type of allergic reaction is it?
{=Anaphy lactic
~Cytotoxic
~Arthus phenomenon type
~Delayed type hypersensitivity
~Stimulating}

Tooth was extracted in a teenager under Novocain anesthesia. Paleness of skin, dyspnea and hypotension
occurred in the patient 10 minutes later. What substance does allergen react with on the surface of mast cells?
{=IgE
~T-lymphocytes
~IgA
~IgD
~IgM}

A 27-year-old woman instilled drops with penicillin to the eyes. Itching and burning pain of skin, edema of lips
and eyelids, cough with whistling, decreasing in BP occur in her some minutes later. What immunoglobulin
participates in development of these allergic reactions?
{=IgE
~IgG and IgM
~IgA
~IgM
~IgG}

A 43-year-old woman is suffering from pneumonia. She began complaining of weakness, face and arms
burning pain in 10 minutes after ampicillinum injection. Cough, dyspnea, pain in the chest developed in her. At
clinical examination patient has cyanosis, eyelids swelling, face red rashes, heart rate — 120 per minute, BP —
120 mmHg, muffled heart sounds, hypopnoe and tachypnea, respiration with different moist rales. What is the
reason for worsening of patient’s condition? {=Anaphylactic shook ~Urticaria ~Quincke’s edema
~Attack of asthma
~Pulmonary thromboembolism}

0.1 ml of horse serum was injected to the guinea-pig for sensitization. What are external signs of sensitization?
{=No external signs
~Skin rashes
~Joints swelling
~Increase in body temperature
~Pain}

A 15-year-old girl is suffering from bronchial asthma. Severe attack of expiratory dyspnea develops in
her during the spring blossoming period. What biological active substance causes spasm of bronchial
smooth muscle in this case? {=Leukotriene
~Thromboxane A2
~Prostacyclin
~Bradykinin
~Serotonin}

Patient addressed to a doctor with complaints of headache, rhinitis, weakness, and increase in body temperature
developing every spring in blossoming period. What type of allergic reaction by Gell and Cumbs underlies this
disease? {=Anaphylactic
~Cytotoxic
~Antibody-dependent cell-mediated cytotoxicity
~Immune complex-mediated
~Delayed type hypersensitivity}

Weakness, itching of skin, acute spasmodic pain in the abdomen, hyperemia and rash on skin, tachycardia, and
decrease in BP down to 70/40 mm Hg suddenly developed in a patient few minutes later Novocain injection
by the dentist. Which type of allergic reactions does this pathology belong to? {=Anaphylactic
~Cytotoxic
~Stimulating
~Cell-mediated
~Immune complex-mediated}

Contact dermatitis of upper extremities developed in the nurse, who has been working in manipulation room for
20 years. Which type of allergic reactions does this pathology belong to?
{=Delayed-type hypersensitivity
~Primary immunodeficiency
~Immediate type hypersensitivity
~B-cells immunodeficiency
~T-cells immunodeficiency}

Pain in the joints and loins, hemorrhagic eruptions on the skin, and increase in body temperature occur in the
patient with toxemic stage of burn disease in 2 hours after allogenic plasma transfusion. What allergic reaction
takes place in this case?
{=Serum sickness
~Urticaria
~Quincke’s edema
~Anaphy lactic
~Autoimmune vasculitis}

Hyperergic inflammation form of upper respiratory tract (larynx, trachea, bronchi) develops at 6-year-old child.
Threat of respiratory impairment develops and then necessity of using anti-inflammatory hormones occurs.
Which hormone has anti-inflammatory property?
{=Cortisol
~Adrenaline
~Growth hormone
~Testosterone
~Insulin}

Swelling, increased BP, proteinuria, hematuria, decreased urine excretion were revealed in the patient, which
suffered severe tonsillitis before. These symptoms are typical of acute glomerulonephritis, resulting from
damage of glomerular basement membrane. What is mechanism of this disease?
{=Anaphy lactic allergic reactions
~Cytotoxic allergic reactions
~Immune complex-mediated allergic reaction
~Delayed-type hypersensitivity
~Stimulating allergic reaction}

An 18-year-old patient has insulin-dependent diabetes mellitus I type. What type of allergic reactions
underlies beta cells damage? {=II type, cytotoxic
~I type, anaphylactic
~III type, immune complex-mediated
~IV type, delayed-type hypersensitivity
~Pseudoallergic reaction}
A patient with periodontitis has developed gingival edema. The gums are of dark red color. What local circulation
disorder prevails in the gums of the patient?
{=Venous hyperemia
~Thrombosis
~Embolism
~Arterial hyperemia
~Ischemia}

A 65 year old patient suddenly died. She suffered from thrombophlebitis of deep veins of shin. Autopsy
revealed: trunk and bifurcation of pulmonary artery contain red loose masses with dull corrugated surface.
What pathological process did the morbid anatomist reveal in pulmonary artery?
{=Thromboembolism
~Fat embolism
~Tissue embolism
~Thrombosis
~Foreign body embolism}

A 28-year-old patient had been diagnosed with multifragmental fracture of the right hip. On the third day after
the injury he began to complain of pain in the right side of chest, difficult respiration. A day later the patient
died of progressive heart and respiratory failure. Histological study of pulmonary and cerebral blood vessels
revealed orange sudanophilic droplets that completely obstructed the vessels of microvasculature. What
complication caused the death of the patient?
{=Fat embolism
~Gas embolism
~Drug-induced embolism
~Microbial embolism
~Thromboembolism}

A 68-year-old man, who had been suffering from essential hypertension for a long time, was delivered to a
resuscitation unit with hemiplegia. The patient died after 7 hours. On autopsy: in the nght cerebral hemisphere
there is a cavity 5x5 cm in size with uneven margins, filled with dark red blood clots. What cerebral circulation
disorder developed in the patient?
{=Thrombosis
~Hemorrhagic infiltration
~Hematoma
~Petechial hemorrhage
~Local venous hyperemia}

A patient with periodontitis has developed gingival edema. The gums are of dark red color. What local circulation
disorder prevails in the gums of the patient?
{=Venous hyperemia
~Thrombosis
~Embolism
~Arterial hyperemia
~Ischemia}

A patient was admitted to the hospital with diagnosis of acute left-ventricle heart failure. Patient’s condition
suddenly became worse and edema of lungs developed in him. What kind of disorders of peripheral blood
circulation causes the lungs edema? {=Congestion (venous hyperemia)
~Arterial hyperemia neuroparalytic type
~Arterial hyperemia metabolic type
~Ischemia
~Arterial hyperemia neurotonic type}

Dependence of blood pressure upon level of peripheral vascular resistance was measured in animal under
experimental conditions. Indicate vessels, which have the most resistance. {=Arterioles
~Arteries
~Aorta
~Capillaries
~Veins}

A patient has obstruction of right calf profound veins, which results in increasing in amount of blood in the
impaired part of tissue. What is the name for increase in amount of blood resulted from impeded blood outflow?
{=Venous hyperemia (congestion)
~Thrombosis
~Ischemia
~Stasis
~Arterial (active) hyperemia}

A patient with paradontitis has edema of gums. His gums are dark red. What kind of local blood flow disturbance
takes place in patient’s gums? {=Venous (passive) hyperemia
~Arterial (active) hyperemia
~Embolism
~Thrombosis
~Ischemia}

Pain in leg at walking, cyanosis, and edema of calf appeared in a patient with varicosity. His foot is cold. What
kind of disturbances of regional blood flow appeared in this patient?
{=Venous (passive) hyperemia
~Ischemic stasis
~Compressive ischemia
~Angiospastic ischemia
~Obstructive ischemia}

Edema and cyanosis of low extremities appear in a food shop assistant at the end of workday. What is the main
factor of edema development in this patient?
{=Orthostatic increase of venous pressure
~Dilatation of resistant vessels
~Increase of number of functional capillaries
~Increase of collateral blood flow
~Increase of tissue drainage}

Redness and increase in volume of affected place of tissue, and increase in local temperature are observed in a
patient with burn of thigh. What pathologic process do indicated symptoms correspond to? {=Arterial hyperemia
~Venous hyperemia
~Thrombosis
~Ischemia
~Stasis}

A 23-year-old woman had intense psycho-emotional excitement. Hyperemia of face skin, tachycardia and
increase in BP were observed in her. What is the mechanism of redness of her face skin?
{=Congestive venous hyperemia
~Neuroparalytic arterial hyperemia
~Post-ischemic arterial hyperemia
~Neurotonic arterial hyperemia
~Stasis}
Dyspnea, sharp pain in the chest, cyanosis, and jugular venous distention suddenly develop in a patient with
thrombophlebitis of lower extremities. What is the most possible disorder of blood circulation developed in the
patient?
{=Thromboembolism of pulmonary artery
~Thromboembolism of mesenteric vessels
~Thrombosis of cerebral vessels
~Thromboembolism of portal vein
~Thromboembolism of coronary vessels}

Burning in the mouth appears after taking of food with pungent dressings. At examination mucosa of oral cavity
is edematous, hyperemic, and bright red. What major reasons underlie disorders of microcirculation?
{=Degranulation of tissue basophiles
~Neuroparalitic arterial hyperemia
~Postischemic arterial hyperemia
~Atonic hyperemia
~Congestion}

A woman experienced a strong psycho-emotional excitement during dentist’s reception. Redness of her face skin
and profuse salivation were noticed at that time. What is the mechanism of these phenomena? {=Neuroparalitic
arterial hyperemia Manifestation of sympathetonic effect
~Stagnant venous hyperemia
~Neuroparalitic arterial hyperemia
~Postischemic arterial hyperemia
~Manifestation of sympathetonic effect}

Patient’s arm was put in plaster cast on account of simple fracture of humeral bone. Swelling, cyanosis, and
decrease in temperature of the traumatized arm appear next day. What disorder of peripheral blood flow do these
symptoms testify to?
{=Venous hyperemia
~Venous hyperemia
~Ischemia
~Embolism
~Thrombosis}
Injury of sympathetic fibers of the sciatic nerve developed in a patient due to trauma. What kind of peripheral
blood flow disorder takes place in the patient?
{=Neuroparalityc arterial hyperemia
~Venous hyperemia
~Angiospastic ischemia
~Neurotonic arterial hyperemia
~Obturative ischemia}

A 42-year-old woman, shop assistant by profession, complains of edema of the lower extremities at the end of a
workday. Her legs are cyanotic, their temperature is decreased. Venous hyperemia of lower extremities was
established, which is due to constitutional weakness of the elastic apparatus of veins and the occupation. What is
the major pathogenic factor causing local changes in venous hyperemia? {=Hypoxia
~Disorders of metabolism
~Atrophy
~Dystrophy
~Sclerosis}

Puncture of abdominal cavity for the extraction of fluid was performed to a 45-year-old patient with diagnosis
“cirrhosis of liver, ascites”. State of unconsciousness suddenly developed in the patient as a result of decrease in
blood pressure after extraction of 5L of fluid. That was considered as a manifestation of brain blood circulation
insufficiency. Which disorder of microcirculation occurs in this case? {=Ischemia
~Arterial hyperemia
~Venous hyperemia
~Thrombosis
~Embolism}

Signs of disorders of microcirculation in form of venous hyperemia were observed in a patient with gingivitis.
This condition manifested by cyanosis and edema of mucosa and decrease in local temperature. What is the major
mechanism of congestion development?
{=Increased permeability of vessel wall
~Perivascular changes
~Disturbances of lymph outflow
~Exudation from vessels
~Blood factors}

Bernard observed increase in secretion of submandibular salivary gland and development of arterial hyperemia
irritating chorda tympani (branch of nervus facialis) in an experiment. What sort of arterial hyperemia it is
according to mechanism of its development?
{=Neurotonic
~Neuroparalitic
~Metabolic
~Reactive
~Working}

Phlebothrombosis of the profound veins of lower extremities with subsequent pulmonary thromboembolism
develop in a patient operated for chronic ulcer of duodenum at the postoperative period. Which functional
disorder of those that develop in this case is the most important?
{=Abrupt drop of arterial blood pressure
~Increase in central venous pressure
~Increase in pressure in pulmonary artery
~Acute right ventricular failure
~Changes of ventilation-perfusion ratio in the lungs}

A 57-year-old man complains of heart pain that has developed after prolonged negative emotions. An emergency
doctor diagnosed ischemic heart disease manifesting by stenocardia. What kind of ischemia is the most probable?
{=Compressive
~Obliterative
~Angiospastic resulting from deficiency of vasodilators ~-
~Obturative}

Condition of a patient with thrombophlebitis of lower extremities suddenly became worse. Weakness, giddiness,
dyspnea, pain in the chest, and cyanosis developed in him. It was established that pulmonary thromboembolism
results in acute cor pulmonale. What is the leading link in forming of this syndrome? {=Increased blood pressure
in pulmonary artery
~Increase in central venous pressure
~Abrupt arterial blood pressure drop in systemic circulation
~Weakening of heart function resulting from myocardial hypoxia
~Disorders of external respiration}

The theory exists that atherosclerosis plays an important role in periodontitis development, affecting vessels of
gums. What kind of local blood circulation disorders develops under atherosclerosis of vessels? {=Active
hyperemia
~Passive hyperemia
~Embolism
~Ischemia
~Disorders of lymph outflow}

Instantaneous death of pilots occurs under depressurization of airplane cabin at the altitude of 19 km. What is the
reason for it?
{=Multiple gas embolisms
~Hemorrhage to the brain
~Gas embolism of cerebral vessels
~Bleeding
~Paralysis of respiratory center}

A patient has acute pain in his chest, dyspnea, tachycardia, cyanosis, and decreased BP. Pulmonary infarction
was diagnosed in this patient. Which factor is the most common cause of pulmonary infarction?
{=Embolism by thrombus from veins of lower extremities
~Congestion in the pulmonary circulation
~Increase in number of platelets
~Activation of fibrinolytic system
~Pneumothorax}

A 50-year-old patient suffers from endarteritis. Which of the following symptoms is the most characteristic one
for this pathology?
{=Decreased temperature of lower extremities
~Cyanosis
~Increased temperature of lower extremities
~Edema
~Enlargement of volume of extremities}

Pulmonary embolism developed in a patient with coxal bone fracture. What kind of embolism does this patient
have? {=Fat
~Thromboembolism
~Tissue ~Gas
~Air}

Gas embolism developed in a diver who was lifted to the surface very fast. It results from fast change:
{=From increased atmospheric pressure to normal
~From normal atmospheric pressure to increased
~From normal atmospheric pressure to decreased
~From decreased atmospheric pressure to normal
~}
Acute heart arrest develops in a patient with thrombophlebitis of calf profound veins. What is its reason?
{=Pulmonary thromboembolism
~Left ventricle hypertrophy
~Myocardial dystrophy
~Endocarditis of mitral valve
~Atherosclerosis}

Convulsions and loss of consciousness develop in a diver during the emergency raising from the depth. What is
the major pathogenic mechanism in these disorders development?
{=Gas embolism
~Hypoxia
~Toxic action of oxygen
~Toxic action of nitrogen
~Hypercapnia}

Destruction of endothelial layer of vessel leads to vasoconstriction. Decreased secretion of endothelial-derived


factor plays the leading role in this phenomena development. What is this factor? {=Nitric oxide
~Adenosine
~Histamine
~Bradykinin
~Adenosine monophosphate}

What changes develop in human organism in case of depressurization of airplane cabin at altitude of 19 km?
{=Gas embolism
~Compensatory augmentation of biologic oxidation in tissues
~Increase in body temperature
~Compensatory augmentation of heart work
~Disorders of muscular coordination}
A man constantly lives in mountains. What changes of blood test indices can found in him? {=Increased
red blood cells number
~Decreased reticulocytes count
~Decreased color index
~Appearance of erythroblasts in blood
~Decreased hemoglobin content}

After autopsy a forensic medical expert established that death of 20-year-old woman resulted from cyanide
poisoning. What process disorder caused the death?
{=Tissue respiration
~Oxy gen transport by hemoglobin
~Hemoglobin synthesis
~Urea synthesis
~Oxidative phosphorilation}

Increased number of erythrocytes in blood test was revealed in people residing in village, which is situated in
mountains at the altitude of 3000 m. What is the reason for changes in their blood? {=Increased erythropoietin
production
~Blood clotting
~Change of spleen function
~Increased vitamin B12 synthesis
~Increased circulating blood volume}

The group of patients from sanatorium went to the mountains for excursion. Tachycardia and breathlessness
developed in part of them in two hours after beginning of excursion. Which type of hypoxia do those disturbances
result from?
{=Hypoxic
~Hemic
~Respiratory
~Tissue
~Circulatory }

A driver slept in garage in a car with working engine. When he woke he had headache and later vomiting began
in him. What compound formation in blood caused this state?
{=Carboxyhemoglobin
~Methemoglobin
~Carbhemoglobin
~Desoxihemoglobin
~Oxihemoglobin}

Which sportsman will have highest content of lactic acid in the blood?
{=Who just had run 5000 m
~Who just had run 100 m
~Weight-lifter
~Who put the shot
~Jumper with pole}

A 40-years-old man took cyanic potassium by mistake. He died instantly. What mitochondrial enzymes do
cyanides block? {=Cytochrome Oxydase
~NAD-dependent Dehydrogenases
~Cytochrome B
~Cytochrome C
~FAD-dependent enzymes}

Pain in muscles appears after physical exertion in people, who move a little (have hypodynamia) for a long time.
What is the possible reason for this? {=Accumulation of lactic acid in muscles
~Increased content of ATP in muscles
~Accumulation of creatinin in muscles
~Decreased content of lipids in muscles
~Enhanced decay of muscle proteins}

The number of erythrocytes in alpinist’s blood before going to mountains is 4,5x10!?/L. What changes of
erythrocytes number can develop at the altitude of 2500 m above sea level?
{=Absolute erythrocytosis
~Absolute erythropenia
~Relative erythropenia
~Relative erythrocytosis
~There won't be any changes}

Dyspnea, increased heart beat rate, weaken of attention, foolishness occurred in sportsmen without
acclimatization during the ascent to the altitude of 3000 m. Then weakness and disturbance of muscular
coordination occurred. What was the reason for that condition?
{=Hypoxic hypoxia
~Circulatory hypoxia
~Respiratory hypoxia
~Hemic hypoxia
~Hypercapnia}

Environmental pollution by nitric compounds occurs after accident at a chemical factory. People, who live at this
region, have sharp weakness, headache, breathlessness, and giddiness. What does hypoxia result from?
{=Methemoglobin formation
~Cytochrome Oxidase inactivation
~Dehydrogenases suppression
~Carboxyhemoglobin formation
~Decreased function of FAD-dependent enzymes}

Total respiratory insufficiency was noticed on examination of blood gases partial pressure in a patient with
bronchial asthma. What is the reason of respiratory hypoxia in this case?
{=Decreased ventilation
~Decreased diffusion
~Increased ventilation
~Increased perfusion
~Increased diffusion}

Total respiratory insufficiency developed in a patient with lung disease. It manifested by decreased pO2 and
increased pCO: in patient’s blood. What caused development of respiratory hypoxia and pronounced respiratory
insufficiency? {=Frequent shallow respiration
~Excessive hyperventilation
~Oxygen deficit in inspired air
~Uneven ventilation of the lungs
~Functional blood shunting in the lungs}

Which factors increase hypoxic injury of a cell under hypoxic conditions?


{=Catecholamines
~Inhibitors of proteolysis
~Glucocorticoids
~Chinon derivations
~Glutation-peroxidase}
Disorders of oxyhemoglobin formation have led to development of hemic hypoxia. What are the reasons for
hemic hypoxia development? {=Methemoglobin formation
~Inhibition of dehydrogenases
~Pronounced polycytemia
~Activation of Glutation Peroxidase
~Deficiency of riboflavinum}

Symptoms of poisoning such as sharp weakness, dizziness, headache, vomiting, dyspnea, tachycardia, cyanosis
of lips, ears, distal phalanges of fingers occurred in one child from group of children, who had eaten sweet juicy
watermelon. Laboratory test of watermelon showed high level of nitrates. What is leading mechanism in
pathogenesis of only one child poisoning?
{=Methemoglobin reductase deficiency
~Cytochrome oxidase blockade
~Glutathione peroxidase deficiency
~Superoxide dismutase deficiency
~Catalase deficiency}

A man has been living high in mountains for a long time. What changes in his blood would develop? {=Increase
in quantity of hemoglobin
~Increase in diameter of blood vessels
~Decrease in number of leukocytes
~Increase in number of leukocytes
~Rare pulse}

Tachypnea and hypopnoe developed in tourists, which had climbed to the altitude of 3000 m. These changes are
consequence of stimulation of {=Chemoreceptors of carotid sinus
~Mechanoreceptors of pulmonary alveoli
~Baroreceptors of arch of aorta
~Neurons of the cerebral cortex
~Motoneurons of spinal cord}

Cyanosis, swelling of lower extremities, dyspnea occurred in a 60-year-old patient suffering from heart failure.
Long-term adaptation of organism to hypoxia, occurring in that condition, can be supplied by {=Increased
hemopoietic activity
~Blood outlet from the depots
~Opening of non-functioning capillaries
~Tachycardia
~Increased blood oxygenation}

A dog has poisoning by unknown substance that caused its immediate death because of oxidation of cytochromes.
What is that substance?
{=Potassium cyanide
~Potassium chloride
~Potassium sulfate
~Potassium orotate
~Potassium permanganate}

Gas alkalosis developed in a group of alpinists due to rise to the Everest’s top. So, carbon dioxide partial pressure
in their arterial blood makes up
{=30 mmHg
~30 mmHg
~40 mmHg
~60 mmHg
~70 mmHg}
Hemic hypoxia occurred in a patient. It is accompanied by methemoglobin accumulation and development of
metabolic acidosis. Which substance accumulation leads to disturbance of acid-base balance? {=Lactates
~Sulfates
~Phosphates
~Hydrocarbonates
~Hydroxybutirates }

During keeping salvage operations, several miners were taken out from the mine. They were unconscious but
without any visible damages. Air in mine had considerable amounts of methane. Which type of hypoxia
developed in those people?
{=Hypoxic
~Hemic
~Circulatory
~Tissue
~Respiratory}

A 29-year-old patient with carbon monoxide poisoning was admitted to the hospital. He had signs of severe
hypoxia: pronounced dyspnea, cyanosis and tachycardia. Which changes of hemoglobin take place during carbon
monoxide poisoning? {=Carboxyhemoglobin formation
~Methemoglobin formation
~Carbhemoglobin formation
~Sulfhemoglobin formation
~Oxyhemo globin inactivation}

A 54-year-old patient was admitted to hospital in hypoxic state, developed due to aspiration of vomit mass. Blood
test was taken after the treatment of patient. Do erythrocytes number in peripheral blood change in the initial
phase of hypoxia?
{=Increase due to outlet of the blood from depots
~No change in initial phase
~Decrease due to hemolysis of erythrocytes
~Decrease due to depot of blood
~Increase due to increase of hemopoiesis}

A patient had course of dosing hypoxic trainings as a part of treatment of hypertensive disease. The most
considerable cellular adaptative mechanism that begins working under such treatment is {=Cellular hypertrophy
~Defense of cell membrane
~Correction of ions transport
~Mobilization of plastic reserve
~Decrease in functional activity of cells}

Dyspnea, cyanosis of mucous membranes of the lips, moderate tachycardia (pulse rate of 80 beats per minute)
suddenly occurred in a patient during the treatment of carious tooth. Which type of hypoxia developed in this
patient? {=Circulatory
~Hemic
~Tissue
~Respiratory
~Hypoxic}

A patient came to a dentist with complaints of prolonged bleeding from his gums, reclining of teeth. Paradontosis
was diagnosed. Which type of hypoxia developed in the patient? {=Tissue
~Circulatory
~Respiratory
~Hemic
~Hypoxic}

Sensitivity of different tissues to oxygen deficit depends on their metabolic rate, power of glycolytic system,
reserve of ATP and ADP, potential possibility of genetic apparatus to provide the plastic consolidation of
hyperdeduction. Which system is in the worst conditions from these points of view?
{=Nervous system
~Cardiovascular system
~Respiratory system
~Gastrointestinal system
~Urinary system}

Decrease in adenylic nucleotides ATP/ADP ratio leads to the increase in glycolysis in tissues under hypoxic
conditions. Which reaction get active in this situation?
{=Lactate dehydrogenase-depended
~Triozophosphate isomerase-depended
~Aldolase-depended
~Endolase-depended
~Phosphofructokinase-depended}

A child has been brought to the admissions office. He had nitrates poisoning symptoms such as cyanosis, dyspnea,
and cramps. What is the mechanism of those symptoms development?
{=Methemoglobin formation
~Reduce hemoglobin formation
~Carbhemoglobin formation
~Oxyhemoglobin formation
~Carboxyhemoglobin formation}

Sense of muscular weakness, dyspnea, and tachycardia appeared in a tourist at the altitude of 5200 m. What was
the reason for appearance of these symptoms? {=Decrease in oxygen partial pressure in expired air
~Decrease in atmospheric pressure
~Increase in carbon dioxide partial pressure in patient’s blood
~Gas embolism
~Air embolism}
A fireman with signs of carbon monoxide poisoning has been brought from the place of accident. Which type of
hypoxia is more possible in that situation?
{=Hemic hypoxia
~Ischemic hypoxia
~Stagnant type of circulatory hypoxia
~Hypoxic hypoxia
~Respiratory hypoxia}

A 60-year-old man was admitted to the hospital with carbon monoxide poisoning. What is mechanism of
disturbances that man has?
{=Inactivation of hemoglobin respiratory function
~Insufficiency of lungs ventilation
~Disturbance of blood circulation in the organs and tissues
~Decrease in partial pressure of oxygen in alveoli
~Increase in adrenaline secretion}
A patient addressed to the doctor with complaints of the attack of breathlessness and dizziness. It was revealed
that he worked at the chemical factory, producing senile acid. Those signs can be explained by disturbance of
enzyme
{=Cytochrome oxydase
~Succinate dehydrogenase
~Catalase
~Lactate dehydrogenase
~Pyruvate dehydrogenase}

Patient has microanizocytosis and poikilocytosis, decreased erythrocytes number, hemoglobin levels, color index,
concentration of iron in serum. These changes were accompanied by development of hypoxia. It name is:
{=Hemic
~Hypoxic
~Circulatory
~Tissue
~Respiratory}

Which reason can lead to hypoxia of exogenous type?


{=Decreased partial pressure in inspired air
~Air embolism
~Gas embolism
~Insufficiency of ventilation
~Acute bleeding}

A patient has been suffering from disease of cardiovascular system for a long time. Chronic blood circulation
insufficiency developed in him eventually. Which mechanism of long-term adaptation to hypoxia takes place in
this patient?
{=Activation of hemopoiesis
~Tachypnea
~Tachycardia
~Mobilization of blood from depots
~Interorganic redistribution of blood circulation}

A patient has been taken to resuscitation department. Considerable content of sulfhemoglobin was revealed in his
blood. Which type of hypoxia does patient have?
{=Hemic
~Respiratory
~Circulatory
~Tissue
~Exogenous }

Alpinists started complaining of breathlessness, palpitation, headache, dizziness, and ringing in their ears during
climbing to mountain at the altitude of 5000 m. Which pathologic factor caused these symptoms? {=Hypoxemia
~Hypokaliemia
~Increased oxygen capacity of blood
~Lactacidemia
~Hypothermia}

Depressurizing of an airplane cabin happened due to emergency conditions at the altitude of 5000 m. What is the
main mechanism of damages inflicted on the people that were in the airplane? {=Hypoxic hypoxia
~Gas embolism
~Tissue embolism
~Respiratory hypoxia
~Circulatory hypoxia}
Symptoms of hypoxia appeared in a patient after the course of treatment with amidopirin and sulfanilamides.
What is the reason for development of this type of hypoxia?
{=Formation of methemoglobin
~Formation of carboxyhemoglobin
~Formation of nitroxyhemoglobin
~Formation of carbhemoglobin
~Formation of oxyhemoglobin}

After an accident in a coalmine, miners were complaining of headache, dizziness, tiredness, glimmer in front of
their eyes, tachycardia. Which type of hypoxia did they have?
{=Hypoxic
~Circulatory
~Respiratory
~Tissue
~Hemic}

Signs of hypoxia such as dyspnea, tachycardia, and cyanosis of visible mucous membranes appeared in a patient
with myocardial infarction. Development of hypoxia connected with
{=Decrease in velocity of blood flow
~Decrease in erythrocytes number
~Decrease in content of hemoglobin
~Insufficiency of blood oxygenation
~Difficult oxyhemoglobin dissociation}

A man has been brought in hospital by ambulance. He was unconscious because of carbon monoxide poisoning.
What substance accumulation in the blood underlies this condition?
{=Carboxyhemoglobin
~Oxyhemoglobin
~Desoxyhemoglobin
~Methemoglobin
~Carbhemoglobin}
A 65-year-old patient suffers from aortic valve stenosis. She has symptoms of heart failure such as dyspnea,
cyanosis, and edema that appeared after viral infection. She was admitted to the therapeutic department. Which
type of hypoxia is in the patient?
{=Circulatory
~Hemic
~Hypoxic
~Respiratory
~Tissue}

A 70-year-old patient underwent medical treatment for ischemic heart disease, heart failure in cardiological
department. Which type of hypoxia was in the patient?
{=Circulatory
~Hemic
~Respiratory
~Tissue
~Mixed}

A 13-year-old girl undergoes treatment for iron-deficiency anemia in hematological department. Which type of
hypoxia does this patient have?
{=Hemic
~Circulatory
~Tissue
~Respiratory
~Mixed}
A 56-year-old woman suffers from thyrotoxicosis for a long time. Which type of hypoxia can develop in this
patient? {=Tissue
~Hemic
~Circulatory
~Respiratory
~Mixed}

Group of tourists ascended to the altitude of 4200 m. Three alpinists complained of headache, pain in their ears
and frontal sinuses, somnolence, considerable muscular weakness, irritability during the ascent. The possible
reason for appearance of these symptoms was
{=Alltitude sickness
~Hyperbaric oxy genetion
~Caisson disease
~Gas saturation in blood
~Altitude decompression}

Measurements of the arterial pCOz2 and pO2 during an attack of bronchial asthma revealed hypercapnia and
hypoxemia respectively. What kind of hypoxia occurred in this case? {=Respiratory
~Hemic
~Circulatory
~Tissue
~Histotoxic}

The resuscitation unit has admitted a patient in grave condition. It is known that he had mistakenly taken sodium
fluoride which blocks cytochrome oxidase. What type of hypoxia developed in the patient? {=Tissue
~Hemic
~Cardiovascular
~Hypoxic
~Respiratory}

A public utility specialist went down into a sewer well without protection and after a while lost consciousness.
Ambulance doctors diagnosed him with hydrogen sulfide intoxication. What type of hypoxia developed?
{=Hemic
~Overload ~Tissue
~Circulatory
~Respiratory}

When ascending to the top of Elbrus, a mountain climber experiences oxygen starvation, dyspnea, palpitations,
and numbness of the extremities. What kind of hypoxia has developed in the mountain climber? {=Circulatory
~Tissue
~Cardiac
~Hypoxic
~Hemic}
A patient with inflammation of trigeminal nerve has been having progressive paradontitis for some years. What
factor is the most important for parodontitis development?
{=Neurodistrophic disorders
~Low activity of leukocytic elastase
~Poor formation of immunoglobulins
~Increased tone of vagus nerve
~Low activity of kallikrein-kinin system}

An animal sensitized with tuberculin had been administered tuberculin intraperitoneally. 24 hours later, during
laparotomy, the animal was found to have venous congestion and peritoneal edema. Impression smears from the
peritoneum contained a large number of lymphocytes and monocytes. What pathological process was detected in
the animal?
{=Allergic inflammation
~Serous inflammation
~Purulent inflammation
~Fibrinous inflammation
~Aseptic inflammation}

A 31-year-old patient has deep caries of the fifth maxillary tooth on the right. Acute purulent inflammation in the
region of the tooth apex has developed. There is marked edema with isolated neutrophils in the perifocal tissues
and soft tissues of cheek and palate. Diagnose the pathologic process in the soft tissues of cheek and palate:
{=Acute serous inflammation
~Acute suppurative periodontitis
~Acute suppurative periostitis
~Phlegmon
~}
A 7-year-old child suffers from acute disease. A pediatrician observed the following during examination: pharynx
mucosa is hyperemic, edematous, swollen, cowered with large amount of mucus. Buccal mucosa has whitish
discoloured spots. On the next day the child came out in rash with large spots covering the skin of his face, neck
and torso. What kind of inflammation causes changes in pharynx mucosa? {=Catarrhal
~Serous
~Serofibrinous
~Fibrinous
~Hemorrhagic}

A woman has scalded her hand with boiling water. The affected area of her skin became red, swollen and painful.
This effect is caused by accumulation of the following substance: {=Histamine
~Lysine
~Thiamine
~Glutamine
~Asparagine}

A patient complains of toothache. On examination he has been diagnosed with pulpitis. Which factor played the
main pathogenic role in the development of pain syndrome in this case?
{=Vasospasm
~Activation of one of the complement system components
~Interleukin action
~Increased intratissular pressure in the dental pulp
~Inadequate stimulation of the mandibular nerve branch}

Toxic pulmonary edema was reproduced on a laboratory rat by means of ammonium chloride solution. What is
the leading pathogenetic factor of this edema? {=Increased permeability of capillars
~Disorder of neural and humoral regulation
~Increase of lymph outflow
~Increase of venous outflow
~Decrease of colloid osmotic pressure}

A patient was stung by a bee. Examination results: his left hand is hot, pink and edematic, there is a big blister on
the spot of the sting. What is the leading mechanism of edema development?
{=Increased vascular permeability
~Vascular damage caused by the sting
~Reduction of osmotic pressure of tissue
~Reduction of oncotic pressure of tissue
~Reduced blood filling of vessels}

It is known that patients with diabetes mellitus are more subject to inflammative processes, they have low
regeneration and slower wound healing. What is the reason for this?
{=Decrease in protheosynthesis
~Accelerated gluconeogenesis
~Intensification of catabolism
~Decrease in lipolysis
~Increase in lipolysis}

A 35-year-old patient complains of pain in the upper jaw, bleeding, a slight loosening of teeth. He has been
diagnosed with periodontitis. What is a typical pathological process in this case? {=Inflammation
~Bleeding
~Caries
~Pain
~Redness}
Autopsy of a 58-year-old man, who had been suffering from rheumatic heart disease and died of cardiopulmonary
decompensation, revealed gray diffuse film- and fiber-shaped coating in his pericardium. What type of
inflammation is characteristic of this pericarditis?
{=Diphtheritic fibrinous
~Suppurative ~Serous
~Hemorrhagic
~Croupous fibrinous}

What factor results in maximal dilation of the gemomicrocirculatory pathway vessels and their increased
permeability? {=Histamine
~Endothelin
~Vasopressin
~Noradrenaline
~Serotonin}

A cook burnt his arm with steam. What substance increased and led to development of redness, edema and
painfulness of affected area of skin?
{=Histamine
~Kalidin
~Thiamine
~Galactosamine
~Glutamine}
A 12-year-old patient suffering from acute leukemia presents with fever up to 39, 80C, acute pain in the throat.
Examination of the oral cavity has revealed swollen tonsils, their surface is covered in deep lesions with uneven
margins, numerous petechial hemorrhages in the pharyngeal mucosa and around the tonsils. Determine the type
of tonsillitis that complicates the disease progress in this case:
{=Necrotic
~Catarrhal
~Fibrinous
~Lacunar
~Purulent}

A 7-year-old child fell ill sharply. Hi has catarrh, cough, lacrimation and large-spot rash on skin; his temperature
is increased up to 38°C. Mucous membrane of patient’s pharynx is edematous and red; buccal mucous membrane
has whitish spots. What is the character of inflammation underlying changes of buccal mucous membrane?
{=Fibrinous
~Serous
~Purulent
~Catarrhal
~Hemorrhagic}

Research of urgentic patient was observed presence in him neutrophilic leucocytosis with the change of leucocytic
formula to the left. In the patient the most likely
{=Purulent inflammation
~Tuberculosis
~Allergic reaction
~Agranulocytosis
~Alimentary leucocytosis}
Enlargement and deformation of joints were revealed in a patient with rheumatism. What type of inflammation
underlies these changes?
{=Proliferetive
~Alterative
~Exudative
~Fibrinous
~Hemorrhagic}

A painful blisters full of opaque fluid has formed in patient after thermal burn. What type of inflammation has
appeared? {=Serous
~Croup
~Granulomatous
~Proliferative
~Diphtheritic}

Prevalence of proliferative processes was revealed in a patient with chronic inflammation of skin and
subcutaneous adipose tissue. Which hormone deficiency can lead to this situation? {=Cortisol
~Aldosterone
~Insulin
~Growth hormone
~Thyroxin}

A patient complains of a pain in joints. Edema and redness are visible above joints. Which enzyme activity has
to be analyzed for establishing the diagnosis?
{=Hyaluronidase
~Creatine kinase
~Alkaline phosphatase
~Acid phosphatase
~Urease}

Condition of biological active substances (BAS) prevalence over there inhibitors usually occurs in inflammation.
Indicate correct correspondence of BAS to their inhibitor
{=Leukotriens - arylsulfatase
~Catecholamines - cholinesterase
~Kinins - monoaminooxidase
~Histamine - carboxypeptidase
~Serotonin-protease inhibitor}

Edema has developed as a result of oncotic pressure elevation in the site of inflammation. What is the main reason
of hyperoncia?
{=Activation of endotheliocytes
~Protein catabolism
~Drainage disorders
~Macromolecules accumulation
~Gluconeogenesis intensification}

What factor promotes proliferation in focus of chronic inflammation?


{=Hypoxia
~Hyperosmia
~Protein catabolism
~Hyperoncia
~Prostacyclin synthesis}

A patient has high body temperature, redness, edema, painfulness on her right forearm. What biological active
substances intensify inflammatory reaction?
{=Kinins
~Prostacyclins
~Phospholipase D
~Proteolysis inhibitors
~Vasopresin}

A patient has an inflammatory process on the outer surface of calf. Redness, edema, pain, enlargement of local
lymph nodes, and body temperature rising are noticed in patient. What are the principles of pathogenetic therapy
of acute inflammation? {=Proteolysis activation blockage
~Energy formation processes improving
~Free radical oxidation increase
~Introduction of mineralocorticoids
~Introduction of thyroid hormones}

A patient with finger inflammation has sharp pain and edema in the inflammatory focus. After opening of an
abscess these symptoms have disappeared. What favors disappearance of pain?
{=Decrease of osmotic pressure
~Increase in c-AMP concentration
~Alkalosis
~Kallikrein-kinin system activation
~Mast cells degranulation}

There are edema, redness and soreness in a place of injury. What is the reason for inflammation’s symptoms
appearance?
{=BAS discharged by cells
~Monoamine oxidase activation
~Sympathetic nervous system activation
~Carboxy peptidase activation
~Limited proteolysis inhibition}

Female patient, aged 32, was stung by wasp. On the skin of left cheek (place of sting) there is a zone of hyperemia
and edema. What is the primary mechanism of edema in this case?
{=Increase in capillary permeability
~Lymph drainage decease
~Increase of oncotic pressure
~Decrease of oncotic pressure
~Increase of hydrostatic pressure in capillaries}

What inflammatory mediator is formed due to limited proteolysis of plasma globulins? {=Bradykinin
~Leukotriens
~Histamine
~Prostaglandins
~Lymphokines}
Fever and increase of antibodies and leukocytes have appeared in animal under experimental modeling of
inflammation. What substances conditioned to all these common reactions in inflammation? {=Interleukins
~Interferons
~Mineralocorticoids
~Glucocorticoids
~Somatomedins}

A patient sustained trauma of knee joint and posttraumatic hemorrhagic bursitis appeared. After 3 months passive
movements limitation in extend was observed. Limitation was a result of scar formation. What inflammatory
component was a basis of this complication?
{=Proliferation
~Exudation
~Tissue hyperplasia
~Secondary alteration
~Primary alteration}

In female patient, 28, abscess opening was performed and fast wound reparation was observed. Reparation was
preceded without score formation. What cells play the main role in proliferation? {=Fibroblasts
~Neutrophils
~Eosinophils
~Lymphocytes
~Monocytes}

In examination of abscess punctate under a microscope different blood cells were revealed. Which of them
appears the first in inflammatory focus?
{=Neutrophils
~Mast cells
~Eosinophils
~Monosytes
~Lymphocytes}
What is the main factor of pain formation in pulpitis?
{=Hydrostatic pressure increasing
~Hydrostatic pressure decreasing
~Excess of pain receptors excitability
~Ischemia
~Excess of brain centers sensitivity}

What is the main factor of edema formation in pulpitis?


{=Capillaries permeability increasing
~Hypoproteinemia
~Tissue hyperosmia
~Blood hyperosmia
~Capillaries hydrostatic pressure increasing}

Which of following inflammatory mediators are formatted under the influence of lypooxygenases? {=Leukotriens
~Prostaglandins El, E2
~Prostacyclins
~Thromboxans
~Thrombocytes activation factor}
Patient has a caries complicated with pulpitis. This condition is accompanied by unbearable pain. What is the
main reason of pain? {=Emigration
~Secondary alteration
~Primary alteration
~Exudation
~Proliferation}

It is known, that inflammatory mediators play the main role in inflammation pathogenesis. What is the histamine
action on inflammation? {=Blood vessel permeability increasing
~Chemotaxis
~Thrombocytes aggregation
~Blood vessels constriction
~Blood coagulation}

Female patient, 19, complains of pain in gums, gingival hemorrhage. These symptoms have appeared 3 days after
sulphamide drugs taking. Objectively: gingival papilli and gum margin are reddened and swollen; there is a
hemorrhage appeared in response of touching. What type of inflammation is observed in patient? {=Hemorrhagic
inflammation
~Acute cattharal inflammation
~Fibrinous inflammation
~Pus inflammation
~Chronic cattharal inflammation}

A patient 6 years old was admitted to the hospital with asphyxia. The membranous coats on mucosa of fauces
were revealed. The coats can be removed easy. What type of inflammation is in patient? {=Fibrinous
~Necrotic
~Pus
~Cattharal
~Hemorrhagic}

There are lots of neutrophils in abdominal pus exudates in a patient with peritonitis. What is the main function of
neutrophils in inflammatory area?
{=Phagocytosis
~Prostaglandins secretion
~Degranulation
~Histamine liberation
~Local blood flow regulation}

Keloid cicatrix has formatted in a place of abscess. Name the stage of inflammation, which caused scar
appearing. {=Proliferation
~Exudation
~Primary alteration
~Secondary alteration
~Emigration}

A male patient, 16, was admitted to the hospital with acute appendicitis. What typical pathological process is the
basis of this disease?
{=Inflammation
~Hypoxia
~Fever
~Tumor
~Allergy}

Eyeball inflammation resulted pus in the anterior chamber of the eye (hypopyon). What stage of inflammation
was it?
{=Exudation
~Proliferation
~Primary alteration
~Secondary alteration
~Emigration}

In a patient with skin pathological process which has 5 typical symptoms: tumor, rubor, calor, dolor, functio laesa.
The development of such process includes 3 stages: Alteration, _? _ , proliferation. What is the second one?
{=Exudation
~Regeneration
~Reparation
~Fibrosis
~Petrification}

The secondary alteration is caused by:


{=Lysosomal enzymes
~Kinins
~Lympokynes
~Prostaglandins
~Complement components C3a and C5a}

In proliferation cells multiplying and defect compensation take place. What is the reason of cells growth in
specific clones?
{=Keylones
~Platelet-derived grow factor
~Interleukins
~Somatomedine
~Lymphocytes peptide}
There are following components in pleural effusion: proteis - 62g/, abundance of neutrophils. What is the type
of inflammation in this case?
{=Serous
~Suppuratuve
~Fibrinous
~Hemorrhagic
~Putrefactive}

Necrotic focus has appeared on burn, swallowing and red skin. What is the main mechanism of necrobyosis
improvement in inflammatory area?
{=Secondary alteration
~Primary alteration
~Emigration of leucocytes
~Dyapedesis of erythrocytes
~Fibroblasts proliferation}
Glucocorticoid treatment is usefull in rheumatic arthritis. What is the basal action of glucocoricoids? {=Cell
membranes stabilization Histamine decreasing
~Emigration inhibition
~Phagocytosis inhibition
~Prostaglandins synthesis inhibition
~Histamine decreasing}

A patient with abscess of finger has some local and general symptoms of inflammation. What from following is
a local inflammatory sing?
{=Swelling
~Fiver
~Headache
~Lymphangitis
~Leucocytosis}

It is known that inflammatory mediators are cellular or humoral. What from the following is a humoral
mediator? {=Kallidine
~Histamine
~Serotonin
~Interleukine-2
~Hydrogenium peroxide}

A patient with inflammation of trigeminal nerve has been having progressive paradontitis for some years. What
factor is the most important for parodontitis development?
{=Neurodistrophic disorders
~Low activity of leukocytic elastase
~Poor formation of immunoglobulins
~Increased tone of vagus nerve
~Low activity of kallikrein-kinin system}

An animal sensitized with tuberculin had been administered tuberculin intraperitoneally. 24 hours later, during
laparotomy, the animal was found to have venous congestion and peritoneal edema. Impression smears from the
peritoneum contained a large number of lymphocytes and monocytes. What pathological process was detected in
the animal?
{=Allergic inflammation
~Serous inflammation
~Purulent inflammation
~Fibrinous inflammation
~Aseptic inflammation}

A 31-year-old patient has deep caries of the fifth maxillary tooth on the right. Acute purulent inflammation in the
region of the tooth apex has developed. There is marked edema with isolated neutrophils in the perifocal tissues
and soft tissues of cheek and palate. Diagnose the pathologic process in the soft tissues of cheek and palate:
{=Acute serous inflammation
~Acute suppurative periodontitis
~Acute suppurative periostitis
~Phlegmon
~}
A 7-year-old child suffers from acute disease. A pediatrician observed the following during examination: pharynx
mucosa is hyperemic, edematous, swollen, cowered with large amount of mucus. Buccal mucosa has whitish
discoloured spots. On the next day the child came out in rash with large spots covering the skin of his face, neck
and torso. What kind of inflammation causes changes in pharynx mucosa? {=Catarrhal
~Serous
~Serofibrinous
~Fibrinous
~Hemorrhagic}

A woman has scalded her hand with boiling water. The affected area of her skin became red, swollen and painful.
This effect is caused by accumulation of the following substance: {=Histamine
~Lysine
~Thiamine
~Glutamine
~Asparagine}

A patient complains of toothache. On examination he has been diagnosed with pulpitis. Which factor played the
main pathogenic role in the development of pain syndrome in this case?
{=Vasospasm
~Activation of one of the complement system components
~Interleukin action
~Increased intratissular pressure in the dental pulp
~Inadequate stimulation of the mandibular nerve branch}

Toxic pulmonary edema was reproduced on a laboratory rat by means of ammonium chloride solution. What is
the leading pathogenetic factor of this edema? {=Increased permeability of capillars
~Disorder of neural and humoral regulation
~Increase of lymph outflow
~Increase of venous outflow
~Decrease of colloid osmotic pressure}

A patient was stung by a bee. Examination results: his left hand is hot, pink and edematic, there is a big blister on
the spot of the sting. What is the leading mechanism of edema development?
{=Increased vascular permeability
~Vascular damage caused by the sting
~Reduction of osmotic pressure of tissue
~Reduction of oncotic pressure of tissue
~Reduced blood filling of vessels}

It is known that patients with diabetes mellitus are more subject to inflammative processes, they have low
regeneration and slower wound healing. What is the reason for this?
{=Decrease in protheosynthesis
~Accelerated gluconeogenesis
~Intensification of catabolism
~Decrease in lipolysis
~Increase in lipolysis}

A 35-year-old patient complains of pain in the upper jaw, bleeding, a slight loosening of teeth. He has been
diagnosed with periodontitis. What is a typical pathological process in this case? {=Inflammation
~Bleeding
~Caries
~Pain
~Redness }

Autopsy of a 58-year-old man, who had been suffering from rheumatic heart disease and died of cardiopulmonary
decompensation, revealed gray diffuse film- and fiber-shaped coating in his pericardium. What type of
inflammation is characteristic of this pericarditis?
{=Diphtheritic fibrinous
~Suppurative ~Serous
~Hemorrhagic
~Croupous fibrinous}

What factor results in maximal dilation of the gemomicrocirculatory pathway vessels and their increased
permeability? {=Histamine
~Endothelin
~Vasopressin
~Noradrenaline
~Serotonin}

A cook burnt his arm with steam. What substance increased and led to development of redness, edema and
painfulness of affected area of skin?
{=Histamine
~Kalidin
~Thiamine
~Galactosamine
~Glutamine}
A 12-year-old patient suffering from acute leukemia presents with fever up to 39, 80C, acute pain in the throat.
Examination of the oral cavity has revealed swollen tonsils, their surface is covered in deep lesions with uneven
margins, numerous petechial hemorrhages in the pharyngeal mucosa and around the tonsils. Determine the type
of tonsillitis that complicates the disease progress in this case:
{=Necrotic
~Catarrhal
~Fibrinous
~Lacunar
~Purulent}

A 7-year-old child fell ill sharply. Hi has catarrh, cough, lacrimation and large-spot rash on skin; his temperature
is increased up to 38°C. Mucous membrane of patient’s pharynx is edematous and red; buccal mucous membrane
has whitish spots. What is the character of inflammation underlying changes of buccal mucous membrane?
{=Fibrinous
~Serous
~Purulent
~Catarrhal
~Hemorrhagic}

Research of urgentic patient was observed presence in him neutrophilic leucocytosis with the change of leucocytic
formula to the left. In the patient the most likely
{=Purulent inflammation
~Tuberculosis
~Allergic reaction
~Agranulocytosis
~Alimentary leucocytosis}

Enlargement and deformation of joints were revealed in a patient with rheumatism. What type of inflammation
underlies these changes?
{=Proliferetive
~Alterative
~Exudative
~Fibrinous
~Hemorrhagic}

A painful blisters full of opaque fluid has formed in patient after thermal burn. What type of inflammation has
appeared? {=Serous
~Croup
~Granulomatous
~Proliferative
~Diphtheritic}

Prevalence of proliferative processes was revealed in a patient with chronic inflammation of skin and
subcutaneous adipose tissue. Which hormone deficiency can lead to this situation? {=Cortisol
~Aldosterone
~Insulin
~Growth hormone
~Thyroxin}

A patient complains of a pain in joints. Edema and redness are visible above joints. Which enzyme activity has
to be analyzed for establishing the diagnosis?
{=Hyaluronidase
~Creatine kinase
~Alkaline phosphatase
~Acid phosphatase
~Urease}

Condition of biological active substances (BAS) prevalence over there inhibitors usually occurs in inflammation.
Indicate correct correspondence of BAS to their inhibitor
{=Leukotriens - arylsulfatase
~Catecholamines - cholinesterase
~Kinins - monoaminooxidase
~Histamine - carboxypeptidase
~Serotonin-protease inhibitor}

Edema has developed as a result of oncotic pressure elevation in the site of inflammation. What is the main reason
of hyperoncia?
{=Activation of endotheliocytes
~Protein catabolism
~Drainage disorders
~Macromolecules accumulation
~Gluconeogenesis intensification}
What factor promotes proliferation in focus of chronic inflammation?
{=Hypoxia
~Hyperosmia
~Protein catabolism
~Hyperoncia
~Prostacyclin synthesis}

A patient has high body temperature, redness, edema, painfulness on her right forearm. What biological active
substances intensify inflammatory reaction?
{=Kinins
~Prostacyclins
~Phospholipase D
~Proteolysis inhibitors
~Vasopresin}

A patient has an inflammatory process on the outer surface of calf. Redness, edema, pain, enlargement of local
lymph nodes, and body temperature rising are noticed in patient. What are the principles of pathogenetic therapy
of acute inflammation? {=Proteolysis activation blockage
~Energy formation processes improving
~Free radical oxidation increase
~Introduction of mineralocorticoids
~Introduction of thyroid hormones}

A patient with finger inflammation has sharp pain and edema in the inflammatory focus. After opening of an
abscess these symptoms have disappeared. What favors disappearance of pain?
{=Decrease of osmotic pressure
~Increase in c-AMP concentration
~Alkalosis
~Kallikrein-kinin system activation
~Mast cells degranulation}

There are edema, redness and soreness in a place of injury. What is the reason for inflammation’s symptoms
appearance?
{=BAS discharged by cells
~Monoamine oxidase activation
~Sympathetic nervous system activation
~Carboxy peptidase activation
~Limited proteolysis inhibition}

Female patient, aged 32, was stung by wasp. On the skin of left cheek (place of sting) there is a zone of hyperemia
and edema. What is the primary mechanism of edema in this case?
{=Increase in capillary permeability
~Lymph drainage decease
~Increase of oncotic pressure
~Decrease of oncotic pressure
~Increase of hydrostatic pressure in capillaries}

What inflammatory mediator is formed due to limited proteolysis of plasma globulins? {=Bradykinin
~Leukotriens
~Histamine
~Prostaglandins
~Lymphokines}

Fever and increase of antibodies and leukocytes have appeared in animal under experimental modeling of
inflammation. What substances conditioned to all these common reactions in inflammation? {=Interleukins
~Interferons
~Mineralocorticoids
~Glucocorticoids
~Somatomedins}

A patient sustained trauma of knee joint and posttraumatic hemorrhagic bursitis appeared. After 3 months passive
movements limitation in extend was observed. Limitation was a result of scar formation. What inflammatory
component was a basis of this complication?
{=Proliferation
~Exudation
~Tissue hyperplasia
~Secondary alteration
~Primary alteration}

In female patient, 28, abscess opening was performed and fast wound reparation was observed. Reparation was
preceded without score formation. What cells play the main role in proliferation? {=Fibroblasts
~Neutrophils
~Eosinophils
~Lymphocytes
~Monocytes}

In examination of abscess punctate under a microscope different blood cells were revealed. Which of them
appears the first in inflammatory focus?
{=Neutrophils
~Mast cells
~Eosinophils
~Monosytes
~Lymphocytes}

What is the main factor of pain formation in pulpitis?


{=Hydrostatic pressure increasing
~Hydrostatic pressure decreasing
~Excess of pain receptors excitability
~Ischemia
~Excess of brain centers sensitivity}

What is the main factor of edema formation in pulpitis?


{=Capillaries permeability increasing
~Hypoproteinemia
~Tissue hyperosmia
~Blood hyperosmia
~Capillaries hydrostatic pressure increasing}

Which of following inflammatory mediators are formatted under the influence of lypooxygenases? {=Leukotriens
~Prostaglandins El, E2
~Prostacyclins
~Thromboxans
~Thrombocytes activation factor}

Patient has a caries complicated with pulpitis. This condition is accompanied by unbearable pain. What is the
main reason of pain? {=Emigration
~Secondary alteration
~Primary alteration
~Exudation
~Proliferation}

It is known, that inflammatory mediators play the main role in inflammation pathogenesis. What is the histamine
action on inflammation? {=Blood vessel permeability increasing
~Chemotaxis
~Thrombocytes aggregation
~Blood vessels constriction
~Blood coagulation}

Female patient, 19, complains of pain in gums, gingival hemorrhage. These symptoms have appeared 3 days after
sulphamide drugs taking. Objectively: gingival papilli and gum margin are reddened and swollen; there is a
hemorrhage appeared in response of touching. What type of inflammation is observed in patient? {=Hemorrhagic
inflammation
~Acute cattharal inflammation
~Fibrinous inflammation
~Pus inflammation
~Chronic cattharal inflammation}

A patient 6 years old was admitted to the hospital with asphyxia. The membranous coats on mucosa of fauces
were revealed. The coats can be removed easy. What type of inflammation is in patient? {=Fibrinous
~Necrotic
~Pus
~Cattharal
~Hemorrhagic}

There are lots of neutrophils in abdominal pus exudates in a patient with peritonitis. What is the main function of
neutrophils in inflammatory area?
{=Phagocytosis
~Prostaglandins secretion
~Degranulation
~Histamine liberation
~Local blood flow regulation}

Keloid cicatrix has formatted in a place of abscess. Name the stage of inflammation, which caused scar
appearing. {=Proliferation
~Exudation
~Primary alteration
~Secondary alteration
~Emigration}
A male patient, 16, was admitted to the hospital with acute appendicitis. What typical pathological process is the
basis of this disease?
{=Inflammation
~Hypoxia
~Fever
~Tumor
~Allergy}

Eyeball inflammation resulted pus in the anterior chamber of the eye (hypopyon). What stage of inflammation
was it?
{=Exudation
~Proliferation
~Primary alteration
~Secondary alteration
~Emigration}

In a patient with skin pathological process which has 5 typical symptoms: tumor, rubor, calor, dolor, functio laesa.
The development of such process includes 3 stages: Alteration, _? _ , proliferation. What is the second one?
{=Exudation
~Regeneration
~Reparation
~Fibrosis
~Petrification}

The secondary alteration is caused by:


{=Lysosomal enzymes
~Kinins
~Lympokynes
~Prostaglandins
~Complement components C3a and C5a}

In proliferation cells multiplying and defect compensation take place. What is the reason of cells growth in
specific clones?
{=Keylones
~Platelet-derived grow factor
~Interleukins
~Somatomedine
~Lymphocytes peptide}

There are following components in pleural effusion: proteis - 62g/, abundance of neutrophils. What is the type
of inflammation in this case?
{=Serous
~Suppuratuve
~Fibrinous
~Hemorrhagic
~Putrefactive}

Necrotic focus has appeared on burn, swallowing and red skin. What is the main mechanism of necrobyosis
improvement in inflammatory area?
{=Secondary alteration
~Primary alteration
~Emigration of leucocytes
~Dyapedesis of erythrocytes
~Fibroblasts proliferation}

Glucocorticoid treatment is usefull in rheumatic arthritis. What is the basal action of glucocoricoids? {=Cell
membranes stabilization Histamine decreasing
~Emigration inhibition
~Phagocytosis inhibition
~Prostaglandins synthesis inhibition
~Histamine decreasing}

A patient with abscess of finger has some local and general symptoms of inflammation. What from following is
a local inflammatory sing?
{=Swelling
~Fiver
~Headache
~Lymphangitis
~Leucocytosis}

It is known that inflammatory mediators are cellular or humoral. What from the following is a humoral
mediator? {=Kallidine
~Histamine
~Serotonin
~Interleukine-2
~Hydrogenium peroxide}
Likely reason of disorders?
{=Deficit of transcortin
~Disorder of metabolism in the liver
~Surplus production of adrenals
~Activation of blood complement
~Liberation of peptidases from cells}

After the carried hepatitis in a patient autoimmune reaction developed as a result of functional disorder in the liver
{=Star macrophages
~Pit cells
~Endotheliocytes
~Hepatocytes
~Fibrocytes}

After the carried hepatitis in a sick child the signs of hypovitaminosis B1 appeared as a result of {=Disorder
of formation of active forms it
~Decrease of absorption it in bowels
~Compete use by microflora
~Insufficient receipt with meal
~Forsing out by ascorbic acid}

After the resection of stomach in a patient in connection with the duodenum ulcer the malabsorption syndrome
developed, reason of which became
{=Diminishing of microvillus amount of thin bowel
~Deficit of villicrinin hormone
~Strengthening of bowel peristalsis
~Activation of fermentative processes
~Loss of electrolytes and water}

After transfered infection with diarrhea and developed malabsorption syndrome with decrease of aminoacids and
glucose adaption. The main role in pathogenesis of activity transport their belong to ions deficit? {=Na
~K
~Mg ~Ca
~CH

At fibroscopy of stomach was detected ulcer in antral part, which associated with contamination Helicobacter
pylori. Role this agent in ulcer formation is led to.
{=Destruction of mucous barrier
~Inhibition of mucous regeneration
~Disorder of mucous microcirculation
~Stimulation of HC] secretion
~Stimulation of pepsin secretion}

In a child got illness on angina was disordered the use of meal as a result of disorder of process {=Swallowing
~Mastication
~Salivation
~Blood supply
~Lymph outflow}
For differentiation of functional disorders of stomach secretion from organic lesion to patient is proposed tests
with straight stimulation of secretion. The most strong stimulating action of secretory cells mucous membrane
cause: {=Gastrin
~Cholecystokinin
~Acetylcholine
~Insulin
~Histamine}

For modeling of ulcer stomach to animal injected in to gastric arteria atophan, which arise sclerosing their. What
mechanism of mucous membrane stomach injury damage is initial in that experiment? {=Hypoxic
~Neurodystrophic ~Mechanical.
~Dysregulative
~Neurohumoral}

In a experimental animal was destroyed one pert of brain. Due to that in the animal bulimia development. What
part of brain have relation development of that state?
{=Hypothalamus
~Thalamus
~Midbrain
~Medulla oblongata
~Cerebellum}

In a man 26 years was revealed stomach dyskinesia on the hyperkinetic type. Reason of stomach motor
disturbances may be {=Increase of vagus tone
~Activating of sympatic nerves
~Systematic use of alcohol
~Increase of motiline level in the blood
~Acidifying of duodenum contents}

In a men 25-year-old which got closed craniocerebral trauma was observed long unrestrained vomiting. What
mechanism stipulated vemiting act
{=Straight irritance of vomiting centre by increased intracranial pressure
~Straight excitement of vomiting centre by toxic substances of blood
~Straight excitement of vomiting centre by impulses from stomach and intestine
~Activation of vomiting centre as a result of movement and secretory stomach function increase ~Excitemeut
of vomiting centre through branches of sympathic nerves}

In a men 35-year-old with ulcer was performed resection of stomach antral part. Secretion of what gastrointestinal
hormone will be disordered in a result of operation? {=Gastrin
~Neurotensin
~Secretin
~Histamine
~Cholecystokinin}
In a men 40-year-old was revealed stomach ulcer after hard emotion stress. What is main mechanism of it
development?
{=Dysbalance between aggressive factors and protective of stomach mucous membranE) ~Mucus
hyperseretion
~Infection of mucus membrane
~Achlorhydria
~Increase of prostaglandine E producing}

In a men 48-year-old which suffers of stomach ulcer was determined secretion increase and increase of gastric
juice acidity . What is mechanism stipulate this phenomenon?
{=Increase histamine output
~Increase secretin output
~Increase motilin output
~Increase catecholamines output
~Increase cholecystokinin output}

In a men 50-year-old which supffer from stomach ulcer was determined increase of secretion and acidity of gastric
juicE) What is mechanism this phenomenon?
{=Increase vagus activity
~Increase sympathic nerves activity
~Decrease vagus activity
~Decrease sympathic nerves activity
~Decrease blood gastrin contents}

After taking a fatty food a patient feels nausea, flaccidity, later the sign of steathorrhea has appeared, cholesterol
in the blood 9,2ml/c. The cause of this state is the deficiency of:
{=Bile acids
~Triglyceroides
~Chylomicrones
~Fatty acids
~Phopholipids}

A patient aged 45 had the diagnosis ulcer of the stomach. On examination of secretory function of the stomach it
was determined that the amount of basal secretion was 100mole/hr, acidity of basal secretion - 6O0mml/hr. What
factors action contributes to the hypersecretion in the stomach? {=Gastrine
~Pancreatic polypeptide
~Somatostatin
~Glucagon
~Betaendorphin}

On laboratory examination increased amount of diastase in the urine and also a large amount of undigested fat in
stool were revealed in a patient female with complain of circular character pain in epigastric area. What form of
gastrointestinal tract pathology are described signs typical for?
{=Acute pancreatitis
~Acute appendicitis
~Infectious gastritis
~Ulcerous disease of the stomach
~Inflammation of large intestine}

What enzyme deficiency is the cause of maldigestion of fats in the gastrointestinal tract and increase of neutral
fats in the stool? {=Pancreatic juice
~Intestinal lipase
~Hepatic lipase
~Enterokinase
~Gastric lipase}

Decrease of synthesis and secretion of trypsin is observed in chronic pancreatitis. The splitting of what substances
is broken?
{=Proteins
~Nucleic acids
~Polysaccharides
~Lipids
~Liposoluble vitamins}

In coprologic investigation it is determined that stool is discoloured, there are drops of neutral fat in it. The most
possible cause of this is the impairment of?
{=Entering of bile into the intestine
~Process of absorption in the intestine
~Secretion of intestinal juice
~Acidity of gastric juice
~Secretion of pancreatic juice}

A patient had been taking antibiotics of a wide spectrum of action for a long period of time that caused decrease
of appetite, nausea, and diarrhea with saprogenic smell. What is the side effect of treatment? {=Dysbacteriosis
~Allergic reaction
~Hepatotoxic action
~Nephrotoxic action
~Direct irritative action}

On examination of a patient suffering from acute pancreatitis increased amount of chylomicrons was determined
in the blood. What enzyme activity is sharply decreased in this pathology? {=Lipoprotienelipase
~Pancreatic lipase
~Pancreatic phospholipase
~Tissue triglyceride lipase
~Tissue diglyceride lipase}

The analysis of gastric juice of an elderly man who complained of unmotivated weakness, sickness, absence of
appetite showed achylia, achlorhydrea, and presence of lactic acids and coagulated blood, decreased of pepsin
secretion. What disease causes such clinical-laboratory symptoms?
{=Cancer of the stomach
~Chronic gastritis
~Chronic pancreatitis
~Cavitary Maldigestion
~Acute gastritias}

A part of patient pancreas was resected. What kinds of product must be limited in his diet? {=Fatty
and fried meal
~Fruits
~Not fatty boiled meat
~Sour milk product
~Vegetables}

A man with chronic hepatitis has dyspeptic disorders: decrease of appetite, nausea, unstable stool, and steatorrhea.
What is the mechanism of dyspeptic disorders in hepatic pathology? {=Hypocholea
~Hypoglycemia
~Cholalemia
~Intoxication
~Hyperbilirubemia}

Spasmodic pains in the abdomen and repeated diarrhoea with mucus appeared in a healthy person 3-5 hours later
after taking meals. This was preceded by nausea and momentaneous vomiting, general weakness, loss of appetite.
What is the most possible cause of the desired symptoms?
{=Food intoxication
~Chronic pancreatitis
~Enterocolitis
~Hyperacid state of the stomach
~Chronic gastritis}

A man who works at a storage battery plants complains of constant feeling of weight and periodical spasmodic
pains in the abdomen, constant retention of stool (not more often than one time per three day). This is accompanied
by frequent headaches, flaccidity, absence of appetite, and bad taste in the mouth. What are the causes of these
disorders?
{=Spastic lead colie with constipation
~Hypoacid state of the stomach
~Hyper acid state of the stomach
~Chronic pancreatitis
~Parietal maldigestion}

Due to chronic gastritis a man has the impaired structure of the mucous membrane, decreased indices of acid
formation function of the stomach. The most essential negative result of this will be the impairment of: {=Protien
digestion
~Pancreatic juice secretion
~Secretory function of small intestine
~Evacuation of chyme into duodenum
~Excretion of secretum}

On laboratory examination increased amount of diastase in the urine and also a large amount of undigested fat in
stool were revealed in a patient with complains of circular character pains in epigastric area. What form of
gastrointestinal tract pathology are the described signs typical for?
{=Acute pancreatitis
~Acute appendicitis
~Infectious hepatitis
~Ulcerous disease of the stomch
~Inflammation of the large intestine}

A patient aged 35 with ulcerous disease had a rejection of antral portion of the stomch. What gastrointestinal
hormone secretion will be impaired due to operation?
{=Gastrin
~Secritin
~Neurotensin
~Histamine
~Cholecystokinin}

A patient aged 57 was admitted to a gastrointestinal department with suspicious on Zolliger-Ellison syndrome,
which was proved by sharp increase of gastrin level in the blood serum. What impairment of secretory gastric
function is the most possible? {=Hyperacid hypersecretion
~Hyperacid hyposecretion
~Hypoacid hypersecretion
~Achylia
~Hypoacid hyposecretion}

A patient with signs of encephalopathy was hospitalised in neurological in- patients department and co relation
between increase of encephalopathy and substances passing from intestine into systemic blood circulation was
determined. What compounds formed in the intestine may cause endotoximia? {=Indole
~Biotin
~Acetoacetat
~Omnitin
~Buthirat}

150 ml of meat broth were introduced into the stomach cavity of experimental dog to a probe the content of what
substance will increase quickly in the animal’s blood? {=Gastrin
~Vasointestinal peptide
~Neurotensin
~Somatostatin
~Insulin}

A patient age 37 was admitted into a surgical department with the signs of acute pancreatitis; vomiting, diarrhea,
bradycardia, hypotension wearness, dehydration of the organism. What preparation must be used first of all?
{=Contrical
~No-spa
~Pephenazine
~Ephedrine
~Platy phy llin}

A patient has a stone in the common bile duct, which has stopped passing bile into the intestines. The impairment
of what digestive process is observed in this case?
{=Digestion of fats
~Absorption of carbohydrates
~Digestion of carbohydrates
~Absorption of proteins
~Digestion of proteins}

On fibroscopy of the stomach ulcer was revealed in antrum portion, which was associated with dissemination of
mucosa by Helicobacter pylori. The role of this agent in the formation of ulcer results in: {=Damage of mucous
barrier
~Inhibition of mucosa regeneration
~Impairment of microcirculation in mucous
~Stimulation of HCL secretion
~Stimulation of pepsin secretion}

A patient with ulcer of stomach has impairment of equilibrium between the factors of aggression and defense.
What factor contributes to the development of gastric ulcer?
{=Helicobacter pylori
~Mucin
~Bicarbonate
~Prostaglandin E2
~Prostacyclin}

After resection of duodenum a patient has developed a syndrome of duodenal in sufficiency due to the impairment
of its endocrine function with signs of cell insufficiency of APUD system. What hormone is produced by A-cells
of this portion of intestine?
{=Glucagon
~Insulin
~Seratonin
~Secrtin
~Histamine}

A patient complains of nausea, which often ends with vomiting. These dyspeptic phenomena become more
laborious with time. What stomach function is infringed in this patient?
{=Evacuation
~Motility
~Reservoir
~Incretory
~Secretory}

A patient aged 42 was admitted to the hospital with gastric hemorrhage (ulcerous disease was excluded by a
patient). Ulcer and hyperplasia of gastric mucose were revealed. What investigation is it necessary to carry out to
confirm the diagnosis of Zollinger-Ellison syndrome in this patient?
{=Definition of gastrin level in the blood
~Tomography of pancreas
~Definition of secretin level in the blood
~Definition of activity level of pancreatic enzymes
~Investigation of activity level of pancreatic secretion}
The amount of protein in the blood was examined in a patient after resection of sizeable part of small intestine.
What change of protein amount may be expected in this patient?
{=Hypoproteinemia
~Hyperproteinemia
~Paraproteinemia
~Hypergammaglobulinemia
~Dysproteinemia}

Ulcerous disease of the stomach is revealed in a woman aged 52, who has been ill with contact dermatitis. On
examination it is determined that the patient had been taking corticosteroid preparations for a long period of time,
but acidity of gastric juice was increased. What mechanism caused the increase of gastric secretion? {=Decrease
of prostaglandin content
~Increase of secretin secretion
~Gastrin hyposecretion
~Increase of histamine content
~Increase of gastrin excretion}

A patient complains of vomiting, eructation, pain in epigastric area, constipation, and emaciation. On examination
a basal secretion of HCL is 6mml/gr; maximal one is 60mml/gr. What state has acid-forming function of this
patient?
{=Hyperacid
~Anacid
~Hypoacid
~Hypersecretory
~Normacid}

A patient aged 42 complains of pains in epigastric area, vomiting; vomitory masses have color of “coffee
grounds”, melena. In anamnesis there is ulcerous disease of the stomach. On examination: skin is pale, heart rate-
110 beats per minute; AP-90/50mm.hg. Blood analyses: erythrocytes-2.8 x 1012/1; leucocytes-8x109/);
hemoglobin-90gr/l. What is the possible complication that developed in the patient? {=Hemorrhage
~Perforation
~Penetration
~Cancerous degeneration
~Pylorostenosis}

The investigation of pancreatic function was carried out in experimental animal by radioisotopic method. The
percentage of excreted with stool 1131- albumin is 53%. This is the evidence of:
{=Inhibition of exocrin function
~Increase of incretory function
~Inhibition of incretory function
~Increase of exocrine function
~Normal function}

Malabsorption syndrome is:


{=Syndrome of intestinal malabsorption
~Syndrome of gastric malabsorption
~Syndrome of malabsorption in small intestine
~Syndrome of membraous maldigestion
~Malabsorption of proteins}

A 67-year-old woman, who has cholecystitis for a long time, suddenly developed sharp pain in the upper part of
abdomen, nausea, and vomiting after food intake. Acute pancreatitis was diagnosed in this patient. What is the
main link in pathogenesis of this disease? {=Preliminary activation of pancreatic enzymes
~Decrease in enzyme levels in pancreatic juice
~Intensification of enzyme activation in duodenum
~Reduction of pancreatic polypeptide secretion
~Increased level of cholecystokinin}

A patient has increased gastric secretion in response to both mechanical and chemical stimuli. He has high acidity
of gastric juice with pH equals 2.0 on an empty stomach. After the breakfast pH restores to normal in 12 minutes
and is continuously dropping during following 2 hours. What type of stomach secretion is being observed in this
patient?
{=Excitable
~Brakable
~Inert
~Asthenic
~Normal}

Frequent liquid stool, signs of intoxication, and dehydration developed ina newbom at 5-6 day. After this newborn
was fed by acid-milk mixtures indicated symptoms disappeared. The conclusion of innate lactose deficiency was
made. What process was broken in this newborn?
{=Membrane digestion
~Cavital digestion
~Excretory function of intestine ~Secretory
function of pancreas
~Secretion of gastric juice}

Ulcers of stomach were found out in a patient, who was treated with glucocorticoids for a long time. What is the
main mechanism of ulcer development in this case? {=Increase in secretion and acidity of gastric juice
~Decrease in histamine concentration in stomach wall
~Increased tonus of sympathetic nervous system
~Increased production of prostaglandins E
~Decreased tonus of parasympathetic nervous system}

A patient with gastric juice hypersecretion has been recommended to exclude from the diet rich broths and
vegetable infused water. A doctor recommended it, because these food products stimulate production of the
following hormone:
{=Gastrin
~Secretin
~Cholecystokinin
~Somatostatin
~Neurotensin}
A patient was admitted to a hospital with poisoning with unsound food. His stomach was lavaged with solution
of potassium permanganate. What is its mechanism of action?
{=Release of atomic oxygen
~Release of chlorine
~Release of iodine
~Disturbance of synthesis of respiratory chain enzymes
~Destruction of bacteria membranes}

A newborn has signs of dyspepsia after milk feeding. Symptoms of dyspepsia disappear when milk is substituted
for glucose solution. The newborn has low activity of the following enzyme: {=Lactase
~Maltase
~Isomaltase
~Amylase
~Invertase}

A 42-year-old patient with gastric ulcer has a disbalance between the aggressive and defensive factors. Which of
the following factors contributes to the development of gastric ulcer? {=Helicobacter pylori
~Mucin
~Hydrocarbonate
~Prostaglandin
~Prostacyclin}

Denture installation has caused excessive salivation in patient. It is caused by the following reflexes:
{=Unconditioned
~Conditioned
~Conditioned and unconditioned
~Local
~}
A patient is diagnosed with pancreatic diabetes with associated hyperglycemia. Glycemia rate can be assessed
retrospectively (4-8 weeks prior to examination) by measuring concentration of the following blood plasma
protein:
{=Glycated hemoglobin
~Albumin
~Fibrinogen
~C-reactive protein
~Ceruloplasmin}

After examining the patient the doctor recommended him to eliminate rich meat and vegetable broth, spices,
smoked products from the diet, since the patient was found to have: {=Increased secretion of hydrochloric acid
by the stomach glands
~Reduced secretion of hydrochloric acid by the stomach glands
~Reduced motility of the gastrointestinal tract
~Reduced salivation
~Biliary dyskinesia}
Roentgenologically confirmed an obstruction of common bile duct that prevents bile from inflowing to the
duodenum. What process is likely to be disturbed?
{=Fat emulgation
~Protein absorption
~Carbohydrate hydrolysis
~Hydrochloric acid secretion in stomach
~Salivation inhibition}

A 2-year-old child presents with mental development retardation, intolerance of proteins, severe
hyperammonemia against the background of low blood urea content. This condition is caused by the congenital
deficiency of the following mitochondrial enzyme:
{=Carbamoy] phosphate synthetase
~Citrate synthase
~Succinate dehydrogenase
~Malate dehydrogenase
~Monoamine oxidase}

A 30-year-old woman has decreased enzyme content in the pancreatic juice. This condition can be caused by
insufficient secretion of the following hormone:
{=Vasoactive intestinal peptide
~Cholecystokinin-pancreozymin
~Secretin
~Somatostatin
~Gastric inhibitory polypeptide}

The patient’s saliva has been tested for antibacterial activity. What saliva component has antibacterial
properties? {=Lysozyme
~Parotin
~Cholesterol
~Amylase
~Ceruloplasmin}

A 40-year-old patient suffers from intolerance of dairy products. This condition has likely developed due to
insufficiency of the following digestive enzyme:
{=Lactase
~Amylase
~Maltase
~Lipase
~Invertase}

After examining a patient a doctor recommended him to exclude rich meat and vegetable broths, spices, and
smoked products from the diet, since the patient was found to have: {=Reduced secretion of hydrochloric acid by
the stomach glands ~-
~Increased secretion of hydrochloric acid by the stomach glands
~Reduced salivation
~Biliary dyskinesia}
Roentgenologically confirmed obstruction of common bile duct resulted in preventing bile from inflowing to the
duodenum. What process is likely to be disturbed?
{=Fat emulgation
~Protein absorption
~Carbohydrate hydrolysis
~Hydrochloric acid secretion in stomach
~Salivation inhibition}

A 30-year-old woman complains of intense thirst and dryness of the mouth that developed after a severe emotional
shock. Laboratory analysis revealed increase of the patient’s blood sugar level up to 10 mmol/L. What endocrine
gland is affected in the patient?
{=Pancreas
~Thyroid gland
~Gonads
~Adrenal glands
~Pineal gland}

During tooth brushing it is not uncommon for oral mucosa to be injured. However, bleeding quickly stops on its
own. What substances in saliva quickly staunch the flow of blood during minor oral injuries? {=Lysozyme and
mucin
~Mineral substances ~Lipolytic
enzymes
~Procoagulants
~Amzylolytic enzymes}

A patient is diagnosed with pancreatitis. Starch decomposition disturbance occurs in the patient’s intestine due to
deficiency of the following pancreatic enzyme:
{=Tripsin
~Amylase
~Chymotrypsin
~Carboxy peptidase
~Lipase}

An experimental animal, a dog, received a weak solution of hydrochloric acid through a tube inserted into the
duodenum. Primarily it will result in increased secretion of the following hormone: {=Gastrin
~Secretin
~Histamine
~Cholecystokinin
~Neurotensin}
A 12 year old boy came home from school and started complaining of headache, sickness, chill, periodical muscle
pain, appetite loss, flabbiness. What period of illness are these symptoms typical for? {=Prodroma
~Latent
~Incubative
~High point of illness
~End of illness}

A patient dropped into an ice hole, froze in the wind and fell ill. Body temperature rose up to 39,7°C and varied
from 39°C to 39,8°C. Name the type of the patients temperature profile? {=Febris continua
~Febris hectica
~Febris recurrens
~Febris intermittens
~Febris remittens}

Pyrogenal administered to a rabbit, in the course of an experiment, resulted in increase of its body temperature.
What substance of those named below acts as a secondary pyrogen that is a part of feverinducing mechanism?
{=Interleukin 1
~Pseudomonas polysaccharide (Piromen)
~Histamine
~Bradykinin
~Immunoglobulin}

A 40-year-old woman suffering from diffuse toxic goiter presents with constant increase of her body temperature.
What mechanism results in such clinical presentation? {=Separation of oxidation and phosphorization in cell
mitochondria
~Increased breakdown of glycogen in hepatic cells
~Increased catabolism of protein in cells
~Increased excitability of nerve cells
~Increased cell sensitivity to catecholamines}

A man submerged into the ice cold water and died soon as a result of abrupt exposure to cold. In such cases an
organism loses heat most intensively by the way of:
{=Heat conduction
~Radiation
~Convection
~Heat conduction and radiation
~No correct answer}

An accident had resulted in a 65-year-old man drowning in a lake. Resuscitation measures allowed to restore his
respiration and cardiac function. What factor prolongs the period of apparent death? {=Hypothermia
~Elderly age
~Hyperthermia

~Prolonged preagony and agony}

In hot weather the bus passengers asked to open the roof hatches. What way of heat transfer is activated in this
situation?
{=Convection
~Conduction
~Radiation
~Conduction and radiation
~Sweat evaporation}

The body temperature of a patient with an infectious disease rises once in two days up to 39,5-40,5°C and stays
so for about an hour and then drops to the initial level. What type of fever is it? {=Intermittent
~Continued
~Remittent
~Hectic
~Atypical}

In the armpits of a patient there are small (1-1.5 mm), dorsoventrally flattened, wingless, blood-sucking insects.
Their larvae have been developing in the armpits as well. What disease is caused by these insects? {=Relapsing
fever ~Phthiriasis ~Chagas’ disease
~Sleeping sickness
~Plague}

A 52-year-old woman complains to bad pain around her abdomen and back. Acute appendicitis was diagnosed.
There is suspicion of complication of pancreatitis by abscess. Which type of fever would corroborate appearance
of abscess?
{=Remittent
~Continuous
~Hectica
~Intermittent
~Atypical}

Polyuria developed in a patient with fever after period of olyguria. What is the leading factor of diuresis change
at the end of the second period of fever?
{=Normalization of hormonogenesis by fascicular zone of adrenal cortex
~Normalization of hormonogenesis by reticular zone of adrenal cortex
~Normalization of hormonogenesis by glomerular zone of adrenal cortex
~Normalization of hormonogenesis by fascicular zone of adrenal medulla ~Normalization
of thyroid gland function}

A patient has felt cold, chills, “goose flesh”, increase of body temperature. Which else changes characterize the
first period of rapid elevation of body temperature? {=Equilibration between heat production and heat loss
~Tachycardia
~Dilation of skin vessels
~Decrease of arterial pressure
~Increase of metabolism on 100-200%}

Body temperature of patient becomes pyretic. Which substances have to act to neurons of thermoregulation for
fever development?
{=Kallidinum
~Interferon
~Prostaglandins
~Free radicals
~Leucotriens}

Fever in a patient develops in following succession of stages:


{=Incrementi; fastigii; decrementi
~Incrementy; decrementy; fastigii
~Fastigii; decrementi; incrementi
~Fastigii; incrementi; decrementi
~Decrementi; fastigii; incrementi}

The temperature of a patient with infectious disease increased to 39.5-40.5°C in a day and kept that level about 1
hour, but then it returned to the normal level. Which type of fever is described in that case? {=Remitten
~Intermittent
~Continuous
~Recurrent
~Atypical}

The body temperature of a patient with infectious disease increased to 39.5-40.5°C in a day and kept this level
about 1 hour, but then became normal again. Which disease is characterized by described type of fever?
{=Tuberculosis
~Influenza
~Peritonitis
~Brucellosis
~Malaria}

A patient had fever after injection of pyrogenal. His skin has become pale, cold; chill appeared in him, oxygen
consumption increased. How do the processes of thermoregulation change in described period of fever?
{=Increase of heat production and decrease of heat loss
~Decrease of heat loss
~Heat loss is equal heat production
~Decrease of heat production and increase of heat loss
~Decrease of heat production}

After being in the room with air temperature 40°C and humidity 80% a patient has been brought to hospital in
grave condition. He was unconscious; he had tachypnea, tachycardia, and body temperature 41°C. Reanimation
was failed. The patient has died. What is the most possible direct reason of death in this case? {=Heart failure
~Collapse
~Coagulation of blood and decrease of volume of circulating blood
~Dehydration
~Paralysis of the breath center}

Fever of the patients with relapsing fever is characterized by several periods of pyretic temperature per day and
several periods of normal temperature. Such type of temperature curve is called:
{=Febris continua
~Febris intermittens ~Febris
remittent
~Febris recurrens
~Febris atypica}

Sharp increase of the temperature to 38.7 °C was marked in a patient with acute purulent periodontitis. His body
temperature has decreased to normal level after opening the pulp cavity. Which type of fever was in the patient?
{=Efemeral
~Septic
~Recurrent ~Remittent
~Continua}
A patient suffers from osteomyelitis of maxilla. His body temperature increases to 40°C and then sharply
decreases to 35.6°C every day. Which type of fever curve is characterized by these changes? {=Continua
~Intermittent
~Reccurens
~Atypica
~Hectica}
Body temperature of the 8-year-old Sasha with meningitis was on the level 39-40°C for a week. After 8 days
under the influence of antibiotics the temperature has decreased to 36.8 °C in 1.5 hours. There were acute
hyperemia of skin, profuse sweating, decrease of arterial pressure, and loss of consciousness in him. Which
medicine is the pathogenetic remedy?
{=Vasoconstrictors
~Antibiotics
~Antipyretics
~Pyrogenal
~Sulfanilamide}

Pallor of the skin, “goose flesh” and increase of oxygen consumption appeared in the patient’s skin after injection
of pyrogenal. Which stage of fever is characterized by these changes? {=The stage of falling temperature by crisis
~Stadium fastigii
~The stadium incrementi
~The stage of falling temperature by lysis}

A man in light clothes is staying in a room with air temperature +14°C. Windows and doors are closed. Which
way of heat loss is the most considerable in this case?
{=Perspiration
~Evaporetion
~Conduction
~Radiation
~Convection}

Experimental mice were kept in a lodge with air temperature 4°C. Which adaptive reaction supplies its thermal
homeostasis?
{=Decrease of oxidation enzyme activity
~Decrease of oxygen consumption
~Anabiosis
~Increase of blood consumption
~Limitation of heat loss}

Inclination of the set point of thermoregulation to higher level due to action of IL-1 is in a patient. What is the
name of this typical pathological process?
{=Fever
~Hyperthermia ~Hypothermia
~Inflammation
~Hypoxia}

The body temperature of a patient with crupous pneumonia is 39°C. The difference between the morning and
evening temperature of his body didn’t exceed 1 °C during 9 days. Which type of the fever curves was that?
{=Continua
~Hectica
~Intermittent
~Hyperpyretic
~Reccurens }

A patient has fever with following stages: incrementi, fastigii, decrementi. Which disease these features can
characterize? {=Acute pneumonia
~Acromegaly
~Diabetes mellitus
~Hyperaldosteronism
~Myocardial hypertrophy}
Acute increase of body temperature, dyspnea, tachycardia, nausea, convulsions, and loss of conciousness
developed in a worker, working in the thick uniform in summer. What was the most possible reason of
development of those symptoms?
{=Equilibration between heat loss and heat production
~Decrease of heat production
~Decrease of heat loss
~Increase of heat production
~Increase of heat loss}

Body temperature of patient is pyretic; his skin is hot and red. What was the correlation between processes of
heat production and heat loss in described stage of fever?
{=Heat loss is less heat production
~Heat loss is more then heat production ~Heat
production is more then heat loss
~Heat loss is equal then heat production
~Heat production is less then heat loss}

The body temperature of a patient with pneumonia was keeping on the level 38.3-38.5°C all the first week of
disease. Such fever is called:
{=Febrile
~Hyperpyretic
~Pyretic
~Subfebrile}

Increase of “acute phase” proteins level in blood such as ceruloplasmin, fibrinogen, C-reactive protein is typical
for development of fever. Indicate the possible mechanism of this phenomenon.
{=Degranulation of mast cells
~Destructive action of elevated temperature to the cells of the organism
~Proliferate action of IL-2 to T-lymphocytes
~Adaptive reaction of the organism to pyrogen
~Stimulative influence of IL-1 on hepatocytes}

Most infectious diseases are characterized by development of fever. It can be explained:


{=Formation of IL-1 during phagocytosis of microorganisms
~Intoxication of the organism
~Degranulation of mast cells
~Activation of T- and B-lymphocytes
~Processes of exudation}
Attacks of fever in a patient occur periodically. During the attack the body temperature sharply increases, keeps
pyrtetic level nearly 2 hours and then decreases to normal level. This type of fever is typical for: {=Brucellosis
~Sepsis
~Pneumonia
~Relapsing fever
~Epidemic typhus}

Body temperature of a patient is 39°C for several hours (stadium fastigii). Indicate which changes of physiological
functions are the most typical for this stage of fever.
{=Bradycardia
~Ingibition of phagocytosis
~Increase of heat production
~Increase of heat production
~Heat production is equal heat loss}

In a patient with pneumonia the increased body temperature was revealed. What sort of biological active
substances play the most important role in development of fever?
{=Interleukin-1
~Leukotriens
~Serotonin
~Histamine
~Bradykinin}

In a patient with prolonged fever after the course of treatment body temperature begins decreasing. What is the
possible mechanism of temperature decrease?
{=Protective activation of immune system
~Decrease of heat production due to reducing metabolic rate
~Decrease of production of pyrogens
~Increase of resistance of organism to action of the pyrogens ~Increase
of heat loss due to peripheral vasodilatation}

What is the most efficient mechanism of heat loss in case of following environmental conditions: 80% air
humidity and +35°C (95 degree Fahrenheit) air temperature?
{=Hyperventilation
~Sweating
~Conduction
~Convection
~Irradiation}

During the examination of the patient following symptoms were revealed: redness of skin, skin is hot and dry to
touch, heart bit rate is 92 per minute, respiratory rate is 22 per minute, body temperature is 39,2°C (102.5 degree
Fahrenheit). What is the correlation between heat production and heat loss in described period of fever? {=Heat
production surpasses heat loss
~Heat production equals to heat loss
~Heat production is less than heat loss
~Decreasing of heat production without changes of heat loss
~Increasing of heat production without changes of heat loss}

Animal was injected with pyrogen to reproduce fever. What mechanism starts process of temperature increasing?
{=Activation of non-shivering thermogenesis
~Activation of shivering thermogenesis
~Rise of set point of thermoregulation in hypothalamus
~Reduction of heat loss
~Dissociation of oxidation and oxidative phosphrilation in tissues}

Patient, who has been suffering from malaria, has weakness of heart activity and tachycardia during the stage of
heart loss. What is the name of this complication?
{=Infectious-toxic collapse
~Bacterial shock
~Hemorrhage collapse
~Ortostatic shock
~Hemorrhage shock}

Adaptation of organism disturbances to decrease of environmental temperature when using medicines


alphaadrenoblokators. Which mechanism is responsible for this?
{=Formation of primary heat
~Constriction of skin vessels
~Contractile thermogenesis
~Sweating}

In a patient with third stage of fever reaction following manifestations observed: abundant sweating, tachypnea
(increase in respiratory rate), decrease of body temperature. What is the mechanism of development of these
symptoms?
{=Reduction of shivering thermogenesis
~Secondary aldosteronism
~Rise of set point of thermoregulation in hypothalamus
~Predomination of heat production over the heat loss
~Peripheral vasodilatation}

After blood trasfusion patient complaints feeling of heat, rigor, increase of body temperature to +40°C. Its known
the cause of elevation temperature is secretion of endogenous pyrogens. Which cells produce endopyrogens?
{=Erythrocytes
~Platelets
~Endotheliocytes
~B-lymphocytes
~Macrophages}

The different types of temperature curve are reported on examination of the infected patients. What other
pathological conditions can lead to the fever?
{=Thyrotoxicosis
~Sistemic immune complex diseases
~Excess production of glucocorticoids
~Protein starvation
~Hypocoagulation}
In a patient which long time suffers of chronical enterocolitic after use meat food arise meteorism, diarrhea,
colics. With deficiency of what enzyme in intestine this associated? {=Peptidase
~Saccharase
~Maltase
~Amylase
~Glycogensynthetase}

In a patient a stone of common biliary duct closed entering of bile into intestine. What disorder digestion process
is observed in that case?
{=Digestion of fat
~Digestion of carbohydrates
~Absorbtion of carbohydrates
~Digestion of proteins
~Absorbtion of proteins}

A patient 42-years-old was admitted to clinic with stomach bleeding (ulcer disease is absent). Ulcer, hyperplasia
of stomach mucous is detected. What research should be make for confirmation of ZollingerEllison syndrome in
the patient?
{=Determination of blood gastrin
~Tomography of pancreas
~Determination of blood secretin
~Determination of pancreas enzymes activity
~Reseach of gastric secretion}

A patient complaints on bad appetites weight loss, pain in epigastric department. For analysis of gastric juice was
revealed achylif. What does term meaning?
{=Lack of free HCI and pepsin
~Lack of free HCl
~Lack of acidity
~Lack of free and binded HCI
~Lack of gastromucoprotein}

A patient long suffer with chronical enterocolitis. After use milk appeare meteorism, diarrhea, colic. It associated
with deficit of what enzyme in intestine?
{=Lactase
~Amylase
~Glycogensynthetase
~Saccharase
~Maltase}

A patient with acute high intestinal ileus was attmited to hospital with long vomiting and decreased arterial
pressure to 60/40 mm Hg. What mechanism of shock development in the patient is main?
{=Hypovolemia
~Exhaustion of arterias — adrenoreactivity
~Acidotic dilatation of metarterioles
~Chlorides loss
~Increase tone of nerve vagus}

After carried plural traumas and blood loss in a victim stomach dyskinesia developed on hypokinetic type. Motore
disorders of stomach in this case arose as a result of:
{=Weakening of general tone of organism
~Use of fatty food
~Appearances of the unpleasant taste feelings
~Permanent phobia for the passed
~Appetite loss}
A patient 52-year-old complains on pain in epigastrical depertment, vomiting. On investigation paleness of skin;
tachycardia, hypotension. Analysis of blood: erythrocytes — 2.8<10!7/1, hemoglobin — 70 g/l. Indicute the most
likely consequence of stomach ulcer, which arose in the patient? {=Bleeding
~Penetration
~Perforation
~Degeneration into cancer
~Pylorostenosis}

A patient 57 years-old with suspicion on Zollinger-Ellison syndrome was addmited to the gastroenterological
department about it testified of charp increase of gastrin level in blood serum. What disorder of the secretory
function of the stomach is most likely?
{=Hyperacidic hypersecretion
~Hyperacidic hyposecretion
~Hyporacidic hyposecretion
~Hyporacidic hypersecretion
~Achylia}

A patient 62-year-old was admitted in heavy state into gastroenterological department with complaints on bloody
vomit black tarlike feces, thirst, dryness in month, dizziness. In anamnesis — ulcer biseas Objectively:
decrease of AP and blood hemoglobin. What compliation of ulcer disease is the most likely in that case? {=Ulcer
bleeding
~Malignization
~Penetration into pancreas
~Perforation
~Stenosis of pylorus}

A woman 33-year-old suffers with hepato-cerebral dystrophy (Wilson’s disease). In the blood — decreased
ceruloplasmin contents. In urine — sharply increased aminoacid contents. Disorder of what process was caused
these changes? {=Copper metabolism
~Glyconeogenesis
~Deamination
~Synthesis of urea
~Desintegration of tissuel proteins}

After intensive antibioticotherapia in a patient the malabsorption syndrome developed for the following
pathogenic scheme: Neomycinum > atrophy of microvillus > .....?..... > disorder of membrane digestion >
disorder of absorption of monomer compounds
{=Deficit of membrane enzymes
~Deficit of pancreatic enzymes
~Congestion of meal in the intestine
~Activating of rotting processes
~Loss of appetite}
After removal of duodenum in a patient syndrom of duodenal insufficiency developted due to disorder its
endocrine function with phenomenon of cells deficiency of APUD-system. What from called hormones is
produced by ?-cells this part of intestine.
{=Glucagon
~Secretin
~Serotonin
~Insulin
~Histamine}

After resection of stomach in a patient appeared complaints to common weakness, heartburn in area of
lingua,dyspnea during physical loading, paresthesias in low extremitus. What is cause is showed disorders?
{=Deficit of vitamine B12
~Deficit of iron
~Achlorhydria
~Hypoproteinemia
~Dyselectrolytemia}

After taking a fatty food a patient feels nausea, flaccidity, later the sign of steathorrhea has appeared, cholesterol
in the blood 9,2ml/c. The cause of this state is the deficiency of: {=Bile acids
~Triglyceroides
~Chylomicrones
~Fatty acids
~Phopholipids}

A patient aged 45 had the diagnosis ulcer of the stomach. On examination of secretory function of the stomach it
was determined that the amount of basal secretion was 100mole/hr, acidity of basal secretion -60mml/hr. What
factors action contributes to the hypersecretion in the stomach? {=Gastrine
~Pancreatic polypeptide
~Somatostatin
~Glucagon
~Betaendorphin}

On laboratory examination increased amount of diastase in the urine and also a large amount of undigested fat in
stool were revealed in a patient female with complain of circular character pain in epigastric area. What form of
gastrointestinal tract pathology are described signs typical for? {=Acute pancreatitis ~Acute appendicitis.
~Infectious gastritis
~Ulcerous disease of the stomach
~Inflammation of large intestine}

A part of patient pancreas was resected. What kinds of product must be limited in his diet? {=Fatty
and fried meal
~Fruits
~Not fatty boiled meat
~Sour milk product
~Vegetables}

What enzyme deficiency is the cause of maldigestion of fats in the gastrointestinal tract and increase of neutral
fats in the stool? {=Pancreatic juice
~Intestinal lipase
~Hepatic lipase
~Enterokinase
~Gastric lipase}

Decrease of synthesis and secretion of trypsin is observed in chronic pancreatitis. The splitting of what substances
is broken?
{=Proteins
~Nucleic acids
~Polysaccharides ~Lipids.
~Liposoluble vitamins}
In coprologic investigation it is determined that stool is discoloured, there are drops of neutral fat in it. The most
possible cause of this is the impairment of?
{=Entering of bile into the intestine
~Process of absorption in the intestine
~Secretion of intestinal juice
~Acidity of gastric juice
~Secretion of pancreatic juice}

A patient had been taking antibiotics of a wide spectrum of action for a long period of time that caused decrease
of appetite, nausea, and diarrhea with saprogenic smell. What is the side effect of treatment? {=Dysbacteriosis
~Allergic reaction
~Hepatotoxic action
~Nephrotoxic action
~Direct irritative action}

On examination of a patient suffering from acute pancreatitis increased amount of chylomicrons was determined
in the blood. What enzyme activity is sharply decreased in this pathology? {=Lipoprotienelipase
~Pancreatic lipase
~Pancreatic phospholipase
~Tissue triglyceride lipase
~Tissue diglyceride lipase}

The analysis of gastric juice of an elderly man who complained of unmotivated weakness, sickness, absence of
appetite showed achylia, achlorhydrea, and presence of lactic acids and coagulated blood, decreased of pepsin
secretion. What disease causes such clinical-laboratory symptoms?
{=Cancer of the stomach
~Chronic gastritis
~Chronic pancreatitis
~Cavitary Maldigestion
~Acute gastritias}

A man with chronic hepatitis has dyspeptic disorders: decrease of appetite, nausea, unstable stool, and steatorrhea.
What is the mechanism of dyspeptic disorders in hepatic pathology? {=Hypocholea
~Hypoglycemia ~Cholalemia
~Intoxication
~Hyperbilirubemia}
Due to chronic gastritis a man has the impaired structure of the mucous membrane, decreased indices of acid
formation function of the stomach. The most essential negative result of this will be the impairment of: {=Protien
digestion
~Pancreatic juice secretion
~Secretory function of small intestine
~Evacuation of chyme into duodenum
~Excretion of secretum}

A patient aged 35 with ulcerous disease had a rejection of antral portion of the stomch. What gastrointestinal
hormone secretion will be impaired due to operation?
{=Gastrin
~Secritin
~Neurotensin
~Histamine
~Cholecystokinin}
Spasmodic pains in the abdomen and repeated diarrhea with mucus appeared in a healthy person 3-5 hours later
after taking meals. This was preceded by nausea and momentancous vomiting, general weakness, loss of appetite.
What is the most possible cause of the desired symptoms?
{=Food intoxication
~Chronic pancreatitis
~Enterocolitis
~Hyperacid state of the stomach
~Chronic gastritis}

A man who works at a storage battery plants complains of constant feeling of weight and periodical spasmodic
pains in the abdomen, constant retention of stool (not more often than one time per three day). This is
accompanied by frequent headaches, flaccidity, absence of appetite, and bad taste in the mouth. What are the
causes of these disorders?
{=Spastic lead colie with constipation
~Hypoacid state of the stomach
~Hyper acid state of the stomach
~Chronic pancreatitis
~Parietal maldigestion}

On laboratory examination increased amount of diastase in the urine and also a large amount of undigested fat in
stool were revealed in a patient with complains of circular character pains in epigastric area. What form of
gastrointestinal tract pathology are the described signs typical for?
{=Acute pancreatitis
~Acute appendicitis
~Infectious hepatitis
~Ulcerous disease of the stomch
~Inflammation of the large intestine}

A patient aged 57 was admitted to a gastrointestinal department with suspicious on Zolliger-Ellison syndrome,
which was proved by sharp increase of gastrin level in the blood serum. What impairment of secretory gastric
function is the most possible? {=Hyperacid hypersecretion
~Hyperacid hyposecretion
~Hypoacid hypersecretion
~Achylia
~Hypoacid hyposecretion}
A patient with signs of encephalopathy was hospitalised in neurological in- patients department and co relation
between increase of encephalopathy and substances passing from intestine into systemic blood circulation was
determined. What compounds formed in the intestine may cause endotoximia? {=Indole
~Biotin
~Acetoacetat
~Omnitin
~Buthirat}

150 ml of meat broth were introduced into the stomach cavity of experimental dog to a probe the content of what
substance will increase quickly in the animal’s blood? {=Gastrin
~Vasointestinal peptide
~Neurotensin
~Somatostatin
~Insulin}

A patient age 37 was admitted into a surgical department with the signs of acute pancreatitis; vomiting, diarrhea,
bradycardia, hypotension wearness, dehydration of the organism. What preparation must be used first of all?
{=Contrical
~No-spa
~Pephenazine
~Ephedrine
~Platy phy llin}

A patient has a stone in the common bile duct, which has stopped passing bile into the intestines. The impairment
of what digestive process is observed in this case?
{=Digestion of fats
~Absorption of carbohydrates
~Digestion of carbohydrates
~Absorption of proteins
~Digestion of proteins}

On fibroscopy of the stomach ulcer was revealed in antrum portion, which was associated with dissemination of
mucosa by Helicobacter pylori. The role of this agent in the formation of ulcer results in: {=Damage of mucous
barrier
~Inhibition of mucosa regeneration
~Impairment of microcirculation in mucous
~Stimulation of HCL secretion
~Stimulation of pepsin secretion}

A patient with ulcer of stomach has impairment of equilibrium between the factors of aggression and defense.
What factor contributes to the development of gastric ulcer?
{=Helicobacter pylori
~Mucin
~Bicarbonate
~Prostaglandin E2
~Prostacyclin}
After resection of duodenum a patient has developed a syndrome of duodenal in sufficiency due to the impairment
of its endocrine function with signs of cell insufficiency of APUD system. What hormone is produced by A-cells
of this portion of intestine?
{=Glucagon ~Insulin
~Seratonin
~Secrtin
~Histamine}

A patient aged 42 complains of pains in epigastric area, vomiting; vomitory masses have color of “coffee
grounds”, melena. In anamnesis there is ulcerous disease of the stomach. On examination: skin is pale, heart rate-
110 beats per minute: AP-90/50mm.hg. Blood analyses: erythrocytes-2.8 x 10!7/; leucocytes-8x107/1;
hemoglobin-90gr/l. What is the possible complication that developed in the patient? {=Hemorrhage
~Perforation
~Penetration
~Cancerous degeneration
~Pylorostenosis}

A patient aged 42 was admitted to the hospital with gastric hemorrhage (ulcerous disease was excluded by a
patient). Ulcer and hyperplasia of gastric mucose were revealed. What investigation is it necessary to carry out
to confirm the diagnosis of Zollinger-Ellison syndrome in this patient?
{=Definition of gastrin level in the blood
~Tomography of pancreas
~Definition of secretin level in the blood
~Definition of activity level of pancreatic enzymes
~Investigation of activity level of pancreatic secretion}

The amount of protein in the blood was examined in a patient after resection of sizeable part of small intestine.
What change of protein amount may be expected in this patient?
{=Hypoproteinemia
~Hyperproteinemia
~Paraproteinemia
~Hypergammaglobulinemia
~Dysproteinemia}

A patient complains of nausea, which often ends with vomiting. These dyspeptic phenomena become more
laborious with time. What stomach function is infringed in this patient? {=Evacuation
~Motility
~Reservoir
~Incretory
~Secretory}

Ulcerous disease of the stomach is revealed in a woman aged 52, who has been ill with contact dermatitis. On
examination it is determined that the patient had been taking corticosteroid preparations for a long period of time,
but acidity of gastric juice was increased. What mechanism caused the increase of gastric secretion? {=Decrease
of prostaglandin content
~Increase of secretin secretion
~Gastrin hyposecretion
~Increase of histamine content
~Increase of gastrin excretion}

Ulcers of stomach were found out in a patient, who was treated with glucocorticoids for a long time. What is the
main mechanism of ulcer development in this case? {=Increase in secretion and acidity of gastric juice
~Decrease in histamine concentration in stomach wall
~Increased tonus of sympathetic nervous system
~Increased production of prostaglandins E
~Decreased tonus of parasympathetic nervous system}

The investigation of pancreatic function was carried out in experimental animal by radioisotopic method. The
percentage of excreted with stool 1!3!- albumin is 53%. This is the evidence of:
{=Inhibition of exocrin function
~Increase of incretory function
~Inhibition of incretory function
~Increase of exocrine function
~Normal function}

Malabsorption syndrome is:


{=Syndrome of intestinal malabsorption
~Syndrome of gastric malabsorption
~Syndrome of malabsorption in small intestine
~Syndrome of membraous maldigestion
~Malabsorption of proteins}
A patient complains of vomiting, eructation, pain in epigastric area, constipation, and emaciation. On examination
a basal secretion of HCL is 6mml/gr; maximal one is 6Omml/gr. What state has acid-forming function of this
patient?
{=Hyperacid
~Anacid
~Hypoacid
~Hypersecretory
~Normacid}

A 67-year-old woman, who has cholecystitis for a long time, suddenly developed sharp pain in the upper part of
abdomen, nausea, and vomiting after food intake. Acute pancreatitis was diagnosed in this patient. What is the
main link in pathogenesis of this disease? {=Preliminary activation of pancreatic enzymes
~Decrease in enzyme levels in pancreatic juice
~Intensification of enzyme activation in duodenum
~Reduction of pancreatic polypeptide secretion
~Increased level of cholecystokinin}

A patient has increased gastric secretion in response to both mechanical and chemical stimuli. He has high acidity
of gastric juice with pH equals 2.0 on an empty stomach. After the breakfast pH restores to normal in 12 minutes
and is continuously dropping during following 2 hours. What type of stomach secretion is being observed in this
patient?
{=Excitable
~Brakable
~Inert
~Asthenic
~Normal}
Frequent liquid stool, signs of intoxication, and dehydration developed in a newborn at 5-6 day. After this
newborn was fed by acid-milk mixtures indicated symptoms disappeared. The conclusion of innate lactose
deficiency was made. What process was broken in this newborn?
{=Membrane digestion
~Cavital digestion
~Excretory function of intestine ~Secretory
function of pancreas
~Secretion of gastric juice}

A patient has normally coloured stool including a large amount of free fatty acids. The reason for this is a
disturbance of the following process:
{=Fat absorption
~Fat hydrolysis
~Biliary excretion ~Choleresis
~Lipase secretion}

A 52 year old man has abdominal pain, hematemesis, and melena. He is not taking NSAIDs (nonsteroidal
antiinflammatory drugs) or aspirin. An upper endoscopy reveals a large posterior wall duodenal ulcer. A biopsy
is obtained from the gastric antrum and a rapid urease test is positive. Which of the following findings is most
likely?
{=Type A autoimmune gastritis
~CagA positive H pylori
~Low gastrin levels after a meal
~Normal gastric acid production
~Absence of gastric metaplasia in the duodenum}

A patient takes cholagogues. What other process besides biliary excretion will be stimulated? {=Intestinal
motility
~Gastric juice secretion
~Pancreatic juice secretion
~Gastric motor activity
~Water absorption}

An 70year old woman has a history of constipation and intermittent left lower quadrant abdominal pain. A
colonoscopy shows extensive diverticulosis in the sigmoid colon. He is at risk for which of the following
conditions?
{=Carcinoma of the colon
~Angiodysplasia
~Gastrointestinal hemorrhage
~Ulcerative colitis
~Malabsorption}

A 42-year-old patient complains of pain in the epigastral area, vomiting; vomit masses have the color of
coffeegrounds; the patient suffers from melena. Anamnesis records gastric ulcer disease. Blood formula:
erythrocytes — 2.8x10'7/1, leukocytes - 8x10°/1, Hb - 90 g/l. What complication is it? {=Perforation
~Penetration
~Hemorrhage
~Canceration
~Pyloric stenosis}

A 35-year-old man with peptic ulcer disease has undergone antrectomy. After the surgery secretion of the
following gastrointestinal hormone will be disrupted the most:
{=Gastrin
~Secretin
~Neurotensin
~Cholecystokinin
~Histamine}

A patient has a critical impairment of protein, fat and hydrocarbon digestion. Most likely it has been caused by
low secretion of the following digestive juice:
{=Pancreatic juice
~Saliva
~Gastric juice
~Bile
~Intestinal juice}
An animal in the terms of starvation demonstrates stupor, decrease of body weight by 20%. Body temperature is
36.2°C, heart rate is 68/min, breathing coefficient 0.7. What is the period of starvation? {=Maximal adaptation
~Inexpedient energy expenditure
~Tissue destruction
~Excitation
~Dying}

After a severe burn a man developed stenosis of the esophagus. Due to impairment of eating a rapid lost of body
weight occurred. Blood analysis: erythrocytes: 3,0x1012/1, Hb-106 g/l, serum protein 57 g/l. What kind of
starvation is present in the patient?
{=Incomplete
~Protein
~Complete
~Water
~Absolute}

In what period of complete starving is found a woman if she has the state of excitation with the feeling of
starvation, blood analysis shows hypoglycemia, negative nitrogen balance, breathing coefficient is 1.0? {=Period
of unconstructive energy expenditure
~Maximal adaptation
~Excitation
~Period of constructive energy expenditure
~Terminal period}

The main matters exchange in a patient is decreased by 15%, negative nitrogen balance is observed in the patient.
Blood analysis demonstrates hypoglycaemia, hyperlipiacidaemia, ketonaemia, acidosis. Breathing coefficient is
0.7. What is the period of starvation?
{=Maximal adaptation
~Excitation
~Period of unconstructive energy expenditure
~Terminal period
~First}

A group of animals has been kept without food for 10 hours. The level if blood sugar at the end of starvation in
all the animals was found on the lowest level of the norm. How can maintenance of sugar concentration on the
lowest level of the norm be explained? {=Activation of glycogenolysis processes
~Activation of glycogenesis processes
~Activation of glyconeogenesis processes
~Depression of glycogenesis processes
~Depression of glyconeogenesis processes}

In a rat after 48 hours of starvation with only water consumption main matters exchange dropped by 20%,
respiratory coefficient is 0.7, lipaemia, and negative nitrogen balance were present. What period of complete
starvation are these changes typical of?
{=Maximum adapration
~Excitation
~Wasteful spending
~Terminal
~First}

In a rat after 72 hours of starvation with only water consumption main matters exchange dropped by 40%,
respiratory coefficient is 0.8, there are ketoneaemia, acidosis, edema. What period of complete starvation are
these changes typical of?
{=Terminal
~Wasteful energy spending
~Maximal adaptation
~First
~Second}

On the 10th hour of fasting with only water consumption a rat developed raised basal metabolism; breathing
coefficient is 1.0; in the blood: increased level of glucocorticoids, catecholamines. Increased urinary nitrogen.
What period of complete starvation are these changes typical of?
{=The period of wasteful energy consumption
~Maximal adaptation
~Period of stimulation
~Period saving energy consumption
~Terminal period}

After chemical burns a patient developed esophageal stenosis. Rapid weight loss occurred as a result of eating
difficulties. Blood tests showed red blood cells count — 3.0*1012/1, hemoglobin - 106 g/l, protein 57 g/l. What
type of starvation has the patient?
{=Quantitative
~Qualitative
~Water
~Complete
~Absolute}

In a starving animal a general depression, lowering basal metabolism by 18%, reduction of body weight by 20%,
hiperlipidaemia were observed. Body temperature 36.2 °C, heart rate 68/min., breathing coefficient 0.7. What
period of complete starvation are these changes typical of?
{=Maximal adaptation
~Period of protein catabolism
~The period of wasteful energy consumption
~Terminal period
~Excitation}

In a starving animal a general excitation, increasing basal metabolism by 25%, breathing coefficient 1.0, heart
rate 78/min were observed. What period of complete starvation are these changes typical of? {=The period of
wasteful energy consumption
~Maximal adaptation
~Period of protein catabolism
~Terminal period
~Excitation}
A pregnant woman developed iron deficient anemia. What type of starving has triggered this disorder?
{=Qualitative ~Quantitative
~Water
~Complete
~Absolute}

A man has an esophageal tumor. Eating is impossible. What is the difference between complete starvation from the
incomplete one?
{=Duration
~Increased basal metabolism
~Nongas alkalosis
~Carbon dioxide acidosis
~Gas alkalosis}

In a starving animal excitation, aggression, and breathing coefficient 1.0 are observed. Basal metabolism is
increased. What is pathophysiological period of starvation?
{=The period of wasteful energy consumption
~Maximal adaptation
~Period of protein catabolism
~Terminal period
~Excitation}

On the 9th day of starvation an experimental animal developed malaise, and apathy; it is lying in a whorled
position. Basic matters exchange is reduced. What is the breathing coefficient in the animal? {=1.0
~2.0
~0.8
~0.7
~0.1}

A man has an esophageal tumor. Eating is impossible. For how long can a man live in the conditions of complete
starvation?
{=65-70 days
~25-30
~12
~15-20
~120-130}

On the 9th day of starvation an experimental animal developed malaise, and apathy; it is lying in a whorled
position. Basic matters exchange is reduced. Respiratory coefficient is 0.7. What is the main energy substrate for
the organism? {=Fats
~Carbohydrates
~Proteins
~Vitamins
~Minerals}

A man who has long worked in the hot shop developed hypovitaminosis. What kind of hypovitaminosis evolved
inthe man? {=Vitamin PP
~Vitamin K
~Vitamin A
~Vitamin E
~Vitamin D}

A starving animal developed drowsiness, indifference. What adaptive phenomena are observed during starvation?
{=Reduction of basal metabolism
~Increase of the basal metabolism
~Increase of temperature production
~Increase in gastric secretion
~Increased urine excretion}

An animal was withdrawn from eating but drinking was not restricted. What are the organs that lose weight when
fasting the least?
{=Liver
~Pancreas
~Lung
~Kidneys
~Heart}

A patient developed kwashiorkor. Which type of starvation does it belongs to? {=Protein
and energy
~Carbohydrate
~Fatty
~Energy
~Vitamin}

A man has an esophageal tumor. Eating is impossible. Which conditions is the increase of complete starvation
duration observed at?
{=At lowering basal metabolism
~When the organism is exposed to high temperatures
~In thyroid hyperfunctions
~At increasing basal metabolism
~In increasing the specific body surface}

In a starving animal signs of excitation, and aggression were observed; breathing coefficient is 1.0. Basal
metabolism is increased. What is the main energy substrate of the organism in these conditions? {=Carbohydrates
~Fats
~Proteins
~Vitamins
~Minerals}

A patient has B12-deficient anemia. What kind of hunger has triggered the disorder?
{=Qualitative ~Quantitative
~Partial
~Full
~Absolute}

In a patient with atrophic gastritis iron is absorbed poorly. What are manifestations of this state?
{=Hypochromic anemia
~Impairment of bone formation
~Tetany ~Goiter
~Caries}
On the second day of starvation a starving animal manifested signs of excitation and aggression. Basal metabolism
became increased. What is the breathing coefficient? {=1.0
~2.0
~0.8
~0.7
~0.1}

On the 9th day of starvation a test animal developed fatigue, apathy and was lying in a whorled position. Basal
metabolism was reduced. Respiratory coefficient is 0.7. What is pathophysiological period of starvation?
{=Maximal adaptation
~Proteins decay
~The period of wasteful energy consumption
~Terminal
~Excitation}

A patients was diagnosed with endemic goiter. Insufficient income of which microelement is responsible for
development of this disease?
{=Iodine
~Potassium
~Fluoride
~Cobalt
~Iron}

In a child abnormalities of bone forming and signs of development of caries were found. Insufficient income of
which microclement is responsible for development of this disease? {=Fluorine
~Potassium
~lodine
~Cobalt
~Iron}

In a patient slowing down of normoblasts maturation and the release of mature red blood cells into the blood
circulation were revealed. Insufficient income of which microelement is responsible for development of this
disease? {=Cobalt
~Potassium
~Fluoride
~Yoda
~Iron}

Experimental animals were fed with a special feed. After some time the animals lost appetite, developed
impairment of the liver, gastrointestinal tract, and kidneys functioning; blood osmotic pressure, and arterial blood
pressure in the animals decreased. Insufficient income of which substance is responsible for development of this
disorders?
{=Sodium chloride
~Potassium
~Fluoride
~Calcium
~Iron}

An experimental animal developed tetany. Insufficient income of which microelement is responsible for
development of this disease?
{=Potassium
~Calcium
~Fluoride
~Cobalt
~Iron}

An experimental animal died on the seventh day of absolute starvation. What causes the most severe changes in
the organism?
{=Deprivation of water
~Deprivation of minerals
~Deprivation of proteins
~Deprivation of fats
~Deprivation of carbohydrates}

After multiple administration of avidin to animals they developed hypovitaminosis. Antagonist of what vitamin is
avidin?
{=Vitamin H
~Vitamin B1
~Vitamin B12
~Vitamin C
~Vitamin D}

In a patient a malignant ling tumor was diagnosed. What type of starvation will the patient develop?
{=Qualitative ~Quantitative
~Partial
~Complete
~Absolute}

In an experimental animal growth retardation, reduction of body weight, muscle weakness and impairment of the
nervous system are observed. The lack of which vitamin may have caused these disorders? {=Vitamin PP
~Vitamin E
~Vitamin A ~Vitamin
Cc
~Vitamin D}

In an experimental animal growth retardation, reduction of body weight, muscle weakness and impairment of the
nervous system are observed. The lack of which vitamin may have caused these disorders? {=Vitamin B1
~Vitamin E
~Vitamin A
~Vitamin C
~Vitamin D}

In an experimental animal growth retardation, reduction of body weight, muscle weakness and impairment of the
nervous system are observed. The lack of which vitamin may have caused these disorders? {=Vitamin B12
~Vitamin E
~Vitamin A
~Vitamin C
~Vitamin D}

In an experimental animal growth retardation, reduction of body weight, muscle weakness and impairment of the
nervous system are observed. The lack of which vitamin may have caused these disorders? {=Vitamin B6
~Vitamin E
~Vitamin A
~Vitamin C
~Vitamin D}

In an experimental animal growth retardation, reduction of body weight, muscle weakness and impairment of the
endocrine system are observed. The lack of which vitamin may have caused these disorders? {=Vitamin E
~Vitamin H
~Vitamin A
~Vitamin C
~Vitamin D}

In an experimental animal growth retardation, reduction of body weight, muscle weakness and impairment of the
endocrine system are observed. The lack of which vitamin may have caused these disorders? {=Vitamin B1
~Vitamin H
~Vitamin A
~Vitamin C
~Vitamin D}

In an experimental animal growth retardation, reduction of body weight, muscle weakness and impairment of the
endocrine system are observed. The lack of which vitamin may have caused these disorders? {=Vitamin B5
~Vitamin H
~Vitamin A
~Vitamin C
~Vitamin D}

In an experimental animal growth retardation, reduction of body weight, muscle weakness and impairment of the
GIT system are observed. The lack of which vitamin may have caused these disorders?
{=Vitamin PP
~Vitamin H
~Vitamin A
~Vitamin C
~Vitamin D}

In an experimental animal growth retardation, reduction of body weight, muscle weakness and impairment of the
endocrine system are observed. The lack of which vitamin may have caused these disorders? {=Vitamin B1
~Vitamin H
~Vitamin A
~Vitamin C
~Vitamin D}

A man who has long worked in the North developed hypovitaminosis. What kind of hypovitaminosis evolved in
the man?
{=Lack of retinol
~Lack of ascorbic acid
~Lack of nicotinic acid
~Biotin deficiency
~Lack of tocopherol}

A man who has long worked in the North developed hypovitaminosis. What kind of hypovitaminosis evolved in
the man?
{=Lack of calciferol
~Lack of ascorbic acid
~Lack of nicotinic acid
~Biotin deficiency
~Lack of tocopherol}

A man who has long worked in the North developed hypovitaminosis. What kind of hypovitaminosis evolved in
the man?
{=Lack of B-group vitamins
~Lack of ascorbic acid
~Lack of nicotinic acid
~Biotin deficiency
~Lack of tocopherol}

A man who has long worked in the hot shop developed hypovitaminosis. What kind of hypovitaminosis evolved
in the man? {=Vitamin B1 ~Vitamin K
~Vitamin A
~Vitamin E
~Vitamin D}

A man who has long worked in the hot shop developed hypovitaminosis. What kind of hypovitaminosis evolved
in the man? {=Vitamin B12
~Vitamin K
~Vitamin A
~Vitamin E
~Vitamin D}
A man who has long worked in the hot shop developed hypovitaminosis. What kind of hypovitaminosis evolved
inthe man? {=Vitamin C
~Vitamin K
~Vitamin A
~Vitamin E
~Vitamin D}

A 10-year-old girl has a history of repeated acute respiratory viral infection. After recovering she presents with
multiple petechial hemorrhages on the sites of friction from clothing rubbing the skin. What kind of
hypovitaminosis has this girl?
{=C
~B6
~Bl
~A
~B2}

A 2-year-old child with mental and physical retardation has been delivered to a hospital. He presents with frequent
vomiting after having meals. There is phenylpyruvic acid in urine. Which metabolism abnormality is the reason
for this pathology?
{=Amino-acid metabolism
~Lipidic metabolism
~Carbohydrate metabolism
~Water-salt metabolism
~Phosphoric calcium metabolism}

A doctor recommends a patient with duodenal ulcer to drink cabbage and potato juice after the therapy course.
Which substances contained in these vegetables help to heal and prevent the ulcers?
{=Vitamin U
~Pantothenic acid
~Vitamin C
~Vitamin B1
~Vitamin K}

A patient has an increased pyruvate concentration in blood, most of it is excreted with the urine. What kind of
avitaminosis has this patient?
{=Bl
~E
~B3
~B6
~B2}

Blood test of a patient suffering from atrophic gastritis gave the following results: RBCs - 2,0x10!7/, Hb - 87 g/l,
colour index - 1,3, WBCs - 4,0x10°/, thrombocytes - 180x10°/1. Anaemia migh have been caused by the
following substance deficiency:
{=Vitamin B12
~Vitamin A
~Vitamin K
~Iron
~Zinc}

A patient diagnosed with focal tuberculosis of the upper lobe of the right lung had been taking isoniazid as a part
of combination therapy. After some time, the patient reported of muscle weakness, decreased skin sensitivity,
blurred vision, impaired motor coordination. Which vitamin preparation should be used to address these
phenomena
{=Vitamin B6
~Vitamin A
~Vitamin
~Vitamin B12
~Vitamin C}

To lose some weight a woman has been limiting the amount of products in her diet. 3 months later she developed
edemas and her diuresis increased. What dietary component deficiency is the cause of this? {=Minerals
~Proteins
~Carbohydrates
~Fats
~Vitamins}

During regular check-up a child is determined to have interrupted mineralization of the bones. What vitamin
deficiency can be the cause?
{=Calciferol
~Tocopherol
~Cobalamin
~Folic acid
~Riboflavin}
Anemia at the diseases of kidneys is conditioned
{=Decrease of erythropoietin production
~Renal azotemia
~Decrease of glomerular filtration
~Increase of tubular resorption
~Decrease of prostaglandins synthesis}

Appearance of what pathological component in urine testifies to the increase permeability of glomerular membrane?
{=High molecular albumens
~Glucose
~Amino acid
~Fresh erythrocytes
~Purulent discharge}

In a patient with kidney insufficiency is decrease of clearance on inulin to 60 ml/min. What function of kidneys is
disordered?
{=Glomerular filtration
~Tubular secretion
~Resorption in the proximal part of nephron
~Resorption in the distal part of nephron
~Resorption in collective kidney tubules}

Index which characterizes the decrease of glomerular filtration, is


{=Azotemia
~Alkalosis
~Aminoaciduria
~Glucosuria
~Polyuria}

Resorption of sodium ions in kidney tubule occur in the use of salt water. What compensatory changes of hormones
secretion will arise in that? {=Decrease of aldosterone excretion
~Decrease of vasopressin excretion
~Increase of aldosterone excretion
~Decrease of natriuretic hormone excretion
~Increase of vasopressin excretion}

The changes in kidneys at the glomerulonephritis as a result of damage basic membrane {=Antibodies
and immune complexes
~Bacterium
~Bacterial toxins
~Biological active substance
~Products of nitrogen metabolism}

There is disorder of excretory kidneys function - oligo-anuria in the II stage of acute kidney insufficiency. Specify
the main characterized index of this phenomenon?
{=Azotemia
~Decrease of hematocrit
~Decrease of arterial pressure
~Hypokalemia
~Hyponatremia}
There is proteinuria as a result of kidney illness. What the most likely mechanism of this phenomenon? {=Damage
of glomerular membranes
~Increase of renin secretion
~Increase of muscles tone
~Increase of protein synthesis in a liver
~Decrease of vasopressin secretion}

What from pathogenetic factors is main in the mechanism of edema development at a nephrotic syndrome?
{=Decrease of oncotic pressures blood plasma
~Increase of capillar permeability
~Increase of hydrostatic pressure in capillaries
~Increase of osmotic pressure in tissues
~Increase of oncotic pressures in tissues}

What origin albumen, the most likely at selective proteinuria of intensity 11 g/day? {=Tubular
~Suprarenal
~Glomerular
~Urethral
~From an urinary bladder}

What quantitative changes of diuresis result in uremia:


{=Anuria
~Dysuria
~Nycturia
~Isosthenuria
~Polyuria}

What type disorder of general blood volume will develop in a patient at the decrease of kidney excretory function?
{=Oligocythemic hypervolemia
~Polycythemic hypovolemia
~Oligocythemic hypovolemia
~Polycythemic hypervolemia
~Simple hypervolemia}

A patient with renal pathology developed massive proteinuria, edema, hypoproteinaemia, retention hyperlipidaemia.
What is the designation of this pathological process?
{=Nephrotic syndrome
~Hypertensive syndrome
~Urinary syndrome
~Anaemic syndrome
~Loss syndrome}

In a patient with chronic renal pathology Zymnytsky’s test found the loss of kidneys osmotic concentration function.
What changes in urination will occur at this pathology?
{=Urorrhagia
~Oliguria
~Anuria
~Polakiuriya
~Hematuria}

In a patient due to nephron tubular pathology Zymnytsky’s test revealed the loss of kidneys osmotic concentration
function. What is the designation of this pathological changes?
{=Isohypostenuria
~Proteinuria
~Haematuria
~Cylinderuria
~Leukocyturia}

As result of poisoning with sublimatum a patient has developed damage to kidneys with the appearance of glycosuria
and proteinuria. Which part of the nephron has sustained the largest injury with this pathological process?
{=Proximal tubule
~Renal glomerulus
~Distal tubule
~Loop of Henle
~Collecting tubules}

As result of poisoning with sublimatum a patient has sustained destruction of nephrons with the loss of substances
with large molecular mass with urine. What is the designation of this pathological changes? {=Tubular proteinuria
~Tubular acidaminuria
~Tubular glycosuria
~Tubular bicarbonateuria
~Tubular phosphaturia}

In a patient with multiple myeloma a protein was determined in urine. Which form of proteinuria has place in this
patient?
{=Suprarenal
~Renal glomerular
~Renal tubular
~Subrenal bullous
~Subrenal uretral}

A 35-year-old female complains of pain in her lower back. Laboratory analysis found in urine: protein - 5 g/,
erythrocytes - 2-3 in the sight field, leukocytes - 20-30 in the sight field. Bacterial culturing found Escherichia coli.
What disease are these changes characteristic of?
{=Acute pyelonephritis
~Acute diffuse glomerulonephritis
~Chronic glomerulonephritis
~Nephrotic syndrome
~Tuberculosis of kidney}
A 58-year-old patient was admitted to hospital with acute heart failure. He was found to have reduced daily urination
volume (oliguria). What factors might have caused this phenomenon?
{=Decreased glomerular filtration rate
~Decreased number of functioning glomeruli
~Decreased oncotic blood pressure
~Increased hydrostatic pressure on the capillary walls
~Decreased permeability of glomerular membranes}

A 30-year-old male, who was admitted to the clinic with the diagnosis of acute hlomerulonephritis, developed
proteinuria. What disorder has caused this pathology?
{=Increased permeability of glomerular membranes
~Delayed excretion of the nitrogen methabolism end products
~Decreased oncotic blood pressure
~Increased hydrostatic pressure on the capillary walls
~Reduced number of functioning nephrons}

2 weeks after the sore throat a 5-year-old child developed acute diffuse glomerulonephritis, which was characterized
by oliguria, proteinuria, haematuria, hyperazotaemia, arterial hypertension, and edema. Abnormality of which renal
function is the most significant in the development of these disorders? {=Glomerular filtration
~Tubular reabsorption
~Tubular secretion
~Urine excretion
~Incretory function}

A patient complains of constant thirst, headache, weakness, massive urination (up to 10 I/day). The patient admitted
that he sustained head injury. Specific weight of urine is 1008; pathological elements are absent. What is the most
feasible mechanism for the development of water-clectrolyte exchange disorder in this case? {=Insufficient
antidiuretic hormone production
~Overproduction of antidiuretic hormone
~Insufficient insulin synthesis
~Overproduction of aldosterone
~Inadequate aldosterone synthesis}

After massive blood loss a patient is marked for oliguria, hyperazotaemia, brain and pulmonary oedema. What
mechanism is the most important in reducing diuresis in this case?
{=Decreased efficiency of filtration pressure
~Increased water reabsorption in canals
~Increased tubular sodium reabsorption
~Redistribution of water in the organism
~Reduced glomerular membrane permeability}

As aresult of failure of safety rules observance a mechanicianin has sustained poisoning with sublimatum (mercury
chloride). After 2 days the patient’s daily diuresis became 620 ml. Headache, vomiting, cramps, shortness of breath,
and moist crepitation in the lungs developed in the patient. What is the designation of such a renal dysfunction?
{=Acute renal failure
~Chronic renal failure
~Uremic coma
~Glomerulonephritis
~Pyelonephritis}

A 38-year-old patient at the 3" year of the disease with systemic lupus erythematosus developed diffuse lesions of
the kidneys accompanied by massive swellings, pronounced proteinuria, hyperlipidaemia, dysprotetnaemia. What is
the most likely mechanism of proteinuria in this clinical situation?
{=Autoimmune damage to nephrons
~Inflammatory damage to nephrons
~Ischemic tubular injury
~Increased level of protein in the blood
~Urinary tract injury}
A 27 year-old man was admitted to the hospital in a critical condition with profuse gastric bleeding. Arterial pressure
is 80/60 mmHg, daily diuresis of the patient constitutes 60 -80 ml with specific weight 1028-1036. Increased levels
of urine nitrogen, urea, and creatine are found in the blood serum. What is the most likely pathogenetic mechanism
that led to the reduction of daily diuresis in this clinical situation?
{=Decrease of hydrostatic pressure in the glomeruli capillaries
~Increase of osmotic pressure of urine
~The high level of residual urine nitrogen in the blood ~Increase
of colloid-osmotic pressure in blood
~Increased hydrostatic pressure in Bowman’s capsule}

Alkiline erythrocytes have been identified in the patient’s daily urine. Which renal pathology is the found symptom
characteristic to?
{=Diffuse glomerulonephritis
~Nephrotic syndrome
~Renal stone disease
~Pyelonephritis
~Acute renal failure}

Different abnormal substances appear in the urine with a renal pathology. Emergence of which pathological
substances in urine confirms the increased glomerular membrane permeability? {=Proteins
~Glucose
~Amine acids
~Homogentisic acid
~Piurtya}

In a patient with acute renal failure (stage of urorrhagia) not only has azotaemia diminished, but continued to grow.
Abnormality of which renal function is it linked to?
{=Secretion
~Filtering
~Reabsorption
~Urine concentration
~Urine dilution}
During urine investigation in a patient with chronic pyelonephritis the decrease of urine specific weight to 1010
associated with increased diuresis were found. What renal function is impaired?
{=Sodium and water reabsorption
~Glomerular filtration
~Tubular secretion
~Glucose reabsorption
~Protein reabsorption}

In a patient with acute renal failure at the 6" day of therapeutic activities urorrhagia appeared (daily diuresis
constituted 2.7 litres). Increase of diuresis at early stages of urorrhagic stage of acute renal failure is caused by:
{=Restoration of filtration in nephrons
~Increased volume of circulating blood
~Increased mass of existing nephrons
~Decrease of plasma aldosterone
~Decrease of plasma vasopressin}

On the background of diabetes mellitus a 30-year-old woman developed swelling of legs and face during a month.
Examination revealed proteinuria (5.87 g/l), hypoproteinaemia, dysproteinaemia, hyperlipidaemia. The
combination of these features is characteristic of:
{=Nephrotic syndrome
~Nephritic syndrome
~Chronic pyelonephritis
~Acute renal failure
~Chronic renal failure}

The levels of total blood protein in a patient with primary nephrotic syndrome are 40 g/l. What reason caused
hy poproteinaemia?
{=Proteinuria
~Extravasations of protein into tissues
~Reduction of protein synthesis in the liver
~Increased proteolysis
~Abnormal protein absorption in the gut}

In a patient with chronic glomerulonephritis glomerular filtration rate (GFR) has decreased to 20% of the normal
rate. What causes the lowering of GFR with chronic renal failure?
{=Reduced number of functioning nephrons
~Tubulopathy
~Obturation of the urinary tract
~Renal ischemia
~Thrombosis of renal artery}

A 55-year-old patient has been suffering from chronic glomerulonephritis for 15 years. What changes in the blood
or urine suggest most of the limitation of the kidneys filtration function?
{=Hyperazotaemia
~Hyperglycaemia
~Hypoproteineamia
~Proteinuria
~Hypo -, isostenuria}

A patient has been diagnosed with renal diabetes mellitus (renal glycosuria). When setting the diagnosis a doctor
hung upon:
{=Decreased glucose reabsorption in the kidneys
~Reduced secretion of PTH
~Reduced insulin secretion
~Reduced insulin sensitivity
~Hyperglycaemia}

In a 18-year-old man laboratory examination revealed glucose in urine at its normal concentrations in plasma. The
most likely reason for this condition is the disorder of:
{=Glomerular filtration
~Tubular secretion
~Insulin secretion
~Tubular reabsorbtsii
~Secretion of glucocorticoids}

A patient developed diabetes insipidus after acute stroke with damage to hypothalamus nuclei. What is the cause of
increased urination in this patient?
{=Reduced water reabsorption
~Reduction of blood pressure
~Acceleration of glomerular filtration rate
~Hyperglycaemia
~Reduced sodium reabsorption}

A 50-year-old man complains of thirst and marked urorrhagia. Levels of blood glucose 4.8 mmol/l; ketone bodies
and glucose in urine are absent; urine is colourless, specific weight - 1.002. What is the reason of urorrhagia in the
patient?
{=Lack of ADH
~Insulin deficiency
~Thyrotoxicosis
~Hyperaldosteronism
~Hypothyroidism}

Diabetic nephropathy with uremia has developed in a patient with pancreatic diabetes. The velocity of glomerular
filtration is 9 ml/min. What mechanism of a decrease in glomerular filtration velocity and chronic renal failure
development is most likely in the case of this patient?
{=Reduction of active nephron mass
~Decrease in systemic arterial pressure
~Obstruction of nephron tubules with hyaline casts
~Tissue acidosis
~Arteriolar spasm}

As aresult of continuous starvation the glomerular filtration rate has increased by 20%. The most probable cause of
the glomerular filtration alteration under the mentioned conditions is:
{=Decrease in the oncotic pressure of blood plasma
~Increase in the systemic arterial pressure
~Increase in the permeability of the renal filter
~Increase of the filtration quotient
~Increase of the renal blood flow}

A patient with massive burns developed acute renal insufficiency characterized by a significant and rapid deceleration
of glomerular filtration. What is the mechanism of its development?
{=Reduction of renal blood flow
~Damage of glomerular filter
~Reduction of functioning nephron number
~Rise of pressure of tubular fluid
~Renal artery embolism}

A 38-year-old male patient has been ill with systemic lupus erythematosus for three years. He was diagnosed with
diffuse renal affection accompanied by massive edema and expressive proteinuria. What is the most likely cause of
proteinuria development? {=Autoimmune renal affection
~Aseptic renal affection
~Ischemic renal affection
~Urinary bladder inflammation
~Urinary tracts inflammation}

The cause of anuria in the course of suprarenal acute renal failure is:
{=Obturation of the urinary tract
~Tubular epithelial necrosis
~Glomerulonephritis
~Poisoning by heavy metals
~Hypoproteinemia}

Anaemia often develops in the course of renal failure. Its reason is:
{=Reduction of erythropoietin synthesis
~Haematuria
~Haemoglobinuria
~Hypoproteinaemia
~Albuminuria}

A child has an acute renal failure. What biochemical factor found in saliva can confirm this diagnosis?
{=Increase in urea concentration
~Increase in glucose concentration
~Decrease in glucose concentration
~Increase in concentration of higher fatty acids
~Decrease in nucleic acid concentration}
A patient with a history of chronic glomerulonephritis presents with azotemia, oliguria, hypo- and isosthenuria,
proteinuria. What is the leading factor in the pathogenesis of these symptoms development under chronic renal
failure?
{=Mass decrease of active nephrons
~Intensification of glomerular filtration
~Tubular hyposecretion
~Disturbed permeability of glomerular membranes
~Intensification of sodium reabsorption}

Due to the use of poor-quality measles vaccine for preventive vaccination, a l-year-old child developed an
autoimmune renal injury. The urine was found to contain macromolecular proteins. What process of urine formation
was disturbed?
{=Filtration
~Reabsorption
~Secretion
~Reabsorption and secretion
~Secretion and filtration}

A man presents with glomerular filtration rate of 180 ml/min., while norm is 125+25 ml/min. The likely cause of it
is the decreased:
{=Effective filtration pressure
~Renal blood flow
~Permeability of the renal filter
~Plasma oncotic pressure
~Hydrostatic blood pressure in the glomerular capillaries}

Urine analysis has shown high levels of protein and erythrocytes in urine. This can be caused by the following:
{=Renal filter permeability
~Hydrostatic blood pressure in glomerular capillaries
~Oncotic pressure of blood plasma
~Hydrostatic primary urine pressure in capsule
~Effective filter pressure}
Due to the use of poor-quality measles vaccine for preventive vaccination, a 1- year-old child developed an
autoimmune renal injury. The urine was found to contain macromolecular proteins. What process of urine formation
was disturbed?
{=Filtration
~Reabsorption
~Secretion
~Reabsorption and secretion
~Secretion and filtration}

Poisoning caused by mercury (II) chloride (corrosive sublimate) occurred in the result of safety rules violation. In 2
days the patient’s diurnal diuresis became 620 ml. The patient developed headache, vomiting, convulsions, dyspnea;
moist crackles are observed in the lungs. Name this pathology
{=Chronic renal failure
~Glomerulonephritis
~Pyelonephritis
~Acute renal failure
~Uremic coma}

A 38-year-old man, who has been suffering from systemic lupus erythematosus for 3 years, developed diffuse renal
lesions accompanied by massive edemas, marked proteinuria, hyperlipidemia, and dysproteinemia. What is the most
likely mechanism of proteinuria development in this case?
{=Inflammatory damage to the nephrons
~Increased blood proteins
~Morbid affection of the urinary tracts
~Autoimmune damage to the nephrons
~Ischemic damage to the tubules}

Poisoning caused by mercury (II) chloride (corrosive sublimate) occurred in the result of safety rules violation. In 2
days the patient’s diurnal diuresis was 620 ml. The patient developed headache, vomiting, convulsions, dyspnea;
moist crackles were observed in the lungs. Name this pathology:
{=Acute renal failure
~Uremic coma
~Pyelonephritis
~Glomerulonephritis
~Chronic renal failure}

Poisoning caused by mercury (II) chloride (corrosive sublimate) occurred in the result of safety rules violation. In 2
days the patient’s diurnal diuresis became 620 ml. The patient developed headache, vomiting, convulsions, dyspnea;
moist crackles are observed in the lungs. Name this pathology: {=Acute renal failure
~Chronic renal failure
~Uremic coma
~Glomerulonephritis
~Pyelonephritis}

14 days after quinsy a 15-year-old child presented with morning facial swelling, high blood pressure, "meat
slops"urine. Immunohistological study of a renal biopsy sample revealed deposition of immune complexes on the
basement membranes of the capillaries and in the glomerular mesangium. What disease developed in the patient?
{=Acute glomerulonephritis
~Acute interstitial nephritis
~Lipoid nephrosis
~Acute pyelonephritis
~Necrotizing nephrosis}
A patient with constant headaches, pain in the occipital region, tinnitus, dizziness has been admitted to the cardiology
department. Objectively: AP- 180/110 mm Hg, heart rate - 95/min. Radiographically, there is a stenosis of one of the
renal arteries. Hypertensive condition in this patient has been caused by the activation of the following system:
{=Renin-angiotensin
~Hemostatic
~Sympathoadrenal
~Kinin
~Immune}
Toxic pulmonary edema was reproduced on a laboratory rat by means of ammonium chloride
solution. What is the leading pathogenetic factor of this edema?
{=Increased permeability of capillars
~Increase of venous outflow
~Decrease of colloid osmotic pressure
~Disorder of neural and humoral regulation
~Increase of lymph outflow}

Examination of experimental rats that have been getting only carbohydrate feed for a long time
revealed accumulation of water in tissues. What is the leading pathogenetic mechanism of
edema development?
{=Hypooncotic
~Hyperosmolar
~Dysregulatory
~Membranogenic
~Lymphogenous}

A newborn child ill with pylorostenosis has frequent vomiting accompanied by apathy,
weakness, muscular hypertonia, sometimes convulsions. What form of acid-base balance
disorder has developed? {=Nongascous alkalosis
~Gaseous acidosis
~Gasceous alkalosis
~Metabolic acidosis
~Excretory acidosis}

A woman with intractable vomiting was admitted to the infectious disease ward. What changes
of water-salt metabolism are likely to be observed?
{=Hypoosmolar dehydration
~Hyperosmolar dehydration
~Isoosmolar dehydration
~Hypoosmolar hyperdehydration
~Hyperosmolar hyperdehydration}

12-year-old teenager has significantly put off weight within 3 months; glucose concentration
rose up to 50 millimole/L. He fell into a coma. What is the main mechanism of its development?
{=Hyperosmolar
~Ketonemic
~Hypoglycemic
~Lactacidemic
~Hypoxic}

A female patient with toxemia of pregnancy has hypersalivation resulting in a daily loss of 34
liters of saliva. What disorder of water-salt metabolism occurs in such cases?
{=Hyperosmolar hypohydration
~Hypoosmolar hypohydration
~Isoosmolar hypohydration
~Hypokalemia
~Hyponatremia}
A patient with enteritis accompanied with intense diarrhea has reduced quantity of water in the
extracellular space and increased quantity of water inside the cells as well as low blood
osmolarity. Name this disorder of water-electrolytic metabolism:
{=Hyposmolar hypohydration
~Osmolar hypohydration
~Hyperosmolar hyperhydration
~Hyposmolar hyperhydration
~Hyperosmolar hypohydration}

A patient was admitted to the infectious diseases department. His skin was dry, with low turgor;
he had rice-water stool. The patient was diagnosed with cholera. This disease is ordinarily
accompanied by the following disorder of water-electrolytic balance:
{=Isoosmotic hypohydration
~Hypoosmotic hypohydration
~Hyperosmotic hyperhydration
~Hyperosmotic hypohydration
~Hyposmotic hyperhydration}

The surgically excised connective tissue of the deformed mitral valve gives a basophilic
reaction when stained with hematoxylin and eosin. When stained with toluidine blue, it turns
purple (metachromasia). What changes of the connective tissue can be detected by these
reactions?
{=Mucoid edema
~Fibrinoid necrosis of connective tissue
~Connective tissue edema
~Petrification
~Hyalinosis}

A patient with liver cirrhosis has been given intravenously 500,0 ml of 5% glucose solution
along with other drugs. There is a high risk of the following water-electrolytic balance
disruption:
{=Hyposmolar hypohydration
~Hyperosmolar hypohydration ~Iso-osmolar
hypohydration
~Hyposmolar dehydration
~Hyperosmolar dehydration}

During postmortem examination of a 9-month-old infant tt was determined that the cause of
death was cerebral edema. What water-electrolyte imbalance is the most likely cause of the
edema development? {=Isoosmolar hyperhydration ~Hyperosmolar dehydration
~Hyperosmolar hyperhydration
~Isoosmolar dehydration
~Hypoosmolar hyperhydration}
An accident had resulted in a 65-year-old man drowning in a lake. Resuscitation measures
allowed to restore his respiration and cardiac function. What factor prolongs the period of
apparent death? {=Hyperthermia
~Elderly age

~Hypothermia
~Prolonged preagony and agony}

In a patient with chronic heart failure the increased hydrostatic pressure in vena cava inferior
was determined; that caused development of pathological process designated as:
{=Cardiac edema
~Liver swelling
~Renal edema
~Lymphocytic edema
~Toxic edema}

A 7-year-old boy after consumption of a seafood developed swellings and skin rash. What is the
cause of such reactions to food?
{=Increased capillary permeability
~High filter pressure
~Reduction of osmotic pressure gradient through the capillary wall
~Venules constriction
~Increased level of plasma proteins}

A patient was admitted to infectious department with complaints of extensive vomiting. What
disorders of water-salt metabolism are present in the patient?
{=Hypoosmotic dehydration
~Isoosmotic dehydration
~Hyperosmotic dehydration
~Hypoosmotic hyperhydration
~Hyperosmotic hyperhydration}

After a catastrophe people found themselves on the ocean island, where there were no fresh
water. What kind of water-salt metabolism has developed in those people?
{=Hyperosmotic hyperhydration
~Isoosmotic dehydration
~Hypoosmotic hyperhydration
~Hyperosmotic dehydration
~Hypoosmotic dehydration}

An experimental animal was placed in the thermostat at the temperature of 36°C. For a long
time the animal has presented with a significant increase in respiratory frequency. What kind
of water-electrolyte exchange disorder developed in this pathological state?
{=Hyperosmotic dehydration
~Hypoosmotic dehydration
~Isoosmotic dehydration
~Positive water balance
~Edema}

Diabetes mellitus is associated with osmotic diuresis. What kind of water-salt metabolism is
observed with this state? {=Hyperosmotic dehydration
~Isoosmotic dehydration
~Hypoosmotic dehydration
~Hypoosmotic hyperhydration
~Hyperosmotic hyperhydration}

A patient with an infectious disease accompanied with severe vomiting and diarrhea developed
hypoosmotic dehydration. What are the reasons of this pathological condition? {=Loss of salts
~Loss of water
~Inflammatory process
~Polydipsia
~Polyphagia}

Sever vomiting in a pregnant women led to the development of hypovolemia due to significant
dehydration. The main endocrine mechanism for compensation of this state is:
{=Increase of aldosterone level
~Increase of vasopressin (ADH) level
~Increase of corticotrope hormone synthesis
~Increase of prolactine secretion
~Increased production of calcitonine}

A patient was admitted to a hospital with complaints of general weakness, headaches, pain in
the lumbar region, massive swelling of the face and limbs, accompanied by pronounced
proteinuria, leukocyturia, cylinders detected in urine analysis. What are the leading
pathogenetic mechanisms of edema? {=Reduction of oncotic blood pressure ~Increase of
vascular permeability
~Increase of hydrostatic blood pressure
~Hormonal balance disorders
~Lymph flow impairment}

Experimental rats that have long received only carbohydrate diet, the accumulation of water in
tissues was observed. What is the main pathogenetic mechanism in the development of edema
in this case? {=Hypooncotic
~Membranogenic
~Dysregulatory
~Lymphogenous
~Hyperosmotic}

With prolonged fasting a human develops "hungry" swellings. What are the reasons of this
phenomenon?
{=Reduction of oncotic blood pressure
~Reduction of vasopressin secretion
~Increased secretion of vasopressin
~Reduction of osmotic pressure in tissues
~Increased oncotic blood pressure}
A 6-year-old girl after she ate an orange, developed edema of eyelids, lips, neck, tongue
mucosa. The girl had the reaction to oranges before, which manifested as skin rashes, and its
itching. What is the pathogenetic mechanism leading to the development of edema in the girl?
{=Increased capillary wall permeability
~Lymph flow obstruction
~Increased oncotic tissue fluid pressure
~Increased hydrostatic pressure of the blood in capillaries
~Decrease of oncotic blood pressure}

A 32-year-old woman has sustained a wasp bite. Edema and hyperemia developed at the site of
the bite. What is the primary mechanism of edema in this case?
{=Increased capillary wall permeability
~Increased hydrostatic pressure of the blood in capillaries
~Increased oncotic tissue fluid pressure
~Lymph flow obstruction
~Decrease of oncotic blood pressure}

A patient is diagnosed with mixedema. What kind of water-clectrolyte exchange disorder is


observed in the patients? {=Isoosmotic hyperhydration
~Hypoosmotic hyperhydration
~Hyperosmotic hyperhydration
~Isoosmotic hypohydration
~Hypoosmotic hypohydration}

A patient is hospitalized due to chronic heart failure. What kind of water-electrolyte exchange
disorder will be the observed in the patient?
{=Isoosmotic hyperhydration
~Hypoosmotic hyperhydration
~Hyperosmotic hyperhydration
~Isoosmotic hypohydration
~Hypoosmotic hypohydration}

After a ship wrack in the ocean, sailors were deprived of fresh water. What changes in relation
to osmotic pressure and extracellular fluid volume are observed as a result of drinking salty
water?
{=Volume increases, osmotic pressure increases
~Volume is reduced, osmotic pressure increases
~Volume increases, osmotic pressure decreases
~The volume does not change osmotic pressure decreases
~Volume decreases with decreasing osmotic pressure}

An experimental animal was given water load supported with vasopressin (ADH). What kind of
water-electrolyte exchange disorder will be the observed in the animal?
{=Hypoosmotic hyperhydration
~Isoosmotic hyperhydration
~Hyperosmotic hyperhydration
~Isoosmotic hypohydration
~Hypoosmotic hypohydration}
A patient with acute glomerulonephritis developed acute renal failure. There is total diuresis
deficiency. What kind of water-electrolyte exchange disorder will be the observed in the
patient?
{=Hypoosmotic hyperhydration
~Isoosmotic hyperhydration
~Hyperosmotic hyperhydration
~Isoosmotic hypohydration
~Hypoosmotic hypohydration}

A patient is diagnosed with glomerulonephritis. The increased blood pressure and swellings are
observed. What is the pathogenesis of edema in glomerulonephritis?
{=Decreased glomerular filtration
~Increased glomerular filtration
~Phlebostasis (congestion in the veins)
~Increased filtration of plasma in capillaries
~Increased capillary wall permeability}

A patient is diagnosed with liver cirrhosis, accompanied by the development of edema. What is
the leading mechanism in the development of hepatic edema?
{=Hypoproteinemia
~Hyperproteinemia
~Phlebostasis (congestion in the veins)
~Increased capillary wall permeability
~Increased filtration of plasma in capillaries}

A patient is diagnosed with angiotrophoneurosis that is accompanied by the development of


edema. What kind of edema by its etiology will be observed in the patient?
{=Neurogenic
~Inflammatory
~Allergic
~Toxic
~Lymphogenous}

A patient is diagnosed with cancer of the stomach, accompanied by the development of edema.
The cause of edema is:
{=Hypoproteinemia
~Hyperproteinemia
~Phlebostasis (congestion in the veins)
~Increased capillary wall permeability
~Increased filtration of plasma in capillaries}

A patient was delivered to a hospital and diagnosed with Quincke’s edema. The main
pathogenetic factor in the development of allergic edema is:
{=Increased capillary wall permeability
~Decreased capillary wall permeability
~Decreased oncotic blood pressure
~Increased oncotic blood pressure
~Phlebostasis (congestion in the veins)}

3 days after acute blood loss a man developed signs of hypovolemia. What kind of
waterelectrolyte exchange disorder will be the observed in the patient?
{=Isoosmotic hypohydration
~Hypoosmotic hypohydration
~Hyperosmotic hypohydration
~Isoosmotic hyperhydration
~Hypoosmotic hyperhydration}

A man, after he had celebrated a wedding, developed signs of poisoning: vomiting, diarrhea,
and fever. As a result of these symptoms he drank small amount of fluids. What kind of water-
electrolyte exchange disorder will be observed in the patient?
{=Hyperosmotic hypohydration
~Isoosmotic hypohydration
~Hyperosmotic hypohydration
~Isoosmotic hyperhydration
~Hypoosmotic hyperhydration}

A patient has developed isoosmotic hypohydration. What reason can lead to this pathology?
{=Acute blood loss
~Diarrhea
~Vomiting
~Sweating
~Hyperventilation}

A patient has developed hypoosmotic hypohydration. What reason can lead to this pathology?
{=Vomiting
~Hypersalivation
~Acute blood loss without drinking water
~Sweating
~Hyperventilation}

An excessive amount of 0.5% solution of sodium chloride was administered to a patient. What
kind of water-clectrolyte exchange disorder will be the observed in the patient?
{=Hypoosmotic hyperhydration
~Isoosmotic hyperhydration
~Hyperosmotic hyperhydratation
~Hypoosmotic hypohydratation
~Hyperosmotic hypohydration}

An excessive amount of 0.9% solution of sodium chloride was administered to a patient. What
kind of water-clectrolyte exchange disorder will be the observed in the patient?
{=Isoosmotic hyperhydration
~Hypoosmotic hyperhydration
~Hyperosmotic hyperhydratation
~Hypoosmotic hypohydratation
~Hyperosmotic hypohydration}

An excessive amount of 0.05% solution of sodium chloride was administered to a patient. What
kind of water-clectrolyte exchange disorder will be the observed in the patient?
{=Hypoosmotic hyperhydration
~Isoosmotic hyperhydration
~Hyperosmotic hyperhydratation
~Hypoosmotic hypohydratation
~Hyperosmotic hypohydration}

A man has sustained a long deprivation of drinking water. What pathological changes may result
from dehydration?
{=Oliguria
~Urorrhagia
~Pulmonary edema
~Cachexia
~Vomiting}

A patient developed dehydration. What pathological conditions can cause dehydration?


{=Vomiting
~Excessive consumption of sodium chloride
~Hypothermia
~Institution of isotonic solutions
~All the listed above}

A patient developed dehydration. What pathological processes can cause dehydration? {=Burns
~Allergies
~Hypothermia
~Infection
~All the listed above}

A patient with liver cirrhosis developed edema. Disorders of which liver functions promote the
development of hepatic edema?
{=Protein synthesis
~Bile-forming
~Deposition of microelements
~Deposition of macroelements
~Synthesis of urea}

After a bee sting a child developed swelling. In the pathogenesis of what kind of edema the
leading role is played by biologically active substances? {=Allergic ~Cardiac
~Kidney
~Starving
~All the listed above}

A patient complains of edema of the legs that emerges in the evening. Increase of what indexes
promote development of edema? {=Hydrostatic blood pressure
~Intercellular fluid hydrostatic pressure
~Oncotic blood pressure
~Osmotic blood pressure
~All the listed above}

A child developed laryngeal edema. In the pathogenesis of what kind of edema the leading role
is played by biologically active substances?
{=Inflammatory
~Cardiac
~Renal
~Starving
~All the listed above}

A patient with thrombophlebitis has developed lower extremities edema in the evening. In the
pathogenesis of what kind of edema the leading role is played by increase of tissue oncotic
pressure? {=Inflammatory
~Cardiac
~Renal
~Starving
~All the listed above}

In a patient who has worked in dangerous conditions, edema began to appear. In the
pathogenesis of what kind of edema the leading role is played by increase of tissue oncotic
pressure? {=Toxic
~Cardiac
~Renal
~Starving
~All the listed above}

A 34-year-old woman, who has long been on a diet, developed swellings. In the pathogenesis
of what kind of edema the leading role is played by hypoproteinemia?
{=Cachectic
~Cardiac
~Allergic
~Inflammatory
~All the listed above}

In a patient, who has long worked in a paintwork factory, swellings began to appear. In the
pathogenesis of what kind of edema the leading role is played by hypoproteinemia?
{=Liver
~Cardiac
~Allergic
~Inflammatory
~All the listed above}

A 44-year-old woman, who has long been on a diet, developed swellings. In the pathogenesis
of what kind of edema the leading role is played by hypoproteinemia?
{=Starving
~Cardiac
~Allergic
~Inflammatory
~All the listed above}

A patient has developed edema. In the pathogenesis of what kind of edema the leading role is
played by increased vessel wall permeability?
{=Allergic
~Cardiac
~Cachectic
~Mixedematous
~Liver}

A patient with thrombophlebitis has developed lower extremities edema in the evening. In the
pathogenesis of what kind of edema the leading role is played by increased vessel wall
permeability? {=Inflammatory
~Cardiac
~Cachectic
~Renal
~Starving}

A patient complains of breathlessness, palpitations, swelling of the lower extremities. In the


pathogenesis of what kind of edema the leading role is played by increased hydrostatic blood
pressure? {=Cardiac
~Cachectic
~Allergic
~Inflammatory
~All the listed above}

There is a sick woman with mixedema in endocrinological clinic. What is the waterelectrolyte
exchange disorder observed in this patient?
{=Isoosmotic hyperhydration
~Hypoosmotic hyperhydration
~Hyperosmotic hyperhydratation
~Isoosmotic hypohydration
~Hypoosmotic hypohydratation}

A patient has been hospitalized due to chronic heart failure. What is the water-clectrolyte
exchange disorder observed in this patient?
{=Isoosmotic hyperhydration
~Hypoosmotic hyperhydration
~Hyperosmotic hyperhydratation ~Isoosmotic
hypohydration
~Hypoosmotic hypohydratation}

A man 3 days after acute blood loss developed signs of hypovolemia. What is the
waterelectrolyte exchange disorder observed in this patient?
{=Isoosmotic hypohydration
~Hypoosmotic hypohydratation
~Hyperosmotic hypohydratation
~Isoosmotic hyperhydration
~Hypoosmotic hyperhydration}

A man, after he had celebrated a wedding, developed signs of food poisoning: vomiting,
diarrhea, and fever. As a result of these symptoms he drinks small amount of fluids. What kind
of water-electrolyte exchange disorder will be observed in the patient?
{=Hyperosmotic hypohydration
~Isoosmotic hypohydration
~Hypoosmotic hypohydration
~Isoosmotic hyperhydration
~Hypoosmotic hyperhydration}

A patient is diagnosed with liver cirrhosis. What is the leading mechanism in the development
of ascites?
{=Increased hydrostatic pressure in the portal venous system
~Increased glomerular filtration
~Phlebostasis (congestion in the veins)
~Increased filtration of plasma in capillaries
~Increased capillary wall permeability}
During earthquake a man was pressed by a large stone that dropped on him and pressed him
the hip. What extreme state has arisen as a result of prolonged crush of the soft tissues of
extremities?
{=Crush syndrome
~Coma
~Traumatic toxemia
~Traumatic shock
~Collapse}

The binder was overlayed to a patient with brachial artery injury. After 4.5 hour the binder was
removed. What extreme state can develop after removal of the binder?
{=Tourniquet shock
~Traumatic shock
~Traumatic toxemia
~Crush syndrome
~Pain shock}

During repair work a worker fell from the roof of a building and injured the liver and spleen.
The patient had developed an internal bleeding. What extreme state can develop due to bleeding
in the patient?
{=Hemorrhagic shock
~Traumatic shock
~Cardiogenic shock
~Anaphylactic shock
~Pain shock}

A patient with shock has an acute blood circulation failure. What mechanisms underlies its
implementation?
{=All the stated
~Reduced volume of circulating blood
~Reduction of heart minute volume
~Reduced total peripheral resistance
~Abnormal rheological properties of the blood}

A patient developed collapse. What is the main feature of collapse?


{=The fall of blood pressure
~Loss of memory
~Absence of reflexes
~Tachycardia
~Bradycardia}

A 60-year-old woman suffering from pneumonia has been bedridden for 2 weeks. After
prolonged stay in the bed the patient stood up and 15 seconds later fainted. Skin became pale,
pulse accelerated, blood pressure lowered, pupils expanded. What extreme situation did the
woman develop? {=Collapse
~Traumatic shock
~Anaphylactic shock
~Cardiogenic shock
~Bum shock}
A 40-year-old man has visited sauna for the first time. First 5 minutes the man felt satisfactory,
but later he suddenly lost consciousness. What kind of extreme state arose in the patient?
{=Vasodilatation collapse
~Hypocapnic collapse
~Cardiogenic collapse
~Toxic-infectious collapse
~Posthemorrhagic collapse}

A 30-year-old female has been hospitalized to psychiatric clinic and stayed bedridden for 2
weeks. Then she was allowed to stand up and walk. The patient got up, but suddenly lost her
consciousness and fell down. Which state is responsible for the loss of consciousness in the
woman?
{=Canting collapse
~Posthemorrhagic collapse
~Hypocapnic collapse
~Hyperthermic collapse
~Arrhythmic collapse}

A 20-year-old man with foot gangrene developed toxic-infectious shock. What is key in the
pathogenesis of this type of shock?
{=Products of tissues decay
~Abnormal microcirculation
~Decrease of blood pressure
~Fever
~Oliguria}

A patient has been diagnosed with the first stage of traumatic shock. What change in the function
of the nervous system is supposed to be observed?
{=Excitation of the central nervous system
~Inhibition of the central nervous system
~Absence of response of the central nervous system
~Function of the central nervous system is not changed
~Nothing is correct}

A patient has been diagnosed with the second stage of traumatic shock. What change in the
function of the nervous system is supposed to be observed?
{=Inhibition of the central nervous system
~Excitation of the central nervous system
~Absence of response of the central nervous system
~The function of the central nervous system is not changed
~Nothing is correct}

What is the approximate beginning and the end of the intermediary period (dominated by acute
renal failure) of the crash-syndrome?
{=From the 3rd_to 12th day
~Before the 3 day
~From the 12th to 24th day
~From the 24th to 30th day
~From the Ist to 2nd month}
A man with peptic ulcer disease, complicated by severe gastric bleeding, has been hospitalized
in an unconscious state. The patient developed posthemorrhagic collapse. Which factor is
responsible for the development of this state?
{=Hypovolemia
~Hypervolemia
~Compensatory angiospasm
~Blood exit from the depot
~All the listed above}

A 40-year-old miner developed crush syndrome due to a mine collapse. Which of the listed
below factors are important in the development of this state?
{=All of them
~Pain
~Traumatic toxemia
~Plasma loss
~Blood loss}

A patient developed hypocorticoid coma. What is the leading pathogenetic mechanism in the
development of this state? {=Water-electrolyte disorder
~Energetic deficiency
~Acid-base disorder
~Abnormal intracranial pressure
~All is correct}

A 25-year-old patient developed hyperosmotic diabetic coma. What abnormality is the leading
one in the pathogenesis of this disorder?
{=Water-electrolyte disorder
~Energetic deficiency mechanism
~Acid-base disorder
~Increased intracranial pressure
~All is correct}

A patient developed hyperthermic coma. What abnormality is the leading one in the
pathogenesis of this disorder? {=Water-electrolyte disorder
~Energetic deficiency mechanism
~Acid-base disorder
~Increased intracranial pressure
~All is correct}

A patient developed hypoxic coma. What abnormality is the leading one in the pathogenesis of
this disorder?
{=Energetic deficiency mechanism
~Water-electrolyte disorder
~Acid-base disorder
~Increased intracranial pressure
~All is correct}

A 30-year-old patient with the insulin dependent diabetes mellitus developed hypoglycaemic
coma. What abnormality is the leading one in the pathogenesis of this disorder?
{=Energetic deficiency mechanism
~Water-electrolyte disorder
~Acid-base disorder
~Increased intracranial pressure
~All is correct}

A patient developed respiratory coma. What abnormality is the leading one in the pathogenesis
of this disorder? {=Energetic deficiency mechanism
~Water-electrolyte disorder
~Acid-base disorder
~Increased intracranial pressure
~All is correct}

As a result of transfusion of incompatible blood a patient developed haemolytic coma. What is


the leading mechanism in the development of this state?
{=Energetic deficiency mechanism
~Water-electrolyte disorder
~Acid-base disorder
~Increased intracranial pressure
~All is correct}

A patient developed acidic diabetic coma. What is the leading mechanism in the development
of this state?
{=Acid-base disorder
~Water-electrolyte disorder
~Energetic deficiency mechanism
~Increased intracranial pressure
~All is correct}

A patient developed lactate-acid coma. What abnormality is the leading one in the pathogenesis
of this disorder? {=Acid-base disorder
~Water-electrolyte disorder
~Energetic deficiency mechanism
~Increased intracranial pressure
~All is correct}

A patient with a cranial-cerebral trauma developed coma. What abnormality is the leading one
in the pathogenesis of this disorder?
{=Increased intracranial pressure
~Water-electrolyte disorder
~Energetic deficiency mechanism
~Acid-base disorder
~All is correct}

A patient with extensive burns of the second and third degrees developed shock. What is (are)
the main factor(s) in the pathogenesis of this type of shock?
{=All these factors
~Pain
~Toxaemia
~Dehydration
~Infection of bur surfaces}

A patient developed hypovolemic collapse. What could be the cause of its development? {=All
these factors
~Profuse diarrhoea
~Extensive areas of burn
~Acute massive bleeding
~Redistribution of blood with its deposition it in the venous vessels}

A patient with insulin dependant diabetes mellitus administered himself a dose of insulin, but
was not able to eat. What complications can develop in the patient?
{=Hypoglycaemic coma
~Hyperglycaemic coma
~Hipocorticoid coma
~Hepatic coma
~Pancreatic coma}

A patient with insulin dependant diabetes mellitus shortly after taking insulin developed
hunger, dizziness, weakness, cold sweat, and irritability. What caused the emergence of these
signs?
{=Carboniferous starvation of the brain
~Increased dissolution of glycogen in the liver
~Increased ketogenesis
~Increased adipogenesis
~Decreased gluconeogenesis}

A 42-year-old woman was hospitalized to the intensive care unit due to lactic coma.
Accumulation of what substance in the patient’s blood caused the coma?
{=Lactate
~Format
~Succinic acid
~Ketone bodies
~Urea}
A patient is drowsy, his conscious is depressed, and his reactions to irritants are suspended. He
has a pale dry and edematous skin, muscular fibrillations, mydriasis, and Cheyne-Stocks’
respiration with ammonium scent from his mouth. Pericardial friction sound was found at
auscultation of the patient’s heart. What kind of coma has developed in this patient?
{=Renal
~Ketoacidotic
~Hyprosmolar ~Hepatic
~Apoplectic}
A patient was hospitalized to hospital with hypoglycaemic comma. What is primary in the
pathogenesis of this kind of coma?
{=Reduced synthesis of ATP in the brain neurons
~Inhibition of neuronal function
~Reduction of ionic gradients and depolarization of neurons
~Loss of consciousness
~Reduced activity of sodium-potassium pump}

A patient with insulin dependant diabetes mellitus developed hypoglycaemic coma shortly after
insulin administration. At which glucose levels in the blood can this coma develop? {=<2.5
mmol / 1
~<3.5 mmol /1
~<4,5 mmol /1
~<5.5 mmol /1
~<6.5 mmol / 1}

A 14-year-old adolescent has sharply lost his body weight for 3-month period and developed
coma. Blood glucose levels are 50 mmol/l. What is the main mechanism of this coma?
{=Hyperosmotic
~Hypoglycaemic
~Ketoacidic
~Lactate-acidic
~Hypoxic}

A 50-year-old female was hospitalized by "ambulance" to the hospital. Hyperosmotic coma was
diagnosed. At which blood glucose levels can this coma develop?
{=> 50 mmol /1
~> 40 mmol /1
~> 30 mmol /1
~> 20 mmol / 1
~> 10 mmol / 1}

A 47-year-old male was hospitalized by "ambulance" to the hospital. Hyperosmotic coma was
diagnosed. What underlies the development of this type of coma?
{=Hyperglycaemia
~Accumulaton of ketone bodies
~Glycosuria
~Hypoglycaemia
~Hypernatraemia}
A victim of the earthquake felt well just after he was extracted from under ruins, but soon
abrupt worsening of his condition developed. He lost consciousness, his BP was of 70/35
mmHg, and his pulse was of 90 per min; edema of previously ischemic tissues increased
quickly. Which is the main cause of patient’s condition worsening?
{=Systemic activation of proteolytic processes and PLO
~Thrombosis of subcutaneous venous vessels
~Dehydration
~Increase in tonus of sympathetic nervous system
~Disorders of renal functions}
A patient with chronic alcoholic hepatitis developed hepatic coma. Impairment of what liver
function led to the development of this coma?
{=Antitoxic function
~Phagocytic function
~Protein synthesis function
~Inactivation of hormones
~Metabolic function}

A patient with hepatitis B developed coma. What is the main mechanism in the pathogenesis of
this type of coma?
{=Increase of neurotoxic substances in the blood
~Decreased synthesis of ATP in the brain neurons
~Impairment of metabolic liver function
~Portal hypertension
~Loss of consciousness}

A patient with liver cirrhosis developed hepatic coma. What changes of acid-base status will be
observed in this type of coma?
{=Metabolic acidosis
~Gas alkalosis
~No change
~Metabolic alkalosis
~Gas acidosis}

A patient with chronic pyelonephritis with chronic renal insufficiency developed uremic coma.
What changes in the blood are characteristic of this pathologic state?
{=Azotemia
~Ketonemia
~Hyperglycemia
~Hematuria
~Proteinuria}

A patient with uremic coma presents with anorexia, nausea, and vomiting. What caused the
development of these symptoms?
{=Toxic damage to the central nervous system
~Toxic liver damage
~Toxic damage to the pancreas
~Toxic damage to the stomach
~Toxic damage to intestine}

A patient was admitted to the hospital in severe state. He does not answer the questions and
does not react to pain stimuli. Patient’s BP is 50/10 mmHg, his pulse is 50 per min. What are
the reasons for disorders of system hemodynamics at torpid phase of shock?
{=Total decrease in peripheral vascular resistance
~Elevation of venous return of blood
~Increase of CBV
~Rise of tonicity of sympathetic nervous system
~Decrease in permeability of exchange vessels}

A patient in comatose state has smell of acetone from his mouth. Content of glucose in his
blood plasma is of 18 mmol/L. What kind of coma is the most possible one in this case?
{=Ketoacidemic ~Hyperosmolar.
~Hypoglycemic
~Toxic
~Lactatacidemic}

A patient with diabetes mellitus developed ketoacidic coma. What caused its development?
{=Accumulation of ketone bodies
~Deficiency of ketone bodies
~Hypoglycaemia
~Metabolic alkalosis
~Accumulation of lactic acid}

A patient with crushed muscular tissue was admitted to the traumatological department. Which
biochemical index of urine is increased in this case?
{=Creatinin
~Mineral salts
~Uric acid
~General lipids
~Glucose}

A 35-year-old man has massive trauma of lower extremities without considerable external
bleeding. The victim is in exited condition. What component of pathogenesis of traumatic
shock is leading and needs immediate correction?
{=Pain
~Internal bleeding
~Intoxication
~Disorder of organ functions
~Internal loss of plasma}

A patient with burn of 30% of body surface has decrease in BP to 75/20 mm Hg, frequent,
filiform pulse (110/min). What is the main factor of decrease in BP in burn shock?
{=Plasmarrhea
~Intoxication by decay products
~Activation of sympathetic nervous system
~Hypoproteinemia
~Activation of parasympathetic nervous system}

Intravenous infusion of blood and blood substitutes does not lead to the stabilization of BP in
a patient with torpid phase of shock. After cessation of infusion patient’s blood pressure
decreases again. What does inefficacy of medical treatment in torpid phase of shock connected
to?
{=Systemic increase in vessel permeability
~Hemodilution
~Increase in velocity of blood flow
~Tachycardia
~Spasm of resistant vessels}
Levels of plasma proteins are sharply increasing, and the number of alveolar macrophages and
lymphocytes is decreasing in bronchoalveorlar contents of a patient with shock. What is the
mechanism of these phenomena appearance?
{=Increase in permeability of lung capillaries
~Spasm of resistant vessels of the lungs
~Increase in inhibitor systems of the lungs
~Increase in pulmonary blood flow
~Hyperventilation}

A patient with torn away finger was admitted to the traumatological department. He is fussy,
verbose, and pale. His pulse is 120 beats per minute, BP 140/90 mmHg. What are the
peculiarities of disorders of microcirculation and systemic hemodynamics in erectile phase of
shock?
{=Increase in systemic peripheral resistance
~Systemic spasm of volume vessels
~Decrease in venous return and volume of blood circulation
~Increase in peripheral resistance in the brain, heart and lungs vessels
~Hypoxia in zones with beta-adrenoreceptors}

Hemodialysis was utilized for treatment for acute renal failure developed in a patient as a result
of traumatic shock. While carrying out the hemodialysis patient’s condition became worse,
aggregation of erythrocytes, leukocytes, and platelets occurred in microvessels, and blood
viscosity increased. What kind of disorders of microcirculation developed in the patient?
{=Sludge syndrome
~Disorders of vascular permeability
~Extravascular disorders
~Capillary-trophic insufficiency
~Lymphatic system insufficiency}

Tourniquet was applied to upper third of hip of wounded driver just at place of car accident.
The patient was admitted to a surgical department in satisfactory condition 3 hours later.
Marked edema of hip tissues, frequent pulse, cold perspiration, and expressed hypotension
develop in the patient after removal of tourniquet. Which pathological process develops in the
patient?
{=Toxic shock
~Anaphylactic shock
~Collapse
~Cardiogenic shock
~Hemorrhagic shock}

Daily diuresis amounts to 250 ml in a patient with anaphylactic shock. The patient has moist
rales in his lungs; his consciousness is intact. In patient’s blood acidosis reveals with base
deficiency of 14.5 mmol/L; plasma contents of potassium is 8.8 mmol/L and urea is 48 mmol/L.
How is this disorder of kidney called?
{=Acute renal failure
~Acute diffuse glomerulonephritis
~Uremic coma
~Chronic renal failure
~Chronic glomerulonephritis}
Spasm of resistant vessels develops in zones with alpha-adrenoreceptors in erectile phase of
shock. What hemodynamic changes will be observed in this case?
{=Decrease of blood flow speed
~Centralization of blood flow
~Bradycardia
~Decrease of venous return
~Systemic decrease of peripheral blood flow}

Sharp decrease of systolic BP down to 60 mmHg, tachycardia of 140 beats per minute, dyspnea,
and loss of consciousness developed in a patient on the second day after myocardial infarction.
What pathways have the most important significance in pathogenesis of this shock? {=Decrease
in stroke volume
~Decrease in blood volume
~Development of paroxysmal tachycardia
~Increase in excitability of the myocardium by products of necrotic decay ~Development
of anaphylactic reaction to myocardial proteins}

A driver was admitted to an emergency department after a car accident. He does not react to
questions; he is indifferent to everything, pale; he has shallow and infrequent respiration and
BP of 75/50 mmHg. Name the principal link in pathogenesis of this pathology.
{=Inhibition of CNS
~Excitement of CNS
~Loss of blood
~Toxemia
~Redistribution of blood}

A patient was admitted to a hospital with acute high bowel obstruction. He had prolonged
vomiting and blood pressure decreased down to 60/40 mmHg. Which mechanism of shock
development is principal one in this case?
{=Hypovolemia
~Exhaustion of arteriolar alpha-adrenoreactivity
~Acidotic dilation of metarteriols ~Loss
of chlorides
~Increase in tonus of vagus nerve}

After a car accident a patient has BP of 70/40 mmHg; he is unconscious; his daily diuresis is
about 300 ml. What is the mechanism of urinopoiesis disturbances in this case?
{=Decrease in glomerular filtration
~Reduction of tubular reabsorption
~Augmentation of glomerular filtration
~Augmentation of tubular reabsorption
~Reduction of tubular secretion}

Severe burn shock has developed in a patient with 40% of body surface burnt. What is the cause
of shock development?
{=Pain
~Mineral metabolism disturbances
~Autoimmunization of the organism
~Protein metabolism disturbances
~Dehydration of the organism}
Novocain solution was injected to a patient for anesthetization at extraction of a carious tooth.
Drop of BP, loss of consciousness, dyspnea, and convulsions develop in the patient in a few
minutes. What is the reason for anaphylactic shock development?
{=Sensitization to Novocain
~Toxic action of Novocain
~Desensitization of the organism
~Autoallergic state
~Para-allergy}

Sharp weakness, paleness of skin, lost of consciousness appeared in a patient the next day after
resection of his stomach. The patient’s BP is 70/40 mmHg; pulse is 160 beats per minute. In
the patient’s blood test Hb is 70 g/L, erythrocytes are 2.3x10'7/L. What pathology appeared in
a patient? {=Hemorrhagic collapse
~Pain shock
~Traumatic shock
~Orthostatic collapse
~Cardiogenic collapse}

To compensate considerable loss of blood resulting from knife wounding of liver, a 30-yerold
patient with blood of TV (AB) Rh (-) group was transfused with blood of I'V(AB) Rh (+) group.
Requirement in repeated blood transfusion appears in several days. What kind of blood is it
possible to use for transfusion?
{=IV(AB) Rh (-)
~I(O) Rh (4+)
~II(A) Rh(-)
~IV(AB) Rh (+)
~II(B) Rh (-)}
A teeth was extracted in a teenager with Novocain utilization. Paleness of skin, dyspnea, and
hypotension developed in the teenager 10 minutes later. What kind of allergic reactions
developed in the teenager?
{=Anaphy lactic ~Stimulating
~Immune complex
~Cytotoxic
~Cell-mediated}

A 45-year-old victim with severe craniocerebral trauma was admitted to the hospital by
emergency team. Shockogenic trauma was diagnosed: loss of consciousness, paleness of skin,
decrease in body temperature down to 35°C, decrease in muscular tonus, absence of reflexes,
frequent and weak pulse, and decrease in BP down to 50/30 mmHg appeared in the patient.
Which clinic stage of traumatic shock is the patient in?
{=Terminal
~Erectile
~Excitation
~Inhibition
~Torpid}

A doctor established shock condition in a patient who fell from high altitude. What is the most
important in traumatic shock development?
{=Disturbances of the neurohumoral regulation
~Toxemia
~Disorders of regulation of the hemodynamics
~Hypovolemia
~Disturbances of renal functions}

A patient developed anaphylactic shock in a dental room after injection of Lidocain solution.
Which immunoglobulins underlie anaphylactic shock development?
{=Ig E
~Ig M
~Ig A
~IgG
~Ig D}

Traumatic shock developed in a patient as a result of severe trauma. Neuroendocrine,


hemodynamic, and metabolic disorders took place during the shock development. Clinically,
after erectile stage of shock, another stage developed, which is referred to as:
{=Torpid
~Septic
~Chronic
~Neurocirculatory
~Ischemic}

Anaphylactic shock developed in a patient after injection of local anesthetic drug. Which
mechanism of blood circulation disorders is the leading one in this condition?
{=Decrease in tonus of vessels
~Hypovolemia
~Pain
~Activation of sympathetic-adrenal system
~Reduction of contractile function of the heart}

Hypovolemic shock and signs of severe hypoxia developed in a victim of aircraft crash due to
severe hemorrhage. In this situation the most severe disorders for the organism develop in:
{=Heart
~Kidneys
~Lungs
~Liver
~§Spleen}

A 27-year-old patient with injury to the neck has lost approximately 30% of the blood volume.
The patient’s condition is severe: blood pressure is 60/40 mm Hg, heart rate is 140/min.,
respiratory rate is 30/min., conscious. Characterize the condition of the patient’s circulatory
system: {=Hypovolemic shock
~Cardiogenic shock
~Collapse
~Coma
~Arterial hypertension}
An unconscious patient was delivered by ambulance to the hospital. On objective examination
the patient was found to present no reflexes, periodical convulsions, irregular breathing. After
laboratory examination the patient was diagnosed with hepatic coma. Disorders of the central
nervous system develop due to accumulation of the following metabolite:
{=Ammonia
~Urea
~Glutamine
~Bilirubin
~Histamine}

A 62-year-old patient has been hospitalized due to massive cerebral hemorrhage. Blood
pressure is 70/30 mm Hg, heart rate is 120/min., respiratory rate is 4/min., unconscious, no
response to external stimuli. Such condition can be determined as:
{=Collapse
~Shock
~Coma
~Stress
~Agony}

After a road accident a victim has tachycardia, arterial blood pressure 130/90 mm Hg,
tachypnoe, the skin is pale and dry, excitation of central nervous system is observed. What
shock stage is the patient most likely in?
{=Erectile
~Terminal
~Torpid
~Preshock (compensation stage)
~Agony}

As a result of a trauma a patient has developed traumatic shock that led to the following
disorders: AP is 140/90 mm Hg, Ps is 120 bpm. The patient is fussy, talkative, pale. Such state
relates to the following shock phase:
{=Erectile
~Latent period
~Terminal
~Torpid
~—}
A patient has been diagnosed with influenza. His condition became drastically worse after
taking antipyretic drugs. His consciousness is confused, AP is 80/50 mm Hg, Ps is 140/m, body
temperature droped down to 35, 80C. What complication developed in this patient? {=Collapse
~Hyperthermia
~Hypovolemia
~Acidosis
~Alkalosis}
Topic 1: Subject, tasks and methods of pathophysiology.
General nosology, etiology and pathogenesis.

1. A 49-year-old man was suffering 12 years ago from rheumatic


myocarditis, endocarditis, and insufficiency of mitral valve.
Examinations showed the absence of inflammatory process, sufficient
minute blood volume. What is it?
A. Pathological reaction
B. Pathological process
C. Typical pathological process
D. Compensatory reaction
E. Pathological condition

2. The 12-year-old boy took part in sport events in light athletic.


Weakness, headache, loss of appetite, and rise in temperature to
37,80C appeared in him on the next day. What period of disease is it?
A. Final period of disease
B. Period of manifestation
C. Contact period
D. Latent period
E. Prodromal period

3. Pain ina left half of the chest and dyspnea appear in a patient during
walking. This pain increases during respiration. The attack of the pain
was stopped by using narcotics. At examination of the patient
following data were revealed: severe condition, respiratory rate is 28
per min., respiration is shallow and is dull in left axillary region. Blood
pressure is 140/80 mmHg. Patient has sputum with blood. This patient
has varicose dilation of veins. In purpose to correct main pathogenetic
link it is necessary to use:
A. Antibiotics
B. Anticoagulants
C. Antihistamin preparations
D. Spasmolytics
12
E. Coronarilytics.
4. A 39-years-old patient has been suffering from gastric ulcer for last 4
years. Pain in epigastric region, heartburn, nausea, and constipation
appear mainly in autumn and spring. Name this condition.
A. Remission
B. Acute period
C. Complication
D. Pathologic condition
E. Relapse
5. Gasping respiration appears in a patient with severe ling pathology.
What terminal condition is this characteristic for?
A. Agony
B. Pre-agony
C. Clinical death
D. Biological death
E. Terminal pause
6. A 10-yeaes-old child endured several rheumatic attacks. At
examination of him it was established that he had inflammatory
process in his joints and signs of mitral valve insufficiency. What
pathological appearance in this patient may be attributed to “disease”?
A. Mitral valve disease
B. Mitral valve insufficiency
C. Rheumatism
D. Inflammation of joints
E. Arthritis

7. Which scientist emphasized senescence of connective tissue cells


cytoplasm?
A. Bogomolets
B. Mechnikov
C. Dilman
D. Frolkis
E. Berdichev
13
8. A 12-year-old boy starts complaining on the headache, nausea, rigor,
periodic myalgia, loss of appetite, tiresome. Which period of disease
characterizes by such symptoms?
A. Prodromal period
B. Latent period
C. Period of contact
D. Period of manifestation
E. Final period

A 28-year-old patient with symptoms of acute appendicitis was


admitted to the surgical department. Acute pains in his right epigastric
area and near the umbilicus were registered during palpation on
examination. Schetkin-Blumberg symptom was positive. Which
period of disease was observed in that patient?
A. Period of manifestation
B. Latent period
C. Prodromal period
D. Final period
E. Period of functional disorder

10 . Increase in pulse rate, respiratory rate, and increase of BP were noticed


ona height of 1000 m above the sea in a 25-year-old woman, coming
for holyday at mountainous resort. In some time all those symptoms
disappeared. Which process was noticed in a woman?
A. Adaptation
B. Compensation
C. Decompensation
D. Stress
E. Parabiosis

11. In a 50-year-old man, who was treated for gastric ulcer, digestion
normalized, pains disappeared, and mood is getting better. But in few
weeks pains in epigastrium, heartburn, and acid cructation develop
again. How can one characterize such course of disease?
A. Relapse
B. Remission
C. Terminal condition
D. Typical pathological process
E. Latent period

12. A man with the complex of deviation of his health is considered sick.
What is the more typically for disease?
A. Disorder of physiological regulation of functions
B. Decrease of ability to work
C. Disorder of immunity
D. Psychological deviations
E. Decrease of adaptation

ANSWERS:
1.-E, 2.-E, 3.-B, 4.-B, 5.-A, 6.-C, 7.-A, 8.-A, 9.-A, 10.-A, 11.-A, 12.-A.
Topic 2. Pathogenic action of environmental factors

1. A worker of radiological department was exposed to radiation once as


result of violation of the rules of safety appliances. Ulcerogangrenous
stomatitis developed in him in 8 days. Patient’s blood test showed: RBC
— 3,2 x 1012/L, reticulocytes — 0.01 %, Hb — 60 g/L, WBC - 2.3 x 109/L,
and platelets — 50 x 109/L. Which period of radiation sickness are
described changes typical for?
A. Period of primary reactions
B. Period of manifestation
C. Latent period
D. Pretended well-being period
E. Outcomes

2. Respiratory standstill developed ina man as a result of action of electric


current from town mains for 0.1 seconds with position of electrodes
“right hand — head”. Indicate the most probable reason for this
complication.
15
A. Paralysis of inspiration center
B. Emotional stress
C. Paralysis of respiratory muscles
D. Reflex respiratory standstill (pain shock)
E. Total paralysis of respiratory center

3. Increase of body temperature up to 38.50C, tachycardia, and BP of


160/85 mmHg were noticed in a worker after prolonged work in
conditions of “warming” microclimate. Disturbances of heart rhythm
were revealed in the patient at auscultation. Which electrolyte primary
loss affects adversely the heart work?
A. Potassium
B. Calcium
C. Magnesium
D. Sodium
E. Phosphate
4. Elevation of arterial pressure, tachycardia, and increase in body
temperature to 38,80C developed in a patient during narcosis with
myorelaxants. Which disturbance of heat exchange appeared in this
patient?
A. Fever
B. Physical hyperemia
C. Hit heat
D. Hypothermia
E. Hyperthermia

5. At violation of the rules of safety appliances an electric welder took


electric wire by unprotected and wet hand. Tonic contractions of
muscles and involuntary urination developed in him. Which effect of
electric current becomes apparent in this case?
A. Electrochemical
B. Biological
C. Heat
D. Mechanical
E. --

6. During long hypothermia of the organism phase of decompensation


developed. Its feature is:
A. Decrease in blood pressure
B. Constriction of peripheral vessels
C. Rare respiration
D. Increase in muscle tone
E. Rigor

7. A man took electric wire with high tension by both hands. He died
momentary in result of:
A. Intracerebral bleeding
B. Respiratory standstill
C. Cardiac fibrillation
D. Burns
E. Tearing extremities off

8. The main link in pathogenesis of kinetosis under radial acceleration is:


A. Excitation of motor nuclei of vagus nerve
B. Irritation of tactile receptors
C. Increase in muscular tone
D. Irritation of nuclei of vestibular nerve
E. Bradycardia

9. A worker, taking part in nuclear power plant breakdown elimination,


was exposed to whole-body irradiation. Which organ or which tissue
will tumor develop in?
A. Lung
B. Bone marrow
C. Stomach
D. Skin
E. Bone tissue
17
10. A worker, taking part in nuclear power plant breakdown
elimination, was exposed to ionizing radiation in dose of 6Gr. What
changes in the blood will happen after 10 days?
A. Agranulocytosis
B. Limpocytosis
C. Leucocytosis and limphocytopenia
D. Basophilia
E. Eosiniphilia

11. A man had electric trauma. Current went through the myocardium.
What dangerous disorders in heart work needing emergency correction
can appear in such situation?
A. Fibrillation of ventricles
B. Fibrillation of atria
C. Bradycardia
D. Tachycardia
E. Extrasystolia

12. A worker was exposed to ionizing radiation in dose of 5Gr after


accident at nuclear power plant. Agranulocytosis was revealed in him
in a week. What is the basic pathogenic mechanism in this case?
A. Increased outcome of mature leukocytes from bone marrow
B. Development of autoimmune processes
C. Increase in migration of granulocytes to tissues
D. Elevated destruction of leukocytes
E. Oppression of leucopoiesis

13. A patient has bone marrow form of radiation diseases. What


lineages of hemopoiesis are affected in this case?
A. All
B. Erythrocytic
C. Megakariocytic
D. Granulocytic
E. Agranulocytic
18
14. Symptoms of hemorrhage syndrome appeared in a patient with
radiation sickness. Which mechanism is leading one in pathogenesis of
this syndrome?
A. Thrombocytopenia
B. Lymphopenia
C. Erythropenia
D. Eosinopenia
E. Neutropenia

15. A doctor revealed in a patient affected by electric current


respiratory arrest and disturbances of heart functions. What pathway did
electric current go in this case?
A. Right hand - head
B. Left leg - right leg
C. Right leg - right hand
D. Left leg - right hand
E. Abdomen - right leg

16. Diver, working at depth of 60 m, was raised very quickly on


surface after an accident. After a time pain in the joints, skin itching
appeared in him. What happened to diver?
A. Decompression diseases
B. High altitude diseases
C. Hyperbaric oxygenation
D. Lowering of reactivity
E. Compensatory adaptation

17. A galvanizer addressed to a doctor because of appearing of pink


itching spots and vesicles on his arms skin. He worked a lot with nickel
compounds recently. What is the mechanism of these lesions
development?
A. Modification of skin proteins due to action of nickel ions
with immunopathological reaction development
B. Irritant action of nickel compounds on skin
19
C. Vegetative disorders related to nickel compounds received
by respiratory tract
D. Infection of skin due to microtraumas
E. Nickel is a complete antigen

18. In consequence of accident in laboratory man was exposed to


radiation in dose of 6Gr. Which symptom is the most typical for latent
period of acute radiation diseases?
A. Leukocytopenia
B. Headache
C. Depression
D. Vomiting
E. Diarrhea
19. A patient was admitted to the hospital with extensive burns of the
body surface. Which pathogenic factor of burn disease has to be
liquidated in the first turn?
A. Intoxication of decay products
B. Plasma loss
C. Pain
D. Infection through the injured skin
E. Autoallergy
20. During running parallel experiments rats were exposed to
prolonged direct solar radiation in open chambers and in ones covered
by glass. Tumor development at hair uncovered places of skin was
marked in animals residing in open chambers. Which factor influence
does this phenomena connected to?
A. Sun heat
B. Ultraviolet radiation
C. Biologic carcinogens
D. Exogenous chemical carcinogens
E. Infrared radiation

21. In consequence of accident at nuclear power plant radio-active


products escape occurred. Peoples, who were in zone of increased
110
radiation, got dose of radiation about 250 — 300 Gr. What symptom
would be the leading one in these people?
A. Lymphocytosis
B. Anemia
C. Thrombocytopenia
D. Neutropenia
E. Leukocytosis

22. A man was admitted to the hospital after exposure to radiation in


dose of 3Gr. Which physiological system develops changes in the first
turn in this patient?
A. Blood system
B. Cardiovascular system
C. Immune system
D. Alimentary system
E. Endocrine system

23. Headache, pain in ears, nasal hemorrhages appeared in the


passengers during flight by balloon. What serves the pathogenic base
for these changes development?
A. Explosive decompression syndrome
B. Decompression syndrome
C. Hypoxia
D. Gas embolism
E. Toxic influence of gases dissolved in the blood

24. What is the reason for pneumonia development in children after


cooling?
A. Microorganisms
B. General cooling
C. Decrease in reactivity due to previous diseases
D. Insufficiency of nutrition
E. All these factors
111
25, Which changes are connected to direct injurious action of ionizing
radiation?
A. Inhibition of DNA-synthesis under the influence of
radiotoxines
. Radiolysis of water
Edema
moa

. Interaction between free radicals and enzymes


. Injury of chromosomes

26. What reasons does high radiosensitivity of cells and tissues depend
on?
A. High mitotic activity
B. Activation of glycolysis
C. Level of differentiation
D. Number of mitochondrias
E. Decrease of DNA-synthesis

27. Which changes in blood are typical for the first period of bone
marrow form of acute radiation diseases?
A. Leukocytosis
B. Agranulocytosis
C. Thrombocytopenia
D. Erithrocytopenia
E. Leukocytopenia

28. Among many factors determining severity of electrical injury the


first place belongs to injury of internal organs localized at the way
which electric current passes through the body. Which organ injury is
the most dangerous?
A. Heart
B. Cerebrum
C. Lungs
D. Liver
E. Kidneys
112
29, A patient with III — IV degree burns of 50% of the body surface
was admitted to the hospital. He has depressed consciousness,
bradycardia, and reduced blood pressure. What is the most possible
diagnosis?
A. Burn shock
B. Intoxication
C. Preagony
D. Collapse
E. Coma

30. A patient very often has diseases of nasopharynx, which appear


under the influence of different factors and in the most cases manifest
by inflammation. Which of listed factors is the most probable reason
for these diseases?
A. Microorganisms
B. General cooling of organism
C. Overstrain
D. Immunodeficiency
E. Insufficiency of nutrition

31. A woman, who took part in liquidation of accident at Chernobyl]


nuclear power plant, was admitted to the hospital with complaints of
headache, pain in the heart, and worsening of general condition.
Changes of subcellular structures were found out in her, except:
A. Proteins
B. Nucleus
C. Lisosomes
D. Mitochondrion
E. Endoplasmic reticulum

32. A 45-year-old man suffering from hematoporphiria complains of


burns of skin, appeared after short-time (no more than 10 to 20 minutes)
exposure to the sun, headache, nausea, and dizziness. Explain the
reason for this pathology?
113
A. Photoallergy
B. Sun stroke
C. Heat stroke
D. Photochemical burn
E. Hyperthermia

33. A liquidator of accident at Chernobyl nuclear power plant after


carrying out works addressed to a doctor with complaints of abrupt
worsening of general state, significant weakness, increase in body
temperature up to 37,80C, multiple punctate hemorrhages on skin and
mucous membranes, diarrhea. In the patient’s blood analysis ESR — 25
mim/h, RBC — 2,4x1012/L, WBC - 2,2x109/L, platelets — 70x 100/L. What
period of radiation disease does this picture typical for?
A. Manifestation
B. First reaction
C. Pretended well-being period
D. Recovery
E. Outcome

ANSWERS:
L.-B, 2.-E, 3.-A, 4.-E, 5.-B, 6.-A, 7-C, 8.-D, 9.-B, 10.-A, 11.-A, 12.-E,
13.-A, 14.-A, 15.-A, 16.-A, 17.-A, 18.-A, 19.-C, 20.-B, 21.-D. 22.-A, 23.-
B, 24.-A, 25.-E, 26.-A, 27.-A, 28.-A, 29.-A, 30.-A, 31.-A, 32.-A,
33.-A.
114
Topic 3. The role of heredity and constitution
(somatotypes) in pathology

1. Three chromosomes of 21st pair were revealed in a baby by


karyotyping. Mark the phenotypic manifestation of this mutation.

A. Patau’s syndrome
B. Edward’s syndrome
C. Down’s syndrome
D. Turner’s syndrome
E. Polisomia Y-syndrome

2. Test of amniotic fluid for determination of sexual chromatin showed


that fetus’s cells include 2 bodies of sexual chromatin (Bar’s bodies).
Which disease was revealed in the fetus?
A. Klinefelter’s syndrome
B. Patau’s syndrome
C. Down’s syndrome
D. Edward;s syndrome
E. Trisomy X-syndrome

3. A 20-year-old man complains of headache, failing sight, muscular


weakness. His height is 2.00 m, weight is 80 kg, he has long
extremities, and kyphosis. On X-ray examination the skull film
showed enlargement of Turkish saddle. Sexual chromatin is absent.
What pathway is the most possible in appearance of gigantism in the
young man?
A. Excess of growth hormone
B. Somatoliberin insufficiency
C. Gonadoliberin insufficiency
D. Testosterone insufficiency
E. Chromosomal mutation
115
4. A woman addressed to the medical-genetic consultation. Short neck
with wing-shaped skin folds (“sphinks’s neck”), broad chest, and
mammary hypoplasia were revealed in her on examination. What is
the most possible diagnosis?
A. Patau’s syndrome
B. Syndrome of “cat’s cry”
C. Turner’s synrome
D. Morris’s syndrome
E. Klinefelter’s syndrome

5. A child being born in late marriage has short stature, mental


retardation, thick geographic tongue, narrow palpebral fissures, and
flat broad face. What disturbance ‘underlies development of described
syndrome?
A. Hereditary chromosomal pathology
B. Birth injury
C. Intrauterine intoxication
D. Intrauterine infection
E. Intrauterine immunoincompatible

6. Mother’s karyotype has 45 chromosomes. There was revealed that it


connects with translocation of the 21st chromosome to the 15th one.
Which disease more possible would develop in her child if the father’s
karyotype is normal?
A. Down’s syndrome
B. Patau’s syndrome
C. Klinefelter syndrome
D. Morris’s syndrome
E. Edward’s syndrome

7. A child with hereditary defects manifested characteristic syndrome


named “cat’s cry” just after the birth. At that newborns have
“meowing” timbre of their voice. In examination of the karyotype of
this child following was revealed:
116
A. Additional Y-chromosome
B. Additional 21st chromosome
C. Deletion of segment of 5th chromosome
D. Additional X-chromosome
E. Lack of X-chromosome

8. A woman addressed to the hospital with complaints of suppurating


incised wound. A doctor was cleaning the wound surface with 3%
solution of H2O2. But foam did not form. What does absence of
preparation activity connected with?
A. Superficial wound
B. Low concentration of H202
C. Hereditary insufficiency of katalase
D. Hereditary insufficiency of erythrocyte
phosphatdchydrogenese E. Presence of pus in the wound

9. Sexual chromatin was revealed at examination of the buccal


epithelium of a man. Which chromosomal disease is characterized by
this symptom?
A. Down’s disease
B. Trisomy X-chromosome
C. Turner’s syndrome
D. Hypophosphatemic rickets
E. Klinefelter’s syndrome

10. A 10-month old baby has fair hair, very white skin and blue eyes. His
parents have dark hair. He had normal appearance, but during last
three months impairment of cerebral circulation and mental
retardation developed in him. The reason for such condition can be:
A. Glycogenose
B. Acute porphyry
C. Hystidinemia
D. Lactosemia
E. Phenylketonuria
117
11. Daltonism (color blindness) was revealed in a 7-year-old boy at
examination. His parents are healthy. But his mother’s father had the
same anomaly. Which is the type of inheritance of this anomaly?
A. X-linked dominant
B. Autosomal dominant
C. X-linked recessive
D. Autosomal recessive
E. Partial dominance

12. Two types of cells with chromosomal set 46XY and 47XXY in the
equal quantity were revealed at examination of karyotype of the
patient.
What is the most possible diagnosis?
A. X-syndrome monosomia
B. Down’s syndrome C. Normal karyotype
D. Kleinefelter’s syndrome
E. Patau’s syndrome

13. A 14-year old girl has short stature, absence of pubescence, very short
neck, broad shoulders, and normal intelligence. What disease does the
girl have?
A. Klinefelter’s syndrome
B. Patau’s syndrome
C. Edward’s syndrome
D. Down’s syndrome
E. Turner’s syndrome

14. A 5-year old girl’s karyotype has 46 chromosomes. But one


chromosome of the 15th pair is longer than normal one, since it joins
chromosome from 21st pair. What type of mutation is present in the
girl?
A. Duplication
B. Deletion
C. Inversion
118
D. Shortage
E. Translocation

15. The father of a pregnant woman suffers from gemeralopia. This is an


X-linked recessive disorder. Her husband’s relatives didn’t suffer
from this disease. What is possibility that her child will suffer from
gemeralopia, if it is a boy?
A. 50%
B. 0%
C. 75%
D. 100%
E. 25%

16. There is regurgitation of milk to the nose in an infant during a meal.


Mark the possible reason for this disturbance?
A. Harelip
B. Fracture of basis of skull
C. -
D. Septal deformity
E. Cleft palate

17. Elementary discrete unit of inheritance in both sexual and asexual


reproduction is:
A. One chain of DNA molecule
B. One gene
C. One pair of nucleotides
D. Two chains of DNA molecule
E. One nucleotide

18. A 12-year-old boy has lower jaw carcinoma. What type of mutation
underlies the tumor development?
A. Non-lethal mutation of somatic cell
B. Lethal mutation of somatic cell
C. Non-lethal mutation of sexual cell mother or father
119
D. Non-lethal mutation of somatic cell in embryogenesis
E. Lethal mutation of somatic cell in embryogenesis

19. Child was born with hereditary pathology - cleft palate. What type of
mutation underlies this pathology?
A. Non-lethal mutation of somatic cell in embryogenesis
B. Non-lethal mutation of somatic cell
C. Lethal mutation of somatic cell
D. Non-lethal mutation of sexual cell mother or father
E. Lethal mutation of somatic cell in embryogenesis

20. A 10-year-old boy with mental retardation has typical appearance:


small head with chamfered nape, oblique slant of palpebral fissures,
lowbridged nose, and half-open mouth. A doctor is keeping him under
observation in connection with congenital heart disease. What
violation of karyotype causes the pathology in this boy?
A. Trisomy of 21 pare of chromosome
B. Trisomy of 13 pare of chromosome
C. Trisomy of 18 pare of chromosome
D. Trisomy X
E. Monosomy of X-chromosome
21. A 25-year-old man complains of sterility. He is tall; he has asthenic
body build and decrease in intelligence. Bar’s bodies were revealed in
biopsy of his buccal epithelium. What pathology it may be?
A. Klinefelter’s syndrome
B. Adipopse-genital dystrophy
C. Acromegaly
D. Adrenogenital syndrome
E. Eunohoidism
22. Few drops of 5% solution of trichloroacetic iron were added to urine
of a newborn. Color of urine did not change. Phenylketonuria with
irreversible brain injury was diagnosed in this child at the age of two.
Why was not phenylketonuria diagnosed in this newborn?
A. Analytic mistake was made
120
B. Wrong substrate was chosen
C. Wrong time was chosen
D. Reagent was prepared incorrectly
E. Wrong interpretation of results

23. Discharge of milk from nose was observed in a newborn during first
feeding. Which inborn defect of oral cavity can cause such condition?
A. This is normal occurrence
B. Absence of soft palate
C. Splitting of alveolar bone
D. Harelip
E. Cleft palate

24. A patient is short; he has mental retardation, epicanthic folds, and


short fingers. The trisomy of 21st pare of chromosome has been
revealed in his karyotype. How is this chromosomal anomaly referred
to?
A. Down’s syndrome
B. Klinefelter’s syndrome
C. Turner’s syndrome
D. Trisomy X-syndrome
E. Specific fetopathy

25. In the second half of pregnancy a woman has disorders of blood


circulation in placenta with violation of all its functions: respiratory,
trophic, protective, excretory, and hormonogenic. This has led to
development of secondary placental insufficiency syndrome. Which
pathologic process can develop in such conditions?
A. Gametopathy
B. Blastopathy
C. Embriopathy
D. Fetopathy
E. Galactosemia
121
26. A patient was exposed to ionizing radiation by accident. It has led to
development of deletion — a loss of chromosome segment. How is this
pathologic happening referred to?
A. Mutation
B. Exudation
C. Alteration
D. Enzymopathy
E. Reparation

27. Mutation, inversion of one chromosome, has developed in a patient as


a consequence of ionizing radiation influence. How is pathogenic
factor called, which evoked these pathologic changes?
A. Physical mutagen
B. Chemical mutagen
C. Biological mutagen
D. Carcinogen
E. Virus

28. A patient has mutation of gene which is responsible for hemoglobin


synthesis. It has lead to development of sickle-cell anemia. What kind
of pathological hemoglobin is found out this case?
A. HbA
B. Hb A;
C. Hb F
D.HbS
E. Bart-Hb

29. A patient has pathological processes resulted from X-linked gene


mutation. This disease is accompanied by deficiency of VIII factor of
blood coagulation and prolongation of blood coagulation time to 25
min.
What is this disease?
A. Glaucoma
B. Hemophilia
122
C. Hemeralopia
D. Daltonism
E. Galactosemia

30. A teenager has asthenic body build, gynecomastia, and testicular


atrophy. For verification of diagnosis he was sent to medical-genetic
consulting room. What karyotype will be found out in this case?
A. 47 XXY, one Bar’s body
B. 47 XXY, two Bar’s bodies
C. 46 XY, Bar’s bodies are absent
D. 45 XO, Bar’s bodies are absent
E. 47 XXX, two Bar’s body

31. Healthy parents gave birth to a girl with blue eyes and fair hair. During
the first months the girl developed irritability, anxiety, disturbances of
nutrition and sleep. Neurologist’s examination established
development delay in her. What genetic method has to be applied in
this case?
A. Biochemical
B. Cytological
C. Gemellary
D. Genealogical
E. Population-statistic

32. In which variant of karyotype in nucleus of somatic cells only one


Bar’s body can be found?
A. 47 XX, 15+
B. 45 XO
C. 45 XY
D.47 XY, 21+
E. 48 XXXY

33. A percentage of concordancy among monozygotic and dizygotic twins


at following diseases comprises relatively: tuberculosis — 66 and 23,
123
schizophrenia - 69 and 10, epilepsy — 56 and 10, harelip — 33 and 5,
endemic goiter — 71 and 70. Taking into account this information,
genotype is not important for formation of
A. Endemic goiter
B. Harelip
C. Epilepsy
D. Schizophrenia
E. Tuberculosis

34. A 32-year-old man has tall stature, high-pitched voice, gynecomastia,


adult woman pattern of hair distribution, mental retardation,
infertility. Klinefelter’s syndrome was diagnosed in him previously.
For diagnosis verification it is important to examine:
A. Karyotype
B. Rhesus-factor
C. Spermatogenesis
D. Blood group
E. Genealogy

35. 46 chromosomes were revealed at patient with signs of Down’s


syndrome. What kind of chromosomal anomalies have led to this
disease?
A. Translocation
B. Inversion
C. Deletion
D. Duplication
E. Circling

36. Father has astigmatism and mother is healthy. What is possibility that
their children will suffer from astigmatism?
A. 50%
B. 100%
C. 75%
D. 25%
124
E. 0%
37. A 3-year-old child has edematous skin, frequent inflammations with
marked exudations, and prolonged allergic reactions. Which type of
diathesis does this child have?
A. Exudative
B. Asthenic
C. Lymphohypoplastic diathesis
D. Gouty diathesis
E. Hemorrhage

38. Congenital diseases are diseases, which are:


A. Manifesting at birth of a child
B. All hereditary diseases
C. Hereditary dominant disorders
D. Result from pathologic labor E. Transplacental infectious
diseases

39. Which disorder is X-linked?


A. Hemophilia A
B. Klinefelter’s syndrome
C. Trisomy X-syndrome
D. Hypophosphatemic rickets
E. Short fingers

40. A patient has hernias, aortic aneurysm, long thin extremities, lens
ectopia. Which hereditary disease are these symptoms typical for?
A. Ehlers-Danlos’ syndrome
B. Marfan’s syndrome
C. Shegren’s syndrome
D. Mucopolysaccharidoses
E. Incomplete osteodgenesys

ANSWERS:
125
L.-C, 2.-E, 3.-A, 4.-C, 5.-A, 6.-A, 7.-C, 8.-C, 9.-E, 10.-E, 11.-C, 12.-D,
13.-E, 14.-E, 15.-A, 16.-E, 17.-B, 18.-A, 19.-A, 20.-A, 21-A, 22.-C, 23.E,
24.-A, 25.-D, 26.-A, 27.-A, 28.-D, 29.-B, 30.-A, 31.-A, 32.-A, 33.-A, 34.-
A, 35.-A, 36-A, 37.-A, 38.-A, 39.-A, 40.-A.
126
Topic 4. Role of reactivity in pathology.
Immunologic reactivity

1. There are no reactions of delayed type hypersensitivity in mice


without hairs (nude mice). The most possible reason for this pathology
is:
A. Defect of phagocytosis
B. Disorders of hematopoiesis
C. Absence of thymus
D. Deficiency of components of complement system
E. Absence of gamma globulins in blood

Transplantation of skin was performed in a patient with wide spread


burns. Graft swelled and changed its color at 8th day and was rejected
at 11th. What cells participate in this process?
A. Erythrocytes
B. Basophils
C. Eosinophils
D. B-lymphocytes
E. T-lymphocytes
Deficient content of immunoglobulins was revealed in a patient. What
cells of immune system produce immmunoglobulins?
A. Plasma cells
B. T-killers
C. B-lymphocytes
D. T-helpers
E. T-suppressors

Three times immunization of population was carried out by


pertussisdiphtheria-tetanus vaccine when the number of diphtheria
cases rose. The levels of anybodies increased in the blood of
immunized people as a consequence of immunization. What cells
produce these proteins?
A. Acidophilic granulocytes
127
B. Neutrophilic granulocytes
C. Macrophages
D. Monocytes
E. Plasmocytes

5. It is known that plasma cells produce specific antibodies against the


given antigen. Number of plasma cells increases after introduction of
antigen. What cells of peripheral blood serve as precursors of plasma
cells?
A. Neutrophils
B. B-lymphocytes
C. Basophils
D. T-lymphocytes
E. Eosinophils

6. Formation of T-helpers is held up in thymus. What processes of


immunogenesis in connective tissue will be violated at first?
A. Phagocytosis of antigens by macrophages
B. Opsonization
C. Conversion of B-lymphocytes to plasma cells
D. Phagocytosis of foreign entities
E. Formation of precursors of T-lymphocytes

7. Transplantation of donor heart was performed in a patient. What


conditions have to be observed to prevent transplant rejection?

A. Selection of donor according to HLA


B. Transplantation of bone marrow
C. Transfusion of donor’s blood
D. Removal of spleen
E. Administration of immunomodulators

8. A patient was operated for acute purulent appendicitis. He cannot be


discharged from the hospital for a long time because of bed healing of
128
post-operative wound. He has diabetes mellitus for many years,
repeated pyoderma, furunculosis, stomatitis, and gingivitis. What is
the reason for decreased immunologic reactivity?
A. Hyperglycemia
B. Hypercholesterolemia
C. Hyperketonemia
D. Hypohydration
E. Protein metabolism violation

It is known that reactivity of the organism is opposite to its resistance


during some pathological processes. What pathological process can
appear in such situation?
A. Fever
B. Shock
C. Inflammation
D. Posthemorrhage anemia
E. Arterial hypertension

10. Macrophages surrounded with alien erythrocytes were found at


microscopic examination of smear of exudation took from a rat
suffering from aseptic peritonitis with addition of bird erythrocytes.
What stage of phagocytosis does this occurrence correspond to?
A. Incomplete phagocytosis
B. Adhesion
C. Hemotaxis
D. Engulfment
E. Intracellular digestion

11. A 3-year-old boy suffering from chronic pneumonia has low indices
of B-lymphocyte system. Bruton’s hypogammaglobulinemia was
diagnosed in him. What is the consequence of B-lymphocyte system
deficiency?
A. Decreased resistance of organism to pyogenic coccus flora
B. Decreased resistance of organism to viruses
129
C. Decreased resistance of organism to fungous and tuberculosis
infection
D. Absence of graft rejection reactions
F. Increased risk of tumor development in organism

12. Considerable edema of lips appeared in a 25-year-old man in dentist’s


office few minutes later washing his mouth with solution of furacillin.
Which type of allergic reaction is observed in this case?
A. Immune complex-mediated
B. Delayed-type hypersensitivity
C. Anaphy lactic
D. Stimulating
E. Cytotoxic

13. Child has congenital heart disease, face defect, absence of thyroid
gland and thymus, and T-lymphocytes in blood. What hereditary
pathology are these symptoms connected with?
A. Di George syndrome
B. Luis-Barr syndrome
C. Turner’s syndrome
D. Down’s disease
F. Bruton’s disease

14. A patient with thyrotoxicosis has antithyroid antibodies in his blood.


Which type of allergic reactions is observed in this case?
A. Cytotoxic
B. Stimulating
C. Delayed-type hypersensitivity
D. Anaphylactic
E. Immune complex-mediated

15.A 2-month-old girl with pneumonia was admitted to pediatric


department. She bore otitis and pneumonia a month ago. Dicrease in
levels of gamma-globulins by 2 times was found at her examination.
130
Which disorder of immune system is it?
A. Luis-Barr syndrome
B. Physiological hypogammaglobulinemia
C. Late hypogammaglobulinemia
D. Swiss type of immune deficiency
E. Di George syndrome

16. A 2-year-old boy has frequent bacterial infections since 2 months.


Allergic reaction to tuberculin (IV type) is positive. Which
immunodeficiency is more possible at this patient?
A. Congenital B-lymphocytes insufficiency
B. Congenital T-lymphocytes insufficiency
C. Congenital T-suppressors defect
D. Congenital total immunodeficiency
E. Acquired immunodeficiency

17. Association of staphylococcus aureus and staphylococcus epidermidis


are found in numerous skin pustules at 25-year-old patient.
Pneumocysta carinia is found in his sputum analysis. Cryptosporidia,
proteus vulgaris and fungi of candida genus are found in his stool.
Which disease is accompanied by such multiple infections with
conditionally-pathogenic microorganisms?
A. Dysbacteriosis
B. AIDS
C. Sepsis
D. Diabetes mellitus
E. Drag agranulocytosis

18. A 25-year-old man complains of frequent inflammation diseases of


different localization. He is injection drug addict. HIV-test was
positive at him. What types of cells are damaged more substantially in
case of HIV?
A. Neutrophilic granulocytes
B. Plasmocytes
131
C. Macrofages
D. T-helpers
E. NK-cells

19. A 12-year-old boy with hereditary disease has eczema, 4 cases of


pneumonia in his anamnesis, bleedings. Low level of T-lymphocytes,
decreased levels of IgM, normal levels of IgA was found in his blood.
What disease does the boy have? A.
Wiskott-Aldrich syndrome
B. Luis-Barr syndrome
C. Bruton’s disease
D. Late hypogammaglobulinemia
E. Early hypogammaglobulinemia

20. A newborn has recurring respiratory tract diseases, pustule affections


of the skin, severe clinical course of herpes infection and recurring
candidiasis (thrush). Inoculation against smallpox and BCG-
vaccination was accompanied by severe complication in him. All
classes of immunoglobulins were decreased in his immunological test.
Delayedtype hypersensitivity reactions were absent in him. What cells
insufficiency underlies this immunodeficiency syndrome?
A. Neutrophiles
B. T- and B-lymphocytes
C. Macrophages
D. Fibroblasts
E. T-suppressors

21. Father has rhesus-positive erythrocytes and mother’s blood is


thesusnegative. Severe hemolytic anemia has developed just after
birth in their second child. What is the mechanism of massive
hemolysis at newborn’s organism?
A. Toxemia of pregnancy
B. Antenatal intoxication
C. Antenatal infection
132
D. Genetic deficit of erythrocytes
E. Immune hemolysis

22. A 1-year-old boy becomes very often ill with respiratory and pustular
skin diseases. Even little damages become complicated by long-time
purulent inflammation. All classes of immunoglobulins are nearly
absent from the child’s blood. Contents of all leukocyte types and
reactions of delayed-type hypersensitivity are normal. What cells
inherited decrease in functional activity underlies this syndrome?
A. T-lymphocytes
B. B-lymphocytes
C. Neutrophils
D. Eosinophils
E. Endoteliocytes

23. Decrease in levels of IgG and particularly IgA, IgM was found during
examination of immune status in 5-year-old boy. B-lymphocytes and
plasma cells are absent from his blood and lymph nodes. Reactions of
T-lymphocytes are normal. This is inherited sex-linked disease. What
diagnosis is more possible?
A. Bruton’s disease
B. Luis-Barr syndrome
C. Wiskott-Aldrich syndrome
D. Swiss type of immunodeficiency
E. Early hypogammaglobulinemia

24. The reasons for disturbances of phagocytosis may be qualitative


disorders of phagocytes. What changes of intracellular structures are
more typical for “lazy leucocytes” syndrome?
A. Chemotaxin and opsonin receptors
B. Microfilaments
C. Specific membrane glycoprotein (GP 110)
D. Microtubules
E. Bactericidal system
133
25. A 9-year-old girl was getting breast feeding during the first year of
life. She suffered prolonged severe pneumonia in the end of the first
year. She started walking late. Her gait was unsteady and her
movements were discoordinated. Telangiectasia appeared in her skin
and conjunctivas. IgA is absent from and levels of T-lymphocytes is
decreased in her blood. What immunodeficiency disease does the girl
have?
A. Luis-Barr syndrome
B. Wiskott-Aldrich syndrome
C. Chediak-Higashi syndrome
D. Swiss type of immunodeficiency
E. Di George syndrome
26. Immune system is depressed in a patient with HIV-infection. What
cells damage contributes more to immunodeficiency at this patient?
A. T-suppressors
B. Macrophages
C. T-helpers
D. B-lymphocytes
E. T-killers
27. A 2-year-old patient suffering from chronic pneumonia has decrease
in quantity of B-lymphocytes, IgM and IgG. What microorganisms
can cause development of this disease?
A. Candidas
B. Mycobacterium tuberculosis
C. Herpes virus
D. Adenovirus
E. Staphylococcus

28. Disorder of cellular immunity was revealed during investigation of


immune system in the patient with chronic fungus affection of the
skin. What indexes decrease more typically in this case?
A. IgG
B. IgE
C. T-lymphocytes
134
D. B-lymphocytes
E. Plasma cells

29. Mantoux test was made to sick child on suspicion of tuberculosis.


Swelling, hyperemia and painfulness occur in the allergen injection
place in 24 hours. What components determinate this reaction of
organism?
A. Mononuclear cells, T-lymphocytes and lymphokines
B. B-lymphocytes and IgM
C. Plasma cells, T-lymphocytes and lymphokines
D. Granulocytes, T-lymphocytes and IgG
E. Macrophages, B-lymphocytes and monocytes

30. A patient was suffering from chronic renal insufficiency. He has renal
transplantation from donor, who died in motorcar accident. Donor had
blood group same as recipient. Renal insufficiency occurs in patient
again after a time. Which system incompatibility takes part in
transplant rejection?
A. ABO
B. RH
C. MNS
D. HLA
E. Kel-Chelano
31. HIV-infection was suspected at a patient of dental clinics. What cells
are damaged at first in patient with AIDS?
A. T-helpers
B. T-suppressors
C. T-killers
D. B-lymphocytes
E. Macrophages
32. A 48-year-old patient suffers from frequent mycotic lesions and
pyoderma at surface of trunk for a long time. He has alcohol abuse in
his anamnesis. What promotes realization of disease reason in this
case?
135
A. Damage of the blood-brain barrier
B. Primary immunodeficiency
C. Decreased barrier function of the liver
D. Allergy
E. Chronic pneumonia

33. A 10-year-old girl suffers from viral and mycotic diseases very often.
She has congenital heart disease and insufficiency of thyroid gland.
Tlymphocytes is absent from her immunological test. What disorder
of immune system takes place in this case?
A. Bruton’s hypogammaglobulinemia
B. Mixed immunodeficiency
C. Terner’s syndrome
D. Hypoplasia of thymus
E. Inherited defect of complement system
34. Pneumonias and pyodermas very often occur in 5-year-old boy, as
well as in his grandfather. B-lymphocytes are absent from their
immunological test. What disorder of immune system takes place in
this case?
A. Bruton’s hypogammaglobulinemia
B. Hypoplasia of thymus
C. Mixed immunodeficiency
D. Terner’s syndrome
E. Inherited defect of complement system
35. Viral and bacterial infections, eczema often occur in a 12-year-old
boy. Decreased quantity of T-lymphocytes and IgM and normal levels
of IgG and IgA are found in him at examination. What kind of immune
pathology is observed in this case?
A. Hypoplasia of thymus
B. Mixed immunodeficiency
C. Bruton’s hypogammaglobulinemia
D. Terner’s syndrome
E. Inherited defect of complement system
136
36. A 20-year-old woman with intestinal polyposis has frequent mycotic
and viral diseases in her anamnesis. What link of immune system is
insufficient more possibly in this case?
A. T-lymphocytes
B. B-lymphocytes
C. Natural killers
D. Complement
E. Phagocytosis

37. Which of following is immunodeficiency disease?


A. Inherited disorder of immune system
B. Decrease of immune reactions under avitaminosis
C. Insufficiency of immune reactions after exposure to ionizing
radiation
D. Disorder of formation antibodies as consequences of protein
starvation
E. Decrease in immune reactions under cytostatic drugs action 38.
A Patient needs renal transplantation. What kind of
transplantation more expedient?
. Syngenic
MOoOaAWSYS

. Xenogenic
. Autogenic
. Allogenic
. Explantation

39, Heart transplantation is indicated for the patient. What kind of


transplantation more expedient?
A. Syngenic
B. Xenogenic
C. Autogenic
D. Allogenic
E. Explantation
40. The main role in pathogenesis of diseases, which occur in case of
Blymphocytes immunodeficiency, plays:
137
A. Immunoglobulin synthesis disorders
B. Cellular type immune reaction disorders
C. Loss of ability to graft rejection
D. Decrease in delayed-type hypersensitivity
E. Decrease in antitumor immunity

41. A newborn has convulsive syndrome and heart interventricular


septum defect. Thymic hypoplasia was revealed by X-ray examination of
his chest. What immunodeficiency may be supposed in this patient?
A. Di George syndrome
B. Bruton’s disease
C. Wiskott-Aldrich syndrome
D. Good’s syndrome
E. Luis-Barr syndrome
42. A 15-year-old patient was admitted to allergologic department
with bronchial asthma. What immunoglobulins excessive production can
result in main clinical symptoms development in this patient?
A. IgEB. IgA
C. IgG
D. IgM
E. IgD
ANSWERS: 1-C, 2-E, 3-A, 4-E, 5-B, 6-C, 7-A, 8-E, 9-B, 10-B, 11-A, 12-
C, 13-A, 14.-B, 15-B, 16-A, 17-B, 18-D, 19-A, 20-B, 21-E, 22-B, 23A,
24-A, 25-A, 26-C, 27-E, 28-C, 29-A, 30-D, 31-A, 32-C, 33-D. 34-A, 35-
B, 36-A, 37-A, 38-A, 39-D, 40-A, 41-A, 42-A.
Topic 5. Allergy

1. Pain in the heart and joints and pneumonia appeared in a patient


three weeks later acute myocardial infarction. What is the main
mechanism of development of post-infarction Dressler’s syndrome?
A. Ischemia of myocardium
B. Resorption of enzymes from necrotized area of myocardium
C. Secondary infection
D. Thrombosis of vessels
138
E. Autoimmune inflammation

2. A patient addressed to a dentist with complaints of redness and


edema of mucous membrane of his mouth a month later dental prosthesis.
Allergic stomatitis was diagnosed in this patient. What type of allergic
reaction by Gell and Cumbs underlies this disease?
A. Cytotoxic
B. Delayed-type hypersensitivity
C. Immune complex-mediated
D. Anaphylactic
E. Stimulating

3. Anaphylactic shock developed in a patient with botulism after


second injection of antitoxic antibotulinus serum mixture. What is the
main mechanism of anaphylaxis?
A. Interaction of T-lymphocytes with mediators
B. Interaction of antigen with IgM
C. Interaction of macrophages with antigens
D. Interaction of antigen with IgE
E. Interaction of T-lymphocytes with tissue basophils

4. In a 27-years-old man tuberculin test was carried out. Following


was observed 24 hours later: infiltration with size of 40x35 mm at the site
of injection and hyperemia of skin above it. What group of biologic active
substances causes development of allergic inflammation in this patient?
A. Lymphokines
B. Biogenic amines
C. Prostaglandins
D. Leukotriens
E. Kinins

5. Purulent endometritis developed in a woman after delivery.


Treating with antibiotics - inhibitors of murein synthesis was ineffective.
Wide spectrum bactericidal antibiotic was administered to her. 6 hours
139
later temperature rapidly increased up to 400C with shivering, pains in
muscles appeared, BP dropped down to 70/40 mmHg, and oligura
developed in this woman. What is the main reason for this condition
development?
A. Endotoxic shock
B. Toxic effect of preparation
C. Internal bleeding
D. Anaphylactic shock
E. Bacteremia

6. A 24-years-old patient has edema of face and increase in BP,


which appeared 1.5 weeks later severe streptococcus tonsillitis. The
patient has hematuria and proteinuria of 1.2 g/L. Anti-streptococcus
antibodies and decrease in content of compliment system components
were revealed in patient’s blood. Which microvessels do deposits of
immune complexes localize in and cause nephropathy?
A. Proximal tubules
B. Glomerule
C. Descendent tubules
D. Loop of Henle
E. Pyramids

7. Nausea, fatigue, stomachache, palpitation, difficult respiration,


and skin blisters developed in a patient 25 minutes later injection of
antibiotics. What stage of allergic reaction is observed in this patient?
A. Pathochemical
B. Biochemical
C. Pathophysiological
D. Immunological
E. Sensibilization

8. Skin tuberculin test was carried out in a patient with chronic lung
tuberculosis. Local hyperemia and edema appeared in the site of
inracutaneous introduction of tuberculin preparation within 24-48 hours.
140
What cells are primary effectors in mechanism of this reaction?
A. Neutrophils
B. T-lymphocytes
C. B-lymphocytes
D. Endotheliocytes
E. Smooth muscle cells of microvessels

9. Hyperemia, swelling and then necrosis of tissue, their rejection


and ulcer (Arthus phenomenon) develop at the rabbit in the place of
secondary intracutaneus injection of a substance with strongly
pronounced antigenic properties (for example horse serum). What factors
play the main role in pathogenesis of this phenomenon?
A. Antibodies presented by IgE
B. Antibodies presented by IgD
C. Antibodies presented by IgA
D. Antibodies presented by IgG and/or IgM
E. Specific T-lymphocytes-effectors
10. Skin rash, itching, swelling and pain in joints, increase in body
temperature, and proteinuria appeared in a patient in 5-8 days after use
lots of medical serum. Serum sickness was diagnosed. What is the main
factor in pathogenesis of this syndrome?
A. Primary systemic accumulation of circulating immune complexes in
the blood
B. Primary systemic degranulation of mast cells in the organism
C. Primary systemic activation of T-killers
D. Primary systemic activation of endoteliocytes
E. Primary systemic cytolysis of blood cells

11. Allergic diagnostic tests are used for the diagnosis of many
infectious diseases (tuberculosis, brucelosis, tularemia etc). Diagnosis is
confirmed if papula and redness appear in the place of the allergen
injection.
Antigens interaction reaction is conditioned by:
A. IgE and lymphokines
14]
B. IgM and macrophages
C. T-lymphocytes and lymphokines
D. IgE and T-lymphocytes
E. IgM and tissue basophiles

12. Catarrhal inflammation of bulbar conjunctiva and nose mucous


membrane develop in patient every year in spring and carly summer, when
trees and flowers are in blossom. Production of specific antibodies to
pollen underlies this syndrome. What cells activate and develop
exocytosis in this syndrome?
A. Neutrophils
B. Macrophages C. Lymphocytes
D. Mast cells
E. Throbocytes

13. An 18-year-old man with shoulder phlegmon got intramuscular


injection of penicillin. Tachycardia, thread-like pulse; decrease in BP
down to 80/60 mmHg occur after that. What kind of pharmacologic
reaction develops?
A. Potentiation
B. Reflex action
C. Central action
D. Anaphylaxis
E. Peripheral action
14. Patient has been wearing removable dental prosthesis for 7 days.
Soreness, swelling and hyperemia of prosthesis bed appear in the patient
after that. Inflammation subsides after prosthesis taking off and develops
again after prosthesis putting on. What process underlies the inflammation
development? A. Mechanical irritation
B. Hypersalivation
C. Hyposalivation
D. Resorption
E. Secretion
142
15. Thyrotoxicosis was diagnosed in a patient. Antithyroid antibodies
were found in his blood. Which type of allergic reaction is observed at
development of this disease?
A. Immune complex-mediated
B. Stimulating
C. Anaphylactic
D. Cytotoxic
E. Delayed type hypersensitivity

16. Hives, itching of the skin, swelling of the skin and mucous
membranes, swelling of lymphatic nodes develop in the patient in 9 days
after injection of medicinal serum. What disease develops?
A. Pollinosis
B. Serum sickness
C. Shwartzman’s phenomenon
D. Overy phenomenon
E. Quincke’s edema

17. Dressler’s syndrome was diagnosed at the patient 1.5 month later
myocardium infarction. It is characterized by pericarditis, pleurisy, and
pneumonia. What is the reason for this syndrome?
A. Sensitization of the organism by myocardium antigens
B. Decrease in resistance to microorganisms
C. Activation of saprophytic microflora
D. Intoxication of organism by products of necrosis
E. Release of myocardial enzymes to the blood

18. It is known that bronchial asthma develops by mechanism of


immediate hypersensitivity, which includes 3 sequential stages: A.
Immunological, pathochemical, pathophisiological
B. Pathochemical, pathophisiological, immunological
C. Pathochemical, immunological, pathophisiological
D. Pathophisiological, immunological, pathochemical
E. Pathophisiological, pathochemical, immunological
143
19. A 20-year-old man has injury of the right testicle. What danger
does it brings for the left (healthy) testicle?
A. Mimicry of antigens and development of antibody -mediated damage
B. Development of infectious process
C. Development of atrophy
D. Development of hypertrophy
E. No danger

20. Guinea-pig’s nephrocytotoxic serum was injected to the rabbit


under the experiment. What human disease is modeled in this case?

A. Nephrotic syndrome
B. Acute pyelonephritis
C. Chronic renal insufficiency
D. Acute diffuse glomerulonephritis
E. Chronic pyelonephritis

21. Acute glomerulonephritis appeared in the patient 2 weeks later


purulent tonsillitis. Antibodies against microorganism antigens were
found at the patient. Which microorganism are these antibodies against?
A. Hemolytic streptococcus
B. Staphylococcus
C. Pneumococcus
D. Mycobacterium tuberculosis
E. Meningococcus

22. Severe edema of soft tissues of upper and lower jaws, rash on the
skin of face, redness, and itching appear in the patient in response to using
anesthetic drag at tooth extraction. Which pathological process underlies
the reaction to anesthetic?
A. Inflammation
B. Drag toxic action
C. Allergy
D. Insufficiency of blood circulation
144
E. Disorder of lymph outflow

23. | Novocain was injected by dentist for anesthesia at tooth extraction.


Symptoms of anaphylactic shock appeared at the patient few minutes
later. Patient has drop of BP, tachypnea, loss of consciousness and
convulsions. What type of reaction is it? A. Immediate type
hypersensitivity
B. Cytolytic or cytotoxic reactions
C. Arthus phenomenon reactions
D. Delayed type hypersensitivity
E. Stimulating allergic reaction

24. Man with the caries is subjected to constant sensitization by


streptococcus antigen. What disease can appear due to this etiological
factor?
A. Glomerulonephritis
B. Pancreatitis
C. Myocarditis
D. Pulpits
E. Periodontitis
25. Antitoxic diphtheria serum was introduced to a child suffering
from diphtheria. Skin eruption accompanied by itching, increase in body
temperature to 380C, and pain in joints occurred in patient 10 days later.
What is the reason for these symptoms?
A. Contact allergy
B. Serum sickness
C. Atopy
D. Anaphylactic reaction
E. Delayed type hypersensitivity

26. Tuberculin was injected intraperitoneally to animal, which was


sensitized by it. Venous hyperemia and peritoneal edema were revealed
24 hours later in laparotomy. High quantity of lymphocytes and
145
monocytes were found in peritoneum smears. What pathological process
does the animal have?
A. Fibrinous inflammation
B. Allergic inflammation
C. Aseptic inflammation
D. Serous inflammation
E. Purulent inflammation

27. A dentist injected ultracain to a patient before tooth extraction for


the purpose of anesthesia. Sensitivity test was not made. Anaphylactic
shook developed in the patient in few minutes after drag injection. What
cells produce reagins, which take part in development of anaphylactic
reaction?
A. Plasma cells
B. B-lymphocytes
C. T-lymphocytes
D. Mast cells
E. Eozinophiles
28. Tooth was extracted in a teenager under Novocain anesthesia.
Paleness of skin, dyspnea and hypotension occurred in the patient 10
minutes later. What type of allergic reaction is it?
A. Anaphylactic
B. Cytotoxic
C. Arthus phenomenon type
D. Delayed type hypersensitivity
E. Stimulating

29, Tooth was extracted in a teenager under Novocain anesthesia.


Paleness of skin, dyspnea and hypotension occurred in the patient 10
minutes later. What substance does allergen react with on the surface of
mast cells?
A. IgE
B. T-lymphocytes
C. IgA
146
D. IgD
E. IgM
30. A 27-year-old woman instilled drops with penicillin to the eyes.
Itching and burning pain of skin, edema of lips and eyelids, cough with
whistling, decreasing in BP occur in her some minutes later. What
immunoglobulin participates in development of these allergic reactions?

A. IgE
B. IgG and IgM
C. IgA
D. IgM
E. IgG

31. A 43-year-old woman is suffering from pneumonia. She began


complaining of weakness, face and arms burning pain in 10 minutes after
ampicillinum injection. Cough, dyspnea, pain in the chest developed in
her. At clinical examination patient has cyanosis, eyelids swelling, face
red rashes, heart rate — 120 per minute, BP — 120 mmHg, muffled heart
sounds, hypopnoe and tachypnea, respiration with different moist rales.
What is the reason for worsening of patient’s condition?
A. Anaphylactic shook
B. Urticaria
C. Quincke’s edema
D. Attack of asthma
E. Pulmonary thromboembolism

32. 0.1 ml of horse serum was injected to the guinea-pig for


sensitization. What are external signs of sensitization?
A. No external signs
B. Skin rashes
C. Joints swelling
D. Increase in body temperature
E. Pain
147
33. A 15-year-old girl is suffering from bronchial asthma. Severe
attack of expiratory dyspnea develops in her during the spring blossoming
period. What biological active substance causes spasm of bronchial
smooth muscle in this case?
A. Leukotriene
B. Thromboxane A2
C. Prostacyclin
D. Bradykinin
E. Serotonin

34. Patient addressed to a doctor with complaints of headache, rhinitis,


weakness, and increase in body temperature developing every spring in
blossoming period. What type of allergic reaction by Gell and Cumbs
underlies this disease?
A. Anaphylactic
B. Cytotoxic
C. Antibody-dependent cell-mediated cytotoxicity
D. Immune complex-mediated
E. Delayed type hypersensitivity

35. Weakness, itching of skin, acute spasmodic pain in the abdomen,


hyperemia and rash on skin, tachycardia, and decrease in BP down to
70/40 mm Hg suddenly developed ina patient few minutes later Novocain
injection by the dentist. Which type of allergic reactions does this
pathology belong to?
A. Anaphylactic
B. Cytotoxic
C. Stimulating
D. Cell-mediated
E. Immune complex-mediated

36. Contact dermatitis of upper extremities developed in the nurse,


who has been working in manipulation room for 20 years. Which type of
allergic reactions does this pathology belong to?
148
A. Delayed-type hypersensitivity
B. Primary immunodeficiency
C. Immediate type hypersensitivity
D. B-cells immunodeficiency
E. T-cells immunodeficiency

37. Pain in the joints and loins, hemorrhagic eruptions on the skin, and
increase in body temperature occur in the patient with toxemic stage of
burn disease in 2 hours after allogenic plasma transfusion. What allergic
reaction takes place in this case?
A. Serum sickness
B. Urticaria
C. Quincke’s edema
D. Anaphylactic
E, Autoimmune vasculitis
38. Hyperergic inflammation form of upper respiratory tract (larynx,
trachea, bronchi) develops at 6-year-old child. Threat of respiratory
impairment develops and then necessity of using anti-inflammatory
hormones occurs. Which hormone has anti-inflammatory property?
A. Cortisol
B. Adrenaline
C. Growth hormone
D. Testosterone
E. Insulin

39. Swelling, increased BP, proteinuria, hematuria, decreased urine


excretion were revealed in the patient, which suffered severe tonsillitis
before. These symptoms are typical of acute glomerulonephritis, resulting
from damage of glomerular basement membrane. What is mechanism of
this disease?
A. Anaphylactic allergic reactions
B. Cytotoxic allergic reactions
C. Immune complex-mediated allergic reaction
D. Delayed-type hypersensitivity
149
E. Stimulating allergic reaction

40. An 18-year-old patient has insulin-dependent diabetes mellitus I


type. What type of allergic reactions underlies beta cells damage?
A. II type, cytotoxic
B. I type, anaphylactic
C. III type, immune complex-mediated
D. IV type, delayed-type hypersensitivity
E. Pseudoallergic reaction

ANSWERS: 1-E, 2-B, 3-D, 4-A, 5-A, 6-B, 7-C, 8-B, 9-D, 10-A, 11-C,
12-D, 13-D, 14-D, 15-B, 16-B, 17-A, 18-A, 19-A, 20-D, 21-A, 22-C, 23A,
24-A, 25-B, 26-B, 27-A, 28-A, 29-A, 30-A, 31-A, 32-A, 33-A, 34-A, 35-
A, 36-A, 37-A, 38-A, 39-C, 40-A.
151
Topic 6. Disorders of microcirculation

1. A patient was admitted to the hospital with diagnosis of acute leftventricle heart failure.
Patient’s condition suddenly became worse and edema of lungs developed in him. What kind
of disorders of peripheral blood circulation causes the lungs edema?
A. Arterial hyperemia neurotonic type
B. Arterial hyperemia neuroparalytic type
C. Arterial hyperemia metabolic type
D. Ischemia
E. Congestion (venous hyperemia)

2. Dependence of blood pressure upon level of peripheral vascular resistance was measured in
animal under experimental conditions. Indicate vessels, which have the most resistance.
A. Capillaries
B. Arteries
C. Aorta
D. Arterioles
E. Veins

3. A patient has obstruction of right calf profound veins, which results in increasing in amount
of blood in the impaired part of tissue. What is the name for increase in amount of blood
resulted from impeded blood outflow?
A. Venous hyperemia (congestion)
B. Thrombosis
C. Ischemia
D. Stasis
E. Arterial (active) hyperemia

4. A patient with paradontitis has edema of gums. His gums are dark red.
What kind of local blood flow disturbance takes place in patient’s gums?
A. Venous (passive) hyperemia
B. Arterial (active) hyperemia
C. Embolism
D. Thrombosis
E. Ischemia

5. Pain in leg at walking, cyanosis, and edema of calf appeared in a patient with varicosity. His
foot is cold. What kind of disturbances of regional blood flow appeared in this patient?
A. Angiospastic ischemia
B. Ischemic stasis
C. Compressive ischemia
D. Venous (passive) hyperemia
E. Obstructive ischemia

Edema and cyanosis of low extremities appear in a food shop assistant at the end of workday.
What is the main factor of edema development in this patient?
A. Dilatation of resistant vessels
B. Orthostatic increase of venous pressure
C. Increase of number of functional capillaries
D. Increase of collateral blood flow
E. Increase of tissue drainage

Redness and increase in volume of affected place of tissue, and increase in local temperature
are observed in a patient with burn of thigh. What pathologic process do indicated symptoms
correspond to?
A. Arterial hyperemia
B. Venous hyperemia
C. Thrombosis
D. Ischemia
E. Stasis
A 23-year-old woman had intense psycho-emotional excitement. Hyperemia of face skin,
tachycardia and increase in BP were observed in her. What is the mechanism of redness of
her face skin?
A. Congestive venous hyperemia
B. Neuroparalytic arterial hyperemia
C. Post-ischemic arterial hyperemia
D. Neurotonic arterial hyperemia
E. Stasis
Dyspnea, sharp pain in the chest, cyanosis, and jugular venous distention suddenly develop in
a patient with thrombophlebitis of lower extremities. What is the most possible disorder of
blood circulation developed in the patient?
A. Thromboembolism of coronary vessels
B. Thromboembolism of mesenteric vessels
C. Thromboembolism of cerebral vessels
D. Thromboembolism of portal vein
E. Thromboembolism of pulmonary artery
10. Burning in the mouth appears after taking of food with pungent dressings. At examination
mucosa of oral cavity is edematous, hyperemic, and bright red. What major reasons underlie
disorders of microcirculation?
A. Congestion
B. Neuroparalitic arterial hyperemia
C. Postischemic arterial hyperemia
D. Atonic hyperemia
E. Degranulation of tissue basophiles
11. A woman experienced a strong psycho-emotional excitement during dentist’s reception.
Redness of her face skin and profuse salivation were noticed at that time. What is the
mechanism of these phenomena?
A. Manifestation of sympathetonic effect
B. Stagnant venous hyperemia
C. Neuroparalitic arterial hyperemia
D. Postischemic arterial hyperemia
E. Neurotonic arterial hyperemia

12. Patient’s arm was put in plaster cast on account of simple fracture of humeral bone. Swelling,
cyanosis, and decrease in temperature of the traumatized arm appear next day. What disorder
of peripheral blood flow do these symptoms testify to?
A. Thrombosis
B. Venous hyperemia
C. Ischemia
D. Embolism
E. Arterial hyperemia

13. Injury of sympathetic fibers of the sciatic nerve developed in a patient due to trauma. What
kind of peripheral blood flow disorder takes place in the patient?
A. Neurotonic arterial hyperemia
B. Venous hyperemia
C. Angiospastic ischemia
D. Neuroparalitic arterial hyperemia
E. Obturative ischemia

14. A 42-year-old woman, shop assistant by profession, complains of edema of the lower
extremities at the end of a workday. Her legs are cyanotic, their temperature is decreased.
Venous hyperemia of lower extremities was established, which is due to constitutional
weakness of the elastic apparatus of veins and the occupation. What is the major pathogenic
factor causing local changes in venous hyperemia?
A. Disorders of metabolism
B. Hypoxia
C. Atrophy
D. Dystrophy
E. Sclerosis
15. Puncture of abdominal cavity for the extraction of fluid was performed to a 45-year-old patient
with diagnosis “cirrhosis of liver, ascites”. State of unconsciousness suddenly developed in
the patient as a result of decrease in blood pressure after extraction of 5L of fluid. That was
considered as a manifestation of brain blood circulation insufficiency. Which disorder of
microcirculation occurs in this case?
A. Ischemia
B. Arterial hyperemia
C. Venous hyperemia
D. Thrombosis
E. Embolism

16. Signs of disorders of microcirculation in form of venous hyperemia were observed in a patient
with gingivitis. This condition manifested by cyanosis and edema of mucosa and decrease in
local temperature. What is the major mechanism of congestion development?
A. Blood factors
B. Perivascular changes
C. Disturbances of lymph outflow
D. Exudation from vessels
E. Increased permeability of vessel wall

17. C. Bernard observed increase in secretion of submandibular salivary gland and development
of arterial hyperemia irritating chorda tympani (branch of nervus facialis) in an experiment.
What sort of arterial hyperemia it is according to mechanism of its development?
A. Neuroparalytic
B. Neurotonic
C. Metabolic
D. Reactive
E. Working
18. Thrombosis of coronary artery results in myocardial infarction.
Which mechanisms will be dominating ones in this disease?
A. Electrolytic-osmotic
B. Acidosis
C. Protein-dependent
D. Lipid-dependent
E. Calcium-dependent
19. One of the most dangerous points in myocardial infarction pathogenesis is enlargement of the
zone of necrosis, dystrophy, and ischemia. Increase in myocardial oxygen consumption plays
important role in development of indicated processes. What substances contribute to this
process?
A. Chlorine ion
B. Cholesterol
C. Catecholamines
D. Acetylcholine
E. Adenosine
20. High level of arterial blood pressure is observed in a patient with renal disease accompanied
by ischemia of renal parenchyma. What is the leading factor of increase in arterial blood
pressure in this patient?
A. Excess of angiotensin I
B. Excess of antidiuretic hormone
C. Augmentation of cardiac output
D. Increase in tonus of sympathetic nervous system
E. Hypercatecholaminemia

21. Patient has acute retrosternal pain irradiating into left arm, which cannot be controlled by the
nitroglycerine for 30 minutes. What kind of changes develop in the patient’s heart?
A. Myocardial ischemia
B. Pathological hypertrophy of myocardium
C. Sharp increase in coronary blood flow
D. Mitral valve insufficiency
E. Inflammation of pericardium
22. Severe stress was induced in an experimental animal. Under this condition, necrotic changes
of myocardium developed in the animal. What is the leading reason for pathogenesis of this
injury? A. Increase in calcium content in cardiomyocytes
B. Decrease in mitochondrial ATP synthesis
C. Affection of Na-K pump functions
D. Coronary blood flow insufficiency
E. Increase in myosin ATP-activity

23. After fast surgical removing of coronary artery occlusion in a patient with ischemic heart
disease, secondary injury of myocardium develop (reperfusion syndrome) characterized by
necrobiotic changes in the focus of previous ischemia. This complication results from:
A. Accumulation of hydrogen ions
B. Deficiency of potassium ions
C. Deficiency of adenosine triphosphate
D. Excessive accumulation of calcium ions
E. Deficiency of creatine phosphate

24. Phlebothrombosis of the profound veins of lower extremities with subsequent pulmonary
thromboembolism develop in a patient operated for chronic ulcer of duodenum at the
postoperative period. Which functional disorder of those that develop in this case is the most
important?
A. Abrupt drop of arterial blood pressure
B. Increase in central venous pressure
C. Increase in pressure in pulmonary artery
D. Acute right ventricular failure
E. Changes of ventilation-perfusion ratio in the lungs
25. A 57-year-old man complains of heart pain that has developed after prolonged negative
emotions. An emergency doctor diagnosed ischemic heart disease manifesting by stenocardia.
What kind of ischemia is the most probable?
A. Compressive
B. Obliterative
C. Angiospastic resulting from deficiency of vasodilators
D. --
E. Obturative

26. Condition of a patient with thrombophlebitis of lower extremities suddenly became worse.
Weakness, giddiness, dyspnea, pain in the chest, and cyanosis developed in him. It was
established that pulmonary thromboembolism results in acute cor pulmonale. What is the
leading link in forming of this syndrome?
A. Increased blood pressure in pulmonary artery
B. Increase in central venous pressure
C. Abrupt arterial blood pressure drop in systemic circulation
D. Weakening of heart function resulting from myocardial hypoxia
E. Disorders of external respiration

27. Atophan, which leads to vessel sclerosis, was introduced to gastric arteries of an experimental
animal with purpose of gastric ulcer modeling. Which mechanism of gastric mucosa injury is
the leading one in this experiment?
A. Hypoxic
B. Neurodystrophic
C. Mechanical
D. Dysregulative
E. Nerohumoral

28. The theory exists that atherosclerosis plays an important role in periodontitis development,
affecting vessels of gums. What kind of local blood circulation disorders develops under
atherosclerosis of vessels?
A. Active hyperemia
B. Passive hyperemia
C. Embolism
D. Ischemia
E. Disorders of lymph outflow

29. Instantaneous death of pilots occurs under depressurization of airplane cabin at the altitude of
19 km. What is the reason for it?
A. Multiple gas embolisms
B. Hemorrhage to the brain
C. Gas embolism of cerebral vessels
D. Bleeding
E. Paralysis of respiratory center
30. A patient has acute pain in his chest, dyspnea, tachycardia, cyanosis, and decreased BP.
Pulmonary infarction was diagnosed in this patient.
Which factor is the most common cause of pulmonary infarction? A. Embolism by thrombus
from veins of lower extremities
B. Congestion in the pulmonary circulation
C. Increase in number of platelets
D. Activation of fibrinolytic system
E. Pneumothorax

31. A 50-year-old patient suffers from endarteritis. Which of the following symptoms is the most
characteristic one for this pathology?
A. Decreased temperature of lower extremities
B. Cyanosis
C. Increased temperature of lower extremities
D. Edema
E. Enlargement of volume of extremities

32. Pulmonary embolism developed in a patient with coxal bone fracture. What kind of embolism
does this patient have?
A. Fat
B. Thromboembolism
C. Tissue
D. Gas
E. Air
33. Gas embolism developed ina diver who was lifted to the surface very fast. It results from fast
change:
A. From increased atmospheric pressure to normal
B. From normal atmospheric pressure to increased
C. From normal atmospheric pressure to decreased
D. From decreased atmospheric pressure to normal
E. --

34. Acute heart arrest develops in a patient with thrombophlebitis of calf profound veins. What is
its reason?
A. Pulmonary thromboembolism
B. Left ventricle hypertrophy
C. Myocardial dystrophy
D. Endocarditis of mitral valve
E. Atherosclerosis

35. Convulsions and loss of consciousness develop in a diver during the emergency raising from
the depth. What is the major pathogenic mechanism in these disorders development?
A. Gas embolism
B. Hypoxia
C. Toxic action of oxygen
D. Toxic action of nitrogen
E. Hypercapnia

36. Destruction of endothelial layer of vessel leads to vasoconstriction. Decreased secretion of


endothelial-derived factor plays the leading role in this phenomena development. What is this
factor?
A. Nitric oxide
B. Adenosine
C. Histamine
D. Bradykinin
E. Adenosine monophosphate
37. What changes develop in human organism in case of depressurization of airplane cabin at
altitude of 19 km?
A. Gas embolism
B. Compensatory augmentation of biologic oxidation in tissues
C. Increase in body temperature
D. Compensatory augmentation of heart work
E. Disorders of muscular coordination

ANSWERS:
1-E, 2-D, 3-A, 4-A, 5-D, 6-B, 7-A, 8-D, 9-E, 10-E, 11-E, 12-B, 13-D, 14-B, 15-A, 16-E, 17-B,
18-E, 19-C, 20-A, 21-A, 22-A, 23-D, 24-D, 25C, 26-A, 27-A, 28-D, 29-A, 30-A, 31-A, 32-A, 33-
A, 34-A, 35-A, 36-A, 37-A.

Topic 7. Hypoxia

. Aman constantly lives in mountains. What changes of blood test indices can found in him?
A. Increased red blood cells number
B. Decreased reticulocytes count
C, Decreased color index
D. Appearance of erythroblasts in blood
E. Decreased hemoglobin content

2. After autopsy a forensic medical expert established that death of 20year-old woman resulted
from cyanide poisoning. What process disorder caused the death?
A. Oxygen transport by hemoglobin
B. Tissue respiration
C, Hemoglobin synthesis
D. Urea synthesis
E, Oxidative phosphorilation

. Increased number of erythrocytes in blood test was revealed in people residing in village,
which is situated in mountains at the altitude of 3000 m. What is the reason for changes in their
blood?
A, Increased vitamin B;2 synthesis
B. Blood clotting
C, Change of spleen function
D. Increased erythropoietin production
E. Increased circulating blood volume

. The group of patients from sanatorium went to the mountains for excursion. Tachycardia and
breathlessness developed in part of them in two hours after beginning of excursion. Which
type of hypoxia do those disturbances result from?
A. Hemic
B. Hypoxic
C, Respiratory
D. Tissue
E. Circulatory

. A driver slept in garage in a car with working engine. When he woke he had headache and
later vomiting began in him. What compound formation in blood caused this state?
A. Carbhemoglobin
B. Methemoglobin
C, Carboxyhemoglobin
D. Desoxihemoglobin
E. Oxihemoglobin

. Which sportsman will have highest content of lactic acid in the blood?
A. Weight-lifter
B. Who just had run 100 m
C, Who just had run 5000 m
D. Who put the shot
E. Jumper with pole

. A 40-years-old man took cyanic potassium by mistake. He died instantly. What mitochondrial
enzymes do cyanides block?
A. NAD-dependent Dehydrogenases
B. Cytochrome Oxydase
C, Cytochrome B
D. Cytochrome C
E. FAD-dependent enzymes

8. Pain in muscles appears after physical exertion in people, who move a little (have
hypodynamia) for a long time. What is the possible reason for this?
A, Decreased content of lipids in muscles
B. Increased content of ATP in muscles
C, Accumulation of creatinin in muscles
D. Accumulation of lactic acid in muscles
E. Enhanced decay of muscle proteins

9. The number of erythrocytes in alpinist’s blood before going to mountains is 4,5x1012/L. What
changes of erythrocytes number can develop at the altitude of 2500 m above sea level?
A, Absolute erythropenia
B. Absolute erythrocytosis
C, Relative erythropenia
D. Relative erythrocytosis
E. There won’t be any changes

10. Dyspnea, increased heart beat rate, weaken of attention, foolishness occurred in sportsmen
without acclimatization during the ascent to the altitude of 3000 m. Then weakness and
disturbance of muscular coordination occurred. What was the reason for that condition?
A, Circulatory hypoxia
B. Hypoxic hypoxia
C, Respiratory hypoxia
D. Hemic hypoxia
E. Hypercapnia

11. Environmental pollution by nitric compounds occurs after accident at a chemical factory.
People, who live at this region, have sharp weakness, headache, breathlessness, and giddiness.
What does hypoxia result from?
A. Cytochrome Oxidase inactivation
B. Dehydrogenases suppression
D. Carboxyhemoglobin formation
E. Decreased function of FAD-dependent enzymes
C, Methemoglobin formation

12. Total respiratory insufficiency was noticed on examination of blood gases partial pressure in
a patient with bronchial asthma. What is the reason of respiratory hypoxia in this case?
A, Increased perfusion
B. Decreased diffusion
C, Increased ventilation
D. Decreased ventilation
E. Increased diffusion

13. Total respiratory insufficiency developed in a patient with ling disease. It manifested by
decreased pOz and increased pCO: in patient’s blood. What caused development of respiratory
hypoxia and pronounced respiratory insufficiency?
A. Excessive hyperventilation
B. Frequent shallow respiration
C, Oxygen deficit in inspired air
D. Uneven ventilation of the lungs
E. Functional blood shunting in the lungs

14. Disorders of oxyhemoglobin formation have led to development of hemic hypoxia. What are
the reasons for hemic hypoxia development?
A. Methemoglobin formation
B. Inhibition of dehydrogenases
C, Pronounced polycytemia
D. Activation of Glutation Peroxidase
E. Deficiency of nboflavinum

15. Which factors increase hypoxic injury of a cell under hypoxic conditions?
A. Chinon derivations
B. Inhibitors of proteolysis
C, Glucocorticoids
D. Catecholamines
E. Glutation-peroxidase

16. Symptoms of poisoning such as sharp weakness, dizziness, headache, vomiting, dyspnea,
tachycardia, cyanosis of lips, ears, distal phalanges of fingers occurred in one child from group
of children, who had eaten sweet juicy watermelon. Laboratory test of watermelon showed
high level of nitrates. What is leading mechanism in pathogenesis of only one child poisoning?
A. Superoxide dismutase deficiency
B. Cytochrome oxidase blockade
C,. Glutathione peroxidase deficiency
D. Methemoglobin reductase deficiency
E. Catalase deficiency

17. A man has been living high in mountains for a long time. What changes in his blood would
develop?
A, Increase in number of leukocytes
B. Increase in diameter of blood vessels
C, Decrease in number of leukocytes
D. Increase in quantity of hemoglobin
E. Rare pulse

18. Tachypnea and hypopnoe developed in tourists, which had climbed to the altitude of 3000 m.
These changes are consequence of stimulation of
A. Chemoreceptors of carotid sinus
B. Mechanoreceptors of pulmonary alveoli
C, Baroreceptors of arch of aorta
D. Neurons of the cerebral cortex
E. Motoncurons of spinal cord

19. Cyanosis, swelling of lower extremities, dyspnea occurred in a 60year-old patient suffering
from heart failure. Long-term adaptation of organism to hypoxia, occurring in that condition,
can be supplied by
A. Blood outlet from the depots
B. Increased hemopoietic activity
C, Opening of non-functioning capillaries
D. Tachycardia
E. Increased blood oxygenation

20. A dog has poisoning by unknown substance that caused its immediate death because of
oxidation of cytochromes. What is that substance?
A. Potassium chloride
B. Potassium cyanide
C, Potassium sulfate
D. Potassium orotate
E. Potassium permanganate

21. Gas alkalosis developed in a group of alpinists due to rise to the Everest’s top. So, carbon
dioxide partial pressure in their arterial blood makes up
A, 40 mmHg
B. 50 mmHg
C. 30 mmHg
D. 60 mmHg
E. 70 mmHg

22. Hemic hypoxia occurred in a patient. It is accompanied by methemoglobin accumulation and


development of metabolic acidosis. Which substance accumulation leads to disturbance of
acid-base balance?
A, Sulfates
B. Lactates
C, Phosphates
D. Hydrocarbonates
E. Hydroxybutirates

23. During keeping salvage operations, several miners were taken out from the mine. They were
unconscious but without any visible damages. Air in mine had considerable amounts of
methane. Which type of hypoxia developed in those people?
A. Hypoxic
B. Hemic
C, Circulatory
D. Tissue
E. Respiratory

24, A 29-year-old patient with carbon monoxide poisoning was admitted to the hospital. He had
signs of severe hypoxia: pronounced dyspnea, cyanosis and tachycardia. Which changes of
hemoglobin take place during carbon monoxide poisoning?
A. Methemoglobin formation
B. Carboxyhemoglobin formation
C, Carbhemoglobin formation
D. Sulfhemoglobin formation
E. Oxyhemoglobin inactivation

25. A 54-year-old patient was admitted to hospital in hypoxic state, developed due to aspiration of
vomit mass. Blood test was taken after the treatment of patient. Does erythrocytes number in
peripheral blood change in the initial phase of hypoxia?
A. Increase due to outlet of the blood from depots
B. No change in initial phase
C, Decrease due to hemolysis of erythrocytes
D. Decrease due to depot of blood
E. Increase due to increase of hemopoiesis

26. A patient had course of dosing hypoxic trainings as a part of treatment of hypertensive disease.
The most considerable cellular adaptative mechanism that begins working under such
treatment is
A. Cellular hypertrophy
B. Defense of cell membrane
C, Correction of ions transport
D. Mobilization of plastic reserve
E. Decrease in functional activity of cells
27. A patient came to a dentist with complaints of prolonged bleeding from his gums, reclining of
teeth. Paradontosis was diagnosed. Which type of hypoxia developed in the patient?
A. Hemic
B. Circulatory
C, Respiratory
D. Tissue
E. Hypoxic

28. Dyspnea, cyanosis of mucous membranes of the lips, moderate tachycardia (pulse rate of 80
beats per minute) suddenly occurred in a patient during the treatment of carious tooth. Which
type of hypoxia developed in this patient?
A, Tissue
B. Hemic
C. Circulatory
D. Respiratory
E. Hypoxic

29. Sensitivity of different tissues to oxygen deficit depends on their metabolic rate, power of
glycolytic system, reserve of ATP and ADP, potential possibility of genetic apparatus to
provide the plastic consolidation of hyperdeduction. Which system is in the worst conditions
from these points of view?
A. Nervous system
B. Cardiovascular system
C, Respiratory system
D. Gastrointestinal system
E. Urinary system

30. A patient addressed to the doctor with complaints of the attack of breathlessness and dizziness.
It was revealed that he worked at the chemical factory, producing senile acid. Those signs can
be explained by disturbance of enzyme
A, Succinate dehydrogenase
B. Cytochrome oxydase
C, Catalase
D. Lactate dehydrogenase
E. Pyruvate dehydrogenase

31. Decrease in adenylic nucleotides ATP/ADP ratio leads to the increase in glycolysis in tissues
under hypoxic conditions. Which reaction get active in this situation?
A, Triozophosphate isomerase-depended
B. Lactate dehydrogenase-depended
C, Aldolase-depended
D. Endolase-depended
E. Phosphofructokinase-depended

32. A child has been brought to the admissions office. He had nitrates poisoning symptoms such
as cyanosis, dyspnea, and cramps. What is the mechanism of those symptoms development?
A. Oxyhemoglobin formation
Reduce hemoglobin formation

IP lO I
Carbhemoglobin formation
Methemoglobin formation
Carboxyhemoglobin formation
=

33. Sense of muscular weakness, dyspnea, and tachycardia appeared in a tourist at the altitude of
5200 m. What was the reason for appearance of these symptoms?
A. Decrease in oxygen partial pressure in expired air
B. Decrease in atmospheric pressure
C, Increase in carbon dioxide partial pressure in patient’s blood
D. Gas embolism
E. Air embolism
34. A fireman with signs of carbon monoxide poisoning has been brought from the place of
accident. Which type of hypoxia is more possible in that situation?
A. Hemic hypoxia
B, Ischemic hypoxia
C, Stagnant type of circulatory hypoxia
D. Hypoxic hypoxia
E. Respiratory hypoxia

35. A 60-year-old man was admitted to the hospital with carbon monoxide poisoning. What is
mechanism of disturbances that man has?
A, Inactivation of hemoglobin respiratory function
B. Insufficiency of lungs ventilation
C, Disturbance of blood circulation in the organs and tissues
D. Decrease in partial pressure of oxygen in alveoli
E. Increase in adrenaline secretion

36. Patient has microanizocytosis and poikilocytosis, decreased erythrocytes number, hemoglobin
levels, color index, concentration of iron in serum. These changes were accompanied by
development of hypoxia. It name is:
A. Hemic
B. Hypoxic
C. Circulatory
D. Tissue
E. Respiratory

37. Which reason can lead to hypoxia of exogenous type?


A. Acute bleeding
B. Air embolism
C, Gas embolism
D. Insufficiency of ventilation
E. Decreased partial pressure in inspired air

38. A patient has been suffering from disease of cardiovascular system for a long time. Chronic
blood circulation insufficiency developed in him eventually. Which mechanism of long-term
adaptation to hypoxia takes place in this patient?
A, Activation of hemopoiesis
B. Tachypnea
C, Tachycardia
D. Mobilization of blood from depots
E. Interorganic redistribution of blood circulation

39, A patient has been taken to resuscitation department. Considerable content of sulfhemoglobin
was revealed in his blood. Which type of hypoxia does patient have?
A. Hemic
B. Respiratory
C. Circulatory
D. Tissue
E. Exogenous

40. Alpinists started complaining of breathlessness, palpitation, headache, dizziness, and ringing
in their ears during climbing to mountain at the altitude of 5000 m. Which pathologic factor
caused these symptoms?
A. Hypoxemia
B. Hypokaliemia
C, Increased oxygen capacity of blood
D. Lactacidemia
E. Hypothermia

41. Depressurizing of an airplane cabin happened due to emergency conditions at the altitude of
5000 m. What is the main mechanism of damages inflicted on the people that were in the
airplane?
A. Hypoxic hypoxia
B. Gas embolism
C, Tissue embolism
D. Respiratory hypoxia
E. Circulatory hypoxia

42. Symptoms of hypoxia appeared in a patient after the course of treatment with amidopirin and
sulfanilamides. What is the reason for development of this type of hypoxia?
A. Formation of methemoglobin
B. Formation of carboxyhemoglobin
C. Formation of nitroxyhemoglobin
D. Formation of carbhemoglobin
E. Formation of oxyhemoglobin

43. After an accident in a coalmine, miners were complaining of headache, dizziness, tiredness,
glimmer in front of their eyes, tachycardia. Which type of hypoxia did they have?
A. Hypoxic
B. Circulatory
C, Respiratory
D. Tissue
E. Hemic

44, Signs of hypoxia such as dyspnea, tachycardia, and cyanosis of visible mucous membranes
appeared in a patient with myocardial infarction. Development of hypoxia connected with
A. Decrease in velocity of blood flow
B. Decrease in erythrocytes number
C, Decrease in content of hemoglobin
D. Insufficiency of blood oxygenation
E. Difficult oxyhemoglobin dissociation
45. A man has been brought in hospital by ambulance. He was unconscious because of carbon
monoxide poisoning. What substance accumulation in the blood underlies this condition?
A, Carboxvhemoglobin
B. Oxyhemoglobin
C, Desoxyhemoglobin
D. Methemoglobin
E. Carbhemoglobin

46. A 65-year-old patient suffers from aortic valve stenosis. She has symptoms of heart failure
such as dyspnea, cyanosis, and edema that appeared after viral infection. She was admitted to
the therapeutic department. Which type of hypoxia is in the patient?
A. Circulatory
B. Hemic
C, Hypoxic
D. Respiratory
E. Tissue

47, A 70-year-old patient underwent medical treatment for ischemic heart disease, heart failure in
cardiological department. Which type of hypoxia was in the patient?
A. Circulatory
B. Hemic
C, Respiratory
D. Tissue
E, Mixed
48.A 13-year-old girl undergoes treatment for iron-deficiency anemia in hematological
department. Which type of hypoxia does this patient have?
A. Hemic
B. Circulatory
C, Tissue
D. Respiratory
E, Mixed

49, A 56-year-old woman suffers from thyrotoxicosis for a long time. Which type of hypoxia can
develop in this patient?
A, Tissue
B. Hemic
C, Circulatory
D. Respiratory
E, Mixed

50. Group of tourists ascended to the altitude of 4200 m. Three alpinists complained of headache,
pain in their ears and frontal sinuses, somnolence, considerable muscular weakness, irritability
during the ascent. The possible reason for appearance of these symptoms was
A, Altitude sickness
B. Hyperbaric oxygenetion
C, Caisson disease
D. Gas saturation in blood
E. Altitude decompression

ANSWERS:
1-A, 2-B, 3-D, 4-B, 5-C, 6-C, 7-B, 8-D, 9-B, 10-B, 11-C, 12-D, 13-B, 14-A, 15-D, 16-D, 17-D,
18-A, 19-B, 20-B, 21-C, 22-B, 23-A, 24-B, 25A, 26-A, 27-D, 28-C, 29-A, 30-B, 31-B, 32-D, 33-
A, 34-A, 35-A, 36-A, 37-E, 38-A, 39-A, 40-A, 41-A, 42-A, 43-A, 44-A, 45-A, 46-A, 47-A, 48-A,
49-A, 50-A
Topic 8. Pathophysiology of cell.

1. Enzyme trypsin is released by the pancreas as a non-active proenzyme trypsinogen. What is


the mechanism of proenzyme trypsinogen activation?
A. Allosteric activation
B. Elevation of de novo synthesis at genetic level
C. Change of quartiary structure of molecule
D. Phoshporilization of molecule
E. Restricted proteolysis

2. Necrosis of cells develops after they were exposed to high dose of ultraviolet irradiation. What
reactions underlie these changes?
A. Hydrolysis of proteins
B. Hydrolysis of lipids
C. Decarboxylation of amino acids
D. Peroxidation of lipids
E. Oxidation of carbohydrates

Increase in levels of Alanine Aminotransferase (AIAT) and Aspartate Aminotransferase


(AsAT) was found during examination of patient’s blood serum. What changes in organism
at cellular level lead to this situation?
A. Disturbances of intercellular interactions
B. Disorders of enzyme systems of cells
C. Impairment of energy supply of cells
D. Destruction of cells
E. Damage of genetic apparatus of cells

Increase in activity of Lactate Dehydrogenase was found during examination of patient’s


blood serum. What changes at cellular level lead to such disturbances?
A. Damage of plasmatic membranes
B. Disorders of energy supply
C. Disturbances of intercellular interactions
D. Damage of genetic apparatus of cells
E. Inhibition of enzyme systems
Cell of an experimental animal was exposed to X-rays. Protein fragments were formed in
cytoplasm as a result. What cellular organelles participate in their utilization?
A. Lysosomes
B. Glogi apparatus
C. Ribosomes
D. Endoplasmatic reticulum
E. Cell center

Activity of Alanine Aminotransferase (AIAT) and Aspartate Aminotransferase (AsAT)


sharply increases in blood plasma at hepatitis and myocardial infarction. What are the reasons
for increase in their activity in blood?
A. Damage of cell membranes and release of enzymes to the blood
B. Augmentation of enzyme activity by hormones
C. Pyridoxine deficiency
D. Increase in rate of amino acids synthesis in tissues
E. Increase in rate of amino acids decay in tissues

Tissue culture was exposed to radiation. Nucleoli in cell nuclei were damaged. What
organelles resumption becomes problematic in cell cytoplasm?
A. Endoplasmatic reticulum
B. Ribosomes C. Lysosomes
D. Golgi apparatus
E. Microtubules

Activation of universal membrane structure injuring mechanism occurs at reperfusion


syndrome. This mechanism is referred to as
A. Beta-oxidation of lipids
B. Oxidation of cytochromes
C. Microsomal oxidation
D. Knoop-Lienen cycle
E. Peroxidation of lipids

It was established that pneumonia was caused by viruses in a 5-yearold child. What is the
leading mechanism of cell injury in this pathology?
A. Protein
B. Electrolyte-osmotic
C. Nucleic
D. Lipid
E. Calcic

10. Destruction of cartilage cells and disorder of their functions are observed in a patient with
rheumatism. Which of cellular organelles takes active part in this pathological process?

A. Ribosomes
B. Microtubules
C. Lysosomes
D. Golgi apparatus
E. Cellular center

11. Which ions accumulation in cytoplasm of muscular cells accounts for stable constriction of
myofibrils?
A. Calcium
B. Sodium
C. Potassium
D. Magnesium
E. Hydrogen
12. What is the direct reason for disorders of calcitum-ions extraction from cytoplasm during cell
injury?
A. ATP deficiency
B. Acidosis
C. Increase of osmotic pressure in cytoplasma
D. Increase of permeability of cell membranes
E. Denaturation of proteins

13. At the phase, preceding diastolic relaxation of myocardium, concentration of calcium ions
sharply increases in sarcoplasm, and uncombined calcium is virtually absent from sarcoplasm
at diastole.
Which of following structures participates in accumulation of calcium?
A. Mitochondrions
B. Lysosomes C. Ribosomes
D. Golgi apparatus
E. Nucleoli

14. During study of cell structure by students the question appears


“What are biological membranes by their structure?”
A. Bimolecular lipid layer with protein constituents
B. Bimolecular lipid layer
C. Monomolecular lipid layer
D. Bimolecular protein layer
E. Bimolecular protein layer with lipid constituents

ANSWERS:
1-E, 2-D, 3-D, 4-A, 5-A, 6-A, 7-B, 8-E, 9-C, 10-C, 11-A, 12-A, 13-A, 14-A
Topic 9. Pathophysiology of inflammation.

1. A 7-year-old child fell ill sharply. Hi has catarrh, cough, lacrimation and large-spot rash on
skin; his temperature is increased up to 380C. Mucous membrane of patient’s pharynx is
edematous and red; buccal mucous membrane has whitish spots. What is the character of
inflammation underlying changes of buccal mucous membrane?
A. Hemorrhagic
B. Serous
C. Purulent
D. Catarrhal
E. Fibrinous

2. Painfulness of tooth and edema of lower part of face at the side of ill tooth are present in a
patient with acute pulpitis. What is leading mechanism of edema development in this case?
A. Increase in production of aldosterone
B. Disturbances of trophic function of nervous system
C. Disorder of neural regulation of water-salt metabolism
D. Disorders of microcirculation at the focus of injury
E. Hypoproteinemia
A 63-year-old woman has signs of rheumatic arthritis. Which of below enumerated indices of
blood test will be most significant for diagnosis verification?
A. R-glycosidase
B. Lipoproteins
C. Acid Phosphatase
D. Summary glycosaminoglycans
E. General cholesterol

A 5-year-old girl has high temperature and sore throat. At the examination of this patient the
following was revealed: edema of soft palate, gray films on tonsils, which were removed
difficulty and left after themselves deep hemorrhagic defects of tissue. Which disease of
bellow enumerated is the most possible?
A. Lacunar tonsillitis
B. Simanovsky-Vincent’s tonsillitis
C. Necrotic tonsillitis
D. Diphtheria of pharynx
E. Infectious mononucleosis

A cook burnt his arm with steam. What substance increased and led to development of redness,
edema and painfulness of affected area of skin?
A. Lysine
B. Histamine
C. Thiamine
D. Galactosamine
E. Glutamine
A child with asphyxia was taken to the hospital. On examination of the child whitish and
difficulty separating films were revealed in his larynx. What type of inflammation is observed
in this case?
A. Croupous inflammation
B. Diphtheritic inflammation
C. Purulent inflammation
D. Catarrhal inflammation
E. Serous inflammation

Enlargement and deformation of joints were revealed in a patient with rheumatism. What type
of inflammation underlies these changes?
A. Alterative
B. Proliferative
C. Exudative
D. Fibrinous
E. Hemorrhagic
A painful blisters full of opaque fluid has formed in patient after thermal burn. What type of
inflammation has appeared?
A. Granulomatous
B. Croup
C. Serous
D. Proliferative
E. Diphtheritic

Prevalence of proliferative processes was revealed in a patient with chronic inflammation of


skin and subcutaneous adipose tissue. Which hormone deficiency can lead to this situation?
A. Cortisol
B. Aldosterone
C. Insulin
D. Growth hormone
E. Thyroxin

10. A patient complains of a pain in joints. Edema and redness are visible above joints. Which
enzyme activity has to be analyzed for establishing the diagnosis?
A. Hyaluronidase
B. Creatine kinase
C. Alkaline phosphatase
D. Acid phosphatase
E. Urease

11. Indicate inflammatory mediators which have to be inhibited for decrease in exudation:
A. Catecholamines
B. Histamine
C. Heparine
D. Thromboxan
E. Interleukine-1
12. Condition of biological active substances (BAS) prevalence over there inhibitors usually
occurs in inflammation. Indicate correct correspondence of BAS to their inhibitor
A. Histamine - carboxypeptidase
B. Catecholamines - cholinesterase
C. Kinins - monoaminooxidase
D. Leukotriens - arylsulfatase
E. Serotonin-protease inhibitor

13. Edema has developed as a result of oncotic pressure elevation in the site of inflammation.
What is the main reason of hyperoncia?
A. Activation of endotheliocytes
B. Protein catabolism
C. Drainage disorders
D. Macromolecules accumulation
E. Gluconeogenesis intensification

14. What factor promotes proliferation in focus of chronic inflammation?


A. Phospholipase activation
B. Hyperosmia
C. Protein catabolism
D. Hypoxia
E. Prostacyclin synthesis

15. A patient has high body temperature, redness, edema, painfulness on her right forearm. What
biological active substances intensify inflammatory reaction?
A. Vasopressin
B. Prostacyclins
C. Phospholipase D
D. Proteolysis inhibitors
E. Kinins
16. A patient with inflammation of a forefinger has acute pain, edema, enlargement of local lymph
nodes, temperature rising to 38,50C. What factors lead to exudation in the inflammatory focus?
A. Lymph flow augmentation
B. Proliferation
C. Resistive vessels tone increase
D. Increase of volumetric flow rate of blood
E. Increasing in endothelial permeability

17. Pleurisy with sequential formation of purulent exudation developed in a patient after the chest
trauma. What is the main factor of pus formation?
A. Change of blood flow rate
B. Chemotaxis
C. Change of physicochemical properties in the site of inflammation
D. ATP production disorders
E. Hyperosmia

18. A patient has an inflammatory process on the outer surface of calf. Redness, edema, pain,
enlargement of local lymph nodes, and body temperature rising are noticed in patient. What
are the principles of pathogenetic therapy of acute inflammation?
A. Proteolysis activation blockage
B. Energy formation processes improving
C. Free radical oxidation increase
D. Introduction of mineralocorticoids
E. Introduction of thyroid hormones
19. A patient with finger inflammation has sharp pain and edema in the inflammatory focus. After
opening of an abscess these symptoms have disappeared. What favors disappearance of pain?
A. Decrease of osmotic pressure
B. Increase in c-AMP concentration
C. Alkalosis
D. Kallikrein-kinin system activation
E. Mast cells degranulation

20. There are edema, redness and soreness in a place of injury. What is the reason for
inflammation’s symptoms appearance?
A. Sympathetic nervous system activation
B. Monoamine oxidase activation
C. BAS discharged by cells
D. Carboxypeptidase activation
E. Limited proteolysis inhibition

21. Condition of biological active substances (BAS) prevalence over there inhibitors usually
occurs in inflammation. Indicate correct correspondence of BAS to their inhibitor
A. Histamine - carboxypeptidase
B. Catecholamines - cholinesterase
C. Kinins — monoamine oxidase
D. Serotonin - arylsulfatase
E. Kallikrein - protease inhibitor

22. A 37-year-old male patient was admitted to the surgical department with symptoms of acute
pancreatitis. Which preparation is the most advisable for use in the first turn?

A. Platyphyllin
B. Contrycal
C. Analgin
D. Ephedrine
E. Nospanum

23. Female patient, aged 32, was stung by wasp. On the skin of left cheek (place of sting) there is
a zone of hyperemia and edema. What is the primary mechanism of edema in this case?
A. Decrease of oncotic pressure
B. Lymph drainage decease
C. Increase of oncotic pressure
D. Increase in capillary permeability
E. Increase of hydrostatic pressure in capillaries

24, What inflammatory mediator is formed due to limited proteolysis of plasma globulins?
A. Histamine
B. Leukotriens
C. Bradykinin
D. Prostaglandins
E. Lymphokines

25. Fever and increase of antibodies and leukocytes have appeared in animal under experimental
modeling of inflammation. What substances conditioned to all these common reactions in
inflammation?
A. Leukotriens
B. Interleukins
C. Mineralocorticoids
D. Glucocorticoids
E. Somatomedins

26. In a patient with eczema there are 5 typical symptoms of inflammation (Celsius - Halen
pentad). Find the one of them:
A. Cyanosis
B. Jaundice
C. Pigmentation
D. Redness
E. Albinism
27. A patient sustained trauma of knee joint and posttraumatic hemorrhagic bursitis appeared.
After 3 months passive movements limitation in extend was observed. Limitation was a result
of scar formation. What inflammatory component was a basis of this complication?
A. Secondary alteration
B. Exudation
C. Tissue hyperplasia D. Proliferation
E. Primary alteration

28. In female patient, 28, abscess opening was performed and fast wound reparation was observed.
Reparation was preceded without score formation. What cells play the main role in
proliferation?
A. Eosinophils
B. Neutrophils
C. Fibroblasts
D. Lymphocytes
E. Monocytes

29. In examination of abscess punctate under a microscope different blood cells were revealed.
Which of them appears the first in inflammatory focus?
A. Monocytes
B. Mast cells
C. Eosinophils
D. Neutrophils
E. Lymphocytes

30. What is the main factor of pain formation in pulpitis?


A. BAS action
B. Hydrostatic pressure increasing
C. Excess of pain receptors excitability
D. Ischemia
E. Excess of brain centers sensitivity

31. What is the main factor of edema formation in pulpitis?


A. Capillaries permeability increasing
B. Hypoproteinemia
C. Tissue hyperosmia
D. Blood hyperosmia
E. Capillaries hydrostatic pressure increasing

32. Which of following inflammatory mediators are formatted under the influence of
lypooxy genases?
A. Leukotriens
B. Prostaglandins El, E2
C. Prostacyclins
D. Thromboxans
E. Thrombocytes activation factor

33. Patient has a caries complicated with pulpitis. This condition is accompanied by unbearable
pain. What is the main reason of pain?
A. Ischemia
B. Emigration
C. Primary alteration
D. Exudation
E. Proliferation

34. It is known, that inflammatory mediators play the main role in inflammation pathogenesis.
What is the histamine action on inflammation?
A. Chemotaxis
B. Blood vessel permeability increasing
C. Thrombocytes aggregation
D. Blood vessels constriction
E. Blood coagulation
35. Female patient, 19, complains of pain in gums, gingival hemorrhage. These symptoms have
appeared 3 days after sulphamide drugs taking. Objectively: gingival papilli and gum margin
are reddened and swollen; there is a hemorrhage appeared in response of touching. What type
of inflammation is observed in patient?
A. Acute cattharal inflammation
B. Hemorrhagic inflammation
C. Fibrinous inflammation
D. Pus inflammation
E. Chronic cattharal inflammation

36. A patient 6 years old was admitted to the hospital with asphyxia. The membranous coats on
mucosa of fauces were revealed. The coats can be removed easy. What type of inflammation
is in patient?
A. Fibrinous
B. Necrotic
C. Pus
D. Cattharal
E. Hemorrhagic
37. In child patient there are following symptoms of inflammation over the skin scratch: pain,
swallowing, redness. These sings are symptoms of immediate hypersensitivity. What blood
cells lead to such changings?
A. Lymphocytes
B. Eosinophils
C. Basophiles
D. Monocytes
E. Neutrophils
38. In a girl patient, 5, there are intense throat pain and fever. Objectively were revealed edema
and membranous coats on mucosa of fauces. The coats cannot be removed easy. What of the
followings are more probably?
A. Simanovsky - Vensann tonsillitis
B. Infectious mononucleosis
C. Diphtheria of fauces
D. Necrotic tonsillitis
E. Lacunar tonsillitis
39, Patient has a caries complicated with pulpitis. This condition is accompanied by unbearable
pain. What is the main factor of pain appearing?
A. High pressure of fluid in inflammatory area
B. Bradykinin
C. Prostaglandins
D. Histamine
E. Serotonin
40. There are lots of neutrophils in abdominal pus exudates in a patient with peritonitis. What is
the main function of neutrophils in inflammatory area?
A. Phagocytosis
B. Prostaglandins secretion
C. Degranulation
D. Histamine liberation
E. Local blood flow regulation

41. Keloid cicatrix has formatted in a place of abscess. Name the stage of inflammation, which
caused scar appearing.
A. Proliferation
B. Exudation
C. Primary alteration
D. Secondary alteration
E. Emigration

42. A male patient, 16, was admitted to the hospital with acute appendicitis. What typical
pathological process is the basis of this disease?
A. Inflammation
B. Hypoxia
C. Fever
D. Tumor
E. Allergy
43. Eyeball inflammation resulted pus in the anterior chamber of the eye (hypopyon). What stage
of inflammation was it?
A. Exudation
B. Proliferation
C. Primary alteration
D. Secondary alteration
E. Emigration
44, In a patient with skin pathological process which has 5 typical symptoms: tumor, rubor, calor,
dolor, functio laesa. The development of such process includes 3 stages: Alteration, .......... ;
proliferation. What is the second one?
A. Exudation
B. Regeneration
C. Reparation
D. Fibrosis
E. Petrification
45. The secondary alteration is caused by:
A. Lysosomal enzymes
B. Kinins
C. Lympokynes
D. Prostaglandins
E. Complement components C3a and C5a

46. In proliferation cells multiplying and defect compensation take place. What is the reason of
cells growth in specific clones?
A. Keylones
B. Platelet-derived grow factor
C. Interleukins
D. Somatomedine
E. Lymphocytes peptide

47. There are following components in pleural effusion: proteis-62g/1, abundance of neutrophils.
What is the type of inflammation in this case?
A. Serous
B. Suppuratuve
C. Fibrinous
D. Hemorrhagic
E. Putrefactive

48. Necrotic focus has appeared on burn, swallowing and red skin. What is the main mechanism
of necrobyosis improvement in inflammatory area?
A. Secondary alteration
B. Primary alteration
C. Emigration of leucocytes
D. Dyapedesis of erythrocytes
E. Fibroblasts proliferation
49. Glucocorticoid treatment is usefull in rheumatic arthritis. What is the basal action of
glucocoricoids?
A. Histamine decreasing
B. Emigration inhibition
C. Phagocytosis inhibition
D. Prostaglandins synthesis inhibition
E. Cell membranes stabilization

50. A patient with abscess of finger has some local and general symptoms of inflammation. What
from following is a local inflammatory sing?
A. Swelling
B. Fiver
C. Headache
D. Lymphangitis
E. Leucocytosis
51. It is known that inflammatory mediators are cellular or humoral.
What from the following is a humoral mediator?
A. Kallidine
B. Histamine
C. Serotonin
D. Interleukine-2
E. Hydrogenium peroxide

ANSWERS:
1-E, 2.-D, 3.-D, 4.-D, 5.-B, 6.-B, 7.-B, 8.-C, 9.-A, 10.-A, > 1 L-B, > 12.-D >

13.-A, 14.-D, 15.-E.16.-E, 17.-B, 18.-A, 19.-A, 20.-C, 21.-E, 22.-B, 23.-D, 24.-C, 25.-B, 26.-D,
27.-D, 28.-C, 29.-D, 30.-B, 31.-A, 32.-A, 33.-D, 34.B, 35.-B, 36.-A, 37.-C, 38.-C, 39.-A, 40.-A,
41.-A, 42.-A' 43.-A, 44.-A, 45.-A, 46.-A, 47.-B, 48.-A, 49.-E, 50.-A, 51.-A.
Topic 10. PATHOPHYSIOLOGY OF EXTREME CONDITIONS. SHOCK.

1. A patient in comatose state has smell of acetone from his mouth. Content of glucose in his
blood plasma is of 18 mmol/L. What kind of coma is the most possible one in this case?
A. Hyperosmolar
B. Ketoacidemic
C. Hypoglycemic
D. Toxic
E. Lactatacidemic

2. A patient with crushed muscular tissue was admitted to the traumatological department.
Which biochemical index of urine is increased in this case?
A. Glucose
B. Mineral salts
C. Uric acid
D. General lipids
E. Creatinin

3. A patient is drowsy, his conscious is depressed, and his reactions to irritants are suspended.
He has a pale dry and edematous skin, muscular fibrillations, mydriasis, and Cheyne-Stocks’
respiration with ammonium scent from his mouth. Pericardial friction sound was found at
auscultation of the patient’s heart. What kind of coma has developed in this patient?
A. Ketoacidotic
B. Renal
C. Hyprosmolar
D. Hepatic
E. Apoplectic
4 A 35-year-old man has massive trauma of lower extremities without considerable external
bleeding. The victim is in exited condition. What component of pathogenesis of traumatic shock
is leading and needs immediate correction?
A. Pain
B. Internal bleeding
C. Intoxication
D. Disorder of organ functions
E. Internal loss of plasma

5. A patient with burn of 30% of body surface has decrease in BP to 75/20 mm Hg, frequent,
filiform pulse (110/min). What is the main factor of decrease in BP in burn shock?
A. Intoxication by decay products
B. Plasmarrhea
C. Activation of sympathetic nervous system
D. Hypoproteinemia
E. Activation of parasympathetic nervous system

6. Spasm of resistant vessels develops in zones with alphaadrenoreceptors in erectile phase


of shock. What hemodynamic changes will be observed in this case?
A. Systemic decrease of peripheral blood flow
B. Centralization of blood flow
C. Bradycardia
D. Decrease of venous return
E. Decrease of blood flow speed

7. A victim of the earthquake felt well just after he was extracted from under ruins, but soon
abrupt worsening of his condition developed. He lost consciousness, his BP was of 70/35 mmHg,
and his pulse was of 90 per min; edema of previously ischemic tissues increased quickly. Which
is the main cause of patient’s condition worsening?
A. Increase in tonus of sympathetic nervous system
B. Thrombosis of subcutaneous venous vessels
C. Dehydration
D. Systemic activation of proteolytic processes and PLO
E. Disorders of renal functions

8. A patient was admitted to the hospital in severe state. He does not answer the questions
and does not react to pain stimuli. Patient’s BP is 50/10 mmHg, his pulse is 50 per min. What are
the reasons for disorders of system hemodynamics at torpid phase of shock?
A. Total decrease in peripheral vascular resistance
B. Elevation of venous return of blood
C. Increase of CBV
D. Rise of tonicity of sympathetic nervous system
E. Decrease in permeability of exchange vessels

9. Intravenous infusion of blood and blood substitutes does not lead to the stabilization of BP
in a patient with torpid phase of shock. After cessation of infusion patient’s blood pressure
decreases again. What does inefficacy of medical treatment in torpid phase of shock connected
to?
A. Spasm of resistant vessels
B. Hemodilution
C. Increase in velocity of blood flow
D. Tachycardia
E. Systemic increase in vessel permeability

10. Levels of plasma proteins are sharply increasing, and the number of alveolar macrophages
and lymphocytes is decreasing in bronchoalveorlar contents of a patient with shock. What is the
mechanism of these phenomena appearance?
A. Increase in permeability of lung capillaries
B. Spasm of resistant vessels of the lungs
C. Increase in inhibitor systems of the lungs
D. Increase in pulmonary blood flow
E. Hyperventilation

11. A patient with torn away finger was admitted to the traumatological department. He is
fussy, verbose, and pale. His pulse is 120 beats per minute, BP 140/90 mmHg. What are the
peculiarities of disorders of microcirculation and systemic hemodynamics in erectile phase of
shock?
A. Systemic spasm of volume vessels
B. Increase in systemic peripheral resistance
C. Decrease in venous return and volume of blood circulation D. Increase in peripheral
resistance in the brain, heart and lungs vessels
E. Hypoxia in zones with beta-adrenoreceptors

12. Hemodialysis was utilized for treatment for acute renal failure developed in a patient as a
result of traumatic shock. While carrying out the hemodialysis patient’s condition became worse,
aggregation of erythrocytes, leukocytes, and platelets occurred in microvessels, and blood
viscosity increased. What kind of disorders of microcirculation developed in the patient?
A. Sludge syndrome
B. Disorders of vascular permeability
C. Extravascular disorders
D. Capillary-trophic insufficiency
E. Lymphatic system insufficiency

13. Tourniquet was applied to upper third of hip of wounded driver just at place of car accident.
The patient was admitted to a surgical department in satisfactory condition 3 hours later. Marked
edema of hip tissues, frequent pulse, cold perspiration, and expressed hypotension develop in the
patient after removal of tourniquet. Which pathological process develops in the patient?

A. Toxic shock
B. Anaphylactic shock
C. Collapse
D. Cardiogenic shock
E. Hemorrhagic shock

14. Daily diuresis amounts to 250 ml in a patient with anaphylactic shock. The patient has
moist rales in his lungs; his consciousness is intact. In patient’s blood acidosis reveals with base
deficiency of 14.5 mmol/L; plasma contents of potassium is 8.8 mmol/L and urea is 48 mmol/L.
How is this disorder of kidney called?
A. Uremic coma
B. Acute diffuse glomerulonephritis
C. Acute renal failure
D. Chronic renal failure
E. Chronic glomerulonephritis

15. Sharp weakness, paleness of skin, lost of consciousness appeared in a patient the next day
after resection of his stomach. The patient’s BP is 70/40 mmHg; pulse is 160 beats per minute. In
the patient’s blood test Hb is 70 g/L, erythrocytes are 2.3x1012/L. What pathology appeared in a
patient?
A. Orthostatic collapse
B. Pain shock
C. Traumatic shock
D. Hemorrhagic collapse
E. Cardiogenic collapse

16. Sharp decrease of systolic BP down to 60 mmHg, tachycardia of 140 beats per minute,
dyspnea, and loss of consciousness developed ina
patient on the second day after myocardial infarction. What pathways have the most important
significance in pathogenesis of this shock?
A. Increase in excitability of the myocardium by products of necrotic decay
B. Decrease in blood volume
C. Development of paroxysmal tachycardia
D. Decrease in stroke volume
E. Development of anaphylactic reaction to myocardial proteins

17. A driver was admitted to an emergency department after a car accident. He does not react
to questions; he is indifferent to everything, pale; he has shallow and infrequent respiration and
BP of 75/50 mmHg.
Name the principal link in pathogenesis of this pathology.
A. Excitement of CNS
B. Inhibition of CNS
C. Loss of blood
D. Toxemia
E. Redistribution of blood

18. To compensate considerable loss of blood resulting from knife wounding of liver, a 30-
yer-old patient with blood of TV (AB) Rh (-) group was transfused with blood of IV(AB) Rh (+)
group. Requirement in repeated blood transfusion appears in several days. What kind of blood is
it possible to use for transfusion?
A. IV(AB) Rh(-)
B. (0) Rh(+)
C. II(A) Rh(-)
D. IV(AB) Rh (+)
E. I1(B) Rh(-)
19. A patient was admitted to a hospital with acute high bowel obstruction. He had prolonged
vomiting and blood pressure decreased down to 60/40 mmHg. Which mechanism of shock
development is principal one in this case?
A. Exhaustion of arteriolar alpha-adrenoreactivity
B. Hypovolemia
C. Acidotic dilation of metarteriols
D. Loss of chlorides
E. Increase in tonus of vagus nerve
20. Novocain solution was injected to a patient for anesthetization at extraction of a carious
tooth. Drop of BP, loss of consciousness, dyspnea, and convulsions develop in the patient in a few
minutes. What is the reason for anaphylactic shock development?
A. Desensitization of the organism
B. Toxic action of Novocain
C. Sensitization to Novocain
D. Autoallergic state
E. Para-allergy

21. Severe burn shock has developed in a patient with 40% of body surface burnt. What is the
cause of shock development?
A. Dehydration of the organism
B. Mineral metabolism disturbances
C. Autoimmunization of the organism
D. Protein metabolism disturbances
E. Pain

22. A teeth was extracted in a teenager with Novocain utilization. Paleness of skin, dyspnea,
and hypotension developed in the teenager 10 minutes later. What kind of allergic reactions
developed in the teenager?
A. Immune complex
B. Stimulating
C. Anaphylactic
D. Cytotoxic
E. Cell-mediated

23. After a car accident a patient has BP of 70/40 mmHg; he is unconscious; his daily diuresis
is about 300 ml. What is the mechanism of urinopoiesis disturbances in this case?
A. Augmentation of glomerular filtration
B. Reduction of tubular reabsorption
C. Decrease in glomerular filtration
D. Augmentation of tubular reabsorption
E. Reduction of tubular secretion

24. A 45-year-old victim with severe craniocerebral trauma was admitted to the hospital by
emergency team. Shockogenic trauma was diagnosed: loss of consciousness, paleness of skin,
decrease in body temperature down to 350C, decrease in muscular tonus, absence of reflexes,
frequent and weak pulse, and decrease in BP down to 50/30 mmHg appeared in the patient. Which
clinic stage of traumatic shock is the patient in?
A. Terminal
B. Erectile
C. Excitation
D. Inhibition
E. Torpid

25. A doctor established shock condition in a patient who fell from high altitude. What is the
most important in traumatic shock development? A. Disturbances of the neurohumoral regulation
B. Toxemia
C. Disorders of regulation of the hemodynamics
D. Hypovolemia
E. Disturbances of renal functions

26. A patient developed anaphylactic shock in a dental room after injection of Lidocain
solution. Which immunoglobulins underlie anaphylactic shock development?
A. IgE
B. IgM
C.IgA
D.IgG
E. IgD
27. Traumatic shock developed in a patient as a result of severe trauma. Neuroendocrine,
hemodynamic, and metabolic disorders took place during the shock development. Clinically, after
erectile stage of shock, another stage developed, which is referred to as:
A. Torpid
B. Septic
C. Chronic
D. Neurocirculatory
E. Ischemic
28. Anaphylactic shock developed in a patient after injection of local anesthetic drug. Which
mechanism of blood circulation disorders is the leading one in this condition?
A. Decrease in tonus of vessels
B. Hypovolemia
C. Pain
D. Activation of sympathetic-adrenal system
E. Reduction of contractile function of the heart

29. | Hypovolemic shock and signs of severe hypoxia developed in a victim of aircraft crash
due to severe hemorrhage. In this situation the most severe disorders for the organism develop in:
A. Heart
B. Kidneys
C. Lungs
D. Liver
E. Spleen

ANSWERS:
1.-B, 2.-E, 3.-B, 4.-A, 5.-B, 6.-E, 7.-D, 8.-A, 9.-E, 10.-A, 11.-B, 12.-A, 13.-A, 14.-C, 15.-D, 16.-
D, 17.-B, 18.-A, 19.-B, 20.-C, 21.-E, 22.-C,
23.-C, 24.-A, 25.-A, 26.-A, 27.-A, 28.-A, 29.-A.
Topic 11. Fever

1. A 52-year-old woman complains to bad pain around her abdomen and back. Acute
appendicitis was diagnosed. There is suspicion of complication of pancreatitis by abscess.
Which type of fever would corroborate appearance of abscess?
A. Hectica
B. Continuous
C. Remittent
D. Intermittent
E. Atypical

2. Polyuria developed in a patient with fever after period of olyguria. What is the leading factor
of diuresis change at the end of the second period of fever?
A. Normalization of hormonogenesis by adrenal medulla
B. Normalization of hormonogenesis by reticular zone of adrenal cortex
C. Normalization of hormonogenesis by glomerular zone of adrenal cortex
D. Normalization of hormonogenesis by fascicular zone of adrenal cortex
E. Normalization of thyroid gland function

3. A patient has felt cold, chills, “goose flesh”, increase of body temperature. Which else changes
characterize the first period of rapid elevation of body temperature?
A. Tachycardia
B. Equilibration between heat production and heat loss
C. Dilation of skin vessels
D. Decrease of arterial pressure
E. Increase of metabolism on 100-200%
4. Body temperature of patient becomes pyretic. Which substances have to act to neurons of
thermoregulation for fever development?
A. Interferon
B. Kallidinum
C. Prostaglandins
D. Free radicals
E. Leucotriens

Fever in a patient develops in following succession of stages:


A. Incrementi; fastigii; decrementi
B. Incrementy; decrementy; fastigit
C. Fastigii; decrementi; incrementi
D. Fastigii; incrementi; decrementi
E. Decrementi; fastigii; incrementi

The temperature of a patient with infectious disease increased to 39.5-40.5 0 C ina day and
kept that level about 1 hour, but then it returned to the normal level. Which type of fever is
described in that case?
A. Continuous
B. Intermittent
C. Remittent
D. Recurrent
E. Atypical

The body temperature of a patient with infectious disease increased to 39.5-40.5 0 C ina day
and kept this level about | hour, but then became normal again. Which disease is characterized
by described type of fever?
A. Tuberculosis
B. Influenza
C. Peritonitis
D. Brucellosis
E. Malaria
A patient had fever after injection of pyrogenal. His skin has become pale, cold; chill appeared
in him, oxygen consumption increased. How do the processes of thermoregulation change in
described period of fever?
A. Increase of heat production and decrease of heat loss
B. Decrease of heat loss
C. Heat loss is equal heat production
D. Decrease of heat production and increase of heat loss
E. Decrease of heat production

Body temperature of the 8-year-old Sasha with meningitis was on the level 39-40 oC for a
week. After 8 days under the influence of antibiotics the temperature has decreased to 36.8 oC
in 1.5 hours. There were acute hyperemia of skin, profuse sweating, decrease of arterial
pressure, and loss of consciousness in him. Which medicine is the pathogenetic remedy?
A. Vasoconstrictors
B. Antibiotics
C. Antipyretics
D. Pyrogenal
E. Sulfanilamide

10. After being in the room with air temperature 40 oC and humidity 80% a patient has been
brought to hospital in grave condition. He was unconscious; he had tachypnea, tachycardia,
and body temperature 41 oC. Reanimation was failed. The patient has died. What is the most
possible direct reason of death in this case?
A. Paralysis of the breath center
B. Collapse
C. Coagulation of blood and decrease of volume of circulating blood
D. Dehydration
E. Heart failure
11. Fever of the patients with relapsing fever is characterized by several periods of pyretic
temperature per day and several periods of normal temperature. Such type of temperature
curve is called:
A. Febris hectica
B. Febris intermittens
C. Febris continua
D. Febris recurrens
E. Febris atypica

12. Sharp increase of the temperature to 38.7 oC was marked in a patient with acute purulent
periodontitis. His body temperature has decreased to normal level after opening the pulp
cavity. Which type of fever was in the patient?
A. Efemeral
B. Septic
C. Recurrent
D. Remittent
E. Continua
13. A patient suffers from osteomyelitis of maxilla. His body temperature increases to 40 oC and
then sharply decreases to 35.6 oC every day. Which type of fever curve is characterized by
these changes?
A. Continua
B. Intermittent
C. Reccurens
D. Atypica
E. Hectica
14. Pallor of the skin, “goose flesh” and increase of oxygen consumption appeared in the patient’s
skin after injection of pyrogenal. Which stage of fever is characterized by these changes?
A. Stadium incrementi
B. Stadium fastigiit
C. The stage of falling temperature by crisis
D. The stage of falling temperature by lysis
15. Acute increase of body temperature, dyspnea, tachycardia, nausea, convulsions, and loss of
conciousness developed in a worker, working in the thick uniform in summer. What was the
most possible reason of development of those symptoms?
A. Equilibration between heat loss and heat production
B. Decrease of heat production
C. Decrease of heat loss
D. Increase of heat production
E. Increase of heat loss

16. A man in light clothes is staying in a room with air temperature +14 oC. Windows and doors
are closed. Which way of heat loss is the most considerable in this case?
A. Evaporation
B. Perspiration
C. Conduction
D. Radiation
E. Convection
17. Experimental mice were kept in a lodge with air temperature 4 oC.
Which adaptive reaction supplies its thermal homeostasis?
A. Limitation of heat loss
B. Decrease of oxygen consumption
C. Anabiosis
D. Increase of blood consumption
E. Decrease of oxidation enzyme activity
18. Inclination of the set point of thermoregulation to higher level due to action of IL-1 is in a
patient. What is the name of this typical pathological process?
A. Fever
B. Hyperthermia
C. Hypothermia
D. Inflammation
E. Hypoxia
19. The body temperature of a patient with crupous pneumonia is 39 oC. The difference between
the morning and evening temperature of his body didn’t exceed | oC during 9 days. Which
type of the fever curves was that?
A. Continua
B. Hectica
C. Intermittent
D. Hyperpyretic
E. Reccurens
20. A patient has fever with following stages: incrementi, fastigii, decrementi. Which disease these
features can characterize?
A. Acute pneumonia
B. Acromegaly
C. Diabetes mellitus
D. Hyperaldosteronism
E. Myocardial hypertrophy

21. Body temperature of patient is pyretic; his skin is hot and red. What was the correlation
between processes of heat production and heat loss in described stage of fever?
A. Heat loss is equal heat production
B. Heat loss is more then heat production
C. Heat production is more then heat loss
D. Heat loss is less then heat production
E. Heat production is less then heat loss

22. The body temperature of a patient with pneumonia was keeping on the level 38.3-38.5 oC all
the first week of disease. Such fever is called:
A. Febrile
B. Hyperpyretic
C. Pyretic
D. Subfebrile
23. Increase of “acute phase” proteins level in blood such as ceruloplasmin, fibrinogen, C-reactive
protein is typical for development of fever. Indicate the possible mechanism of this
phenomenon.
A. Stimulative influence of IL-1 on hepatocytes
B. Destructive action of elevated temperature to the cells of the organism
C. Proliferate action of IL-2 to T-lymphocytes
D. Adaptive reaction of the organism to pyrogen
E. Degranulation of mast cells
24. Most infectious diseases are characterized by development of fever. It can be explained:
A. Formation of IL-1 during phagocytosis of microorganisms
B. Intoxication of the organism
C. Degranulation of mast cells
D. Activation of T- and B-lymphocytes
E. Processes of exudation
25. Attacks of fever in a patient occur periodically. During the attack the body temperature sharply
increases, keeps pyretic level nearly 2 hours and then decreases to normal level. This type of
fever is typical for:
A. Tertian fever
B. Sepsis
C. Brucellosis
D. Relapsing fever
E. Epidemic typhus
26. Body temperature of a patient is 39 oC for several hours (stadium fastigii). Indicate which
changes of physiological functions are the most typical for this stage of fever.
A. Bradycardia
B. Ingibition of phagocytosis
C. Increase of heat production
D. Increase of heat production
E. Heat production is equal heat loss
27. In a patient with pneumonia the increased body temperature was revealed. What sort of
biological active substances play the most important role in development of fever?
A. Interleukin-1
B. Leukotriens
C. Serotonin
D. Histamine
E. Bradykinin
28. In a patient with prolonged fever after the course of treatment body temperature begins
decreasing. What is the possible mechanism of temperature decrease?
A. Protective activation of immune system
B. Decrease of heat production due to reducing metabolic rate
C. Decrease of production of pyrogens
D. Increase of resistance of organism to action of the pyrogens
E. Increase of heat loss due to peripheral vasodilatation
29. What is the most efficient mechanism of heat loss in case of following environmental
conditions: 80% air humidity and +350C (95 degree Fahrenheit) air temperature?
A. Hyperventilation
B. Sweating
C. Conduction D. Convection
E. Irradiation
30. During the examination of the patient following symptoms were revealed: redness of skin,
skin is hot and dry to touch, heart bit rate is 92 per minute, respiratory rate is 22 per minute,
body temperature is 39,20C (102.5 degree Fahrenheit). What is the correlation between heat
production and heat loss in described period of fever?
A. Heat production surpasses heat loss
B. Heat production equals to heat loss
C. Heat production is less than heat loss
D. Decreasing of heat production without changes of heat loss
E. Increasing of heat production without changes of heat loss
31. Animal was injected with pyrogen to reproduce fever. What mechanism starts process of
temperature increasing?
A. Activation of non-shivering thermogenesis
B. Activation of shivering thermogenesis
C. Rise of set point of thermoregulation in hypothalamus
D. Reduction of heat loss
E. Dissociation of oxidation and oxidative phosphrilation in tissues
32. In a patient with third stage of fever reaction following manifestations observed: abundant
sweating, tachypnea (increase in respiratory rate), decrease of body temperature. What is the
mechanism of development of these symptoms?
A. Reduction of shivering thermogenesis
B. Secondary aldosteronism
C. Rise of set point of thermoregulation in hypothalamus
D. Predomination of heat production over the heat loss
E. Peripheral vasodilatation
33. Patient, who has been suffering from malaria, has weakness of heart activity and tachycardia
during the stage of heart loss. What is the name of this complication?
A. Infectious-toxic collapse
B. Bacterial shock
C. Hemorrhage collapse
D. Ortostatic shock
E. Hemorrhage shock
34. Adaptation of organism disturbances to decrease of environmental temperature when using
medicines alpha-adrenoblokators. Which mechanism is responsible for this?
A. Formation of primary heat
B. Constriction of skin vessels
C. Contractile thermogenesis
D. Sweating
35. After blood trasfusion patient complaints feeling of heat, rigor, increase of body temperature
to +400C. Its known the cause of elevation temperature is secretion of endogenous pyrogens.
Which cells produce endopyrogens?
A. Erythrocytes
B. Platelets
C. Endotheliocytes
D. B-lymphocytes
E. Macrophages
36. The different types of temperature curve are reported on examination of the infected patients.
What other pathological conditions can lead to the fever?
A. Isoosmotic hyperhydratation
B. Sistemic immune complex diseases
C. Excess production of glucocorticoids
D. Protein starvation
E. Hypohonadism
ANSWERS:
1.-C. 2.-D. 3.-B. 4.-B. 5.-A. 6.-A. 7.-A. 8.-B. 9.-A. 10.-E. 11.-C. 12.-B. 13.-A. 14.-C. 15.-A 16.-
B. 17.-E. 18.-A. 19.-A. 20.-A. 21.-D. 22.-A. 23.E, 24.-A. 25.-C. 26.-D 27.-A. 28.-A. 29.-A. 30.-
A. 31.-A. 32.-A. 33.-A.
34.-A. 35.-A. 36.-E.

Topic 12. Pathophysiology of tissue growth. Tumors.

1. Rate of appearance of tumors is increased in elderly people. One of the main reasons for this
is:
A. Increase in activity of cellular immunity
B. Decrease in activity of cellular immunity
C. Rise of disorders of mitoses
D. Decrease of intensity of antibody production
E. Increase of intensity of antibody production

2. Ina patient with metastases of lung carcinoma introduction of cytostatics led to suspension of
metastases growth at first but later metastases resumed spread. What is the most possible
mechanism of secondary growth of metastases?
A. Absence of contact braking
B. Absence of Heiflik’s limit
C. Rise of genetic heterogeneity of tumor cells
D. Increased glucose consumption by tumour
E. Increased amino acids consumption by tumour

3. A 56-years-old patient, who had contact with diethylnitrozamine at his work place, complains
of pain in right subcostal area, weakness, loss of appetite, and decreased workability. At
examination of this patient: surface of his liver is rough, splenomegaly and ascites are present
in him; his body temperature is 37.20C; in his blood analysis ESR is 25 mm/hour, besides
neutrophilic leukocytosis, and hypochromic anemia were found. What disease developed in
the patient’s organism?
A. Cancer of liver
B. Hepatitis
C. Cirrhosis of liver
D. Gallstone disease
E. Dyskinesia of bile ducts
4. Approximately 60% of tryptophan is oxidized through serotonin pathway in case of malignant
intestine carcinoma. What vitamin demand is increased for a patient with malignant carcinoma
of intestine?
A. Nicotinic acid
B. Pantothenic acid
C. Folic acid
D. Pyridoxine
E. Riboflavin
Malignant tumor of lung was diagnosed in a patient. What feature of tumour growth testifies
its malignancy?
A. Infiltrative growth
B. Unregulated growth
C. Unlimited growth
D. Expansive growth
E. Appearance from one cell
Tumor has developed in a patient with innate immunodeficiency. What factor of non-specific
immunity participates in anti-tumor defense?
A. Lysozyme
B. Interleukin-1
C. Lactoferrine
D. Arylsulfatase
E. Properdine

The woman complained to the doctor for changing of voice, appearance of hair on the face,
and reduction of breast. Where would a tumor develop that could lead to these symptoms?
A. Tumor of zona reticulata of adrenal glands
B. Tumor of ovaries
C. Tumor of anterior lobe of pituitary gland
D. Tumor of zona glomerulata of adrenal glands
E. Tumor of zona fasciculata of adrenal glands
Gamma-interferon was used for treatment of the patient, suffering from tumor. What property
of this substance was used for treatment of this tumor disease?
A. Activation of synthesis of Ig
B. Activation of B-lymphocytes
C. Activation of killer effect
D. Activation of complement system
E. Activation of macrophages

Following changes can occur in development of tumor:


A. Pheochromocitoma — hypotension
B. Insulinoma — hypoglycemia
C. Aldosteroma — hypohydratation
D. Tumor of zona reticulata of adrenal glands — inhibition of sexual growth
E. Tumor of thyroid gland — hypothyroidism
10. Cancer of lung developed in a patient, smoking for a long time. Which cancerogenous
substance is present in tobacco smoke, related to polycyclic aromatic carbohydrates?
A. Benspyren
B. Dimethylaminobenzol
C. Beta-naphthylamine
D. Dietilnitrozamine
E. Orthoaminoazotoluol

11. A male patient, 40, has stenotic (without metastases) esophagus cancer. The following changes
were revealed in that patient: muscular and fat tissue atrophy, brownish color of the skin, thin
epidermis, and cardiac atrophy. What’s the reason of such symptoms?

A. Alimentary cachexy
B. Myasthenia
C. Addison’s disease
D. Cancer cachexy
E. Brown induration

12. In 1910 Rhauss managed to cause sarcoma in chickens by cell-free infiltrate inserting. What
was the method of experimental modeling?
A. Induction
B. Explantation
C. Isotransplantation
D. Homotransplantation
E. Heterotransplantation

13. There is high stage of interaction between lung cancer and tobacco smoking. What chemical
carcinogen is contained in tobacco smog?
A. 3,4-benspyren
B. Orhtoaninotoluol
C. Aphlatoxin
D. Methylcholatren
E. Dyethylnitrosamine

14. Patient complained of weight loss and weakness, in blood analysis hypoglycemia and
hyperinsulinemia were revealed. An additional study tumor of beta-cells was discovered.
Insulin synthesis improvement in this case is a result of:
A. Biochemical athypia
B. Morphological athypia
C. Functional athypia
D. Differentiation athypia
E. Immunological athypia
15. What biological process augmentation is typical for tumor cells?
A. Anaerobic glycolysis
B. Decarboxilation
C. Tissue respiration
D. Lipolysis
E. Gluconeogenesis

16. Erlich’s tumor was transplanted to animal. What is the evidence of tumor progression?
A. Unlimited growth
B. Resistance to cytostatics
C. Anaplasia
D. Infiltration
E. Tumor weight increasing

17. Unpainted formation under the jaw was appeared in liquidator of Chernobyl’s disaster after
12 years of accident. The size of it has increased till last month. The blood analysis is in norm.
What pathological process is most suspicious in this case?
A. Lymphadenitis
B. Syaloadenitis
C. Abscess
D. Malignant tumor
E. Cyst

18. What is the most effective manner of experimental transplantation of tumor?


A. Isotransplantation
B. Autotransplantation
C. Heterotransplantation
D. Homotransplantation
E. Allotransplantation

19. There is a tumor of tongue in patient. What characteristics of tumor may be considered as
malignant?
A. Dysplasia
B. Expansion
C. Infiltration
D. Pasteur’s positive effect
E. Mitosis increasing

20. What cell structure is a «target» for chemical cancerogens?


A. Nuclear DNA
B. Lysosomes
C. Mitochondria
D. Cytoplasmic membrane
E. Ribosomes

21. A patient with lung cancer has been smoking 30 cigarettes per day for 20 years. What the
group of cancerogens is in tobacco smog?
A. Polycyclic carbohydrates
B. Aminoasosubstances
C. Nitrosamines
D. Amines
E. Heterocyclic carbohydrates

22. A female patient was admitted to the hospital with diagnosis «uterine cervix cancer. What
from the following processes are not typical for tumor cells?
A. Anaerobic glycolysis
B. High glucose metabolism
C. Aerobic glucolysis
D. High aminoacides metabolism
E. Organospecificity absence in aminoacides consumption
23. A patient with urinary bladder cancer was working in coke factory. What substance was the
most probable reason of this pathological condition?
A. Naphtvlamine
B. Dichlorethane
C. Vinegar acid
D. Alcohol
E. Pethroleynic aether
24. It is established that tumor tissue receives in 20-25 times less of glucose that intact tissue in
equal glucose amount. What metabolic changings lead to such event?
A. Aerobic glycolysis enhancement
B. Oxydation improvement
C. Normal interaction of these processes
D. Tissue respiration improvement
E. Decreasing of anaerobic glycolysis

25. They got nitrogenous nitrite to experimental animals. A tumor was developed in 80% of
animals. What was the group of cancerogens?
A. Nitrosamines
B. Aminoasosubstances
C. Polycyclic carbohydrates
D. Simple chemical substances
E. Hormones

26. After Chernoby] disaster morbidity of tumors has been increasing.


What action of the radiation has been appearing?
A. Oncogenic
B. Thermal
C. Mutagenic
D. Cytostatics
E. Immunostimulative

ANSWERS:
1.-B. 2.-C. 3.-A. 4.-A. 5.-A. 6.-E. 7.-A. 8.-C. 9.-B. 10.-A. 11.-A. 12.-A. 13.-A. 14.-C. 15.-A. 16.-
B. 17.-D. 18.-B. 19.-C. 20.-A. 21.-A 22.-A. 23.A. 24.-A. 25.-A. 26.-A.
Topic 13.Pathophysiology of metabolism. Diabetes mellitus.

1. A patient was admitted to the hospital in comatose state. Accompanying people said that hi
lost consciousness at training while he was finishing the Marathon distance. What sort of coma
is the most possible in this patient?
A. Hyperglycemic
B. Hypoglycemic
C. Hypothyroid
D. Hepatic
E. Diabetic ketoacidosis
2. Patient has hyperglycemia, glucosuria, polydipsia, polyphagia, and polyuria. What hormone
hyposecretion do these changes develop due to?
A. Antidiuretic hormone
B. Atriopeptide
C. Glucagon
D. Insulin
E. Cortisone

3. A patient, aged 80, complains of increased appetite, thirst, elevated urination, and worsening
of general condition after the taking some sweet food. What disease is it?
A. Hypercortisolism
B. Hyperthyroidism
C. Hypothyroidism
D. Diabetes insipidus
E. Diabetes mellitus

4. A man, aged 38, is under the course of treatment for schizophrenia at in-patient department.
Contents of glucose, ketone bodies, and urea in his blood are normal. Shock therapy with
regular injections of insulin has led to development of insular coma, and after that state of
patient becomes better. What is the most possible reason for insular coma?
A. Glucosuria
B. Dehydration of tissues
C. Hypoglycemia
D. Metabolic acidosis
E. Ketonemia

In worker at polar station, who has been working there for a long time, hemorrhage form gums
occur, his teeth sway and pull out. What vitamin deficiency leads to these changes?
A. Tocopherol
B. Ergocalciferol
C. Ascorbic acid
D. Folic acid
E. Nicotinic acid

A patient, aged 50, complains of increased appetite, thirst, and loss of body weight, weakness.
At laboratory examination rise of amount of glucose in his blood revealed. What type of cells
is injured in case of this disease development?
A. Lipotropocytes
B. Thytocytes
C. B-cells of Langerhans islets D. A-cells of Langerhans islets
E. Pancreatocytes.

In 18-years-old patient, while laboratory examining presence of glucose in urine and normal
concentration of glucose in blood plasma were revealed. What disorder is the most possible
cause of these changes?
A. Disorders of glomerular filtration
B. Disorders of tubular secretion
C. Disorders of glucocorticoids secretion
D. Disorders of insulin secretion
E. Disorders of tubul I .
. Dyspepsia and vomiting are observed in a newborn after feeding with milk. These phenomena
disappear after feeding with glucose solution. What enzyme that takes part in carbohydrate
digestion is deficient in case of these changes development?
A. Amylase
B. Lactase
C. Maltase
D. Isomaltase
E. Saccharase

In patient painfulness along large nervous trunks and increase of pyruvate in blood are
revealed. What vitamin deficiency may lead to these changes?
A. Pantothenic acid
B. Nicotinic acid (PP)
C. Biotin
D. Riboflavin (B2)
E. Thiamin (Bi)

10. A newborn was admitted to the emergency department with following symptoms: vomiting,
diarrhea, disorders of growth and development, cataract, and mental retardation. Galactosemia
was diagnosed. What enzyme deficiency takes place in this case?
A. Glucose-1-phosphate uridiltransferase
B. Glucokinase
C. Glucose-6-phosphate dehydrogenase
D. UDP glucose pyrophosphorilase
E. UDP glucose-4-epimerase

11. Flatulence, bowel spasms, abdominal pain and diarrhea often develop in some people after
taking milk. These symptoms arise in | - 4 hours after intake only one glass of milk. What
component of milk these symptoms develop due to?
A. Galactose
B. Lactose
C. Maltose
D. Saccharose
E. Fructose

12. Newborn has been refusing food, having vomiting and diarrhea, and some time later its
crystalline lens become opaque. At examination of newborn: glucose in blood — 8.5 mmol/L
and in urine — 1%. What is the most possible diagnosis?
A. Galactosemia
B. Phenylketonuria
C. Tyrosinosis
D. Cystinuria
E. Alkaptonuria

13. In woman, aged 45, without symptoms of diabetes mellitus, content of glucose in blood on an
empty stomach reaches 7.5 mmol/L. What test is necessary to be performed?
A. Determination of residual nitrogen in blood
B. Determination of glucose in blood on an empty stomach
C. Determination of tolerance to glucose
D. Determination of ketone bodies in urine
E. Determination of glycosylated hemoglobin

14. In man, aged 60, who is 170 cm tall and 110 kg weight, content of glucose in blood is 6.8—7.0
mmol/L. Content of insulin in his blood is normal. Tolerance to glucose is decreased, as well
as number of insulin receptors, in this patient. What signs allow us to evaluate this diabetes
mellitus as non-insulin dependent?
A. Decreased tolerance to carbohydrates
B. Recurrent hyperglycemia
C. Patient’s age
D. Normal content of insulin in blood
E. Obesity

15. In a patient, suffered from frequent hemorrhages from internal organs and mucous membranes,
proline and lysine were found in structure of collagen fibers. What vitamin deficiency
contributes disorders of these amino acids hydroxylation?
A. Vitamin C
B. Vitamin E
C. Vitamin K
D. Vitamin A
E. Vitamin Bi

16. In patients who suffered from alcoholism Bi hypovitaminosis is often observed, as a


consequence of nutrition disturbances. Symptoms of vitamin B; deficiency are disorders of
nervous system, psychoses, and amnesia. Why cells of nervous tissue are particularly
susceptible to thiamine deficiency?
A. Aerobic decay of glucose is broken
B. Liplysis in adipose tissue is increased
C. Oxidation of fatty acids is broken
D. Glycolysis is intensified
E. Glycolysis is decreased
17. According to results of gastric juice analysis following traits were revealed: common acidity
— 24 mmol/L, free hydrochloric acid — 1.5 mmol/L, content of gastric mucoprotein is
decreased. What vitamin deficiency is observed in organism?
A. Cobalamin
B. Folic acid
C. Pantothenic acid
D. Nicotinamide
E. Bioflavonoids

18. A 24-years-old woman complains of dryness in mouth and loss of weight in spite of good
appetite. At examination of the patient: height — 162 cm, weight — 65 kg, content of glucose in
blood — 8.3 mmol/L, and presence of glucose in urine. What disease does these symptoms
characteristic for?
A. Diabetes mellitus B. Steroid diabetes
C. Diabetes insipidus
D. Alimentary glucosuria
E. Renal diabetes

19. A man, who have been suffering from diabetes mellitus for a long time, was admitted to the
hospital because of rapid worsening of his condition: general malaise, polyuria, polydipsia,
nausea and vomiting, confusion, sleepiness. Kussmaul respiration and scent of acetone from
mouth were observed in this patient. In his urine high contents of glucose and acetone bodies
were found. What is the reason for worsening of patient’s condition?
A. Diabetic ketoacidosis
B. Gas acidosis
C. Heart failure
D. Renal failure
E. Hypoglycemic coma

20. Treatment of the child for rachitis using vitamin D3 was not efficient.
What is the most possible reason for ineffectiveness of treatment?
A. Disorders of hydroxylation of vitamin D3
B. Deficit of lipids in food
C. Disorders of including vitamin D3 into enzyme
D. Intensified using vitamin D3 by intestine microflora
E. Disorders of transport of vitamin D3 with blood plasma proteins
21. Content of glucose in patient’s blood is: on an empty stomach — 5.65 mmol/I, in one hour after
taking sugar — 8.55 mmol/I, and in two hours after taking sugar — 4.95 mmol/l. These signs are
characteristic for:
A. Healthy person
B. Person suffered from hidden diabetes mellitus
C. Person suffered from non-insulin-dependent diabetes mellitus
D. Person suffered from insulin-dependent diabetes mellitus
E. Person suffered from thyrotoxicosis
22. Patient, aged 26, who suffered from hypoglycemic coma resulted from insulin overdosage,
was intravenously infused with 20% solution of glucose. After this manipulation patient’s
condition improved. What process helps glucose enter the cell?
A. Osmotic transport
B. Pinocytosis
C. Active transport
D. Secretion
E. Phagocytosis

23. A 40-years-old man is suffering from diabetes mellitus. After he has endured tonsillitis,
reinforcement of thirst, nausea, vomiting, abdominal pain, and sleepiness develop in him.
Patient’s BP is 80/45 mmHg, pulse rate 125 bpm, and his skin is dry. Content of glucose in
blood is 28 mmol/L. What complication of diabetes mellitus appears in this patient?
A. Lactic acidosis
B. Diabetic ketoacidosis
C. Hepatic coma
D. Hyperosmolar coma
E. Hypoglycemic coma

24. After the break of diet (taking the easy for assimilation carbohydrates) in a woman, who has
been suffering from diabetes mellitus for a long time, general malaise and increase of blood
pressure gradually develop and hallucinations and cramps appear. Woman has dry skin and
distinct signs of dehydration. What is the reason for worsening of patient’s condition?
A. Hypoglycemic coma
B. Hyperosmolar hyperglycemic coma
C. Diabetic ketoacidosis
D. Heart failure
E. Respiratory failure

25. During the experiment rat was injected with 5% alloxan solution in dose 200 mg per kg of rat
weight. What kind of pathology arises in this case?
A. Arterial hypertension
B. Acute renal failure
C. Diabetes insipidus D. Diabetes mellitus
E. Hepatic failure
26 One-year infant lags in mental development from infants of the same age. The infant has
vomiting, cramps, and loss of consciousness in the mornings. What enzyme deficiency these
changes are connected to?
A. Phosphorylase
B. Arginase
C. Glycogen syntase
D. Saccharase
E. Lactase
27. In patient with constant hypoglycemia blood analysis does not change after injection of
adrenalin. A doctor supposes hepatic disorder.
What function disorder it is?
A. Cholesterol formation
B. Excretion
C. Glycolysis
D. Ketogenesis
E. Glycogen deposition

28. A woman, aged 58, was admitted to the hospital in severe condition. She has confused
consciousness; dry skin, hollow eyes, cyanosis, and scent of rotten apples from her mouth. At
laboratory examination of her: glucose in blood — 15.1 mmol/L, glucose in urine — 3.5%. What is
the most possible reason for this condition?
Hypoglycemic coma

MOOD >
Hyperglycemic coma
Hypovolemic coma
Uremic coma
Anaphy lactic shock

29. A doctor reveals in child symmetric roughness on cheeks, diarrhea, and disorders of neural
activity. What nutrition factors deficit underlies this condition?
Methionine, lipoic acid
HUA S

Lysine, ascorbic acid


Nicotinic acid, tryptophan
Threonine, pantothenic acid
Phenylalanine, pangamic acid

30. The most of participants of Magellan expedition to America died of avitaminosis. This
disease displays as general malaise, subcutaneous hemorrhage, pulling teeth out, hemorrhage from
gums. What is the name for this avitaminosis?
Scurvy
HUA S

Pellagra
Addison-Birmer’s anaemia
Polyneuritis (beriberi)
Rachitis

31. In a woman, of 52 years old and of 125 kg weight, diabetes mellitus develops. It happens
due to:
Decrease of number of insulin receptors
MoOOw>

High-affinity binding insulin to synalbumin


Reduced cell susceptibility to insulin
Increase activity of insulinase
Broken insulin synthesis

32. Unconscious patient was admitted to the hospital. He has Kussmaul respiration, blood
pressure 80/50 mmHg, and acetone scent from his mouth. What substances accumulation in
organism may lead to these disturbances?
Complex carbohydrates
moOOw>

Carbonic acid
Lactic acid
Modified lipoproteins
Ketone bodies
33. Inpatient suffered from diabetes mellitus metabolic acidosis develops due to accumulation
of ketone bodies (beta-oxybutyric acid and acetoacetic acid). At this condition pH of arterial blood
is:
A. 7.40
B. 7.48
Cc. 7.56
D. 7.32
E. 7.66
34. A patient, 56 years old woman, who have been suffering from diabetes mellitus for 6 years,
complains of compressing pain behind her sternum. What mechanism of heart affection is the most
possible in this case?
Myocardial dystrophy
Microangiopathy of myocardial vessels
How,

Macroangiopathy of coronary vessels


Myocarditis
Vegetative neuropathy of heart

35. Content of glucose in patient’s blood is: on an empty stomach — 4.52 mmol/l, in one hour
after taking sugar — 6.23 mmol/I, and in two hours after taking sugar — 2.56 mmol/l. These signs
are characteristic for:
Healthy person
MoOOw>

Person suffered from insulinoma


Person suffered from hidden diabetes mellitus
Person suffered from insulin-dependent diabetes mellitus
Person suffered from thyrotoxicosis

36. What complication may develop when treating diabetic ketoacidosis with large doses of
insulin?
Arterial hypertension
MoOOw>

Leukocytosis
Arterial hypotension
Hypoglycemia
Hyperkalemia

37. A female patient, aged 24, complains of dryness in her mouth, loss of weight despite good
appetite. Her height is 157 cm her weight is 72 kg.
What analysis we have to perform at fist in this patient?
Urinalysis by Zemnitsky
VOw>

Determination of glucose content in 24-hour urine quantity


General urinalysis
Determination of protein fractions in blood serum
E. Coagulogram

38. A patient D., 40-years-old woman, was admitted to the hospital with complaints of
weakness, giddiness, hunger, cold sweat, and cramps. At examination of the patient: distension of
pupils of the eyes, weakening of respiration, and BP is 90/50 mmHg. Biochemical analysis of her
blood shows: general bilirubin is 16.0 mcmol/L, urea is 4.7 mmol/L, creatinine is 98 mcmol/L,
and glucose is 2.0 mmol/L. What kind of coma may develop in this patient?
A. Hyperglycemic
B. Hepatic
C. Renal
D. Hypoglycemic
E. At adrenal glands deficiency
39, The woman B., aged 45, was admitted to the emergency department from a street. At
examination of patient following symptoms was revealed: loss of consciousness, loss of reflexes,
absence of reflexes from pupil of the eye and sclera, Kussmaul respiration, BP is 70/40 mmHg,
and body temperature is 350C. Biochemical analysis of patient’s blood displays: general bilirubin
is 16.0 mcmol/L, urea is 3.6 mmol/L, creatinine is 108 mcmol/L, and glucose is 22 mmol/L. What
kind of coma has developed in this patient?
Hypoglycemic
MoOOw>

Hepatic
Renal
At adrenal glands deficiency
Hyperglycemic

40. Diabetes mellitus develops in animals after injection to them some alloxan. What is the
main mechanism of this type of diabetes mellitus?
Selective damage of lambda-cells of pancreatic islets
moOOw>

Damage of beta- and lambda-cells of pancreatic islets


Formation of antibodies to insulin
Selective damage of beta-cells of pancreatic islets
Gluconeogenesis activation

41. A patient address to a doctor with complaints of constant thirst. Hyperglycemia, polyuria
and increased content of 17-ketosteroids in urine were revealed. What is the most probable disease
in this case?
Addison’s disease
MoOOw>

Myxedema
Glycogenosis of I type
Insulin dependent diabetes mellitus
Steroid diabetes
42. In 62-years-old woman cataract (crystalline lens dimness) develops at the diabetes mellitus
background. What process enhancement due to diabetes mellitus is the cause of cataract?
Ketogenesis
Lipogenesis

MOOS
Proteolysis
Glycosilation of proteins
Gluconeogenesis

43. A 19-years-old patient has suffered from diabetes mellitus since he was 8. He took cure
irregularly. He was admitted to the hospital in connection to diabetes ketoacidosis development.
What kind of respiration is the most possible in this condition?
Kussmaul respiration
HOW pS

Biot’s respiration
Chane-Stocks respiration
Inspiratory breathlessness
Expiratory breathlessness

44, A patient aged 60 has been suffering from diabetes mellitus for 18 years. He complains of
cold of his lower extremities and intermittent lameness during last years. What is the mechanism
of indicated symptoms development?
Macroangiopathy of lower extremities
MoOOw>

Hyperketonemia
Neuropathy
Disturbances of metabolism of muscles
Hyperglycemia

45, An adolescent aged 17 addressed to a physician in connection with enter a college. He has
no complaints. His height is 178 cm; his weight is 96 kg without essential changes during a year.
He has even distribution of subcutancous adipose tissue. His pulse rate is 82 bpm; his BP is 115/80
mmHg. At laboratory examination of a youth: content of glucose in blood is 8.2 mmol/L;
glucosuria — 4.6 g/L. What type of diabetes mellitus is the most probable in this patient?
A. Non-insulin-dependent diabetes mellitus with obesity
B. Insulin-dependent diabetes mellitus
C. Symptomatic diabetes mellitus is due to dyencephalic syndrome
D. Steroid diabetes mellitus is due to Cushing’s syndrome
E. Symptomatic diabetes mellitus is due to acromegaly
46. Signs of fatty dystrophy of liver are revealed in 38-years-old female patient who has been
suffering from diabetes mellitus for a long time. What factor deficiency is leading in this state
development?
A. Lipocain
B. Lecithin
C. Insulin
D. Glycogen
E. Acetyl-CoA
47. | Hyperglycemia and decreased number of insulin receptors on the lipocytes are established
in mice with hereditary obesity. What is the primary mechanism of lipogenesis intensification in
these animals?
MoOOw> Hyperinsulinemia
Hypoinsulinemia
Hyperfunction of lipocytes
Decreased tolerance to glucose
Increased fat deposition

48. A patient with diabetes mellitus did not take the dose of insulin in time. Thus
gyperglycemic coma developed (content of glucose in patient’s blood is 50 mmol/L). What is the
leading factor for such coma development?
Hyperosmolarity of blood plasma
MoOOw>

Hypokalemia
Hypoxia
Hyponatremia
Acidosis

49, What is the mast important mechanism of hypoglycemic coma development?


Carbohydrate starvation of brain
moOOw>

Reinforcement of glycogenilysis
Oppression of glycogenesis
Oppression of gluconeogenesis
Intensification of ketogenesis

50. What is the reason for glucosuria appearance under diabetes mellitus?
Hyperglycemia
MoOOw>

Increase of renal threshold


Rise of hexokinase activity
Increase of glomerular membrane permeability
Polyuria

51. ‘Ina patient with diabetes mellitus loss of consciousness and cramps were observed after
insulin injection. What is a possible result of blood test for glucose in this case?
5.5 mmol/L
MoOOw>

3.3 mmol/L
8.0 mmol/L
10 mmol/L
2.5 mmol/L
52. A patient was admitted to the hospital in connection to osteomyelitis of lower jaw. When
laboratory examining of the patient glucose was found in her urine. Content of glucose in her
blood is normal. What is the reason for glucosuria?
A. Insufficiency of tubular enzyme systems
B. Increase of glomerular filtration
C. Enhancement of tubular secretion
Dz. Decrease of renal concentration function
E. Rise of osmotic pressure in tubules
53. In a patient, who has poisoning with flodzine, glucosuria was revealed; concentration of
glucose in patient’s blood is 5.6 mmol/L. What is the mechanism of glucosuria in this case? A.
Disturbances of tubular reabsorption
B. Reduction of glomerular filtration
C. Increase of glomerular filtration
D. Disorders of tubular secretion
E. Rise of oncotic pressure of blood

ANSWERS: 1.- B.2.-D.3.- E.4.-C. 5.-C. 6.- C.7.- E.8.-B.9.-E 10.-A. 11.- B.12.- A.13.-C. 14.-D.
15.-A.16.-A.17.-A.18.-A.19.-A.20.-A.21.-A.22.C.23.-D.24.-B.25.-D.26.-C.27.-E.28.-B.29.-
C€.30.-A.31.-A.32.-E.33.-D. 34.-C.35.-B.36.-D.37.-B.38.-D.39.-E.40.-D.41.-E.42.-D.43.-A.44.-
A.
45.-A.46.-A.47.-A.48.-A.49.-A.50.-A.51.-E.52.-A.53.-A.
Topic 14. Pathophysiology of water-salt metabolism.

. It is characteristically for cholera to lose large quantity of water and sodium ions from the
organism. The basis of biochemical action of cholera toxin is:
A. Oxidation of aldosterone in the cortex of adrenal glands
B. Activation of adenilat cyclase in enterocytes of small intestine
C. Intensification of renin secretion by the cells of juxtaglomerular apparatus
D. Decrease of synthesis of antidiuretic hormone in hypothalamus
E. Activation of synthesis of atrial natriuretic hormone

. Wasp has stung a woman. Oedema and hyperemia develop at woman’s left cheek (site of the
bite). What mechanism of oedema development is primary in this case?
A. Decrease of osmotic pressure of blood
B. Impediment for lymphatic drainage
C. Increase of oncotic pressure of tissue fluid
D. Increase of capillary permeability
E. Increase of hydrostatic pressure of blood in capillaries

. In worker of fusing workshop thirst develops due to enhanced sweating. He drinks a lot of
water without salt. What kind of water-salt balance disturbance the most possibly develops in
this case?
A. Hyperosmolar hyperhydration
B. Hyperosmolar hypohydration
C. Hypoosmolar hyperhydration
D. Hypoosmolar hypohydration
E. Isoosmolar hypohydration

. A boy, aged 9, with oedemas was admitted to the hospital. What proteins content in blood is
decreased in this case?
A. Albumins
B. Protamines
C. Globulins
D. Hemoglobin
E. Hystons

. Worker of fusing workshop, man of 23 years old and of 60 kg weight, was admitted to the
emergency department. Examination of water-salt exchange in this patient displays: content of
general water is 33 liters (55% of body weight), extracellular sector constitutes 28.6% of body
weight (17.2 liters), intravascular fluid constitutes 4% of body weight (2.4 liters), and
intracellular sector constitutes 26.3% of body weight (15.8 liters). Osmotic pressure of
patient’s blood is 340 mosm/1; content of sodium in it - 160 mmol/l. Patient’s urination is 0.4
liters per 24 hours. Determine the type of dyshydration.
A. Hyperosmolar hyperhydration
B. Hyperosmolar hypohydration
C. There is no dyshydration
D. Isoosmolar hyperhydration
E. Hypoosmolar hypohydration

A patient with severe nephropathy accompanied by severe oedema syndrome that develops as
complication of bronchiectasis. Laboratory examination of this patient displays abundant
proteinuria, cylinderuria, distinct decrease of protein content in blood serum, hyperlipidemia,
hypokalemia, and other pathological changes. What is the most important link in development
of oedemas in this patient? A, Decrease of oncotic pressure of blood
B. Increase of osmotic pressure of interstitial fluid
C. Increase of hydrostatic pressure of blood
D. Blockade of lymphatic drainage
E. Increase of microvessel permeability
A patient has increased osmolarity of urine and decreased urination due to intensive sweating
and dehydration. What hormone secretion changes provide compensatory retention of water at
first?
A. Antidiuretic hormone
B. Aldosterone
C. Corticosterone
D. Thyroxin
E. Insulin
Content of sodium in patient’s blood serum is 100 mmol/l. What does this condition may
manifest in?
A. Edemas
B. Arrhythmias
C. Dehydration
D. Heart arrest
E. Tachycardia
In a patient, aged 44, thirst develops after burns. What receptors generate impulses that
underlie thirst development in this case?
A. Osmoreceptors
B. Pain receptors
C. Thermal receptors
D. Tactile receptors
E. Chemoreceptors

10. Isoosmolar hypohydration has been formed ina patient due to severe diarrhea. What symptoms
are characteristic for this disturbance of water exchange?
A. Edematous syndrome
B. Decrease of content of water inside cells
C. Hypovolemic shock
D. Polyuria
E. Arterial hypertension
11. In patient with affection of kidneys hypoosmolar hyperhydration (water poisoning) has
developed. What is the main pathogenic factor of this syndrome?
A. Anuria
B. Hypoaldosteronism
C. Polyuria
D. Hypoproteinemia
E. Increase of microvessel permeability

12. Hyperosmolar hypohydration has been formed while prolonged water starvation. Which of
following manifestations are typical for this condition?
A. Arterial hypertension
B. Hypoisostenuria
C. Hypothermia
D. Increased salivation
E. Cramps and hallucinations

13. In patient suffered from severe chronic glomerulonephritis retention of isoosmolar fluid in
organism and distinct edematous syndrome occur. What is the major factor of edema
development in case of glomerulonephritis?
A. Hyperproteinemia
B. Secondary aldoseronism
C. Hypoproteinemia
D. Hypoaldosteronism
E. Arterial hypertension

14. In patients with myeloma content of proteins in blood plasma is increased up to 200 g/l. This
leads to redistribution of water between intracellular, interstitial, and intravascular spaces.
What direction does water mainly move at in this case?
A. From interstitium to blood vessels
B. From cells to interstitium
C. From interstitium to cells
D. From blood vessels to interstitium
E. From lymphatic vessels to interstitium

15. A patient of 18 years old and of 60 kg weight was admitted to the hospital with signs of hemic
hypoxia resulting from poisoning by nitric compounds. Examination of water exchange of this
patient displays: general water — 64% of body weight, extracellular fluid — 18%, intravascular
fluid — 5%, and intracellular fluid — 46%; osmotic pressure of blood plasma is 250 mosm/1;
urination is 0.8 L per 24 hours. Define the type of dyshydration?
A. There is no dyshydration
B. Isoosmolar hyperhydration
C. Isoosmolar hypohydration
D. Hypoosmolar hyperhydration
E. Hypoosmolar hypohydration

16. Edemas at lower extremities occur in a patient suffered from severe heart failure. What is the
leading mechanism of edema development in this case?
A. Centralization of blood circulation
B. Lowering of hydrostatic pressure
C. Secondary hyperaldosteronism
D. Orthostatic increase of venous pressure
E. Hypoproteinemia

17. What is the initial link in formation of heart edemas?


A. Increase of vascular permeability
B. Decrease of minute heart volume
C. Activation of renin-angiotensin system
D. Increase of content of aldosteron in blood
E. Increase of secretion of antidiuretic hormone
18. What is the leading factor of edema development in case of nephrotic syndrome?
A. Increase of hydrostatic pressure in capillaries
B. Increase of vascular permeability
C. Hypoalbuminemia
D. Dynamic lymphatic insufficiency
E. Increase of blood volume

19. Patient suffered from cirrhosis of liver was given with 500 ml of 5% glucose solution with
medicines. What disturbances of water-salt balance may appear in this patient?
A. Hypoosmolar hyperhvdration
B. Hyperosmolar hyperhydration
C. Isoosmolar hyperhydration
D. Hypoosmolar hypohydration
E. There is no dyshydrotation
20. Patient has edema of right lower part of face and pulsing pain in tooth, which intensifies when
taking some hot food. Dentist has diagnosed acute pulpitis. What is the leading mechanism of
edema development in this case?
A. Disorders of microcirculation in the focus of inflammation
B. Disorders of trophic function of nervous system
C. Hypoproteinemia
D. Hyperosmia
E. Lymphocytosis
21. Toxic lung edema was modeled in rat using solution of ammonium chloride. What is the
leading mechanism of edema development in this case?
A. Reducing of colloid-osmotic pressure
B. Rising of venous pressure
C. Increase of vascular permeability
D. Disorders of neural and humoral regulation
E. Intensification of lymphatic drainage
22. A rat was intravenously injected by 10 ml of 40% glucose solution. In 60 min coma develops
due to hyperosmolar dehydration in this rat.
What is the mechanism of edema development in this case?
A. Loss of water and salts
B. Reduction of vasopessin synthesis
C. Increase of oncotic pressure of extracellular fluid
D. Increase of osmotic pressure of extracellular fluid
E. Disturbance of acid-base balance

23. Patient has extracellular edema of tissues (dimensions of soft tissues of extremities, liver, and
others are enlarged). What parameter of homeostasis decrease do these changes result from?
A. Viscosity
B. pH
C. Hematocrite
D. Oncotic pressure of blood plasma
E. Osmotic pressure of blood plasma

24. At complete starvation (with taking water) generalized edemas develop. What is the leading
pathogenic factor in this case?
A. Increase of oncotic pressure of interstitial fluid
B. Decrease of osmotic pressure of blood plasma
C. Decrease of oncotic pressure of blood plasma
D. Increase of osmotic pressure of interstitial fluid
E. Decrease of hydrostatic pressure of interstitial fluid

25. When treating for dehydration by means of salt-poor fluids at the background of sharply
reduced excretory renal function resulted from tubular necrosis the worsening of general
condition, confused consciousness, convulsive readiness, and brain edema with vomiting
develop. What kind of water-salt exchange disturbances takes place in this case?
A. Hypoosmolar hyperhydration
B. Isoosmolar hyperhydration
C. Hyperoosmolar hyperohydration
D. Hypoosmolar hypohydration
E. Hyperosmolar hypohydration
26. As it is known, general amount of water in an organism depends on age, body weight, and sex.
Besides, pathogenetic classification of dehydration is important. This classification includes
forms of mineral salt deficiency, forms of water deficiency, and transitional forms. What
reasons, according to this classification, belong to form of mineral salt deficiency?
A. Loss of electrolytes through the stomach
B. Loss of water is due to polyuria
C. Reduced intake of water
D. Loss of water with a bleeding
E. Loss water is due to hyperventilation

27. A patient has uneasiness in the chest and difficult breathing after physical exertion. Some time
later cough with foamy liquid phlegm appears. Significant cyanosis develops in the patient.
What is the leading mechanism for edema development in this case?
A. Hydrodynamic
B. Colloid
C. Membranogenous
D. Lymphogenic
E. Osmotic

28. In inflammation increased vascular permeability and increase of hydrostatic pressure are
observed in microcirculatory vessels. The level of colloid-osmotic pressure of blood does not
significantly changes. There are shift of pH towards acid state, rise of osmotic pressure, and
increase in dispersion of proteins in the interstitial fluid. What kind of edema is observed in
this case?
A. Mixed
B. Hydrodynamic
C. Colloid-osmotic
D. Lymphogenic
E. Membranogenous
29. A person, who has been starving for along time, has edemas. What is the main mechanism of
edema development in this case?
A. Decrease of oncotic pressure of the blood
B. Increase of oncotic pressure of tissues
C. Increase of hydrostatic pressure of venous blood
D. Decrease of hydrostatic pressure of tissues
E. Decrease of blood circulating volume

30. In person, who was bitten by bees, edema of the face and the upper extremities developed.
What is the main mechanism of edema development in this case?
A. Increase of vascular permeability
B. Increase of hydrostatic pressure in capillaries
C. Decrease of hydrostatic pressure of tissues
D. Increase of oncotic pressure of tissues
E. Decrease of oncotic pressure of blood

31. Pulmonary hypertension and right-ventricle heart insufficiency with ascites and edemas
develop in a patient suffered from pneumosclerosis.
What is the main mechanism of edema development in this case?
A. Increase of hydrostatic pressure in veins
B. Increase of oncotic pressure of tissues
C. Decrease of oncotic pressure of blood
D. Reduction of heart stroke volume
E. Increase of vascular permeability

32. Which of followings may cause the isoosmolar hypohydration?


A. Acute blood loss
B. Diarrhea
C. Vomiting
D. Perspiration
E. Hyperventilation

33. Where does fluid accumulate predominantly in case of edema?


A. Intercellular space
B. Intracellular space
C. Pleural cavity
D. Abdominal cavity
E. Intravascular space

ANSWERS:
1.-B. 2.-D. 3.-C. 4.-A. 5.-B. 6.-A. 7.-A. 8.-C. 9.-A. 10.-C. 11.-A. 12.-E. 13.-B. 14.-A. 15.-D. 16.-
C. 17.-B. 18.-C. 19.-A. 20.-A. 21.-C, 22.-D. 23.-D, 24.-C. 25.-A. 26.-A. 27.-A. 28/-A. 29.-A. 30.-
A. 31.-A. 32.-A.
33.-A.
Topic 15. Pathophysiology of acid-base balance and phosphate-calcium metabolism.

1. At examination of patient following were found: hyperglycemia, ketonuria, polyuria,


hyperstenuria, and glucosuria. What kind of acidbase balance disturbances occurs in this case?
A. Gas alkalosis
B. Non-gas alkalosis
C. Metabolic alkalosis
D. Metabolic acidosis
E. Gas alkalosis

2. Acidosis develops in case of severe form of diabetes mellitus. What buffer system
components change at first?
A, Bicarbonate
B. Phosphate
C. Hemoglobin
D. Oxihemoglobin
E. Protein

3. Prolonged convulsions occur in patient suffered from epilepsy. After that following data
of laboratory analysis of this patient were received: pH— 7.14, pCO2 —- 45 mmHg, HCOs. — 14
mmol/l, Na+ - 140 mmol/, Cl- - 98 mmol/l. What kind of acid-base balance disturbances occurs
in this patient?
A. Metabolic ketoacidosis
B. Metabolic lactoacidosis
C. Respiratory alkalosis
D. Metabolic alkalosis
E. There are no disorders of acid-base balance

4. While ascending to mountain in alpinist excitation developed that was replaced with
headache, giddiness, breathlessness, and after that apnea occurred. What kind of acid-base balance
disturbances occurs in this case?
A. Non-gas acidosis
B. Excretory acidosis
C. Gas alkalosis
D. Non-gas alkalosis
E. Gas acidosis

5. A patient suffered from chronic glomerulonephritis has increasing general malaise,


tachycardia with recurrent arrhythmia, confusion, and sleepiness. What kind of acid-base balance
disturbance accompanies uremic coma?
A.Non-gas excretory acidosis
B. Non-gas excretory alkalosis
C. Gas acidosis
D. Gas alkalosis
E. Respiratory alkalosis

6. Pregnant woman has toxicosis accompanied by vomiting of 24 hours duration. After that
tetany cramps and dehydration develop. What kind of shift of acid-base balance leads to described
changes?

A. Gas alkalosis
B. Gas acidosis
C. Non-gas metabolic acidosis
D. Non-gas metabolic alkalosis
E.Non-gas excretory alkalosis

7.A patient suffered from diabetes mellitus was admitted to the hospital because of worsening of
his condition. He has general malaise, polyuria, lethargy, and sleepiness. Kussmaul respiration,
heart arrhythmia, and acetone scent in expired air are noticed in this patient. What kind of shift of
acid-base balance contributes these symptoms?
A. Gas alkalosis
B. Gas acidosis
C. Non-gas metabolic alkalosis
D. Non-gas metabolic acidosis
E. Non-gas excretory alkalosis

8. A group of alpinists was undergone blood analysis in mountains at height 3000 meters.
Following was revealed: decrease of HCO3- down to 15 mmol/l (norm is 22-26 mmol/l). What
is the mechanism of decrease of HCO3- in the blood?
A. Decrease of reabsorption of bicarbonate in kidneys
B. Hyperventilation
C. Intensification of acidogenesis
D. Hypoventilation
E. Reduction of ammoniogenesis

pH of blood of patient suffered from diabetes mellitus sets to 7.3. What component of buffer
system determination is used to diagnose disorders of acid-base balance?
A. Bicarbonate
B. Oxihemoglobin
C. Phosphate
D. Hemoglobin
E. Protein
10. Buffer capacity of blood decreases in worker as a result of exhausting muscle work. What acid
substance income to the blood this may be explained?
A. Alpha-ketoglutaric acid
B. 3-phosphoglycerate
C. Lactic acid
D. Pyruvate
E. 1,3-biphosphoglycerate
11. Repeated vomiting occurs in patient suffered from pylorostenosis which is accompanied by
loss of chloride ions from the organism and development of non-gas alkalosis. What conditions
these changes of acid-base balance may result from?
A. Hyperchloridemia
B. Hyponatremia
C. Hypokalemia
D. Hypernatremia
E. Hyperhposphatemia

12 . 48-years-old patient with diabetes mellitus was admitted to the hospital in severe pre-coma
state. When examining of acid-base balance metabolic acidosis was revealed. Patient was
treated with complex therapy including insulin intramuscular injections and sodium
bicarbonate solution intravenous infusion. What is the main possible mechanism of found
changes development?
A. Disorders of O2 using in cells
B. Disorders of buffer systems of blood
C. Decrease of CO2 removing
D. Excretion of alkali elements with urine
E. Formation of products of incomplete oxidation

13. Patient has disturbances of airways passage at the level of small and medium bronchi. What
kind of acid-base disorders may develop in this patient?
A. Respiratory alkalosis
B. Metabolic alkalosis
C. Respiratory acidosis
D. Metabolic acidosis
E. Acid-base balance does not change

14. In patient suffered from diabetes mellitus coma has developed due to disorders of acid-base
balance. What kind of acid-base disorders develops in this case?
A. Exogenous acidosis
B. Respiratory acidosis
C. Metabolic acidosis
D. Gas alkalosis
E. Non-gas alkalosis
15. A woman has distinct hypersalivation syndrome. Patient has to remove saliva from her mouth
with napkin because of hard pain when she is swallowing. What kind of acid-base disorders
may develop in this patient in some time?
A. Non-gas excretory acidosis
B. Gas alkalosis
C. Metabolic acidosis
D. Non-gas excretory alkalosis
E. Gas acidosis
16. During bronchial asthma attack patient developed gas acidosis (hypercapnia). What buffer
system of blood plays the main role in compensation of this condition?
A. Bicarbonate
B. Hemoglobin
C. Phosphate
D. Protein
E. Ammoniogenesis
17. At examination of a patient hyperglycemia, ketonuria, polyuria, hyperstenuria, and glucosuria
were revealed. What type of acid-base balance disorder takes place in this case?
A. Metabolic acidosis
B. Gas acidosis
C. Metabolic alkalosis
D. Non-gas alkalosis
E. Gas alkalosis

18. A 65-years-old patient with multiple fractures of ribs was admitted to the hospital. What type
of acid-base balance disorder may develop in him?
A. Gas acidosis
B. Gas alkalosis
C. Non-gas acidosis
D. Non-gas alkalosis
E. There are no disorders of acid-base balance

19. A pregnant woman has toxicosis, which accompanied by prolonged vomiting. Data of her
biochemical analyses show: pH of blood — 7.38, pCOz of arterial blood — 46 mmHg, SB — 17
mmol/L, BE — (+ 6 mmol/L). What type of acid-base balance disorder takes place in this case?
A. Compensated non-gas alkalosis
B. Decompensated non-gas alkalosis
C. Compensated non-gas acidosis
D. Decompensated non-gas acidosis
E. Compensated gas alkalosis
20. A 58-years-old female patient was admitted to the hospital in severe state. Data of her
biochemical analyses show: pH of blood — 7.33, pCO: of arterial blood — 36 mmHg, SB — 17
mmol/L, BE — (+ 6 mmol/L).
What type of acid-base balance disorder takes place in this case?
A. Compensated non-gas alkalosis
B. Decompensated non-gas alkalosis
C. Compensated non-gas acidosis
D. Decompensated non-gas acidosis
E. Compensated gas alkalosis

21. What is the reason for gas alkalosis? A. Pulmonary hyperventilation


B. Loss of gastric juice
C. Loss of intestine juice
D. Pulmonary hypoventilation
E. Hyperaldosteronism

22. What kind of acid-base balance disturbances may be observed in case of diabetes mellitus?
A. Non-gas acidosis
B. Gas acidosis
C. Gas alkalosis
D. Non-gas alkalosis
E. Excretory acidosis

ANSWERS:
1.-D, 2.-A. 3.-B. 4.-E. 5.-A. 6.-E. 7.-D. 8.-B. 9.-A. 10.-C. 11.-C. 12.-E.
13.-C. 14.-C. 15.-A. 16.-B. 17.-A. 18.-A. 19.-A. 20.-B. 21.-A. 22.-A.
Topic 16. Pathophysiology of endocrine system.

. The height of an adult is 100cm, in proportional constitution and normal mental development.
What hormone insufficiency do the indicated signs characterize?
Growth hormone
I= |S 10 I [>
Thyroxin
Antidiuretic
Mineralocorticoids
Gonadotropic hormone.

. Under the influence of harmful ecological factors the normal formation of lysosomes is
inhibited in thyrocytes. What stage of hormone production of thyroid gland will be impaired?
Synthesis of colloid
>

Iodinisation of colloid
Synthesis of thyroglobulin
IF |F lO

Reabsorption of colloid
Proteolysis of phagocytosed colloid from follicles.

. Diabetes insipidus appeared in a patient after cerebral infarction with impairment of


hypothalamus nuclei. What is the cause for increased urination in this patient?
Decrease of reabsorption of sodium
I= |S 10 I [>

Acceleration of glomerular filtration


Decrease of water reabsorption
Decrease of arterial pressure
Hyperglycemia

. What hormone stimulates inclusion of calcium in osteoblasts of bone tissue in tooth?


A. Insulin
B. Thyroxin
Cc. Parathormone
D. Hydrocortisone
E. Calcitonin

. A 10-year-old child has a height of 178 cm and weight of 64 kg. What endocrine gland disorder
does this connected with?
A. Sexual glands
B. Adrenal glands
CG Parathyroid glands D. Pituitary gland
E. Thyroid gland.

. A patient with thyrotoxicosis has hyperthermia, bulimia, and loss of weight. What kind of
impairment does this connected to?
Reaction of fat synthesis

I= |F 10 Ie |>
Lyses of adenosintriposphoric acid
Conjunction oxidation and phosphorylation
Reaction of citric acid cycle
Reaction of beta-oxidation of fatty acid.

7. During an experiment on animals the nervous tracts leading to the crus of hypophysis were
broken. That impaired the entrance of the fallowing hormones into the blood.
Hypophysis hormones
I= |F 10 Ie |>

Vasopressin and oxytocin


Adenohypophysis hormone
Thyrotropic hormone
Adenocorticotropic hormone

8. Acareless student suddenly meets his dean. What hormone concentration will be increased in
student’s blood more rapidly?
A. Somatotropin
B. Hydrocortisone
CG Corticotropin
D. Thyroliberine
E. Adrenalin

9. A women aged 44 complains of general malaise, pain in the heart area, and considerable
increase of body weight. At examination of this patient following symptoms are revealed: lunar
face, hirsutism, BP 165/100 mmHg, height is 164 cm, weight is 103kg, the adipose deposits
mainly on the neck, shoulders, abdomen. What is the main pathogenic mechanism of the
woman’s obesity?
Decrease of thyroid hormone production
I= |F 10 Ie |>

Decrease of glycogen production


Increase of glucocorticoid production
Increase of insulin production
Increase of minerelocorticoid production.

10. A child has a time disturbance of teeth eruption, enamel anomalies, and its lips and tongue are
enlarged. What hormone deficiency causes these changes?
Thyroxin
I= |F 10 Is |>

Parathormone
Thyrocalcitonin
Insulin
Somatotropin
11. At clinical examination of a woman it was determined increase of basal metabolism rate by
40%, increased perspiration, tachycardia, and leanness. What endocrine gland functions are
impaired and what direction in?
Sexual glands, hypofunction

I= |S 10 I [>
Cortical substance of adrenal glands, hyperfunction
Medulla of adrenal glands, hyperfunction
Thyroid gland, hyperfunction
Pancreatic gland, hyperfunction
12 . A young man an aged 17 has the signs of retention of sexual development; his height is 184 cm;
he is asthenic; his secondary sexual characters are weakly developed. Sex chromatin is absent
in this patient’s cells. At the age of 4 he endured epidemic parotitis. What was the cause of
hypogonodism?
A. Deficiency of gonadoliberine
B. Deficiency of gonadotropin
C Cryptorchism
D. Orchitis
E. Chromosomal anomalies (XXY)

13. A patient aged 20 complains of polydipsia and abundant urination (up to 10 liters for 24 hrs);
concentration of glucose in his blood is normal; in his urine glucose is absent. What hormone
absence may cause such clinical picture?
Oxytocin
IF |F Oe>

Vasopressin
Insulin
Hydrocortisone
Tniodothyronine

14. A woman with primary hyperparathyroidism has periodically repeated attacks of renal colic.
Ultrasound examination shows presence of small stones in her kidneys. What is the cause of
the formation of these stones?
Hypercholesterolemia
I= |S 10 I [>

Hyperuricemia
Hyperkalemia
Hyperphosphatemia
Hypercalcemia

15. Substances which imitate morphine effects but which are produced in CNS may be used for
analgesia. Indicate them:
Somatotropin
IS | I [>

Oxytocin
Beta-endorphin
Vasopressin
E. Calcitonin

16. Dilated pupils, dryness of mucous membranes, accelerated heart rate, constipation, and tremor
are observed in some people during emotional exertion. What system activation will lead to
such changes in the organism?
Pituitary-adrenal system
I= |F 10 Ie |>
Sympathetic nervous system
Sympathoadrenal system
Somatic nervous system
Parasympathetic nervous system

17. In ancient India those people who were suspected in committing crimes were given swallow a
handful of dry rice. Criminals couldn’t swallow rice due to decreased salivation
(oligosalivation) caused by:
Decrease of blood supply of salivary glands
I= |F [0 I |>

Activation of sympathoadrenal system


Activation of parasympathetic nucleus of glossopharyngeal nerve
Inhibition of sympathoadrenal system
Activation of parasympathetic nucleus of facial nerve

18. A patient aged 23 complains of headache, changes of his appearance (increase of size of feet,
fingers, features of the face), hoarseness of the voice, worsening of the memory. The disease
has begun 3 years ago without any causes. Increase of superciliarry arches, nose, and tongue
were found during physical examination of this patient; the blood sugar was normal; the
urinalysis is out of changes. What is the cause of such condition?
Excessive production of corticosteroids
I= |F 10 Ie |>

Excessive production of somatotropin


Insufficiency of aldosterone
Insufficiency of glucagon
Insufficiency of thyroxin

19. Adrenalin is used to prolong Novocain action in infiltration anesthesia. What adrenalin action
is this effect connected with?
Potentiation of Novocain action at the level of CNS
IF |Ol >

Inhibition of function of nerve ending and conductors


Dilatation of vessels
Constriction of vessels
Inhibition of tissue esterases

20. A patient aged 80 complains of increased appetite, thirst, excretion of large amount of urine,
and worsening of general condition after taking sweet food. What disease is it?
A, Hypercorticoidism
Hyperthyroidism

IF |S 10 I=
Hypothyroidism
Diabetes insipidus
Diabetes mellitus

21. Persistent hyperglycemia developed in a patient with Cushing’s syndrome under the influence
of excessive amount of glucocorticoids.
What is the mechanism of hyperglycemia in this case?
Activation of glycogenolysis at the liver
I= |F [0 I |>

Activation of gluconeogenesis
Activation of insulinaze of the liver
Decrease of hexokinase activation
Increase of absorption of glucose in intestine

22. A man aged 38 is in stress state due to industrial conflict. Which of below mentioned hormones
participate in starting stress reaction of the organism?
A. Adrenalin
B. ACTH
C Glucagon
D. Hydrocortisone
E. Thyroxin

23. A woman aged 28 addressed to a doctor a month later after delivery with complaints of
decreased breast milk formation. What hormone insufficiency caused this condition?
ACTH
IF |S [0 Ie |>

Glucagon
Insulin
Prolactin
GH

24, While examining a patient a doctor suspected Cushing’s syndrome. What substance
determination in patient’s blood will prove doctor’s supposition?
A. Tocopherol
B. Adrenalin
Cc. Cholesterol
D. Retinol
E. Cortisol

25. The attacks of convulsions appeared in a patient after resection of thyroid gland. What
preparation must be administered in this case?
A. Somatotropin
B. Insulin
Thyroxin

IF [FIA
Prednisolon
Parathyroidin

26. A dog with endocrine pathology had decreased oxygen usage in the state of rest, decreased
body temperature, and decrease of glucose tolerance. What hormone insufficiency may explain
the discovered changes?
Growth hormone
I= |F 10 ls [>

Thyroxin
Gonadotropin
Adrenocorticotropic hormone
Insulin

27. A patient aged 50 complains of thirst, he drinks much water, has marked polyuria. His blood
glucose - 4.8 mmol/L, glucose and acetone bodies are absent in the urine; urine is colourless,
its specific gravity - 1.002-1.004. What is the cause of polyuria?
Insufficiency of ADH
IF |F 10 I |>

Hypothyrosis
Insulin insufficiency
Aldosteonism
Thyrotoxicosis

28. A patient aged 40 was hospitalized with complaints of general malaise, convulsions of upper
and lower extremities, his BP - 160/100 mm Hg. The results of laboratory examination of him:
blood glucose - 6.5 mmol/L, calctum - 2 mmol/L, phosphates - 1 mmol/L, sodium-160
mmol/L. Urination-700 ml for 24 hours. What pathology causes such state?
Hyperaldosteronism
I= |F 10 I |>

Hypoaldosteronism
Hyperparathyroidism
Thyrotoxycosis
Rickets

29. Pilosis by male type and increase of muscular mass began appearing in a boy of 5 years old
who had developed previously without declination from the age norms. At examination of the
patient marked secondary male sexual signs were revealed, but the size of his testes
corresponds to his age. What is the cause of precocious puberty?
Androgen producing adrenal tumor
I= |F 10 I [>

Hormone producing testis tumor


Increase of gonadotropin production
Increase of adrenocorticotropin production
Increase of gonadoliberin production
30. Why must a patient, who has taken prednisolone for rheumatoid arthritis for a long time, avoid
contacts with infectious patients?
A. Because of the development of secondary
immunodeficiency
B. Because of risk of exacerbation of arthritis
Cc. Because of risk of thromboembolic complications development
D. Because of the development of lymphopenia
E. Because of the blockade of interferon formation
31. Liquidator of an accident at Chernobilskaya AES began complaining of increased excitability,
nervousness, heartbeat, decrease of body weight, constant weakness, body tremor, feeling of
fever, bad heat endurance. What gland hyperfunction may be the cause of such state?
Thyroid gland
I= |F 10 I |>

Adenohypophysis
Adrenal gland
Mediulla of adrenal gland
Parathyroid gland

32. Muscular weakness, adynamia, decrease of body temperature, hypoglycemia, developed in a


dog after two-sided resection of adrenal gland. What other manifestation of adrenal
insufficiency may be noted?
A. Lymphopenia
B. Arterial hypotension
C. Increase of glycogen synthesis
D. Hypermatriemia
E. Hypokalemia

33. A woman complains of increased irritability, perspiration, weakness, loss of body weight,
tremor of extremities, increased heartbeat rate, and exophthalmia. What metabolic impairment
in the organism accompanies this disease?
A. Increase of adenosine triphosphoric acid synthesis
B. Decrease of organism sensitivity to hypoxia
C. Weakening of phospholipase activation
D. Increase of basal metabolism
E. Decrease of cholesterol lysis

34. A patient who suffered from severe thyrotoxicosis had been operated on for strumectomy after
that weakness, sensitivity to cold, increase of body weight, paleness and dryness of skin
developed. What are the other manifestations of hypothyrosis?
A. Inhibition of CNS activity
B. Tachycardia
C. Increase of basal metabolism
D. Increase of intestinal peristalsis
E. Decrease of tolerance to carbohydrates

35. In case of hypercortisolism — Cushing’s disease — the following changes in the organism take
place:
A. Development of cachexia
B. Impoverishment of the liver with glycogen
C. Hyperglycemia
D. Hypotension
E. Lymphocytosis

36. A patient was admitted to the hospital with complaints of loss of weight, quick fatigability,
darkening of skin. His heart sounds are dull.
What are the other manifestations of adrenal insufficiency?
A. Lymphopenia
B. Adynamia
C. Arterial hypertension
D. Increase of minute blood volume
E. Increased appetite

37.A patient admitted to the hospital complains of quick fatigability, loss of weight,
hyperpigmentation of the skin. Her heart sounds are dull. She has pulse rate - 96 beats per
minute and BP - 90/50 Hg. What metabolic impairments are observed in hypocortisolism?
Hyperkalemia
I= |F 10 I [>

Increase of glycogen synthesis


Hypernatremia
Hyperhydration
Hypoglycemia

38. The manifestations of hypoparathyrosis developed in a patient after stramectomy. What


changes in the organism are observed in this case?
Hypophosphatemia
I= |F 10 I >

Resorption of bone tissue


Acidosis
Hypocalcaemia
Decrease of neuromuscular excitability

39. Small height, disproportional development of the body, and insufficient mental development
were found in a boy of 10 during examination. What hormone deficiency caused these
changes?
A. Thyroxin
B. Parathormone
CG Thyrocalcitonin
D. Adenocorticotropic hormone
E. Oxytocin

40. With the help of indirect calorimetrical measurement it was determined that the basal
metabolism of the patient was 40% lower than the proper one. What endocrine gland
hypofunction is the cause of described changes?
Adrenal glands
I= |F 10 Ie |>

Thyroid gland
Epiphysis
Thymus
Pancreas

41. After suffered sepsis a bronze color of the skin typical for Addison’s disease appeared in a
woman aged 27. Hyperpigmentation occurs due to the increased secretion of:
Melanocytstimulating hormone
I= |F 10 I >

Growth hormone
Adrenocorticotropic hormone
B-lipotropic hormone
Thyrotropic hormone

42. A patient with rheumatic arthritis was given hydrocortisone for a long period of time. He
developed hyperglycemia, polyuria, glycosuria, and thirst. These complications after treatment
develop due to the activation of the process of:
Gluconeogenesis
I= |F 10 I |>

Glycogenolysis
Glycogenesis
Glycolysis
Lipolysis

43. Fibrillary muscular tics of arms, legs and face appeared in a woman aged 46 after the operation
on thyroid gland. These disorders may be eliminated by injection of:
Parathyroidin
I= |F 10 I [>

Triiodthyronine
Thyrotropin
Thyroxin
Calcitonin.

44, Atrophy of the testis developed in an athlete who systemically used androgen hormones. This
phenomenon is due to the inhibition of secretion of:
A, Gonadotropic hormone
B. Corticoliberine
CG Prolactoliberin
D. Gonadoliberine
E. Testosterone.

45. During the examination of the patient a doctor found out Cushing’s disease that is characterized
by obesity. It is connected with:
A. Excessive use of fats with meal
B. Impairment of ventromedial nuclei of hypothalamus
C Inhibition of adrenalin synthesis by adrenal glands D. Production of
excessive amount of glycocoticoids
E. Hereditary tendency to hyperlipemia.

46. A patient with diffuse toxic goiter has marked exophthalmia. The appearance of exophthalmia
is explained in this pathology by:
Antibody circulation against thyroglobulin
I= |F 10 I [>

Presence of immunoglobulins in the blood


Production of exophthalmia factor by hypophysis
Increase of adrenorecpetor sensitivity to catecholamines
Pathogenic action of thyroid prostaglandins.

47. A patient with myxedema came to a doctor. Her face was puffy with poor facial expression;
she had thickened nose and lips. These signs can be explained by:
Free filtration of sodium in glomeruli
I= |S 10 I [>

Accumulation of hydrophilic mucous substances


Impairment of sensitivity of volumo- and osmoreceptor
Increase of sodium reapsorption in tubules (canaliculi)
Increased permeability of capillary walls.

48. After physic exertion a patient with pheochromocytoma complains of tachycardia, increased
arterial pressure, and sharp pain in epigastric area.
These attacks may be explained by:
Massive release of catecholamines by adrenal glands
I= |F 10 I |>

Release of norepinephrine by sympathetic nerves


Activation of hypothalamus vegetative nuclei
Increase of thyroid hormone secretion
Increased synthesis of adrenocorticotrophic hormone

49, Excessive secretion of a certain hormone was observed in a patient with pheochromocytoma -
a tumor, which appears from the medulla of adrenal glands. How is this hormone called?
A. Glucagon
B. Insulin
Cc. Thyroxin
D. Adrenalin
E. Somatotropin

50. Adenoma, which grows from the cells of glomerular zone of adrenal cortex and is the source
of excessive formation of aldosterone was found in a patient. This resulted in a development
of primary hyperaldosteronism or Conn’s disease. What electrolyte exchange does this
hormone influence?
Ferric ion
I= |F 10 I [>

Calcium ion
Magnesium ion
Chlorine ion
Sodium ion
51. Weakness, thirst, sharp increase of neuromuscular excitability with the development of
parathyroid tetany were observed ina dog 1-2 days later after resection of thyroid glands. What
disturbance of electrolyte exchange takes place in this condition?
A. Hypercalcaemia B. Hypocalcaemia
C. Hypomagnemia
D. Hypermagnemia
E. Hyponatremia

52. Removal of cancer tumor of the testis in a patient before the period of sexual maturity resulted
in the development of eunochoidism and was accompanied by the deficiency of the production
of:
Androgens
I= |F 10 I |>

Estrogens
Kinines
Prostaglandins
Cytokines

53. Male patient aged 55 had an increase of pituitary gland dimensions and hyperplasia of adrenal
cortex. At examination of the patient: BP-190/90 mm Hg, content of blood glucose is 20
mmol/L; there are glucosuria, obesity, and histurism. What pathology are these changes typical
for?
Barraker-Simmond disease
I= |F 10 I >

Adipose-genital dystrophy
Cushing’s syndrome
Addison’s disease
Cushing’s disease

54. A patient aged 29 had acute massive blood loss during delivery. Then the following changes
developed: acute loss of weight, atrophy of skeletal muscles, thinning of skin, decrease of body
temperature, hypotension, and hypoglycemia. What pathology of pituitary gland is the most
possible in this case?
Sheehan’s disease

I= |F 10 I |>
Parhon’s syndrome
Diabetes insipidus
Adipose-genital dystrophy
Pituitary dwarfism

55. A 28-years-old patient complains of flaccidity, quick mental and physical fatigue, and
dyspeptic disorders. During the examination following was found out: positive TB tests,
hypoglycemia, BP - 90/60 mm Hg, hyponatremia, skin pigmentation. What disease of adrenal
glands underlies observed symptoms?
Cushing’s syndrome
IF |F Ol>

Addison’s disease
Acute insufficiency of adrenal cortex
Hypofunction of medullar layer of adrenal glands
Conn’s syndrome

56. Which of the signs that develops in hyperthyrosis is the most important for making diagnosis?

Tachycardia
I= |F 10 Ie |>

Subfebrile temperature
Increase of basal metabolism
Increase excitability
Disorder of sleep

57. A patient aged 41 complains of weakness, sweating, fever, tremor of hands, BP-160/90 mm
Hg. Diffuse toxic goiter was diagnosed (Basedow’s diseas). What is the main mechanism of
impairment of the function of cardio-vascular system in this disease?
Decrease of tonus of sympathetic nervous system tonus
10 Ie [>

Increase of tonus of sympathetic nervous system tonus


IS

Auto immune reactions


Increase of catabolism
E. Hyperthermia
58. A girl aged 5 had the symptoms of premature sexual development, menses, growth of
mammary gland, adipose deposits in the field of pelvis and femur. The most possible cause of
premature sexual development is:
Adrenal adenoma
I= |F 10 I [>

Hypopituitarism
Polycystic ovary syndrome
Hyperplasia of adrenal glands
Hormone- active tumor of ovaries
59. Conn’s syndrome was diagnosed in a patient who complained of muscular weakness, increased
urination (at night), and increased arterial blood pressure. What is typical for this syndrome?
Increase of renin, increase of aldosterone, and increase of potassium

I= |F 10 Ie |>
Decrease of renin, increase of aldosterone, and increase of potassium
Decrease of renin, increase of aldosterone, and decrease of potassium
Decrease of renin, decrease of aldosterone, and decrease of potassium
Increase of renin, decrease of aldosterone, and increase of potassium

60. A female patient aged 44 complains of general malaise, large increase of body weight, growth
of hair on the face, arrest of menses, BP-165/100 mm Hg. What diagnostic test will help to
differentiate Cushing’s disease from Cushing’s syndrome?
Level of ACTH in blood plasma
i>

Level of cortisol in plasma


Excretion of 17-oxyketosteroids with urine
IF |S lO

X-ray of the skull


Number of eosinophils in blood
ol. A patient aged 42 complains of sharp loss of body weight, weakness, appearance of pigmental
spots on the skin and mucus membranes. Addison’s disease was diagnosed in this patient.
Which of the following tests is the most important for diagnostics of Addison’s disease?
Content of 17-oxyketosteroids in urine
I= |F 10 I |>

Level of potassium and sodium in blood plasma


Tests with injection of ACTH
Level of cortisol in plasma
Content of blood glucose

62. A 55-years-old man had an attack of angina pectoris which w as accompanied by a severe pain
behind the breastbone and resulted in anuria. What mechanism influenced on the appearance
of anuria?
Excessive somatotropin secretion
I= |F [0 I |>

Excessive vasopressin secretion


Decrease of corticotropin secretion
Vasopressin deficiency
Somatotropin deficiency

63. Enlargement of thyroid gland, aloofness, increased basal metabolism, tachycardia, tremor of
fingers were found out in a women aged 52. Psychic excitability was increased in her. The
leading part of pathogenesis of this state is:
Increase of mitochondrial membrane permeability
IF |F 10 I |>

Negative nitrogen balance


Hypocholesteremia
Decrease of protein synthesis
Hyperglycemia
64. Thyrotoxicosis was produced in an animal experimentally. For confirming this state it is
necessary to determine the level of metabolism according to food taking and physical load.
What experimental method must be used in this case?
Determination of iodine content in the thyroid gland

I= |F 10 Ie |>
Determination of concentration of organic iodine in blood
Direct calorimetry
Determination of oxygen tension in the blood
Determination of the content of the gland hormone in the blood

65. Three months later after delivery female patient C., aged 30, began to complain of increase of
body weight, excessive adipose deposits, mainly in the area of the neck and face, growth of
hair on the upper lip. The increase of size of cella turcica was determined by X-ray
examination. What pathogenic mechanism underlies determined pathology?
Hypersecretion of glucocorticoids
IF |F 10 I |>

Increase of food taking


Hyposecretion of thyroxin
Development of diabetes mellitus
Birth of a child

66. A boy of 14 years old visited endocrinologist. His mother complains of his being behind in
physical development and growth. The boy has proportional constitution, his is 104 cm tall,
and his secondary sexual characters are not marked. The cause of this pathology is
hyposecretion of:
A. Gonadotropic hormone
B. Growth hormone
Cc. All hormones of adenohypophysis
D. ACTH
E. Thyroid stimulating hormone

67. Patient female aged 47 was admitted to the hospital with complaints on increased nervous
excitability disorder of sleep, heartbeat, pain in the heart area, and muscular weakness.
Objectively: the patient is thin, has exophthalmia, tremor of hands, increase of reflexes. She
has body temperature - 37.50C, heartbeat - 150 beats per minute; at her ECG sinus tachycardia
was revealed. Thyroid gland is increased. Arrhythmia in this patient develops is due to:
Increase of tonus of sympathetic nervous system
I= |F 10 Ie [>

Decrease of tonus of vagus nerve


Intoxication
Metabolic increase in myocardium
Increase of catchecolamine level

68 . A 51-years-old male patient has been suffering from tuberculosis for 10 years. He is abusing
alcohol. At last time, complaints of irritability, fast fatigability, syncope, muscular weakness,
and loss of weight appear in him. At examination of the patient: hyperpigmentation of his skin;
his BP is 90/60 mmHg. What is the most possible reason for disease development in this
patient?
Chronic alcohol intoxication

I= |F [0 I |>
Impairment of adrenal gland due to tuberculosis
Alcoholic injury of the liver
Hypothalamic tumor
Thyroid gland impairment

69. A woman aged 40 complains of general malaise, pain in the heart area, increase of body
weight. At examination of the patient: she is 164 cm height and 104 kg weight; she has
predominant deposits of fat at face, neck, shoulders, and abdomen; her BP — 165/100 mmHg;
content of glucose in her blood — 7.8 mmol/L. What endocrine gland function is infringed and
what direction at?
Pituitary gland, basophilic adenoma, hyperfunction
I= |F 10 I |>

Pituitary gland, eosinophilic adenoma, hyperfunction


Sexual glands, hypofunction
Cortex of adrenal glands, hypofunction
Thyroid gland, hypofunction
70. In a patient
t aged 23 BP rapidly drops down to 70/40 mmHg after surgical removal of left
adrenal gland. Patient has rapid, weak pulse, vomiting, and cramps; he is covered by cold
sweat. What is the most possible cause for this condition?
Hypertrophy of the right adrenal gland
IF |F 10 Ie |>

Hypofunction of adenohypophisis
Atrophy of the right adrenal gland
Hyperfunction of adenohypophisis
Hypofunction of neurohypophisis

71. A 17-years-old female patient, who has been ill for 6 months, complains of appearance of
mustache and beard, roughness of voice, and absence of menses. At examination of the patient
hirsutism and undeveloped mammal glands and sexual organs were revealed. What glands
impairment has lead to this pathology?
Adrenal glands
IF |F 10 I |>

Thyroid gland
Neurohypohpisis
Adenohypophisis
Sexual glands

72. A child aged 4 lags behind at physical development, sleeps badly, and has increased irritability,
thirst, and polyuria. Glucose is absent in patient’s urine. Patient has negative reaction to
vasopressin injection.
What is the mechanism of revealed disorders?
Disturbance of realization of hormone biological activity

I= |F 10 I |>
Impairment of neural regulation of endocrine glands
Disorders of biosynthesis and secretion of hormones
Disorders of neuroendocrine regulation
Disorders of feedback mechanisms

73. Loss of body weight, fatigue, pulling teeth out, and coming hairs out began in a patient in
several months after delivery. She often became ill. At examination of the patient:
subcutaneous adipose tissue is virtually absent; BP — 90/55 mmHg; body temperature — 36.0
oC. Biochemical analysis of patient’s blood: glucose content — 3.0 mmol/L. There is increased
level of growth hormone and adrenocorticotropic hormone in her blood. What kind of pituitary
functions is impaired in this patient?
Pituitary dwarfism
IF |F [0 I |>

Panhypopituitarism
Acromegaly
Cushing’s disease
Diabetes insipidus

74. Hans Sallie, author of doctrine of stress, showed that different stimuli (heat, cold, pain) always
evoke standard non-specific reaction: 1) involution in thymico-lymphatic system; 2)
hemorrhage acute ulcers of stomach and duodenum. Name the third compound of classical
triad.

Hypertrophy of adrenal cortex


IF |F 10 I [>

Hypertrophy of beta cells of Langerhans’ islets


Myocardial infarction
Parodontitis
Cerebral hemorrhage

75. Polyuria and polydipsia developed in a 40-years-old patient accordingly to impairment of


hypothalamic-pituitary connective way.
What hormone deficiency evokes this changes?
Oxytocin
I= |F 10 I >

Antidiuretic hormone
Adrenocorticotropic hormone
Growth hormone
Thyroid stimulating hormone

76. In a patient excessive development of bones and soft tissues of his face, enlargement of his
tongue, and widened interspaces between teeth in enlarged teeth arch were found out. What
changes of hormone secretion are the most possible in this patient?
A, Elevated secretion of insulin
Increased secretion of thyroxin

IF [9 19I
Increased secretion of growth hormone
Reduced secretion of growth hormone
Elevated secretion of vasopressin

77. A boy aged 9 is in endocrinology department because of increase bone fragility. What
endocrine organ is impaired?
Adrenal glands
I= |S 10 I [>

Thymus
Thyroid gland
Parathyroid gland
Epiphysis

78. A child has infringements of teeth enamel and dentin formation because of reduced content of
calcium in his blood. What hormone deficiency may evoke these changes?
Growth hormone
IF |F ll>

Triiodthyronin
Parathormone
Thyroxin
Thyrocalcitonin

79. By X-ray examination of scull bones enlargement of cella turcica, thinned anterior inclined
appendices, and destruction of different places of cella turcica were revealed. What endocrine
gland tumor may lead to such bone destruction?
Thyroid gland
IS [9 IF>

Epiphysis
Adrenal gland
Thymus
Pituitary gland
=

80. After the operation on thyroid gland tetany developed in a patient that manifested in
convulsions of striped muscles. What does this complication may be connected to?
Mistaken removal of parathyroid glands
I= |F 10 I |>

Insufficient removal of thyroid tissue


Hyperfunction of adrenal glands
Hypofunction of adrenal glands
Elevated blood content of thyroid-stimulating hormone

81. A female patient with hyperthyroidism has increased body temperature. What is the
mechanism of body temperature elevation?
Separation of oxidation and oxidative phosphorilation
>

B. Increase utilization of glucose by tissues


Cc. Intensification of glycogenolysis
D. Intensification of protein catabolism
E. Elevation of oxidation of fat in liver

82. A woman has fatigue, sleepiness, apathy, worsening of memory, and edemas. These symptoms
appeared in several years after change of dwelling. “Endemic goiter” is diagnosed in this
patient. What substance deficit in water and food may lead to this disease?
Iodine
I= |F 10 I |>

Fluoride
Tron
Calcium
Magnesium

83. A 38-years-old patient complains of thirst (he drinks 8 liters of water daily), polyuria, loss of
weight, and general malaise. He has been sick for 6 months. Urinalysis of the patient shows:
special gravity — 1.001, leukocytes — 1 to 2 in field of vision, protein — trace amount. What
reason evokes the constant polyuria in this patient?
Reduction of ADH production
I= |F 10 I |>

Injury of renal glomerules


Injury of renal tubules
Elevation of osmotic pressure of urine
Increase of oncotic pressure of urine

84. When stomatological examining the patient aged 18 the following symptoms were revealed:
woman features of his face, anomalies of form of crowns in some teeth, catarrhal gingivitis,
and resorption of interdental junctures. Cytological examination of mucous membrane
epithelium discovers that cells contain 1 Barr’s body. What endocrine pathology are these
manifestations the most probably connected to?
Hypogonadism
I= |F 10 I |>

Hypergonadism
Cretinism
Thyrotoxicosis
Chronic hypoparathyroidism

85. A 20-years-old patient has hypoplasia of enamel and impaired formation of dentin in his teeth
after surgical removal of thyroid gland.
What hormone deficiency became the reason for indicated pathology?
Parathormone
I= |F 10 I [>

Thyroxin
Thyroid-stimulating hormone
Thyroid-stimulating hormone-releasing factor
Thyrocalcitonin
86. A patient, who lives in mountain region, was admitted to the hospital with diagnosis of
endemic goiter. What is characteristic for this disease?
Decreasing of thyroxin and triiodthyronin synthesis

IS [0 IF [>
Increase of thyroid hormone production
Toxic action of thyroxin and triiodthyronin
Chronic hypoparathyroidism

87. A child aged 2.5 has retardation in physical development, absence of appetite, thirst, and
polyuria; he sleeps badly. Sugar was not found in the patient’ urine. Which of endocrine
pathologies is the reason for disorders of water-salt metabolism?
Hyposecretion of antidiuretic hormone
IF |F 0 I >

Hypersecretion of antidiuretic hormone


Secondary hyperaldosteronism
Hyposecretion of growth hormone
Hyposecretion of adrenocorticotropic hormone

88. What pathological conditions the tumor growing from reticular zone of adrenal gland may lead
to?
Virilization of female organism
IS [9 I |>

Elevation in content of glucocorticoids in blood


Tachycardia
Elevation of content of mineralocorticoids in blood

89. What changes result from separation in oxidation and oxidative phosphorilation?
Elevation in heat production
I= |S 10 I [>

Decrease of free oxidation part


Decrease in heat production
Enhancement of functional activity of organs
Increase in ATP production

90. What changes in adult organism may be evoked by hypersecretion of growth hormone?
Development of diabetes mellitus
I= |S 10 I [>

Intensification of lipolysis
Intensification of urea production
Reduction of transport of amino acids to the cell
Increase in content of calcium in blood

91. A 40-years-old patient complains of decreased capability of working, sleepiness, sensitivity to


cold, fragility and coming out of hairs, dryness of skin, and edema of face and extremities.
There is no formation of pit under the pressing at the anterior surface of patient’s calves. Patient
has slow tendon reflexes and bradicardia. What infringement do these changes result from?
——__ A, —__ Hypothyroidism
Adrenal insufficiency

IF |S 10 I=
Hypoparathyroidism
Vitamin C deficiency
Iron deficiency

92. Influence of unfavorable psycho-emotional factor upon a patient results in development of


non-specific pathological process, which course consists of 3 stages: 1) anxiety reaction; 2)
stage of resistance; 3) stage of exhaustion. This process involves system hypothalamus —
pituitary — adrenal cortex. What is the name for this process?
A. Stress
B. Parabiosis
C Adaptation
D. Compensation
E. Dominance

93. A doctor suspects the hypothyroid goiter in a patient with enlarged thyroid gland. What sign
has a decisive importance for establishing the diagnosis?
A. Decrease of basal metabolism
B. Bradicardia
CG Edemas
D. Arterial hypotension
E. Hypodynamism

94. Arterial hypotension, muscular weakness, and periodic convulsions appear in the patient with
hepatic cirrhosis. Content of sodium is increased and content of potassium is decreased in
patient’s blood. What kind of endocrine disorders underlies this symptoms?
Secondary hyperaldosteronism
I= |S [0 I |>

Hypopituitarism
Primary hyperaldosteronism
Hyperpituitarism
Hypoaldosteronism
95. Prolonged intake of mineralocorticoids led to development of muscular weakness in a patient.
What gives rise to these symptoms?
Hypokalemia
I= |S 10 I [>

Hyperkalemia
Hypernatremia
Hyponatremia
Hypervolemia

96. What hormone increased secretion causes becoming thin at the period of enhanced growth?
A Growth hormone
B. Prolactin
Glucagon

IF [FIA
Progesterone
Insulin

97. What kind of endocrine disorders lead to obesity?


Hypercortisolism
I= |F 10 I |>
Hypofunction of adrenal glands
Hyperparathyroidism
Hypergonadism
Hyperaldosteronism

98. A 50-years-old female patient, who was operated on thyroid gland for diffuse toxic goiter,
begins complaining that she is spiritless and slow, has fast fatigability, increased working
ability, sleepiness, worsening of memory, and increasing of body weight. At examination the
patient has dry skin, edematous face, and striped nails with broken edges. What is your
assumable diagnosis?
Myxedema
IF |F 10 I |>

Cushing’s disease
Acromegaly
Obesity
Thyrotoxicosis

99. A 25-years-old male patient, who endured influenza, complains of increased thirst (he drinks
5 to 6 liters of water daily), frequent abundant urination, and loss of body weight. Patient has
dry mucous membranes and dry skin with reduced turgor. Urinalysis of this patient displays:
urine is colorless, its special gravity — 1.000-1.004, leucocytes and erythrocytes — 1 to 2 in field
of vision. What pathology does this patient suffers from?
Diabetes insipidus
I= |F 10 Is |>

Diabetes mellitus
Conn’s disease
Pheochromocytoma
Secondary hyperaldosteronism

100. When prophylactic examining of the first-year pupils in two of them lag in their height by
2-3 sigma deviations was indicated. No another disorders were revealed. What hormone
deficiency could lead to this pathology?
Growth hormone
I= |F 10 I [>

Sexual hormones
Insulin
Thyroid hormones
Glucocorticoids
ANSWERS:
1.-A2.-E3.-C4.-E5.-D6.-C7.-B8.-E9.-C10.-Al1.-D12.-D13.-B14.-E15.C16.-C17.-B18.-B19.-
D20.-E21.-B22.-A23.-D24.-E25.-E26.-B27.A28.-A29.-A30.-A3 1.-A32.-B33.-D34.-A35.-C36.-
B37.-E38.-D39.A40.-B41.-A42.-A43.-A44.-A45.-D46.-D47.-B48.-A49.-D50.-E51.B52.-A53.-
E54.-A55.-B56.-C57.-B58.-E59.-C60.-A61.-C62.-B63.A64.-C65.-A66.-A67.-A68.-B69.-A70.-
C71.-A72.-A73.-B74.-A75.-76.C77.-D78.-C79.-E80.-A8 1.-A82.-A83.-A84.-A85.-A86.-A87.-
A88.A89.-A90.-A91.-A92.-A93.-A94.-A95.-A96.-A97.-A98.-A99.-A100.-A
Topic. 17. Pathophysiology of blood and hemopoietic organs.

1. Anemia, leuko- and thrombocytopenia, color index-1.3, presence of megaloblasts and


megalocytes were determined in the laboratory analysis of blood of a patient a year later after
he was operated on for subtotal resection of the stomach for the ulcer of lesser curvature of the
stomach. What factor deficiency results in these changes?
Gastromucoprotein
I= |F 10 I >

Gastrin
Pepsin
Chlorine hydrate
Mucin

Amino acids replacement in alpha and beta chains of hemoglobin takes place in a number of
hemoglobimopathies. Which of them is typical for HbS (sickle-cell anemia)?
Glycine to serine
I= |S 10 Ie [>

Aspartate to lysine
Methionine to histidine
Glutamate to valine
Alanine to serine

Hereditary microspherocytic hemolytic anemia (Mincovsky-Shoffar disease) was diagnosed


in a woman aged 34. What mechanism caused hemolysis of erythrocytes in the patient?
Enzymopathy
I= |S 10 I [>

Hemoglobinopathy
Autoimmune impairment
Membranopathy
Hypoplasia of bone marrow

Megaloblastic anemia was diagnosed in a patient. What substance deficiency may cause the
development of this disease?
Cyanocobalamin
I= |F 10 Is |>

Cholecalciferol
Magnesium
Glycine
Copper

Three years ago a man aged 45 was operated on for stomach resection. After the operation the
content of erythrocytes in the blood is 2.0x1012, Hb 85 g/l, color index-1.27. What vitamin
absorption is impaired that causes the change of erythropoiesis?
A C
BP
Gc A
A patient, carrier of hereditary sickle-cell erythrocytes anomaly, had pneumonia accompanied
by hemolytic crisis and development of anemia. What is the main cause of hemolytic crisis in
this case?
A. Hyperoxia
B. Heterozygosis HbS
CG Mutation of structural gene
D. Hypoxia caused by pneumonia
E. Blood osmolarity change

Examining the oral cavity of a patient, a dentist paid attention to the presence of inflammatory-
dystrophy process in the mucous membrane (Gunter’s glossitis, atrophic stomatitis). Blood
analysis revealed hyperchromic anemia. What factor is a cause of this disease?
Hypovitaminosis Bs
I= |S 10 I [>

Hypovitaminosis A
Increase of stomach juice acidity
Hypovitaminosis Bi

A female patient complains of malaise, weakness, breathlessness, rapid fatigability, and


dizziness. Her blood test data: erythrocytes-1.8x1012/L, Hb-80 g/L, leukocytes-3.2x100/L,
color index-1.5. Anisocytosis, poikilocytosis, megaloblasts, megalocytes were found in smear.
What is the possible diagnosis?
Bi2-deficiency anemia
I= |F 10 I |>

Posthemorragic anemia
Acute leukemia
Iron deficiency anemia
Immunohemolytic anemia

A patient, aged 50, complains of a bad appetite, loss of weight, weakness, pain in the stomach
area, and eructation. At laboratory examination of him: Hb-2x1012/L, stomach secretion 0.4 1,
pH of stomach juice-7. Pernicious anemia is diagnosed in this patient. What compound of
gastric juice deficiency is the cause of the disease?
A. Pepsin
B. Renin
Cc Secretin
D. HCL
E. Castle’s factor
10. Funicular myelosis and hyperchromic anemia developed in a man 7 years later after the
stomach resection due to ulcer. What pathogenic mechanism of changes in spinal cord is the
most possible one?
Hypoxia impairment in anemia

I= |F [0 I |>
Accumulation of methylmalonic acid in cyanocobalamin deficiency
Impairment of DNA in cyanocobalamin deficiency
Deficiency of folic acid
Deficiency of iron containing enzyme

11. On the seventh day after hemorrhage caused by a trauma, the following was revealed in a
patient’s blood: erythrocytes - 2.8x 1012/1, Hb - 3.9mmol/I, reticulocytes - 15%, acidophilic and
polychromatophilic normoblasts were found in smear. What is the mechanism of appearance
of regenerative forms of blood erythrocytes?
Going out of blood deposition
IF |F 10 Ie I>

Increase permeability of erythrocyte sprout of bone marrow


Intensification of regeneration of erythroid cells in bone marrow
Inhibition of maturation of erythroid cells in bone marrow
Inhibition of synthesis of erythroprotein inhibitor

12. When treating a patient for malaria with primachine, hemolysis occurs in him. At examination
deficiency of Glucose-6-phosphate Dehydrogenase in patient’s erythrocytes was revealed.
What metabolic process is impaired in this patient?
A. Aerobic carbohydrate oxidation
B. Glycogenesis
CG Glycolysis
D. Gluconeogenesis
E. Pentose-phosphate pathway

13. Bi2-folate deficient anemia was diagnosed in a patient aged 55. What hematological index will
be the most sensitive for pathogenetic treatment?
Increase of number of erythrocytes
I= |F 10 Ie |>

Increase of number of reticulocytes


Reduction of anisocytosis
Decrease of ESC
Decrease of amount of hemoglobin

14. In a pregnant woman, in the 7th month of pregnancy, anemia begins rising fast. In her blood
test: number of erythrocytes is 2.7x1012/L, amount of hemoglobin is 90 g/L, anisocytosis,
poicilocytosis, megaloblasts and megalocytes were found, reticulocytes of 0%. What sort of
anemia develops in this case?
A. Thalassemia
B. Bi2-deficiency anemia
Post-hemorrhagic anemia

IF [FIA
Iron-deficiency anemia
Hemolytic anemia

15. In an infant, who is under an artificial nutrition with cow milk, severe anemia has developed.
At the blood count of the infant: number of erythrocytes is 4x1012/L, content of hemoglobin is
68 g/L, reticulocytes of 0%. What kind of anemia developed in the infant?
Sickle-cell anemia
I= |S [0 I [>

Inborn hemolytic anemia


By2-deficiency anemia
Hypopastic anemia
Iron-deficiency anemia

16. Predomination of erythroblasts, normoblasts and megaloblasts was revealed in blood analysis
of a patient with anemia. The same cells were found in bone marrow. What type of anemia do
these changes characteristic for?
Post-hemorrhagic
I= |F 10 I [>

Hemolytic
Aplastic
Bi2-folate deficiency anemia
Iron-deficiency anemia

17. Two years ago a patient was operated on for resection of pyloric portion of the stomach. There
is weakness, periodical appearance of dark circles under eyes, breathlessness in this patient. In
the blood analysis: HB-70g/L, erythrocytes-3.3x1012/L, color index-0.7. What erythrocyte
changes in the blood smear are the most typical for the given state?
A. Microcytes
B. Megalocytes
CG Schizocytes
D. Ovalocytes
E. Macrocytes

18. Singular oxyphilic normoblasts appeared in the blood of a patient after acute post-traumatic
hemorrhage composing 15% of blood volume. On supravital staining 25% of reticulocytes
were found. What is the patient anemia according to its ability of regeneration?
A. Hyperregenerative
B. Regenerative
C Hyporegenerative
D. Aregenerative
E, Hypo- and aregenerative

19. What index of blood analysis is the most typical for betathalassemia?
Considerable decrease of erythrocytes and hemoglobin

I= |F 10 Ie |>
Erythrocytes with basophilic stippling
Increase of fetal hemoglobin
Target-like erythrocytes
Increase of met-hemoglobin

20. A patient had anemia due to profuse blood loss. What blood changes are typical at the
beginning of development of acute post-hemorrhagic anemia?
A. Presence of megalocytes in the blood
B. Absence of reticulocytes
CG Poikilocytosis, anisocytosis
D. Hyperchromia
E. Normochromia

21. Erythropenia, hyperchromia, normocytes, macrocytes, megalocytes, poikilocytosis were


found out in a patient’s blood at examination. What is the cause of this pathology?
Ascariasis
I= |F IO le [>

Deficiency of gastromucoprotein
Iron deficiency in food
Trichocephaliasis
Fequent loss of blood

22. While studying a blood smear different forms of erythrocytes were found. Which of them are
regenerative?
A. Anisocytes
B. Poikilocytes
CG Ploychromatophils
D. Ovalocytes
E. Megalocytes

23. The presence of agranulocytosis was determined in the blood analysis of a liquidator of
accident at Chernobyl atomic power station who had got 5 Gr dose of irradiation. What
pathogenetic mechanism is the leading one in its appearance?
4 _tabibatton of Jeuropoiesss
Increased penetration of granulocytes into tissues
IF |S 10 I=

Increase of leucocytes destruction


Impairment of going out of mature leukocytes from bone marrow
Development of autoimmune process.

24. During the development of acute pulpitis a patient complained of paroxysm of pain in the
upper jaw, which is increasing at night, fever. At examination leucocytosis was established in
the blood. What kind of leucocytosis is possible in this case?
A. Basophilic leucocytosis
B. Lymphocytosis
CG Eosinophilic leucocytosis
D. Monocytosis
E. Neutrophilic leuccytosis.
25. Considerable increase of the number of eosinophils in a unit of blood volume was determined
during the examination of a 5 years old boy.
What may cause eosinophilia in this patient?
A. Helminthic invasion
B. Obesity
CG Hypodynamia
D. Hypothermia
E. Physical exertion

26. In the patient’s blood analysis the number of leukocytes is 250*109/ L. What syndrome does
this patient have?
A. Leukemia
B. Leucocytosis
C. Leucopoenia D. Leucomoid reaction
E. Hyperleucocytosis.

27. Leucocytosis was found out in a person who didn’t complain of his health. The cause of this
may be that fact that the blood was taken for analysis after:
Physical load
I= |S 10 I [>

Mental work
Rest at a health resort
Considerable use of water
Usage of alcohol

28. Lymphocytosis was revealed in a patient. What diseases may be accompanied by


lymphocytosis?
Sepsis
I= |S 10 I [>

Helminthic invasion
Somatotropin insufficiency
Pertussis, chicken pox
Bronchial asthma

29. Neutropoenia is found out in a patient who has the manifestations of immunodeficiency. What
diseases may neutropoenia be determined in?
After profuse haemorrhage
IP I[>

Fuzarium fungus poisoning


Myeloleukemia
D. Insufficiency of sexual gland function
E. Septic process

30. Increase of the number of eosinophils was determined in a patient with endocrine pathology
during examination. Which of the named diseases may be accompanied by cosinophilia?
A. Pheochromocytoma
B. Conn’s disease
Cc. Acromegaly
D. Cushing disease
E. Addison’s disease

31. Functional insufficiency of monocytes is accompanied by immunodeficiency. What BAS


produced by monocytes stimulate specific response?
A. Interleukin-2
B. Lysocime
C Interleukin-1
D. Fibronectin
E. Myeloperoxidase

32. Relative neutropoenia with degenerative shift was revealed in the blood of a patient with TB
during examination. What change of differential blood count corresponds to this state?
A. Decrease of the number of segmented forms and increase of band forms
of neutrophils B. Decrease of lymphocyte number
C. Increase of monocyte number
D. Increase of basophil number
E. Decrease of eosinophil number

33. Neutrophil leukocytosis was revealed in worker during examination.


What pathologic condition may neutrophil leukocytosis result from?
Chronic loss of blood
I= |F [0 I |>

Septic condition
Viral infection
Radiation sickness
Benzene poisoning

34. A patient aged 32 with massive hemorrhage due to car accident trauma was admitted to the
hospital. Pluse-100 beats per min, respiratory rate-22 per 2 min, BP-100/60 mmHg. What
blood change will be the most characteristic in an hour after hemorrhage?
Erythropoenia
IS 10 IF |>

Hypoproteinemia
Hypovolemia
Leucopoenia
E. Erythrocyte hypochromia

35. During the examination of peripheral blood of a patient aged 42, it was revealed: Hb-80g/L,
erythrocytes-3.2 x 1012/L, leukocytes-25 x 109/ L, leukocytic formula: basophils-5%,
eosinophils- 9%, myeoblasts- 3 %, promyelocytes-8%, neutrophils-17%, segmented- 19%,
lymphocytes- 3 %, monocytes- 3%. What blood pathology is the most possible in this patient?
Panmyclophthisis
I= |F 10 I >

Erythroleukemia
Acute myeloblast leukemia
Chronic myelogenous leukemia
Promyelocytic leukemia

36. The experiment was carried out on a rabbit. The increase of the number of erythrocytes and
hemoglobin in the blood due to the stimulation of erythropoiesis by erythropoictin was
determined 2 weeks later after the narrowing of renal artery. What increases the formation of
erythropoietin?
A. Hypoosmia
B. Hypercapnia
CG Hypoxemia
D. Hyperosmia
E. Hypovolemia

37. Hemolytic anemia with decrease of osmotic erythrocyte resistance that averaged 0.6-0.5 was
revealed in a smear of venous blood during microscopic examination. What substance
accumulation in the blood plasma may also indicate the development oh hemolytic anemia?
A. Creatinine
B. Urea
CG Indirect bilirubin
D. Lactic acid
E. Inorganic phosphate
38. Hypochromia of erythrocytes, micro-, anisocytosis, poikilocytosis, are determined in the blood
in case of development of iron deficiency and iron refractory anemia. What index must be
determined to carry out differential diagnosis of these anemias?
A. Serum chlorine
B. Serum phosphorous
CG Serum magnesium
D. Serum calcium
E. Serum iron
39. Biz folic deficiency anemia developed in a patient after stomach resection. What color index
is typical for this disease?
A. 130
B. 1.15
C10
D. 0.85
E. 0.70
40. A victim of a car accident has lost much blood. What impairment of general blood volume
takes place?
Simple hypovolemia
I= |S 10 I [>
Polycythemic normovolemia
Olygocysthemic normovolemia
Polycystehmic hypovolemia
Oligocysthemis hypovolemia

41. Excessive flow of estrogens into the blood due to follicle persistence (a state when follicle
does not reach complete maturation and ovulation does not take place) often cause uterine
bleeding. What anemia may develop in this case?
A. Iron deficiency
B. Sideroachrestic
Cc. Sickle cell
D. Hypoplastic
E. Metaplastic.

42. Which of the below named anemias relevant to hemoglobinopathies?


A. Minkovsky-Shoffar disease
B. Iron deficiency anemia
Cc. Biz deficiency
D. Thalassemia
E. Iron refractory anemia
43. Evaluate the blood analysis: erythrocytes 3.10; Hb 90g\L reticulocytes 0.5%. In the smear
there are poikilocytes hypochromatic erythrocytes. Blood serum iron is 80 micromol/l. What
pathology is this typical for?
Minkovsky-Shoffer disease
10 I >

Iron deficiency anemia


IS

Biz deficiency
Sickle cell anemia
E. Iron refractory anemia
44, What conditions are accompanied by neutrophilic leukocytosis with shift of differential count
to the left?
A. Purulent inflammation
B. Tuberculosis
C Infections mononucleiosis
D. Agranulocytosis
E. Alimentaray leukocytosis
45. What blood pathology is the presence of Philadelphia chromosomes in the blood cells and
bone marrow cells typical for?
Acute myelogenous leukemia

I= |F [0 I |>
Chronic myelogenous leukemia
Hodgkin’s disease
Burkitt’s lymphoma
Chronic lymphocyte leukemia

46. Patient M, aged 20 was admitted to the hospital complaining of high temperature, pain in the
bones, and hemorrhage from his gums. Blood analysis of this patient shows: erythrocytes-
2.5x1012/L; Hb-80g/L; leucocytes-2.0x109/L; thrombocytes-6.0x109/L; differential count:
eosinophils-1%; stab neutrophils-1%; segmented neutrophils-10%; lymphocytes-10%;
monocytes-3%; blast cells-75%. What pathology is this blood analysis typical for?
Hodgkin’s disease
I= |S 10 Ie |>

Burkitt’s lymphoma
Acute leukemia
Infections mononucleosis
Chronic leukemia

47. General amount of leucocytes is 90x109/l. In differential count: cosinophils-1%; basophils-


0%; juvenile neutrophils-0%; stab neutrophils-2%; segmented neutrophils-20%;
prolymphocytes-2%; lymphocytes-70%; Botkin-Gumprecht cells. Cervical, submandibular
lymph nodes are enlarged. What pathology is such blood picture typical for?
Acute lymphoblastic leukemia
I= |S 10 I |>

Hodgkin’s disease
Infectious mononucleosis
Chronic lymphocytic leakemia
Chronic myelogenous leukemia

48. Hypochromic anemia was found out in a patient aged 54, who had a prolonged contact with
lead at his work. Treatment with iron preparations for a month didn’t give any effect. The
increased amount of iron was determined in blood serum. What is this anemia due to?
Vitamin B12 deficiency
I= |S 10 I [>

Porphirin synthesis impairment


Folic acid deficiency
Hypoplasia of red bone marrow
Erythrocyte hemolysis

49. A patient aged 20, has periodically yellowness of sclera and skin that is accompanied by
weakness. Minkovski-Shoffar disease is diagnosed.
What is the most typical for blood picture in this disease?
A. Reticulositosis
Agranulocytosis

IF IF 19I
Macrocytosis
Microspherocytosis
Thrombocytosis

50. Posthemorrhagic anemia has developed in a patient with periodical bleeding due to uterus
fribromyoma. What type of chronic post hemorrhagic anemia takes place in this case?
Megaloblastic, hyperchromic, hyperegenerative.
10 IF [>

Megaloblastic, hypochromic, hyporegenerative.


IP

Erythroblastic, hyperchromic, hyporegenerative.


Erythroblastic, hypochromic, hyperregenerative.
E. Erythroblastic, hypochronic, hyporegenerative.
51. A female patient with impairment of menstrual cycle accompanied by prolonged bleeding the
blood analysis was made which revealed hypochromia, decrease of reticulocytes,
microcytosis. What group does this anemia belong to according to pathogenesis?
Biz folic deficiency anemia
I= |F 10 I [>

Iron deficiency anemia


Hypoplastic anemia
Hemolytic anemia
Metaplastic anemia

52. Patient R. aged 12 was operated on for acute phlegmonous appendicitis. At the examination
of his blood, the amount of leukocytes is 12.100/L. On the blood smear there are: basophils —
0%; eosinophils — 2%; monocytes — 2%; juvenile neutrophils — 0%; stab neutrophils — 30%;
segmented neutrophils — 43%; lymphocytes — 23%; monocytes — 0%. Stab (immature)
neutrophils have pyknosis of nuclei. There is anisocytosis of neutrophils; some of them have
toxic granulation. What form of change of leukocyte blood composition takes place in this
case? A. Neutrophilia with degenerative shift to the left.
B. Neutrophilia with regenerative shift to the left.
C. Neutrophilia with hyperegenerative shift to the left.
D. Leukemoid reaction of neutrophil type.
E. Neutrophilia with the shift to the right.

53. Which of the below given hematological characteristics corresponds to chronic post-
hemorrhagic anemia?
Hyperregenerative, hypochromic with erythroblasts type of bleeding.
I= |F 10 I [>

Regenerative, hypochromic with erythroblasts type of bleeding.


Hyporegenerative, hypochromic with megaloblasts type of bleeding.
Hyporegenerative, hypochromic with erythroblasts type of bleeding.
Hyporegenarative, hyperchromic with megaloblasts type of bleeding.
54. A female patient, aged 25, was admitted to the hematological department with complains of
the appearance of hemorrhages of different sizes on the body; during menstruation there are
uterine bleedings. She has been ill for ten years. Paleness of skin and mucous membranes were
determined on examination; there are hemorrhages of different size and color on the upper and
lower extremities. Pulse = 100 beats/minute, AP 110/70 mmHg. Blood analysis shows:
erythrocytes 3.3*10i12 /L, Hb 80g/L, thrombocytes 33*109/L; time of blood coagulation:
beginning is at 2nd minute, end is at 6th minute; time of bleeding (according to Duke) - 15
minutes. What is the possible diagnosis?
Marchiafava-Michelli disease.
IF |F [0 I |>

Thrombocytopoenic purpura.
Glanzman’s thrombasthenia.
Willeberandt-Yurgens thrombocytopathia
Chronic myelogenous leukemia.

55. Neutrophil leukocytosis is determined in a patient with chronic myelogenous leukemia. Which
variant of nuclear shift of differential count to the left is the most typical for chronic
myelogenous leukemia?
Regenerative
I= |S 10 I [>

Degenerative.
Hyper regenerative.
Regenerative and degenerative.
Hyporegenerative.

56. A patient with atrophic gastritis has vitamin Biz deficiency. What variant of nuclear shift of
differential count is most typical for B12 hypovitaminosis?
Hyper regenerative to the left
I |S 10 I [>

Degenerative to the left.


Regenerative and degenerative to the left.
To the right
Regenerative to the left

57. Hypochromic anemia was diagnosed in a patient on the 7th day after acute hemorrhage. What
mechanism is the leading one in its development?
Increase of iron excretion from the organism;
I= |S 10 I [>

Impairment of globin synthesis;


Increased erythrocyte destruction in the spleen;
Impairment of iron absorption in the intestine;
Increased penetration of immature erythrocytes from bone marrow.
58. Hemiparesis appeared in a patient with acute promyelocytic leukemia. What is the main
mechanism of the impairment of CNS in this case?
Intoxication by leukemic cells decay products;
I= |>

Formation of leukemic infiltrates;


C.Impairment of desintoxicative function of the liver;
D. Cachexia;
E. Increase of thrombogenesis.
59. Acute pain in the lower jaw, swelling of the cheek, temperature of 37.60C occurred in the
patient who had had dental caries for some years.
What changes in the patient’s blood may be observed in this case?
A. Neutrophilic leukocytosis;
B. Leucopoenia;
CG Moncytosis;
D. Anemia;
E. Eosinophilia.
60. A patient aged 43 has stomatitis, glossitis, the tongue is crimson colour, smooth. Blood
analysis revealed: Hb 100 g/l; erythrocytes 2.3x1019/1; color index 1.30. What is the patient’s
state due to?
Erythrocytes haemolysis
I= |F [0 Ie [>

Hypoplasia of red bone marrow


Impairment of porphyrin synthesis
Iron deficiency
Vitamin Bi2 deficiency

61. After resection of small intestine a patient complained of increased fatigability, infringement
of taste, brittleness of nails, quick decay of dental enamel, and appearance of breathlessness
on physical exertion. Which of the below given substances is the source of impairments in the
patient’s organism?
A. Bi2-folic deficiency anemia
B. Hypocalcaemia
Cc Vitamin D deficiency
D. Hyponatriaemia
E. Iron deficiency anemia

62. A female patient took analgin because of toothache. Dark urine, icteric sclera, weakness
appeared in her two days later. Which of the causes given below is the most possible one?
A. Immune hemolytic anemia
B. Hypoplastic anemia
CG Thrombocytopoenia
D. Thrombocytopathy
E. Agranulocytosis

63. A patient with acute pulpitis has an increased body temperature and a number of leukocytes
up to 14.109/L, differential count represents: basophils-0%; eosinophils-2%; megakaryocytes-
0%; juvenile neutrophils-2%; stab neutrophils-8%,; segmented neutrophils-58%; lymphocytes-
26%; monocytes-4%. How can you evaluate such changes in blood?
A. Neutrophilic lekocytosis with regenerative shift to the left
B. Neutrophilic lekocytosis degenerative shift to the left

C Neutrophilic lekocytosis with hyperregenerative shift to the


left
D. Lymphocytosis
E. Neutrophilic leukocytosis with shift to the right

64. A patient aged 30 took sulfacthidole for infectious process of mucous membrane in the mouth
locally (as powder). Preparation has hemotoxic action and the treatment was complicated by
the development of agranulocytosis. At examination it was revealed:
A. Increase of agranulocytes in the blood;
B. Decrease of the number of granulocytes in the blood on the background of
leukocytosis;
CG Decrease of the number of granulocytes in the blood on the background
of leucopoenia;
D. Decrease of the number of neutrophylic granules with their simultaneous
increase;
E. Loss of their granules by granulocytes.

65. A women fell ill with purulent stomatitis. What index of complete blood count is characteristic
for this disease?
A. Lymphocytosis
B. Thrombocytosis
C Leukocytosis
D. Anemia
E. Monocytosis
66. It is recommended to perform a clinical examination of blood in an empty stomach. What
compounds of peripheral blood may be changed after taking food?
Decrease in number of platelets
I= |F 10 Ie |>

Elevation of number of leukocytes


Elevation of number of erythrocytes
Increase in content of plasma proteins
Reduction of number of erythrocytes

67. Acute inflammatory disease of upper airways and eyes appears in 45years-old woman at the
period of blossom of grass. Symptoms of this disease are hyperemia, edema, and mucous
secretions. What kind of leukocytosis is the most characteristic one for this disease?
A. Neutrophilia
B. Monocytosis
CG Eosinophilia
D. Basophilia
E. Lymphocytosis

68. Enlargement of liver and spleen, anemia, and myeloblasts in peripheral blood were revealed
in a patient with acute leukemia. What is the main trait, which allows to distinguish acute
myelogenous leukemia from chronic one?

Pancytopoenia
I= |S 10 I [>

Blast cells in peripheral blood


Anemia
Leukemic gap
Thrombocytopoenia

69. Extraction of a tooth, in a patient with chronic lymphocytic leukemia, was complicated by
prolonged bleeding. What may cause the hemorrhagic syndrome in this patient?
Anemia
I= |S 10 I [>

Lymphocytosis
Eosinopoenia
Neutropoenia
Thrombocytopoenia

70. In blood analysis of 37-years-old woman following data were revealed: content of hemoglobin
is 60 g/L, number of erythrocytes is 3.0x1012/L, and color index of 0.6; differential count of
leukocytes without any changes; number of platelets is 200x109/L; reticulocytes count of
0.3%; ESR of 18 mm/hour; microcytosis and poikilocytosis of erythrocytes. Indicate the most
probable type of anemia according to mechanisms of its development.
Hypoplastic anemia
I= 1S 10 I [>

Hemolytic anemia
Acute post-hemorrhagic anemia
Iron deficiency anemia
By2-folate deficiency anemia

71. In a patient aged 35 autoimmune hemolytic anemia developed. What index of blood serum is
the most increased in this case?
Mesobilinogen
I= |S 10 I [>

Stercobilinogen
Direct bilirubin
Indirect bilirubin
Protoporphirin

72. A patient addressed a dentist with complaints of affections of mucous membrane of his mouth.
During the examination of the patient ulcerous stomatitis with necrosis in center was revealed
at him in area of his palate. In the history of disease of the patient recently endued pneumonia
and taking of medicines (sulfonilamides) were present. After administering treatment the
doctor pointed the patient for blood analysis.
What pathology from below mentioned does doctor suppose?
Immune agranulocytosis
SIF IS 19 I >
Iron deficiency anemia
Thrombocytopoenia
Infectious mononucleosis
Infectious lymphocytosis
73. A 40-years- old male patient, who was bitten by snake, was admitted to the hospital. What
place does hemolysis occur at in this case?
In vessels
I= |S 10 I [>

In hepatic cells
In the spleen
In the bone marrow
In renal parenchyma

74. Myocardial infarction was diagnosed in 65-years-old man. Neutrophilic leukocytosis with left
shift is present in the blood of this patient. What factors underlie this phenomenon?
Products of tissue decay
I= 1S 10 Is |>

Elevation of mass of muscular fibers


Disorders of alveolar ventilation
Decrease in glycogen content in the myocardium
Increase of arterial pressure

75. Biz deficiency anemia was diagnosed in a man aged 57 after examination of him. Treatment
was administered to this patient. Control blood test in this patient was performed in 3 years.
What is the most adequate criterion for enhancement of erythropoiesis?
Increase in number of reticulocytes
IF |F 0 I >

Increase in hemoglobin content


Decrease in color index
Normoblastic hematopoiesis
Increase in number of leukocytes

76. Patient with chronic hypoacidic gastritis has hypochromic anemia. In the blood smear of this
patient agranulocytes, micro- and anisocytosis, and poikilocytosis are revealed. Name this
anemia.
Iron deficiency anemia
I= |S [0 I |>

Acute post-hemorrhagic anemia


Thalassemia
Sickle-cell anemia
Pernitious anemia
77. Increased concentration of leukopoietins in blood was found in a patient with acute
appendicitis. What kind of lekocytosis occurs in these conditions?
Neutrophilic

I= |S [0 I |>
Basophilic
Eosinophilic
Lymphocytosis
Monocytosis
78. A patient has deficiency of cyancobalamine and folic acid that leads to disorder of
leucopoiesis. What changes takes place in these conditions?
Leucopoenia
I= |S 10 I [>

Eosinophilia
Basophilia
Hemophilia
Hyperemia

79. A patient, who was exposed to ionizing radiation, has panmyelophtisis and secondary
infections. What changes occur in blood analysis in this case?
Agranulocytosis
I= |F 10 I >

Leucocytosis
Eosinophilia
Basophilia
Hyperemia

80. A patient with leukemia has general number of leukocytes of 120.0x100/L. What kind of
leukemia does this patient have?
Leukemic
I= |S 10 I [>

Leucopenic
Subleukemic
Aleukemic
Erythremia

81. In a woman hypochromic anemia was diagnosed after delivery accompanied by marked
bleeding. What pathologic forms of erythrocytes are characteristic ones for this type of
anemia?
Anisocytes
I= |F 10 I |>

Target-like erythrocytes
Sickle-cell anemia
Spherocytes
Microcytes
82. Patient with chronic leukemia has sharply increased temperature, breathlessness, marked
muscular weakness at insignificant physical exertion, increased sweating, cough. What
mechanism of leukemia influence upon organism underlies complications in this patient?
A. Immunodeficiency due to functional inability of leukocytes
B. Internal bleeding because metastases into vessel wall
Cc. Anemia
D. Tumor progression
E. Airway obstruction because of development of metastases

83. When examining a blood in a patient, who endued bleeding three days ago, following data was
revealed: number of leukocytes is 12x109/ L, basophils count is 0%, eosinophils count is 3%,
myclocytes count 0%, juvenile neutrophils count is 3%, stab neutrophils count is 12%,
segmented neutrophils count is 62%, lymphocytes count 16%, and monocytes count is 4%.
What kind of changes of leukocyte differential count takes place in this case?

Neutrophilia with regenerative shift to the left


I= |F 10 I |>

Neutrophilia with degenerative shift to the left


Neutrophilia with shift to the right
Absolute lymphopoenia
Absolute monocytopoenia

84. A 40-years-old patient, who was admitted to the surgical department with diagnosis of
phlegmona of thigh, had high temperature, tachycardia, and breathlessness. On the blood test
of this patient: number of leukocytes is 25x109/L; eosinophils count is 1%, myelocytes count
is 1%, juvenile neutrophils count is 15%, band neutrophils count is 25%, segmented
neutrophils count is 40%, lymphocytes count 14%, monocytes count is 4%. What kind of shift
in differential count is present in this case?
A. Regenerative
B. Hyperregenerative
CG Degenerative
D. Regenerative-degenerative
E. Leukemoid

85. What is the cause of intracellular hemolysis? A. Genetic infringements of erythrocytes


B. Malaria
C. Action of hemolytic poison
D. Infection with hemolytic streptococcus
E. Transfusion of incompatible blood

86. What kind of disorders of total blood volume appears in case of absolute erythrocy tosis?
A. Polycytemic hypervolemia
B. Oligocytemic hypervolemia
Oligocytemic hypovolemia

I= |S IA
Simple hypervolemia
Simple hypovolemia
87. Beta-thalassemia was revealed in a patient, who came from Tunis. The disease was
accompanied by hemolysis and jaundice. The disease was diagnosed on the base of presence
in blood:
Target-like erythrocytes
I= |F 10 I |>

Grained erythrocytes
Polychromatophil erythrocytes
Normocytes
Reticulocytes

88. During the examination of adolescents that reside in mounting region increase in level of
erythrocytes and hemoglobin in peripheral blood was found out. What is the reason for
indicated erythrocytosis?

Exogenous hypoxia
I= |S 10 I [>

Diseases of lungs
Congenital heart disease
Condensation of blood due to large loss of water
Vakes’s disease

89. Content of hemoglobin and number of erythrocytes significantly decreases in a patient’s blood
from time to time. It was found out that such attacks appear after taking some horse beans.
What kind of anemia takes place in this case?
Enzymopathy
I= |S 10 I [>

Membranopathy
Iron deficiency anemia
Hemoglobinopathy
Acquired hemolytic anemia
ANSWERS:
1.-A. 2.-D. 3.-D. 4.-A. 5.-E. 6.-D. 7.-E. 8.-A. 9.-E. 10.-B. 11.-C. 12.-E. 13.-C. 14.-B. 15.-E. 16.-
D. 17.-A. 18.-A. 19.-C. 20.-E. 21.-B. 22.-C. 23.A. 24.-E. 25.-A. 26.-A. 27.-A. 28.-D. 29.-B. 30.-
E. 31.-C. 32.-A. 33.-B. 34.-C. 35.-D. 36.-C. 37.-C. 38.-E. 39.-A. 40.-E. 41.-A. 42.-D. 43.-E. 44.A.
45.-B. 46.-C. 47.-D. 48.-D. 49.-D. 50.-E. 51.-B. 52.-D. 53.-D. 54.-B. 55.-C. 56.-D. 57.-E. 58.-B.
59.-A. 60.-E. 61.-E. 62.-A 63.-A 64.-C. 65.C. 66.-B. 67.-C. 68.-D. 69.-E. 70.-D. 71.-D. 72.-A. 73.-
A. 74.-A. 75.-A. 76.-A. 77.-A. 78.-A. 79.-A. 80.-A. 81.-A. 82.-A. 83.-A. 84.-B. 85.-A.
86.-A. 87.-A. 88.-A. 89.-A.
Topic 18. Pathophysiology of hemostasis and antihemostasis.

1. A 32-years-old male patient has hemorrhage after injury of vessel that connected to
formation of friable thrombi. What coagulation factor deficiency has led to this disorder?
A. II (prothrombin)
B. IT (thromboplastin)
C. VII (proconvertin)
D. XII (Hageman’s factor)
E. XIII (fibrin stabilizing factor)

2. A man, who has been to Arctic for a long time, has hemorrhage from gums, his teeth sway
and pull out. What is the initial mechanism in scurvy development?
Infringement of elastin synthesis
HUA S

Affection of alveolar process


Impairment of collagen synthesis
Fragility of capillaries
Insufficient tightness of round ligament of tooth

3. Preliminary exfoliation of placenta and hemorrhagic shock develop in a pregnant woman


during the delivery. While taking woman’s blood for analysis it coagulates in syringe. The acute
DIC syndrome is diagnosed in this woman. What pathogenic mechanism appears to be initial in
disorder of hemostasis in this case?
A. Activation of sympathetic-adrenal system
B. Elevation of platelet aggregation
C. Activation of tissue thromboplastin derived from
destroyed tissues
D. Cascade activation of plasma coagulation factors
E. Oppression of aticoagulative system of blood
4, It was found out before operating on that the patient has bleeding time increased up to 9
min. What blood cells deficiency do these changes result from?
A. Monocytes
B. Platelets
C. Erythrocytes
D. Lymphocytes
E. Leukocytes

5. Breathlessness, acute pain in the chest, cyanosis, and enlargement of neck veins rapidly
develop in a patient suffered from thrombophlebitis.
What is the most possible disorder of blood circulation in this patient?
A. Pulmonary thromboembolism
B. Cerebral thromboembolism
C. Coronary thromboembolism
D. Mesenterial thromboembolism
E. Portal thromboembolism
6. Numerous subcutaneous hemorrhages appear in a patient suffered from hepatic cirrhosis.
What is a possible reason for this?
A. Excessive decay of vitamin C
B. Disorder of vitamin K synthesis
C. Hypocalemia
D. Reduction in synthesis of factor II (prothrombin)
E. Deficiency of factor III (thromboplastin) in blood plasma 7. Hemorrhagic
syndrome with disorders of the third stage of blood coagulation develops in a patient after
she was operated on uterus. What is the most probable mechanism of hemostasis disorder
in this case?
A. Qualitative abnormalities of fibrinogen
B. Activation of fibrinolysis
C. Deficit of fibrin stabilizing factor
D. Decrease in ptothrombin synthesis
E. Decrease in fibrinogen synthesis
8. Hemorrhagic diathesis has developed in a patient with streptococcus infection. What is the
reason for hemorrhage development?
Elevation of heparin content in the blood
moOOw>

Vitamin A deficiency
Increase in callicrein content in blood
Enhanced fibrinolysis
Vitamin C deficit

9. Point hemorrhages appear on a forearm of a patient after putting a tourniquet on a patient’s


arm. What functions of blood cells do these changes connect to?
Neutrophils
MoOOw>

Platelets
Erythrocytes
Basophils
Macrophages

10. A teeth was pulled out from the patient with chronic hepatitis. Bleeding, which developed after
that, could not be seized for 2 hours. Performed blood analysis establishes increase in content
of several coagulation factors. What kind of hemostasis is damaged in this case?
Platelet-vascular
MoOOw>

Coagulation
Vascular stasis
Platelet reaction
Vascular reaction
11. Numerous hemorrhages and bruises were found out on the patient’s body. At examination of
this patient: his bleeding time by Duke is 25 min, his number of platelets is 25x109/L. What
disease are these symptoms characteristic for?
A. Hemophilia B
B. Von Willebrand’s disease
C. Vitamin C deficiency
D. Hereditary defect of platelet formation
E. Hemophilia A
12. A 65-years-old male patient suffered from atherosclerosis was admitted to the surgical
department because of diffuse purulent peritonitis. When the patient was operated on
thrombosis of mesenterial vessels was diagnosed in him. What is the most possible reason for
peritonitis development?
A. Hemorrhagic infarction
B. Agiospastic ischemia
C. Ischemic infarction
D. Stasis
E. Compressive ischemia
13. Hemorrhagic syndrome connected to disorders of the third phase of coagulation developed in
a patient after hi was operated on pancreas. What is the possible mechanism of development
of hemostasis disorder?

Elevation in content of heparin in patient’s blood


HUA S

Reduction in fibrinogen synthesis


Activation of fibrinolysis
Deficit of fibrin stabilizing factor
Reduction in prothrombin synthesis

14. Sharp pain and edema of tissues develop in a patient with thrombosis of veins of his right calf.
What is the possible consequence of thrombosis?
Hypertrophy of tissues
mMoOw>

Circulatory hypoxia
Enhancement of drainage of tissues
Intensification of metabolism
Enhancement of functional activity of tissues

15. Thrombosis of coronary arteries occurs in a patient with atherosclerosis of cordial vessels.
What is the reason for formation of thrombi?
Heparin deficiency
MoOOw>

Thrombocytopenia
Acceleration of blood flow
Activation of fibrinolysis
Vitamin K deficiency
16. An elderly patient was admitted to the hospital with thrombosis of veins of his calf. What is
the reason for thrombosis development?
Decrease in prothrombin concentration in blood

moOw>
Increase in heparin concentration in blood
Injury of vessel wall
Slowing-down of blood flow
Activation of plasmin

17. When examining a patient increased blood coagulation (thrombophilia) was found out in him.
What reasons favor such state development?
Intensification of prostacyclin synthesis in vessel wall
moOOw>

Low concentration of thrombin in blood


High concentration of heparin in blood
Deficit of inhibitors of proteolytic enzymes
High concentration of adrenalin in blood

18. Thrombocytopenia was found out in blood of a patient with Werlgoff’s disease. Why the
pathology of platelets leads to hemorrhage?
Decrease in thromboplastin formation
MoOOw>

Decrease in concentration of prothrombin in blood


Decrease in concentration of heparin in blood
Activation of fibrinolytic system
Decrease in concentration of fibrinogen in blood

19. Hemophilia A was diagnosed in a patient with hemorrhagic syndrome. What is the reason for
this disease development?
Absence of Stewart’s factor (factor X)
MoOOw>

Absence of Christmas’s factor (factor [X)


Absence of factor VIII
Absence of Rosenthal’s factor (factor XT)
Low concentration of Hageman’s factor (factor XII) in blood

20. A patient was admitted to the hospital with abundant hemorrhoid bleeding. This patient has
been suffering from hepatic cirrhosis for a long time. What is the reason for hemorrhage
development under hepatic cirrhosis?
Activation of fibrinolysis
MoOOw>

Plasmin deficiency
Prothrombin deficiency
Low concentration of thrombostenin in blood
Excess of heparin
21. Shonlein-Henoch disease was diagnosed in a patient. What changes in blood cells number are
characteristic for this disease?
Eosinophilia

MOODS
Thrombocytopenia
Erythropoenia
Polycytemia
Eosinopoenia

22. Patient has hemorrhage from gums, subcutaneous hemorrhages, and frequent nasal bleedings.
Significant Thrombocytopenia was revealed in blood test of this patient. What is the reason
for bleeding development in case of thrombocytopenia?
Thrombin deficiency
HOW pS

Reduction of thromboplastin formation


Excessive heparin formation
Activation of fibrinolysis
Excess of prostacyclins

23. While performing intravenous injections trauma of venous vessels leads to thrombosis. What
favors thrombosis under injury of vessel wall?
Activation of plasmin
moOOw>

Activation of fibrinolysis
Activation of phospholipase D
Enhanced production of prostacyclin by endotheloicytes
Accumulation of thromboxan A2

24. Changes of some indices of blood were revealed at examination of a patient suffered from
hemophilia. What of enumerated indices corresponds to this condition?
Thrombocytopenia
moOOw>

Bleeding time by Duke takes longer


Eosinophilia
Time of coagulation takes longer
Afibrinogenemia

25. A boy has congenital disorder of hemostasis: he has prolonged hemorrhage even in case of
insignificant injuries, subcutaneous bruises, and bleedings in joint cavities, which restrict the
movement activity. Patient’s blood does not coagulate for a long time if it is taken out from
the organism and it does not contain coagulation factor VIII. Blood cells count including
number of platelets is within norm. What underlies the congenital disease of this boy?
Hereditary gene defect linked with X chromosome
Vow>

Toxicosis of pregnancy in boy’s mother


Intrauterine infection
Intrauterine intoxication
E. Intrauterine immune conflict

26. Hemophilia B was diagnosed in a child who has hemorrhagic syndrome. This type of
hemophilia results from absence of:
Coagulation factor IX (Christmas’s factor)
MoOOw> Coagulation factor II (prothrombin)
Coagulation factor VII (antihemophilic globulin)
Coagulation factor XI (thromboplastin)
Coagulation factor XII (Hageman’s factor)

27. It is known that thrombus undergo same changes after its formation. What sort of finishing of
thrombus formation is the most dangerous for a patient?
Aseptic lysis
moOw>

Septic lysis
Organization without recanalization
Organization with recanalization
Calsification of thrombus

28. Such appearances as petechiae and ecchymoses develop ina boy aged 7, who often fall ill with
acute respiratory diseases. Pathology of internal organs is absent in this patient. What
pathology is present in this case?
Hypoplastic anemia
mMoOw>

Thrombocytopenia
Acute leukemia
Hemophilia
Chronic leukemia

29. Course of atherosclerosis in 70-years-old patient is complicated by thrombosis of vessels of


lower extremities and development of gangrene of left foot toes. The beginning of thrombosis
is connected to:
Activation of prothrombinase
mMoOw>

Adhesion, aggregation and agglutination of platelets


Conversion of prothrombin to thrombin
Conversion of fibrinogen to fibrin
Decreased heparin synthesis

30. Antihemophilic globulin A (factor VHD) is absent in the blood plasma of a boy with significant
hemorrhagic syndrome. What phase of hemostasis is infringed primarily in this boy?
Retraction of blood clot
GOW p

Conversion of fibrinogen to fibrin


Conversion of prothrombin to thrombin
Extrinsic pathway of prothrombinase (thrombokinase) activation
E. Intrinsic pathway of prothrombinase (thrombokinase) activation

31. Which of essential factors of thrombus formation the most often serves as the reason for
thrombosis?
Decrease in activity of anticoagulative system
MOOD pActivation of systems of blood sedimentation
Injury of vessel wall
Acceleration of blood flow
Turbulence of blood

32. A patient complains of hemorrhage from gums. What vitamin deficiency may lead to this
phenomenon? A. By
B.D
C. Ba
D.A
E.K

33. Prolonged bleeding complicated pulling out of a tooth in a patient, who suffered from chronic
leukemia. What may serve a reason for hemorrhagic syndrome in this patient?
Thrombocytopenia
MoOOw>

Anemia
Erythropenia
Lymphocytosis
Neutropenia

34. Bleeding developed in a 15-years-old patient after her teeth was pulled out. It was found out
that after endued hepatitis A this patient has point hemorrhages (petechiae and ecchymoses)
and bruises on her skin, and she had three accidents of nasal bleeding. Before this she did not
have hemorrhages even after pulling out of her tooth. It was established under examination
that the bleeding time is increased in this patient. At the patient’s blood test hypochromic
anemia, decrease in number of platelets, and markedly reduced adhesion of platelets are
present. Prothrombin and thrombin time are normal in this patient. Her differential blood count
is normal too. What is the most possible diagnosis?
A. Hereditary thrombocytopathy
B. Acquired thrombocytopenia
C. Hereditary thrombocytopenia
D. Hemophilia
E. Acquired thrombocytopathy

35. A patient complains of frequent hemorrhages from gums. When carrying out blood test in this
patient deficiency of factor of hemocoagulation (prothrombin) was revealed. What phase of
blood coagulation is primarily impaired in the patient?
A. Formation of thrombin
B. Fibrinolysis
C. Formation of fibrin
Dz. Retraction of blood clot
E. Formation of prothrombinase

36. A 12-years-old patient was admitted to the hospital with hemarthrosis of knee joint
(hemorrhage into joint cavity). This patient has been suffering from hemorrhages since early
childhood. What disease does this boy suffer from?
A. Hemophilia
B. Hemorrhagic vasculitis
C. Iron deficiency anemia
D. Vitamin Bi2 deficiency anemia
E. Thrombocytic purpura
37. A 30-years-old female patient, who is suffering from megrim, often takes analgin.
Hemorrhages on skin and frequent nasal bleedings appear in her at the recent time. At her
blood analysis: number of platelets is 30x109/L; bleeding time is increased. What do these
changes result from?
A. Autoimmune thrombocytopenia
B. Hemorrhagic vasculitis
C. Hemolytic anemia
D. Angiohemophilia
E. Thrombocytopathy

38. What kind of pathologic processes underlie formation of thrombus?


Astringent metamorphose of thrombocytes
MoOOw>

Increase of osmotic pressure


Decrease in number of thrombocytes
Increase of oncotic pressure
Decrease in number of erythrocytes

39. A worker at pharmaceutical plant addresses the doctor with complaints of general malaise,
significant hemorrhages from gums, nasal bleedings, and numerous subcutaneous
hemorrhages. At blood analysis of this patient following was revealed: number of erythrocytes
is 2.2x1012, content of hemoglobin is 48 g/L, presence of neutropenia with relative
lymphocytosis, and number of platelets is 35x109/L. What is the possible pathogenesis of
thrombocytopenia in this patient?
Decrease of platelet production
mMoOOw>

Enhanced platelet destruction


Increase platelet utilization
Redistribution of platelets
Increased loss of platelets
40. Intravenous injection of mercury chloride to experimental animal evokes development of
thrombosis in it. What is the basic pathogenic factor in development of this pathologic process?
A. Injury of vessel wall
C. Activation of coagulation system
B. Activation of platelets adhesion
D. Slowing-down of blood flow
E. Decrease in anticoagulants activity

L-E, 2.-C, 3.-C, 4.-B, 5.-A, 6.-D, 7.-B, 8.-D, 9.-B, 10.-B, 11.-D, 12.-C,
13.-C, 14.-B, 15.-A, 16.-C, 17.-D, 18.-A, 19.-C, 20.-C, 21.-A, 22.-B, 23.-E, 24.-D, 25.-A, 26.-A,
27.-B, 28.-B, 29.-B, 30.-E, 31.-C, 32.-E, 33.A, 34.-E, 35.-A, 36.-A, 37.-A, 38.-A, 39.-A, 40.-A.
Topic 19. Pathophysiology of cardiovascular system.

Thrombosis of coronary artery caused the development of myocardial infarction. What


mechanism of the impairment will be dominating in this disease?
A. Electrolyte-osmotic;
B. Asidotic;
C. Protein;
D. Lipid;
E. Calcic.

Acute cardiac insufficiency appeared in a patient with arterial hypertension due to


hypertensive crisis. What mechanism of cardiac insufficiency is the main in this case?
A. Overload of heart by resistance;
B. Absolute coronary insufficiency;
C. Relative incompetence;
D. Overload of heart by rush of blood;
E. Myocardial impairment.

A patient aged 59 was hospitalised at cardiological department in a severe state with the
diagnosis of acute myocardial infarction of the posterior wall of the left ventricle and septum,
and starting pulmonary edema. What is primary mechanism which causes the development of
pulmonary oedema in the patient?

A. Pulmonary arterial hypertension;


B. Left ventricular failure;
C. Pulmonary venous hypertension;
D. Hypoxemia;
E. Decrease of alveolocapillary diffusion of oxygen.

A female patient aged 18, complains of general weakness, quick fatigability, depressed mood.
She has asthenic type of constitution. Pulse 68 per min., AP-90/60mm Hg. Primary
neurociculatory arterial hypertension was diagnosed. What is the main factor of decreasing of
arterial pressure in a patient?
A. Decrease of minute volume of the blood;
B. Decrease of cardiac output;
C. Decrease of the tension of resistant vessels:
D. Hypovolemia;
E. Deposition of the blood in the vines of systemic circulation.

A patient has stable and marked increase of arterial pressure, increased extracellular fluid
volume, increased content of Na+ and decrease of K+ in the blood, positive effect of saluretic
treatment. What is the mechanism of development of hypertension in the patient?
A. Mineralocorticoid B. Renin-angiotensine
C. Renovascular
D. Reflexogenic
E. Cento-ischemic

While climbing upstairs on the 5tn floor a patient has got an increased arterial pressure. The
cause is the increase of:
A. Minute volume of the blood
B. The number of functioning capillaries
C. Content of ions in blood plasma
D. Viscosity of the blood
E. Circulating volume of the blood

On the diagnosis of myocardial infarction the main role belongs to enzymodiagnosis. The
definition of content level in the blood of what enzyme is the most important during the first
2-4 hours after infarction?
A. Aldolase
B. Lipoprotein lipase
C. Alanine aminotranspherase
D. Creatin phosphokinase
E. Acetylcholinesterase

On analysis ECG it was determined: sinus rhythm, correct, interval RR is 0.58 sec, location
and duration of other intervals, waves and segments are not changed. Call the type of
arrhythmia:
A. Sinus tachycardia
B. Sinus bradycardia
C. Indioventricular rhythm
D. Sinus arrhythmia
E. Ciliary arrhythmia
One of the most dangerous moments in pathogenesis of myocardial necrosis is the further
increase of the zones of necrosis, dystrophy and ischemia. The important role in this belongs
to the increase of the use of the oxygen by myocardium. What substance contributes to this
process?
A. Chlorine ion
B. Cholesterol
C. Catecholamine
D. Acetylcholine
E. Adenosine
10. The functioning of certain structures is stopped on the isolated heart by means of cooling.
What structure is cooled if due to this the contractions stopped at first, but then they began
with a rate 2 times slower than initial one?
A. Sinoatrial node B. Purkinje’s fibres
C. Limbs of His’ bundle
D. Atrioventricular node
E. His’ bundle

11. A patient with chronic glomerulonephritis has edema, BP is 210/100 mmHg; the rate of
heartbeat is 85 per minute; the borders of the heart are dilated. What is the leading mechanism
in the development of arterial hypertension?
A. Increase of the activity of sympathetic adrenal system
B. Hyperfunction of the heart
C. Activation of renin-angiotensin-aldosterone system
D. Increase of circulating volume of the blood
E. Increase of vasopressin output

12. The patient’s ECG shows that interval RR=1.5 sec, heart rate - 40 per min. What is the
pacemaker of the heart?
A. Left limb of His’ bundle
B. Sinus node
C. His’ bundle
D. Right limb of the His bundle
E. Atrioventricular node

13. Pulmonary edema developed in a patient with hypertonic crisis.


What is the main factor in the pathogenesis of his state?
A. Increase of arterial pressure
B. Permeability increase of the vessels of pulmonary circulation
C. Increase of hydrostatic pressure in the capillary of the lungs.
D. Resistibility increase of the lung vessels
E. Decrease of oncotic pressure of blood plasma.

14. During the examination of blood for activity of AsAT and AIAT in the patient who
complained of pain in the chest and in upper part of the abdomen, the following results were
received: activity of ASAT 2 times higher than AIAT activity. What disease does the patient
have?
A. Acute infectious hepatitis
B. Acute pancreatitis
C. Myocardial infarction
D. Chronic hepatitis
E. Cirrhosis of the liver
15. Redistribution of organ blood supply took place in a young man, aged 20 during the load.
What organ did the blood flow increase in most of all?
A. Brain
B. Kidneys
C. Liver
D. Skeletal muscles
E. Heart

16. Clinical signs of developing pulmonary edema appeared in a patient with cardiac insufficiency
of left ventricular type. Which of the pointed pathogenic mechanism is the primary in such
pathology?
A. Hydrodynamic
B. Congestive
C. Colloid-osmotic
D. Lymphogenous
E. Membranogenous

17. A patient has cyanosis, increase of the liver, oedema of the lower extremities due to the right
ventricular insufficiency. What is the cause of the development of right ventricular failure?
A. Cardiogenic cirrhosis of the liver.
B. Functional shunting in lungs
C. Hypercatecholaminemia
D. Increase of venous pressure
E. Hypotension of pulmonary circulation

18. A woman, aged 25, complains of constant pain in the heart area, breathlessness on movement,
and general malaise. She has pale and cold skin, acrocyanosis. Her pulse is 96/min and her
BP is 105/70 mmHg. Heart border in her shifted 2 cm left. The first sound is weakened over
the apex of heart; there is systolic murmur over the apex. Diagnosis is insufficiency of the
mitral valve of the heart. What is the cause of the blood circulation failure?
A. Myocardial overload by the increased blood volume
B. Myocardial overload by the increased of resistance of blood outflow
C. Myocardial failure
D. Volume decreased of circulating blood
E. Volume increased of vascular bed

19. A patient with acute myocardial infarction was being given 1500ml of different solutions
intravenously during 8 hours, oxygen intranasally. Death occurs due to pulmonary edema.
What was the cause of the pulmonary edema?
A. Overload of the left ventricle by the volume
B. Decrease of oncotic pressure due to hemodilution
C. Allergic reaction
D. Neurogenic reaction
E. Oxygen inhalation

20. Functional hypertrophy of the left ventricle of the heart developed in a sportsman, aged 20,
due to constant physical load. What morphofunctional process do these changes result from?
A. Increase of cell size and number of contractile organelles
B. Increase of fibroblast number
C. Increase of the number of conductive cardiomyocytes
D. Increase of the amount of connective tissue
E. Increase of the amount of fat tissue

21. A patient with renal disease accompanied by parenchyma ischemia has a high arterial
pressure. What leading factor is the cause of the increase of AP in this patient?
A. Excess of angiotensin IT
B. Excess of antidiuretic hormone
C. Increase of heart output
D. Increase of sympathetic nervous system tonus
E. Hypercatecholaminemia
22. During the operation reflex increased of vagus nerve influence on the heart happened. What
may occur in this case?
A. Cardiac arrest
B. Increase of atrioventricular node conduction
C. Increase of myocardium conduction
D. Intensification of myocardium contractions
E. Increase of heart rate
23. Decrease of R-R interval was revealed on ECG of a man. What changes in the heart work are
observed in this case?
A. Increase of heart rate
B. Decrease of heart rate
C. Increase of force of contractions
D. Decrease of force contraction
E. Decrease of force and rate of contractions
24. Large amount of isoenzymes of creatine kinase of MV-form was revealed in the blood of the
patient with destructive changes in the muscular tissue. What is the most possible diagnosis?
A. Myocardial infarction
B. Muscular atrophy
C. Muscular dystrophy
D. Polymyositis
E. Myopathy
25. Patient’s attack of tachycardia was stopped by pressing on the eyeballs (Danini-Ashner
reflex). In the decrease of the heart rate there is intensification of the influence on the sinoatrial
node of:
A. Vagus nerves
B. Sympathetic nerves
C. Autonomic nervous system
D. Sympathoadrenal system
E. Catecholamines
26. Considerable increase of PQ interval was found out on ECG. It means that conduction of
stimulation is delayed by:
A. AV node
B. Atria
C. His’ bundle
D. Purkinje’s fibres
E. Ventricles

27. Sharp marked pains in the substernal area that radiate to the left arm cannot be controled by
nitro-glycerine for 30 minutes. What changes developed in the patient’s hearts?
A. Myocardial ischemia
B. Pathological myocardial hypertrophy
C. Sharp increase of coronary blood flow
D. Mitral incompetence
E. Inflammation of pericardium

28. The activity of what enzymes is it necessary to determine in pathology of cardiac muscle with
diagnostic and prognostic aim?
A. Arginase, peptidase, phosphatase
B. Decarboxylase, decanidase, lactate dehydrogenase
C. Creatin kinase, transaminase, lactate dehydrogenase D. Lysozyme, citrate
synthatase, succinate dehydrogenase
E. Neuroaminase, aldolase, hexakinase.

29. A severe stress was caused in an experimental animal. Necrotic injuries of myocardium
developed in this stage. What is the main cause in the pathogenesis of these injuries?
A. Increase of calcium content in cardiomyocytes
B. Decrease of adenosine triphosphoric acid synthesis in mitochondria
C. Changes in the work of Na+-K+ pump
D. Insufficiency of coronary circulation
E. Decrease of adenosine triphosphoric acid activity of myosin
30. A patient male has had a chronic disease of kidneys for 12 years. AP - 200/130mmHg. Pulse
-75 beats per min. The main factor that causes the increase of pressure in this case is the
increase of:
A. Minute volume of the heart
B. Heart rate
C. Circulating volume of the blood D. Systemic peripheral resistance
E. Venous recurrence.

31. During the attack of heartbeat a patient with thyrotoxicosis has an irregular pulse of different
filling, pulse deficiency is observed. Waves P are absent; small in amplitude, disorderly
undulations (P waves), and irregular ventricular complexes of ordinary configuration are
noted.
What kind of rhythm impairment is observed in a patient?
A. Sinus tachycardia
B. Sinoatrial block
C. Ciliary arrhythmia
D. Paroxysmal tachycardia
E. Ventricular extrasystole

32. At examination of a patient strengthening of a second pulmonic sound, hypertrophy of the


right ventricle wall are determined. What changes of hemodynamic take place in pulmonary
circulation?
A. Hyperperfusion of the lungs
B. Hypoperfusion of the lungs
C. Manifestation of Hering-Breuer reflex
D. Development of broncho-alveolar vascular anastomosis
E. Spasm of resistant vessels in the lungs.

33. A patient with mitral failure has an enlargement of the liver, edema of lower limbs. What is
the leading mechanism of the development of cardiac edema?
A. Increase of venous recurrence
B. Increase of tissue drainage
C. Participation of renin-angiotensin-aldosteron system D. Decrease of oncotic
pressure
E. Decrease of transudation.

34. At examination a patient’s arterial pressure is 190/100 hg. What factors leads to increase of
arterial pressure?
A. Spasm of resistance vessels
B. Increase of venous recurrence
C. toxygenic dilation of cardiac muscle D. Aler-Lilestrand reflex
E. Kitaevs reflex.

35. A patient with myocardial infarction has a mark paleness of skin, oliguria, AP 100/90 mm Hg,
and pulse 100 beats/min. What compensative mechanism maintains relative high level of AP?
A. Hypokalemia
B. Hypoperfusion of the lungs
C. Centralization of blood circulation
D. Increase of the level of vasodilators in blood
E. Secondary aldosteronism

36. Marked frequency of the patients pulse was determined during the examination, what is sinus
tachycardia due to?
A. Hypothyrosis
B. Hypokalemia
C. Tonus increase of vagus nerve
D. Increase of speed of slow diastolic depolarisation
E. Excess of acetylcholine

37. Considerably slow pulse was determined in a patient at examination. What is sinus
bradycardia due to?
A. Decrease of speed _of slow diastolic depolarisation
B. Hypercatecholaminemia
C. Decrease of circulating blood volume
D. Hyperkalemia
E. Haemic hypoxia

38. In cardiac pathology homeometric mechanism of compensation in the work of the left
ventricle takes place in:
A. Stenosis of atrioventricular foramen
B. Mitral incompetence
C. Aortic incompetence
D. Hypertension of pulmonary circulation
E. Hypertension of systemic circulation.

39, In cardiac pathology heterometric mechanism of compensation connected with overloading


of left ventricle volume takes place in:
A. Stenosis of atrioventricular foramen
B. Stenosis of aortic osteum
C. Aortic incompetence
D. Hyper tension of pulmonary circulation
E. Hypertension of systemic circulation.

40. Sharp increase of AsAT activity was determined in the blood serum of a patient 12 hrs later
after an acute attack of pain in the substernal area. Which of the pathogenesis is the most
possible?
A. Collagenosis
B. Diabetes mellitus
C. Myocardial infarction
D. Virus hepatitis
E. Diabetes insipidus

41. Pressing pain in the heart area with irradiation to the left arm, neck and under the left shoulder
blade suddenly appeared in a male patient aged 45, after considerable psycho-emotional
exertion. His face became pale and covered with cold perspiration. Nitro-glycerin relieved the
attack of pain. What process in the patient?
A. Angina pectoris
B. Perforation of stomach ulcer
C. Psychogenetic shock
D. Myocardial infarction
E. Insult

42. In recreation of the arterial hypertension in a dog a thickness in the left ventricle wall increased
1.7 times in a month, but the circulating blood volume was not changed in comparison with
the initial data. What stage of myocardial hypertrophy is observed in the animal?
A. Initial
B. Repair
C. Complete hypertrophy
D. Decompensation
E. Progressive cardiosclerosis

43. The rate of spread of pulse wave turned out to be considerably higher in a man aged 70, than
in a man aged 25. The cause of this is decrease of:
A. Arterial pressure
B. Cardiac output
C. Elasticity of vascular wall
D. Rate of cardiac contractions
E. Circulation rate

44, On recording ECG of a patient with hyperfunction of the thyroid gland increase of rate of
cardiac contractions was registered. Shortening of what ECG element indicates this?
A. Interval R-R
B. Interval P-Q
C. Interval P-T
D. Segment P-Q
E. Complex QRS
45. A patient excretes water from the organism less than he uses it for 24 hours. What disease
may lead to this state?
A. Cardiac insufficiency
B. Pancreatitis
C. Cystitis
D. Hepatitis
E. Infectious diseases

46. A man has got an electro trauma. Current went through the cardiac muscle. What dangerous
impairment in the work of the heart demanding urgent measures may appear in this situation?
A. Bradycardia B. Extrasystole
C. Atrial fibrillation
D. Ventricular fibrillation
E. Tachycardia

47. A patient who underwent myocardial infarction a month and a half ago had Dressler’s
syndrome with typical triad: pericarditis, pleurisy, and pneumonia. The cause of its
development is:
A. Sensibilization of the organism by mvocardial antigens
B. Decrease of resistance to infectious agents
C. Activation of saprophytic micro flora
D. Intoxication of the organism by necrotic products
E. Injection of myocardial enzymes in the blood.

48 A patient with rheumatism had incompetence of the left atrioventricular foramen and
decompensation of cardiac activity. The characteristic hemodynamic index of this state is:
A. Decrease of cardiac output
B. Slowing down of blood flow
C. Decrease of arterial pressure
D. Increase of venous pressure
E. Widening of microcirculatory bed
49. A patient with rheumatic myocarditis began to feel intermissions in the work of heart. By
means of auscultation of the heart, feeling the pulse and ECG investigation it was determined that
this was connected with the appearance of ventricular extra systoles the characteristic feature of
which is compensatory pause. Its appearance is due to: A. Refractivity of ventricular
myocardium to the next impulse
B. Retention of stimulation in atrioventricular node
C. Retrograde conduction of stimulation to atria
D. Inhibition of function of sinoatrial node
E. Impairment of contraction of ventricular myocardium

50. Considerable increase of myocardial mass of left ventricle was determined in a patient with
hypertension. It was due to:
A. Increase of cardiomyocyte volume
B. Increase in amount of cardiomyocytes
Enlargement of connective tissue

IF [FIA
Retention of water in myocardium
Fat infiltration of myocardium

51. A patient with aortic atherosclerosis has left ventricular hypertrophy as a compensatory
phenomenon. Compensatory role of hypertrophy comes to:
Normalization of load on each cardiomyocyte
IF |F 10 I |>

Improvement of delivery of oxygen to myocardium


Activation of synthesis of macroergs in myocardium
Economical use of energy by cardiomyocytes
Increase of stroke volume of the blood

52. A patient who is treated for myocardial infarction at the in-patient department has lost
suddenly his consciousness several times a day. During the attack pulse is absent, heart sounds are
not heard, face is cyanotic, and convulsions appear, arterial pressure is not determined.
The diagnosis is Morgagni-Adams-Stokes syndrome. It appears due to:
Development of full atrioventricular blockade
IF |F 10 I >

Weakness of sinoatrial node


Attack of ventricular paroxysmal tachycardia
Impairment of excitability of ventricular myocardium
Cardiosclerotic changes in the heart

53. _Electrocardiography examination of a patient with hypertension showed such results: right
sinus rhythm, rate of cardiac constrictions is 92 per min; duration of PQ - 0.2 sec, QRS without
change, a patient has disturbance of:
A. Refractivity
B. Conduction
Cc. Automatism
D. Stimulation
E. Contractility

54. By electrocardiogram investigation the following data were determined in a patient who
had suffered the grippe; the rate of heart is 140 beats per min, sinus rhythm, the range of R-R is
not more than 0.15 c; duration of PQ-0.2 c; QRS is not changed. These indicate of the development
of:
Sinus tachyarrhythmia
I= 15 10 I [>

Sinus tachycardia
Nonparoxysmal tachycardia
Paroxysmal tachycardia
Ventricular fibrillation
55. The signs of heart failure appeared in a man aged 56, during carrying out hard work,
feelings of air shortage, heart beating, and general weakness. Objectively heart borders are dilated,
the heart rate is 92 beats per min and arterial pressure is 180/110 mm hg. These signs are due to:
Increase of peripheral resistance

I= |F 10 I |>
General hypoxia of an organism
Insufficiency of coronary circulation
Increase of diastolic filling
Neurotrophic disturbances

56. A patient aged 47 with mitral incompetence has the symptoms of cardiac insufficiency:
breathlessness, cyanosis, oedema of lower limbs. Objectively: the borders of the heart are dilated,
the heart rate is 104 beats per min, and arterial pressure is 125/85 mm hg. These symptoms are
due to:
Increase of peripheral resistance
IF |F 10 Ie I>

Autoallergic changes of myocardium


Increase of diastolic filling
Insufficiency of coronary circulation
Neurotrophic disturbances

57. Dilation of the heart, AP-155/100 mm hg, the heart rate 95 beats per min, was revealed in
a patient aged 63 with hypertension during the examination. The most effective mechanism, which
will contribute to the normalization of AP, is:
Inhibition of aldosteron synthesis
I= |S 10 I [>

Blocking of angiotensin synthesis


Inhibition of catecholamine action
Blocking of vasopressin action
Administration of salt low diet

58. After suffered rheumatism a patient had aortic stenosis. Point out what mechanism of
compensation takes place in the left ventricle:
A. Homeometric
B. Heterometric
Cc. Systolic
D. Diastolic
E. Coronary
59. After suffered rheumatism a patient had aortic incompetence. Indicate what mechanism of
compensation takes place in the left ventricle in this case:
A. Homeometric
B. Heterometric
CG Systolic
D. Diastolic
E. Coronary
60. Acute myocardial infarction of a patient includes three areas: zone of necrosis, dystrophy
and ischemia. The changes of what ECG wave characterises the place of necrosis in this case?
A R
BS
c& = Q
D OT
E P
61. A patient with acute myocardial infarction, which was caused by thrombosis of coronary
arteries, was treated with fibrinolytc preparation urkinase that led to the development of
reperfusion syndrome. Which of the mentioned syndromes is associated with reperfusion?
A. Dressler’s
B. Chediak-Higashi
CG Morgagni-Adams-Stokes
D. No-reflow
E. Wolff-Parkinson-White

62. Which of the factors is the most frequent cause of formation of acquired valvular heart
disease?
A. Septic endocarditis
B. Syphilis
CG Atherosclerosis
D. Mechanical factor
E. Rheumatism
63. Acute pain in the heart area, marked skin paleness and loss of consciousness developed in
a patient after short intensive physical excretion. What heart failure may be suspected in this
patient?
Mitral incompetence
IF |S Ol>

Stenosis of aortic osteum


Stenosis of mitral orifice
Aortic incompetence
Stenosis of the right atrioventricular opening
64. A patient aged 40, complains of increased fatigue, appearance of pain in the heart area on
physical exertion. Objectively: cyanotic skin (especially on the fingers of the hands and legs,
lobule of the ear), pulse rate 96 beats/min, AP-110/85 mm of hg. There are oedemas in the lower
and middle third of the cruses. The edge of the liver projects 3 cm from under the costal arch.
Which form of circulatory insufficiency may be suspected in this patient?
Acute left ventricular failure
I= |S 10 I |>

Acute vascular insufficiency


Chronic left ventricular failure
Chronic right ventricular failure
Acute right ventricular failure
65. A patient complains of breathlessness that appears even on slight physical exertion, cough
with sputum, which recently has obtained bloody character. Objectively: the skin of the face and
mucous membrane of the lips particularly are cyanotic. Which impairment of the heart work may
be suspected in this patient?
Stenosis of mitral orifice
Mitral incompetence
Stenosis of aortic osteum
Pirin

Aortic incompetence
Stenosis of the right atrioventricular opening
66. A patient has constant high arterial pressure- 160/110 mm of hg. Periodically it may rise
up to 220-240/120-130 mm of hg. What type of hypertension has this patient?
A. Psycho emotional
B. Cerebral
CG Renal
D. Adrenal
E. Essential

67. A patient, female, aged 48 complains of breathlessness on slight physical exertion, pains
of pressing character in substernal area, appearance of syncope on exertion. According to a case
history the patient suffered rheumatism. Heart failure was diagnosed - stenosis of aortic osteum,
myocardial hypertrophy. What is the main mechanism of the development of myocardial
hypertrophy?
Hypertrophy of myocardiocytes
IF |F 10 Ie |>

Diffuse oedema of interstice


Synthesis increase of contractile proteins in myocardium
Hyperplasia of intermediate tissue of myocardium
Accumulation of water and electrolytes in myocardiocytes

68. A patient 45 complains of breathlessness on slight physical exertion, oedema of the legs,
frequent quinsies are pointed out in case history, she is being ill for 2 years. Rheumocarditis,
combined mitral incompetence, insufficiency of blood circulation were diagnosed. What is
hemodynamic mechanism of decompensation?
Decrease of circulating blood volume
I= |S 10 I [>

Decrease of venous pressure


Increase of arterial pressure
Decrease of minute volume of the heart
Tachycardia

69. A patient with ischemic heart disease had a sudden severe attack of angina pectoris: the
face is pale, cold, damp skin, AP-70/50 mm Hg, extrasystolia. Myocardial infarction and
cardiogenic shock were diagnosed. Name the leading link of pathogenesis?
A. Hypotension
Pain syndrome

PIP lol
Toxaemia
Extrasystolia
E. Decrease of minute blood volume
70. Mitral incompetence without impairment of blood circulation was revealed in a patient
aged 25. What mechanism provided the state of the heart compensation?
Heterometric mechanism
I= |S 10 I [>

Homeometric mechanism
Inotropic action of catecholamines
Increase of the heart mass
Strengthening of protein synthesis

71. Aortic stenosis was revealed in a girl aged 15, but without impairments of blood
circulation. What the mechanism provided the condition of heart compensation?
Homeometric mechanism
I= |S 10 I [>

Inotropic action of catecholamines


Heterometric mechanism
Decrease of heart mass
Strengthening of protein synthesis

72. ECG of a child aged 5 shows the impairment of action of cardiac rhythm. On holding
breathing the cardiac rhythm becomes normal. What kind of impairment was found out on ECG?
Sinus extrasystole
I= |F 10 I [>

Ciliary arrhythmia
Respiratory arrhythmia
Atrial extrasystole
Transversal heart block

73. Strong palpitation, pain in the heart, sharp weakness, increase of AP, irregular pulse with
deficiency developed suddenly in a man aged 50. ECG showed absence of wave P and different
R-R intervals. What impairment of cardiac rhythm has a patient?
Transversal heart block
I= |S 10 I [>

Paroxysmal tachycardia
Respiratory arrhythmia
Ciliary arrhythmia
Sinus extrasystole

74. | ECGshow that wave P overlaps the wave T what is observed in case of atrial obstruction.
At what rate of the heart contractions will it occur?
A. 150 beats per minute
B. 160 beats per minute
CG 170 beats pr minute
140 beats per minute

I= |S
130 beats per minute

75. During the experiment a white rat was given intraperitoneal injection of 0.1% adrenalin
solution at 1 mg/100g of body mass. 30 minutes later respiration become hurried and shallow, then
sharp and convulsive, foamy fluid appeared from the nasal cavity, there was acrocyanosis. The
animal died when acute pulmonary edema developed. What pathogenetic mechanism is the
leading in this case?
Toxic
>

Membranogenous
Lymphogenous
IF |F lO

Hydrodynamic
Colloid-osmotic

76. Patient V., aged 67 suffers from atherosclerosis of heart vessels and brain vessels.
Hyperlipidemia was revealed on examination. What class of lipoproteins of blood serum is of
greatest importance in the pathogenesis of atherosclerosis?
Lipoproteins of low density
I= |F 10 I |>

Chylomicrones
Beta lipoproteins
Lipoproteins of high density
Complex of fat acids with albumins

77. The planned investigation of cardiohemodinamics readings of patient M. with chronic


cardiac insufficiency was carried out. Which of the named readings is the main sign of the
development of cardiac decompensation?
Development of tachycardia
I= |F 10 Ie |>

Decrease of stroke volume


Tonogenic dilatation
Increase of peripheral vascular resistance
Increase of central venous pressure

78. While analyzing the ECG of the patient with myocardial infarction attention was paid to
absence of wave P, presence of wavy isoelectric line with plenty of small waves, frequent and
irregular location of QRS complex. What impairment of rhythm does the described picture of ECG
correspond to?
A. Ventricular fibrillation
B. Idioventricular fibrillation
CG Atrial fibrillation
D. Paroxysmal tachycardia
E. Ventricular extrasystole
79. Periodical appearance of ventricular extrasystole was revealed during ECG investigation
of patient P. It was determined that wave P was absent before extrasystole. What is the cause of
its disappearance?
Appearance if refractory period in atria

I= |S 10 I [>
Impulse block in sinus node
Appearance of refractory period in ventricles
Block of impulse conduction round the atria
Impossibility of retrograde conduction through A-V_ node 80. During
ultra-sound investigation dilation of heart cavities was revealed in patient 8. who suffered
from hypertension. Which of the below mentioned signs are evidence of development of
tonogenic dilation?
A, Dilation of heart cavities with increase if stroke volume
B. Dilation of cavities without change if stroke volume
C, Dilation of cavities with change of stroke volume
D. Uniform dilations of heart borders
E. Irregular dilations of heart borders

81. Quick fatigue, breathlessness on physical exertion, feeling of “sink” and momentary cardiac
arrest appeared in a patient D, aged 13, 2 weeks later after secondary suffered tonsillitis. ECG
showed decrease of voltage of waves periodical prolapse of some cardiac cycles PORST,
RCC-55 beats/minute. What is the most possible mechanism of the impairment of heart
functions?
A. Toxic impairment of sinus node
B. Impairment of sinus node by immune complexes
C. Sclerotic changes in sinus node
D. Ischemic impairment of sinus node
E. Parasympathetic irritation of sinus node

82. During ECG investigation irregular atrioventricular extrasystoles were determined in a patient
aged 38. The impairment of what properties of myocardium composes the base of extra systole
pathogenesis?
A. Excitability
B. Automatism
C. Conduction
D. Contractility
E. Tonicity

83. Patient Y, aged 40, is treated for pulmonary tuberculosis, acrocyanosis, breathlessness,
extension of heart borders, increase of AP and number of erythrocytes, pachyemia,
neutrophilic leukocytosis are marked on examination. Which of the below mentioned
symptoms of chronic hypoxia are regarded as lasting compensatory mechanisms of organism.
A. Pachvemia
B. Dilation of the cavities and myocardial hypertrophy
C. Leucocytosis
D. Increase of AP
E. Increase of respiratory rate.

84. Increase of blood viscosity was revealed in a patient with chronic cardiac insufficiency, the
damage of the walls of vessels of microcirculatory channel was found out on capillaroscopy.
Which of the impairments of peripheral circulation are possible in this case?
A. Slage phenomenon
B. Thrombosis
C. Arterial hyperemia
D. Embolism
E. Venous hyperaemia.

85. Changes that testify to the impairment of cardiomyocytes were recorded on the ECG of an
experimental animal after the injection of uabain (substance which blockades K+/Na+-
depending ATP). What molecular mechanisms had decisive significance in this case?
A. Electrono-osmotic
B. Lipid
C. Acidotic
D. Calcic
E. Protein

86. Patient, aged 44, complains of strangulation, palpitation, pains in the right costal interspace,
and edemas on the legs. Pulsation of cervical veins, enlargement of the liver, edemas of the
lower extremities were found out on examination. ECG shows the signs of hypertrophy of both
ventricles and right auricle. Tricuspid incompetence was diagnosed.
What is pathophysiological variant of this incompetence?
A. Overloading of the heart by exertion
B. Primary myocardial insufficiency
C. Overloading of the heart by volume
D. Coronary insufficiency
E. Cardiac tamponade.

87. Acceleration of the heartbeat during breathing in and deceleration during breathing out were
revealed in a youth, aged 16, on examination. ESG showed shortening of RR interval during
inspiration and its lengthening during expiration. Name the kind of arrhythmia.
A. Ciliary arrhythmia
B. Sinus tachycardia
C. Idioventricular rhythm
D. Sinus arrhythmia
E. Sinus bradycardia
88. Choose in what form below mentioned states that cause of cardiac insufficiency heterometric
mechanism of compensation (Frank-Starling) takes place:
A. Hypertension of systemic circulation
B. Hypertension of pulmonary circulation
C. Stenosis of mitral orifice
D. Stenosis of aortic osteum
E. Aortic incompetence

89. A patient, aged 58, was hospitalised with complaints of unpleasant feelings in the heart area;
there was myocardial infarction in anamnesis. ECG shows registration of fibrillation waves
(F) instead of P-waves; QRST complexes are widened and deformed. Ventricular rhythm is
right; the rate of contractions of ventricles is 48 per minute. What is the most possible
impairment of the rhythm in this case?
A. Frederic’s syndrome
B. Morgagni-Adams-Stokes syndrome
C. Wolf-Parkinson-White syndrome
D. Atrioventricular block of II stage. Mobits’ type I
E. Atrioventricular block of II stage. Mobits’ type II
90. Name the condition, which accompanied by pressure overload of left ventricle.
A. Stenosis of mitral orifice
B.S is of ic orifi

C. Incompetence of mitral valve


D. Incompetence of aortic valve
E. Pulmonary hypertension

91. A 24-year-old female patient was admitted to the hospital with complaints of headache, pain
in kidney area, and general weakness. She was suffered from tonsillitis one month before. At
examination patient has BP of 180/110 mmHg; in patient’s blood analysis erythrocytes —
3.1x10%12/L, leucocytes — 12.6x10°9/L, ESR — 28 mm/hour; in patient’s urinalysis — marked
proteinuria, hematuria, leukocyturia. What is the mechanism of hypertension development?
A. Renovascular
B. Reflexogenic
C. Aldosteron-induced
D. Renal
E. Mineralocorticoid-induced

92. In a patient with hypertension the pulse rate dropped from 72 to 52 bpm during hypertonic
crisis and maintained at this level for 10 days. Intramuscular injection of 1 mg of atropine led
to increasing of pulse rate at 16 bpm. What group of arrhythmias does described disorder of
cardiac rhythm belong to?
A. Ventricular fibrillation
B. Atrial palpitation
C. Disturbance of rhythm formation
D. Disturbance of rhythm conduction
E. Disturbance of automatism
93. Decrease in pulse rate down to 50 bpm was observed in a patient with cerebral hemorrhage.
His pulse was rhythmic. What is the mechanism of this disorder of cardiac rhythm?
A. Irritation of nervus vagus
B. Reduction of rate of free diastolic repolarization
C. Influence of sympathetic mediator
D. Extension of site of sinoatrial node
E. Elevation of rate of free diastolic repolarization

94. A patient died from myocardial infarction, which was proved by data of clinical and
electrocardiographical examinations. No changes of coronary vessels were found at autopsy
of this patient. What evokes myocardial infarction in this patient?
A. Increase in tonus of sympathetic nervous system
B. Enhanced secretion of catecholamines
C. Obstruction of vessel with embolus
D. Thrombosis of coronary vessels
E. Rheumatic coronaritis

95. A patient with neuro-circulatory distony has heartbeat rate increased up to 130 bpm. Clinical
symptoms of organic heart injury were not found in this patient. At pressing on carotid sinus
heart rate decreased, but after returned to previous frequency. What is the origin of this disorder
of heart rhythm?
A. Fluctuations of parasympathetic tonus
B. Inflammatory lesion of myocardium
C. Enhanced influence of sympathetic nervous system
D. Ischemic injury of myocardium
E. Toxic injury of myocardium

96 . Thrombosis of anterior intraventricular coronary artery developed after coronarography in a


patient with ischemic heart disease and atherosclerosis of coronary arteries. What mechanism
is the most significant for development of this complication?
A. Slowing of blood flow
B. Injury of endothelium of vessel wall
C. Increase in blood coagulant concentrations
D. Decrease in blood anticoagulant concentrations
E. Reduction of fibrinolytic system activity

97. Under hypertrophy of myocardium mass of heart increases due to:


A. Enlargement of each muscular fiber
B. Growth of connective tissue
C. Increase in number of myocardiocytes
D. Growth of adipose tissue
E. Enhancement of blood supply of heart muscle

98. Changes in ECG was shown at first by lengthening of P-Q interval, after that by falling out of
single QRS complexes, later by increasing in number of fallen out ventricle complexes, and at
last atriums constrict with frequency of 70 bpm and ventricles constrict with frequency of 35
bpm. Described changes are characteristic for:
A. Intraatrial block
B. Cross-sectional block
C. Intraventricular block
D. Arrhythmia due to disturbances of automatism
E. Arrhythmia due to disturbances of conduction

99. The first link in atherosclerosis development is:


A. Slowing of blood flow
B. Degenerative-proliferative changes of internal layer of arteries C. Excessive depositions
of blood plasma lipoproteins in internal layer of arteries
D. Formation of fibrous plaque on internal layer of arteries
E. Disorders of intactness of arterial wall
100. A patient suddenly lost consciousness and develop cramps. At his ECG followings were
revealed: 2 to 3 P cogs related to 1 QRST complex. What property of heart conductive system
is impaired?
A. Automatism
B. Excitability
C. Conduction
D. Constriction
E. Reproduction of frequency of excitement

ANSWERS: 1.-E. 2.-A. 3.-B. 4.-C. 5.-A. 6.-A. 7.-D. 8.-A. 9.-C. 10.-A. 11.-C. 12.-B. 13.-C. 14.-
C. 15.-D. 16.-B. 17.-E. 18.-A. 19.-A. 20.-A. 21.A. 22.-A. 23.-A. 24.-A. 25.-A. 26.-A. 27.-A. 28.-
C. 29.-D. 30.-D. 31.-C.
32.-E. 33.-C. 34.-A. 35.-C. 36.-D. 37.-A. 38.-E. 39.-C. 40.-C. 41.-A. 42.C. 43.-C. 44.-A. 45.-
>

46.-D. 47.-A. 48.-A. 49.-A. 50.-A. 51.-A. 52.-A. 53.-C. 54.-B. 55.-A. 56.-C. 57.-C. 58.-A. 59.-
Ow

60.-C. 61.-D. 62.-E. 63.B. 64.-D. 65.-A. 66.-D. 67.-C. 68.-D. 69.-E. 70.-A. 71.-A. 72.-C. 73.-D
74.-C. 75.-D. 76.-A. 77.-B. 78.-C. 79.-E. 80.-A. 81.-B. 82.-A. 83.-A. 84.-A. 85.-A. 86.-C. 87. D.
88.-E. 89.-A. 90.-B. 91.-D. 92.-E. 93.-A.
94.-B. 95.-C. 96.-B. 97.-A. 98.-B. 99.-E. 100.-C.
Topic 20. Pathophysiology of respiratory system.

1. A young man with suspicion on narcotic poisoning was admitted into neurological department.
Which of the disorders of external respiration can be expected?
A. Asphyxia
B. Alveolar hyperventilation
C. Kussmal respiration
D. Biot’s Respiration
E. Alveolar hypoventilation

2. In pathogenesis of which types of respiration the main link is the fall of excitability of
respiratory centre to carbon dioxide due to oxygen starvation of this centre?
A. Accelerated and deep breathing
B. Expiratory dyspnoea
C. Periodic respiration
D. Inspiratory dyspnoea
E. Combined dyspnoea

3. As a result of casualty the obturation of the lung trachea occurred.


Which stage of respiration will be impaired first?
A. Tissue respiration
B. Lung ventilation
C. Exchange of gases in the lungs
D. Exchange of gases in tissues
E. Transport of oxygen and carbon dioxide

4. The functional state of respiratory system was examined in a patient with emphysema. What is
more characteristic for this state?
A. Increase of tidal volume
B. Increase of vital volume of the lungs
C. Increase of inspiratory reserve volume
D. Decrease of total volume of the lungs
E. Decrease of inspiratory reserve volume

5. The syndrome of respiratory insufficiency is often observed in premature bor children. What
is the main cause of this?
A. Swallowing of amniotic water
B. Immaturity of lung alveoli due to deficiency of surfactant
C. Intrauterine hypercapnia
D. Imperfection of the nervous regulation of the respiratory act
E. Intrauterine asphyxia

6. Paleness of the skin, accelerated superficial respiration is observed in a newborn. Numerous


diffused atelectasis are revealed at X-ray examination. What is the most possible cause of this
condition?
A. Pneumothorax
B. Hydrothorax C. Tuberculosis
D. Bronchial asthma
E. Surfactant deficiency

7. A patient who was at a resuscitation department with skull injury suddenly developed
convulsions on the background of regaining consciousness, and short arrest of breathing was
changed by solitary sighs with calming down character. What types of respiration appeared in
the patient?
A. Gasping respiration
B. Cheyne-Stokes respiration
C. Biot’s respiration
D. Kussmaul respiration
E. Apnoeystic respiration
8. During meal a child breathe in a seed. What respiratory changes will develop in the child?
A. First expiratory dyspnoea, then inspiratory one
B. At first inspiratory dyspnoea, then expiratory one
C. At first arrest of breathing, then expiratory dyspnoea
D. Expiratory dyspnoea, then Cheyne-Stokes respiration
E. Inspiratory dyspnoea, then Biot’s respiration

9. A partial respiratory insufficiency developed in a patient due to a chronic impairment of the


organs of respiratory system. What is a

characteristic sign of partial respiratory insufficiency?

A. Hypoxemia and hypercapnia


B. Hypoxemia and decrease of alveolar ventilation
C. Decrease of alveolar ventilation and hypercapnia
D. Hypoxemia and gas acidosis
E. Hypoxemia without hyperapnia

10. When developing of pulmonary emphysema in a patient the following is notified:


A. Increase of functional “dead space”.
B. Inspiratory dyspnoea
C. Cardiac insufficiency (left ventricular type)
D. Decrease of resistance to the airflow in the respiratory tract
E. Decrease of functional “dead space”

11. Decrease of the passage at the level of middle and small bronchi is observed in a patient.
What process will be the leading in the development of respiratory insufficiency?
A. Disturbance of diffusion
B. Hyperperfusion
C. Hypoventilation
D. Hypoperfusion
E. Hyperventilation

12. ‘In the decrease of the middle and small bronchial passages in a patient the following is
observed:
A. Development of gas alkalosis
B. Decrease of pressure in pulmonary circulation
C. Development of inspiratory dyspnoea
D. Hypocapnia
E. Decrease of pQ2 and increase of pCO: in the alveolar air

13. In obstructive type of respiratory disturbances in a patient it will be determined


A. Increase in forced vital volume of the lungs
B. Expiratory dyspnoea
C. Increase in vital volume of the lungs
D. Decrease of the tidal volume
E. Decrease of the total volume of the lungs
14. In restrictive type of respiratory disturbances in a patient the following is revealed
A. Decrease of rate and increase of depth of respiration
B. Expiratory dyspnoea
C. Increase of vital capacity of the lungs
D. Increase of tidal volume of the lungs
E. Decrease of tidal volume of the lungs

15. Gas alkolosis is revealed in the patient is blood. What process impairment is connected
with this development?

A. Impairment of diffusion
B. Hyperventilation
C. Hyperperfusion
D. Hypoventilation
E. Hypoperfusion

16. The application of oxygen did not eliminate hypoxemia in a patient with partial respiratory
insufficiency. What is the mechanism of respiratory insufficiency?
A. Disturbance of diffusion
B. Hyperperfusion (functional shunt)
C. Hypoventilation
D. Hyperperfusion (anatomical shunt)
E. Hyperventilation
17. Patient aged 62 was admitted into neurological department due to cerebral haemorrhage.
Grave condition. Increase of depth and rate of respiration and then its decrease and apnoea is
observed. After that the cycle of respiratory movements start again. What type of respiration
appeared in the patient?
A. Apneustic respiration
B. Kussmaul respiration
C. Gasping respiration
D. Cheyne-Stokes respiration
E. Biot’s respiration

18. After Typhno’s test index decreased to 30% in the patient. The development of what
pathological process in the organs of respiratory systems does it indicate?
A. Lobular pneumonia
B. Pneumothorax
C. Obstructive bronchitis
D. Tuberculous pleurisy
E. Pneumosclerosis

19. A patient has developed athelectasis, which was accompanied by alveolar collapse. What
contributes to this?
A. Hyperventilation
B. Spasm of lung vessels
C. Arterial hypertension
D. Surfactant deficiency
E. Respiratory acidosis

20. A patient has the pathology of the lung with the disturbance of external respiration and
development of hypercapnia and hypoxia. How many times is the ability of COz2 to diffusion
through alveolar-capillary membrane higher than the same one of oxygen?
A.5
B. 10
C. 15
D. 20
E. 25
21. Patient’s respiratory rate is 10 per min, respiratory volume - 1.1 1 whistling rals are heard
over the lungs on auscultation. Which of pathogenetic forms of external respiratory disturbance
has this patient:
A. Diffuse-pneumonia
B. Diffuse-restrictive
C. Ventilative-restrictive
D. Primary-dyskinetic
E. Obstructive

22. Which of the pathogenetic forms of external respiratory disturbance may develop in
traumatic costal fracture?
A. Primary-dyskinetic B. Ventilative-restrictive.
C. Obstructive
D. Diffuse-restrictive
E. Diffuse pneumonia

23. Which of pathogenetic forms of external respiratory disturbance may appear in


cerebrospinal trauma of the level of cervico-thoracic part of the spine?
A. Obstructive
B. Primary-dyskinetic
C. Ventilative-restrictive
D. Diffuse-restrictive
E. Diffuse pneumonic
24, In which of pathologic processes does obstructive form of disturbance of external
respiration appear?
A. Pulmonary edema
B. Pneumonia
C. Pleurisy
D. Bronchial asthma
E. Open pneumothorax
25, In which of pathologic processes does ventilative-restrictive form of external respiratory
disturbance develop?
A. Pulmonary edema
B. Bronchial asthma
C. Poliomyelitis
D. Syringomyelitis
E. Bronchitis

26. In what form of dyspnea develops in insufficiency of surfactant system of the lungs?

A. Rare shallow respiration


B. Fast deep respiration
C. Fast shallow respiration
D. Expiratory dyspnea
E. Rare deep respiration

27. Which of pathogenetic forms of external respiratory disturbance may develop in


craniocerebral trauma?
A. Diffuse-pneumonic
B. Diffuse-restrictive
C. Ventilative-restrictive
D. Primary dyskinetic
E. Obstructive

28. A patient aged 23 was admitted to the hospital with a craniocerebral injury in a poor
condition. Respiration is characterised by spasmatic effort of inspiration which does not stop
and sometimes is broken by expiration. What type of respiration is this typical for?
A. Apneustic respiration
B. Gasping respiration
C. Kussmal’s respiration
D. Cheyne-Stokes’ respiration
E. Biot’s respiration

29. A patient aged 64 was admitted to the hospital with complaints of cough with sputum,
marred dyspnea. Objectively forced position, respiratory rate - 32 per min: additional muscles
take part in the act of respiration. X-ray examination: the lungs have increased transparency.
What is the main link in the pathogenesis of respiratory insufficiency of this patient?
A. Decrease of elastic abilities of the lungs
B. Accumulation of sputum in the lungs
C. Thin mucus membranes of bronchi
D. Impairment of surfactant system of the lung
E. Pneumosclerosis

30. Both vagus nerves were cut during experiment on animal. How may vagus respiration be
characterized?
A. Frequent and deep
B. Very rare and deep
C. Rare and shallow
D. Frequent and shallow
E. Periodic

31. A child with diphtheria developed larynx edema. What type of respiration is observed?

A. Gasping respiration
B. Apneustic respiration
C. Kussmal’s respiration
D. Dyspnea
E. Biot’s respiration
32. A female patient was admitted to the hospital with exertion of chronic bronchitis.
Antibiotic treatment was administered. On the fourth day general condition became worse: the
temperature persisted, dyspnea and cough increased, itching eruptions appeared on the skin,
diffused dry whistles were heard on auscultation. There is eosinophilia (10%) in the blood.
Aggregation of symptoms of the patient is caused by the development of:
A. Asthmatic bronchitis
B. Bronchial asthma
C. Bronchopneumonia
D. Allergic reaction
E. Drug allergic reaction

33, The deficiency of alpha-l-antitrypsine was determined in a patient with pulmonary


emphysema. What is the leading mechanism of the development of emphysema in this patient?
A. Excessive discharge of elastase by lymphocytes
B. Increase of histamine discharge
C. Dilation of bronchioles
D. Activation of elastolysis
E. Decrease of elastase discharge by neutrophilic granulocytes 34. A patient with bronchial
asthma has developed acute respiratory insufficiency. What type of respiratory
insufficiency appears in this case?
. Disregulative disorder of alveolar ventilation
MoUoawP

. Diffuse
. Perfuse
. Restrictive disturbance of alveolar ventilation
Obstructive disturbance of alveolar ventilation
35. 0.5 ml of air was introduced into a rat’s pleural cavity. What type of respiratory insufficiency
appears in this case?
A. Restrictive disturbance of alveolar respiration
B. Obstructive disturbance of alveolar ventilation
C. Perfuse
D. Diffuse
E. Disregulative disturbance of alveolar ventilation
36. Acute respiratory insufficiency appeared in a patient with tetanus.
What type of respiratory insufficiency develops in this case?
A. Restrictive disturbance of alveolar respiration
B. Disregulative disturbance of alveolar ventilation
C, Obstructive disturbance of alveolar ventilation
D. Perfuse
E. Diffuse

37. Increased content of carbon dioxide is in a room. How does respiration change (its depth and
rate) in a man entered this room?
A. Depth of respiration increases
B. Rate of respiration increases
C. Depth and rate of respiration increase
D. Depth of respiration decreases
E. Rate of respiration decreases

38. When eating a child aspired some food. Strong cough, cyanosis of skin and mucous
membranes, increase in pulse rate, rare respiration, and lengthening of inspiration developed
in him. What kind of disorder of external respiration developed in this child?
A. Stenotic respiration
B. Expiratory dyspnea stage in asphyxia
C, Inpiratory dyspnea stage in asphyxia
D. Biot’s respiration
E. Alternative respiration

39, A 56-year-old patient, who is fitter by occupation, has been suffering form fibrous-cavernous
tuberculosis of lungs for 10 years. During later 3 weeks cough and weakness intensify, amount
of purulent-mucous phlegm with blood increases in him. What is the reason for developed
ventilation insufficiency in this case?
A. Decrease in number of functioning alveoli
B. Disturbances of functions of respiratory center
C, Disturbances of functions of neuro-muscular apparatus
D. Disturbances of agility of the thorax
E. Disturbances of airways passage

40. A 50-year-old unconscious patient was admitted to the endocrinology department. It is known
that patient has been suffering from diabetes mellitus for many years. What kind of disorders
of external respiration will be present in this patient?
A. Kussmaul’s respiration
Asphyxia
BIS [Ol

. Biot’s respiration
Stenotic respiration
. Chain-Stocks respiration

41. Attack of suffocation developed in a 45-year-old woman suffered from bronchial asthma for
a long time. What pathogenetic mechanism does this phenomenon have?
A. Expiratory narrowing of small bronchi
B. Loose of elasticity of lung tissue
C. Decrease in sensitivity of respiratory center
D. Disorders of agility of the thorax
E. Impairment of perfusion of lung tissue

42. Edema of larynx developed in a child on the background of allergic reaction. What kind of
respiratory failure has developed in this case?
. Obstructive

>
. Restrictive
I=| FIO|
. Disregulative
. Parenchimatous

43. A patient with both-sided lung edema was in the emergency department. What kind of dyspnea
is observed in this case?
A. Shallow frequent respiration
B. Deep frequent respiration
C, Shallow rare respiration
D. Shallow respiration
E. Asphyxic respiration

44, Lung fibrosis with disorders of alveolar ventilation developed in a miner. What is the leading
mechanism in this disorder development?
A. Restriction of respiratory surface of lungs
B. Narrowing of upper airways
C, Disorders of neural regulation of respiration
D. Restriction of agility of the thorax
E. Spasm of bronchi

45. A 60-year-old male patient in severe condition was admitted to the hospital. The patient has
been suffering from diabetes mellitus for 10 years. At second day of his stay at the in-patient
department his condition sharply worsened: coma developed, noisy respiration appeared,
which was accompanied by deep inspirations with forced expirations and participation of
expiratory muscles. What kind of disorders of respiration is observed in the patient?
A. Kussmaul’s respiration
B. Stenotic respiration
C, Tachipnea
D. Chain-Stocks’ respiration
E. Biot’s respiration

46. A patient with craniocerebral trauma has respiration, which is characterized by respiratory
movements becoming deeper and then becoming shallower and after that short pause in
respiration occurs.
What periodical respiration is this type characteristic for?
A. Chain-Stocks’ respiration
B. Biot’s respiration
C, Kussmaul’s respiration
D. Gasping respiration
E. Apneastic respiration
47. A female patient N., aged 16, with asphyxia was admitted to the emergency department. She
has expiratory dyspnea with respiration rate of 8 per minute, BP of 80/50 mmHg, heartbeat
rate of 40 bpm, and narrowed pupils. What phase of asphyxia does this patient have?
A. IL phase
I phase
IS jo ls

II phase
IV phase E. -

48. A boy was admitted to resuscitation department. He has inspiratory dyspnea, general
excitation, widened pupils, tachycardia, and BP of 160/100 mmHg. What phase of asphyxia
does this patient have?
A. [phase
B. II phase
C, TI phase
D. IV phase E. -

49, A 15-year-old girl has been suffering from bronchial asthma for 5 years. What kind of hypoxia
does this patient have?
A. Respiratory
B. Circulatory
C, Hemic
D. Tissue
E, Mixed
ANSWERS. 1.-E.2.-A.3.-B.4.-A.5.-B.6.-E.7.-A.8.-B.9.-E.10.-A.11.-C. 12.-E.13.-B.14.-E. 15.-
B.16.-D.17.-D.18.-C.19.-D.20.-E.21.-E.22.-B.23.B.24.-D.25.-A.26.-C. 27.-D.28.-A.29.-A.30.-
B.31.-D.32.-E.33.-D.34.-E. 35.-A.36.-B.37.-C.38.-B.39.-A. 40.-A. 41.-A. 42.-A. 43.-A. 44.-A.
45.A. 46.-A. 47.-A. 48.-A. 49.-A.
Topic 21. Pathophysiology of digestive system.

1. After taking a fatty food a patient feels nausea, flaccidity, later the sign of steathorrhea has
appeared, cholesterol in the blood 9,2ml/c. The cause of this state is the deficiency of: A.
Chylomicrones.
B. Triglyceroides.
C. Bile acids.
D. Fatty acids.
E. Phopholipids.

2. A patient aged 45 had the diagnosis ulcer of the stomach. On examination of secretory
function of the stomach it was determined that the amount of basal secretion was 100mole/hr,
acidity of basal secretion -60mml/hr. What factors action contributes to the hypersecretion in the
stomach?
A. Gastrine.
B. Pancreatic polypeptide.
C, Somatostatin.
D. Glucagon.
E. Betaendorphin.

3. On laboratory examination increased amount of diastase in the urine and also a large
amount of undigested fat in stool were revealed in a patient female with complain of circular
character pain in epigastric area. What form of gastrointestinal tract pathology are described signs
typical for?
A, Inflammation of large intestine.
B. Acute appendicitis.
C, Infectious gastritis.
D. Ulcerous disease of the stomach.
E. Acute pancreatitis.

A part of patient pancreas was resected. What kinds of product must be limited in his diet?
A, Sour milk product
B, Fruits.
C, Not fatty boiled meat.
D. Fatty and fried meal.
E. Vegetables.
5. What enzyme deficiency is the cause of maldigestion of fats in the gastrointestinal tract
and increase of neutral fats in the stool?
A, Gastric lipase B. Intestinal lipase. C. Hepatic lipase,
D. Enterokinase.
E. Pancreatic juice.
6. Decrease of synthesis and secretion of trypsin is observed in chronic pancreatitis. The
splitting of what substances is broken? A. Proteins.
B. Nucleic acids C. Polysaccharides.
D. Lipids.
E. Liposoluble vitamins.

7. In coprologic investigation it is determined that stool is discoloured, there are drops of


neutral fat in it. The most possible cause of this is the impairment of?
A, Secretion of intestinal juice.
B. Process of absorption in the intestine.
C. Entering of bile into the intestine.
D. Acidity of gastric juice.
E. Secretion of pancreatic juice.

8. A patient had been taking antibiotics of a wide spectrum of action for a long period of time
that caused decrease of appetite, nausea, and diarrhea with saprogenic smell. What is the side
effect of treatment?
A. Hepatotoxic action.
B,. Allergic reaction.
C, Dysbacteriosis.
D. Nephrotoxic action.
E. Direct irritative action

9. On examination of a patient suffering from acute pancreatitis increased amount of


chylomicrons was determined in the blood. What enzyme activity is sharply decreased in this
pathology? A. Lipoprotienelipase.
B. Pancreatic lipase.
C. Pancreatic phospholipase.
D. Tissue triglyceride lipase.
E. Tissue diglyceride lipase.

10. — The analysis of gastric juice of an elderly man who complained of unmotivated weakness,
sickness, absence of appetite showed achylia, achlorhydrea, and presence of lactic acids and
coagulated blood, decreased of pepsin secretion. What disease causes such clinicallaboratory
symptoms?
A. Cancer of the stomach.
B. Chronic gastritis.
C, Chronic pancreatitis.
D. Cavitary Maldigestion.
E. Acute gastritias.
11. A man with chronic hepatitis has dyspeptic disorders: decrease of appetite, nausea,
unstable stool, and steatorrhea. What is the mechanism of dyspeptic disorders in hepatic
pathology?
A, Intoxication
B. Hypoglycemia
C, Cholalemia
D. Hypocholea
E. Hyperbilirubemia
12. Spasmodic pains in the abdomen and repeated diarrhoea with mucus appeared in a healthy
person 3-5 hours later after taking meals. This was preceded by nausea and momentanecous
vomiting, general weakness, loss of appetite. What is the most possible cause of the desired
symptoms?
A, Chronic pancreatitis
B. Food intoxication
C. Enterocolitis
D. Hyperacid state of the stomach
E. Chronic gastritis

13. A man who works at a storage battery plants complains of constant feeling of weight and
periodical spasmodic pains in the abdomen, constant retention of stool (not more often than one
time per three day). This is accompanied by frequent headaches, flaccidity, absence of appetite,
and bad taste in the mouth. What are the causes of these disorders?
A. Hyper acid state of the stomach
B. Hypoacid state of the stomach
C, Spastic lead colie with constipation
D. Chronic pancreatitis
E. Parietal maldigestion

14. Due to chronic gastritis a man has the impaired structure of the mucous membrane,
decreased indices of acid formation function of the stomach. The most essential negative result of
this will be the impairment of:
A. Excretion of secretum
B. Pancreatic juice secretion
C, Secretory function of small intestine
D. Evacuation of chyme into duodenum
E. Protien digestion
15. | On laboratory examination increased amount of diastase in the urine and also a large
amount of undigested fat in stool were revealed in a patient with complains of circular character
pains in epigastric area. What form of gastrointestinal tract pathology are the described signs
typical for?
A, Inflammation of the large intestine
B. Acute appendicitis
C, Infectious hepatitis
D. Ulcerous disease of the stomch
E. Acute pancreatitis

16. A patient aged 35 with ulcerous disease had a rejection of antral portion of the stomch.
What gastrointestinal hormone secretion will be impaired due to operation?
A. Gastrin
B. Secritin
C, Neurotensin
D. Histamine
E. Cholecystokinin

17. A patient aged 57 was admitted to a gastrointestinal department with suspicious on


Zolliger-Ellison syndrome, which was proved by sharp increase of gastrin level in the blood
serum. What impairment of secretory gastric function is the most possible?
A. Hyperacid hyposecretion
B. Hyperacid hypersecretion
C, Hypoacid hypersecretion
D. Achylia
E. Hypoacid hyposecretion

18. A patient with signs of encephalopathy was hospitalised in neurological in- patients
department and co relation between increase of encephalopathy and substances passing from
intestine into systemic blood circulation was determined. What compounds formed in the intestine
may cause endotoximia?
A, Acetoacetat
B. Biotin
C, Indole
D. Omitin
E. Buthirat

19. 150 ml of meat broth were introduced into the stomach cavity of experimental dog to a
probe the content of what substance will increase quickly in the animal’s blood?
A, Insulin
B. Vasointestinal peptide
C, Neurotensin
D. Somatostatin
E. Gastrin

20. A patient age 37 was admitted into a surgical department with the signs of acute
pancreatitis; vomiting, diarrhea, bradycardia, hypotension wearness, dehydration of the organism.
What preparation must be used first of all?
A. Ephedrine
B. No-spa
C, Pephenazine
D. Contrical
E. Platyphyllin
21. A patient has a stone in the common bile duct, which has stopped passing bile into the
intestines. The impairment of what digestive process is observed in this case?
A, Digestion of carbohydrates
B. Absorption of carbohydrates
C, Digestion of fats
D. Absorption of proteins
E. Digestion of proteins
22. On fibroscopy of the stomach ulcer was revealed in antrum portion, which was associated
with dissemination of mucosa by Helicobacter pylori. The role of this agent in the formation of
ulcer results in:
A. Damage of mucous barrier
B. Inhibition of mucosa regeneration
C, Impairment of microcirculation in mucous
D. Stimulation of HCL secretion
E. Stimulation of pepsin secretion

23. A patient with ulcer of stomach has impairment of equilibrium between the factors of
aggression and defense. What factor contributes to the development of gastric ulcer?
A. Mucin
B. Helicobacter pylori
C, Bicarbonate
D. Prostaglandin E2
E. Prostacyclin

24. After resection of duodenum a patient has developed a syndrome of duodenal in sufficiency
due to the impairment of its endocrine function with signs of cell insufficiency of APUD system.
What hormone is produced by A-cells of this portion of intestine?
A. Histamine
B. Insulin
C, Seratonin
D. Secrtin
E. Glucagon

25. A patient complains of nausea, which often ends with vomiting. These dyspeptic
phenomena become more laborious with time. What stomach function is infringed in this patient?
A. Evacuation
B. Motility
C, Reservoir
D. Incretory
E. Secretory

26 A patient aged 42 was admitted to the hospital with gastric hemorrhage (ulcerous disease was
excluded by a patient). Ulcer and hyperplasia of gastric mucose were revealed. What
investigation is it necessary to carry out to confirm the diagnosis of Zollinger-Ellison syndrome
in this patient?
A. Definition of gastrin level in the blood.
B. Tomography of pancreas
C, Definition of secretin level in the blood
D. Definition of activity level of pancreatic enzymes
E. Investigation of activity level of pancreatic secretion

27 The amount of protein in the blood was examined in a patient after resection of sizeable part of
small intestine. What change of protein amount may be expected in this patient?
A. Hyperproteinemia
B. Hypoproteinemia
C, Paraproteinemia
D. Hypergammaglobulinemia
E. Dysproteinemia

28 Ulcerous disease of the stomach is revealed in a woman aged 52, who has been ill with contact
dermatitis. On examination it is determined that the patient had been taking corticosteroid
preparations for a long period of time, but acidity of gastric juice was increased. What
mechanism caused the increase of gastric secretion?
A. Decrease of prostaglandin content
B. Increase of secretin secretion
C, Gastrin hyposecretion
D. Increase of histamine content
E. Increase of gastrin excretion
29 A patient complains of vomiting, eructation, pain in epigastric area, constipation, and
emaciation. On examination a basal secretion of HCL is 6mml/gr; maximal one is 60mml/gr.
What state has acid-forming function of this patient?
A. Hypoacid
B. Anacid
C,Hyperacid
_
D. Hypersecretory
E. Normacid

30 A patient aged 42 complains of pains in epigastric area, vomiting; vomitory masses have color
of “coffee grounds”, melena. In anamnesis there is ulcerous disease of the stomach. On
examination: skin is pale, heart rate-110 beats per minute; AP-90/50mm.hg. Blood analyses:
erythrocytes-2.8 x 1012/1; leucocytes-8x109/l; hemoglobin-90gr/l. What is the possible
complication that developed in the patient?
A, Penetration
B. Perforation
C, Hemorrhage
D. Cancerous degeneration
E. Pylorostenosis
31 The investigation of pancreatic function was carried out in experimental animal by
radioisotopic method. The percentage of excreted with stool 1131- albumin is 53%. This is the
evidence of:
A, Increase of exocrine function
B. Increase of incretory function
C, Inhibition of incretory function
D. Inhibition of exocrin function
E. Normal function
32 Malabsorption syndrome is:
A. Syndrome of membraous maldigestion
B. Syndrome of gastric malabsorption
C, Syndrome of malabsorption in small intestine
D. Syndrome of intestinal malabsorption
E. Malabsorption of proteins

33. A patient complains of dyspeptic disorders, melaena, hemorrhoidal bleedings. While


examination of the patient enlargement of veins at the anterior surface of abdomen and increased
abdomen dimensions were revealed. What pathology of GIT may manifest by such symptoms?
A. Colitis
B. Portal hypertension
C. Intestinal autointoxication
D. Peptic ulcer
E. Enteritis

34. A 67-year-old woman, who has cholecystitis for a long time, suddenly developed sharp
pain in the upper part of abdomen, nausea, and vomiting after food intake. Acute pancreatitis was
diagnosed in this patient. What is the main link in pathogenesis of this disease?
A. Preliminary activation of pancreatic enzymes
B. Decrease in enzyme levels in pancreatic juice
C, Intensification of enzyme activation in duodenum
D. Reduction of pancreatic polypeptide secretion
E. Increased level of cholecystokinin
35. A patient has increased gastric secretion in response to both mechanical and chemical
stimuli. He has high acidity of gastric juice with pH equals 2.0 on an empty stomach. After the
breakfast pH restores to normal in 12 minutes and is continuously dropping during following 2
hours. What type of stomach secretion is being observed in this patient?
A. Excitable
B,. Brakable
C, Inert
D. Asthenic
E. Normal
36. Frequent liquid stool, signs of intoxication, and dehydration developed in a newborn at 5-
6 day. After this newborn was fed by acidmilk mixtures indicated symptoms disappeared. The
conclusion of innate lactose deficiency was made. What process was broken in this newborn?
A. Membrane digestion
B. Cavital digestion
C. Excretory function of intestine
D. Secretory function of pancreas
E. Secretion of gastric juice
37. Ulcers of stomach were found out in a patient, who was treated with glucocorticoids for a
long time. What is the main mechanism of ulcer development in this case?
A, Increase in secretion and acidity of gastric juice
B. Decrease in histamine concentration in stomach wall
C, Increased tonus of sympathetic nervous system
D. Increased production of prostaglandins E
E. Decreased tonus of parasympathetic nervous system

ANSWERS: 1.-C.2.-A.3.-E.4.-D.5.-E.6.-A.7.-C.8.-C.9.-A.10.-A.11.-D. 12.-B.13.-C.14.-E.15.-


E.16.-A.17.-B.18.-C.19.-E.20.-D.21.-C.22.-A.23_B.24.-E.25.-A.26,-A.27.-B.28.-A.29.-C.30.-
C.31.-D.32.-D, 33.-B. 34.-A.
35.-A. 36.-A. 37.-A.
Topic 22. Pathophysiology of the liver.

1.Flabby contraction of gall bladder was revealed in a woman aged 55 after introducing some of
vegetable oil into duodenum. What hormone insufficiency with such state?
A. Gastrin
B. Enterogastrin
C. VIP
D. Pancreozymin
E. Cholecystokinin
2A patient aged 25 has a diagnosis of chronic hepatitis. A patient has lost 10 kg of his body weight
for 2 months. Objectively: the skin is dry, desquamative, and pale with yellowish color, small
punctate hemorrhages on the skin, stomatorrhagia. The impairment of what hepatic function do
petechial hemorrhage and stomatorrhagia prove?
A. Glycogen synthetic
B. ekromogenic
C. Detoxicative
D. Depositing
E. Albumin synthetic
3.0On examination bile congestion in the liver and cholelitiasis were revealed in a patient. Point out
the main component of cholelitiasis in this state:

A. Calcium bilirubinate
B. Triglycerides
C. Protein
D. Cholesterol
E. Mineral salts

4.What kind of jaundice is characterized by increased amount of direct bilirubin in the blood,
appearance of bilirubin in urine, acholic stool?
A. Hemolytic
B. Obstructive
C. Parenchymatous
D. -E. -

5.Residual nitrogen and urea were determined in the patient’s blood analysis. The amount of urea
in the residual nitrogen is considerably reduced. The disease of what organ is characterized by this
analysis?
A. Intestine
B. Kidneys
C. Stomach
D. Heart
E. Liver
6.A patient complains of general weakness, boring pain in the abdomen, bad appetite, suspicion
onjaundice. Blood serum contains 77.3 mcml/L of total bilirubin and 70.76 mcml/L of conjugative
bilirubin. What is the most possible type of jaundice?
A. Mechanical jaundice
B. Acute hepatitis
C, Hepatic cirrhosis
D. Parenchymatous jaundice
E. Hemolytic jaundice
7.1n 70’s the scientists determined that the cause of severe jaundice in newborns was the
impairment of connection of bilirubin in hepatocytes.
What substance is used for the formation of conjugate?
A. Glucoronic acid
B, Pyruvic acid
C, Uric acid
D. Sulphuric acid
E. Lactic acid
8.A man aged 38 with ecteric skin has anemia,enlarged spleen, hyperbilirubinemia, urobilinuria,
hypercholic stool. What condition are these changes typical for?
A. Suprahepatogenous jaundice
B. Subhepatogenous jaundice
C, Cellular-hepatogenous jaundice
D. Gilbert’s syndrome
E. Syndrome of hepatic insufficiency

9 It’s determined that a patient with jaundice has increased amount of total bilirubin instead of
indirect one (free) in blood plasma,high content of stercolbilin in stool and urine,the level of direct
(connected) bilirubin in blood plasma is normal.what kind of jaundice is it?
A. Mechanical
B. Icterus of new borns
C. Parenchymatous (hepatic)
D. Hemolytic
E. Gilbert’s disease

10.In inflammatory process colloidal properties of bile are impaired in gall bladder and this results
in the formation of gall stones. What substance crystallization is the main cause of the formation?
A. Cholesterol
B. Urate
C, Chloride
D. Oxalate
E. Phosphate

11.After an accident a completely crashed man’s liver was removed.what disorders do of hepatic
absence may cause death during the first hours after operation?
A. Hypoglycemia
B. Intoxication
C, Fall of AP
D. Sharp ascites
E. Hemophilia and hemorrhage
12.rked isoosmotic hyperhydration has developed in a patient with hepatocirrhosis. What is the
leading mechanism of dyshydria development?
A. Cardio-vascular insufficiency
B. Growth of wall capillary permeability
C. Rushyer-petrovsky reflex
D. Secondary aldosteronism
E. Hypoproteinemia
13.On the background of pain in the right hypochondrium and yellowness ‘a patient with hepatic
cirrhosis has constant dyspeptic disorders in a kind of bitter taste in the mouth, feeling of heaviness
in epigatric area, nausea, unstable stool, steatorrhea. What is the main cause of the described
disorders?
A. Hypoglycemia
B. Hypocholia and intoxication
C. Increase of stercobilin
D. Hyperbilirubinemia
E. Hypoproteinemia
14.In a severe course of viral hepatitis, a patient has developed adynamia, sleepiness at day time
and insomnia at night, inadequate behavior (delirated ideas). There is fetor hepaticus, Kussmaul’s
respiration. What kind of metabolism impairment causes these symptoms?
. Carbohydrate metabolism and hypoglycemia
mMmOOWp

Water and salt metabolism and hyperhydration


Fat metabolism
. Pigmental metabolism and hyperbilirubinemia
Nitrogenous metabolism

15.There is increase of indole amount and decrease of indican | in the patient’s urine. This indicate
the impairment of
A. Filtering function of kidneys
B. Detoxication function of liver
C. Reabsorptive funtion of kidney
D. Albumin synthetic function of liver
E. Secretory function of pancreas

16.Increase of direct and indirect bilirubin is determined in blood of a patient with marked
yellowness of sclerae and skin. There is great amount of bilirubin and urobilin in the urine, traces
of stercobilin, decrease of stercobilin in stool. Define pathogenic type of jaundice in a patient.
A. Parenchymatous
B. Hemolytic
C, Mechanical
D. By-pass
E. Transmissible
17.A patient admitted to the hospital has clearly marked widened subcutaneous veins in the area
of umbilicus (“the head of medusa”).
Which of the large venous vessels has the impaired passage?
A. V. renalis
B. V. porta
C. V. iliaca inferior
D. V. mesenterica superior
E. V. mesenterica inferion

18.In coprologic investigation it is determined that stool is colorless; there are drops of neutral fat
in it. The most possible case of this is impairment of:
A, Entering the bile into intestine
B. Secretion of intestine juice
C, Acidity of gastric juice
D. Processes of absorption in the intestine
E. Secretion of pancreatic juice
19.A patient had nausea and malaise after taking fatty foodstuffs. Sings of steatorrhea developed
in this patient some time later. Content of cholesterol in patient’s blood is 9.2 mmol/L. This
condition results from deficiency of:
A. Chylomicrones
B. Triglycerids
C. Bile acids
D. Phospholipids
E. Lipase

20.Low level of albumins and fibrinogen were revealed in the patient’s blood. The decreased
activity of what hepatocyte organells cause this phenomenon?
A. Agranular endoplasmic network
B. Mitochondria
C. Granular endoplasmic network D. Goldgi’s complex
E. Lysosoms.

21.A patient was admitted into a clinic with signs of acute alcohol poisoning. What changes of
carbohydrate metabolism are typical for this condition?
A. Gluconeogenesis increases in the liver
B. Glycogen decomposition increases in the liver
C. Aerobic decomposition of glucose increases in the muscles D. Anaerobic
decomposition of glucose predominates in the muscles.
E. The rate of gluconeogenesis decreases in the liver.
22A man aged 54 was admitted into the clinic with complaints of pains in the right hypochondrium
vomiting with blood. Objectively: enlargement of hepatic size, varicose of esophagus and
stomach, bleeding from them. The functional disorder of what vessel took place?
A. Vena hepatica
B. Vena cave superior
C. Vena porta
D. Aorta abdominalis
E. Vena cava inferior

23.The development of acute pancreatitis in a patient is accompanied by the impairment of


permeability of common bile duct. What pathologic process may this result in?
A. Hemolytic jaundice
B. Parenchymatous jaundice
C. Hepatic coma D. Mechanical jaundice
E. Portal hypertension.

24.Which of the factors plays the leading role in the development of encephalopathy in hepatic
insufficiency?
A. Increase of concentration of toxic substances in the blood
B. Hyperaldosteronism
C, Hyperbilirubinemia
D. Hypofibrinogenia
E. Hypoproteinemia

25.A patient has a diagnosis of ascites. There are no edemas in the other parts of the body. There
are large vessels of cyanotic color on the abdomen. What pathology has this patient?
A. Essential hypertension
B. Hypertension of pulmonary circulation
C. Portal hypertension
D. Chronic circulatory insufficiency
E. Hepatic hypertension
26.Arterial hypertension may develop in a case of hepatic insufficiency. Which of these factors
causes the increase of arterial pressure in such cases?
A. Aldosteron
B. Adrenaline
C, Renin
D. Noradrenalin
E. Angiotensin I

27.A patient aged 25 is ill with jaundice, his skin has got yellow and green color, there’s skin itch,
hypocholic stool throbbing gall bladder.
What is the origin of jaundice?
A. Serum hepatitis
B. Obturative genesis
C. Hepatic cirrhosis
D. Leptospirosis
E. Post-transfusion

28.A patient complains of general weakness, breathlessness. Decrease of AP, ascites, widening of
superficial veins of anterior abdominal wall, splenomegaly were established in this patient. What
impairment of hemodynamic has this patient?
A, Portal hypertension syndrome
B,. Left ventricular failure
C, Right ventricular failure
D. Collapse
E. Arterial hypotension.

29.A patient with jaundice complains of erythrism, headache, and insomnia. Objectively: pulse-
54 beats/min. AP- 90/60 mmHg. Coagulation of blood decreases. There are traces scratching on
skin. The action of what component causes these symptoms?

A. Bilirubin
B. Cholesterol
C. Bile pigment
D. Bile acid
E. Fatty acid

30.A patient was admitted to the hospital with complaints of dyspeptic disorders, melena,
hemorrhoidal bleeding. Extensions of the vessels on the anterior abdominal wall in combination
with the enlargement of size of the abdomen were revealed on examination of this patient. What
pathology of gastrointestinal tract shows these symptoms?
A. Ulcerous disease
B. Intestinal autointoxication
C. Portal hypertension
D. Colitis
E. Enteritis.

31.Yellow color of the skin and the sclera, dark urine appeared in a patient after fungus poisoning.
What pigment causes the color of urine in the patient with hemolytic jaundice?
A. Biliverdin
B. Verdoglobin
C. Unconjugated bilirubin
D. Monoglucoronide bilirubin
E. Stercobilin
32A female patient with chronic hepatitis complains of increase sensitivity to barbiturates which
she used previously without any symptoms of intoxication. The disorder of what hepatic jaundice
is responsible for this state?
A. Bile formation
B. Hemodynamic
C. Metabolic
D. Hemopoetic
E. Phagocytotic
33.Marked jaundice appeared in a patient 3 months later after the operation on his upper jaw.
What pathological process may this patient have?
A. Suprahepatic jaundice
B. Hereditary hemolytic jaundice
C. Hepatic jaundice
D. Subhapetic jaundice
E. Cholecystitis

34.A female patient aged 45 was admitted to a hospital with complains of sudden pain in the
abdominal cavity, increase of temperature, and leukocytosis. Which of factors caused these
changes in the woman’s condition?
A, Gall stones
B. Bacteria
C. Mechanical energy
D. Acids
E. Viruses

35. In a 38-year-old patient, who endued viral hepatitis C and is abusing alcohol, symptoms of
hepatic cirrhosis with ascites and edemas on lower extremities developed. What changes in blood
composition underlies edema development?
A. Hypoglycemia
B. Hypoalbuminemia
C. Hypoglobulinemia
D. Hypokalemia
E. Hypocholesterolemia
ANSWERS: 1.-E.2.-E.3.-D.4.-B.5.-E.6.-A.7.-A.8.-A.9.-D.10.-A.11.-A. 12.-D.13.-B.14.-E.15.-
B.16.-A.17.-B.18.-A.19.-C.20.-C.21.-E.22.-C.23.D.24.-A.25.-C.26.-A.27.-B.28.-A.29.-D.30.-
C.31.-E.32.-C.33.-C.34.-A.
35.-B.
Topic 23. Renal pathophysiology

1.A patient has severe nephropathy with a massive edematous syndrome, which complicated
bronchiectatic diseases. Laboratory investigations show massive proteinuria, cylindruria,
considerable decrease of protein content in the blood serum, hyperlipidemia, hypokalemia and
other deviations. What is the main link in the pathogenesis of the patient’s edemas?
A. Decrease of oncotic blood pressure.
B. Increase of extracellular fluid pressure.
C, Increase of hydrostatic blood pressure.
D. Blockade of lymph drainage.
E. Hyperpermeability of microvessels

2.A patient with renal insufficiency has a disorder of hemopoietin synthesis produced in the
kidneys. What formed blood elements development is impaired?
A, Erythrocytes
B. Granulocytes
C, Thrombocytes
D. B-lymphocytes
E. T-lymphocytes
3.The narrowing of afferent glomerular renal arteriole caused the decrease of urination. The cause
of this is a decrease of:
A. Reabsorption of glucose
B. Reabsorption of ions
C. Reabsorption of urea
D. Reabsorption of water
E. Effective filtration pressure
4.A man aged 32 has been ill with chronic glomerulonephritis for 4 years. He was hospitalized
with the signs of anasarca. AP-185/105 mm. Hg. Blood analysis shows: Hb-110gm/1; erythrocytes-
2.6x1012/1; leucocytes-9.5x109/1; residual nitrogen-32 mmol/l; total protein-50 gm/1.
What change points to glomerulonephritis with nephrotic syndrome?
A. Anemia
B. Arterial hupertension
C. Hyperzotemia
D. Hypoproteinemia
E. Leukocytosis

5.Sharp decrease in sodium content in the blood serum was revealed in a patient with renal
insufficiency. There are pale crumby edemas of the face manifesting in the morning. What
substance composing intercellular matrix comes from blood bed?
A. Collagen
B. Elastin
C. Procollegen
D. Hyaluronic acid
E. Fibronectin
6.Edemas appeared in a patient after Streptococcus infection. The urinalysis was made and allowed
to make a diagnosis of acute glomerulonephritis. What pathologic changes in urine confirm the
diagnosis?
A. Fresh erythrocytes
B. Protein with high molecular mass and lixiviated erythrocytes
C, Protein only with low molecular weight (up to 40000.
D. Increased excretion of sodium
E. Leukocyturia

7.After poisoning by salts of heavy metals a patient has developed nongascous acidosis, urine
pH=6.0, glucosuria without hyperglycemia, polyuria, natriuresis. Which of the structures is
damaged?
A. B-cells of Lanhergans islets
B. Nephron glomeruli
C. Nephron tubules
D. Nuclei of hypothalamus
E. Adrenal cortex
8.A patient age 48 with fibro-cavernous tuberculosis complains of general weakness decrease of
diurnal urination, edemas of the trauma and extremities, increase of AP up to 180/90 mm. Hg
Urinalysis shows increase of protein, hyaline and granular cylinder, erythrocytes. A month later
he died from renal insufficiency. Enlargement of the heart and large “fatty kidneys” were revealed
at autopsy (weight of kidneys is 240.0 gm -260.0 gm). What disease was fibro-cavernous
tuberculosis of the lungs complicated by?

A. Glomerulonephritis
B. Nephrotic syndrome
C. Amyloidosis
D. Pyelonephritis
E. Nephrosclerosis

9.A patient with chronic glomerulonephritis has edemas, AP-210/100 mmHg, and the rate of heart
beats-85 per minute. The borders of the heart are dilated. What is the leading mechanism in the
development of arterial hypertension?
A. Increase of sympathetic-adrenal system activity
B. Hyperfunction of the heart
C. Activation of renin- angiotensin -aldosteron system
D. Increase of circulatory blood volume
E. Increase of vasopresin discharge
10.The presence of glucose in the urine in its normal concentration in the blood serum was
determined in a patient aged 18 on laboratory examination. The most possible cause of this is the
impairment of
A. Tubular reabsorption
B. Glomerular filtration
C, Tubular secretion
D. Secretion of the glucocorticoids
E. Insulin secretion

11.In a week and a half after a severe streptococcal tonsillitis a patient aged 24 developed
edematous face, increased arterial pressure, hematuria, and proteinuria of 1.2 g/L .His blood
analysis shows antistreptococcal antibodies and decrease of complement components. In the
microvessels of what structures is the most possible localization of immune complex deposits
which caused the development of nephropathy.
A. Proximal tubules
B. Gomeruli
C. Descendent tubules
D. Henle’s loop
E. Pyramids

12.Increased amount of proteins in the urine was revealed in a patient with acute
glomerulonephritis. The impaired function of what nephron structures is the presence of protein
in the urine connected to?
A. Basal membrane of glomerulus capillaries
B. Epithelium of parietal layer of glomerulus capsule
C, Epithelium of thin tubules
D. Epithelium of distal tubules
E. Epithelium of Henle”s loop

13.Proteinuria (5 g/l) due to low molecular weight proteins and hematuria with lixiviated
erythrocytes were revealed in patient’s urinalysis. What renal function disorder do these findings
point out?
A, Increase in glomerular permeability
B. Increase in tubular secretion
C, Increase in tubular excretion
D. Decrease in tubular reabsorption
E. Extrarenal disorders

14.A patient with chronic glomerulonephritis has symptoms of anemia.


What causes these symptoms?
A. Decrease in erythropoietin synthesis
B. Loss of erythrocytes in urine
C, Increased destruction of normal erythrocytes
D. Erythrocytes hemolysis
E. Iron deficiency for hemoglobin synthesis

15.A patient with chronic renal disease is edematous, pale, and his BP is increased. Laboratory
examination shows protein and erythrocytes in patient’s urine. Protein content in the blood is the
normal. What is the main link in the pathogenesis of edematous syndrome?
A. Arterial hypertension
B. Hyperazotomia
C. Secondary aldosteronism
D. Hematuria
E. Poteinemia

16.A patient with chronic renal disease is edematous, pale, and his BP is increased. Laboratory
examination revealed protein and erythrocytes in his urine, hyperazotemia, and decrease in
erythrocytes and hemoglobin in his blood. What is the main link in pathogenesis of arterial
hypertension of this patient?
A. Activation of renin-angiotensin system
B. Anemia
C, Hyperazotemia
D. Increase of circulatory blood volume
E. Proteinuria

17.A patient with diabetes mellitus has developed chronic renal failure with the development of
uremia and the rate of glomerular filtration of 8 ml/min. What is the mainly possible mechanism
of decrease in glomerular filtration rate and the development of chronic renal failure in this
patient?
A. Spasm of afferent glomerular arteriole
B. Occlusion of tubular lumen of nephron by hyaline cylinders
C. Increase of osmotic blood pressure
D. Decrease of systemic BP
E. Decrease of the number of working nephron
18.Severe poisoning by mercury salts has lead to decrease and then to ceasing of patient’s
urination. There are headache, nausea, and vomiting in this patient. Laboratory investigation
reveals quickly increasing azotemia. The established diagnosis is the acute renal failure, stage of
oligouria-anuria. What disorders of aqueous and osmotic homeostatic does this patient have at this
stage of the disease?
A. Hypoosmolar hypohydration
B. Hyperosmolar hypohydration
C. Isoosmolar hypohydration
D. Hypoosmolar hyperhydration
E. Hyperosmolar hyperhydration
19.A patient with chronic renal disease is edematous, pale, his AP is increased, and he has
vomiting and diarrhea with ammoniac smell. Laboratory investigation reveals protein and
erythrocytes in his urine, hyperazotemia, decrease of erythrocytes and hemoglobin in his blood.
What is the main link pathogenesis of anemia which complicated renal disease?
A. Loss of erythrocytes with urine
B. Toxic influence of urea on bone marrow
C. Deficiency of erythropoietin
D. Impaired of iron absorption due to diarrhea
E. Impaired of regulation of erythropoiesis by endocrine glands 20.A man
suffering from gout complains of pains in the area of kidneys. Ultra sound examination
reveals the presence of renal calculi. What substance increased concentration causes the
formation of calculi in this case?
A. Bilirubin
B. Uric acid
C. Urea
D. Cystine
E. Cholesterol
21.What hormone increased secretion is caused by activation of reninangiotensin system in renal
hypoxia?
A. Hydrocortisone
B. Thyroxin
C. Aldosteron
D. Insulin
E. Parathormone

22.A man has decreased urination, hypernatremia, hypokalemia. What hormone hypersecretion is
the cause of such changes?
A. Vasopressin
B. Atrial natriuuretic factor
C. Parathormone
D. Aldosterone
E. Adrenalin

23.A patient with acute renal insufficiency developed anuria (diumal urination - 50 ml). Which of
the bellow-mentioned mechanisms is the main one in its development?
A. Decrease of glomerular filtration
B. Impairment of renal blood circulation
C, Increase of water reabsorption
D. Increase of sodium reabsorption
E. Difficulty of urine outflow
24.A patient with chronic glomerulonephritis has vegetation of collagenic fibers in interstitium
near tubules in which reabsorption of sodium ions decreases. What underlies these changes in
reabsorption of sodium ions in tubules?
A. Inhibition of energy metabolism
B. Activation of glycolysis
C, Inhibition of lipid peroxidation
D. Stabilization of lysosomal membranes
E. Increase of antioxidative activity

25.The damage of proximal portion of nephron with decrease of sodium ion reabsorption was
observed in a patient after poisoning by corrosive sublimate. How much maximally may sodium
ions be reabsorbed in this part of nephron?
A.65%
B. 80%
C. 50%
D. 35%
E. 20%

26.The damage of proximal portion of nephron was held in a patient in 24 hours later after the
poisoning by corrosive sublimate. What electrolyte reabsorption is impaired in this pathology?
A. Tons of potassium
B. Ions of chloride C. Ions of sodium
D. Ions of calctum
E. Ions of magnesium

27.A patient aged 35 complains of pain in the lumbar area, edema under eyes, and increased
fatigability in usual physical loads. Protein was revealed in patient’s urine (0.99gr/l). Patient’s BP
is -160/110 mmHg. What pathology has the patient?
A. Pyelitis
B. Nephritic syndrome
C. Cystitis
D. Acute renal failure
E. Nephrotic syndrome

28.What pathological process is non-selective, non-massive proteinuria is characteristic for?


A. Nephritic syndrome
B. Urethritis
C, Chronic renal failure
D. Acute renal failure
E. Nephrotic syndrome

29.What origin of protein is the most possible in selective proteinuria with intensity 12 gr/day?
A. Suprarenal
B. Tubular
C. Glomurular
D. Urethra
E. From urinary bladder

30.What pathological process is characterized by combination of massive proteinuria (25gr/day)


with generalized edema?
A. Urethritis
B. Chronic renal failure
C. Nephrotic syndrome
D. Cystitis
E. Nephritic syndrome
31.What pathological process may be complicated by acute renal failure?
A. Chronic lung abscess
B. Viral hepatitis
C. Polycystitic kidneys
D. Extensive burn
E. Hypertensive disease

32.Acute pylonephritis was diagnosed in a patient with complains of increased temperature, pain
in the lumbar area, and frequent and painful urination .Which of the infectious agents is a cause
of the disease?
A. Staphylococci
B. Streptococci
C. Escherichia coli
D. Brucella
D. Anaerobic flora

33.A patient suddenly has developed colicky pains in the are of kidney with irradiation to grain,
nausea, and vomiting; the discharge of urine for
24 hours is 90 ml. Determine mechanism of anuria
A. Impairment of filtration
B. Increase of reabsorption
C. Blocking of a work of a part of nephrons
D. Reflex anuria
E. Obturation of ureters

34.A man of aged 72 is ill with chronic glomerulonephritis. On examination following was
determined: absence of appetite, vomiting, diarrhea, skin itching, anemia, the content of residual
nitrogen in the blood is 45mm. The indicated signs are caused by:
A. Increase of glomerular membrane permeability
B. Disturbance of nephron function
C. Autoimmune damage of nephrone function
D. Renal ischemia
E. Disturbance of concentrating mechanism

35.A patient of age 32 with acute glomerulonephritis, who did not follow regime of NaCl and
water limitation, suddenly has lost his consciousness and convulsions appeared in him. His BP is
220/120 mmHg, he has mydriatic pupils and bradycardia. What complication has appeared in this
patient?
A. Acute heart failure
B. Hypertensive crisis
C. Renal coma
D. Epilepsy
E. Eclampsy
36.A woman aged 25 with frequent prolonged tonsillitis in her anamnesis came to a doctor with
complaints of periodical headaches, undue fatigability, periorbital edemas. Clinic-laboratory
investigations show moderate arterial hypertension, proteinuria, hypoproteinemia,
hyperlipidemia. Bilateral disturbance with insignificant increase of echogenity of renal
parenchyma was diagnosed by ultrasound examination of kidneys. What disease is the mast
possible in this case?
A. Chronic pyelonephritis
B. Acute glomerulonephritis
C. Lipoid nephrosis
D. Chronic glomerulonephritis
E. Acute renal failure

37.Protein, which level didn’t exceed 1g/L, was revealed in the urine of physically healthy young
military men after a hard physical exertion during one day foot match (50 km). What kind of
proteinuria takes place first of all?
A. False proteinuria
B. Organic proteinuria
C. Alimentary proteinuria
D. Dehydrated proteinuria
E. Cyclic proteinuria

38.After a severe trauma a patient developed shock with signs of acute renal failure. What is the
leading mechanism in the development of acute renal failure in this case?

A. Decrease of oncotic blood pressure


B. Increase of pressure in renal arteries
C. Increase of pressure in glomerular capsule D. Damage of outflow of urine
E. Fall of arterial pressure.
39.Decrease in insulin clearance down to 60 ml/min was determined in a patient with chronic renal
failure. What renal function impairment is this connected with?
A. Glomerular filtration
B, Tubular secretion
C, Reabsorption in proximal part of nephron
D. Reabsorption in distal part of nephron
E. Reabsorption in collecting renal tubules.

40.A patient with chronic renal insufficiency has developed anorexia, dyspepsia, impairment of
cardiac rhythm, skin itching. What is the main mechanism of the development of these
impairments?
A. Disturbance of lipid metabolism
B. Accumulation of products of nitrogen metabolism in the blood
C. Changes of carbohydrate metabolism
D. Renal acidosis
E. Disturbance of water and electrolyte metabolism.

41.A patient who has been suffering from osteomyelitis of the mandible for many years has
extensive edemas; marked massive proteinuria is revealed in his urine. Which form of
complications in the course of osteomyelitis has this patient?
A. Pyelitis
B. Nephrotic syndrome
C. Nephritic syndrome
D. Chronic renal failure
E. Renal calculi disease

42.After automobile accident a patient’s BP is 70/40 mm Hg. The patient is unconscious. His
diurnal urination is about 500 ml. There are periodical convulsions, Kussmaul’s respiration in him.
What is the cause of the disorder of urination?
A. Considerable acute hypotension
B. Increase of glomerular filtration
C, Decrease of tubular reabsorption
D. Increase of tubular reabsorption
E. Intoxication by metabolites of nitric exchange

43 In a man rate of glomerular filtration rises in 20% as a result of prolonged starvation. What is
the most possible reason for changes of filtration under indicated conditions?
A. Increased permeability of renal filter
B. Increase in filtration coefficient
C. Increased systemic arterial pressure
D. Decreased oncotic pressure of blood plasma
E. Increase in renal blood flow
44 Under experiment morphological impairment of epithelial cells of distal part of nephron was
induced in rats. What functional processes in kidneys are weakened in this case?
A. Filtration
B. Reabsorption of sodium and glucose
C. Reabsorption of glucose
D. Reabsorption of water and salts
E. Reabsorption of proteins

45.Glucosuria and aminoaciduria were found out in a patient with nephritis. What mechanism of
reabsorption of glucose and amino acids is impaired in this case?
A. Pinocytosis
B. Primary active transport
C. Simple diffusion
D. Secondary Na-dependent transport
E. Phagocytosis
46.Following changes were observed in a patient after poisoning with salts of heavy metals:
increased level of residual nitrogen, hyperphosphatemia, hypersulfatemia, hyperacidemia, and
decreased alkaline reserve. What structures impairment has led to described changes?
A. Glomerulus of nephron
B. Langerhgans’ islets
C, Hepatocytes
D. Cortex of adrenal glands
E. Tubules of nephron

47.4 48-year-old man was admitted to the hospital with aggravation of chronic
glomerulonephritis. Examination of this patient found out a presence of chronic renal failure in
him. What does azotemia in chronic renal failure results from?
A. Decreased glomerular filtration
B. Decreased tubular reabsorption
C, Decreased tubular excretion
D. Disorders of protein metabolism
E. Disorders of water-salt balance

48.A patient was admitted to the hospital with complaints of absence of urination during 24 hours
and pains in lumbar area. Catheterization of patient’s urinary bladder does not relief the patient.
What pathology may be cause of anuria in this patient?
A, Bilateral renal calculi
B. Heart failure
C. Bleeding
D. Vomiting
E. Severe intoxication
49 Inhibitor of phosphorilation in cells of nephron tubules — floridzine, was introduced to an
animal in experiment. What substances reabsorption impairment may develop in this case?
A. Disorders of reabsorption of glucose
B. Disorders of reabsorption of non-organic phosphate and calcium
C. Disorders of reabsorption of amino acids
D. Disorders of reabsorption of proteins
E. Disorders of reabsorption of sodium ions and water

50.What pathological changes are characteristic for glomerulonephritis?


A. Presence of high molecular weight proteins (70,000 D) in urine
B. Presence of fresh erythrocytes in urine
C, Presence of low molecular weight proteins (about 40,000 D) in urine
D. Increased excretion of sodium and hyaline cylinders in urine
E. Leukocyturia without hematuria

ANSWERS:
1.-A.2.-A.3.-E.4.-D.5.-D.6.-A.7.-C.8.-C.9.-C. 10.-A.11.-B.12.-A.13.-A.
14.-A.15.-C.16.-A.17.-E.18.-D.19.-C.20.-B.21.-C.22.-D.23.-A.24.-A.
25.-B.26.-C.27.-B.28.-A.29.-B.30.-C.31.-D.32.-C.33.-D.34.-B.35.-E.36.D.37.-E.38.-E.39.-A.40.-
B.41.-B.42.-A.43.-D.44.-D. 45.-D. 46.-A. 47.-
A. 48.-A. 49.-A. 50.-A.
Topic24.Pathophysiology of the nervous system

1. A 38-year-old man is going undertreatment in a hospital for schizophrenia. Blood contents


of glucose, ketone bodies, and urea in him are norm. Shock therapy with regular injections of
insulin was led to the development of insulin coma, after that the patient’s condition improved.
What was the most possible reason for development of insulin coma?
Glucosuria
moOw>

Dehydratation of tissues
Hypoglycemia
Metabolic acidosis
Ketonemia

2. Smell of apples is felt from the patient in comatose state. The content of glucose is 18
mmol/L. Which coma is more possible in this case?
Hypoglycemic
moOOw>

Toxic
Hyperosmolar
Ketoacidotic
Lactatacidotic
3. A patient is drowsy, with cloudiness of consciousness, his reaction to the strong stimuli is
slow. His skin is pale and dry. He has edemas, muscular fibrillar tremor, midriasis, Cheyne-Stokes
respiration with ammonium smell. Pericardial rub was revealed during auscultation.
Which type of coma has been developed in this patient?
Ketoacidotic
HoOAwWS

Renal
Hyperosmolar
Hepatic
Apoplectic
4. Increase of blood pressure and rapid pulse are noticed in a sportsman at the start before
competitions. Influence of which part of the CNS can above-mentioned changes be explained?
Cortex of hemispheares
moOOw>

Medulla oblongata
Mesencephalon
Diencephalons
Hypothalamus

5. The close clinical examination of the patient, admitted to the reanimation department in
unconscious condition allowed to make a conclusion that the patient was in uremic comatose state.
Which symptom is more characteristic for this type of coma?
Hyperglycemic
moOOw>

Acetone smell from his mouth


Polyuria
Hypernitremia
Non-gas alkalosis

6. The close clinical examination of the patient, admitted to the reanimation department in
unconscious condition allowed to make a conclusion that the patient was in state of diabetic coma.
Which symptom is more characteristic for this type of coma?
Hyperglycemia
mMoOOw>

Smell of ammonium from the mouth


Anuria
Metabolic alkalosis
Decrease of content of nonprotein nitrogen in serum

7. Ptyalism, bradycardia (heart rate 45 beats per minute), miosis are observed in a man. What
is the most possible reason for such changes?
Increase of sympathetic influence
moOOw>

Increase of parasympathetic influence


Decrease of sympathetic influence
Decrease of parasympathetic influence
Decrease of vegetative influence
8. The reactions, typical for stimulation of parasympathetic nerves, appeared after irritation
of throphogenic zone of hypothalamus, including preoptic nucleas and anterior hypothalamic area.
What appears in this case?
Tachycardia
moOOw>

Mydriasis
Exophthalmus
Decrease of BP
Hyperglycemia

9. After amputating the upper extremity a patient had a bad pain in it. Which mechanism of
the pain feeling formation is more possible in this case?
Phantom
moOOw>

Reflex
Hyposecretion of endorphin
Hypersecretion of endorphin
Hyposecretion of encephalin

10. A patient had hemiplegia after insult. What disorder is observed in this case?
Taste
moa >

Balance
Movement
Vision
Hearing

ANSWERS:
1.-C. 2.-D. 3.-B. 4.-A. 5.-D. 6.-A. 7.-B. 8.-D. 9.-A. 10.-C.

Study about disease

1. A 40-year-old man sustained trauma of the cornea and after long treatment cataract appeared.

What is the designation of such an ending of the disease?

A.Pathological state

B.Pathological process

C.Pathological reaction

D.Chronic disease

E.Remission

2. In a patient after long lasting sever disease blood pressure decreased (60/40 mm Hg), it is
accompanied by tachycardia, dispnoae, dizziness. This state can be regarded as:

A.Preagony

B.Agony

C.Shock

D.Clinical death

E.Relapse

3. What are the criteria of mountain disease classification?

A.Ecological

B.Topographical-anatomical

C.Etiological

D.Pathogenetic

E.Statistic

4. What are the criteria of acute leukemia classification?

A.Topographical-anatomical

B.Etiological

C.Ecological

D.Statistic

E.Pathogenetic

5. What are the criteria of tumours classification?

A.Pathogenetic

B.Etiological

C.Ecological

D.Statistic

E.Topographical-anatomical

6. What are the criteria of chicken pox classification?

A.Etiological

B.Ecological

C.Pathogenetic
D.Topographical-anatomical

E.Statistic

7. What are the criteria of allergic disorders classification?

A.Pathogenetic

B.Etiological

C.Ecological

D.Topographical-anatomical

E.Statistic

8. What are the criteria of tropical malaria classification?

A.Ecological

B.Topographical-anatomical

C.Etiological

D.Pathogenetic

E.Statistic

9, What are the criteria of fibromioma of uterus classification?

A.By sex

B.By age

C.By topographical-anatomical principle

D.By ecological principle

E.By pathogenetic principle

10. What are the criteria of Alzheimer disease classification?

A.By age

B.By sex

C.By course

D.By ecological

E.By pathogenetic principle

11. What are the criteria of miners’disease (antracosis) classification?

A.By ecological
B.By pathogenetic

C.By age

D.By sex

E.By statistic

12. What are the criteria of Down disease classification?

A.By etiological

B.By pathogenetic

C.By age

D.By sex

E.By statistic

13. What are the criteria of Kleinfelter disease classification?

A.By etiological

B.By pathogenetic

C.By age

D.By sex

E.By statistic

14. On the second day after sport events a 12-year-schoolboy developed malaise,headache,

increased body temperature up to 37.80 C. What period of disease are these symptoms

characteristic of?

A.Prodromal

B.Latent

C.Contact

D.Period of outburst

E.Final period

15.A 39-year-old patient has been suffering from a stomach disease for the last three years. The

patient’s state worsens in autumn and spring and manifests as pain in the upper part of abdomen,

nausea, heartburns, constipations. What is the designation of this period?


A.Relapse

B.Remission

C.Pathologic state

D.Complication

E.Pathologic reaction

16. In a patient with a sever obstructive pulmonary disease gasp-breathing appeared. What

terminal state is this breathing characteristic of?

A.Agony

B.Preagony

C.Terminal pause

D.Clinical death

E.Biological death

17. Astudent-girl after she came from university classes felt malaise, nausea, headache, flash.

What is the period of disease in the girl?

A.Prodromal

B.Contact

C.Latent

D.Outburst

E.Final

18. A 45-year-old man has complained for the past week on running nose, malaise, headaches,

subfebrile body temperature. The day before hospitalization jaundice, dark urine emerged. Blood

laboratory analysis revealed direct hyperbilirubinaemia; stool is white. What is the period of

jaundice in the patient?

A.Qutburst

B.Contact

C.Prodromal

D.Latent

E.Final
19. A scientist has sustained ionizing radiation while an accident at a laboratory. A disease

manifested by severe reduction of RBC, WBC and platelets count, depression of the organism’s

protection forces. What is period of the disease?

A.Period of outburst

B.Latent

C.Final

D.Contact

E.Prodromal

20. A 7-year-old boy became felt malaise and fatigue, increased temperature in the evening. In

the morning catharal symptoms, fever and lesions on the oral cavity mucosa —Belsky-Filatov-

Coplic plaques appeared. What is period of the disease?

A.Prodromal

B.Period of outburst

C.Latent

D.Final

E.Contact

21. Clinical death is characterized by:

A.Stop of heart work, absence of breathing and reflexes, anaerobic glycolysis;

B.Stop of breathing, presence of pulse and iris reflex, anaerobic glycolysis;

C.Stop of heart work, gasp-breathing, presence of iris reflex, anaerobic glycolysis;

D.Stop of heart work, absence of breathing, reflexes and complete cessation of matters

metabolism;

E.Heart fibrillation; gasp-breathing, presence of iris reflex, anaerobic glycolysis.

22. Development ofiron-deficient anemia results in heavy brain hypoxia. What type of disorder

is the anemia in this case?

A.Typical pathological process

B.Disease

C.Pathological state
D.Pathological reaction

E.Pathological process

1.23. A 33-year-old woman has fallen down and received an open fracture of ulnar bone. On the

fractured bone a plaster of Paris was applied. Because of inobservance of recommendations by

the woman she soon presented with movements disorders in the ulnar joint. Give the designation

to this type of state.

A.Pathological state

B.Typical pathological process

C.Pathological reaction

D.Pathological process

E.Disease

1.24. While an attempt to experimentally reproduce anaphylactic shock a mice developed

seizures, gasp-breathing which lasted for 3 min. What type of terminal state developed in the

mice?

A.Agony

B.Preagony

C.Clinical death

D.Biological death

E.Terminal pause

1.25. What factors are the first to determine duration of clinical death?

A.ATP and creatinine phosphate stores

B.Patient’s age

C.Disease that caused death

D.Rate of the organism’s injury

E.Patient’s sex

1.26. Which factor can substantially prolong duration of clinical death?

A.Hypothermia

B.Hyperthermia
C.Indirect heart massage

D.Artificial breathing

E.Defibrillation

1.27. A 37-year-old woman has been suffering on peptic ulcer disease for 4 years. Every spring

she must take stationary treatment due to:

A.Relapse

B.Remission

C.Exacerbation

D.Complication

E.Pathological reaction

1.28. A 65-year-old man complaints on frequent infections with fevers, fatigue, headaches. After

detailed examination of the patient the diagnosis of chronic leucosis was established. The

patient’s state and laboratory analysis after treatment improved. The patient has:

A.Remission

B.Curing

C.Relapse

D.Pathological state

E.Pathological reaction

1.1.29. A patient with ulcerative disease of the stomach complicated by bleeding, after stationary

treatment felt better. Nevertheless endoscopic investigation revealed a scar and pylorus

narrowing on the place of the previous ulcer. That should be regarded as:

A.Pathological state

B.Pathological process

C.Pathological reaction

D.Exacerbation

E.Complication

1.1.30. A student bought .meat pie on the street near university.After several hours he felt pain

in the stomach, nausea, headache, fluttering before his eyes. Soon vomiting and diarrhea
emerged. The student developed:

A.Pathological process

B.Pathological reaction

C.Pathological state

D.Relapse

E.Exacerbation

1.1.31. During dentist review of the 37-year-old patient’s oral cavity absence of the 1stupper

premolar tooth was discovered. Such a phenomenonis designated as:

A.Pathological state

B.Pathological reaction

C.Pathological process

D.Chronic process

E.Complication

1.1.32. A druggist, that has worked for many years on chemical-pharmaceutical enterprise has

poisoned with cyanides. What criterion should this disorder be classified by?

A.Etiological

B.Topographical-anatomical

C.Sex

D.Course

E.Pathogenetical

1.1.33. Aman after a car accident developed acute blood loss, which was accompanied by acute

cardiovascular deficiency. After half an hour the bleeding stopped, blood pressure began rising,

tachycardia and tachypnoe appeared. What mechanism has come into play?

A.Urgentrecovery mechanisms

B.Relatively stable recovery mechanisms

C.Prolonged recovery mechanisms

D.Pathological reactions

E.Renewal reactions
1.1.34. A 10-year-old-child’s mother decided to become a donor for her boy suffering from

chronic renal failure, and to give him her kidney for transplantation. Compensation due to what

mechanisms would allow the mother to restore her viability?

A.Prolonged recovery mechanisms

B.Urgent recovery mechanisms

C.Relatively stable recovery mechanisms

D.Adaptation reactions

E.Pathological reactions

1.1.35. Which of the following terminal states is characterized by irreversible changes in the

organism?

A.Biological death

B.Clinical death

C.Preagony

D.Agony

E.Terminal pause

1.1.36. In the process of dying the first cells to die are:

A.Neurons of the brain cortex

B.Liver cells

C.Myocardium cells

D.Neurons of the spinal brain

E.Epithelial cells of the skin

1.1.37. A patient who has sustained an electrical shock is lying unconscious, without breathing,

pulse is impalpable. What terminal state is the patient in?

A.Clinical death

B.Biological death

C.Preagony

D.Agony

E.Terminal pause
1.1.38. Resuscitation is possible ifthe timeof clinical death not more than:

A.5 min

B.10min

C.20 min

D.30 min

E.60 min

1.1.39. Chose the indication for electrical heart defibrillation:

A.Ventricles fibrillations

B.Absence of arterial pressure

C.Asystoly

D.“Ineffective heart”

E.Absent pulse above carotid artery

1.1.40. Resuscitation measures includes administrationof sodium hydrocarbon solution with the

purpose of:

A.Correction of metabolic acidosis

B.Prevention of respiratory acidosis

C.Liquidation of hypoxia

D.Correction of metabolic alkalosis

E.Prevention of brain edema

1.1.41. Arterial blood pressure in a patient witha massive brain hemorrhage is 60/30 mm Hg,

pulse -110/ min; breathing rate —36 / min, the patient is unconscious. Hoe can this state be

designated?

A.Preagony

B.Shock

C.Collapse

D.Clinical death

E.Agony

1.1.42. Clinical death in the terms of normothermia lasts for:


A.5-6 min

B.7-8 min

C.1-3 min

D.9-12 min

E.up to 10 min

1.1.43. In a woman suffering from lung tuberculosis calcifications were diagnosed during

roentgenogram; they may be regarded as:

A.Pathological state

B.Relapse

C.Period of maximally developed clinical symptoms

D.Pathological process

E.Complication

1.1.44. A 56-yeal-old man was delivered to a hospital with complaints of the pain in abdomen,

nausea, vomiting,and constipations.He said that he had been suffering for three days. After

objective examination the diagnosis “Intestinal obliteration”. What period of the disease is the

patient in?

A.Period of maximally developed clinical symptoms

B.Latent period

C.Reconvalescent period

D.Prodromal period

E.Contact period

1.1.45. Awoman complaints of an acute pulsating pain in the 2ndlow right tooth, that aggravates

in the night. On examination caries and pulpitiswere diagnosed. What of the following is a

typical pathological process?

A.Inflammation

B.Pulpitis

C.Pain

D.Caries
E.Redness

1.1.46. A 50-year-old man has received treatment for ulcerative disease of the stomach after

which he got relief: the process of digestion normalized, pain disappeared, general state was

improved. Howeverseveral days after the pain and heartburn in the stomach appeared again.

How can this process be designated?

A.Relapse of disease

B.Terminal state

C.Typical pathological process

D.Latent period

E.Remission

1.1.47. What category of pathology can be inborn thigh luxation taken to?

A.Pathological state

B.Disease

C.Pathological reaction

D.Pathological process

E.Complication

1.1.48.A 28-year-old man was hospitalized to surgical department with the symptoms of acute

appendicitis. Objectively was found: pain on palpation in the right iliac region, positive

Shchotkin-Blumberg’s symptom. What is the period of disease in the patient?

A.Period of maximally developed clinical symptoms

B.Contact period

C.Latent period

D.Prodromal period

E.Reconvalescent period

1.1.49. A 16-year-old patient was admitted to a surgical department with the diagnosis “Acute

appendicitis”. What typical pathological processis the main one in development of the disease?

A.Inflammation

B.Hypoxia
C.Fever

D.Tumor

E.Disorder of microcirculation

1.1.50. A 28-year-old woman came to a doctor with complaints of the sore throat, headache and

increased body temperature. What category of pathology can these states be taken to?

A.Pathological process

B.Pathological reaction

C.Pathological state

D.Pathological reflex

E.Protective reaction

INFLUENCE OF ENVIRONMENTAL FACTORS ON THE ORGANISM

1.2.1. What develops during gradual decrease of barometric pressure?

A.Mountain disease

B.Barometric trauma

C.Explosive decompression

D.Pain shock

E.Caisson disease

1.2.2 Alpinists slowly climbed up a mountain slope. They had already 6 hours of climbing

behind. Every step was hard to make due to general lassitude, it was difficult to breath, they felt

palpitation. The pulse rate approached 140 / min. Headache, depression, anorexia and dizziness

were present. What is the main reason of these disorders?

A.Decrease of oxygen partial pressure in the air inhaled

B.Physical load

C.Decrease of atmospheric pressure

D.Decrease of arterial pressure

E.Changes of air temperature

1.2.3. Tourists who have ascended on the height of 2500 m developed mountain disease, that was

accompanied by hypoxic hypoxia. What leading mechanism lies in the basis of this type of
hypoxia development?

A.Decrease of pO2 in the air inhaled

B.Decrease of pCO2 in the air inhaled

C.lncrease of pO2 in the air inhaled

D.Relative blood circulation deficiency

E.Increase of pCO2 in the air inhaled

1.2.4. An alpinist,who was climbing onto the high of 6000 m above the see level,developed

euphoria, inadequate orientation, and hallucinations. What is the main reason of development of

these mountain disease manifestations?

A.Decrease of oxygen partial pressure in the air inhaled

B.Physical load

C.Decrease of atmospheric pressure

D.Decrease of arterial pressure

E.Hyperinflation of frontal sinuses

1.2.5. Tourists during their climbing on the mountains developed signs of mountain disease.

What is the main moment in the pathogenesis of this disease development?

A.Hypoxia

B.Hypocapnia

C.Relative erythrocytosis

D.Absolute erythrocytosis

E.Hypercapnia

1.2.6. A man during his ascending on mountainshas developed mountain disease, which is

characterized by a number of sequential changes in the organism. Point out what from the

following is the primary pathogeneticalfactor in the development of the disease.

A.Hypoxemia

B.Hypocapnia

C.Breathlessness

D.Tachycardia
E.Decrease of mobile activity

1.2.7. Frequent and deep breathing developed in alpinists while their climbing on a mountain

slope, pulse rate was up to 120 /min. Whatis the initiating factor in the arising of these

symptoms?

A.Hypoxemia

B.Hyperoxya

C.Hypocapnia

D.Hypercapnia

E.Hypertonia

1.2.8. During prolonged travel in the mountains partial pressure of oxygen in arterial blood has

decreased to 60 mm Hg (8.0 kPa). What is the reaction of the respiratory system to sucha

change in homeostasis?

A.Hyperventilation

B.Hypoventilation

C.Tissue hyperoxygenation

D.Hyperoxya

E.Hypercapnia

1.2.9. A traveling man on his ascending on mountains has developed symptoms of mountain

disease. What is the main compensatory reaction of the human organism to hypoxia during

ascending on the mountains?

A.Lung hyperventilation

B.Bradycardia

C.Decrease of red blood cells count

D.Decrease of hemoglobin level

E.Decrease of affinity of hemoglobin to oxygen

1.2.10. On physical load in tourists ascending on mountains changes in partial pressure of

oxygen, CO2, and pH are observed. The most effective irritator of carotid sinus chemoreceptors,

which increases lung ventilation is:


A.Low 02 level in the blood

B.Low CO2level in the blood

C.lncreased partial pressure of O2 in the blood

D.Accumulation of lactate in the blood

E.Increase of pH in the blood

1.2.11. A 40-year-old man on his ascending on the height of 3800 m above the sea level

developed mountain disease manifested by changes of breathing pattern. What is the breathing

pattern on the initial stages of mountain disease?

A.Superficial frequent

B.Superficial rare

C.Periodical

D.Slowed deep

E.Rare deep

1.2.12. Tourists who have ascended on the height of 3000 m, developed frequent and deep

breathing. These changes are the result of the stimulation of which structures?

A.Chemoreceptors of carotid sinuses

B.Mechanoreceptors of lung alveoli

C.Baroreceptors of the aorta arc

D.Brain cortex neurons

E.Myocytes of breathing muscles

1.2.13. What can be regarded as compensatory mechanism of mountain disease?

A.Erythrocytosis

B.Bradycardia

C.Anemia

D.Bradypnoae

E.Leukocytosis

1.2.14. Euphoria was observed in a group of studentsduring their mountain climbing; the

students also developed tachypnoae, tachycardia, increased level of erythrocytes and hemoglobin
in the blood.What changes in the blood accompany this state?

A.Erythrocytosis

B.Genuine erythraemia

C.Leukocytosis

D.Leukopenia

E.Thrombocytopenia

1.2.15. A 36-year-old man after prolonged staying on mountains developed increased oxygen

capacity of the blood. Point out the reason of such changes in the organism.

A.Increased erythropoesis

B.Increased lung ventilation

C.Increased blood volume

D.Increase of heart rate

E.Deficiency of iodine in ground and water

1.2.16. People living in the mountain region have increased level of erythrocytes; what is the

mechanism of the red blood cells increased production?

A.Erythropoietin

B.Renin

C.Urokinasa

D.Prostaglandins

E.Provitamin D

1.2.17. People suffering from lung deficiency are recommended to live for some time in a

mountain region. What is the mechanism of their symptoms improvement?

A.Reaction of the organism to hypoxia

B.Low temperature of environment

C.Clean air

D.Decrease of nervous tension

E.Decrease of physical load

1.2.18. What are the adaptive mechanisms playing an important role during breathing on a high
level above the sea level?

A.All thestatedabove

B.Increase of alveolar ventilation

C.Shift of hemoglobin dissociation curve to the left

D.Increase of the lungs diffuse capacity

E.Increase of erythrocytes and hemoglobin level

1.2.19. An alpinist on his mountain climbing developed euphoria, whichlater changed on

headache, dizziness, palpitation and breathlessness; apnoae emerged some time after. What

changes in the acid-base balance developed in this case?

A.Gas alkalosis

B.Nongas acidosis

C.Nongas alkalosis

D.Excretion alkalosis

E.Gas acidosis

1.2.20. Analpinists while his mountain climbing has developed mountain disease signs:

euphoria, tachycardia, impairment of breathing rhythm with the development of Chein-Stocks

pattern of breathing. What type of acid-base disorder is observed in this patient?

A.Respiratory alkalosis

B.Respiratory acidosis

C.Metabolic alkalosis

D.Metabolic acidosis

E.Mixed acidosis

1.2.21. An alpinists while his mountain climbing has developed mountain disease signs:

euphoria, tachycardia, impairment of breathing rhythm with the development of Chein-Stocks

pattern of breathing. Which pathogenetic factor underlies development of mountain disease

symptoms?

F.Hypocapnia
G.Hypercapnia

H.Hyperoxya

|.Hyperasotemia

J.Hypokaliemia

1.2.22. What is the direct reason of hypocapnia in mountain disease?

A.lncrease offrequency and deepness of breathing

B.Decrease of breathing frequency

C.Decrease of breathing deepness

D.Increase of heart rate

E.Increase of blood output per minute

1.2.23. After several intensive arbitrary breathing movements (hyperventilation) made by an

alpinists climbing at a high mountains, he felt unwillingness to breath for some time. What can

be the reason of such a state development?

A.Decrease of the breathing centre excitation

B.Increase of breathing centre excitation

C.lIncrease of CO2partial pressure

D.Decrease of O2partial pressure

E.Increase of blood pH

1.2.24. A traveling man on his mountain climbing developed sighs of mountain disease. Which

of the compensatory mechanisms can deteriorate his state?

A.Lung hyperventilation

B.Tachycardia

C.Increase of red blood cells count

D.Increase of hemoglobin level

E.Increase of hemoglobin affinity to oxygen

1.2.25. Aman who has ascended on the height of 2500 m felt general fatigue, dizziness, his

breathing became frequent and deep. After some time theman lost his consciousness.

Hypocapnia that underlies this state develops due to:


A.Lung hyperventilation

B.Metabolism depression

C.Binding of carbon dioxide with proteins

D.Neutralization of carbonic acid by bicarbonates

E.Absorption of carbonic acid by erythrocytes

1.2.26. Amember of a high mountain expedition developed dizziness and weakness on the

height of 6 km. The alpinists lost consciousness, his breathing stopped. These disorders

developed as a result of:

A.Excessive CO2removal from the organism

B.Excessivecreation of CO2in tissues

C.Deficient O2income to the organism

D.Insufficient O2utilization by tissues

E.Insufficient release of O2by oxyhemoglobin

1.2.27. What will be the effect of decrease of O2pressure in the blood passing through carotid

sinus on systemic arterial pressure?

A.Decrease of systemic arterial pressure

B.Increase of systemic arterial pressure

C.No change

D.Increase and later decrease

E.Decrease and then increase

1.2.28. A 27-year-old alpinist during sleep on the height of 5000 m above the sea level

manifested changes in his breathing pattern. After several deep inspirations breathing stops, after

which deep breathing movements follow again and so on. What is the most plausible reason of

changes in external breathing?

A.Decrease of the breathing centre excitation

B.Increase of the breathing centre excitation

C.Increase of O2partial pressure

D.Increase of blood circulation speed


E.Decrease of air temperature

1.2.29. Due to a long staying of a woman in the mountains on the height of 1000 m above the sea

level she developed an increased oxygen capacity of the blood. What is the direct reason of this

state?

A.Increased production of erythropoietin

B.Increased production of carboxyhemoglobin

C.Increased production of carbhemoglobin

D.Increased production of of catecholamines

E.Increased production of 2,3-dyphosfoglycerate

1.2.30. A frog and mouse were placed in a barocamera and then atmospheric pressure began to

be decreased. It was the mouse that first reacted to decrease of pressure. The frog appeared more

stable to thatpressure. This is due to:

A.Lower grade of frog evolutional development

B.Higher dependence of metabolic processes of the frog’s organism on the oxygen content

in environment

C.Higher grade of frog evolutional development

D.Sufficient development of the frog’s systems responsible for utilization

E.Preliminary saturation of the frog’s organism with oxygen

1.2.31. Anewborn and sexually mature rats were placed in a barocamera, in which pressure was

gradually decreased. The newborn rat appeared to be more stable to hypoxia due to:

A.Lower grade of central nervous system development

B.Sufficient development of systems responsible for oxygen utilization by cells

C.Accumulation of toxic products in the organism

D.Higher grade of evolutional development

E.Saturation of the organism with oxygen

1.2.32. A mouse was placed in a barocamera and the model of mountain disease was reproduced,

the first manifestation of which became appearance of seizures. This is connected with the fact

that the most sensitiveto hypoxia are:


A.Brain neurons

B.Muscle tissue

C.Bone marrow

D.Lungs

E.Kidneys

1.2.33. Two rats were placed in a barocamerawith thepressure in it beinggradually decreased.

To one rat the somnolent substance Nembutal was administered, the second rat was a control

one. In the rat to whom Nembutal was administered the appearance of seizures and terminal

breathing came later than in the control one. How can it be explained?

A.Decrease of the organism’s reactivity

B.Increase of the organism’s reactivity

C.Decrease of anaerobic glycolysis intensity

D.Increase of anaerobic glycolysis intensity

E.Increase of carbohydrate metabolism intensity

1.2.34. Two rats were placed in a barocamera with the pressure in it being gradually decreased.

To one rat the adrenalin was administered, the second rat was a control one. In the rat to whom

adrenalin was administered the appearance of seizures and terminal breathing came earlier than

in the control one. How can it be explained?

A.lncrease of the main metabolism intensity

B.Decrease of the main metabolism intensity

C.Decrease of anaerobic glycolysis intensity

D.Increase of anaerobic glycolysis intensity

E.Increase of carbohydrate metabolism intensity

1.2.35. When the plane is flying up some people feel pain in the ears. This is due to:

A.Expansion of gases in tympanic cavities

B.Expansion of gases in maxillary cavities (sinuses)

C.Expansion of gases in frontal cavities (sinuses)

D.Inflammation of the inner ear


E.Inflammation
of maxillary cavities (sinuses)

1.2.36. Passengers of a plane with impaired hermetic conditions rapidly developed symptoms of

pain in ears, frontal sinuses and accumulations of gases in the intestines. What lies in the bases of

pathogenesis of these symptoms?

A.lncrease of gases volume

B.Decrease of gases volume

C.Saturation of gases

D.Desaturation of gases

E.Acute compression

1.2.37. On the height of 10000 m an abrupt lost of hermetic conditions has appeared on a plane

board. Some seconds after the barometric pressure inside the plane became as atmospheric one

on this height (180 mm Hg). Pilots could not perform quick descending, the plane continued its

fly on this height for some time. What pathological process develops in passengers of the plane?

A.Syndrome of explosive decompression

B.Mountain disease

C.Deepness sickness

D.Caisson disease

E.Pain shock

1.2.38. After a plane cabin lostits hermetic conditions a pilot developed sensor functions

impairment and lost consciousness. What is the reason of this condition?

A.Acute hypoxia

B.Decrease of gases volume

C.Speed
of air movement

D.Phenomenon
of saturation

E.Air humidity

1.2.39. As a result of an accident at a space ship on the height of 70000 m the ship lost its

hermetic conditionsthat led to abrupt death of the ship members. What is the mechanism of their

death?
A.Boiling of all the organisms’ fluids

B.Toxic action of nitrogen

C.Toxic action of oxygen

D.Decrease of gases volume

E.Saturation of gases

1.2.40. Which of the following processes causes death of humans in the terms of lost hermetic

conditions in flying objects at the height of more than 19000 m without oxygen devises?

A.Boiling of the blood

B.Saturation of gases in fluids

C.Toxic action of oxygen

D.Toxic action of nitrogen

E.Low concentration of oxygen in the air inhaled

1.2.41. A diver while working on depth developed signs of the central nervous system disorder,

which manifested as light excitation resembling euphoria. In these conditions CNS is damaged

predominantly by:

A.Nitrogen

B.Carbon dioxide

C.Combined action of oxygen and carbon dioxide

D.Oxygen

E.Lactic acid

1.2.42. On submerging on the depth of 100 m a diver developed impairment of central nervous

system function. What is the most probable reason of the CNS disorders?

A.Saturation of nitrogen

B.Desaturation of nitrogen

C.Hypocapnia

D.Hypoxemia

E.Hypercapnia

1.2.43. On submerging on the depth of 60 m a diver developed impairment of central nervous


system function: euphoria, lost of attention and professional mistakes. These symptoms are

related to the toxic action on neurons of:

A.Nitrogen

B.Oxygen

C.Lactate

D.Carbon acid

E.Amonium

1.2.44. A diver working on the depth of 40 m using breathing mixture of nitrogen, oxygen and

carbon dioxide developed light excitation and euphoria, his movement became uncoordinated,

the sense of responsibility for the work performed was diminished. What is the predominance of

neurological disorders conditioned by?

A.Increased nitrogen dissolving in lipids

B.Decreased nitrogen dissolving in lipids

C.lIncreased oxygen dissolving

D.Decreased oxygen dissolving

E.Increased CO2dissolving

1.2.45. A diver working on the depth of 40 m using breathing mixture of nitrogen, oxygen and

carbon dioxide developed light excitation and euphoria, his movement became uncoordinated,

the sense of responsibility for the work performed was diminished. How is it possible to predict

the development of these symptoms?

F.Replace nitrogen by helium

G.Replace CO2 by helium

H.Carry out the preventive treatment with vitamins before the work

|.Conduct a course of psychological training before the work

J.Replaceoxygen by helium

1.2.46. What is the most likely cause of toxicity of hyperbaric oxygenation?

A.Formation of active oxygen compounds

B.Braking of chemoreceptors reflexogenic zones


C.Mechanical damage to blood vessels

D.Excessive activation of oxidative reactions

E.Mechanical damage to the lungs.

1.2.47. The man, who was in a vacuum chamber with high oxygen pressure over 1 atmosphere,

developed significantly increased levels of lipid peroxidation products. How will be vessel’s

lumen changed in these conditions?

A.Vessels narrow due to increased permeability of cell membranes for Ca ++

B.Vessels dilate due toreducing membrane permeability to Ca ++

C.Vessels dilate due to increased level of cAMP

D.Vessels narrow due to increased permeability of cell membranes to Na +

E.Vessels narrow due to increased cGMP

1.2.48. After the rapid rise of a diver from the depth of 25 m, he developed: joint pain, itching of

the skin, and poor vision. This syndrome is called:

A.Decompression sickness

B.Deep disease

C.Altitude sickness

D.Mountaindisease

E.Sea disease

1.2.49. Elevation of divers after workat depth is conductingvery slowly. This is done with the

purpose to prevent the development of:

A.Gas embolism

B.Compression ischemia

C.Saturation

D.Venostasis

E.Thrombosis

1.2.50. A diverhasperformed caisson works at the depth of 15 m for one hour, then quickly

raised to the surface. After half an hour he felt weakness, increasing pain in muscles and joints,

headache, visual and hearing impairment. What is the reason of the disorders describedabove?
A.Gas embolism

B.Thromboembolism

C.Saturation

D.Venostasis

E.Compressionischemia

1.2.51. An accident happened during works performed at a depth and necessity f an urgent

salvage of a diver emerged. Soon his state got worse. What can be the possible reasonof this

deterioration?

A.Gas embolism

B.Hyperoxia

C.Trombembolism

D.Hypercapnia

E.Hypocapnia

1.2.52. A diver has carried out a caisson work at the depth of 15 m for an hour, and then he was

quickly raised on the surface. After half an hour he felt weakness, increasing pain in muscles and

joints, headache, impairment of hearing and vision. What measures should be carried outfor

removal of these disorders?

A.Put the patient into barocamera with an increased pressure.

B.Put the patient into barocamera with an decreased pressure.

C.Administerpain-relieving medicines

D.Conduct general strengthening of the organism

E.Administer membrane stabilizers

1.3. EFFECT OF IONIZING RADIATION ON THEORGANISM

1.3.1. A 45-year-old man,who worked on liquidation of the consequences of Chernobyl disaster,

sustained ionizing radiation andin the course of acute radiation sickness developed hemorrhagic

syndrome. What has the greatest importancein the pathogenesis of this syndrome?

Thrombocytopenia

Abnormalstructure of the vascular wall


Increasedactivity of anticoagulation factors

Increasedactivity of fibrinolytic factors

Dicreasedactivity of coagulation factors

1.3.2. Liquidators of theconsequences ofChernobyl disasterhave sustained radiation exposure

ofdifferent dosages. What isthe mechanism of direct action of ionizing radiation on the

organism?

Formation of free radicals

Damage to nervouscells

Damage to the red bone marrow

Changeofthe hereditary information in DNA

Destruction of the cell membranes

1.3.3. One of the molecular mechanisms of indirect effects of ionizing radiation is the formation

of lipid and quinone radiotoxins. What is the mechanism of radiotoxinseffects on cells?

A.Inhibition of nucleic acids synthesis

B.Activation of LPO (lipid peroxidation)

C.Stimulation of pyrimidine bases oxidation

D.Inhibition of the antioxidant system activity

E.Deamination ofpurine bases

1.3.4. One of the manifestations of acute radiation sickness is the hematological syndrome. What

changes in the peripheral blood are characteristic of the period of initial reactions?

A.Leukocytosis, limphocytopenia

B.Leukopenia, limphocytopenia

C.Leukopenia, thrombocytopenia

D.Leukocytosis, thrombocytopenia

E.Leukopenia, anemia, thrombocytopenia

1.3.5. The leakageof radioactive products occurred as a result of damage to anuclear reactor.

There were three people in the area of increased radiation. Tentatively, they got 2.50 -3.0 Gy.

They were immediately hospitalized. What consequences should be expected concerning the
patients?

A.Medullary form of acute radiation disease

B.Intestinal form of acute radiation disease

C.Chronic radiation disease

D.Cerebral form of acute radiation disease

E.Acute leukemia

1.3.6. Which investigationsshould be perfomedfor the interpretation of severity of hjury in the

hospitalized patients with acute radiation disease?

A.Total bloodcell count, bone marrow aspiration, immunological investigations of blood,

coagulation assay

B.Immunological investigations of blood

C.Coagulation assay, total bloodcell count, bone marrow aspiration

D.Bone marrow aspiration

E.Bone marrow aspiration, immunological investigations of blood

1.3.7. What pathological changes occur with thepatientafter ionizing radiationin the second

period of acute radiationdisease?

Disappearance of manifestationsof the nervous systemoverexcitation, leukopenia,

thrombocytopenia

Leukopenia, thrombocytopenia, anemia, infectious complications

Headache, anxiety, lability of blood pressureand pulse

Headache, anxiety, lability of blood pressure, limphopenia

Dyspeptic disorders, autointoxication

1.3.8. A patientafter exposure to ionizing radiation in the dosagesof 3.5 Gydeveloped anemia,

leukocytopenia, thrombocytopenia, limphopenia. What period of acute radiation diseaseare

these signscharacteristic of?

A.Third

B.Second

C.First
D.First and second

E.Second and third

A patient applied to the doctor with complaints of hypodermic hemorrhage at insignificant

mechanical traumas. What from enumerated below can be reason of such phenomenon?

Thrombocytopenia

Leukopenia

Decrease of hemoglobin concentration

Erythropenia

Lymphocytosis

A patient applied to the policlinic with complaints of general weakness, dyspnea. Shortly before it

he accepted levomycetin for the prophylaxis of intestinal infection. In blood: erythrocytes —

1,9x1012/I, Hb — 58 g/I, colour index— 0,9, leukocytes — 2,2x109/I. What type of anemia does it

testify about?

Hypoplastic

lron deficiency

~Hemolytic

~Metaplastic

~Aplastic}

A patient with paradontosis takes antimicrobial drugs during the long time. Results of blood test:

erythrocytes — 2,x1012/I, hemoglobin — 60 g/I, colour index — 0,85, leukocytes — 2,5x109/I. What

type of anemia has this patient?

Metaplastic

lron deficiency

Hemolytic

Protein deficiency anemia

Hypoplastic
A woman 55 years old suffers from thyrotoxicosis. Results of blood test: amount of erythrocytes —

5,9x1012 /I, content of hemoglobin — 171 g/l, colour index — 0,9, amount of leukocytes — 4,9x109/I.

How these changes of blood are named?

Absolute erythrocytosis

Hypoplastic anemia

Absolute leukocytosis

Leukemoid reaction

Relative leukopenia

Ina patient tumor of head of pancreas was revealed which is accompanied with steatorrhea and

hemorrhage in skin. Concentration of prothrombin and other factors of coagulation is diminished.

What Is the mechanism of hemorrhage origin in this case?

Avitaminosis C

Deficit of trypsin

Cholemia

Deficit of pepsin

Avitaminosis K

Aman appealed to the doctor with complaints of appearance of hypodermic hematomas. In the

blood test: erythrocytes — 2,5x1012/l; hemoglobin — 80,0 g/l; colour index — 0,9. Thrombocytes —

40,0x109/I. Leucocytes — 5,8x109/I. Leucocytic formula: basophils — 5 %, eosinophils — 15 %,

myeloblasts — 6 %, myelocytes — 10 %, metamyelocytes — 18 %, stab neutrophils — 26 %,

segmentonucleonic neutrophils — 10 %, lymphocytes — 8 %, monocytes — 2 %. What pathology is in

the patient?

Chronic myelocytic leucosis

Leukemoid reaction

Acute myeloblastic leucosis

Basophilic leucocytosis

Eosinophilic leucocytosis

A patient 33 years old was admitted to the hospital with an abscess of femur. Leukogram:
leucocytes — 20,0x109/I, basophils — 0 %, eosinophils — 1 %; neutrophils: myelocytes — 5 %,

metamyelocytes — 10 %, stab neutrophils — 12 %, segmentonuclear neutrophils — 60 %,

lymphocytes — 10 %, monocytes — 2 %. This leukogram testifies about

Shift of leukocyte formula to the left

Acute myeloblastic leucosis

Degenerative shift to the left

Chronic myelocytic leucosis

Leukemoid reaction

A patient 40 years old was operated for acute phlegmonous appendicitis. Amount of leucocytes —

12,0x109/lI. In the smear of blood: basophils — 0 %, eosinophils — 2 %, myelocytes — 0%,

metamyelocytes — 1%, stab neutrophils — 35 %, segmentonucleonic neutrophils — 50 %,

lymphocytes — 10 %, monocytes — 2 %. Stab neutrophils ere with piknosis of nucleus, in some from

them there are toxogenic granulas. How such change of leucocytic formula is named?

Regenerative-degenerative shift to the left

Hyperregenerative shift to the left

Regenerative shift to the left

Leukemoid reaction

Nucleonic shift to the right

A patient appealed to stomatologist with complaints of hemorrhagic from gingiva. Blood test:

erythrocytes — 3,6x1012/I, hemoglobin — 120 g/l, colour index — 1,0; leucocytes — 56,0x109/I,

myeloblasts — 3 %, promyelocytes — 2 %, myelocytes — 3 %, metamyelocytes — 1%, stab

neutrophils — 12%, segmentonucleonic neutrophils — 4 %, eosinophils — 8 %, basophils — 8 %,

lymphocytes — 16 %, monocytes — 3 %, thrombocytes — 30x109/I. What pathology of blood is in the

patient?

Chronic myelocytic leucosis

Leukemoid reaction

Chronic monocytic leucosis

Chronic erythromyelosis
Chronic eosinophilic leucosis

At the analysis of leukogram of a patient was revealed: amount of leucocytes — 250,0x109/I,

leucocytic formula: myeloblasts — 1%, promyelocytes — 2 %, myelocytes — 16 %, metamyelocytes

— 18 %, stab neutrophils — 20 %, segmentonucleonic neutrophils — 15 %, basophils — 8 %,

eosinophils — 9 %, lymphocytes — 9 %, monocytes — 2 %. What type of chromosomal anomaly may

leads to illness in this patient ?

Translocation of segment of 22th chromosome on 9th

Translocation segment of 8th chromosome on 14th

Inversion segment of 7th chromosome

Inversion segment of 15th chromosome

Trisomia on a 8th pair

Blood test of child: amount of leucocytes — 20,0x109/I; basophils — 1 %, eosinophils — 16 %, stab

neutrophils — 3 %, segmentonucleonic neutrophils — 60 %, lymphocytes — 18 %, monocytes — 2 %.

These results testify about presence

Chronic myeloid leucosis

Leukemoid reaction

Chronic lymphoid leucosis

Neutrophilic leucocytosis

Eosinophilic leucocytosis

In a patient at examination of peripheral blood there are: hemoglobin — 80 g/I, erythrocytes —

3,2x1012/I, leukocytes — 3,5x109/I. Leycocytic formula: basophils — 5 %, eosinophils — 9 %,

myeloblasts — 3 %, promyelocytes — 8 %, myelocytes — 11 %, metamyelocytes — 22 %, stab

neutrophils — 17 %, segmentonucleonic neutrophils — 19 %, lymphocytes — 3 %, monocytes — 3 %.

What pathology is in the patient the most likely?

Chronic myelocytic leucosis

Acute promyelocytic leucosis

Chronic erythroblastic leucosis

Acute myeloblastic leucosis


Chronic eosinophilic leucosis

In a patient is increase of lymphatic nodes, hepatosplenomegalia. Hemoglobin — 80 g/l, leucocytes —

90,0x109 /I, lymphocytes — 75 %, ESR — 35 mm/hour. In the smear of peripheral blood there are a

lot of Gumprecht’s shadow. What disease is in a patient?

Chronic lymphocytic leucosis

Acute lymphoblastic leucosis

Acute myeloblastic leucosis

Pernicious anemia

lron deficiency anemia

In a patient with the chronic abscess of lungs there are leucopenia, increase of segmentonucleonic

leucocytes, increase of stab neutrophils, metamyelocytes is absent. Among of stab there are

leucocytes with vacuolated cytoplasm, toxogenic granulas, Dele’s bodies. How shift of leucocytic

formula is named in this patient?

Regenerative

Hyperregenerative

Hyporegenerative

Regenerative-degenerative

Degenerative

In a woman after days after abortion neutrophilic leukocytosis with regenerative shift to the left was

revealed. What is the mechanism of its development?

Increase of leukopoiesis

Redistribution of leukocytes in vessels

Decrease of leukocytes destruction

Delay of leukocytes emigration

Transport of leukocytes from tissues in vessels

Ina worker which worked with benzol the amount of leukocytes in blood is 1,0x109/I. Leucocytic

formula: basophils — 0 %, eosinophils — 1 %, stab neutrophils — 0 %, segmentonucleonic neutrophils

—15 %, lymphocytes — 60 %, monocytes — 24 %. What variant of pathology of leucocytes is in this


case?

Agranulocytosis

Aleukia

Monocytosis

Neutrophilia

Lymphocytosis

In the blood of a patient there are leucocytosis, sharp increase of amount of stab granulocytes and

metamyelocytes, myelocytes and promyelocytes. What is type of nuclear shift in the patient?

Hyperregenerative shift to the left

Regenerative shift to the left

Nucleonic shift to the right

Degenerative shift to the left

Regenerative-degenerative shift to the left

In the Itsenko-Cushing’s illness which is characterized by surplus formation of adrenocorticotropic

hormone, is arose

Eosinopenia

Neutropenia

Lymphocytopenia

Basophilia

Monocytosis

In the purulent endometriris is arose leucopenia in base which lie

Inhibition of leucopoiesis

Destruction of leucocytes in blood

The lysis of leucocytes in spleen

Redistribution of leucocytes to internal organs

Secretion of leucocytes from an organism

Leucosogenic viruses which get to the cell cause leucosis because they

Activate cellular oncogene


Disorder process of transcription

Disorder process of translation

Stimulate formation of energy

Activate lysosomal enzymes

Nuclear Gumprecht’s shadow - it is:

Destroyed lymphocytes

Rest of nucleus in megalocytes

Basophilic including in leucocytes

Rest of nuclear membrane

Alkalized erythrocytes

Simultaneous increase of basophils and eosinophils {basophilic-eosinophilic association) are

characterized for

Chronic myelocytic leucosis

Chronic lymphocytic leucosis

Chronic erythroblastic leucosis

Acute myeloblastic leucosis

Acute megacaryoblastic leucosis

The decrease of thyroid function (hypothyroidism) is accompanied

Eosinophilia

Neutrophilia

Lymphocytosis

Monocytosis

Basophilia

The hereditary neutropenia is a result

Defect of neutrophils differentiations

Intravascular leucolysis

Delay of leucocytes in depot

Draw of leucocytes to mucous membranes


Destruction of leucocytes in spleen

The philadelphian chromosome formates as a result

Translocation of segment 9th chromosome on 22th

Translocation of segment 21th chromosome on 22th

Translocation of segment 21th chromosome on 14th

Translocation of segment 21th chromosome on 15th

Translocation of segment 22th chromosome on 9th

Choose the correct sequence of organism reactions, which represents pathogenesis of edema

development at heart insufficiency.

Slow down of blood flow - exit of blood liquid out the vessels - activation of suprarenal glands -

releasing of aldosterone - retention of sodium

Slow down of blood flow - increase of diuresis - activation of suprarenal glands - releasing of

vasopressin - retention of water

Slow down of blood flow - activating of sympathetic-adrenal system - releasing of adrenalin -

retention of calcium

Slow down of blood flow - activation of hypophysis - releasing of thyroxin retention of sodium and

calcium

Slow down of blood flow - exit of blood liquid out the vessels - activation of thyroid glands -

retention of iodum

Choose the indexes, which characterize the stage of exhaustion of heart hypertrophy

Diminishing of activity of cardiomyocytes genetical structures, diminishing of proteins synthesis,

accumulation of calcium

Increase of activity of cardiomyocytes genetical structures, diminishing of proteins synthesis,

accumulation of sodium

Increase of activity of cardiomyocytes genetical structures, activating of proteins synthesis,

accumulation of hydrogen

Diminishing of activity of cardiomyocytes genetical structures, diminishing of nucleic acids

concentration, deficit of calcium


Diminishing of activity of cardiomyocytes genetical structures, increase of nucleic acids

concentration, deficit of calcium

Choose the sequence of reactions, which represent pathogeny of acid-base balance violation at heart

insufficiency in the stage of decompensation.

Hypoxia - glucolysis activation - surplus synthesis of lactic acid - acidosis

Hypoxia - glucolysis activation - increase synthesis of adenosinthreephosphate acid - acidosis

Hypoxia - activating of glucogenesis - increase synthesis of glucogen - alkalosis

Hypoxia - activating of lipolysis - surplus synthesis of fat acids - alkalosis

Hypoxia - activating of gluconeogenesis - disintegration of glucogen — acidosis

Contractile damage of cardiomyocytes in zone of ischemia at development of reperfusion syndrome

is the result of

Accumulations of calcium in cardiomyocytes

Accumulations of lactic acid in cardiomyocytes

Deficit of magnesium in blood

Deficit of glucose in blood

Accumulation of sodium in cardiomyocytes

Development of sinus tachycardia is the result of pace-makers activity change, namely

Increase of slow diastole depolarization speed, decrease of crutical threshold potential

Decrease of slow diastole depolarization speed, increase of crutical threshold potential

Decrease of slow diastole depolarization speed, decrease of crutical threshold potential

Increase of slow diastole depolarization speed, increase of crutical threshold potential

Increase of slow diastole depolarization speed, hyperpolarization

Dispnoe, tachycardia, cyanosys of visible mucous membranes appeared in sick with myocardium

infarction that testified hypoxia in t he result of heart insufficiency. These signs are caused by

Reducing of red blood cells number

Insufficient concentration of haemoglobin

Insufficient concentration of oxigene in a blood

Violation of oxyhemoglobin dissociation


Reducing of blood flow speed

For diagnose of sinus arrhythmia it is necessary to discover on an electrocardiogram

Difference of R-R duration = 0,2 sec, wave P is placed before QRS, complex QRS is not changed

Difference of R-R duration = 0,1 sec, wave P is placed before QRS, complex QRS is not changed

Difference of R-R duration = 0,25 sec, wave P is placed after QRS, complex QRS is not changed

Difference of R-R duration = 0,45 sec, wave P is placed after QRS, complex QRS is deformed

Difference of R-R duration = 0,15 sec, wave P is placed before QRS, complex QRS is deformed

It is known that compensatory hyperfunction {it is essence of emergency phase of heart

insufficiency) conduces for cardiomyocites hypertrophy. It is happened because so-called ,,limited

mechanisms”, which cause depression of cardiomyocites contractility. A leading role in

pathogenesis of this phenomenon belongs to

Deficit of energy and accumulation of hydrogene ions

Deficit of energy and accumulation of calcium ions

Activation of nuclea and surplus of sodium ions

Activation of glucolysis and accumulation of calcium ions

Deficit of energy and accumulation of sodium ions

It is proved that appearance of the state which has name ,,hibernal myocardium” can be

complication of myocardium infarction. This state is characterized by the changes of functional

descriptions of heart, namely by

Reducing of pumping function in condition of rest without cardiomyocytes cytolysis

Reducing pumping function at the physical loading with cardiomyocytes cytolysis

Increase of pumping function at the increase of adrenalin level without cardiomyocytes cytolysis

Reducing of pumping function in the condition of stress with cardiomyocytes cytolysis

Increase of pumping function in the conditions of rest with cardiomyocytes cytolysis

What changes of pace-makers activity do cause development of sinus bradycardia?

Decrease of slow diastole depolarization speed, decrease of crutical threshold potential

Increase of slow diastole depolarization speed, increase of crutical threshold potential

Increase of slow diastole depolarization speed, decrease of crutical threshold potential


Increase of slow diastole depolarization speed, hyperpolarization

Decrease of slow diastole depolarization speed, increase of crutical threshold potential

What is the main in formation of ectopic areas of excitation in heart?

Damage of the conducting ways, electric unhomogenity of myocardium, depression of sinus node

Anomaly of the conducting ways, hypoxia of myocardium, activation of sinus node

Additional conducting ways, activation of sinus node, depression of atrium-ventricular node

Activation of sinus node, activation of atrium-ventricular node, additional conducting ways

Activation of sinus node, anomaly of the conducting system

What properties of modified lipoproteins do conduce for development of atherosclerosis?

They increase of vessels permeability, activate macrophages; stimulate smooth musclle cels

proliferation

They co-operate with apoB,E receptors, promote of hyperlipoproteinemia, activate of

endotheliocytes

They penetrate easily through the vessel wall, promote formation of connective tissue, well

co-operate with apoC receptors

They activate thrombocytes and phagocytes, activate the synthesis of apoproteins

They oppress of endotheliocytes proliferation, cause damage of vascular wall, oppress activity of

phagocytes

What is the main in pathogenesis of Dresler’s syndrome, which arises after carried myocardium

infarction?

Organism sensitization by the myocardium antigens

Decrease of resistance to the infection

Activating of saprophyte microflora

Intoxication of organism by the products of necrosis

Excretion in blood of myocardium enzymes

What is the role of smooth muscle cells in pathogenesis of atherosclerosis?

They migrate in to intima, proliferate actively, absorb cholesterol, and stimulate the synthesis of

connective tissue
They conduce for synthesis of connective tissue, absorb threeglicerides, and excrete lipolytic

enzymes

They distroy a cholesterol, distroy a collagen, proliferate actively, synthesize the high density

lipoproteins

They metabolize carbohydrates, synthesize a collagen, activate endotheliocytes, synthesize of low

density lipoproteins

They conduce for synthesis of connective tissue, absorb cholesterol, and excrete lipolytic enzymes

A patient, 64 years old, was hospitalized with complaints of a cough with sputum, expressed

dyspnea . During examiation the next signs were revealed: position is forced, breath rate — 32/min,

the intercostales muscles take part in the breathing. During X-ray examination the increased

transparency of lungs were determined. What is the most important in the pathogenesis of

respiratory failure in this patient?

Decrease of elastic properties of lungs

Accumulation of sputum in the bronchial tubes

Thinning of mucus shell of bronchial tubes

Insufficiency of the surfactant system of lungs

Fibrosis of lungs

A patiet with the penetrating wound of pectoral wall was hospitalized to a clinic. What form

disorders of the external breathing may develop in this patient?

Ventilative-restrictive

Obstructive

Primary-diskinetic

Difusion-restrictive

Difusion-pneumonosis

A young man having a cranial-cerebral trauma is in the serious condition. The breathing is

characterized with the convulsive attempts to breathe, which is not halted, that is sometimes broken

by exhalation. What type of breathing has this patient?

Gasping
Kussmaul’s

Chein-Stocks’

Biot’s

Apneic

An young man with the signs of morphin poisoning was hospitalized to the emergency department.

His breathing is shallow and slow as a result of oppression of respiratory center. What type of

breathing disorders is present in this patient?

Disregulative disorders of alveolar ventilation

Perfusive

Ventilation restrictive

Diffusive

Ventilative obstructive

In the laboratory animal has Gasping. It develops as a result of:

Stimulation cells caudal part of medulla oblongata

Acceleration reflex Hering-Breyer’s

Increase of haemoglobin in a blood

Deceleration of the Hering-Dreyers reflex

Irritation respiratory center by the surplus CO2

Put the scheme of pathogeny of hepatic icterus at protein starvation: decrease in the hepatocytes of

proteines Y and Z > ?> disorder of bilirubin transport > decreased transfer of it from blood through

cytoplasmatic membrane in the hepatocyte > icterus

Disorder of bilirubin cupture by ligandes

Decrease of glucuroniltransferase activity

Strong connection of bilirubin proteins

Disorder of blood circulation in the liver

Energetic insufficiency

The sequence of development of acute pancreatitis includes next links: damage of cells >

?>bradykinin > edema, pain, inflammation. Define the absent link of pathogeny
Activating of kallikreinogen

Activating of trypsin

Development of autoimmune reaction>>

Disorders of microcirculation

Irritation of nervous ending

Ina patient which long time suffers of chronical enterocolitic after use meat food arise meteorism,

diarrhea, colics. With deficiency of what enzyme in intestine this associated?

Peptidase

Saccharase

Maltase

Amylase

Glycogensynthetase

A boy 16-year-old complains of weakness, nausee, pain into right hypochondrium. On examenation

was revealed jaundiced sclera and skin. In the blood is large conjugated bilirubin, in the

feces-stercobilin in litlle amount, in the urine-urobilinogen and bilirubin. What type jaundice it is

characterized for?

Mechanical

Functional

Hemolytic

Hereditary

Parenchymatous

A patient complains of dyspeptic disorders, melena, hemorrhoidal bleeding. Belly and liver

increased, sclera slightly jaundice What is pathology may be that symptoms in?

Portal hypertension

Enteritis

Intestine autoinxication

Colitis

Ulcer disease
A patient complains of irritability, sleeplessness, rapit weakness, itch skin, yellowing skin and

mucous membrane, acholic feces. AP 100/75 mm Hg, cardial rate — 56/min. What jaundice it is

characterized for?

Mechanical

Hemolytic

Parenchymatous

Hereditary

Hepatic

A patient complains of weakness, subfebrile temperature, jaundiced of sclera, dark urine, slightly

stained feces. Analysis of blood: conjucated bilirubin — 27,4 mmol/l, unconjucated bilirubin — 51,3

mmol/l. In the urine — urobilin. What is pathology take place in the patient?

Parenchymatous jaundice

Mechanical jaundice

Hemolytic jaundice

Cholemic syndrome

Portal hypertension syndrome

A patient complaints on bad appetites weight loss, pain in epigastric department. For analysis of

gastric juice was revealed achylia. What does term meaning?

Lack of free HCl

Lack of acidity

Lack of free and binded HCl

Lack of gastromucoprotein

Lack of free HCI and pepsin

A woman 33-year-old suffers of hepato-cerebral dystrophy (Wilson’s disease). In the blood

decreased ceruloplasmin contents, in urine — sharply increased aminoacid contents. Effort of what

process was stipulated that changes?

Complex formation of aminoacid with copper

Desintegration of tissuel proteins


Synthesis of urea

Glyconeogenesis

Deamination

A woman 33-year-old suffers with hepato-cerebral dystrophy (Wilson’s disease)/ In the blood —

decreased ceruloplasmin contents. In urine — sharply increased aminoacid contents. Disorder of

what process was caused these changes?

Copper metabolism

Glyconeogenesis

Deamination

Synthesis of urea

Desintegration of tissuel proteins

After carried plural traumas and blood loss in a victim stomach dyskinesia developed on

hypokinetic type. Motore disorders of stomach in this case arose as a result of

Weakening of general tone of organism

Use of fatty food

Appearances of the unpleasant taste feelings

Permanent phobia for the passed

Appetite loss

After removal of duodenum in a patient syndrom of duodenal insufficiency developted due to

disorder its endocrine function with phenomenon of cells deficiency of APUD-system. What from

called hormones is produced by ?-cells this part of intestine

Secretin

Serotonin

Insulin

Histamine

Glucagon

After the carried hepatitis Ain a patient maintenance of free corticosteroids was increased in blood.

Likely reason of disorders?


Deficit of transcortin

Disorder of metabolism in the liver

Surplus production of adrenals

Activation of blood complement

Liberation of peptidases
from cells

After the carried hepatitis in a sick child the signs of hypovitaminosis B1 appeared as a result of

Disorder of formation of active forms it?

Decrease of absorption
it in bowels

Compete use by microflora

Insufficient receipt with meal

Forsing out by ascorbic acid

After the carried rheumatism in a sick 33 years during a few years was formated insufficiency of

mitral valve of heart and superhepatic portal hypertension reason of which is

Rightventriculary insufficiency of heart

Compression of portal vein by transsudate

Embolism of portal vein

Thrombosis of lower cavity vein

Congestion of blood in lower cavity vein

After the resection


of stomach in a patient in connection with the duodenum ulcer the malabsorption

syndrome developed, reason of which became

Diminishing of microvillus amount of thin bowel

Deficit of villicrinin hormone

Strengthening of bowel peristalsis

Activation of fermentative processes

Loss of electrolytes and water

After transfered infection with diarrhea and developed malabsorption syndrome with decrease of

aminoacids and glucose adaption. The main role in pathogenesis of activity transport their belong to

ions deficit?
Na

Mg
Ca

cl

As a result of liver insufficiency in a patient the signs of hyperaldosteronism developed due to

Disorder of hormone excretionis with urine

Surplus formation in adrenal glands

Insufficient binding of hormone with proteins

Increase of sensitiveness of receptors to the hormone

Hormone destruction disorder in a liver

At fiberoscopic research ofa patient 56 years was revealed cancer of stomach in lated stage and

intrahepatic portal hypertension reason of which is probably

Metastases of tumor in the liver

Compression of portal vein by the tumor

Metastases of tumor in the mesenteric nodes

Compression of stomach veins by the tumor

Decline of tone of portal vein

At research of urine of a patient with hepatitis was revealed increase maintenance of amino acid.

Likely reason of this disorder?

Disorder of oxidative deamination

Use of meat meal

Increase of secretion in the kidneys

Disintegration of cells in the liver

Deficit in the meal of nicotine acid

At the height of hepatitis in a patient was icreased maintenance of histamine in the blood that testify

about

Strengthening of decarboxylation of histidin in the liver


Development of allergic reaction in the liver

Decreasing of histaminase activity

Decrease of binding of histamine by red corpuscles

Strengthening of liberation of histamine from cells

For differentiation of functional disorders of stomach secretion from organic lesion to patient is

proposed tests with straight stimulation of secretion. The most strong stimulating action of secretory

cells mucous membrane cause:

Cholecystokinin

Acetylcholine

Insulin

Histamine

Gastrin

Heavy ecephalopathy at the Crigler-Najjar syndrome develops in the pathogenic sequence: inherited

defect of enzyme of UDPh-glucuroniltransferase > ? > ecephalopathy > nuclear icterus. Complete

the scheme of pathogeny

High level of free bilirubin

Lesion of hepatocytes

Decrease of permeability of hepatocytes membrane

Disorder of transport of conjucated birubin in blood

Decrease of bilirubin secretion in the bile

Ina child 8 months was revealed disorder of digestion in the bowels, signs of pellagra with

disorders of neuro-psychic development. Likely reason of disorders?

Congenital disorder of tryptophan absorption

Decrease of pepsin secretion by mucus stomach

Insufficient secretion of pancreatic juice

Deficit in the meal of nicotine acid

Disorder of metabolic function of liver

Ina child with hemolytic disease of the new born developed encephalopathy. Increase of what
matter in blood caused lesion of CNS?

Unconjucated bilirubin

Complex bilirubin-albumin

Conjucated bilirubin

Verdoglobin

Biliverdin

In a man 38-year-old is observed jaundiced skin, anemia, spleen increased, hyperbilirubinemia at

the expense of unconjugated bilirubin., stercobilirubinuris, hypercholic dark feces. What state the

most characterized that changes for?

Hemolytic jaundice

Mechanical jaundice

Hepatic jaundice

Gilber’s syndrome

Hepatic insufficiency syndrome

Ina men 25-year-old which got closed craniocerebral trauma was observed long unrestrained

vomiting. What mechanism stipulated vomiting act

Straight irritance of vomiting centre by increased intracranial pressure

Straight excitement of vomiting centre by toxic substances of blood

Straight excitement of vomiting centre by impulses from stomach and intestine

Activation of vomiting centre as a result of movement and secretory stomach function increase

Excitemeut of vomiting centre through branches of sympathic nerves

In a men 48-year-old which suffers of stomach ulcer was determined secretion increase and increase

of gastric juice acidity . What is mechanism stipulate this phenomenon?

Increase histamine output

Increase secretin output

Increase motilin output

Increase catecholamines output

Increase cholecystokinin output


In a men was revealed hypersecretion of gastric juice at the time examination. Physician

recommended to except from diet saturated broth and vegetable-water because they contets matters

which stimulate gastric secretion, and namely:

Extractive substancies

Hydrochloric acid

Large of carbohydrates

Large of fat

Gastrin

Ina patient 27-year-old was revealed patholodical changes in liver and brain. In the plasma — sharp

slowing, and urine increase copper contents. Activity of what enzyme in serum blood necessary to

defermine
in that case?

Carbonic anhydrase

Xanthine oxidase

Alcoholdehydrogenase

Leucinaminopeptidase

Ceruloplasmin

Ina patient 45 years duodenitis developed, but for that secretory function of pancreas was

disordered as a result of decrease of secretion of

Secretin

Gastrin

Gastroinhibiting of peptide

Cholecystokinin

Somatostatin

In a patient was disordered digestion of proteins in the intestine. Deficit of what enzymes due to that

process with?

Peptidase

Lipase

Synthetase
Amylase

Transferase

Ina patient with acute pancreatitis was revealed sharply increased trypsin activity in blood and

sharp decrease of arterial pressure — 80/60 mm Hg. What may explain influence of trypsin onto

arterial pressure with?

Activation of kinin synthesis in blood

Inhibition of kinin synthesis in blood

Activation of angiotensin Il synthesis

Inhibition of angiotensin II synthesis

Activation of aldosterone synthesis

Ina patient with hepatic jaundice was revealed disorder of bilirubin conjugation with glucuronic

acid due to deficit UDP-glucuroniltransferase, which catalysed this process. What hereditary

disease characterized that disorder for?

Crigler-Najjar’s syndrome

Gilbert’s syndrome

Dubin-Johnson’s syndrome

Rotor’s syndrome

Down’s syndrome

Ina patient with stable hypoglycemia analysis of blood after injection of adrenalin essentialy non

changed. Physician supposed about liver function disorder. About changes of what liver function

may be question?

Excretory

Cholesterol formating

Deintoxicatic

Ketogenic

Glycogensynthesis

In a woman 45-year-old was revealed in gallbloodder biliary stones as manifestation of dyscholia in

a result of excess take of fat food. By main mechanism of arising of lithogenic bile is in that case?
Decrease of biliary acids and lecithin to bile cholesterol relation

Inhibition of cholic acid synthesis in liver

Acceleration of biliary acid absorption by gallbladder mucous

Decrease of ?-hydroxilase activity

Decrease of hepato-intensinal circulation of cholic acids

In blood of a patient was revealed low level of albumin and fibrinogen. Increase activity of what

hepatocyte organelle stipulate this phenomenon?

Granular endoplasmatic reticulum

Mitochondrium

Agranular endoplasmatic reticulum

Lysosome

Golgi complex

In examination ofa patient with stomach ulcer was determined gastric secretion rise maximum for 5

minutes after eat and fall down it arise through Lhour 25 minutes. What type of gastric secretion in

that patient?

Normal

Braking

Asthenic

Inert

Excitable

To a patient with hypersecretion of gastric juice physician was recomended to exclude from diet

concentrated broth and vegetable decoctions because they stimulate gastric secretion. What is

dominating mechanism of gastric secretion stimulation in this patient?

Stimulation of gastrin output by G-cells

Irritation of gastric mechanoreceptors

Irritation of taste receptors

Irritation of gastric chemoreceptors

Stimulation of secretin output in duodenum


A 23-year-old patient who had frequent strep infections of the throat complained of periodic

headache, rapid tiredness, periorbital edema. Arterial hypertension, proteinuria, hypoproteinemia,

hyperlipidemia were revealed. What disease was most likely in this case?

Chronic glomerulonephritis

Acute pyelonephritis

Chronic pyelonephritis

Lipoid nephrosis

Diabetic nephropathy

A 23-year-old woman has complaints of flank pain, loos of appetite, increase of temperature to

39°C. Positive symptom Pastepnatsky was found. There is leukocytosis , ESR is raised. In the urine:

albumen 0,9 g/I, leucocytes 250-300 in area of view, red blood cells 8-10 in area of view, mucus

+++, bacteria +++. What disease has the patient?

Acute pyelonephritis

Acute glomerulonephritis

Chronic glomerulonephritis

Nephrolithiasis

Acute renal failure

A 24-year-old pregnant woman has frequent exsessive vomiting. What disorder of acid-basic

balance can be in her?

Respiratory acidosis

Not respiratory acidosis

Respiratory alkalosis

Metabolic acidosis

Metabolic alkalosis

A 35-year-old patient after cranial trauma complains on permanent thirst, increased urine output (to

10 I/per day). Specific gravity of urine is 1010, pathological components are absent. What is the

cause of these symptoms in the patient?

Antidiuretic hormone deficiency


Increased antidiuretic hormone production

Decrease of insulin synthesis

Hyperproduction of aldosterone

Decrease of aldosterone synthesis

A 42-year-old man has chronic glomerulonephritis. Hypertension, edema, vomiting, diarrhea, itch

of skin, arthritis are observed. Contents of residual nitrogen of blood - 43 mmol/l. The reason of

these signs is

Reduced renal excretion

Increased glomerular filter permeability

Ischemia of kidneys

Disorder of reabsorption of bicarbonates

Disorder of concentrating mechanism

A 45-year-old patient with hypovolemic shock has acute renal failure. The main mechanism of this

disorder is

Decrease of blood flow to the kidneys

Increase of reabsorption of sodium

Violation of urine passage

Obstruction of urine outflow

Increase of reabsorption of water

A 50-year-old patient with chronic pyelonephritis has arterial hypertension. What is the reason of

this symptom?

Increased formation of kidney prostaglandins

Activation of central cholinergic mechanism

Decreased partial pressure of oxygen in kidney

Activation of angiotensinase synthesis in kidney

Activation of acidogenesis and ammoniagenesis in kidneys

A 75-year-old male has anuria. The arterial pressure is 55/20 mm Hg. What kidney function is

altered?
Glomerular filtration

Obligate reabsorption

Facultative reabsorption

Tubular secretion

All enumerated processes

A patient has arterial hypertension for 10 years. After using of furosemyd loss of appetite, general

weakness, decrease of blood pressure and intestine peristaltic appeared. The cause of such changes

is

Hyponatremia

Hyperuricemia

Hypercalcemia

Hyperkalemia

Hypokalemia

A patient has diabetes mellitus for 25 years. This disorder led to impairment of renal function.

Glomerular filtration rate is 9 ml/min. What is main mechanism of deminished glomerular filtration

rate?

Gradual tissue destruction

Decrease of systemic arterial pressure

Occlusion of tubules

Development of acidosis in tissues

Spasm of afferent glomerular arteriole

A patient has isoosmolar dehydration as a result of prolonged diarrhea. What complication may

develop, if it to compensate by hypotensive solution?

Swelling of cells

Dehydration of cells

Edema

Ascites

Arterial hypertension
A patient has unlateral loin pain, fever, malaise, anorexia.Full blood count shows elevated white

cell count with neutrophilia. Microscopy of urine shows pyuria, proteinuria. What is disease in this

patient?

Acute pyelonephritis

Acute glomerulonephritis

Chronic glomerulonephritis

Nephrolithiasis

Chronic kidney insufficiency

A patient with polycystic kidney disease has chronic renal failure. Glomerular filtration rate is

reduced to 15 % from normal. What is the main cause of the decline of glomerular filtration in this

case?

Decreased amounts of functioning nephrons

Tubulopathy

Obstruction of urinary tract

Ischemia of kidneys

Thrombosis of kidney arteries

A 31-year-old woman, who has two healthy children, notes that she has had no menstrual periods

for the past 6 months, but she is not pregnant and takes no medications. Within the past week, she

has noted some milk production from her breasts. She has been bothered by headaches for the past 3

months. After nearly hitting a bus while changing lanes driving her vehicle, she is concerned with

her vision. An optometrist finds her lateral vision to be reduced. On physical examination she is

afebrile and normotensive. Which of the following laboratory test findings is most likely to be

present in this woman?

Hyperprolactinemia

Lack of growth hormone suppression

Increased serum alkaline phosphatase

Increased serum cortisol

Hyponatremia
A 28-year-old woman has had difficulty concentrating at work for the past month. She is constantly

getting up and walking around to visit co-workers. She complains that the work area is too hot. She

seems nervous and often spills her coffee. She has been eating more but has lost 5 kg in the past 2

months. On physical examination her temperature is 37.5°C, pulse 101/minute, respiratory rate

22/minute, and blood pressure 145/85 mm Hg. Which of the following laboratory findings is most

likely to be present in this woman?

Decreased iodine uptake

Decreased plasma insulin

Increased calcitonin

Increased ACTH

Decreased TSH

A 19-year-old previously healthy woman has had a mild pharyngitis followed by a high fever over

the past 24 hours. When seen in the emergency room, her skin now shows extensive areas of

purpura. Vital signs include temperature 39°C, pulse rate 102/minute, respiratory rate 21/minute,

and blood pressure 80/55 mm Hg. Laboratory studies show a serum sodium of 115 mmol/L,

potassium 5.3 mmol/L, chloride 92 mmol/L, CO2 22 mmol/L, glucose 42 mg/dL, and creatinine 1.1

mg/dL. Which of the following is the most likely diagnosis?

Meningococcemia

Disseminated tuberculosis

Reactive systemic amyloidosis

Sheehan syndrome

Idiopathic adrenalitis

A 37-year-old man experiences abdominal pain, nausea, and constipation for the past 3 days. On

physical examination he has no palpable abdominal masses and bowel sounds are present. His lungs

are clear to auscultation. He has a heart rate of 80/min with an irregular rhythm. An

electrocardiogram demonstrates a shortened QT(corrected) interval and a prolonged PR interval. He

has a stool positive for occult blood. Upper GI endoscopy reveals multiple 1 cm diameter shallow

ulcerations of the gastric antrum. Which of the following laboratory test findings is most likely to
be present in this man?

Thyroid peroxidase antibody of4 IU/mL

Blood glucose of 225 mg/dL

Total serum thyroxine of 21 ng/mL

Plasma cortisol of 45 microgm/dL at 8 am

Serum calcium of 12.4 mg/dL

A 58-year-old man with a history of diabetes mellitus has noted the presence of bone pain,

especially of his hands, for the past 6 months. On physical examination there is no swelling or

redness of his hands, no joint deformity, but the range of motion is slightly decreased. Laboratory

studies show sodium 139 mmol/L, potassium 4.0 mmol/L, chloride 98 mmol/L, CO2 22 mmol/L,

glucose 153 mg/dL, creatinine 7.8 mg/dL, calcium 7.8 mg/dL, phosphorus 5.7 mg/dL, total protein

6.2 g/dL, and albumin 4.0 g/dL. Which of the following conditions is this man most likely to have?

Adrenal adenoma

Medullary thyroid carcinoma

Extra-adrenal pheochromocytoma

Parathyroid hyperplasia

Pituitary adenoma

A 49-year-old woman has had increasing cold intolerance, weight gain of4 kg, and sluggishness

over the past two years. A physical examination reveals dry, coarse skin and alopecia of the scalp.

Her thyroid is not palpably enlarged. Her serum TSH is 11.7 mU/L with thyroxine of 2.1

micrograms/dL. A year ago, anti-thyroglobulin and anti-microsomal autoantibodies were detected at

high titer. Which of the following thyroid diseases is she most likely to have?

Hashimoto thyroiditis

Papillary carcinoma

DeQuervain disease

Multinodular goiter

Graves disease

A 48-year-old woman has experienced constant back pain exacerbated by movement over the past
month. She reports increasing weakness over the past 3 months. On physical examination her blood

pressure is 165/110 mm Hg. She is overweight, with a BMI of 28. A radiograph of the spine reveals

a compressed fracture at T10. Laboratory findings include a serum glucose of 155 mg/dL. Which of

the following pathologic lesions is most likely to explain her findings?

Adrenal cortical carcinoma

Anaplastic thyroid carcinoma

Empty sella syndrome

Pheochromocytoma

Multinodular goiter

An 35-year-old woman has had insomnia for the past 4 months, as well as episodes of diarrhea with

up to 4 loose stools per day. On physical examination, she exhibits bilateral proptosis. Her

outstretched hands have a fine tremor. On palpation of her neck, the thyroid gland does not appear

to be enlarged and no masses are palpable. Laboratory studies show a serum TSH of 8.8

microU/mL in association with a serum total thyroxine of 15.1 microgram/dL. Which of the

following is the most likely diagnosis?

Pituitary adenoma

Graves disease

Chronic thyroiditis

Prior thyroidectomy

Multinodular goiter

A 50-year-old man has episodic headaches for 3 months. On physical examination his blood

pressure is 185/110 mm Hg, with no other remarkable findings. Laboratory studies show sodium

145 mmol/L, potassium 4.3 mmol/L, chloride 103 mmol/L, CO2 26 mmol/L, glucose 91 mg/dL, and

creatinine 1.3 mg/dL. An abdominal CT scan shows a 7 cm left adrenal mass. During surgery, as

the left adrenal gland is removed, there a marked rise in blood pressure. Which of the following

laboratory test findings most likely explains his findings?

Increased urinary free catecholamines

Decreased urinary homovanillic acid


Increased serum ACTH

Decreased serum cortisol

Elevated serum ANCA

A 56-year-old woman has had diffuse, dull, constant abdominal pain for the past 2 months. On

physical examination no abnormal findings are noted. An abdominal CT scan shows a 3 cm right

adrenal mass. The right adrenal is excised and on microscopic examination the mass is composed of

cells resembling adrenal cortex. Which of the following features is the most reliable indicator that

this mass is malignant?

Invasion

Presence of mitoses

Cellular atypia

Size of the mass

Cellular necrosis

A 33-year-old previously healthy man has lateral visual field deficits, but his residual vision is

20/20. His facial features have changed over the past year. His shoe size has increased. A head CT

scan reveals enlargement of the sella turcica. Which of the following hormones is most likely being

secreted in excessive amounts in this man?

Growth hormone

Prolactin

ACTH

Antidiuretic hormone

Luteinizing hormone

A 45-year-old man has a 4 month history of nonfocal, generalized headaches. On physical

examination {=he is found to have a blood pressure of 170/110 mm Hg. Laboratory studies show a

serum sodium of 146 mmol/L, potassium 2.3 mmol/L, chloride 103 mmol/L, CO2 27 mmol/L,

glucose 82 mg/dL, and creatinine 1.2 mg/dL. His plasma renin activity is 0.1 ng/mL/hr and his

serum aldosterone 65 ng/mL. Which of the following is the most likely cause for his findings?

21-hydroxylase enzyme deficiency


Pituitary adenoma

Exogenous corticosteroid administration

Renal cell carcinoma

Adrenal cortical adenoma

A 45-year-old woman with severe rheumatoid arthritis has been on chronic corticosteroid therapy

for the past 15 years. She is admitted for an orthopedic procedure to correct joint deformity from

her disease. She continues to receive her regular dosage of5 mg of prednisone per day. Three days

postoperatively, she develops an aspiration pneumonia with Klebsiella pneumoniae cultured from

sputum. Five days following the surgery, she becomes obtunded. Laboratory findings at that time

include: sodium 105 mmol/L, potassium 5.4 mmol/L, chloride 87 mmol/L, CO2 16 mmol/L,

glucose 40 mg/dL, and creatinine 1.1 mg/dL. Which of the following complications is most likely to

have occurred in this patient?

Waterhouse-Friderichsen syndrome

Anterior pituitary necrosis

Conn syndrome

21-hydroxylase deficiency

Acute Addisonian crisis

A 41-year-old man has been drinking large quantities of water--up to 20 liters per day--for the past

week. On physical examination he has diminished skin turgor and dry mucous membranes.

Laboratory studies show sodium 162 mmol/L, potassium 4.1 mmol/L, chloride 121 mmol/L, and

bicarbonate 27 mmol/L. His serum glucose is 75 mg/dL and creatinine 1.0 mg/dL. His serum

osmolality is 343 mOsm/kg. A deficiency of which of the following hormones is most likely present

in this man?

Vasopressin

Oxytocin

Insulin

Growth hormone

Prolactin
A 50-year-old man has a lump on his neck found at physical examination. A fine needle aspirate of

this lump shows a follicular neoplasm of the thyroid. He undergoes a total thyroidectomy. Within a

day following surgery, he is noted to have tingling sensations and neuromuscular irritability. Which

of the following serum laboratory tests should be ordered immediately to determine further therapy

for this man?

Calcium

Parathormone

Thyroxine

TSH

Calcitonin

A 57-year-old man is found comatose. On physical examination he has decreased skin turgor.

Laboratory studies show a blood glucose of 780 mg/dL. Urinalysis reveals no ketosis or proteinuria,

though there is 4+ glucosuria. Which of the following is the most likely diagnosis?

Type | diabetes mellitus

Cushing syndrome

Ingestion of a large quantity of sugar

Neuroendocrine tumor secreting glucagon

Type II diabetes mellitus

Acromegaly is most frequently diagnosed in:

Middle-aged adults

Newborns

Children ages 2 to5

Adults age 65 and older

A patient presents with weight loss, sweating, palpitations, tachycardia, tremors, lid lag,

exopthalmosis, and a goiter. Upon taking a thyroid scan, there appears to be an increased amount of

iodine. What is the pathogenesis of his disease?

Autoimmune mediated

Immune complex mediated


Unknown

Infection

None of the above

A patient presents with lethargy, tiredness, cold intolerance, dryness of skin and hair, hoarsness of

voice and weight gain. He also has psychosis and his heart rate appears to be lower than normal.

Which of the following is not associated with the presenting symptoms?

Cretinism

Protruding tongue, potbelly appearance, dwarflike features

lodine deficiency

Increased iodine levels

Hashimoto's disease

A girl presents with delayed puberty, absent secondary sexual characteristics, and primary

amenorrhea. She also appears to be hypertensive and hypokalemia. Which of the following

enzymes is increased if there is virilization of a person?

11bHydroxylase

17ahydroxylase

21 b-hydroxylase

Pyruvate kinase

Alcohol degidrohenase

A baby with pupillar dilation, visual disturbances, difficulty breathing, and enlarged head is

diagnosed with Dandy-Walker syndrome. What is underlying possible mechanism of his disease?

BandD

Problem with arachnoid granulations

Tumor

Alzheimers

Hemorrhage/inflammations}

The most common cause of intracerebral hemorrhage is associated with

BandCc
Tumors

Cerebral amyloid angiopathy

High cholesterol

Liver failure}

A patient presents with a bump on his neck. Upon taking a tissue sample, there appears to be

parafollicular cells. The tumor is also associated with MEN II, although most of the time it is

sporadic. What is the best way to moniter treatment?

Measure calcitonin levels

Measure iodine levels

Measure TSH levels

Measure thyroid hormone levels

Measure iron levels

A patient with Cushing syndrome might present with any of the following EXCEPT:

Bronze or hyperpigmented skin

Obesity

Buffalo hump

Moon facies

Glucose intolerance

Patients with diabetes have an increased risk of all of the following EXCEPT:

Infections

Increased atherosclerosis

Peripheral neuropathy

Cataracts

Pancreatic carcinoma

Which of the following is true regarding multiple endocrine neoplasia (MEN) | syndrome? .

Many patients develop parathyroid hyperplasia

The genetic abnormality involves the RET gene


The genetic abnormality involves a proto-oncogene

Virtually all patients develop medullary thyroid carcinoma

Some patients may have a Marfanoid habitus

Which of the following is true regarding pheochromocytoma?

It is usually an aggressive, malignant tumor

It only occurs in the adrenal gland

It is derived from neural crest cells

Many tumors have a 1p deletion

It can cause hypertension

Which of the following is true regarding Addison’s disease?

It is also called primary chronic adrenal insufficiency

Most cases are due to infection

It is characterized by an overproduction of catecholamines

The onset is sudden

Patients are often hypertensive actin level.

Which sysndrom belongs to hypothalamical?

Hyperhonadotropic hypogonadism

Syndrom of persistent lactation

Cushing syndrom

Frelich syndrom

DIC syndrom

What can be provoked with Shuchan syndrom?

Afterdeliverical hypochinadism

Meningitis

Pernicious anemia

Hemolisis of erythrocytes

DIC syndrom

Poin the state which leads to blood pressure changes in Addison disease
Hypovolemia

Increase of vessels tonus

Hypervolemia

Tachicardia

Decrease of n.vagus tonus

Which clinical state is not caracterised for Kone syndrom?

Hypervolemia

Decrease of n.vagus tonus

Decrease of muscular tonus

Thirst and poliuria

Tachicardia

In which type of tissures break down of protein is intensified during Cushing syndrome?

Muscular tissures

Adrenal glands

Skin

Limphoid tissure

Bones

What is the reason of peripheral resistancy to hormons?

Disorders of realising hormons syntesis

Atrophy of target cells

Low concentration of hormon un bloodstream

Hereditary deficit of cellular programs

Block of receptors with autoantibodies

Which mechanism of lipid metabolisms takes place in cushing disease?

Decrease of fat mobilisation from depot under action of glucocorticoids

Low rateof fat synthesis

Intensification of lipolysis

Intensification of gluconeogenesis
Intensification of fat mobilisation from depot under action of catecholamines

Poin the basic change in Kone syndrom

Hypertension

Disorders of heart functions

Megacolon

Lethargy

Hypovolemia

Which syndrom is characterised for hypopituitarism?

Alimentary obisity

Lactorrea-amenorrea

Chican syndrom

Frelich syndrome

Characteristic of Simmonds disease:

Coused with adenohypophysis destruction

Primary hypocorticism is manifestation

Coused due to hypotalamus destruction

Clinically manifests like ACT deficiency

Clinically manifests like deficiency of somatotropin

What is the manifestation of primary hyperaldosteronism?

Increasing of vasoconstrictive effect of catecholamines

Arterial hypotension

Muscular hypertonus

Alopecia

Poliphagia

Point what is not characterised for total adrenal insufficiency?

Arterial hypotension

Hyponatriemia
Hyperkaliemia

Increase of mucular tonus

Gigantism and acromegaly are due to

Hyperpituitarism

Hyperthyroidism

Hypopituitarism

Hypothyroidism

What is Sheeshan’s syndrome?

The pituitary adenoma is hemorrhagic and necrotic

Pituitary gland is infected with virus

Pituitary gland is infected with Bacteria

It is the malignancy of the pituitary

Uniform painless enlargement of the thyroid gland is noted in:

long standing goiter

Riedel’s thyroiditis

Subacute granulomatous thyroiditis

Grave’s disease

A 50 -year -old male presented with a history of recurrent gastric ulcers and multiple renal stones.

The most likely histologic finding is:

Adrenal cortical adenoma

Pituitary adenoma

Pheochromocytom

Thyroid adenoma

Parathyroid adenoma

A parathyroid adenoma may cause all of the following except:

Tetanus

Renal calculi

Elevated levels of serum calcium


Osteomalacia

Aman 52, years old, complains on a general weakness and obesity. Objectively: height 168 cm,

weight 111 kg, obesity with extreme deposits of fat on the face, neck, stomach. BP — 195/100

mmHg, level of glucose in blood 8,9 mmol/I. What function of what endocrine gland is changed?

Hyperfunction of neurohypophysis

Hyperfunction of thyroid gland

Hypofunction of adrenal cortex

Hypofunction of pancreas

A man, 58 years old (growth 170 sm, mass 103 kg), has an obesity, filling out person, apathy,

hypothermia, bradycardia. The dysfunction of what gland leads to that state of the patient?

Thyroid gland

Parathyroid gland

Adrenal gland

Pancreas

Hypophysis

A patient complains on loss of weight, agitation, fatigue, muscular weakness, skin

hyperpigmentation, hypotension. It is known from anamnesis, that patient suffers on pulmonary

tuberculosis and is addicted to alcohol for a long time. Name the most possible reason of disease at

this patient.

Tubercular impairment of adrenal glands

Alcoholic illness of liver

Tumour of adenohypophysis

Impairment of parathyroid glands

Chronic intoxication by alcohol}

A patient complains on polydypsia and poliuria (8 liters per day). Objectively no changes in

carbohydrates metabolism were found. What endocrine gland dysfunction is a cause of such

changes?

Neurohypophysis
Adenohypophysis

Pancreas

Adrenal cortex

Thyroid gland

A patient complains on rapid fatigue, dyspepsia. It is discovered at the inspection: BP — 95/65

mmkg, glucopenia, hyponatremia, skin hyperpigmentation. What disease are these symptoms

typical for?

Addison disease

Syndrome of Itsenko-Cushing

Conn’s Syndrome

Acute insufficiency of adrenal cortex

Pheochromocytoma

A patient for two months used glucocorticoids occasionally for the treatment of joint syndrome. He

abolished abruptly the reception of preparation by his own decision. He complains on dizziness,

decline of arterial pressure, general weakness, apathy, nausea, vomiting and hypothermia. How can

you explain these symptoms?

Development of insufficiency of adrenal glands

Resistance to preparation

Development of hepatic insufficiency

Development of kidney insufficiency

After sprinting untrained people feel muscular pain as a result of lactate accumulation. | may be
connected with intensification of the following biochemical process:

{=Glycolysis

~Glyconeogenesis

~Pentose-phosphate cycle

~Lipogenesis

~Glycogenesis}
What vitamin deficiency leads to both disorder of reproductive function and distrophy of skeletal
muscles?

{=Vitamin E

“Vitamin A

“Vitamin K

“Vitamin D

“Vitamin Bi}

Toxic pulmonary edema was reproduced on a laboratory rat by means of ammonium chloride solution.
What is the leading pathogenetic factor of this edema?

{=Increased permeability of capillars

~Increase of venous outflow

~Decrease of colloid osmotic pressure

~Disorder of neural and humoral regulation

~Increase of lymph outflow}

A patient has disorder of airways patency at the level of small and middle bronchs. What changes of
acid-base balance may take place?

{=Respiratory acidosis

“Respiratory akcalosis

~Metabolic acidosis

~Metabolic alkalosis

~Acid-base balance won't change}

The body temperature of a patient with an infectious disease rises once in two days up to 39,540,5°C
and stays so for about an hour and then drops to the initial level. What type of fever is it? {=Intermittent

~Continued

~Remittent
~Hectic

~Atypical}

Climbing the mountains at a height of 5000 m climbers started complaining of breath shortness,
palpitation, vertigo, ring in the ears. What pathogenetic factor determines the development of these
occurances? {=Hypoxemia
~Hypokalemia

~Decreased oxygen capacity of blood

~Lactacidemia

~Hypernatremia}

A 12 year old boy came home from school and started complaining of headache, sickness, chill,
periodical muscle pain, appetite loss, flabbiness. What period of illness are these symptoms typical for?

{=Prodroma

~Latent

~Incubative

~High point of illness


~End of illness}

After a long-lasting and grave illness the blood pressure ofa patient fell up to 60/40 mm; he has
tachicardia, dyspnea, black-out. How can this state be defined?

{=Preagony

~Agony

~Shock

~Apparent death
~-}

A newborn child ill with pylorostenosis has frequent vomiting accompanied by apathy, weakness,
muscular hypertonia, sometimes convulsions. What form of acid-base balance disorder has developed?

{=Nongaseous alkalosis

~Gaseous alkalosis

~Gaseous acidosis

~Metabolic acidosis

~Excretory acidosis}

After recovering from epidemic parotiditis a patient began to lose weight, he was permanently thirsty,
drank a lot of water, had frequent urination, voracious appetite. Now he has complaints of skin itch,
weakness, furunculosis. His blood contains: glucose - 16 mmole/L, ketone bodies - 100 mcmole/L;
glucosuria. What disease has developed?

{=Insulin-dependent diabetes
~Insulin-independent diabetes

~Steroid diabetes

~Diabetes insipidus

~Malnutrition diabetes}

A patient dropped into an ice hole, froze in the wind and fell ill. Body temperature rose up to 39,7°C and
varied from 39,0°C to 39,8°C. Name the type of the patient's temperature profile? {=Febris continua

~Febris recurrens

~Febris hectica

~Febris intermittens
~Febris remittens}

A diver who has been staying at the depth of 40 m for a long time fell ill with caisson disease as a result
of decompression. The main pathogenetic factor is the following embolism:

{=Gaseous

~Air

~Fat

~Paradoxical
~Tissue}

A 65 year old patient suddenly died. She suffered from thrombophlebitis of deep veins of shin. Autopsy
revealed: trunk and bifurcation of pulmonary artery contain red loose masses with dull corrugated
surface. What pathological process did the morbid anatomist reveal in pulmonary artery?
{=Thromboembolism

~Thrombosis

~Tissue embolism

~Foreign body embolism

~Fat embolism}

Skin samples ofa patient with bronchial asthma revealed allergen sensitization of poplar fuzz. What
factor of immune system plays the main part in development of this immunopathological state? {=lgE

~IgD

~IgM

~Sensitized T-lymphocytes
~-}

A patient with insulin-dependent diabetes had an insulin injection. Some time later he felt weakness,
irritability, excessive sweating. What is the main reason of these disorders?

{=Carbohydrate starvation of brain

~Intensified glycogenolysis

~Intensified ketogenesis

~Intensified lypogenesis

~Reduced glyconeogenesis}

For the purpose of disinfection of nonmetallic surgical instruments the formaldehyde solution was used.
What group does this anticeptic preparation belong to according to its chemical structure? {=Aliphatics

~Aromatics

~Alcohols

~Halogenated compounds

~Detergents}

A patient with inflammation of trigeminal nerve has been having progressive paradontitis for some
years. What factor is the most important for parodontitis development?

{=Neurodistrophic disorders

~Low activity of leukocytic elastase

~Poor formation of immunoglobulins

~Increased tone of vagus nerve

~Low activity of kallikrein-kinin system}

Potassium cyanide that is a poison came into a patient's organism and caused death a few minutes
after it. The most probable cause of its toxic effect was abnormal activity of: {=Cytochrome oxidase

~Catalase

~ATP-synthetase

~NADP-H-dehydrogenase

~Haemoglobin synthesis}
Hydrocyanic acid and cyanides are the most violent poisons. According to the dose the death
follows after a few seconds or minutes. The death is caused by the inhibited activity of the
following enzyme: {=Cytochrome oxidase

~Acetylcholinesterase

~ATP-synthetase

~Catalase

~Methemoglobin reductase}

A newborn has signs of dyspepsia after milk feeding. Symptoms of dyspepsia disappear when milk is
substituted for glucose solution. The newborn has low activity of the following enzyme: {=Lactase

~Invertase

~Maltase

~Amylase

~lsomaltase}

A married couple applied to the genetic consultation in order to consult about their child with multiple
abnormalities (microcephaly, idiocy etc). The woman has had an illnesses during her pregnancy but she
didnt take any teratogens or mutagens. The parents and the childs karyotype is normal. Anamnesis
study revealed that the family kept a cat. What gravidic disease caused the childs abnormalities?

{=Toxoplasmosis

~Dysentery

~Trichomoniasis
~Balantidiasis

~Leishmaniasis}

As a result of improper feeding an infant got full-blown diarrhea. One of its main consequences is
excretion of large amount of sodium bicarbonate. What form of acid-base balance disturbance is it?
{=Excretory acidosis

~Metabolic alkalosis

“Respiratory alkalosis

~Acid-base balance wont be disturbed

“Respiratory acidosis}

After sprinting untrained people feel muscular pain as a result of lactate accumulation. | may be
connected with intensification of the following biochemical process:
{=Glycolysis

~Lipogenesis

~Glycogenesis

~Glyconeogenesis

~Pentose-phosphate cycle}

What vitamin deficiency leads to both disorder of reproductive function and distrophy of skeletal
muscles? {=Vitamin E

“Vitamin K

“Vitamin B1
“Vitamin D

“Vitamin A}

Toxic pulmonary edema was reproduced on a laboratory rat by means of ammonium chloride solution.
What is the leading pathogenetic factor of this edema?

{=Increased permeability of capillars

~Disorder of neural and humoral regulation

~Increase of lymph outflow

~Increase of venous outflow

~Decrease of colloid osmotic pressure}

A pregnant women developed severe toxemia with exhausting recurrent vomiting throughout a day. By
the end of the day she developed tetanic convulsions and bodily dehydration. The described changes
were caused by the following type of acid-base disbalance:

{=Nongaseous excretory alkalosis

~Gaseous acidosis

~Nongaseous excretory acidosis

~Nongaseous metabolic acidosis


~Gaseous alkalosis}

After a diver had dived to a depth of 60 meters he got the following symptoms of CNS dysfunction:
anxiety, euphoria, lack of attention, professional errors. These symptoms are associated with neurons
being under a toxic effect of:

{=Nitrogen
~Ammonia

~Lactate

~Oxygen

~Carbon dioxide}

A patient has disorder of airways patency at the level of small and middle bronchs. What changes of
acid-base balance may take place?

{=Respiratory acidosis

~Metabolic alkalosis

~Acid-base balance wont change

“Respiratory alkalosis

~Metabolic acidosis}

The body temperature of a patient with an infectious disease rises once in two days up to 39,540,50C
and stays so for about an hour and then drops to the initial level. What type of fever is it? {=Intermittent

~Hectic

~Atypical

~Continued

~Remittent}

A patient with periodontitis has developed gingival edema. The gums are of dark red colour. What local
circulation disorder prevails in the gums of the patient?

{=Venous hyperemia

~Thrombosis

~Embolism

~Arterial hyperemia

~Ischemia}

To prevent the transplant rejection after organ transplantation it is required to administer


hormonotherapy for the purpose of immunosuppression. What hormones are used for this purpose?
{=Glucocorticoids

~Mineralocorticoids

~Catecholamines
~Thyroid

~Sexual hormones}

Climbing the mountains at a height of 5000 m climbers started complaining of breath shortness,
palpitation, vertigo, ring in the ears. What pathogenetic factor determines the development of these
occurances? {= Hypoxemia

~Hypokalemia
~Lactacidemia

~Hypernatremia

~Decreased oxygen capacity of blood}

A 45 year old patient was taken to the hospital by an emergency team with serious cranial trauma in
shock condition. Objectively: unconscious, skin is pale, body temperature- 35°C, low muscular tonus,
reflexes are absent, pulse is rapid and weak, AP- 50/30 mm Hg. What clinical shock stage is it?

{=Terminal stage

~Erectile stage

~Inhibition stage

~Torpid stage

~Excitement stage}

A teenger had his tooth extracted under novocain anaesthesia. 10 minutes later he presented with skin
pallor, dyspnea, hypotension. When this reaction is developed and the allergen achieves tissue
basophils, it reacts with:

{=IgE

~IgA

~IgM

~T-lymphocytes

~IgD}

A 12 year old boy came home from school and started complaining of headache, sickness, chill,
periodical muscle pain, appetite loss, flabbiness. What period of illness are these symptoms typical for?

{=Prodromal

~Latent

~High point of illness


~End of illness

~Incubative}

Examination of experimental rats that have been getting only carbohydrate feed for a long time
revealed accumulation of water in tissues. What is the leading pathogenetic mechanism of edema
development? {=Hypooncotic

~Hyperosmolar

~Dysregulatory

~Membranogenic

~Lymphogenous}

A man who took part in disaster-management at a nuclear power plant had hemorrhagic syndrome at
the same time with acute radiation sickness. What is the most important thing for the pathogenesis of
this syndrome?

{=Thrombocytopenia

~High activity of fibrinolysis factors

~Destructed structure of vessel walls

~High activity of anticoagulative blood system

~Low activity of anticoagulative blood system}

After a long-lasting and grave illness the blood pressure ofa patient fell up to 60/40 mm; he has
tachicardia, dyspnea, black-out. How can this state be defined?

{=Preagony

~Shock

~Apparent death

~Agony}

A newborn child ill with pylorostenosis has frequent vomiting accompanied by apathy, weakness,
muscular hypertonia, sometimes convulsions. What form of acid-base balance disorder has developed?

{=Nongaseous alkalosis

~Gaseous acidosis

~Gaseous alkalosis
~Metabolic acidosis

~Excretory acidosis}

A 10-year-old child cut his leg with a piece of glass and was sent to a clinic for an anti-tetanus serum
injection. In order to prevent the development of anaphylactic shock the Besredka desensitisation
method was applied. What mechanism underlies this method?

{=Binding to IgE fixed to mast cells

~Binding of IgE receptors on mast cells

~Stimulation of the immunological antigen tolerance

~Inhibited synthesis of mast cells mediators

~Stimulation of antigen-specific |IgG2 synthesis}

After recovering from epidemic parotiditis a patient began to lose weight, he was permanently thirsty,
drank a lot of water, had frequent urination, voracious appetite. Now he has complaints of skin itch,
weakness, furunculosis. His blood contains: glucose - 16 mmole/L, ketone bodies - 100 mcmole/L;
glucosuria. What disease has developed?

{=Insulin-dependent diabetes

~Malnutrition diabetes

~Steroid diabetes

~Insulin-independent diabetes

~Diabetes insipidus}

A patient dropped into an ice hole, froze in the wind and fell ill. Body temperature rose up to 39,7°C and
varied from 39°C to 39,8°C. Name the type of the patients temperature profile? {=Febris continua

~Febris hectica

~Febris recurrens

~Febris intermittens
~Febris remittens}

A woman with intractable vomiting was admitted to the infectious disease ward. What changes of
water-salt metabolism are likely to be observed?

{=Hypo-osmolar dehydration

~Hyperosmolar dehydration

~lso-osmolar dehydration
~Hypo-osmolar hyperdehydration

~Hyper-osmolar hyperdehydration}

A 65 year old patient suddenly died. She suffered from thrombophlebitis of deep veins of shin. Autopsy
revealed: trunk and bifurcation of pulmonary artery contain red loose masses with dull corrugated
surface. What pathological process did the morbid anatomist reveal in pulmonary artery?
{=Thromboembolism

~Fat embolism

~Tissue embolism

~Thrombosis

~Foreign body embolism}

Skin samples ofa patient with bronchial asthma revealed allergen sensitization of poplar fuzz. What
factor of immune system plays the main part in development of this immunopathological state? {=IgE

~IgM

~IgD

~Sensitized T-lymphocytes

~-}

A patient with insulin-dependent diabetes had an insulin injection. Some time later he felt weakness,
irritability, excessive sweating. What is the main reason of these disorders?

{=Carbohydrate starvation of brain

~Reduced glyconeogenesis

~Intensified ketogenesis

~Intensified glycogenolysis ~Intensified


lypogenesis}

A patient with inflammation of trigeminal nerve has been having progressive paradontitis for some
years. What factor is the most important for parodontitis development?

{=Neurodistrophic disorders

~Low activity of kallikrein-kinin system

~Poor formation of immunoglobulins

~Low activity of leukocytic elastase

~Increased tone of vagus nerve}


During a prophylactic medical examination a 7-year-old boy was diagnosed with daltonism. His parents
are healthy and have normal colour vision, but his grandfather on his mother’s side has the same
abnormality. What is the type of the abnormality inheritance?

{=Recessive, sex-linked

~Autosomal dominant

~Semidominance

~Dominant, sex-linked

~Autosomal recessive}

12-year-old teenager has significantly put off weight within 3 months; glucose concentration rose up to
50 millimole/L. He fell into a coma. What is the main mechanism of its development? {=Hyperosmolar

~Ketonemic

~Hypoglycemic

~Lactacidemic

~Hypoxic}

A patient was stung by a bee. Examination results: his left hand is hot, pink and edematic, there is a big
blister on the spot of the sting. What is the leading mechanism of edema development?

{=Increased vascular permeability

~Vascular damage caused by the sting

~Reduction of osmotic pressure of tissue

~Reduction of oncotic pressure of tissue ~Reduced


blood filling of vessels}

surgeon used novocaine as an anaesthetic during surgical manipulations. 10 minutes after it the patient
became pale, he got dyspnea and hypotension. What type of allergic reaction is it? {=Anaphylactic

~Immune complex

~Cytotoxic

~Stimulating

~Cell-mediated}

Examination of a patient revealed glycosuria and hyperglycemia. He complains of dry mouth, itchy skin,
frequent urination, thirst. He has been diagnosed with diabetes mellitus. What is the cause of polyuria
in this patient?
{=Increased urine osmotic pressure

~Increased filtration pressure

~Decreased plasma oncotic pressure

~Decreased cardiac output

~Increased plasma oncotic pressure}

Hydrocyanic acid and cyanides are the most violent poisons. According to the dose the death
follows after a few seconds or minutes. The death is caused by the inhibited activity of the
following enzyme: {=Cytochrome oxidase

~ATP-synthetase

~Methemoglobin reductase

~Catalase

~Acetylcholinesterase}

A typical symptom of cholera is great loss of water and sodium ions. What mechanism underlies
development of diarrhea in this case?

{=Activation of adenylate cyclase of enterocytes

~Inhibition of vasopressin synthesis in hypothalamus

~Increased corticotropin synthesis

~Increased secretion of renin by the cells of renal arterioles

~Aldosterone oxidation in adrenal cortex}

A patient with enteritis accompanied with intense diarrhea has reduced quantity of water in the
extracellular space and increased quantity of water inside the cells as well as low blood osmolarity.
Name this disorder of water-electrolytic metabolism:

{=Hyposmolar hypohydration

~Osmolar hypohydration

~Hyperosmolar hyperhydration

~Hyposmolar hyperhydration

~Hyperosmolar hypohydration}

It is known that patients with diabetes mellitus are more subject to inflammative processes, they have
low regeneration and slower wound healing. What is the reason for this?

{=Decrease in protheosynthesis
~Accelerated gluconeogenesis

~Intensification of catabolism

~Decrease in lipolysis

~Increase in lipolysis}

A patient was admitted to the infectious diseases department. His skin was dry, with low turgor; he had
rice-water stool. The patient was diagnosed with cholera. This disease is ordinarily accompanied by the
folowing disorder of water-electrolytic balance:

{=lsoosmotic hypohydration

~Hypoosmotic hypohydration

~Hyperosmotic hyperhydration

~Hyperosmotic hypohydration ~Hyposmotic


hyperhydration}

Prophylactic examination of a patient revealed hyperglycemia, ketonuria, polyuria, glycosuria. What


form of acid-base balance disorder is the case?

{=Metabolic acidosis

~Nongaseous acidosis

~Gaseous acidosis

~Gaseous alkalosis

~Metabolic alkalosis}

As a result of a trauma a patient has developed traumatic shock that led to the following disorders: AP is
140/90 mm Hg, Ps is 120 bpm. The patient is fussy, talkative, pale. Such state relates to the following
shock phase:

{=Erectile
~Terminal

~Latent period

~Torpid
~-}

A patient consulted a dentis about a lesion of his oral mucosa. He was diagnosed with herpetic
stomatitis. Which of the following drugs will have an effect on etiotropic factor? {=Acyclovir

~Levamisole

~Furacilinum
~Dimedrol

~Paracetamol}

Examination of an 18-year-old girl revealed the following features: ovarian hypoplasia, broad shoulders,
narrow hips, shortening of the lower extremities, webbed neck. Mental development is normal. The
patient has been diagnosed with Turner’s syndrome. What chromosomal abnormality does this patient
have?

{=Monosomy X

~Trisomy X “Trisomy
13

“Trisomy 18

~Nullisomy X}

Mother and father are healthy. Mother underwent amniocentesis for fetal karyotyping. The fetal
karyotype turned out to be 45, XO. What syndrome can be expected in a newborn baby? {=Turner’s

~Edwards’

~Patau’s

~Cri du chat

~"Superwoman"}

Genealogical study of a family with hereditary enamel hypoplasia has revealed that the disease occurs in
every generation. In women, the anomaly occurs more frequently than in men. Male patients only pass
this trait to their daughters. What type of inheritance takes place in this case?

{=X-linked dominant

~Autosomal dominant

~Autosomal recessive

~Y-linked

~X-linked recessive}

A female patient with toxemia of pregnancy has hypersalivation resulting in a daily loss of 3-4 liters of
saliva. What disorder of water-salt metabolism occurs in such cases?

{=Hyperosmolar hypohydration

~Hypoosmolar hypohydration

~lsoosmolar hypohydration
~Hypokalemia

~Hyponatremia}

A 28-year-old liquidator of Chernobyl disaster consulted a doctor about marked weakness, skin
hemorrhages, diarrhea. Blood test results: ESR- 25 mm/h, RBC- 2, 4 - 1012/L, WBC - 2, 2 - 109/L,
platelets - 70 - 109/L. What stage of acute radiation sickness are these presentations typical for?
{=Manifest illness stage

~Prodromal stage

~Latent stage

~Recovery stage

~Outcome of the disease}

A 10-year-old child cut his leg with a piece of glass and was sent to a clinic for an anti-tetanus serum
injection. In order to prevent the development of anaphylactic shock, the Besredka desensitization
method was applied. What mechanism underlies this method?

{=Binding to IgE fixed to mast cells

~Inhibited synthesis of mast cells mediators

~Stimulation of the immunological antigen tolerance

~Stimulation of antigen-specific IgG2 synthesis

~Binding of IgE receptors on mast cells}

A 28-year-old patient had been diagnosed with multifragmental fracture of the right hip. On the third
day after the injury he began to complain of pain in the right side of chest, diffi- cult respiration. A day
later the patient died of progressive heart and respiratory failure. Histological study of pulmonary and
cerebral blood vessels revealed orange sudanophilic droplets that completely obstructed the vessels of
microvasculature. What complication caused the death of the patient?

{=Fat embolism

~Gas embolism

~Drug-induced embolism

~Microbial embolism

~Thromboembolism}

An animal sensitized with tuberculin had been administered tuberculin intraperitoneally. 24 hours later,
during laparotomy, the animal was found to have venous congestion and peritoneal edema.
Impression smears from the peritoneum contained a large number of lymphocytes and monocytes.
What pathological process was detected in the animal?

{=Allergic inflammation

~Serous inflammation

~Purulent inflammation

~Fibrinous inflammation ~Aseptic


inflammation}

As a result of treatment of viral RNA with nitrous acid, UCA triplet mutated to UGA triplet. What kind of
mutation occurred?

{=Transition

~Nucleotide deletion

~Missense

~Nucleotide insertion

~Inversion}

A 36-year-old female patient who has been limiting the number of foodstuffs in her diet for 3 months
presents with a decrease in body weight, deterioration of physical and mental health, face edemata.
These changes may be caused by the deficiency of the following nutrients: {=Proteins

“Vitamins

~Fats

~Carbohydrates

~Micronutrients}

During anesthesia of the oral mucosa a 37-year-old patient has had anaphylactic reaction

(widespread vasodilation, increased vascular permeability with liquid exiting the blood vessels and
penetrating in the tissues). What type of hypersensitivity reaction occurred in the patient? {=Type |
(anaphylactic)

~Type Il (antibody-dependent)

~Type Ill {immune complex)

~Type IV (cell cytotoxicity)

~Type V (granulomatosis)}
In patients with glycogenolysis, that is von Gierke’s disease, the conversion of glucose-6-phosphate into
glucose is inhibited, which is accompanied by the improper breakdown of glycogen in the liver. The
cause of this condition is the following enzyme deficiency:

{=Glucose-6-phosphatase

~Glycogen phosphorylase

~Glucose-6-phosphate dehydrogenase

~Phosphofructokinase

~Phosphoglucomutase}

After arriving in the polar region, researchers from Australia have complained of nervous disorders, loss
of appetite, aggravation of chronic diseases for 6 months. What process has been disrupted in extreme
conditions?

{=Adaptation

~Tolerance

~Tachyphylaxis

~Stress

~Reparation}

The surgically excised connective tissue of the deformed mitral valve gives a basophilic reaction when
stained with hematoxylin and eosin. When stained with toluidine blue, it turns purple (metachromasia).
What changes of the connective tissue can be detected by these reactions? {=-Mucoid edema

~Fibrinoid necrosis of connective tissue

~Connective tissue edema

~Petrification

~Hyalinosis}

If a trait is determined mostly by genetic factors, the percentage of concordance between the twins is
much higher in monozygotic twins than in dizygotic ones. What is the percentage of blood group
concordance in monozygotic twins?

{=100%
~75%
~50%
~25%
~O%}

During auscultation a 26-year-old patient was asked to breathe deep. After 10 breaths the patient lost
consciousness, which is associated with the development of the following condition: {=Respiratory
alkalosis

~Carbon dioxide acidosis

~Erythropenia

~Polycythemia

~Reduced oxygen capacity of blood}

A 31-year-old patient has deep caries of the fifth maxillary tooth on the right. Acute purulent
inflammation in the region of the tooth apex has developed. There is marked edema with isolated
neutrophils in the perifocal tissues and soft tissues of cheek and palate. Diagnose the pathologic process
in the soft tissues of cheek and palate:

{=Acute serous inflammation

~Acute suppurative periodontitis

~Acute suppurative periostitis

~Phlegmon

~-}

A 35-year-old patient complains of pain in the upper jaw, bleeding, a slight loosening of teeth. He has
been diagnosed with periodontitis. What is a typical pathological process in this case? {=Inflammation

~Bleeding

~Caries

~Pain

~Redness}

A female patient has scalded her hand with boiling water. The affected skin area became red, swollen
and painful. This effect is caused by accumulation of the following substance: {=Histamine

~Lysine

~Thiamine

~Glutamine

~Asparagine}
Mother had noticed her 5-year-old child’s urine to become dark in colour. Bile pigments in urine were
not detected. The diagnosis of alkaptonuria was made. What pigment is deficient?

{=Homogentisic acid oxidase

~Phenylalanine hydroxylase

~Tyrosinase

~Oxyphenylpyruvate oxidase

~Phenylpyruvate decarboxylase}

There are various diseases that cause sharp increase of active oxygen, leading to cell membranes
destruction. Antioxidants are used to prevent it from happening. The most potent natural antioxidant is:

{=Alpha-tocopherol

~Glucose

“Vitamin D

~Fatty acids

~Glycero}

A patient with liver cirrhosis has been given intravenously 500,0 ml of 5% glucose solution along with
other drugs. There is a high risk of the following water-electrolytic balance disruption:

{=Hyposmolar hypohydration

~Hyperosmolar hypohydration

~lso-osmolar hypohydration

~Hyposmolar dehydration

~Hyperosmolar dehydration}

A patient has oliguria caused by acute renal failure. What daily amount of urine corresponds with this
symptom? A. 100-500 ml B. 1500-2000 ml C. 1000-1500 ml D. 500-1000 ml E. 50-100 ml 42. A 7-year-old
child suffers from acute disease. A pediatrician observed the following during examination: pharynx
mucosa is hyperemic, edematous, swollen, cowered with large amount of mucus. Buccal mucosa has
whitish discoloured spots. On the next day the child came out in rash with large spots covering the skin
of his face, neck and torso. What kind of inflammation causes changes in pharynx mucosa?

{=Catarrhal

~Serous

~Serofibrinous
~Fibrinous

~Hemorrhagic}

A 28-year-old patient had been diagnosed with multifragmental fracture of the right hip. On the third
day after the injury he began to complain of pain in the right side of chest, difficult respiration. A day
later the patient died of progressive heart and respiratory failure. Histological study of pulmonary and
cerebral blood vessels revealed orange sudanophilic droplets that completely obstructed the vessels of
microvasculature. What complication caused the death of the patient? {=Fat embolism

~Gas embolism

~Drug-induced embolism

~Microbial embolism

~Thromboembolism}

A 39-year-old patient has been suffering from gastric ulcer for the last 4 years. Krok 1 Stomatology
2015 7 His condition exacerbates in spring and autumn: he suffers from pain in epigastrium, heartburn,
nausea, constipation. What nosological term can be applied to the period between two exacerbations?
{=Pathological state

~Pathological process

~Pathological reaction

“Typical pathological process ~Good


health}

A patient at the early stage of diabetes mellitus was found to have polyuria. What is its cause?
{=Hyperglycemia

~Ketonemia

~Hypocholesterolemia

~Hypercholesterolemia ~Hyperkaliemia}

As a result of a trauma a patient has developed traumatic shock. The patient is fussy, talkative, pale. AP-
140/90 mm Hg, Ps-120 bpm. This condition is consistent with the following shock phase: {=Erectile ~Latent
~Terminal

~Torpid

~-}

In Western Europe nearly half of all congenital malformations occur in the children conceived in the
period when pesticides were used extensively in the region. Those congenital conditions result from the
following influence:
{=Teratogenic

~Carcinogenic

~Malignization

~Mutagenic

~Mechanical}

A 50-year-old patient has been referred for treatment of neck lymphadenitis. His individual penicillin
sensitivity was tested. In 30 seconds full-body fever raised in the patient and his arterial blood pressure
dropped to 0 mm Hg, followed by cardiac arrest. Resuscitation was unsuccessful. Autopsy revealed
acute venous hyperemia of viscera. Histological study revealed mast cells (tissue basocytes)
degranulation in the skin (at the area of injections), myocardium and lungs. What kind of
hypersensitivity reaction occurred in patient?

{=Anaphylactic

~Delayed-type hypersensitivity

~Complement-mediated cytotoxic

~Immune complex-mediated

~-}

After arriving in the polar region, researchers from Australia have complained of nervous disorders, loss
of appetite, aggravation of chronic diseases for 6 months. What process has been disrupted in extreme
conditions?
{=Adaptation

~Tolerance

~Tachyphylaxis

~Stress

~Reparation}

Pyrogenal administered to a rabbit, in the course of an experiment, resulted in increase of its body
temperature. What substance of those named below acts as a secondary pyrogen that is a part of
feverinducing mechanism?

{=Interleukin 1

~Pseudomonas polysaccharide (Piromen)

“Histamine

~Bradykinin
~Immunoglobulin}

Phenylketonuria is a disease caused by a recessive gene that is localized in the autosome. The parents
are heterozygous for this gene. They already have two sons with phenylketonuria and one healthy
daughter. What is the probability that their fourth child will have the disease too?

{=25%

~0%

~50%

~75%

~100%}

X-chromatin test of somatic cells is used for quick diagnostics of hereditary diseases caused by
variations of sex chromosomes number. What is the karyotype of a male, whose cells mostly contain
one X-chromatin body?

{=47, XXY
~45, XO
~46, XY
~48, XXXY
~A9, XXXXY}

When examining a female patient a doctor observed the following: misshapen auricles, elevated palate,
teeth growth disorder; mental retardation; no disruption of reproductive function. Provisional diagnosis
is the "super woman"syndrome. Point out the karyotype of this disease. {=(47, XXX)

~(47, XXY)
~(47, YYY)
~(47, XYY)
~(45, XO)}

The patient’s condition after blood transfusion has been aggravated by posttransfusion shock. Name the
type of allergic reaction causing this pathology.

{=Cytotoxic

~Anaphylactic

~Immune complex
~Delayed-type hypersensitivity

~Receptor-mediated}

A patient with gastric juice hypersecretion has been recommended to exclude from the diet rich broths
and vegetable infused water. A doctor recommended it, because these food products stimulate
production of the following hormone:

{=Gastrin

~Secretin

~Cholecystokinin

~Somatostatin

~Neurotensin}

An oculist detected increased time of darkness adaptation of a patient’s eye. What vitamin deficiency
can cause such symptom?

{=A

~E

~C

~K

~D}

A woman has scalded her hand with boiling water. The affected area of her skin became red, swollen
and painful. This effect is caused by accumulation of the following substance: {=Histamine

~Lysine

~Thiamine

~Glutamine

~Asparagine}

There are various diseases that cause sharp increase of active oxygen, thus leading to cell membranes
destruction. Antioxidants are used to prevent it from happening. The most potent natural antioxidant
is:

{=a-tocopherol

~Glucose

“Vitamin D
~Fatty acids
~Glycerol}

A pregnant woman developed severe toxemia with exhausting recurrent vomiting throughout a day. By
the end of the day she developed tetanic convulsions and bodily dehydration. The described changes
were caused by the following type of acid-base disbalance:

{=Nongaseous excretory alkalosis

~Gaseous alkalosis

~Gaseous acidosis

~Nongaseous metabolic acidosis

~Nongaseous excretory acidosis}

Due to recurring vomiting a patient has lost significant amount of gastric juice, which led to
development of acidbase dysbalance. What type of acid-base dysbalance has developed? {=Nongaseous
alkalosis

~Gaseous acidosis

~Nongaseous acidosis

~Gaseous alkalosis

~Metabolic acidosis}

A 28-year-old patient had been diagnosed with multifragmental fracture of the right hip. On the third
day after the injury he began to complain of pain in the right side of the chest, difficult respiration. One
day later the patient died of progressive heart and respiratory failure. Histological study of the
pulmonary and cerebral blood vessels revealed orange sudanophilic droplets that completely
obstructed the vessels of microvasculature. What complication caused the death of the patient?

{=Fat embolism

~Gas embolism

~Drug-induced embolism

~Microbial embolism ~Thromboembolism}

A woman who had been consuming alcohol excessively during her pregnancy had a child with cleft
palate and upper lip. These presentationsm are indicative of some chromosomal anomalies. What
process do they result from?

{=Teratogenesis

~Carcinogenesis
~Mutagenesis

~Phylogenesis

~Ontogenesis}

What factor results in maximal dilation of the gemomicrocirculatory pahtway vessels and their
increased permeability?

{=Histamine

~Endothelin

~Vasopressin

~Noradrenaline
~Serotonin}

A patient with diabetes mellitus has been delivered to a hospital unconscious. BP is low, Kussmaul’s
respiration is observed, the smell of acetone can be detected from the patient’s mouth. What
mechanism is leading in the coma development in this case?

{=Accumulation of ketone bodies in blood

~Accumulation of potassium ions

~Accumulation of sodium ions

~Accumulation of chlorine ions

~Accumulation of urea}

In Western Europe nearly half of all congenital malformations occur in the children conceived in the
period, when pesticides were used extensively in the region. Those congenital conditions result from
the following influence:

{=Teratogenic

~Carcinogenic

~Malignization

~Mutagenic

~Mechanical}

A 40-year-old woman suffering from diffuse toxic goiter presents with constant increase of her body
temperature. What mechanism results in such clinical presentation? {=Separation of oxidation and
phosphorization in cell mitochondria

~Increased breakdown of glycogen in hepatic cells


~Increased catabolism of protein in cells

~Increased excitability of nerve cells

~Increased cell sensitivity to catecholamines}

Periodontitis induces development of lipid peroxidation in the periodontal tissues, as well as increase in
malondialdehyde and hydrogen peroxide concentration in the oral cavity. Which of the following
enzymes provides antioxidant protection?

{=Catalase

~Amylase

~Maltase

~Lactase

~Invertase}

A victim of an earthquake has been remaining under debris for 7 days without food or water. What type
of starvation is it?

{=Complete

~Complete with continued hydration

~Quantitative

~Qualitative

~Incomplete}

A 50-year-old patient has been referred for treatment of neck lymphadenitis. His individual penicillin
sensitivity was tested. In 30 seconds fullbody fever raised in the patient and his arterial blood pressure
dropped to 0 mm Hg followed by cardiac arrest. Resuscitation was unsuccessful. Autopsy revealed acute
venous hyperemia of viscera. Histological study revealed mast cells (tissue basocytes) degranulation in
the skin (at the area of injections), myocardium and lungs. What kind of hypersensitivity reaction
occurred in the patient?

{=Anaphylactic

~Delayed-type hypersensitivity

~ Complement-mediated cytotoxic

~Immune complex-mediated

~-}
A 42-year-old woman has been administered propranolol for the ischemic heart disease. Yet she has
been found to have a concomitant condition that renders propranolol to be contraindicated. What
disease is it? {=Bronchial asthma

~Cholecystitis

~Arterial hypertension

~Duodenal ulcer

~Myasthenia}

A man submerged into the ice cold water and died soon as a result of abrupt exposure to cold. In such
cases an organism loses heat most intensively by the way of:

{=Heat conduction

~Radiation

~Convection

~Heat conduction and radiation

~No correct answer}

A woman had been taking synthetic hormones during her pregnancy. Her newborn girl presents with
excessive hairiness which has formal resemblance to adrenogenital syndrome. This sign of variability
iscalled:
{=Phenocopy

~Mutation

~Recombination

~Heterosis

~Replication}

Ketoacidosis that develops due to accumulation of ketone bodies in blood serum is a primary
complication of diabetes mellitus. What acid-base disbalance develops during this condition?
{=Metabolic acidosis

~Metabolic alkalosis

“Respiratory acidosis

“Respiratory alkalosis

~-}
During auscultation a 26-year-old patient was asked to breathe deep. After 10 breaths the patient lost
consciousness, which is associated with the development of the following condition: {=Respiratory
alkalosis

~Carbon dioxide acidosis

~Erythropenia

~Polycythemia

~Reduced oxygen capacity of blood}

Phenylketonuria is a disease caused by a recessive gene that is localized in an autosome. Parents are
heterozygous for this gene. They already have two sons with phenylketonuria and one healthy
daughter. What is the probability that their fourth child will have the disease too?

{=25%
~0%
~50%
~75%
~100%}

A doctor examines a 17-year-old girl. The following is detected: pharyngitis, cervical lymphadenopathy,
fever. The preliminary diagnosis is infectious mononucleosis. What method of investigation allows to
confirm this diagnosis at the disease onset?

{=Determining antibodies IgM to Epstein-Barr virus

~Microscopy of blood smear according to Giemsa method

~Determining antibodies IgG to Epstein-Barr virus

~Sabin-Feldman dye test

~Determining the amount of C-reactive Protein}

When examining a female patient a doctor observed the following: misshapen auricles, elevated
palate, teeth growth disorder; mental retardation; no disruption of reproductive function. Provisional
diagnosis is the"super woman"syndrome. Point out the karyotype of this disease: {=(47, XXX)

~(47, XXY)
~(47, YYY)
~(47, XYY)
~(45, XO)}
A patient has been administered conduction anesthesia with novocaine in preparation for tooth
extraction. After the anesthesia administration the patient developed swelling and hyperemy around
the injection site, skin itch, general fatigue, motor agitation. Name the developed complication:
{=Allergy

~Idiosyncrasy

~Tachyphylaxis

~Drug dependence
~Inflammation}

A woman is diagnosed with Turner’s syndrome (karyotype 45, XO). How many autosomal pairs would
her somatic cells contain?

{=22
~24
~23

~A4
~45}

A diet must include fats. Fats perform plastic function in an organism due to their inclusion in: {=Cell
membranes

~Cell ion channel

~Cell ion pumps

~Cell end-organs ~Glycocalyx}

A 30-year-old man addressed a doctor with complaints of enlarged submandibular lymph nodes.
Anamnesis states that the patient previously had been treated for acute apical periodontitis of the 36th
and 46th teeth. Histologically the following could be detected in the removed lymph node: hyperemia,
edema, increased amount of plasmocytes and plasmablasts in the medullary area and germinal centers
of follicles. What can cause such changes in lymph nodes?

{=Antigenic stimulation

~Immunodeficiency state

~Metastasis of a malignant tumor

~Granulomatous inflammation

~Lymphoma}
A 12-year-old patient suffering from acute leukemia presents with fever up to 39, 80C, acute pain in the
throat. Examination of the oral cavity has revealed swollen tonsils, their surface is covered in deep
lesions with uneven margins, numerous petechial hemorrhages in the pharyngeal mucosa and around
the tonsils. Determine the type of tonsillitis that complicates the disease progress in this case:
{=Necrotic

~Catarrhal

~Fibrinous

~Lacunar

~Purulent}

Heterozygous parents with A (II) and B (III) blood groups according to ABO system gave birth to a child.
What is the probability of the child having 0 (I) blood group?

{=50%

~0%

~100%
~75%

~25%}

An oculist has detected increased time of darkness adaptation of a patient’s eye. What vitamin
deficiency can cause such symptom?

{=C
~E

~A

~K

After anaesthetic application during tooth extraction the patient developed marked soft tissue edema of
the upper and lower jaw, skin rash on the face, reddening, and itching. What pathological process
results in such reaction to the anaesthetic?

{=Disturbed lymph drainage

~Allergy

~Inflammation

~Toxic action of a drug

~Circulatory deficiency}
After introdiction of adrenaline the patient’s blood glucose level increased. It is caused by intensified:

{=Glycolysis in the skeletal muscles

~Glycolysis in the liver

~Glycogenolysis in the liver

~Glycogen synthesis

~Glycogenolysis in the muscles}

A 20-year-old young man, who started to train systematically in athletics, has the following restingstate
blood values: erythrocytes — 5,5x1012/I, reticulocytes — 12%, hemoglobin — 160 g/l, color index —
1,03. Such blood values indicate erythropoiesis stimulation due to the following occurring in the process
of his training:

{=Physical activity

~Hypercapnia

~Hypoxemia

~Hyperventilation

~Hyperglycemia}

Inthe armpits of a patient there are small (1-1,5 mm), dorsoventrally flattened, wingless, bloodsucking
insects. Their larvae have been developing in the armpits as well.What disease is caused by these
insects? {=Relapsing fever

~Phthiriasis

~Chagas’ disease

~Sleeping sickness
~Plague}

In student, who unexpectedly met his girlfriend, developed an increase in systemic arterial pressure.
This pressure change was caused by the intensified realization of the following reflexes:

{=Conditioned parasympathetic

~Unconditioned parasympathetic

~Unconditioned sympathetic

~Conditioned sympathetic

~Conditioned sympathetic and parasympathetic}


A patient with diabetes mellitus has been delivered to a hospital unconscious. BP is low, Kussmaul’s
respiration is observed, the smell of acetone can be detected from the patient’s mouth. What mechanism
is leading in the coma development in this case?
{=Accumulation of potassium ions

~Accumulation of chlorine ions

~Accumulation of urea

~Accumulation of ketone bodies in blood

~Accumulation of sodium ions}

A patient complains of toothache. On examination he has been diagnosed with pulpitis. Which factor
played the main pathogenic role in the development of pain syndrome in this case? {=Vasospasm

~Activation of one of the complement system components

~Interleukin action

~Increased intratissular pressure in the dental pulp

~Inadequate stimulation of the mandibular nerve branch}

A victim of an earthquake has been remaining under debris for 7 days without food or water. What type
of starvation is it?

{=Incomplete

~Complete

~Quantitative

~Complete with continued hydration

~Qualitative}

A 42-year-old woman, who has been keeping to a vegetarian diet for a long period of time, consulted a
doctor. Examination revealed negative nitrogen balance in the patient. What factor is the most likely
cause of such a condition? {=Excessive amount of fats in the diet

~Insufficient amount of dietary fiber

~Insufficient amount of proteins in the diet

~Insufficient amount of fats in the diet

~Decreased rate of metabolic processses}

Cytogenetic analysis allowed to determine the patient’s karyotype — 47, XY, +21/46, XY. Name this
condition:
{=Genocopy

~Phenocopy

~Deletion

~Translocation

~Mosaicism}

A patient with acute bronchitis has been prescribed sulfanilamide drugs for treatment. In an hour after
administration the patient developed itching and vesicles filled with light transparent liquid on the face,
palms and soles. Name the mechanism of immune response:

{=Reaginic reaction

~Cell cytotoxicity
~

~Immune complex-mediated hypersensitivity

~Antibody-mediated cytolysis}

A 68-year-old man, who had been suffering from essential hypertension for a long time, was delivered
to a resuscitation unit with hemiplegia. The patient died after 7 hours. On autopsy: in the right cerebral
hemisphere there is a cavity 5x5 cm in size with uneven margins, filled with dark red blood clots. What
cerebral circulation disorder developed in the patient?

{=Thrombosis

~Hemorrhagic infiltration

~Hematoma

~Petechial hemorrhage

~Local venous hyperemia}

What factor results in the highest energy expenditure under the normal vital activity conditions?

{=Mental work

Food rich in calories

Increase of environment temperature

Decrease of environment temperature


Action of skeletal muscles}

During thermal stimulation it is characteristic of oral cavity blood vessels to:

{=Respond with constriction to hot stimuli


~Respond depending on the vessel functional condition

~Present no response towards thermal stimuli

~Respond with constriction to cold stimuli

~Dilate in response to both cold and hot stimuli}

Deaf parents with genotypes DDee and ddEE gave birth to a girl with normal hearing. Specify the form of
D and E genes interaction:

{=Polymery

~Overdominance

~Complete dominance

~Epistasis

~Complementary interaction}

During postmortem examination ofa 9-month-old infant it was determined that the cause of death was
cerebral edema. What water-electrolyte imbalance is the most likely cause of the edema development?

{=lsoosmolar hyperhydration

~Hyperosmolar dehydration

~Hyperosmolar hyperhydration

~lsoosmolar dehydration

~Hypoosmolar hyperhydration}

An accident had resulted in a 65-year-old man drowning in a lake. Resuscitation measures allowed to
restore his respiration and cardiac function. What factor prolongs the period of apparent death?
{=Hyperthermia

~Elderly age
~

~Hypothermia

~Prolonged preagony and agony}

A girl is diagnosed with primary microcephaly that is a monogenic autosomal recessive disease. Her
natural brother develops normally. What genotypes do the parents of these children have? {=aa x aa

~AAX AA

~Aa x Aa
~AAXaa

~AABB x AABB}

High-altitude dwellers typically demonstrate chronically intensified respiration and decreased pCO2
value of blood. What mechanism is leading in the compensation of their asid-base imbalance?

{=Decreased pulmonary ventilation

~Increased ammonia excretion with urine

~Desreased renal reabsorption of bicarbonate

~Increased pulmonary ventilation}

A 50-year-old man came to a hospital with complaints of memory disorders, painful sensations along
the nerve trunks, decreased mental ability, circulatory disorders and dyspepsia. Anamnesis states
excessive alcohol consumption. What vitamin deficiency can result in such symptoms? {=Retinol

~Niacin

~Thiamine

~Calciferol

~Riboflavin}

A patient presents with lymphocytic-monocytic leukogram pattern. It is characteristic of: {=Chronic


radiation sickness

~Acute inflammatory process

~Allergies

~Chronic inflammatory process}

A patient has undergone recurring blood tests that revealed sharp fluctuations of glucose content:

significant increase in absorptive state and significant decrease in postabsorptive state. What pathology
can be the cause of it?

{=Diabetes mellitus type II

~Diabetes mellitus type |

~Aglycogenosis (glycogenosis type 0)

~Acromegaly

~Endemic goiter}
Ulcer disease of the duodenum has been detected in a 38-year-old man. A treatment was prescribed
after which the patient considered himself to be healthy. However, half a year later the patient
developed pain in the epigastrium, heartburn, and insomnia. The patient’s condition can be estimated
as a: {=Latent period

~Pathological state

~Remission

~Development of chronic disease

~Relapse}

Autosomal nondisjunction had occurred in a woman during meiosis. An ovum with the third copy of the
18th chromosome was formed. The ovum was impregnated by normal spermatozoon. The resulting
child will suffer from:

{=Down’s syndrome

~Edwards’ syndrome

~Patau’s syndrome

~Klinefelter’s syndrome

~Turner’s syndrome}

Autopsy ofa 58-year-old man, who had been suffering from rheumatic heart disease and died of
cardiopulmonary decompensation, revealed gray diffuse film- and fiber-shaped coating in his
pericardium. What type of inflammation is characteristic of this pericarditis?

{=Diphtheritic fibrinous

~Suppurative

~Serous

~Hemorrhagic

~Croupous fibrinous}

A patient was diagnosed with a monogenic hereditary disease. Name this disease:

{=Hemophilia

~Hypertension

~Peptic ulcer disease of the stomach

~Poliomyelitis

~Hymenolepiasis}
A diabetus mellitus patient developed unconsciousness and convulsions after administration of insulin.
What result of blood glucose analysis is the most likely in this case?

{=1.5 mmol/L

~3.3 mmol/L

~8 mmol/L

~10 mmol/L

~5.5 mmol/L}

A pregnant woman developed severe toxemia with exhausting recurrent vomiting throughout a day. By
the end of the day she developed tetanic convulsions and dehydration. The described changes were
caused by the following type of acid-base imbalance:

{=Nongaseous excretory alkalosis

~Gaseous alkalosis

~Gaseous acidosis

~Nongaseous metabolic acidosis

~Nongaseous excretory acidosis}

Often the cause of secondary immunodeficiency is an infectious affection of an organism, when agents
reproduce directly in the cells of immune system and destroy them. Specify the diseases, during which
the described above occurs:

{=Infectious mononucleosis, AIDS

~Tuberculosis, mycobacteriosis

~Poliomyelitis, viral hepatitis type A

~Dysentery, cholera
~Q fever, typhus}

A patient presents with disturbed patency of the airways at the level of small and medium bronchial
tubes. What acidbase imbalance can the patient develop?

{=Respiratory acidosis

“Respiratory alkalosis

~Metabolic acidosis

~Metabolic alkalosis

~Acid-base balance remains unchanged}


A woman had been taking synthetic hormones during her pregnancy. Her newborn girl presents with
excessive hairiness which has formal resemblance to adrenogenital syndrome. This sign of variability
is called:
{=Phenocopy

~Mutation

~Recombination

~Heterosis

~Replication}

A 42-year-old woman, who has been keeping to a vegetarian diet for a long period of time, consulted a
doctor. Examination revealed negative nitrogen balance in the patient. What factor is the most likely
cause of such a condition? {=Insufficient amount of proteins in the diet

~Insufficient amount of dietary fiber

~Excessive amount of fats in the diet

~Insufficient amount of fats in the diet

~Decreased rate of metabolic processses}

In hot weather the bus passengers asked to open the roof hatches. What way of heat transfer is
activated in this situation?

{=Convection

~Conduction

~Radiation

~Conduction and radiation


~Sweat evaporation}

At the end of winter a student, who had been lately in the state of nervous tension, developed a case of
URTI after overexposure to cold. What is the case of this disease?

{=Pathogenic agent

~Nervous stress

~Overexposure to cold

~Improper diet

~Hypovitaminosis}

A patient was diagnosed with Klinefelter’s syndrome. The patient with this disease will have the
karyotype (47, XXY). How many sex chromosomes are in this complement? {=Three
~Zero

~One

~Two

~Forty four}

30 minutes after dental treatment the patient developed red itching spots on the face and oral mucosa.
The patient was diagnosed with urticaria. What bioactive substance with vasodilating and pruriginous
effect is produced during this type of allergic reaction?

{=Histamine

~Prostaglandin E2

~Leukotriene B4

~Interleukin-1

~Bradykinin}

A patient has been administered conduction anesthesia with novocaine in preparation for tooth
extraction. After the anesthesia administration the patient developed swelling and hyperemy around
the injection site, skin itch, general fatigue, motor agitation. Name the developed complication:
{=Allergy

~Idiosyncrasy

~Tachyphylaxis

~Drug dependence

~Inflammation}

A 16-year-old girl, who has been starving herself for a long time to lose weight, developed an edema.
This phenomenon is mainly caused by:

{=Hypoproteinemia due to protein synthesis disturbance

~Hypoglycemia due to glycogen synthesis disturbance

~Venous congestion and increased venous pressure

~Deceleration of glomerular filtration rate

~Decreased production of vasopressin in the hypothalamus}

A woman is diagnosed with Turner’s syndrome (karyotype 45, XO). How many autosomal pairs would
her somatic cells contain?

{=22
~24
~23
~A4
~A5}

A 50-year-old man came to a hospital with complaints of memory disorders, painful sensations along
the nerve trunks, decreased mental ability, circulatory disorders and dyspepsia. Anamnesis states
excessive alcohol consumption. What vitamin deficiency can result in such symptoms? {=Thiamine

~Niacin
~Retinol

~Calciferol

~Riboflavin}

Differentiation of B-lymphocytes into plasma cells leads to synthesis of immunoglobulins that ensure
specific immune response of the body. Differentiation of B-lymphocytes takes place in the following
organ of immune system:

{=Tonsils

~Red bone marrow

~Liver

~Thymus

“Thyroid gland}

A patient who takes a blocker of membrane cytoreceptors of efferent conductor synapses of autonomic
nervous system complains about dry mouth. What receptors are blocked?

{=Muscarinic cholinoreceptors

~Nicotinic cholinoreceptors

~H2-receptors

~a-adrenoreceptors

~B-adrenoreceptors}

Examination of a patient revealed enlargement of some body parts (jaw, nose, ears, feet, hands), but
body proportions were conserved. It might be caused by intensified secretion of the following hormone:
{=Somatotropin

~Somatostatin
~Tetraiodothyronine

~Triiodothyronine
~Cortisol}

15 minutes after a car accident examination of a 35 year old man revealed massive injury of
lower extremities without serious external loos of blood. The victim is in excited state. What component
of pathogenesis of traumatic shock is basic and requires urgent correction?

{=Pain

~Acute renal insufficience

~Intoxication

~Cardiac function disorder


~Internal loss of plasma}

A 23 year patient was admitted to the hospital in grave condition with craniocerebral trauma. His
respiration is characterized by a spasmodic long inspiration interrupted by a short expiration. What
respiration type is it typical for?

{=Apneustic

~Gasping

~Kussmaul's respiration

~Cheyne-Stokes respiration

~Biot's respiration}

Two days after myocardial infarction a patient had a sudden systolic pressure decrease up to 60 mm,
tachycardia up to 140/min, dyspnea; the patient lost consciousness. What mechanism is principal for
the shock pathogenesis? {=Decrease of cardiac volume

~Intoxication

~Decrease of circulating blood volume

~Paroxysmal tachycardia ~Anaphylactic


reaction}

After a psychoemotional stress a 48 year old patient had a sudden attack of acute heart pain with
irradiation to the left hand. Nitroglycerine suppressed pain in 10 minutes. What pathogenetic
mechanism is principal for the pain development?

{=Spasm of coronary vessels

~Dilatation of peripheral vessels


~Coronary vessel occlusion

~Embarrassement of coronary vessels

~Increased need of myocardium in oxygen}

A 40 year old man who took part in disaster-management at a nuclear power plant fell sick with
paradontitis. What etiological agent is the most important for the development of this pathology?
{=Emotional stress

~lron deficit

~Malnutrition

~Increased load of dentoalveolar apparatus

~Streptococcus}

A woman after labor lost 20 kg of body weight, her hair and teeth fall out, she has muscle atrophy
(hypophysial cachexia). Synthesis of what hypophysis hormone is disturbed?

{=Somatotropic

~Corticotrophic

~Thyreotropic

~Gonadotropic
~Prolactin}

Damage of one of the reactors at a nuclear power plant resulted in runout of radioactive products.
People who were present in the high-radiation area got approximately 250-300 R. They were urgently
taken to the hospital. What blood changes will be typical for this period?

{=Lymphopenia

~Leukopenia

~Anemia

~Thrombocytopenia

~Neutropenia}

A year after subtotal stomach resection on account of ulcer of lesser curvature the following blood
changes were revealed: anemia, leukocytopenia and thrombocytopenia, color index - 1,3, megaloblasts
and megalocytes. What factor deficiency caused the development of thos pathology? {=Castle's factor

~Hydrochloride acid

~Mucin
~Pepsin

~Gastrin}

A 45 year old patient was taken to the hospital by an emergency team with serious cranial trauma in
shock condition. Objectively: unconscious, skin is pale, body t°- 35,0°C, low muscular tonus, reflexes are
absent, pulse is rapid and weak, AP- 50/30 mm Hg. What clinical shock stage is it? {=Terminal stage

~Erectile stage

~Excitement stage

~Inhibition stage

~Torpid stage}

After traumatic tooth extraction a patient complains of a severe dull pain without accurate localization
in his gum, body temperature rise up to 37,5°C. He was diagnosed with alveolitis. What type of pain
does the patient have?

{=Protopathic

~Epicritic

“Visceral

~Referred

~Phantom}

After poisoning with an unknown drug a 37 year old patient has stereotypical face muscle
contractions that imitate blinking and squinting. What form of motor function disorder of nervous
system is it? {=Hyperkinesia

~Hypokinesia

~Akinesia

~Ataxy

~-}

Rabbits lived on food with addition of cholesterol. Five months later the atherosclerotic aorta changes
were revealed. Name the main cause of atherogenesis in this case:

{=Exogenous hypercholesterolemia

~Overeating

~Hypodynamia
~Endogenous hypercholesterolemia ~-
}
A patient with primary nephrotic syndrome has the following content of whole protein: 40 g/l. What
factor caused hypoproteinemia?

{=Proteinuria

“Transition of protein from vessels to tissues

~Reduced protein synthesis in liver

~Increased proteolysis

~Disturbance of intestinal protein absorption}

A man who took part in disaster-management at a nuclear power plant had hemorrhagic syndrome at
the same time with acute radiation sickness. What is the most important thing for the pathogenesis of
this syndrome?

{=Thrombocytopenia

~Destructed structure of vessel walls

~High activity of fibrinolysis factors

~High activity of anticoagulative blood system

~Low activity of anticoagulative blood system}

Four months ago a 43 year old patient had a traumatic amputation of his lower extrimity. Now he
complains of sensing the amputated extremity and having constantly grave, sometimes unbearable pain
in it. What type of pain does he have?

{=Phantom

~Causalgia

~Neuralgia

~Thalamic

~Reflex}

In crisis period a 14 year old child ill with diphtheria has AP- 70/50 mm Hg accompanied by abrupt fall in
temperature and tachycardia. What form of vascular tone disturbance is it?

{=Acute hypotension

~Chronic hypotension

~Vegetovascular dystonia
~Essential arterial hypotension

~-}
A patient ill with jaundice has increased content of conjugated bilirubin and bile acids in blood, no
stercobilinogen in urine. What jaundice are these symptomps typical for?

{=Obstructive

~Hepatic

~Hepatocellular

~Hemolytic

~Cythemolytic}

A 19 year old patient was diagnosed with chronic acquired hemolytic anemia. What is the leading
pathogenetic mechanism of this pathology's development?

{=Autoimmune hemolysis

~Toxic hemolysis

~Intracellular hemolysis

~Hyposmolarity of plasm

~Osmotic hemolysis}

A 56 year old man was taken to the hospital with complaints of general weakness, pain and burning in
the region of tongue, extremity numbness. In the past he had resection of cardiac part of ventricle.
Blood test: Hb- 80 g/L; RBC- 2,0+1072/L; colour index of blood- 1,2; leukocytes - 3,5*10°/L. What {=type
of anemy is it?

{=B12 folic-deficient

~Hemolytic

~Posthemorrhagic

~Aplastic

~lron-deficient}

A patient has the following diagnosis: renal hypertension. What is the initial pathogenetic factor of
arterial hypertension development in this case?

{=Renal ischemia

~Hypernatremia

~Hyperaldosteronism
~Intensified renin synthesis

~Intensified angiotensin synthesis}

A patient with an acute myocarditis has the clinic presentations of cardiogenic shock. What
pathogenetic mechanism plays the main part in shock development?

{=Disorder of pumping ability of heart

~Depositing of blood in veins

~Decrease of diastolic flow to the hear

~Decrease of vascular tone

~Increase of vascular tone}

A patient was admitted to a hospital with poisoning with unsound food. His stomach was lavaged with
solution of potassium permanganate. What is its mechanism of action?

{=Release of atomic oxygen

~Release of chlorine

~Release of iodine

~Disturbance of synthesis of respiratory chain enzymes

~Destruction of bacteria membranes}

A typical symptom of cholera is great loss of water and sodium ions. What mechanism underlies
development of diarrhea in this case?

{=Activation of adenylate cyclase of enterocytes

~Increased secretion of renin by the cells of renal arterioles

~Aldosterone oxidation in adrenal cortex

~Inhibition of vasopressin synthesis in hypothalamus

~Increased corticotropin synthesis}

A patient underwent a surgery for excision of a cyst on pancreas. After this he developed
haemorrhagic syndrome with apparent disorder of blood coagulation. Development of this
complication can be explained by: {=Activation of fibrinolytic system

~Insufficient fibrin production

~Activation of anticoagulation system

“Activation of Christmas factor


~Reduced number of thrombocytes}

ECG of a 44-year-old patient shows signs of hypertrophy of both ventricles and the right atrium. The
patient was diagnosed with the tricuspid valve insufficiency. What pathogenetic variant of cardiac
dysfunction is usually observed in case of such insufficiency?

{=Heart overload by volume

~Heart overload by resistance

~Coronary insufficiency

~Cardiac tamponade

~Primary myocardial insufficiency}

A patient who takes a blocker of membrane cytoreceptors of efferent conductor synapses of autonomic
nervous system complains about dry mouth. What receptors are blocked?

{=Muscarinic cholinoreceptors

~Beta-adrenoreceptors

~H2-receptors

~Nicotinic cholinoreceptors

~Alpha-adrenoreceptors}

Examination of a patient revealed enlargement of some body parts (jaw, nose, ears, feet, hands), but
body proportions were conserved. It might be caused by intensified secretion of the following hormone:
{=Somatotropin

~Tetraiodothyronine

~Cortisol

~Triiodothyronine
~Somatostatin}

Blood of patients ill with diabetes mellitus has high content of free fatty acids. It may be caused by:

{=High activity of triglyceride lipase of adipocytes

~Activation of ketone bodies utilization

~Low activity of phosphatidylcholine-cholestein-acyltransferase of plasma

~Activation of synthesis of apolipoproteins A-1, A-2, A-4

~Accumulation of palmitoyl-CoA in the cytosol}


An experimental rat got intra-abdominal injection of 10 ml of 40% glucose solution. 60 minutes later the
rat passed into a comatose state as a result of dehydratation. What is the mechanism of development of
this state?

{=Rise of osmotic pressure of extracellular fluid

~Reduction of vasopressin secretion

~Rise of oncotic pressure of extracellular fluid

~Loss of salts and water

~Acid-base disbalance}

15 minutes after a car accident examination of a 35 year old man revealed massive injury of lower
extremities without serious external loos of blood. The victim is in excited state. What component of
pathogenesis of traumatic shock is basic and requires urgent correction? {=Pain

~Internal loss of plasma

~Intoxication

~Acute renal insufficience

~Cardiac function disorder}

A 50-year-old patient has been examined by a dentist and found to have crimson smooth tongue.

Blood analysis revealed a decrease in RBC level and hemoglobin concentration, colour index of 1,3,
symptoms of megaloblastic hematopoiesis, degenerative changes in WBCs. What blood disorder
was found in this patient? {=B12-folic-acid-deficiency anemia

~Myeloid leukemia

~lron deficiency anemia

~Aplastic anemia

~Hemolytic anemia}

A 23 year patient was admitted to the hospital in grave condition with craniocerebral trauma. His
respiration is characterized by a spasmodic long inspiration interrupted by a short expiration. What
respiration type is it typical for?

{=Apneustic

~Biots respiration

~Kussmauls respiration

~Gasping
~Cheyne-Stokes respiratio}

Two days after myocardial infarction a patient had a sudden systolic pressure decrease up to 60 mm,
tachycardia up to 140/min, dyspnea; the patient lost consciousness. What mechanism is principal for
the shock pathogenesis? {=Decrease of cardiac volume

~Decrease of circulating blood volume

~Anaphylactic reaction

~Paroxysmal tachycardia

~Intoxication}

After a psychoemotional stress a 48 year old patient had a sudden attack of acute heart pain with
irradiation to the left hand. Nitroglycerine suppressed pain in 10 minutes. What pathogenetic
mechanism is principal for the pain development?

{=Spasm of coronary vessels

~Coronary vessel occlusion

~Increased need of myocardium in oxygen

~Embarrassement of coronary vessels

~Dilatation of peripheral vessels}

A 46-year-old patient suffering from the diffuse toxic goiter underwent resection of the thyroid gland.
After the surgery the patient presents with appetite loss, dyspepsia, increased neuromuscular
excitement. The body weight remained unchanged. Body temperature is normal. Which of the following
has caused such a condition in this patient?

{=Reduced production of parathormone

~Increased production of thyroliberin

~Reduced production of thyroxin

~Increased production of thyroxin

~Increased production of calcitonin}

18-year-old patient complains of general weakness, fatigue, low spirits. The patient is of the asthenic
constitution type. Ps- 68/min., AP- 90/60 mm Hg. She has been found to have primary neurocirculatory
hypotension. What is the leading factor of the arterial pressure drop in this patient? {=Decreased tonus
of resistive vessels

~Decreased cardiac output

~Hypovolemia
~Decreased minute blood volume

~Deposition of blood in the veins of the systemic circulation}

18-year-old patient complains of general weakness, fatigue, low spirits. The patient is of the asthenic
constitution type. Ps- 68/min., AP- 90/60 mm Hg. She has been found to have primary neurocirculatory
hypotension. What is the leading factor of the arterial pressure drop in this patient?

{=Decreased cardiac output

~Decreased tonus of resistive vessels

~Hypovolemia

~Decreased minute blood volume

~Deposition of blood in the veins of the systemic circulation}

A woman after labor lost 20 kg of body weight, her hair and teeth fall out, she has muscle atrophy
(hypophysial cachexia). Synthesis of what hypophysis hormone is disturbed?

{=Somatotropic

~Thyreotropic

~Prolactin

~Gonadotropic

~Corticotrophic}

Damage of one of the reactors at a nuclear power plant resulted in runout of radioactive products.
People who were present in the high-radiation area got approximately 250-300 R. They were urgently
taken to the hospital. What blood changes will be typical for this period?

{=Lymphopenia

~Thrombocytopenia

~Neutropenia

~Leukopenia

~Anemia}

A year after subtotal stomach resection on account of ulcer of lesser curvature the following blood
changes were revealed: anemia, leukocytopenia and thrombocytopenia, color index - 1,3, megaloblasts
and megalocytes. What factor deficiency caused the development of those pathology? {=Castles factor

~Hydrochloride acid

~Pepsin
~Gastrin

~Mucin}

A patient with a craniocerebral injury presents with respiration characterized by progressively deeper
respiratory movements followed by a gradual decrease that results in a temporary stop in breathing.
What pattern of abnormal respiration are these features typical for?

{= Cheyne-Stokes

~Gasping

~Apneustic

~Biots

~Kussmauls}

After the traumatic tooth extraction a patient is complaining of acute, dull, poorly-localized pain in
gingiva, body temperature rise up to 37,5°C. The patient has been diagnosed with alveolitis. Specify the
kind of pain in this patient:

{=Protopathic

“Visceral

~Epicritic

~Heterotopic

~Phantom}

After poisoning with an unknown drug a 37 year old patient has stereotypical face muscle
contractions that imitate blinking and squinting. What form of motor function disorder of nervous
system is it? {=Hyperkinesia

~Hypokinesia

~Ataxy

~~Akinesia
}

Rabbits lived on food with addition of cholesterol. Five months later the atherosclerotic aorta changes
were revealed. Name the main cause of atherogenesis in this case:

{=Exogenous hypercholesterolemia ~Overeating

~Endogenous hypercholesterolemia
~
~Hypodynamia}

A patient with primary nephrotic syndrome has the following content of whole protein: 40 g/l. What
factor caused hypoproteinemia?

{=Proteinuria

~Disturbance of intestinal protein absorption

~Reduced protein synthesis in liver

“Transition of protein from vessels to tissues

~Increased proteolysis}

Four months ago a 43 year old patient had a traumatic amputation of his lower extrimity. Now he
complains of sensing the amputated extremity and having constantly grave, sometimes unbearable pain
in it. What type of pain does he have?

{=Phantom

~Reflex

~Neuralgia
~Causalgia

~Thalamic}

In crisis period a 14 year old child ill with diphtheria has AP- 70/50 mm Hg accompanied by abrupt fall in
temperature and tachycardia. What form of vascular tone disturbance is it?

{=Acute hypotension

~Essential arterial hypotension


~

~Chronic hypotension

~Vegetovascular dystonia}

A patient ill with jaundice has increased content of conjugated bilirubin and bile acids in blood, no
stercobilinogen in urine. What jaundice are these symptomps typical for?

{=Obstructive

~Hepatocellular

~Cythemolytic

~Hemolytic

~Hepatic}
19 year old patient was diagnosed with chronic acquired hemolytic anemia. What is the leading
pathogenetic mechanism of this pathologys development?

{=Autoimmune hemolysis

~Toxic hemolysis

~Hyposmolarity of plasm

~Osmotic hemolysis

~Intracellular hemolysis}

A 56 year old man was taken to the hospital with complaints of general weakness, pain and burning in
the region of tongue, extremity numbness. In the past he had resection of cardiac part of ventricle.
Blood test: Hb- 80 g/L; RBC- 2,0x1012/L; colour index of blood- 1,2; leukocytes - 3,5x109/L. What type of
anemia is it?

{=B12 folic-deficient

~Posthemorrhagic

~lron-deficient

~Aplastic

~Hemolytic}

A patient has the following diagnosis: renal hypertension. What is the initial pathogenetic factor of
arterial hypertension development in this case?

{=Renal ischemia

~Hypernatremia

~Intensified renin synthesis

~Intensified angiotensin synthesis

~Hyperaldosteronism}

A diver who has been staying at the depth of 40 m for a long time fell ill with caisson disease as a
result of decompression. The main pathogenetic factor is the following embolism: {=Fat

~Air

~Gaseous

~Paradoxical
~Tissue}
ECG of a patient shows such alterations: P-wave is normal, P-Q-interval is short, ventricular QRST
complex is wide, R-wave is double-peak or two-phase. What form of arrhythmia is it?

{=WPW syndrome (Wolff-Parkinson-White)

~Atrioventricular block

~Fredericks syndrome (atrial flutter)

“Ventricular fibrillation

~Ciliary arrhythmia}

A patient with an acute myocarditis has the clinic presentations of cardiogenic shock. What
pathogenetic mechanism plays the main part in shock development?

{=Disorder of pumping ability of heart

~Decrease of vascular tone

~Increase of vascular tone

~Depositing of blood in veins

~Decrease of diastolic flow to the heart}

Potassium cyanide that is a poison came into a patients organism and caused death a few minutes after
it. The most probable cause of its toxic effect was abnormal activity of:

{=Cytochrome oxidase

~ATP-synthetase

~Haemoglobin synthesis

~NADP-H-dehydrogenase

~Catalase}

After the exposure to ionizing radiation a person was found to have a decrease in blood granulocyte
level. What mechanism underlies these changes?

{=Leikopoiesis inhibition

~Autoimmune process development

~Increased passage of granulocytes into the tissues

~Increased disintegration of leucocytes

~Disturbed release of mature leukocytes from the bone marrow}


An unconscious patient had been delivered to a hospital by the ambulance. Objectively: absent reflexes,
occasional convulsions, irregular breathing. After a laboratory examination he was diagnosed with
hepatic coma. What metabolite accumulation is essential for the development of the central nervous
system disorders?

{=Ammonia

“Histamine

~Glutamine

~Urea

~Bilirubin}

A patient was admitted to a hospital with poisoning with unsound food. His stomach was lavaged with
solution of potassium permanganate. What is its mechanism of action?

{=Release of atomic oxygen

~Release of iodine

~Release of chlorine

~Disturbance of synthesis of respiratory chain enzymes

~Destruction of bacteria membranes}

A newborn has signs of dyspepsia after milk feeding. Symptoms of dyspepsia disappear when milk is
substituted for glucose solution. The newborn has low activity of the following enzyme: {=Lactase

~Maltase

~lsomaltase

~Amylase

~Invertase}

Injection of an anaesthetic before the tooth extraction resulted in development of anaphylactic shock
accompanied by oliguria. What pathogenetic mechanism caused a decrease in diuresis in this case?

{=Decrease in hydrostatic pressure in the renal corpuscle capillaries

~Increase in vasopressin secretion

~Damage of glomerular filter

~Increase in hydrostatic pressure in the Bowmans capsule

~Increase in oncotic pressure of blood plasma}


A patient presents with icteritiousness of skin, scleras and mucous membranes. Blood plasma the total
bilirubin is increased, stercobilin is increased in feces, urobilin is increased in urine. What type of
jaundice is it?

{=Haemolytic

~Parenchymatous

~Gilberts disease

~Obturational
~Cholestatic}

A patient suffers from the haemorrhagic syndrome that shows itself in frequent nasal bleedings,
posttraumatic and spontaneous intracutaneous and intra-articular haemorrhages. After a laboratory
study a patient was diagnosed with the type B haemophilia. This disease is provoked by the deficiency of
the following factor of blood coagulation:

{=IX
“VIII

Vv

“VII

~XI}

A 23-year-old patient with acute pulpitis has elevated body temperature and an increase in the WBC
count up to 14- 109/L. The leucogram is as follows: basophils - 0, eosinophils - 2, monocytes - 0,
immature neutrophils - 4, stab neutrophils - 8, segmented neutrophils - 56, lymphocytes - 26,
monocytes - 4. How can we interpret these changes in the white blood cells?

{=Neutrophilia with a regenerative left shift

~Neutrophilia with a degenerative left shift

~Neutrophilia with a hyperregenerative left shift

~Lymphocytosis

~Neutrophilic leukocytosis with a right shift}

A 39-year-old patient underwent hematologic tests. The following results were obtained: RBC- 2, 8

- 1012/L, Hb- 80 g/L, color index - 0,85, reticulocytes - 0,1%, platelets - 160 - 109/L, WBC - 60 - 109/L.
Basophils - 2, eosinophils - 8, promyelocytes - 5, myelocytes - 5, immature neutrophils - 16, stab
neutrophils - 20, segmented neutrophils - 34, lymphocytes - 5, monocytes - 5. What form of blood
pathology are these results indicative of?

{=Chronic myeloid leukemia


~Acute myeloid leukemia

~Hypoplastic anemia

~Undifferentiated leukemia

~Hemolytic anemia}

A 49-year-old patient was found to have a disproportionate enlargement of hands, feet, nose, ears,
superciliary arches and cheek bones. Blood test revealed hyperglycemia, impaired glucose tolerance.
What is the most likely cause of this pathology development?

{=Hypersecretion of growth hormone

~Posterior pituitary hormone hypersecretion

~Insulin hyposecretion

~Vasopressin hyposecretion

~Glucocorticoid hypersecretion}

A 44-year-old patient with obstructive jaundice has been admitted to a hospital with the symptoms of
cholemic syndrome. On the ECG arrhythmia shows up. What kind of arrhythmia is the patient most
likely to have? {=Sinus bradycardia

~Sinus tachycardia

~Atrial premature contraction

“Ventricular premature contraction

~Atrioventricular block}

Ina 52-year-old patient with chronic glomerulonephritis, the glomerular


fi- Itration rate (GFR) was
reduced by 20% compared to normal. What causes the decrease in GFR in patients with chronic renal
failure?

{=Reduced number of active nephrons

~Tubulopathy

~Obstruction of the urinary tract

~Renal ischemia

~Renal artery thrombosis}

A 36-year-old patient with diabetes mellitus had seizures with loss of consciousness after an insulin
injection. What was the result of blood glucose test?

{=2,5 mmol/|
~3,3 mmol/|

~8,0 mmol/|

~10 mmol/|

~5,5 mmol/l}

Following thyroid surgery, a 47-yearold female patient had fibrillary twitching of muscles in the arms,
legs and face. These disorders can be treated by the introduction of the following hormone:
{=Parathyroid hormone

~Triiodothyronine

~Thyrotropin

~Thyroxine

~Thyroid-stimulating hormone}

A 42-year-old patient with tetanus developed an acute respiratory failure. What type of respiratory
failure occurs in this case?

{=Disregulatory impairment of alveolar ventilation

“Restrictive impairment of alveolar ventilation

~Obstructive impairment of alveolar ventilation

~Perfusion impairment

~Diffusion impairment}

When students pass an exam, they often complain of having "dry mouth". The mechanism underlying
the development of this condition is the activation of the following processes:

{=Conditioned sympathetic

~Unconditioned parasympathetic

~Conditioned parasympathetic

~Unconditioned sympathetic

~Unconditioned peripheral}

A female with Rh-negative blood of A (II) type has a child with AB (IV) type who has been diagnosed with
hemolytic disease resulting from Rh-conflict. What blood type may the baby’s father have?

{=lll (B), Rh-positive

~| (0), Rh-positive
~II (A), Rh-positive

~|V (AB), Rh-negative

~IIl (B), Rh-negative}

A 66-year-old male patient has liver carcinoma with syndrome of portal hypertension. What kind of
portal hypertension does the patient have?

{=Intrahepatic

~Suprahepatic

~Subhepatic

~Combined

~-}

A 39-year-old patient with pyelonephritis has been found to have hyposthenuria combined with
polyuria. According to this data, what process is most likely to be disrupted?

{=Tubular reabsorption

~Glomerular filtration

~Tubular secretion

~Tubular excretion ~-
}

A 12-year-old child is of short stature, has disproportionate body structure and mental retardation.
These characteristics might be caused by the hyposecretion of the following hormone: {=Thyroxine

~Insulin

~Cortisol

~Somatotropin

~Glucagon}

A 48-year-old female patient with a history of cholelithiasis has recurring steatorrhea. What vitamin
deficiency may develop as a complication of the current disease?

{=K
~B6
~C
~PP
~B12}

Transfusion of Rh-incompatible blood resulted in hemolytic jaundice development in the patient.

What laboratory blood value confirms this type of jaundice?

{=Accumulation of unconjugated bilirubin

~Reduction of unconjugated bilirubin

~Accumulation of urobilinogen

~Reduction of stercobilin

~Reduction of conjugated bilirubin}

For several days a 55-year-old female patient has had pain attacks in the right upper quadrant after
eating fatty foods. Visually, there is yellowness of sclera and skin. The patient has acholous stool, beer-
colored urine. What substance present in the patient’s urine causes its dark color? {=Conjugated
bilirubin

~Ketone bodies

~Unconjugated bilirubin

~Stercobilin

~Bilirubin glucuronides}

A 23-year-old patient with diabetes has hyperglycemia at the rate of 19 mmol/l which is clinically
manifested by glucosuria, polyuria, polydipsia. Which of the listed below mechanisms is responsible
for the development of glycosuria? {=Exceedence of glucose renal threshold

~Non-enzymatic glycosylation of proteins

~Polyuria

~Polydipsia

~Tissue dehydration}

Experimental stimulation of the peripheral segment of the vagus nerve of a cat will result in the
following changes: {=Decreased heart rate

~Increased heart rate

~Dilated pupils

~Increased respiratory rate

~Bronchiectasis}
Arterial pH is 7,4; primary urine - 7,4; final urine - 5,8. Decrease in the pH of final urine is the result of
the secretion of the following ions in the nephron tubules:

{=Hydrogen ions

~Potassium ions

~Hydrogen carbonate ions

~Urea

~Creatinine}

A 49-year-old male patient with myocardial infarction has been admitted to the cardiology department.
What changes in the peripheral blood cells are induced by the necrotic changes in the myocardium?

{=Neutrophilic leukocytosis

~Monocytosis

~Eosinophilia

~Thrombocytopenia

~Lymphopenia}

During ABO blood grouping by using coliclons (diagnostic monoclonal antibodies), haemagglutination did
not occur with any of the coliclons. What is the blood group of the patient under examination?

{=0 (1)
~A (Il)
~B (III)
~AB (IV) ~-}

A 64-year-old male patient died with symptoms of acute cardiovascular failure. Autopsy results: the
section of the anterior wall of the left ventricle showed a yellowish flaccid 1,5-2 cm focus surrounded by
a reddish rim. The convoluted coronary arteries had lumen irregularly narrowed by 75%. The vessel
intima was thickened, dense, covered with whitish plaques, crunched when cut. What disease can you
think of?

{=Acute myocardial infarction

~Continuously recurrent myocardial infarction

~Postinfarction cardiosclerosis

~Microfocal cardiosclerosis

~Recurrent myocardial infarction}


During a hypertensive crisis a patient has had a hemorrhagic stroke resulting in a lack of voluntary
movements, increased tendon reflexes and muscle tone of the left arm and leg. What is this motor
dysfunction called?

{=Hemiplegia

~Paraplegia

~Tetraplegia

~Monoplegia

~Flaccid paralysis}

A 42-year-old patient with gastric ulcer has a disbalance between the aggressive and defensive
factors. Which of the following factors contributes to the development of gastric ulcer?
{=Helicobacter pylori
~Mucin

~Hydrocarbonate

~Prostaglandin ~Prostacyclin}

In the solution being used for perfusing the isolated heart of rat, the K+ concentration has been
increased to 8 mmol/L. What changes in the heart are to be expected?

{=Diastolic arrest

~Systolic arrest

~Heart force increase

“Heart rate increase

~There will be no changes}

ECG of a 46-year-old patient shows an increase in the QRS duration. This might be caused by:

{=Increased ventricular activation time

~Conduction disturbances in the AV node

~Increased atrial excitability

~Increased atrial and ventricular excitability

~Increased atrial activation time}

A 28-year-old patient complains of frequent gingival haemorrhages. Blood test revealed the clotting
factor Il (prothrombin) deficiency. What phase of blood coagulation is impaired in this patient?

{=Thrombin generation
~Vascular-platelet haemostasis

~Fibrinolysis

~Clot retraction

~-}

A 40-year-old patient was revealed to have blood clotting time of 2 minutes under a stressful condition.
It is primarily caused by the following hormon affecting hemocoagulation: {=Catecholamine

~Cortisol

~Aldosterone

~Somatotropin
~Vasopressin}

A 38-year-old female patient has been brought into admission room with uterine bleeding. What
will be revealed by blood test? {=Decrease of packed cell volume

~Eosinophilia

~Decreased erythrocyte sedimentation rate

~Leukocytosis

~Increased color index of blood}

There is high content of proteine and erythrocytes in urine. This can be caused by increased:

{=Permeability of renal filter permeability

~Effective filtration pressure

~Hydrostatic blood pressure in glomerular capillaries

~Hydrostatic pressure of primary urine in capsule

~Oncotic pressure of blood plasma}

A 32-year-old patient has purulent wound in the lower third of forearm. Smear of purulent wound
content has been made. What cells will be generally detected, if it is stained using RomanovskyGiemsa
stain?

{=Neutrophil

~Eosinophil

~Lymphocyte
~Erythrocyte
~Basocyte}

A pregnant woman complains of vaginal mucosa irritation, itching and genital tracts secretion.
Bacterioscopy of vaginal smears revealed large gram-positive oval oblong cells that form
pseudomicelium. What is the most probable channel of infection?

{=Endogenous infection

~Sexual transmission

~Contact infection

~Vector-borne transmission

~Wound infection}

A patient has sustained a traumatic injury of the greater pectoral muscle. This resulted in a decrease of:

{=Inspiratory reserve volume

~Expiratory reserve volume

~Tidal volume

~Residual volume

~Functional residual lung capacity}

Denture installation has caused excessive salivation in patient. It is caused by the following reflexes:

{=Unconditioned

~Conditioned

~Conditioned and unconditioned

~Local

~-}

A 53-year-old patient with a long history of nephrolithiasis underwent nephrectomy. The kidney looks
as a thin-walled sac filled with urine. Renal parenchyma is atrophied. Specify this complication of
nephrolithiasis:

{=Hydronephrosis

~Pyelonephritis

~Pyonephrosis

~Multicystic kidney disease


~Nephrosclerosis}

A patient from Prykarpattia (at the foot of the Carpathian mountains) with endemic goiter consulted a
doctor about suppuration of gingival angles and loosening of teeth. What is a major factor of
periodontitis development in this case?

{=Endocrine disorders

~Stress effects

~Hypersalivation

“Violation of swallowing

~Malnutrition}

The total number of leukocytes in patient’s blood is 90 - 109/I. Leukogram: eosinophils - 0%, basophils -
0%, juvenile - 0%, stab neutrophils - 2%, segmentonuclear cells - 20%, lymphoblasts - 1%,
prolymphocytes - 2%, lymphocytes - 70%, monocytes - 5%, Botkin-Gumprecht cells. Clinical examination
revealed enlarged cervical and submandibular lymph nodes. Such clinical presentations are typi- Krok 1
Stomatology 2015 8 cal for the following pathology:

{=Chronic lympholeukosis

~Acute lympholeukosis

~Lymphogranulomatosis

~Infectious mononucleosis

~Chronic myeloleukosis}

A 67-year-old patient has atherosclerosis of cardiac and cerebral vessels. Examination revealed
hyperlipidemia. What class of blood plasma lipoproteids is most important in atherosclerosis
pathogenesis?

{=Low-density lipoproteids

~Chylomicrons

~a-lipoproteids

~High-density lipoproteids

~-}

A 42-year-old patient with tetanus developed an acute respiratory failure. What type of respiratory
failure occurs in this case?

{=Disregulatory impairment of alveolar ventilation

“Restrictive impairment of alveolar ventilation


~Obstructive impairment of alveolar ventilation

~Perfusion impairment

~Diffusion impairment}

A patient, who has been suffering from severe injury of thorax, went into shock followed by symptoms
of acute renal failure. What is the primary mechanism of acute renal failure development in this case?

{=Arterial pressure drop

~Disruption of urinary outflow

~Increase of pressure in glomerular capsule

~Increase of pressure in renal arteries

~Decrease of oncotic blood pressure}

A 12-year-old child is of short stature, has disproportionate body structure and mental retardation.
These characteristics might be caused by the hyposecretion of the following hormone: {=Thyroxine

~Insulin

~Cortisol

~Somatotropin

~Glucagon}

A patient with acute retention of urine has been brought to an admission room. During examination a
doctor found out that the patient has urethral obturation caused by pathology of the surrounding
organ. Name this organ.

{=Prostate

~Testicle

~Seminal vesicle

~Spermatic cord

~Epididymis}

A 43-year-old female complains of weight loss, hyperhidrosis, low-grade fever, increased irritability. She
has been found to have hyperfunction of the sympatheticadrenal system and basal metabolism. These
disorders can be caused by hypersecretion of the following hormone:

{=Thyroxine

~Somatotropin
~Corticotropin

~Insulin

~Aldosterone}

As a result of dysfunction of protein synthesis in liver a patient with hepatic insufficiency has disturbed
synthesis of procoagulants, prothrombin, fibrinogen. Which of the listed syndromes can be expected in
this patient?

{=Haemorrhagic

~Portal haemorrhagic syndrome

~Hepatolienal syndrome

~Acholia syndrome

~Cholaemia syndrome}

A 49-year-old male patient with myocardial infarction has been admitted to the cardiology department.
What changes in the peripheral blood cells are induced by the necrotic changes in the myocardium?

{=Neutrophilic leukocytosis

~Monocytosis

~Eosinophilia

~Thrombocytopenia

~Lymphopenia}

Tooth extraction in a patient with chronic persistent hepatitis was complicated by a prolonged bleeding.
What is the cause of hemorrhagic syndrome?

{=Decreased production of thrombin

~Increased production of thromboplastin

~Decreased production of fibrin

~Increased synthesis of fibrinogen

~Increased fibrinolysis}

A patient presented to a hospital with complaints about quick fatigability and significant muscle
weakness. Examination revealed an autoimmune disease that causes functional disorder of receptors in
the neuromuscular synapses. This will result in the disturbed activity of the following mediator:
{=Acetylcholine

~Noradrenaline
~Dopamine
~Serotonin

~Glycine}

After the transfusion of the concentrated red blood cells the patient developed posttransfusion shock.
What is the leading mechanism of acute renal failure in this case?

{=Glomerular filtration disorder

~Tubular reabsorption disorder

~Tubular secretion disorder

~Urinary excretion disorder

~Impairment of the renal incretory function}

A patient with pituitary tumor complains of increased daily diuresis (polyuria). Glucose concentration in
blood plasma equals 4,8 mmol/l. What hormone can be the cause of this if its secretion is disturbed?

{=Vasopressin

~Aldosterone

~Natriuretic hormone

~Insulin

~Angiotensin |}

During ABO blood grouping by using coliclons (diagnostic monoclonal antibodies), haemagglutination did
not occur with any of the coliclons. What is the blood group of the patient under examination?

{=0 (1)
~A (Il)
~B (III)
~AB (IV)
~-}

Throughout a year a 37-year-old woman periodically got infectious diseases of bacterial origin, their
course was extremely lingering, remissions were short. Examination revealed low level of major classes
of immunoglobulins. The direct cause of this phenomenon may be the following cell dysfunction:

{=Plasmocytes

~Phagocytes
~Neutrophils

~Macrophages

~Lymphocytes}

A patient is diagnosed with pancreatic diabetes with associated hyperglycemia. Krok 1 Stomatology
2015 15 Glycemia rate can be assessed retrospectively (4-8 weeks prior to examination) by measuring
concentration of the following blood plasma protein:

{=Glycated hemoglobin

~Albumin

~Fibrinogen

~C-reactive protein

~Ceruloplasmin}

A patient consulted an immunologist about diarrhea, weight loss within several months, low-grade
fever, enlarged lymph nodes. The doctor suspected HIV infection. What immunocompetent cells must
be studied in the first place?

{=Helper T-lymphocytes

~Suppressor T-lymphocytes

~B-lymphocytes

~Monocytes

~Plasma cells}

After examining the patient the doctor recommended him to eliminate rich meat and vegetable broth,
spices, smoked products from the diet, since the patient was found to have:

{=Increased secretion of hydrochloric acid by the stomach glands

~Reduced secretion of hydrochloric acid by the stomach glands

~Reduced motility of the gastrointestinal tract

~Reduced salivation ~Biliary


dyskinesia}

A patient suffering from pericarditis with rapid progression has developed acute cardiac tamponade.
What regulation mechanism is most likely to compensate for this pathology?

{=Tachycardia

~Heterometric
~Homeometric

~Inotropic effect of catecholamines

~Vasoconstriction}

A 64-year-old male patient died with symptoms of acute cardiovascular failure. Autopsy results: the
section of the anterior wall of the left ventricle showed a yellowish flaccid 1,5-2 cm focus surrounded by
a reddish rim. The convoluted coronary arteries had lumen irregularly narrowed by 75%. The vessel
intima was thickened, dense, covered with whitish plaques, crunched when cut. What disease can you
think of?

{=Acute myocardial infarction

~Continuously recurrent myocardial infarction

~Postinfarction cardiosclerosis

~Microfocal cardiosclerosis

~Recurrent myocardial infarction}

A 43-year-old patient has acute pancreatitis with concomitant disruption of common bile duct patency.
What condition can it result in?

{=Mechanical jaundice

~Hemolytic jaundice

~Hepatocellular jaundice

~Hepatic coma

~Portal hypertension}

An inflammatory process in tissues is characterised by hyperemia and edema. What leukocytes


situated in connective tissue provide for vasodilatation and increased blood vessel capacity under
these conditions? {=Basocytes

~Neutrophils

~Eosinophils

~T-lymphocytes

~B-lymphocytes}

In course of an experiment researchers stimulate a branch of a sympathetic nerve that innervates heart.
What changes in cardiac activity should be registered?

{=Increase in heart rate and heart force


~Decrease in heart force

~Increase in heart rate

~Increase in heart force

~Increase in arterial pressure}

A patient has myocardial infarction. The first several hours of such medical condition will be
characterized by signifi- cant increase of activity of the following enzyme in his blood serum: {=Creatine
phosphokinase

~Lactate dehydrogenase4

“Aspartate aminotransferase

~Lactate dehydrogenase5S

~Alanine-aminotransferase}

A 54-year-old patient with viral hepatitis has complication of hepatic coma caused by massive necrosis
of liver epithelial cells. What kind of hepatic coma is it characteristic of?

{=Parenchymatous

~Shunt

~Mixed type ~Porto-


caval

~Ketoacidotic}

A patient has petechial hemorrhages on the gums, hard and soft palate, buccal mucosa. This is caused
by the dysfunction of the following blood corpuscles:

{=Platelets

~Eosinophils

~Monocytes

~Lymphocytes

~Erythrocytes}

Examination of a patient revealed dermatitis, diarrhea, dementia. What vitamin deficiency is the cause
of this condition?

{=Nicotinamide

~Ascorbic acid
~Folic acid

~Biotin

~Rutin}

A 49-year-old patient was found to have a disproportionate enlargement of hands, feet, nose, ears,
superciliary arches and cheek bones. Blood test revealed hyperglycemia, impaired glucose tolerance.
What is the most likely cause of this pathology development?

{=Hypersecretion of growth hormone

~Posterior pituitary hormone hypersecretion

~Insulin hyposecretion

~Vasopressin hyposecretion

~Glucocorticoid hypersecretion}

A patient suffers from mutation of a gene that corresponds with hemoglobin synthesis. This condition
led to development of sicklecell disease. Name the pathological hemoglobin characteristic of this
disease:

{=HbS

~HbA

~HbF

~HbA1

~Bart-Hb}

A patient, who had suffered severe blood loss three days ago, underwent blood test. The following data
was obtained in leukogram: leukocytes -12 - 109/I, basophils - 0, eosinophils -3, myelocytes - 0, juvenile -
3, stabneutrophils - 12, segmented neutrophils- 62, lymphocytes - 16, monocytes -4.

What change of leukocyte content occurred in this case?

{=Neutrophilia with regenerative leftshift

~Neutrophilia with degenerative leftshift

~Neutrophilia with right-shift

~Absolute lymphopenia

~Absolute monocytopenia}
After the traumatic tooth extraction a patient is complaining of severe dull poorly-localized pain in
gingiva, body temperature rise up to 37, 50C. The patient has been diagnosed with alveolitis. Specify the
kind of pain in this patient:

{=Protopathic

~Epicritic

“Visceral

~Heterotopic

~Phantom}

The total number of leukocytesm in the patient’s blood is 90 - 109/I. Leukogram: eosinophils - 0%,
basophils - 0%, juvenile - 0%, stab neutrophils - 2%, segmented neutrophils - 20%, lymphoblasts - 1%,
prolymphocytes -2%, lymphocytes - 70%, monocytes -5%, Botkin-Gumprecht cells. Clinical examination
revealed enlarged cervical and submandibular lymph nodes. Such clinical presentations are typical of
the following pathology:

{=Chronic lympholeukosis

~Acute lympholeukosis

~Lymphogranulomatosis

~Infectious mononucleosis

~Chronic myeloleukosis}

A patient has a history of chronic heart failure. Which of the following hemodynamic parameters is a
major symptom of cardiac decompensation development?

{=Decreased stroke volume

~Tachycardia development

~Tonogenic dilatation

~Increased peripheral vascular resistance

~Increased central venous pressure}

After a serious psychoemothional stress a 48-year-old patient suddenly developed acute heart ache
irradiating to the left arm. Nitroglycerine relieved the pain attack after 10 minutes. What is the leading
pathogenetic mechanism of this process development?

{=Spasm of coronary arteries

~Dilatation of peripheral vessels

~Obstruction of coronary vessels


~Compression of coronary vessels

~Increase in myocardial oxygen consumption}

A patient with chronic hepatitis complains of increased sensitivity to barbiturates that previously
induced no symptoms of intoxication. What hepatic function is disrupted and primarily responsible for
such reaction in this patient?

{=Metabolic
~Bilification

~Hemodynamic

~Hemopoietic

~Phagocytic}

Glucose content of blood keeps at sufficient level after one week of starvation. Is it caused by activation
of the following process:

{=Gluconeogenesis

~Glycolysis

~Glycogenolysis

~Tricarboxylic acid cycle

~Glycogen phosphorolysis}

A student, who unexpectedly met his girlfriend, developed an increase in systemic arterial pressure. This
pressure change was caused by the intensified realization of the following reflexes:

{=Conditional sympathetic

~Conditional parasympathetic

~Conditional sympathetic and parasympathetic

~Unconditional parasympathetic

~Unconditional sympathetic}

After the exposure to ionizing radiation a person was found to have a decreased blood granulocyte
level. What mechanism underlies these changes?

{=Leikopoiesis inhibition

~Increased passage of granulocytes into the tissues

~Autoimmune process development


~Increased disintegration of leucocytes

~Disrupted release of mature leukocytes from the bone marrow}

On the 4th day of treatment with diclofenac sodium a 55-year-old patient has developed gastric
hemorrhage due to an ulcer appearing on the gastric mucosa. Ulcerogenic action of this drug is caused
by decreased secretion of:

{=Prostaglandin E2

~Leukotriene

~Prostaglandin E1

~Cyclic endoperoxides

~Thromboxane}

A 43-year-old woman complains of weight loss, hyperhidrosis, low-grade fever, increased irritability. She
has been found to have hyperfunction of the sympathetic-adrenal system and basal metabolism. These
disorders can be caused by hypersecretion of the following hormone: {=Thyroxine

~Somatotropin

~Corticotropin
~Insulin

~Aldosterone}

A blood test was performed for a patient with allergic rhinitis. Blood smear stained after Romanowsky
reveals large number of cells with the following structure: segmented nucleus consists of 2-3 segments;
cytoplasm is fi- led with bright-pink oxyphil granularity; granules are large. Name these cells:

{=Eosinophils

~Lymphocytes

~Monocytes

~Basocytes

~Neutrophils}

For several days a 55-year-old woman has been suffering from pain attacks in the right upper quadrant
after eating fatty foods. Visually, there is yellowness of sclera and skin. The patient has acholic stool,
beer-colored urine. What substance present in the patient’s urine causes its dark color? {=Conjugated
bilirubin

~Ketone bodies

~Unconjugated bilrubin
~Stercobilin

~Bilirubin glucuronides}

A 49-year-old man with myocardial infarction has been admitted to a cardiology department. What
changes in the peripheral blood cells are induced by the necrotic changes in the myocardium?
{=Neutrophilic leukocytosis

~Monocytosis

~Eosinophilia

~Thrombocytopenia

~Lymphopenia}

A 19-year-old young man has been examined in a nephrological hospital. Increased potassium content
was detected in secondary urine of the patient. Such changes have been most likely caused by the
increased secretion of the following hormone:

{=Aldosterone

~Oxytocin

~Adrenalin

~Glucagon

~Testosterone}

Injection of an anaesthetic before the tooth extraction resulted in development of anaphylactic shock
accompanied by oliguria. What pathogenetic mechanism caused the decrease in diuresis in this case?

{=Decrease in hydrostatic pressure in the renal corpuscle capillaries

~Increase in hydrostatic pressure in the Bowman’s capsule

~Damage of the glomerular filter

~Increase in oncotic pressure of blood plasma

~Increase in vasopressin secretion}

A patient complains of toothache. On examination he has been diagnosed with pulpitis. Which factor
played the main pathogenic role in the development of pain syndrome in this case? {=Increased
intratissular pressure in the dental pulp

~Vasospasm

~Inadequate stimulation of the mandibular nerve branch


~Activation of one of the components of the complement system ~Interleukin
action}

After the transfusion of the concentrated red blood cells the patient developed posttransfusion shock.
What is the leading mechanism of the acute renal failure in this case?

{=Glomerular filtration disorder

~Tubular reabsorption disorder

~Tubular secretion disorder

~Urinary excretion disorder

~Impairment of the renal incretory function}

A 78-year-old patient suffering from atherosclerosis has been delivered to a surgical ward with signs of
acute abdomen. Laparoscopy revealed blackened and flaccid small intestine loops; demarcation line is
not clear. Diagnose the changes that occurred in the patient’s small intestine:

{=Hemorrhagic infarction complicated with humid gangrene

Hemorrhagic infarction complicated with dry gangrene

Ischemic stroke complicated with humid gangrene

Ischemic stroke complicated with dry gangrene -}

A patient with pituitary tumor complains of increased daily diuresis (polyuria). Glucose concentration in
blood plasma equals 4,8 mmol/I. What hormone can be the cause of this, if its secretion is disturbed?

{=Vasopressin

~Aldosterone

~Natriuretic hormone

~Insulin

~Angiotensin |}

A woman suffering from essential hypertension had suddenly lost consciousness; she was delivered to a
resuscitation unit in a comatose state with the diagnosis of disturbed cerebral circulation. The patient
died one day after her hospitalization. Autopsy revealed a cavity in the left hemisphere of the brain. The
cavity is 5x4 cm in size and filled with blood clots and liquid blood. What hemorrhage is it according to
the mechanism of its origin?

{=Hemorrhage caused by vessel rupture

~Hemorrhage caused by vessel erosion

~Diapedetic hemorrhage
~Hemorrhagic extravasation

~Petechial hemorrhage}

During ABO blood grouping by using zoliclons (diagnostic monoclonal antibodies), hemagglutination did
not occur with any of the zoliclons. What is the blood group of the patient under examination?

{=0 (1)
~A (Il)
~B (III)
~AB (IV)}

A 40-year-old patient suffers from intolerance of dairy food products. This condition has likely
developed due to insufficiency of the following digestive enzyme:

{=Lactase

~Lipase

~Maltase

~Invertase

~Amylase}

A patient has been diagnosed with severe B12-deficient anemia with hemopoiesis. Anamnesis states
totalgastrectomy. What cells allow to confirm this diagnosis, if they are absent in the peripheral blood?
{=Megalocytes
~Microcytes

~Ovalocytes

~Normocytes

~Anulocytes}

Roentgenologically confirmed an obstruction of common bile duct that prevents bile from inflowing to
the duodenum. What process is likely to be disturbed?

{=Fat emulgation

~Protein absorption

~Carbohydrate hydrolysis

~Hydrochloric acid secretion in stomach

~Salivation inhibition}
During ascent into mountains at the altitude of 5000 meters the group of climbers has developed the
following complaints: dyspnea, increased heart rate, headache, vertigo, tinnitus. What is the cause of
such symptoms? {=Hypoxemia

~Hypokalemia

~Hypothermia

~Erythropenia
~Leucopenia}

A 65-year-old patient had been treated for 3 days in a resuscitation unit for a cardiac pathology.
Suddenly he developed ventricular fibrillation that became the immediate cause of death of this patient.
Microscopy of the left ventricular myocard revealed a large focus of cardiomyocyte
karyolysisdemarcated by the zone of hyperaemia. What cardiac pathology was the causeof death?
{=Acute myocardial infarction

~Ischemic myocardial degeneration

~Acute myocarditis

~Diffuse cardiosclerosis

~Postinfarction cardiosclerosis}

Which of the named below is the substrate of activated Christmas factor that takes part in blood
coagulation? {=Factor X

“Vitamin K

~Fibrinogen

~Fibrin

~Thrombin}

Students have been remaining for a long time in a badly ventilated room. They developed respiratory
changes caused by irritation of their peripheral chemoreceptors that react primarily to:

{=Decrease of oxygen tension in arterial blood

~Increase of oxygen tension in arterial blood

~Decrease of carbon dioxide tension in arterial blood

~Increase of hydrogen ion concentration in arterial blood

~Decrease of hydrogen ion concentration in arterial blood}


A 2-year-old child presents with mental development retardation, intolerance of proteins, severe
hyperammonemiia against the background of low blood urea content. This condition is caused by the
congenital deficiency of the following mitochondrial enzyme:

{=Carbamoyl phosphate synthetase

~Citrate synthase

~Succinate dehydrogenase

~Malate dehydrogenase

~Monoamine oxidase}

Corticosteroid analogues induce breakdown of muscle proteins into free amino acids. Under such
conditions these amino acids become involved with the following processes:

{=Gluconeogenesis in liver

~Glycolysis in muscles

~Synthesis of higher fatty acids

~Glycogenolysis

~Decarboxylation}

Parkinson’s disease is caused by disrupted dopamine synthesis. What brain structure synthesizes this
neurotransmitter?

{=Substantia nigra

~Pallidum

~Quadrigeminal plate

~Red nuclei

~Hypothalamus}

A 30-year-old man died from electrocution. What was the cause of death?

{=Central respiratory arrest

~Acute renal failure

~Acute respiratory failure

~Shock

~Internal hemorrhage}
A 35-year-old woman complains of swollen neck. Subtotal thyreoidectomy is performed. On histological
examination of the removed part of the thyroid gland the following was detected: atrophy of
parenchyma, moderate sclerosis development, duffuse infiltration by lymphocytes and plasma cells
leading to formation of lymphatic follicles. What pathology has developed in the thyroid gland?

{=Hashimoto’s thyroiditis

~Follicular adenoma

~Riedel’s thyroiditis

~Papillary carcinoma of the thyroid gland

~Diffuse toxic goiter}

A 30-year-old woman has decreased enzyme content in the pancreatic juice. This condition can be
caused by insufficient secretion of the following hormone:

{=Vasoactive intestinal peptide

~Cholecystokinin-pancreozymin

~Secretin

~Somatostatin

~Gastric inhibitory polypeptide}

Psychological evaluation determined that a person is able to quickly adapt to changing situation, has
good memory, is emotionally stable, possesses of high working ability. This person is the most likely to
be: {=Melancholic

~Choleric

~Sanguine

~Phlegmatic}

A patient suffers from diabetes mellitus with fasting hyperglycemia over 7,2 mmol/I. What blood plasma
protein would allow to assess the patient’s glycemia level retrospectively (4-8 weeks prior to
examination)? {=Fibrinogen

~Albumin

~Glycated hemoglobin

~C-reactive protein

~Ceruloplasmin}

A patient is in the state of hypoglycemic coma. What hormone can cause this condition if
overdosed? {=Progesterone
~Somatotropin

~Corticotropin

~Insulin

~Cortisol}

Along with normal hemoglobin types there can be pathological ones in the body of an adult. Specify one
of them:

{=HbA1
~HbF
~HbS
~HbA2
~HbO2}

Chronic overdosage of glucocorticoids leads to the development of hyperglycemia. What process of


carbohydrate metabolism is responsible for this effect?

{=Pentose-phosphate cycle

~Glycogenesis

~Glycogenolysis

~Aerobic glycolisis ~Gluconeogenesis}

Due to morbid affection of the supraoptic and paraventricular nuclei of the hypothalamus a 40-yearold
patient has developed polyuria (10-12 liters per day) and polydipsia. The following hormone is deficient,
thus leading to this disturbance:

{=Oxytocin

~Somatotropin

~Thyrotropin

~Vasopressin

~Corticotropin}

A patient with glossitis presents with disappearance of lingual papillae, reddening and burning pain in
the tongue. Blood test: erythrocytes 2, 2x1012/l, hemoglobin — 103 g/I, color index — 1,4. What type
of anemia is it? {=B12 folate-deficient

~a-thalassemia
~lron refractory

~B-thalassemia

~lron deficiency}

A patient with chronic renal failure presents with reduced inulin clearance of 60 ml/min. The following
renal function is disturbed:

{=Glomerular filtration

~Reabsorption in the proximal tubular segment of the nephron

~Reabsorption in the tubules of collecting duct

~Reabsorption in the distal tubular segment of the nephron}

Often the cause of secondary immunodeficiency is an infectious affection of an organism, when agents
reproduce directly in the cells of immune system and destroy them. Specify the diseases, during which
the described above occurs:

{=Dysentery, cholera

~Q fever, typhus

~Tuberculosis, mycobacteriosis

~Poliomyelitis, viral hepatitis type A

~Infectious mononucleosis, AIDS}

Premature babies often develop respiratory distress syndrome. This pathology is caused by the
deficiency of a certain component of the blood—air barrier. Name this component: {=Alveolocytes

~Surfactant

~Endothelial basement membrane

~Capillary endothelium

~Alveolar basement membrane}

A patient has a history of chronic heart failure. Which of the following hemodynamic parameters is a
major symptom of cardiac decompensation development?

{=Increased peripheral vascular resistance

“Increased central venous pressure

~Tachycardia development

~Tonogenic dilatation
~Decreased stroke volume}

Due to trauma the patient’s parathyroid glands have been removed, which resulted in inertness,
thirst, sharp increase of neuromuscular excitability. Metabolism of the following substance is
disturbed: {=Chlorine

~Manganese

~Calcium

~Molybdenum

~Zinc}

A doctor has established significant growth retardation, disproportional body build, and mental
deficiency of a child. What is the most likely cause of this pathology?

{=Hypothyroidism

~Hyperthyroidism

~Hypopituitarism

~Genetic defects

~Insufficient nutrition}

During development of a frostbite the exposed skin becomes pale and its temperature drops. What
mechanism is the basis of these developments?

{=Visceral vasoconstriction

~Dermal and subcutaneous vasodilatation

~Reflex vasoconstriction

~Decreased heart rate

~Closure of arteriovenous anastomoses}

A patient during fasting developed ketoacidosis as a result of increased fatty acids decomposition. This
decomposition can be inhibited with:

{=Thyroxin

~Cortisol

~Glucagon

~Adrenaline

~Insulin}
Examination of a patient revealed glycosuria and hyperglycemia. He complains of dry mouth, itchy skin,
frequent urination, thirst. He has been diagnosed with diabetes mellitus. What is the cause of polyuria
in this patient?

{=Increased urine osmotic pressure

~Increased filtration pressure

~Increased plasma oncotic pressure

~Decreased cardiac output

~Decreased plasma oncotic pressure}

A patient, who had been eating only polished rice, developed polyneuritis caused by thiamine
deficiency. What compound can be indicative of this kind of avitaminosis when excreted with urine?

{=Methylmalonic acid

~Malate

~Pyruvic acid

~Uric acid

~Phenylpyruvate}

A doctor examined a patient, studied the patient’s blood analyses and concluded that the peripheral
organs of immunogenesis are damaged. What organs are the most likely to be affected? {=Kidneys

~Thymus

~Tonsils

~Red bone marrow

~Yellow bone marrow}

The patient’s saliva has been tested for antibacterial activity. What saliva component has antibacterial
properties?

{=Amylase

~Parotin

~Cholesterol

~Lysozyme

~Ceruloplasmin}
During an experiment aimed as study of respiration regulation processes the peripheral chemoreceptors
of test animals were stimulated, which resulted in changed respiratory rate and depth. Where are these
receptors localized?

{=Capillary bed, aortic arch, carotid sinus

~Capillary bed, carotid sinus

~Atria, carotid sinus

~Aortic arch, carotid sinus

~Capillary bed, aortic arch}

After an abortion a 25-year-old woman developed oliguria, anuria, and increasing azotemia. The patient
died of acute renal failure. Autopsy revealed degeneration and necrosis of the convoluted renal tubules
epithelium. What disease was the cause of death in this case?

{=Chronic glomerulonephritis

~Acute pyelonephritis

~Acute necrotizing nephrosis

~Renal amyloidosis

~Rapidly progressive glomerulonephritis}

A patient on the 2nd day after cardiac infarction presents with acute decrease of systolic blood
pressure down to 60 mm Hg with tachycardia 140/min., dyspnea, loss of consciousness. What
mechanism is essential in the pathogenesis of shock developed in this case? {=Increased
myocardial excitability caused by products of necrotic disintegration

~Development of paroxysmal tachycardia

~Development of anaphylactic reaction to myocardial proteins

~Decreased cardiac output

~Decreased circulating blood volume}

For biochemical diagnostics of cardiac infarction it is necessary to determine activity of a number of


enzymes and their izoenzymes in the blood. What enzyme assay is considered to be optimal for
confirming or ruling out cardiac infarction at the early stage, after the patient develops thoracic pain?

{=LDH1 isoenzyme

~Creatine kinase MM isoenzyme

~Creatine kinase MB isoenzyme

~LDH5 isoenzyme
~Cytoplasmic isoenzyme of aspartate aminotransferase}

A 56-year-old man presents with parathyroid tumor. The following is observed: muscle weakness,
osteoporosis, bone deformation, nephroliths consisting of oxalates and phosphates. The patient’s
condition is caused by:

{=Increased secretion of thyroxin

~Increased secretion of parathyroid hormone

~Increased secretion of calcitonin

~Decreased secretion of parathyroid hormone

~Decreased secretion of calcitriol}

There is increased activity of AST, LDH1, LDH2, and CPK in the patient’s blood. Pathological process most
likely occurs in the:

{=Liver

~Adrenal glands

~Skeletal muscles

~Kidneys

~Heart}

A patient with autoimmune thyroiditis has been prescribed a peptide hormonal agent. Specify this
agent:

{=Trimethoprim

~Triquilar

~Tamoxifen ~L-
thyroxin

~Triamcinolone}

Red bone marrow has been damaged under radioactive emission of 5 Gy. What determines the red
bone marrow sensitivity towards ionizing radiation?

{=High content of peroxides in the cells

~High content of free radicals in the cells

~Intensive cell division

~Radiosensitizers in the cells


~Destructive effect of radiotoxins on DNA synthesis}

A 40-year-old patient suffers from intolerance of dairy products. This condition has likely developed due
to insufficiency of the following digestive enzyme:

{=Amylase

~Lactase

~Maltase

~Lipase

~Invertase}

After examining a patient a doctor recommended him to exclude rich meat and vegetable broths,
spices, and smoked products from the diet, since the patient was found to have:

{=Reduced motility of the gastrointestinal tract

~Reduced secretion of hydrochloric acid by the stomach glands

~Increased secretion of hydrochloric acid by the stomach glands

~Reduced salivation

~Biliary dyskinesia}

Roentgenologically confirmed obstruction of common bile duct resulted in preventing bile from
inflowing to the duodenum. What process is likely to be disturbed?

{=Carbohydrate hydrolysis

~Protein absorption

~Fat emulgation

~Hydrochloric acid secretion in stomach

~Salivation inhibition}

Typical signs of food poisoning caused by C. botulinum include diplopia, swallowing and respiration
disorders. These signs develop due to:

{=Enterotoxic shock development

~Enterotoxin action

~Exotoxin action

~Adenylate cyclase activation

~Adhesion of the agent to enterocyte receptors}


In the course of an experiment researchers stimulate a branch of the sympathetic nerve that innervates
heart. What alterations of cardiac activity should be registered?

{=Increase in arterial pressure

~Increase in heart rate and heart force

~Increase in heart rate

~Decrease in heart force

~Increase in heart force}

During tooth brushing it is not uncommon for oral mucosa to be injured. However, bleeding quickly
stops on its own.What substances in saliva quickly staunch the flow of blood during minor oral injuries?

{=Lipolytic enzymes

~Mineral substances

“Lysozyme and mucin

~Procoagulants

~Amylolytic enzymes}

An athlete (a long-distance runner) during competition has developed acute heart failure. This
pathology developed due to: {=Coronary blood flow disturbance

~Pericardial pathology

~Pressure overload

~Volume overload

~Direct damage to myocardium}

A 50-year-old man, who has been suffering for a long time from viral hepatitis, developed mental
impairments, impairments of consciousness, and motor disturbances (tremor, ataxia, etc.). What is the
mechanism of such condition?

{=Decreased synthesis of albumins and globulins in the liver

~Insufficient phagocytic function of stellate macrophages

~Decreased detoxification function of the liver

~Disturbed lipid exchange in the liver

~Alterations in the lipid composition of blood}


A 30-year-old man had suffered a thoracic trauma in a traffic accident, which resulted in disturbance of
external respiration. What ventilatory failure can be observed in this case?

{=Dysregulatory

~Mixed type

~Pulmonary restrictive

~Obstructive

~Extrapulmonary restrictive}

A 30-year-old man with glomerulonephritis has developed nephrotic syndrome. What symptom
invariably accompanies nephrotic syndrome?

{=Glucosuria

~Anemia

~Azotemia

~Proteinuria

~Low urine specific gravity}

A patient is diagnosed with pancreatitis. Starch decomposition disturbance occurs in the patient’s
intestine due to deficiency of the following pancreatic enzyme:

{=Carboxypeptidase

~Amylase

~Chymotrypsin

~Tripsin

~Lipase}

An experimental animal, a dog, received a weak solution of hydrochloric acid through a tube inserted
into the duodenum. Primarily it will result in increased secretion of the following hormone:

{=Secretin

~Gastrin

~Histamine

~Cholecystokinin
~Neurotensin}
A woman presents with edemas. In her urine there is a large amount of protein excreted. What nephron
segment is functionally disturbed in this case?

{=Renal corpuscle

~Proximal convoluted tubule

~Distal convoluted tubule

~Descending limb of loop of Henle

~Ascending limb of loop of Henle}

The patient exhausted by starvation presents with intensification of the following process in the
liver and kidneys: {=Gluconeogenesis

~Urea synthesis

~Bilirubin synthesis

~Hippuric acid synthesis


~Uric acid synthesis}

A patient with glossitis presents with disappearance of lingual papillae, reddening and burning pain in
the tongue. Blood test: erythrocytes - 2.2 - 1012/l, hemoglobin - 103 g/l, color index - 1.4. What type of
anemia is it? {=B12 folate-deficiency

~lron deficiency

~a-thalassemia ~B-
thalassemia

~lron refractory}

A 13-year-old girl is an in-patient at the hematology department of the regional children’s hospital. She
was diagnosed with iron-deficiency anemia. What type of hypoxia does this patient have?

{=Hemic

~Circulatory

“Tissue

“Respiratory

~Mixed}

A 55-year-old man was diagnosed with acute glomerulonephritis. Name the main mechanism of
anemia development in this case: {=Decreased erythropoietin synthesis

~Decreased glomerular filtration


~Decreased synthesis of renal prostaglandins

~Renal azotemia

~Decreased tubular reabsorption}

A newborn failed to take his first breath. Autopsy revealed that despite unobstructed airways the lungs
of the newborn were unable to stretch. What is the most likely cause of this condition? {=Absence of
surfactant

~Bronchial narrowing

~Bronchial rupture

~Pleural thickening

~Alveolar enlargement}

A 30-year-old woman complains of intense thirst and dryness of the mouth that developed after a
severe emotional shock. Laboratory analysis revealed increase of the patient’s blood sugar level up to
10 mmol/L. What endocrine gland is affected in the patient?

{=Pancreas

~Thyroid gland

~Gonads

~Adrenal glands
~Pineal gland}

Due to trauma the patient’s parathyroid glands have been removed, which resulted in inertness,
thirst, sharp increase of neuromuscular excitability. Metabolism of the following substance is
disturbed: {=Calcium

~Manganese

~Chlorine

~Molybdenum

~Zinc}

A patient presents with high content of vasopressin (antidiuretic hormone) in the blood. What changes
in the patient’s diuresis will occur?

{=Oliguria

~Polyuria

~Anuria
~Glycosuria

~Natriuria}

A patient presents with osteoporosis; hypercalcemia and hypophosphatemia are observed in the
patient’s blood. What is the cause of this condition?

{=Increased parathormone secretion

~Increased thyroxin secretion

~Inhibited parathormone secretion

~Increased corticosteroid secretion

~Inhibited corticosteroid secretion}

Detoxification of bilirubin occurs in the membranes of endoplasmic reticulum of hepatocytes. Bilirubin is


secreted by hepatocytes into bile for the most part as:

{=Bilirubin diglucuronide

~Unconjugated bilirubin

~Bilirubin monoglucuronide

~Indirect reacting bilirubin

~-}

In the course of experiment the vagus nerve of the test animal was severed, which resulted in the
animal developing constant tachycardia. What effect of parasympathetic nervous system on cardiac
performance is demonstrated by this experiment?

{=Inhibition

~Stimulation

~Stimulus summation

~Paradoxical response
~Mixed effect}

A victim of a traffic accident has lost thoracic respiration but retaines diaphragmal. The spinal cord is
most likely to be damaged at:

{=VI-VIl cervical segments

~I-Il cervical segments

~XI-XIl cervical segments


~|-II lumbar segments

~|-Il sacral segments}

A 50-year-old man declined anaesthesia during dental manipulations. Due to severe pain he developed
anuria caused by acute increase in production of:

{=Adrenaline

“Renin

~Thymosin

~Thyroxin

~Glucagon}

After spinal trauma the patient presents with absence of voluntary movements and tendon reflexes;
sensitivity is retained only in the lower extremities. What is the mechanism of these disturbances and
what part of the spine was injured?

{=Spinal shock, thoracic spine

~Spinal shock, cervical spine

~Peripheral paralysis, cervical spine

~Central paralysis, coccyx

~-}
A man has developed downturning mouth and smoothed out nasolabial fold due to influenza
complication. What nerve is damaged?

{=Facial nerve

~Maxillary nerve

~Mandibular nerve

~Trochlear nerve

~Oculomotor nerve}

A 20-year-old woman came to the doctor with complaints of general weight loss, loss of appetite,
weakness, skin discoloration resembling bronze tan. In addtition to hyperpigmentation, examination in
the hospital revealed bilateral adrenal tuberculosis. What substance leads to skin hyperpigmentation,
when accumulated excessively?

{=Melanin

~Bilirubin
~Hemozoin

~Lipofuscin

~Adrenochrome}

Erythrocytes of the patient with hemolytic anemia present with significant decrease of pyruvate kinase
activity. What methabolic process is disturbed in this case?

{=Glycolysis

~Glycogenolysis

~Gluconeogenesis

~Pentose-phosphate pathway of glucose oxidation ~Glycogen


synthesis}

When determining comparative tissue radiosensitivity, it was revealed that different tissues have
different level of sensitivity toward ionizing radiation. What tissue of those listed below is the most
radiosensitive? {=Hematopoietic

~Cartilaginous

~Bone

~Muscular
~Nerve}

A 28-year-old patient complains of frequent gingival hemorrhages. Blood test revealed the clotting
factor (prothrombin) deficiency. What phase of blood coagulation is impaired in this patient?
{=Thrombin generation

~Vascular-platelet haemostasis

~Clot retraction

~Fibrinolysis

~-}

Increased stumulation rate of isolated heart of a rabbit leads to incomplete relaxation of the heart
ventricles due to:

{=Calcium accumulation in cardiomyocytes

~Increased sodium content in cardiomyocytes

~Inhibition of K - Na pump

~Increased potassium content in cardiomyocytes


~Increased potassium content in the interstitial tissue}

A 25-year-old man has lost all sensitivity due to damage of his peripheral nerves. Name this disorder:
{=Anesthesia

~Ataxia

~Hypoesthesia

~Hyperesthesia
~-}

A 60-year-old woman with hepatocirrhosis developed hemorrhagic syndrome. What mechanism leads
to the development of this condition?

{=Decreased synthesis of prothrombin and fibrinogen

~Increased portal venous pressure

~Deceased blood oncotic pressure

~Reduction of hepatic glycogen stores

~Emergence of neurotoxins in the blood}

Dopamine precursor - dioxyphenylalanine (DOPA) - is used in treatment of Parkinson’s disease. This


active substance is produced from the following amino acid:

{=Tyrosine

~Alanine

~Cysteine

“Histidine

~Tryptophan}

An 84-year-old patient suffers from parkinsonism. One of the pathogenetic development elements of
this disease is deficiency of a certain mediator in some of the brain structures. Name this mediator:
{=Dopamine

~Adrenaline

~Noradrenaline

~Histamine

~Acetylcholine}

A tumor is detected in one of the regions of the patient’s brain, resulting in the patient’s inability to
maintain normal body temperature. What brain structure is damaged?
{=Hypothalamus

~Thalamus

~Cerebellum

~Striatum

~Substantia nigra}

An experiment was conducted to measure the threshold of tactile receptors stimulation with various
stimuli. What stimulus will have the lowest threshold?

{=Mechanical stimulus

~Chemical stimulus

~Photic stimulus

~Cold stimulus

~Heat stimulus}

Miners’ work at the coal-face often leads to development of anthracosis. What type of respiratory
failure arises along with this disease?

{=Restrictive

~Obstructive

~Dysregulatory

~Thoracic

~Diaphragmatic}

During ultrasound a patient with atherosclerosis was diagnosed with bilateral stenosis of the renal
arteries. Specify the bioactive substance that is the key pathogenetic link in the development of arterial
hypertension in this case:

{=Renin

~Adrenaline

~Vasopressin

~Cortisol

~Thyroxin}

A 72-year-old man with hepatocirrhosis developed hepatic coma. Its development is caused by the
substances, that are being neutralized in the liver, entering into general circulation through portacaval
shunts (portal hypertension syndrome) and necrosis of hepatic cells. What type of hepatic coma is
characterized by these presentations?

{=Mixed

~Parenchymatous

~Shunt

~Hepatocellular
~Ketoacidotic}

After a traffic accident a man presents with severe blood loss, consciousness disturbance, low blood
pressure, as well as compensatory activation of the reninangiotensin system, which results in:

{=Hyperproduction of aldosterone

~Increased blood coagulation

~Intensification of erythropoiesis

~Hyperproduction of vasopressin

~Intensification of heart contractions}

Laboratory analysis revealed UDPglucuronyl transferase deficiency in the patient. What blood values can
confirm this enzymopathy?

{=Hyperbilirubinemia
~Indicanuria

~Phenylketonuria

~Ketoacidosis

~Uremia}

Autopsy ofa 72-year-old man with recurrent transmural myocardial infarction revealed his epicardium
and pericardium membranes to be swollen, thickened, coarce, as if covered in hair. Name the type of
inflammation that occurred in the cardiac membranes:

{=Croupous

~Diphtheritic

~Serous

~Suppurative
~Catarrhal}

A patient came to the doctor with complaints of general weakness and sleep disturbances.
Objectively the patient’s skin is yellow. In blood there is increased concentration of direct bilirubin and
bile acids. Acholic stool is observed. What condition can be characterized by these changes?
{=Mechanical jaundice

~Hemolytic jaundice

~Parenchymatous jaundice

~Familial nonhemolytic (Gilbert’s) syndrome

~Chronic cholecystitis}

Ina patient viral hepatitis complicated acute hepatic insufficiency, which transfered to coma. By the
main chain in pathogenesis of hepatic coma is increase in blood neurotoxic substances, at first all:
{=Ammonia
~Phenol

~Indole

~Scatole

~Acetoin}

Ina child 12 years after the carried hepatitis the signs of anemia appeared as a result of
{=Depositing of iron disorder

“Transport of iron disorder

~Enhance excretion of iron with bile

~Insufficient absorption of iron in bowels

~Formation erythropoietin disorder}

Ina child with hemolytic disease of the new born developed encephalopathy. Increase of what matter in
blood caused lesion of CNS? {=Unconjucated bilirubin

~Complex bilirubin-albumin

~Conjucated bilirubin

~Verdoglobin

~Biliverdin}

Ina children 7 years on the base of the developed hepatitis B resistance to the staphylococcus infection.
sharply decreased as a result of disorder by Kupffer’s cells {=Phagocytosis
~Pinocytosis

~Chemotaxis

~Apoptosis

~Production of cytokines}

In a dog was removed liver. After few hours it died from

{=Hypoglycemic coma

~Hepatic coma

~Ketoacidotic coma

~Lactatacidotic coma}

In a man 38-year-old is observed jaundiced skin, anemia, spleen increased, hyperbilirubinemia at the
expense of unconjugated bilirubin., stercobilirubinuris, hypercholic dark feces. What state the most
characterized that changes for?

{=Hemolytic jaundice

~Mechanical
jaundice ~Hepatic
jaundice
~Gilber’s syndrome

~Hepatic insufficiency syndrome}

A patient complains of general weakness, boring pain in the abdomen, bad appetite, suspicion on
jaundice. Blood serum contains 77.3 mcml/L of total bilirubin and 70.76 mcml/L of conjugative bilirubin.
What is the most possible type of jaundice?

{=Mechanical jaundice

~Acute hepatitis

~Hepatic cirrhosis

~Parenchymatous jaundice

~Hemolytic jaundice}

Flabby contraction of gall bladder was revealed in a woman aged 55 after introducing some of vegetable
oil into duodenum. What hormone insufficiency with such state?
{=Cholecystokinin

~Enterogastrin

~VIP

~Pancreozymin

~Gastrin}

A patient aged 25 has a diagnosis of chronic hepatitis. A patient has lost 10 kg of his body weight for 2
months. Objectively: the skin is dry, desquamative, and pale with yellowish color, small punctate
hemorrhages on the skin, stomatorrhagia. The impairment of what hepatic function do petechial
hemorrhage and stomatorrhagia prove?

{=Albumin synthetic

~Ekromogenic

~Detoxicative

~Depositing

~Glycogen synthetic}

On examination bile congestion in the liver and cholelitiasis were revealed in a patient. Point out the
main component of cholelitiasis in this state:

{=Cholesterol

“Triglycerides

~Protein

~Calcium bilirubinate

~Mineral salts}

What kind of jaundice is characterized by increased amount of direct bilirubin in the blood, appearance
of bilirubin in urine, acholic stool?

{=Obstructive

~Hemolytic

~Parenchymatous

~-}
Residual nitrogen and urea were determined in the patient’s blood analysis. The amount of urea in the
residual nitrogen is considerably reduced. The disease of what organ is characterized by this analysis?
{=Liver

~Kidneys

~Stomach

~Heart

~Intestine}

In 70’s the scientists determined that the cause of severe jaundice in newborns was the impairment of
connection of bilirubin in hepatocytes. What substance is used for the formation of conjugate?
{=Glucoronic acid

~Pyruvic acid

~Uric acid

~Sulphuric acid

~Lactic acid}

A man aged 38 with ecteric skin has anemia, enlarged spleen, hyperbilirubinemia, urobilinuria,
hypercholic stool. What condition are these changes typical for?

{=Suprahepatogenous jaundice

~Subhepatogenous jaundice

~Cellular-hepatogenous jaundice
~Gilbert’s syndrome

~Syndrome of hepatic insufficiency}

It’s determined that a patient with jaundice has increased amount of total bilirubin instead of indirect
one (free) in blood plasma, high content of stercolbilin in stool and urine, the level of direct (connected)
bilirubin in blood plasma is normal. What kind of jaundice is it?

{=Hemolytic

~Icterus of new borns

~Parenchymatous (hepatic)

~Mechanical

~Gilbert’s disease}
In inflammatory process colloidal properties of bile are impaired in gall bladder and this results in the
formation of gall stones. What substance crystallization is the main cause of the formation?
{=Cholesterol
~Urate

~Chloride

~Oxalate

~Phosphate}

After an accident a completely crashed man’s liver was removed.what disorders do of hepatic absence
may cause death during the first hours after operation?

{=Hypoglycemia

~Intoxication

~Fall of AP

~Sharp ascites

~Hemophilia and hemorrhage}

Marked isoosmotic hyperhydration has developed in a patient with hepatocirrhosis. What is the leading
mechanism of dyshydria development? {=Secondary aldosteronism

~Growth of wall capillary permeability

~Rushyer-petrovsky reflex

~Cardio-vascular insufficiency

~Hypoproteinemia}

On the background of pain in the right hypochondrium and yellowness ‘a patient with hepatic cirrhosis
has constant dyspeptic disorders in a kind of bitter taste in the mouth, feeling of heaviness in epigatric
area, nausea, unstable stool, steatorrhea. What is the main cause of the described disorders?
{=Hypocholia and intoxication

~Hypoglycemia

~Increase of stercobilin

~Hyperbilirubinemia

~Hypoproteinemia}
There is increase of indole amount and decrease of indican 1 in the patient’s urine. This indicate the
impairment of

{=Detoxication function of liver

~Filtering function of kidneys

~Reabsorptive funtion of kidney

~Albumin synthetic function of liver

~Secretory function of pancreas}

Which of the factors plays the leading role in the development of encephalopathy in hepatic
insufficiency? {=Increase of concentration of toxic substances in the blood

~Hyperaldosteronism

~Hyperbilirubinemia

~Hypofibrinogenia

~Hypoproteinemia}

A patient admitted to the hospital has clearly marked widened subcutaneous veins in the area of
umbilicus (“the head of medusa”). Which of the large venous vessels has the impaired passage? {=V.
porta
~V. renalis

~V. iliaca inferior

~V. mesenterica superior

~V. mesenterica inferion}

In coprologic investigation it is determined that stool is colorless; there are drops of neutral fat in it. The
most possible case of this is impairment of:

{=Entering the bile into intestine

~Secretion of intestine juice

~Acidity of gastric juice

~Processes of absorption in the intestine

~Secretion of pancreatic juice}


A patient had nausea and malaise after taking fatty foodstuffs. Sings of steatorrhea developed in this
patient some time later. Content of cholesterol in patient’s blood is 9.2 mmol/L. This condition results
from deficiency of:

{=Bile acids

~Triglycerids

~Chylomicrones

~Phospholipids

~Lipase}

Low level of albumins and fibrinogen were revealed in the patient’s blood. The decreased activity of
what hepatocyte organells cause this phenomenon?

{=Granular endoplasmic network

~Mitochondria

~Agranular endoplasmic network


~Goldgi’s complex

~Lysosoms}

A patient was admitted into a clinic with signs of acute alcohol poisoning. What changes of carbohydrate
metabolism are typical for this condition?

{=The rate of gluconeogenesis decreases in the liver.

~Glycogen decomposition increases in the liver

~Aerobic decomposition of glucose increases in the muscles

~Anaerobic decomposition of glucose predominates in the muscles.

~Gluconeogenesis increases in the liver}

A patient has a diagnosis of ascites. There are no edemas in the other parts of the body. There are large
vessels of cyanotic color on the abdomen. What pathology has this patient?

{=Portal hypertension

~Hypertension of pulmonary circulation

~Essential hypertension

~Chronic circulatory insufficiency


~Hepatic hypertension}

The development of acute pancreatitis in a patient is accompanied by the impairment of permeability of


common bile duct. What pathologic process may this result in?

{=Mechanical jaundice

~Parenchymatous jaundice

~Hepatic coma

~Hemolytic jaundice

~Portal hypertension}

A man aged 54 was admitted into the clinic with complaints of pains in the right hypochondrium
vomiting with blood. Objectively: enlargement of hepatic size, varicose of esophagus and stomach,
bleeding from them. The functional disorder of what vessel took place?

{=Vena porta

~Vena cave superior

~Vena hepatica

~Aorta abdominalis

~Vena cava inferior}

Increase of direct and indirect bilirubin is determined in blood of a patient with marked yellowness of
sclerae and skin. There is great amount of bilirubin and urobilin in the urine, traces of stercobilin,
decrease of stercobilin in stool. Define pathogenic type of jaundice in a patient.

{=Parenchymatous

~Hemolytic

~Mechanical

~By-pass

~Transmissible}

In a severe course of viral hepatitis, a patient has developed adynamia, sleepiness at day time and
insomnia at night, inadequate behavior (delirated ideas). There is fetor hepaticus, Kussmaul’s
respiration. What kind of metabolism impairment causes these symptoms?

{=Nitrogenous metabolism

~Water and salt metabolism and hyperhydration


~Fat metabolism

~Pigmental metabolism and hyperbilirubinemia

~Carbohydrate metabolism and hypoglycemia}

Arterial hypertension may develop in a case of hepatic insufficiency. Which of these factors causes the
increase of arterial pressure in such cases?

{=Aldosteron

~Adrenaline

~Renin

~Noradrenalin

~Angiotensin Il}

A patient was admitted to the hospital with complaints of dyspeptic disorders, melena, hemorrhoidal
bleeding. Extensions of the vessels on the anterior abdominal wall in combination with the enlargement
of size of the abdomen were revealed on examination of this patient. What pathology of gastrointestinal
tract shows these symptoms?

{=Portal hypertension

~Intestinal autointoxication

~Ulcerous disease

~Colitis

~Enteritis}

A patient complains of general weakness, breathlessness. Decrease of AP, ascites, widening of


superficial veins of anterior abdominal wall, splenomegaly were established in this patient. What
impairment of hemodynamic has this patient?

{=Portal hypertension syndrome

~Left ventricular failure

~Right ventricular failure

~Collapse

~Arterial hypotension}
A patient aged 25 is ill with jaundice, his skin has got yellow and green color, there’s skin itch, hypocholic
stool throbbing gall bladder. What is the origin of jaundice?

{=Obturative genesis

~Serum hepatitis

~Hepatic cirrhosis

~Leptospirosis

~Post-transfusion}

A patient with jaundice complains of erythrism, headache, and insomnia. Objectively: pulse-54
beats/min. AP- 90/60 mmHg. Coagulation of blood decreases. There are traces scratching on skin. The
action of what component causes these symptoms?

{=Bile acid

~Cholesterol

~Bile pigment

~Bilirubin

~Fatty acid}

Yellow color of the skin and the sclera, dark urine appeared in a patient after fungus poisoning. What
pigment causes the color of urine in the patient with hemolytic jaundice?

{=Stercobilin

~Verdoglobin

~Unconjugated bilirubin

~Monoglucoronide bilirubin

~Biliverdin}

A female patient with chronic hepatitis complains of increase sensitivity to barbiturates which she used
previously without any symptoms of intoxication. The disorder of what hepatic jaundice is responsible
for this state?

{=Metabolic

~Hemodynamic

~Bile formation
~Hemopoetic

~Phagocytotic}

Marked jaundice appeared in a patient 3 months later after the operation on his upper jaw. What
pathological process may this patient have?

{=Hepatic jaundice

~Hereditary hemolytic jaundice

~Suprahepatic jaundice

~Subhapetic jaundice

~Cholecystitis}

A female patient aged 45 was admitted to a hospital with complains of sudden pain in the abdominal
cavity, increase of temperature, and leukocytosis. Which of factors caused these changes in the
woman’s condition?
{=Gall stones

~Bacteria

~Mechanical energy

~Acids

~Viruses}

In a 38-year-old patient, who endued viral hepatitis C and is abusing alcohol, symptoms of hepatic
cirrhosis with ascites and edemas on lower extremities developed. What changes in blood composition
underlies edema development? {=Hypoalbuminemia

~Hypoglycemia

~Hypoglobulinemia

~Hypokalemia

~Hypocholesterolemia}

A 67-year-old woman, who has cholecystitis for a long time, suddenly developed sharp pain in the upper
part of abdomen, nausea, and vomiting after food intake. Acute pancreatitis was diagnosed in this
patient. What is the main link in pathogenesis of this disease? {=Preliminary activation of pancreatic
enzymes

~Decrease in enzyme levels in pancreatic juice


~Intensification of enzyme activation in duodenum

~Reduction of pancreatic polypeptide secretion

~Increased level of cholecystokinin}

A patient with obstructive jaundice presents with bradycardia, low arterial pressure, itching, irritability,
asthenia. What is the cause of these presentations?

{=Cholemia

~Anacholia

~Hypercholesterolemia

~Hypocholesterolemia

~Hyperbilirubinemia}

A newborn born to an Rh-negative mother (3rd pregnancy) presents withprogressing jaundice,


symptoms of CNS excitation, anemia. What type of jaundice is it?

{=Hemolytic

~Parenchymatous

~Obstructive

~Parasitic

~Toxic}

A patient has obstruction of the common bile duct. Which of these substances is usually found in urine
in such cases? {=Bilirubin

~Ketone bodies

~Uric acid

~Creatinine

~Glucose}

A patient with alcoholic cirrhosis complains of general weakness, dyspnea. He has been found to have
decreased blood pressure, ascites, enlargement of superficial veins of the anterior abdominal wall,
esophageal varices, splenomegaly. What hemodynamic disorder is observed in the patient?

{=Portal hypertension
~Left ventricular failure

~Right ventricular failure

~Heart failure

~Collapse}

A 71-year-old woman developed mechanical jaundice due to obstruction of the bile duct with a
chololith. Decrease of blood pressure and bradycardia are detected. These changes in functioning of the
patient’s cardiovascular system are caused by increased blood content of the following substance:
{=Bile acids

~Direct bilirubin

~Indirect bilirubin

~Urobilin

~Stercobilin}

During calculous cholecystitis attack the patient has developed the following symptoms: saponated
feces and steatorrhea. What stage of fats metabolism is disrupted according to those symptoms? {=Fat
digestion, absorption and secretion

~Fat absorption

~Intermediary metabolism of fats

~Fats metabolism in adipose tissue

~Depositing disruption}

A newborn infant has hemolytic jaundice caused by rhesus incompatibility. What bile pigment will be
concentrated highest in the blood of this infant?

{=Unconjugated bilirubin

~Conjugated bilirubin

~Urobilinogen

~Stercobilinogen

~Bile acids}

The patient with alcoholic cirrhosis complains of general weakness and dyspnea. The following is
revealed: decrease of arterial pressure, ascites, dilation of stomach anterior wall superficial veins,
esophageal varicose veins dilatation, splenomegaly. What haemodynamics disorder does the patient
uffer from? {=Portal hypertension

~Left ventricular failure

~Right ventricular failure

~Cardiac insufficiency

~Collapse}

The patient with mushroom poisoning has developed the following symptoms: yellow coloring of skin
and sclera, dark-colored urine. Hemolytic jaundice was diagnosed. What pigment causes such coloring
of the patient’s urine? {=Stercobilin

~Conjugated bilirubin

~Biliverdin

~Unconjugated bilirubin

~Verdohemoglobin}

The patient has icteritous skin; unconjugated bilirubin content in blood is high; conjugated bilirubin in
urine is not detected. There is significant amount of urobilin in urine and stercobilin in feces. Name the
pathology characterized by given symptoms.

{=Hemolytic jaundice

~Obstructive jaundice

~Jaundice of the newborn

~Hepatocellular jaundice

~Atherosclerosis}

A newborn child born from Rhnegative mother in the result of her third pregnancy presents with
gradually worsening jaundice, irritated central nervous system, anemia. What type of jaundice does the
infant suffer from? {=Hemolytic

~Hepatocellular

~Obstructive

~Parasitic

~Toxic}
The patient with alcoholic cirrhosis complains of general weakness and dyspnea. The following is
revealed: decrease of arterial pressure, ascites, dilation of the superficial veins of the stomach anterior
wall, esophageal varicose veins dilatation, splenomegaly. What haemodynamics disorder does the
patient suffer from? {=Portal hypertension

~Left ventricular failure

~Right ventricular failure

~Cardiac insufficiency

~Collapse}

Inhibition of the synthesis of bile acids from cholesterol in liver of an experimental animals has caused
maldigestion of lipids. What is the role of these acids in the enteral lipidic metabolism? {=They emulsify
dietary lipids

~They keep balance of alkaline environment in the intestines

~They participate in the synthesis of lipids

~They are part of LDL

~They activate the formation of chylomicrons}

The patient has icteric skin; unconjugated bilirubin content in blood is high; conjugated bilirubin in urine
is not detected. There is significant amount of urobilin in urine and stercobilin in feces. Name the
pathology characterized by the given symptoms:

{=Hemolytic jaundice

~Obstructive jaundice

~Jaundice of the newborn

~Hepatocellular jaundice

~Atherosclerosis}

A patient has icteric skin; unconjugated bilirubin content in blood is high; conjugated bilirubin in urine is
not detected. There is significant amount of urobilin in urine and stercobilin in feces. Name the
pathology characterized by given symptoms:

{=Hemolytic jaundice

~Obstructive jaundice

~Jaundice of the newborn

~Hepatocellular jaundice
~Atherosclerosis}

A woman complains of nausea, vomiting, skin itch. She was diagnosed with mechanical jaundice. What
is the possible cause of skin itch in such a condition? {=Bile acids accumulating in the blood

~Increased blood content of indirect bilirubin

~Cholesterol accumulating in the blood

~Direct bilirubin appearing in the blood

~Erythrocyte disintegration products accumulating in the blood}

A patient presents with icteric sclera and mucous tunics; urine is dark; feces are light-colored. Blood
content of direct and indirect bilirubin is increased, urine content of direct bilirubin is increased. What
pathology can be characterized by these signs?

{=Obstructive jaundice

~Hemolytic jaundice

~Hepatocellular jaundice

~Jaundice of the newborn

~Atherosclerosis}

A patient has a gallstone lodged in the common bile duct, which blocks bile supply to the intestine.
What digestive process will be disturbed in this case?

{=Fat digestion

~Protein absorption

~Carbohydrate digestion

~Carbohydrate absorption

~Protein digestion}

On examination the patient’s sclera and oral mucosa are icteric. What biochemical blood value can be
expected to be increased?

{=Bilirubin

~Amylase

~Glucose
~Albumin}

After taking a fatty food a patient feels nausea, flaccidity, later the sign of steathorrhea has appeared,
cholesterol in the blood 9,2ml/c. The cause of this state is the deficiency of: {=Bile acids

~Triglyceroides

~Chylomicrones

~Fatty acids

~Phopholipids}

A patient aged 45 had the diagnosis ulcer of the stomach. On examination of secretory function of the
stomach it was determined that the amount of basal secretion was 100mole/hr, acidity of basal
secretion -6O0mml/hr. What factors action contributes to the hypersecretion in the stomach? {=Gastrine

~Pancreatic polypeptide

~Somatostatin

~Glucagon

~Betaendorphin}

On laboratory examination increased amount of diastase in the urine and also a large amount of
undigested fat in stool were revealed in a patient female with complain of circular character pain in
epigastric area. What form of gastrointestinal tract pathology are described signs typical for?

{=Acute pancreatitis

~Acute appendicitis

~Infectious gastritis

~Ulcerous disease of the stomach

~Inflammation of large intestine}

A part of patient pancreas was resected. What kinds of product must be limited in his
diet? {=Fatty and fried meal

~Fruits

~Not fatty boiled meat

~Sour milk product


~Vegetables}

What enzyme deficiency is the cause of maldigestion of fats in the gastrointestinal tract and increase of
neutral fats in the stool? {=Pancreatic juice

~Intestinal
lipase
~Hepatic
lipase

~Enterokinase

~Gastric lipase}

Decrease of synthesis and secretion of trypsin is observed in chronic pancreatitis. The splitting of what
substances is broken?

{=Proteins

~Nucleic acids

~Polysaccharides

~Lipids

~Liposoluble vitamins}

In coprologic investigation it is determined that stool is discoloured, there are drops of neutral fat in it.
The most possible cause of this is the impairment of?

{=Entering of bile into the intestine

~Process of absorption in the intestine

~Secretion of intestinal juice

~Acidity of gastric juice

~Secretion of pancreatic juice}

A patient had been taking antibiotics of a wide spectrum of action for a long period of time that caused
decrease of appetite, nausea, and diarrhea with saprogenic smell. What is the side effect of treatment?
{=Dysbacteriosis
~Allergic reaction

~Hepatotoxic action

~Nephrotoxic action

~Direct irritative action}


On examination of a patient suffering from acute pancreatitis increased amount of chylomicrons was
determined in the blood. What enzyme activity is sharply decreased in this pathology?
{=Lipoprotienelipase

~Pancreatic lipase

~Pancreatic phospholipase

“Tissue triglyceride lipase

“Tissue diglyceride lipase}

The analysis of gastric juice of an elderly man who complained of unmotivated weakness, sickness,
absence of appetite showed achylia, achlorhydrea, and presence of lactic acids and coagulated blood,
decreased of pepsin secretion. What disease causes such clinical-laboratory symptoms?

{=Cancer of the stomach

~Chronic gastritis

~Chronic pancreatitis

~Cavitary Maldigestion

~Acute gastritias}

A man with chronic hepatitis has dyspeptic disorders: decrease of appetite, nausea, unstable stool, and
steatorrhea. What is the mechanism of dyspeptic disorders in hepatic pathology? {=Hypocholea

~Hypoglycemia

~Cholalemia

~Intoxication

~Hyperbilirubemia}

Spasmodic pains in the abdomen and repeated diarrhea with mucus appeared in a healthy person 3-5
hours later after taking meals. This was preceded by nausea and momentaneous vomiting, general
weakness, loss of appetite. What is the most possible cause of the desired symptoms? {=Food
intoxication

~Chronic pancreatitis

~Enterocolitis

~Hyperacid state of the stomach

~Chronic gastritis}
A man who works at a storage battery plants complains of constant feeling of weight and periodical
spasmodic pains in the abdomen, constant retention of stool (not more often than one time per three
day). This is accompanied by frequent headaches, flaccidity, absence of appetite, and bad taste in the
mouth. What are the causes of these disorders?

{=Spastic lead colie with constipation

~Hypoacid state of the stomach

~Hyper acid state of the stomach

~Chronic pancreatitis

~Parietal maldigestion}

Due to chronic gastritis a man has the impaired structure of the mucous membrane, decreased indices
of acid formation function of the stomach. The most essential negative result of this will be the
impairment of: {=Protien digestion

~Pancreatic juice secretion

~Secretory function of small intestine

~Evacuation of chyme into duodenum

~Excretion of secretum}

A patient complains of nausea, which often ends with vomiting. These dyspeptic phenomena become
more laborious with time. What stomach function is infringed in this patient? {=Evacuation

~Motility

~Reservoir

~Incretory

~Secretory}

A patient aged 35 with ulcerous disease had a rejection of antral portion of the stomch. What
gastrointestinal hormone secretion will be impaired due to operation?

{=Gastrin

~Secritin

~Neurotensin

~Histamine
~Cholecystokinin}

150 ml of meat broth were introduced into the stomach cavity of experimental dog to a probe the
content of what substance will increase quickly in the animal’s blood? {=Gastrin

~Vasointestinal peptide

~Neurotensin

~Somatostatin

~Insulin}

A patient age 37 was admitted into a surgical department with the signs of acute pancreatitis; vomiting,
diarrhea, bradycardia, hypotension wearness, dehydration of the organism. What preparation must be
used first of all? {=Contrical

~No-spa

~Pephenazine

~Ephedrine

~Platyphyllin}

A patient has a stone in the common bile duct, which has stopped passing bile into the intestines. The
impairment of what digestive process is observed in this case?

{=Digestion of fats

~Absorption of carbohydrates

~Digestion of carbohydrates

~Absorption of proteins

~Digestion of proteins}

On fibroscopy of the stomach ulcer was revealed in antrum portion, which was associated with
dissemination of mucosa by Helicobacter pylori. The role of this agent in the formation of ulcer results
in: {=_Damage of mucous barrier
~Inhibition of mucosa regeneration

~Impairment of microcirculation in mucous

~Stimulation of HCL secretion

~Stimulation of pepsin secretion}


On laboratory examination increased amount of diastase in the urine and also a large amount of
undigested fat in stool were revealed in a patient with complains of circular character pains in epigastric
area. What form of gastrointestinal tract pathology are the described signs typical for?

{=Acute pancreatitis

~Acute appendicitis

~Infectious hepatitis

~Ulcerous disease of the stomach

~Inflammation of the large intestine}

A patient aged 57 was admitted to a gastrointestinal department with suspicious on Zolliger-Ellison


syndrome, which was proved by sharp increase of gastrin level in the blood serum. What impairment of
secretory gastric function is the most possible? {=Hyperacid hypersecretion

~Hyperacid hyposecretion

~Hypoacid hypersecretion

~Achylia

~Hypoacid hyposecretion}

A patient with signs of encephalopathy was hospitalised in neurological in- patients department and co
relation between increase of encephalopathy and substances passing from intestine into systemic blood
circulation was determined. What compounds formed in the intestine may cause endotoximia? {=Indole

~Biotin

~Acetoacetat

~Ornitin

~Buthirat}

A patient with ulcer of stomach has impairment of equilibrium between the factors of aggression and
defense. What factor contributes to the development of gastric ulcer?

{=Helicobacter pylori

~Mucin

~Bicarbonate

~Prostaglandin E2

~Prostacyclin}
After resection of duodenum a patient has developed a syndrome of duodenal in sufficiency due to the
impairment of its endocrine function with signs of cell insufficiency of APUD system. What hormone is
produced by A-cells of this portion of intestine?

{=Glucagon

~Insulin

~Seratonin

~Secrtin

~Histamine}

A patient aged 42 was admitted to the hospital with gastric hemorrhage (ulcerous disease was excluded
by a patient). Ulcer and hyperplasia of gastric mucose were revealed. What investigation is it necessary
to carry out to confirm the diagnosis of Zollinger-Ellison syndrome in this patient?

{=Definition of gastrin level in the blood.

~Tomography of pancreas

~Definition of secretin level in the blood

~Definition of activity level of pancreatic enzymes

~Investigation of activity level of pancreatic secretion}

The amount of protein in the blood was examined in a patient after resection of sizeable part of small
intestine. What change of protein amount may be expected in this patient?

{=Hypoproteinemia

~Hyperproteinemia

~Paraproteinemia

~Hypergammaglobulinemia

~Dysproteinemia}

Ulcerous disease of the stomach is revealed in a woman aged 52, who has been ill with contact
dermatitis. On examination it is determined that the patient had been taking corticosteroid preparations
for a long period of time, but acidity of gastric juice was increased. What mechanism caused the
increase of gastric secretion? {=Decrease of prostaglandin content

~Increase of secretin secretion

~Gastrin hyposecretion
~Increase of histamine content

~Increase of gastrin excretion}

A patient complains of vomiting, eructation, pain in epigastric area, constipation, and emaciation. On
examination a basal secretion of HCL is 6mml/gr; maximal one is G(Omml/gr. What state has acid-forming
function of this patient?

{=Hyperacid

~Anacid

~Hypoacid

~Hypersecretory

~Normacid}

A patient aged 42 complains of pains in epigastric area, vomiting; vomitory masses have color of “coffee
grounds”, melena. In anamnesis there is ulcerous disease of the stomach. On examination: skin is pale,
heart rate-110 beats per minute; AP-90/50mm.hg. Blood analyses: erythrocytes-2.8 x 1012/I; leucocytes-
8x10°/l; hemoglobin-90gr/I. What is the possible complication that developed in the patient?
{=Hemorrhage

~Perforation

~Penetration

~Cancerous degeneration

~Pylorostenosis}

The investigation of pancreatic function was carried out in experimental animal by radioisotopic
method. The percentage of excreted with stool 1131- albumin is 53%. This is the evidence of:

{=Inhibition of exocrin function

~Increase of incretory function

~Inhibition of incretory function

~Increase of exocrine function

~Normal function}

A 67-year-old woman, who has cholecystitis for a long time, suddenly developed sharp pain in the upper
part of abdomen, nausea, and vomiting after food intake. Acute pancreatitis was diagnosed in this
patient. What is the main link in pathogenesis of this disease? {=Preliminary activation of pancreatic
enzymes

~Decrease in enzyme levels in pancreatic juice

~Intensification of enzyme activation in duodenum

~Reduction of pancreatic polypeptide secretion

~Increased level of cholecystokinin}

Malabsorption syndrome is:

{=Syndrome of intestinal malabsorption

~Syndrome of gastric malabsorption

~Syndrome of malabsorption in small intestine

~Syndrome of membraous maldigestion

~Malabsorption of proteins}

Frequent liquid stool, signs of intoxication, and dehydration developed in a newborn at 5-6 day. After
this newborn was fed by acid-milk mixtures indicated symptoms disappeared. The conclusion of innate
lactose deficiency was made. What process was broken in this newborn?

{=Membrane digestion

~Cavital digestion

~Excretory function of
intestine ~Secretory function
of pancreas

~Secretion of gastric juice}

Ulcers of stomach were found out in a patient, who was treated with glucocorticoids for a long time.
What is the main mechanism of ulcer development in this case? {=Increase in secretion and acidity of
gastric juice

~Decrease in histamine concentration in stomach wall

~Increased tonus of sympathetic nervous system

~Increased production of prostaglandins E

~Decreased tonus of parasympathetic nervous system}


A patient 62-year-old was admitted in heavy state into gastroenterological department with complaints
on bloody vomit black tarlike feces, thirst, dryness in month, dizziness. In anamnesis — ulcer biseas
Objectively: decrease of AP and blood hemoglobin. What compliation of ulcer disease is the most likely
in that case? {=Ulcer bleeding

~Malignization

~Penetration into pancreas

~Perforation

~Stenosis of pylorus}

A patient complaints on bad appetites weight loss, pain in epigastric department. For analysis of gastric
juice was revealed achyliA) What does term meaning?

{=Lack of free HCl and pepsin

~Lack of free HCl

~Lack of acidity

~Lack of free and binded HCI

~Lack of gastromucoprotein}

A patient long suffer with chronical enterocolitis. Affer use milk appeare meteorism, diarrhea, coliC) It
associated with deficit of what enzyme in intestine?

{=Lactase

~Amylase

~Glycogensynthetase

~Saccharase

~Maltase}

A patient with acute high intestinal ileus was attmited to hospital with long vomiting and decreased
arterial pressure to 60/40 mm/Hg. What mechanism of shock development in the patient is main?

{=Hypovolemia

~Exhaustion of arterias ?— adrenoreactivity

~Acidotic dilatation of metarterioles

~Chlorides loss
~Increase tone of nerve vagus}

A patient has increased gastric secretion in response to both mechanical and chemical stimuli. He has
high acidity of gastric juice with pH equals 2.0 on an empty stomach. After the breakfast pH restores to
normal in 12 minutes and is continuously dropping during following 2 hours. What type of stomach
secretion is being observed in this patient?

{=Excitable

~Brakable

~Inert

~Asthenic

~Normal}

A woman 33-year-old suffers with hepato-cerebral dystrophy (Wilson’s disease)/ In the blood —
decreased ceruloplasmin contents. In urine — sharply increased aminoacid contents. Disorder of what
process was caused these changes? {=Copper metabolism

~Glyconeogenesis

~Deamination

~Synthesis of urea

~Desintegration of tissuel proteins}

After carried plural traumas and blood loss in a victim stomach dyskinesia developed on hypokinetic
type. Motore disorders of stomach in this case arose as a result of

{=Weakening of general tone of organism

~Use of fatty food

~Appearances of the unpleasant taste feelings

~Permanent phobia for the passed

~Appetite loss}

After intensive antibioticotherapia in a patient the malabsorption syndrome developed for the following
pathogenic scheme: Neomycinum > atrophy of microvillus > ? > disorder of membrane digestion >
disorder of absorption of monomer compounds {=Deficit of membrane enzymes

~Deficit of pancreatic enzymes

~Congestion of meal in the intestine


~Activating of rotting processes

~Loss of appetite}

After removal of duodenum in a patient syndrom of duodenal insufficiency developted due to disorder
its endocrine function with phenomenon of cells deficiency of APUD-system. What from called
hormones is produced by ?-cells this part of intestinE)

{=Glucagon

~Secretin

~Serotonin

~Insulin

~Histamine}

After resection of stomach in a patient appeared complaints to common weakness, heartburn in area of
lingua,dyspnea during physical loading, paresthesias in low extremitus. What is cause is showed
disorders? {=Deficit of vitamine B12

~Deficit of iron

~Achlorhydria

~Hypoproteinemia

~Dyselectrolytemia}

After the carried hepatitis Ain a patient maintenance of free corticosteroids was increased in blood.
Likely reason of disorders? {=Deficit of transcortin

~Disorder of metabolism in the liver

~Surplus production of adrenals

~Activation of blood complement

~Liberation of peptidases from cells}

After the carried hepatitis in a patient autoimmune reaction developed as a result of functional disorder
in the liver

{=Star macrophages

~Pit cells

~Endotheliocytes
~Hepatocytes

~Fibrocytes}

After the carried hepatitis in a sick child the signs of hypovitaminosis B1 appeared as a result of
{=Disorder of formation of active forms it

~Decrease of absorption it in bowels

~Compete use by microflora

~Insufficient receipt with meal

~Forsing out by ascorbic acid}

After the resection of stomach in a patient in connection with the duodenum ulcer the malabsorption
syndrome developed, reason of which became

{=Diminishing of microvillus amount of thin bowel

~Deficit of villicrinin hormone

~Strengthening of bowel peristalsis

~Activation of fermentative processes

~Loss of electrolytes and water}

After transfered infection with diarrhea and developed malabsorption syndrome with decrease of
aminoacids and glucose adaption. The main role in pathogenesis of activity transport their belong to
ions deficit? {=Na

~K

~Ca

~Cl}

At fibroscopy of stomach was detected ulcer in antral part, which associated with contamination
Helicobacter pylori. Role this agent in ulcer formation is led to.

{=Destruction of mucous barrier

~Inhibition of mucous regeneration

~Disorder of mucous microcirculation


~Stimulation of HCl secretion

~Stimulation of pepsin secretion}

Gastric juice of a patient has decreased concentration of enzymes. What secretory cells of stomach
display disfunction?

{=Chief cells of glands

~Parietal cells of glands

~Gland mucocytes

~Cells of tegumental epithelium

~G-cells}

A patient who had been continuously treated with glucocorticoids was found to have a duodenal ulcer.
What mechanism plays a major part in its development?

{=Increase of gastric juice secretion and acidity

~Acceleration of histamine inactivation in the stomach

~Inhibition of gastrin secretion in the stomach

~Excess production of prostaglandin E

~Hyperglycemia}

During gastric secretory function research decrease of hydrochloric acid concentration in gastric juice
was detected. What enzyme will be less active in such a condition?

{=Pepsin

~Amylase

~Lipase

~Dipeptidase

~Hexokinase}

A newborn infant has hemolytic jaundice caused by rhesus incompatibility. What bile pigment will be
concentrated highest in the blood of this infant?

{=Unconjugated bilirubin

~Conjugated bilirubin
~Urobilinogen

~Stercobilinogen

~Bile acids}

The 55-year-old female patient has developed a case of acute pancreatitis caused by greasy food. What
is the main pathogenesis step of this disorder?

{=Premature activation af enzymes in gland ducts and cells

~Pancreatic juice deficiency

~Low bile production in liver

~Fats digestion disruption

~Acute bowel obstruction}

What kind of hypoxia does arise at patients with heart insufficiency in the stage of decompensation?
{=Circulative

~Hemic

~Hypoxial

~Tissueal

~Respiratory}

What kind of lipoproteins does protect the vessel wall against accumulation of cholesterol?
{=High density lipoproteins

~Chilomicrones

~Low density lipoproteins

~Very low density lipoproteins

~Phospholipids}

A child has poisoning of nitrites. What is a main mechanism in pathogenesis of this disorders?
{=Insufficiency of met-Hb-reductase

~Insufficiency of superoxyddesmutase

~Blocade of cytochromoxydase
~Insufficiency of glutationperoxydase

~Insufficiency of katalase}

A child with defect of heart was hospitalized to emergency department. At patients with the nonclosure
of the Botal’s duct develops:

{=Hypoxic hypoxia

~Mixed hypoxia

~Hemic hypoxia

~Circulatory hypoxia

~Histotoxic hypoxia}

A patient has circulatory hypoxia. What is mechanism of circulatory hypoxia:

{=Deceleration of blood stream

~Decrease of oxygen capacity of blood

~Decrease of oxygen partial pressure in air

~Inactivation of haemoglobin

~Decrease of haemoglobin}

A patient has hemic hypoxia. Hemic hypoxia is characterized by such changes of blood:
{=Anemia

~Erythrocytosis

~Leukocytosis

~Leukopenia

~Lymphocytosis}

A patient has hemic hypoxia. What is cause of this:

{=Decrease of blood oxygen capacity

~Deceleration of speed of blood stream

~Decrease of partial pressure of oxygen in air

~Poisoning by cyanides
~Disease of lungs}

A patient has hypercapnia. It is:

{=Increase pCO2

~Decrease pCO2

~Decrease pO2

~Increase pO2

~Increase pO2 and pCO2}

A girl has stenosis of pulmonary artery. The dyspnea is increased during a walk, the cyanosis appears
and he lost consciousness. What is the basic mechanism of development this state?

{=Acute brain hypoxia

~Dilatation of peripheral vessels

~Decrease of arterial pressure

~Disorders of pulmonary ventilation

~Disorders of gases diffusion in the lungs}

A patient has hypocapnia. It is:

{=Decrease pCO2

~Increase pCO2

~Decrease pO2

~Increase pO2

~Increase pO2 and pCO2}

A patient has hypoxic hypoxia. Erythrocytosis at hypoxia develops under influencing:


{=Erythropoietin

~Adrenaline

~Insulin

~Glucagon

~Lymphoquin}
A patient has hypoxic hypoxia. Hypoxia is the state when tissues:

{=Got insufficient of oxygen

~Glucose is not got

~The increased amount of oxygen is got

~The insufficient amount of hemoglobin is got

~The insufficient amount of red corpuscles is got}

A patient has hypoxic hypoxia. Hypoxic hypoxia is characterized by such changes of blood:
{=Erythrocytosis

~Anemia

~Leukocytosis

~Leukopenia

~Lymphocytosis}

A patient has massive hemorrhage. What type of hypoxia in this case?


{=Hemic

~Hypoxic

~Mixed

~Circulatory

~Tissue}

A patient has stenocardia with cardiovascular insufficiency. As a results can develops:


{=Circulatory hypoxia

~Respiratory hypoxia

~Hemic hypoxia

~Hypoxic hypoxia

~Histotoxic hypoxia}

A patient with stenocardia has cyanotic skin. This is develops as a result of increase in the blood
capillaries: {=Deoxyhemoglobin

~Carboxyhaemoglobin
~Erythrocytes

~Methemoglobin

~Carbhaemoglobin}

A patient was poisoned by carbon monooxide. In how many times the CO affinity for hemoglobin is
higher than for oxygen?

{=300
~20
~15
~10
~5}

A student executed intensive arbitrary hyperventilation. What changes will be observed in his organism?
{=Respiratory alcalosis

“Respiratory acidosis

~Hypoxemia

~Hypercapnia

~Hypoxemia and hypercapnia}

A tourist at high altitudes lost consciousness suddenly. The cause is a hypocapnia as a result of

{=Hyperventilation

~Decrease of metabolism rate

~Binding of carbonic acids by proteins

~Neutralizations of carbonic acid by bicarbonates

~Adsorption of carbonic acids by red blood cells}

Students made injection to potassium cyan to experiment frog. What pathological process can develop
in animal?

{=Histotoxic hypoxia

~Mixed hypoxia

~Hypoxic hypoxia
~Circulatory hypoxia

~Hemic hypoxia}

Alpinists forgot oxygen bulbs. During climbing up highly in mountains without oxygen bulbs in the
alpinists can develop: {=Hypoxic hypoxia

~Mixed hypoxia

~Hemic hypoxia

~Circulatory hypoxia

~Histotoxic hypoxia}

An alpinist felt dizziness and severe weakness on the height of 7 km. He lost consciousness, his
breathing stopped. These disorders were a result of

{=Surplus of CO2 is discharged from his organism

~Insufficient formation of CO2 in tissues


~Insufficient suply of organism by O2

~Insufficient utilization of O2 by tissues

~Insufficient release of O2 from oxyhemoglobin}

A patient with congenital stenosis of pulmonary artery the dyspnea increase during a walk, the cyanosis
appears and he lost consciousness. What is the basic mechanism development of this state? {=Acute
brain hypoxia
~Dilatation of peripheral vessels

~Decrease of arterial pressure

~Disorders of pulmonary ventilation

~Disorders of gases diffusion in the lungs}

A young man has nitrites poisoning. What is a main mechanism in pathogenesis of this disorder?

{=Insufficiency of met-Hb-reductase

~Insufficiency of superoxyddesmutase

~Blockade of cytochromoxydase

~Insufficiency of glutationperoxydase

~Insufficiency of catalase}
Blood oxygen capacity decrease at anemia of. As a result develops:

{=Hemic hypoxia

~Mixed hypoxia

~Hypoxic hypoxia

~Circulatory hypoxia

~Histotoxic hypoxia}

During climbing up to the mountains without oxygen bulbs in the alpinists can develop:
{=Hypoxic hypoxia

~Mixed hypoxia

~Hemic hypoxia

~Circulatory hypoxia

~Histotoxic hypoxia}

During dehermetization of airplane at height 9000m in the passengers will develop:

{=Hypoxic hypoxia

~Mixed hypoxia

~Hemic hypoxia

~Circulatory hypoxia

~Histotoxic hypoxia}

Ina patients with the nonclosed of the Botal’s duct may develop:

{=Hypoxic hypoxia

~Mixed hypoxia

~Hemic hypoxia

~Circulatory hypoxia

~Histotoxic hypoxia}

In the patient is observed violation of the external breathing. It is result of


development: {=Hypoxic hypoxia ~Mixed hypoxia
~Hemic hypoxia

~Circulatory hypoxia

~Histotoxic hypoxia}

On height 8000 m was depressurization of airplane. Passengers will have:

{=Hypoxic hypoxia

~Mixed hypoxia

~Hemic hypoxia

~Circulatory hypoxia

~Histotoxic hypoxia}

Patient has bronchial asthma. For this disease is typical:

{=Hypoxic hypoxia

~Mixed hypoxia

~Hemic hypoxia

~Circulatory hypoxia

~Histotoxic hypoxia}

Patient has pneumonia. For this disease is typical:

{=Hypoxic hypoxia

~Mixed hypoxia

~Hemic hypoxia

~Circulatory hypoxia

~Histotoxic hypoxia}

Students study some pathological process in class. Pathological process which develops as a result of
low supply to tissues of oxygen or violation of the use by it tissues is named: {=Hypoxia

~Hypercapnia

~Bradypnea

~Tachypnea
~Polypnea}

The alpinists respire of rarefied air. The decrease of partial pressure of oxygen in air will result of
development:

{=Hypoxic hypoxia

~Mixed hypoxia

~Hemic hypoxia

~Circulatory hypoxia

~Histotoxic hypoxia}

The child has foramen interventriculare. Mixing of arterial and vein blood will result of development:

{=Hypoxic hypoxia

~Mixed hypoxia

~Hemic hypoxia

~Circulatory hypoxia

~Histotoxic hypoxia}

The group of tourists went on mountains. In two hours after the beginning of ascent the tachicardia and
dyspnea was revealed in some tourists. What type of hypoxia do they have?

{=Hypoxic

~Hemic

“Tissue

~Circulatory

~Histotoxic}

The patient was asked to breath deep during auscultation. After 10 respiratory movements she felt
dizziness. What is the cause of this disorder?

{=Respiratory alcalosis

~Decrease concentrations of hemoglobin

~Disorder of diffusions of gases in lungs

“Respiratory acidosis
~Decrease quantity of erythrocytes}

The tourist climb on mountains. In three hours after the beginning of hike the tachicardia and dyspnea
were developed. What type of hypoxia tourist has?

{=Hypoxic

~Hemic

“Tissue

~Circulatory

~Histotoxic}

The massive bleeding developed in a patient. What type of hypoxia in this case?
{=Hemic

~Hypoxic

~Mixed

~Circulatory

~Tissue}

A patient has been hospitalized for chronic heart failure. Objectively: skin and mucous membranes are
cyanotic, the patient has tachycardia, tachypnea. What type of hypoxia has developed in the patient?
{=Circulatory

~Anemic

~Hemic

~Tissue

~Hypoxic}

A group of alpinists climbing to the top had their blood tested. The test revealed erythrocytosis and an
increase in hemoglobin rate. What type of hypoxia caused the stimulation of erythropoiesis in the bone
marrow? {=Hypoxic

~Combined

~Hemic

~Circulatory

~Tissue}
A patient has been hospitalized for chronic heart failure. Objectively: skin and mucous membranes are
cyanotic, the patient has tachycardia, tachypnea. What type of hypoxia has developed in the patient?
{=Circulatory

~Anemic

~Hemic

~Tissue

~Hypoxic}

The 55-year-old patient has been hospitalised due to chronic cardiac failure. Objectively: skin and
mucosa are cyanotic, tachycardia, tachypnea. What kind of hypoxia does the patient have? {=Circulatory

~Anemic

~Hemic

~Tissue

~Hypoxic}

The patient with acute cardiac failure has developed dyspnea, tachycardia and cyanosis during physical
exertion. Name the type of hypoxia.

{=Circulatory

“Respiratory

~Hemic

~Hypoxic

~Tissue}

A patient presents with hypoxia. What metabolic process activates when oxygen supply is insufficient?
{=Anaerobic glycolysis

~Urea cycle

~Pentose-phosphate pathway

~Oxidative decarboxylation of keto acids

~Tricarboxylic acid cycle}


Ina patient the sense of periodic “heart fading” appeared. On ECG: complexes, which are preceded by
decreased interval TP, reduced voltage of R wave, shortened RQ interval, complex QRST - normal, that is
followed by extended interval TR. What form of cardiac arrhythmia has place in this case? {=Atrial
extrasystole

~Atrial fibrillation

~Incomplete atrioventricular block

“Ventricular fibrillation

“Ventricular extrasystole}

An athlete -stayer (distance runner) during competition developed acute heart failure. What is the
reason of this pathology?

{=Volume overload of the heart

~Coronary circulation disorder

~Direct myocardial damage

~Pathology of pericardium

~Overload of the heart with increased resistance}

ECG of a women with ischemic heart disease showed heart rate 230 / minute, slightly deformed P wave,
ventricular complexes are intact. What heart rate disorder is observed in the patient? {=Paroxysmal
atrial tachycardia

~Fibrillation

~Atrial fibrillation

“Ventricular fibrillation

“Ventricular extrasystole}

A patient with essential hypertension developed myocardium hypertrophy. What is the primary
mechanism of myocardium hypertrophy? {=Hyperplasia of cardiomyocytes

~Increased number of mitochondria

~Increased myocardium capillary networks

~Increased endoplasmic reticulum

~Increased synthesis of retractive proteins}


A 55-year-old man, who has for years suffered from mitral valve insufficiency, developed acute heart
failure. Which pathophysiological type of heart failure is observed in this case?

{=Due to heart overload

~Due to hypoxic damage of the heart

~As a result of coronary heart damage

~Due to neurogenic heart damage

~As a result of acute tamponade of the heart}

A 25-year-old patient after acute endocarditis developed aortic valves insufficiency due to which left
ventricular hypertrophy emerged. After eight weeks, heart weight increased by 100% and then its
growth ceased. What stopped the growth of myocardium ? {=Decreased load per unit of muscle mass

~Coronary circulation impairment

~Deterioration of cardiomiocytes energetic supply

~Disorder of the regulatory heart supply

~Decreased protein synthesis and impairment of cardiomyocytes supply with plastic material}

In the experiment conducted on animals, which have undergone removal of adrenals, a significant delay
of potassium in the organism that led to the development of hyperkaliemia was observed. What heart
rate disorder is most likely to occur in such animals?

{=Sinus bradycardia

~Sinus tachycardia

~Atrial extrasystole

“Ventricular extrasystole

~Atrioventricular block}

While recording a patient’s ECG a doctor easily pressed with his fingers on the patient’s eyes and
continued to record ECG. Slowing down of the heart rate — bradycardia emerged. Impairment of what
heart conduction system property underlies this phenomenon?

{=Automatism

~Nervous

~Contractile
~Conductivity

~Dissociation}

While taking electrocardiogram of a patient, the periodic appearance of ventricular extrasystoles was
identified. It was found that P-waves are missing before extrasystoles. What is the most likely reason of
P-wave disappearance in this clinical situation?

{=Failure of retrograde impulse conduction through AV-node

~Blockade of impulse conduction through the atriums

~Emergence of refractory period in the ventricles

~Blockade of impulses in the sinus node

~The emergence of refractory period in atriums}

A patients with extensive myocardial infarction developed heart failure (shortness of breath began to
intensify, crepitation in the lungs appeared). What pathogenetic mechanism underlies the development
of heart failure in the patient?

{=Decrease of the mass of functioning cardiomyocytes

~Pressure overload

~Volume overload

~Acute heart tamponade

~Reperfusion myocardial damage}

In patients with coronary cardiosclerosis atriums and ventricles contract independently — each by its
own rhythm:

atrial contraction rate is about 70, ventricles contraction rate is about 35 per 1 min. What kind of
rhythm is it? {=ldioventricular

~Atrioventricular

~Slow atrial

~Alternation

~Heart rate driver migration}

A patient suffers from mitral valve insufficiency resulting in heart volume overload. What mechanism of
urgent compensation is the main one in heart volume overload?
{=Heterometric

~Homeometric

~Chronotropic

~Inotropic effect of catecholamines

~Compensatory myocardial hypertrophy}

A patient suffers from tachycardia, and dyspnea at rest. He acquires a sitting position. An attempt of the
patient to get a horizontal position leads to the development of cardiac asthma attack. What type of
circulatory decompensation developed?

{=Left ventricle

~Right ventricle

“Total

~Vascular

~Extracardial}

A 27-year-old patient periodically developed recurrent attacks of palpitations, which lasted for several
minutes. Cardiac work is rhythmic, heart contraction rate is up to 220 beats/min. What is the type of
arrhythmia? {=Paroxysmal tachycardia

~Sinus tachycardia

~Fibrillation

~Sinus arrhythmia

~Sinus extrasystole}

A patient with myocarditis developed clinical signs of cardiogenic shock. Which of the following
pathogenetic mechanisms is the leading one?

{=Impairment of the heart pump function

~Deposition blood in the organs

~Decrease of diastolic inflow to the heart

~Decrease of vascular tone

~Increase of peripheral vessels resistance}


Arrhythmia with generation of atrial contractions up to 400/ min was observed in a patient with
cardiosclerosis. This was accompanied by reduced pulse rate comparatively to the heart rate.
Impairment of what heart muscle function is observed in this case?

{=Generation of an impulse and its conduction

~Automatism

~Generation of an impulse

~Contractile

~Conduction}

A patient with heart failure developed arrhythmia in the form of generation of early impulses in the His
bunch. Impairment of what heart muscle function is observed in this case?

{=Generation of an impulse

~Automatism

~Conduction

~Generation of an impulse and its conduction

~Contractile}

Exstrasystole was identified in a patient. On ECG: absent P-wave, deformed QRS-complex, complete
compensatory break is present. What kind of exstrasystole is it?

{=Ventricular

~Atrial

~Atrioventricular

~Sinus

~Extracardial}

A patient with heart failure developed arrhythmia associated with decrease of the atrial contraction
frequency to 70/min, and ventricles contraction frequency to 35 / min. Impairment of what heart
muscle function is observed in this case?

{=AV conduction

~Generation of an impulse

~Automatism
~Generation of an impulse and its conduction

~Contractile}

The analysis of ECG revealed: missing P-waves, instead of them - numerous f-waves, QRST-complexes go
in different intervals, RR differ, R-waves are of different voltage. Name the type of arrhythmia. {=Atrial

~Sinus

~Paroxysmal
tachycardia
~Sinus
tachycardia

~|dioventricular rhythm}

One of the most severe complications of myocardial infarction is cardiogenic shock, which is manifested
by which of the following signs? {=Oliguria

~Long-lasting pain

~Dyzuriya

~Tachycardia

~Acute heart failure}

A patient has congenital mitral stenosis. Which of the following processes will develop in the patient’s
heart? {=Hypertrophy

~Hypotrophy

~Anaplasia

~Atrophy

~Regeneration}

In myocardium hypertrophy the heart muscle volume is increased due to:

{=Thickening each muscle fiber

~Overgrowth of connective tissue

~Increased number of cardiomyocytes

~Buildup of adipose tissue

~Increased blood supply of the heart}


Resorption of necrotized areas of damaged heart cells in myocardial infarction is responsible for:

{=Appearance of creatinecinase in the blood

~Increased activity of alaninaminotransferase

~Slowing down of ESR

~Leukopenia

~Decrease of free fatty acids}

Ventricles contract before they are filled with blood and their contractions do not illicit pulse waves. In
this case, pulse rate is less than heart contraction rate (pulse deficit). This disorder of heart rhythm is
called: {=Fibrillation

~Sinus fibrillation

~Extrasystole

“Ventricular fibrillation

~Blockade}

A 56-year-old male addressed a doctor with complaints of pain in heart area, shortness of breath.
Objective findings: edema of the lower extremities, cyanosis of the lips. From the patient’s history: long
and steady increase of arterial pressure. What is the mechanism of heart failure in this case?

{=Overload with increased peripheral resistance

~Overload with blood volume

~Immune destruction of the myocardium

~Electrolyte-steroid cardiopathy

~Hypoxic myocardium necrosis}

Changes on ECG manifested initially by extended PQ-interval, then — by occasional loss of QRS-
complexes, later - by increasing number of missing ventricular complexes and, ultimately, - by reducing
atrial contraction rate to 70 beats/min, and the ventricles contraction rate to 35 beats/min. These
changes are characteristic of: {=Atrioventricular block

~Intraatrial blockade

~Intraventricular blockade

~Arrhythmia due to automatism disorder

~Arrhythmia due to rhythm accepting impairment}


In a patient with neurocirculatory dystonia heart rate increased up to 130 beats/min. Clinical symptoms
of organic heart lesions were not found. On pressing on the carotid sinus the heart rhythm slows down
for a short time, and then is restored. What is the cause of the dysrhythmia? {=Increased impact of
sympathetic autonomic nervous system

~Fluctuations in parasympathetic tone

~Inflammatory injury of myocardium

~Ischemic myocardial damage

~Toxic injury of myocardium}

The pulse rate of a patient who sustained cerebral haemorrhage slowed down to 50/min. Pulse is
rhythmical. What is the mechanism of such a heart rhythm disorder?

{=Irritation of nervus vagus

~Replacement of glutamic acid in the globin beta chain with serine

~Influence of sympathetic mediator

~Stretching sinoatrial node zone

~Increase of slow diastolic depolarization speed}

In a patient with arterial hypertension the pulse rate during crisis fell from 72 to 52 beats/ min. and
holds for 10 days. Intramuscular administration of 1 mg atropine led to the pulse deceleration to 16
beats / min. Which group of arrhythmias does the described heart disorder belong to?

{=Automatism disorder

~AV conduction

~Impairment rate adoption

~Atrial fibrillation

“Ventricular fibrillation}

What compensatory mechanism is triggered in conditions of heart failure caused by blood volume
overload? {=Heterometric (Frank - Starling)

~Tachycardia

~Homeometric

~Hypertrophy of myocardium
~Chronoinotropic}

What compensatory mechanism is triggered in conditions of heart failure caused by increased resistance
of blood outflow?

{=Homeometric

~Heterometric (Frank - Starling)

~Bradycardia

~Hypertrophy of myocardium

~Increased frequency of breathing rate}

What form of heart failure develops with rheumatism?

{=Mixed

~Overload

~Miocardial

~Right heart

~Left heart}

What kind of compensation in heart failure develops in conditions of hypertension?

{=Hypertrophy of myocardium

~Heterometric (Frank - Starling)

~Homeometric

~Tachycardia

~The increase of minute blood volume}

A 50-year-old patient complains of having dyspnea under a considerable physical stress, leg edemata.
Examination reveals chronic myocarditis and circulatory failure. What is the evidence of cardiac
decompensation in the patient? {=Decreased cardiac output

~Increased blood flow velocity

~Increased vascular resistance

~Decreased venous pressure


~Increased hydrostatic pressure in the}

A patient with acute myocarditis exhibits rapid fatigability, shortness of breath, edemata of legs,
hepatomegaly. Classify the type of heart failure by the mechanism of its development:

{=Myocardial

~Overload

~Compensated

~Subcompensated

~Combined}

Heart rate ofa person at rest is 40/min. What structure is the pacemaker of heart in this man?
{=Atrioventricular node

~Sinoatrial node
~His’ bundle
~His’ bundle branches

~Purkinje fibers}

A patient suffering from coronary heart disease, who had had two myocardial infarctions of left
ventricular wall, presents with bubbling breathing and dyspnea. Pulmonary auscultation reveals
numerous moist crackles. What kind of heart failure is it?

{=Left ventricular

~Right ventricular

~Compensated

~Subcompensated

~Combined}

A 46-year-old patient was found to have hyperactivity of creatine kinase in the blood serum. What
pathology can be suspected?

{=Myocardial infarction

~Acute pancreatitis

~Chronic hepatitis

~Hemolytic anemia

~Renal failure}
A 55-year-old man, who had been suffering from mitral insufficiency for many years, developed acute
heart failure. What pathophysiological type of heart failure can be observed in this case? {=Due to
cardiac overload

~Due to hypoxic damage to the heart

~Due to coronarogenic damage to the heart

~Due to neurogenic damage to the heart

~Due to acute cardiac tamponade}

The patient’s large-focal myocardial infarction is complicated with pulmonary edema. What disturbance
of cardiohemodynamics contributed to the pulmonary edema development?

{=Acute left ventricular failure

~Acute right ventricular failure

~Autoimmune myocarditis

~Cardiogenic shock

~Reperfusion syndrome}

After severe emotional strain a 53- year-old man suddenly developed acute pain in the heart area,
which irradiates to the left hand, to the neck, and under the left scapula. He noted numbness of his left
hand. His face is pale and covered in cold sweat. Nitroglycerine administration stopped the pain attack
after 10 minutes had passed. What is the most likely disease in this case?

{=Angina pectoris

~Stroke

~Myocardial infarction

~Pulmonary embolism

~Somatoform autonomic dysfunction}

Stable contraction of myofibrilla of muscle fibers takes place due to accumulation of the following ions
in the cytoplasm: {=Calcium
~Potassium

~Sodium

~Magnesium
~Hydrogen}

A male patient developed fever up to 400C, there are vomiting, diarrhea, the patient is in grave
condition. Blood osmolality is 270 mOsm/I. What disorder of water-salt metabolism is observed in the
patient? {=Hypoosmolar hypohydration
~lsoosmolar hypohydration

~Hyperosmolar hypohydration

~lsoosmolar hyperhydration

~Hypoosmolar hyperhydration}

As a result of an emergency situation (shipwreck) a man had to drink sea (salty) water. What form of
water-salt imbalance may occur in this case? {=Hyperosmolar hyperhydration

~Hypoosmolar hyperhydration

~Hypotonic hyperhydration

~lsoosmolar hyperhydration

~lsotonic hyperhydration}

The patient has been admitted to the hospital with complaints of general fatigue, headache, lumbago,
edema of face and extremities. Urine analysis revealed proteinuria, hematuria and cylindruria. What is
the main pathogenetic mechanism of edema formation during glomerulonephritis?

{=Decrease of oncotic blood pressure

~Increase of vascular permeability

~Increase of hydrodynamic blood pressure

~Hormonal disbalance

~Lymph flow disruption}

A 22-year-old man was stung by bees; the affected area became hyperemic and edematous. What is the
leading mechanism of edema development in this patient?

{=Increased permeability of the capillaries

~Decreased hydrostatic blood pressure in the capillaries

~Increased oncotic pressure of tissue fluid

~Impaired lymphatic efflux


~Reduced oncotic pressure of blood}

If the amount ofa high-molecular substance added to the sol is very small, it can not increase but
decrease its stability. This phenomenon is called:

{=Sensibilization

~Solubilization

~Mutual coagulation

~Colloidal protection

~Sol adaptation}

A child with evident hypotrophy got edemata on his lower extremities, ascites. What is the main
mechanism of pathogenesis of cachectic edema?

{=Drop of oncotic pressure of blood plasma

~Rise of hydrostatic blood pressure

~Rise of oncotic pressure of intercellular fluid

~Increased permeability of vascular wall

~Disturbance of lymph outflow}

A 22-year-old male was stung by bees, the affected region became hyperemic and edematous. What is
the leading mechanism of edema development in this patient?

{=Increased permeability of the capillaries

~Decreased hydrostatic blood pressure in the capillaries

~Increased oncotic pressure of tissue fluid

~Impaired lymphatic efflux

~Reduced oncotic pressure of blood}

Ina patient with chronic heart failure the increased hydrostatic pressure in vena cava inferior was
determined; that caused development of pathological process designated as:

{=Cardiac edema

~Liver swelling
~Renal edema

~Lymphocytic edema

~Toxic edema}

A 7-year-old boy after consumption of a seafood developed swellings and skin rash. What is the cause of
such reactions to food?

{=Increased capillary permeability

~High filter pressure

~Reduction of osmotic pressure gradient through the capillary wall

~Venules constriction

~Increased level of plasma proteins}

A patient was admitted to infectious department with complaints of extensive vomiting. What disorders
of water-salt metabolism are present in the patient?

{=Hypoosmotic dehydration
~lsoosmotic dehydration

~Hyperosmotic dehydration

~Hypoosmotic hyperhydration

~Hyperosmotic hyperhydration}

After a catastrophe people found themselves on the ocean island, where there were no fresh water.
What kind of water-salt metabolism has developed in those people?

{=Hyperosmotic hyperhydration

~lsoosmotic dehydration

~Hypoosmotic hyperhydration

~Hyperosmotic dehydration

~Hypoosmotic dehydration}

Diabetes mellitus is associated with osmotic diuresis. What kind of water-salt metabolism is observed
with this state?

{=Hyperosmotic dehydration
~lsoosmotic dehydration

~Hypoosmotic dehydration

~Hypoosmotic hyperhydration

~Hyperosmotic hyperhydration}

A patient with an infectious disease accompanied with severe vomiting and diarrhea developed
hypoosmotic dehydration. What are the reasons of this pathological condition?

{=Loss of salts

~Loss of water

~Inflammatory process

~Polydipsia

~Polifagia}

Sever vomiting in a pregnant women led to the development of hypovolemia due to significant
dehydration. The main endocrine mechanism for compensation of this state is:

{=Increase of aldosterone level

~Increase of vasopressin (ADH) level

~Increase of corticotrope hormone synthesis

~Increase of prolactine secretion

~Increased production of calcitonine}

A patient was admitted to a hospital with complaints of general weakness, headaches, pain in the
lumbar region, massive swelling of the face and limbs, accompanied by pronounced proteinuria,
leukocyturia, cylinders detected in urine analysis. What are the leading pathogenetic mechanisms of
edema?

{=Reduction of oncotic blood pressure

~Increase of vascular permeability

~Increase of hydrostatic blood pressure

~Hormonal balance disorders

~Lymph flow impairment}


An experimental animal was placed in the thermostat at the temperature of 36 oC. For a long time the
animal has presented with a significant increase in respiratory frequency. What kind of water-electrolyte
exchange disorder developed in this pathological state?

{=Hyperosmotic dehydration

~Hypoosmotic dehydration

~lsoosmotic dehydration

~Positive water balance

~Edema}

Experimental rats, that have long received only carbohydrate diet, the accumulation of water in tissues
was observed. What is the main pathogenetic mechanism in the development of edema in this case?
{=Hypooncotic

~Membranogenic

~Dysregulatory

~Lymphogenous

~Hyperosmotic}

With prolonged fasting a human develops "hungry" swellings. What are the reasons of this
phenomenon? {=Reduction of oncotic blood pressure

~Reduction of vasopressin secretion

~Increased secretion of vasopressin

~Reduction of osmotic pressure in tissues

~Increased oncotic blood pressure}

A 6-year-old girl after she ate an orange, developed edema of eyelids, lips, neck, tongue mucosa. The
girl had the reaction to oranges before, which manifested as skin rashes, and its itching. What is the
pathogenetic mechanism leading to the development of edema in the girl?

{=Increased capillary wall permeability

~Lymph flow obstruction

~Increased oncotic tissue fluid pressure

~Increased hydrostatic pressure of the blood in capillaries


~Decrease of oncotic blood pressure}

A 32-year-old woman has sustained a wasp bite. Edema and hyperemia developed at the site of the bite.
What is the primary mechanism of edema in this case?

{=Increased capillary wall permeability

~Increased hydrostatic pressure of the blood in capillaries

~Increased oncotic tissue fluid pressure

~Lymph flow obstruction

~Decrease of oncotic blood pressure}

A patient is diagnosed with mixedema. What kind of water-electrolyte exchange disorder is observed in
the patients?

{=lsoosmotic hyperhydration

~Hypoosmotic hyperhydration

~Hyperosmotic hyperhydration

~lsoosmotic hypohydration

~Hypoosmotic hypohydration}

A patient is hospitalized due to chronic heart failure. What kind of water-electrolyte exchange disorder
will be the observed in the patient? {=lsoosmotic hyperhydration

~Hypoosmotic hyperhydration

~Hyperosmotic hyperhydration

~lsoosmotic hypohydration

~Hypoosmotic hypohydration}

After a ship wrack in the ocean, sailors were deprived of fresh water. What changes in relation to
osmotic pressure and extracellular fluid volume are observed as a result of drinking salty water?

{=Volume increases, osmotic pressure increases

~Volume is reduced, osmotic pressure increases

~Volume increases, osmotic pressure decreases

~The volume does not change osmotic pressure decreases


~Volume decreases with decreasing osmotic pressure}

A patient with acute glomerulonephritis developed acute renal failure. There is total diuresis deficiency.
What kind of water-electrolyte exchange disorder will be the observed in the patient?

{=Hypoosmotic hyperhydration

~lsoosmotic hyperhydration

~Hyperosmotic hyperhydration

~lsoosmotic hypohydration

~Hypoosmotic hypohydration}

An experimental animal was given water load supported with vasopressin (ADH). What kind of
waterelectrolyte exchange disorder will be the observed in the animal?

{=Hypoosmotic hyperhydration

~lsoosmotic hyperhydration

~Hyperosmotic hyperhydration

~lsoosmotic hypohydration

~Hypoosmotic hypohydration}

A patient is diagnosed with glomerulonephritis. The increased blood pressure and swellings are
observed. What is the pathogenesis of edema in glomerulonephritis?

{=Decreased glomerular filtration

~Increased glomerular filtration

~Phlebostasis (congestion in the veins)

~Increased filtration of plasma in capillaries

~Increased capillary wall permeability}

A patient is diagnosed with liver cirrhosis, accompanied by the development of edema. What is the
leading mechanism in the development of hepatic edema?

{=Hypoproteinemia

~Hyperproteinemia
~Phlebostasis (congestion in the veins)

~Increased capillary wall permeability

~Increased filtration of plasma in capillaries}

A patient is diagnosed with angiotrophoneurosis that is accompanied by the development of edema.


What kind of edema by its etiology will be observed in the patient? {=_Neurogenic

~Inflammatory

~Allergic

~Toxic

~Lymphogenous}

A man, after he had celebrated a wedding, developed signs of poisoning: vomiting, diarrhea, and fever.
As a result of these symptoms he drank small amount of fluids. What kind of water-electrolyte exchange
disorder will be observed in the patient? {=Hyperosmotic hypohydration

~lsoosmotic hypohydration

~Hyperosmotic hypohydration

~lsoosmotic hyperhydration

~Hypoosmotic hyperhydration}

A patient is diagnosed with cancer of the stomach, accompanied by the development of edema. The
cause of edema is:

{=Hypoproteinemia

~Hyperproteinemia

~Phlebostasis (congestion in the veins)

~Increased capillary wall permeability

~Increased filtration of plasma in capillaries}

A patient was delivered to a hospital and diagnosed with Quincke’s edema. The main pathogenetic
factor in the development of allergic edema is: {=Increased capillary wall permeability

~Decreased capillary wall permeability

~Decreased oncotic blood pressure


~Increased oncotic blood pressure

~Phlebostasis (congestion in the veins)}

3 days after acute blood loss a man developed signs of hypovolemia. What kind of water-electrolyte
exchange disorder will be the observed in the patient?

{=lsoosmotic hypohydration

~Hypoosmotic hypohydration

~Hyperosmotic hypohydration

~lsoosmotic hyperhydration

~Hypoosmotic hyperhydration}

A patient has developed isoosmotic hypohydration. What reason can lead to this pathology?
{=Acute blood loss

~Diarrhea

~Vomiting

~Sweating

~Hyperventilation}

A patient has developed hypoosmotic hypohydration. What reason can lead to this pathology?
{=Vomiting

~Hypersalivation

~Acute blood loss without drinking water

~Sweating

~Hyperventilation}

An excessive amount of 0.5% solution of sodium chloride was administered to a patient. What kind of
waterelectrolyte exchange disorder will be the observed in the patient?

{=Hypoosmotic hyperhydration

~lsoosmotic hyperhydration

~Hyperosmotic hyperhydratation

~Hypoosmotic hypohydratation
~Hyperosmotic hypohydration}

A patient developed dehydration. What pathological conditions can cause dehydration?

{=Vomiting

~Excessive consumption of sodium chloride

~Hypothermia

~Institution of isotonic solutions

~All the listed above}

An excessive amount of 0.9% solution of sodium chloride was administered to a patient. What kind of
waterelectrolyte exchange disorder will be the observed in the patient?

{=lsoosmotic hyperhydration

~Hypoosmotic hyperhydration

~Hyperosmotic hyperhydratation

~Hypoosmotic hypohydratation

~Hyperosmotic hypohydration}

An excessive amount of 0.05% solution of sodium chloride was administered to a patient. What kind of
waterelectrolyte exchange disorder will be the observed in the patient?

{=Hypoosmotic hyperhydration

~lsoosmotic hyperhydration

~Hyperosmotic hyperhydratation

~Hypoosmotic hypohydratation

~Hyperosmotic hypohydration}

A man has sustained a long deprivation of drinking water. What pathological changes may result from
dehydration? {=Oliguria
~Urorrhagia

~Pulmonary edema

~Cachexia

~Vomiting}
A patient with liver cirrhosis developed edema. Disorders of which liver functions promote the
development of hepatic edema? {=Protein synthesis

~Bile-forming

~Deposition of microelements

~Deposition of macroelements

~Synthesis of urea}

After a bee sting a child developed swelling. In the pathogenesis of what kind of edema the leading role
is played by biologically active substances?

{=A
ller
gic
~C
ard
iac
“Ki
dn
ey
~Starving

~All the listed above}

A patient complains of edema of the legs that emerges in the evening. Increase of what indexes
promote development of edema? {=Hydrostatic blood pressure

~Intercellular fluid hydrostatic pressure

~Oncotic blood pressure

~Osmotic blood pressure

~All the listed above}

A child developed laryngeal edema. In the pathogenesis of what kind of edema the leading role is played
by biologically active substances?

{=Inflammatory

~Cardiac

~Renal

~Starving
~All the listed above}

A patient developed dehydration. What pathological processes can cause


dehydration? {=Burns

~Allergies

~Hypothermia

~Infection

~All the listed above}

A patient with thrombophlebitis has developed lower extremities edema in the evening. In the
pathogenesis of what kind of edema the leading role is played by increase of tissue oncotic pressure?
{=Inflammatory

~Cardiac

~Renal

~Starving

~All the listed above}

In a patient who has worked in dangerous conditions, edema began to appear. In the pathogenesis of
what kind of edema the leading role is played by increase of tissue oncotic pressure? {=Toxic

~Cardiac

~Renal

~Starving

~All the listed above}

A 34-year-old woman, who has long been on a diet, developed swellings. In the pathogenesis of what
kind of edema the leading role is played by hypoproteinemia?

{=Cachectic

~Cardiac

~Allergic

~Inflammatory

~All the listed above}


In a patient, who has long worked in a paintwork factory, swellings began to appear. In the pathogenesis
of what kind of edema the leading role is played by hypoproteinemia? {=Liver

~Cardiac

~Allergic

~Inflammatory

~All the listed above}

A 44-year-old woman, who has long been on a diet, developed swellings. In the pathogenesis of what
kind of edema the leading role is played by hypoproteinemia?

{=Starving

~Cardiac

~Allergic

~Inflammatory

~All the listed above}

A patient has developed edema. In the pathogenesis of what kind of edema the leading role is played by
increased vessel wall permeability?

{=Allergic

~Cardiac

~Cachectic

~Mixedematous

~Liver}

A patient with thrombophlebitis has developed lower extremities edema in the evening. In the
pathogenesis of what kind of edema the leading role is played by increased vessel wall permeability?
{=Inflammatory

~Cardiac

~Cachectic

~Renal

~Starving}
A patient complains of breathlessness, palpitations, swelling of the lower extremities. In the
pathogenesis of what kind of edema the leading role is played by increased hydrostatic blood pressure?
{=Cardiac

~Cachectic

~Allergic

~Inflammatory

~All the listed above}

There is a sick woman with mixedema in endocrinological clinic. What is the water-electrolyte exchange
disorder observed in this patient? {=lsoosmotic hyperhydration

~Hypoosmotic hyperhydration

~Hyperosmotic hyperhydratation

~lsoosmotic hypohydration

~Hypoosmotic hypohydratation}

A patient has been hospitalized due to chronic heart failure. What is the water-electrolyte exchange
disorder observed in this patient? {=lsoosmotic hyperhydration

~Hypoosmotic hyperhydration

~Hyperosmotic hyperhydratation

~lsoosmotic hypohydration

~Hypoosmotic hypohydratation}

A man 3 days after acute blood loss developed signs of hypovolemia. What is the water-electrolyte
exchange disorder observed in this patient? {=lsoosmotic hypohydration

~Hypoosmotic hypohydratation

~Hyperosmotic hypohydratation

~lsoosmotic hyperhydration

~Hypoosmotic hyperhydration}
A man, after he had celebrated a wedding, developed signs of food poisoning: vomiting, diarrhea, and
fever. As a result of these symptoms he drinks small amount of fluids. What kind of water-electrolyte
exchange disorder will be observed in the patient? {=Hyperosmotic hypohydration

~lsoosmotic hypohydration

~Hypoosmotic hypohydration

~lsoosmotic hyperhydration

~Hypoosmotic hyperhydration}

A patient is diagnosed with liver cirrhosis. What is the leading mechanism in the development of
ascites? {=Increased hydrostatic pressure in the portal venous system

~Increased glomerular filtration

~Phlebostasis (congestion in the veins)

~Increased filtration of plasma in capillaries

~Increased capillary wall permeability}

Stable contraction of myofibrilla of muscle fibers takes place due to accumulation of the following ions
in the cytoplasm: {=Calcium
~Potassium

~Sodium

~Magnesium

~Hydrogen}

A male patient developed fever up to 400C, there are vomiting, diarrhea, the patient is in grave
condition. Blood osmolality is 270 mOsm/I. What disorder of water-salt metabolism is observed in the
patient? {=Hypoosmolar hypohydration ~lsoosmolar hypohydration
~Hyperosmolar hypohydration

~lsoosmolar hyperhydration

~Hypoosmolar hyperhydration}

As a result of an emergency situation (shipwreck) a man had to drink sea (salty) water. What form of
water-salt imbalance may occur in this case? {=Hyperosmolar hyperhydration

~Hypoosmolar hyperhydration

~Hypotonic hyperhydration
~lsoosmolar hyperhydration

~lsotonic hyperhydration}

The patient has been admitted to the hospital with complaints of general fatigue, headache, lumbago,
edema of face and extremities. Urine analysis revealed proteinuria, hematuria and cylindruria. What is
the main pathogenetic mechanism of edema formation during glomerulonephritis?

{=Decrease of oncotic blood pressure

~Increase of vascular permeability

~Increase of hydrodynamic blood pressure

~Hormonal disbalance

~Lymph flow disruption}

A 22-year-old man was stung by bees; the affected area became hyperemic and edematous. What is the
leading mechanism of edema development in this patient?

{=Increased permeability of the capillaries

~Decreased hydrostatic blood pressure in the capillaries

~Increased oncotic pressure of tissue fluid

~Impaired lymphatic efflux

~Reduced oncotic pressure of blood}

If the amount ofa high-molecular substance added to the sol is very small, it can not increase but
decrease its stability. This phenomenon is called:

{=Sensibilization

~Solubilization

~Mutual coagulation

~Colloidal protection

~Sol adaptation}

A child with evident hypotrophy got edemata on his lower extremities, ascites. What is the main
mechanism of pathogenesis of cachectic edema?

{=Drop of oncotic pressure of blood plasma


~Rise of hydrostatic blood pressure

~Rise of oncotic pressure of intercellular fluid

~Increased permeability of vascular wall

~Disturbance of lymph outflow}

A 22-year-old male was stung by bees, the affected region became hyperemic and edematous. What is
the leading mechanism of edema development in this patient?

{=Increased permeability of the capillaries

~Decreased hydrostatic blood pressure in the capillaries

~Increased oncotic pressure of tissue fluid

~Impaired lymphatic efflux

~Reduced oncotic pressure of blood}

Flatulence, bowel spasms, abdominal pain and diarrhea often develop in some people after taking milk.
These symptoms arise in 1 - 4 hours after intake only one glass of milk. What component of milk these
symptoms develop due to?

{=Galactose

~Lactose

~Maltose

~Saccharose

~Fructose}

Newborn has been refusing food, having vomiting and diarrhea, and some time later its crystalline lens
become opaque. At examination of newborn: glucose in blood — 8.5 mmol/L and in urine — 1%. What is
the most possible diagnosis? {=Galactosemia

~Phenylketonuria

~Tyrosinosis

~Cystinuria

~Alkaptonuria}
In woman, aged 45, without symptoms of diabetes mellitus, content of glucose in blood on an empty
stomach reaches 7.5 mmol/L. What test is necessary to be performed?

{=Determination of residual nitrogen in blood

~Determination of glucose in blood on an empty stomach

~Determination of tolerance to glucose

~Determination of ketone bodies in urine

~Determination of glycosylated hemoglobin}

In man, aged 60, who is 170 cm tall and 110 kg weight, content of glucose in blood is 6.8-7.0 mmol/L.
Content of insulin in his blood is normal. Tolerance to glucose is decreased, as well as number of insulin
receptors, in this patient. What signs allow us to evaluate this diabetes mellitus as non-insulin
dependent? {=Decreased tolerance to carbohydrates

~Recurrent hyperglycemia
~Patient’s age

~Normal content of insulin in blood

~Obesity}

A 24-years-old woman complains of dryness in mouth and loss of weight in spite of good appetite. At
examination of the patient: height — 162 cm, weight — 65 kg, content of glucose in blood — 8.3 mmol/L,
and presence of glucose in urine. What disease does these symptoms characteristic for? {=Diabetes
mellitus

~Steroid diabetes

~Diabetes insipidus

~Alimentary glucosuria

~Renal diabetes}

Content of glucose in patient’s blood is: on an empty stomach — 5.65 mmol/l, in one hour after taking
sugar— 8.55 mmol/l, and in two hours after taking sugar — 4.95 mmol/I. These signs are characteristic
for:

{=Healthy person

~Person suffered from hidden diabetes mellitus

~Person suffered from non-insulin-dependent diabetes mellitus

~Person suffered from insulin-dependent diabetes mellitus


~Person suffered from thyrotoxicosis}

A man, who have been suffering from diabetes mellitus for a long time, was admitted to the hospital
because of rapid worsening of his condition: general malaise, polyuria, polydipsia, nausea and vomiting,
confusion, sleepiness. Kussmaul respiration and scent of acetone from mouth were observed in this
patient. In his urine high contents of glucose and acetone bodies were found. What is the reason for
worsening of patient’s condition?

{=Diabetic ketoacidosis

~Gas acidosis

~Heart failure

~Renal failure

~Hypoglycemic coma}

Patient, aged 26, who suffered from hypoglycemic coma resulted from insulin overdosage, was
intravenously infused with 20% solution of glucose. After this manipulation patient’s condition
improved. What process helps glucose enter the cell? {=Active transport

~Osmotic transport

~Pinocytosis

~Secretion

~Phagocytosis}

A 40-years-old man is suffering from diabetes mellitus. After he has endured tonsillitis, reinforcement of
thirst, nausea, vomiting, abdominal pain, and sleepiness develop in him. Patient’s BP is 80/45 mmHg,
pulse rate 125 bpm, and his skin is dry. Content of glucose in blood is 28 mmol/L. What complication of
diabetes mellitus appears in this patient? {=Hyperosmolar coma

~Diabetic ketoacidosis

~Hepatic coma

~Lactic acidosis

~Hypoglycemic coma}

After the break of diet (taking the easy for assimilation carbohydrates) in a woman, who has been
suffering from diabetes mellitus for a long time, general malaise and increase of blood pressure
gradually develop and hallucinations and cramps appear. Woman has dry skin and distinct signs of
dehydration. What is the reason for worsening of patient’s condition?
{=Hyperosmolar hyperglycemic coma

~Hypoglycemic coma

~Diabetic ketoacidosis

~Heart failure

“Respiratory failure}

During the experiment rat was injected with 5% alloxan solution in dose 200 mg per kg of rat weight.
What kind of pathology arises in this case?

{=Diabetes mellitus

~Acute renal failure

~Diabetes insipidus

~Arterial hypertension

~Hepatic failure}

In patient with constant hypoglycemia blood analysis does not change after injection of adrenalin. A
doctor supposes hepatic disorder. What function disorder it is?

{=Glycogen deposition

~Excretion

~Glycolysis

~Ketogenesis

~Cholesterol formation}

A woman, aged 58, was admitted to the hospital in severe condition. She has confused consciousness;
dry skin, hollow eyes, cyanosis, and scent of rotten apples from her mouth. At laboratory examination of
her: glucose in blood — 15.1 mmol/L, glucose in urine — 3.5%. What is the most possible reason for this
condition? {=Hyperglycemic coma

~Hypoglycemic coma

~Hypovolemic coma

~Uremic coma

~Anaphylactic shock}
In a woman, of 52 years old and of 125 kg weight, diabetes mellitus develops. It happens due to:

{=Decrease of number of insulin receptors

~High-affinity binding insulin to synalbumin

~Reduced cell susceptibility to insulin

~Increase activity of insulinase

~Broken insulin synthesis}

Unconscious patient was admitted to the hospital. He has Kussmaul respiration, blood pressure 80/50
mmHg, and acetone scent from his mouth. What substances accumulation in organism may lead to
these disturbances? {=Ketone bodies complex carbohydrates

~Carbonic acid

~Lactic acid

~Modified lipoproteins

~Complex carbohydrates}

In patient suffered from diabetes mellitus metabolic acidosis develops due to accumulation of ketone
bodies (beta-oxybutyric acid and acetoacetic acid). At this condition pH of arterial blood is: {=7.56 ~7.48

~7.40

~7.32

~7.66}

A patient D., 40-years-old woman, was admitted to the hospital with complaints of weakness, giddiness,
hunger, cold sweat, and cramps. At examination of the patient: distension of pupils of the eyes,
weakening of respiration, and BP is 90/50 mmHg. Biochemical analysis of her blood shows: general
bilirubin is 16.0 mcmol/L, urea is 4.7 mmol/L, creatinine is 98 mcmol/L, and glucose is 2.0 mmol/L. What
kind of coma may develop in this patient?

{=Hypoglycemic

~Hepatic

~Renal

~Hyperglycemic

~At adrenal glands deficiency}


A patient, 56 years old woman, who have been suffering from diabetes mellitus for 6 years, complains of
compressing pain behind her sternum. What mechanism of heart affection is the most possible in this
case?

{=Macroangiopathy of coronary vessels

~Microangiopathy of myocardial vessels

~Myocardial dystrophy

~Myocarditis

~Vegetative neuropathy of heart}

Content of glucose in patient’s blood is: on an empty stomach — 4.52 mmol/l, in one hour after taking
sugar— 6.23 mmol/l, and in two hours after taking sugar — 2.56 mmol/|. These signs are characteristic
for:

{=Person suffered from insulinoma

~Healthy person

~Person suffered from hidden diabetes mellitus

~Person suffered from insulin-dependent diabetes mellitus

~Person suffered from thyrotoxicosis}

A female patient, aged 24, complains of dryness in her mouth, loss of weight despite good appetite. Her
height is 157 cm her weight is 72 kg. What analysis we have to perform at fist in this patient?

{=Determination of glucose content in 24-hour urine quantity

~Urinalysis by Zemnitsky

~General urinalysis

~Determination of protein fractions in blood serum

~Coagulogram}

Diabetes mellitus develops in animals after injection to them some alloxan. What is the main
mechanism of this type of diabetes mellitus?

{=Selective damage of beta-cells of pancreatic islets

~Damage of beta- and lambda-cells of pancreatic islets

~Formation of antibodies to insulin


~Selective damage of alfa-cells of pancreatic islets

~Gluconeogenesis activation}

A patient address to a doctor with complaints of constant thirst. Hyperglycemia, polyuria and increased
content of 17-ketosteroids in urine were revealed. What is the most probable disease in this case?
{=Steroid diabetes

~Myxedema

~Glycogenosis of | type

~Insulin dependent diabetes mellitus


~Addison’s disease}

In 62-years-old woman cataract (crystalline lens dimness) develops at the diabetes mellitus background.
What process enhancement due to diabetes mellitus is the cause of cataract?

{=Glycosilation of proteins

~Lipogenesis

~Proteolysis

~Ketogenesis

~Gluconeogenesis}

A 19-years-old patient has suffered from diabetes mellitus since he was 8. He took cure irregularly. He
was admitted to the hospital in connection to diabetes ketoacidosis development. What kind of
respiration is the most possible in this condition? {=Kussmaul respiration
~Biot’s respiration

~Chane-Stocks respiration

~Inspiratory breathlessness

~Expiratory breathlessness}

A patient aged 60 has been suffering from diabetes mellitus for 18 years. He complains of cold of his
lower extremities and intermittent lameness during last years. What is the mechanism of indicated
symptoms development?

{=Macroangiopathy of lower extremities

~Hyperketonemia
~Neuropathy

~Disturbances of metabolism of muscles

~Hyperglycemia}

An adolescent aged 17 addressed to a physician in connection with enter a college. He has no


complaints. His height is 178 cm; his weight is 96 kg without essential changes during a year. He has
even distribution of subcutaneous adipose tissue. His pulse rate is 82 bpm; his BP is 115/80 mmHg. At
laboratory examination of a youth: content of glucose in blood is 8.2 mmol/L; glucosuria — 4.6 g/L. What
type of diabetes mellitus is the most probable in this patient?

{=Non-insulin-dependent diabetes mellitus with obesity

~Insulin-dependent diabetes mellitus

~Symptomatic diabetes mellitus is due to dyencephalic syndrome


~Steroid diabetes mellitus is due to Cushing’s syndrome

~Symptomatic diabetes mellitus is due to acromegaly}

Signs of fatty dystrophy of liver are revealed in 38-years-old female patient who has been suffering from
diabetes mellitus for a long time. What factor deficiency is leading in this state development? {=Lipocain

“Lecithin

~Insulin

~Glycogen

~Acetyl-CoA}

Hyperglycemia and decreased number of insulin receptors on the lipocytes are established in mice with
hereditary obesity. What is the primary mechanism of lipogenesis intensification in these animals?
{=Hyperinsulinemia

~Hypoinsulinemia

~Hyperfunction of lipocytes

~Decreased tolerance to glucose

~Increased fat deposition}

A patient with diabetes mellitus did not take the dose of insulin in time. Thus gyperglycemic coma
developed {content of glucose in patient’s blood is 50 mmol/L). What is the leading factor for such coma
development? {=Hyperosmolarity of blood plasma
~Hypokalemia

~Hypoxia

~Hyponatremia

~Acidosis}

What complication may develop when treating diabetic ketoacidosis with large doses of insulin?
{=Hypoglycemia

~Leukocytosis

~Arterial hypotension

~Arterial hypertension

~Hyperkalemia}

What is the most important mechanism of hypoglycemic coma development?

{=Carbohydrate starvation of brain

~Reinforcement of glycogenilysis

~Oppression of glycogenesis

~Oppression of gluconeogenesis

~Intensification of ketogenesis}

What is the reason for glucosuria appearance under diabetes mellitus?

{=Hyperglycemia

~Increase of renal threshold

~Rise of hexokinase activity

~Increase of glomerular membrane permeability

~Polyuria}

Ina patient with diabetes mellitus loss of consciousness and cramps were observed after insulin
injection. What is a possible result of blood test for glucose in this case?

{=5.5 mmol/L

~3.3 mmol/L

~8.0 mmol/L
~10 mmol/L

~2.5 mmol/L}

A patient was admitted to the hospital in connection to osteomyelitis of lower jaw. When laboratory
examining of the patient glucose was found in her urine. Content of glucose in her blood is normal.
What is the reason for glucosuria?
{=Insufficiency of tubular enzyme systems

~Increase of glomerular filtration

~Enhancement of tubular secretion

~Decrease of renal concentration function

~Rise of osmotic pressure in tubules}

Ina patient, who has poisoning with flodzine, glucosuria was revealed; concentration of glucose in
patient’s blood is 5.6 mmol/L. What is the mechanism of glucosuria in this case?

{=Disturbances of tubular reabsorption

~Reduction of glomerular filtration

~Increase of glomerular filtration

~Disorders of tubular secretion

~Rise of oncotic pressure of blood}

At the diabetes mellitus is developed:

{=Hyperosmolar dehydration

~lsoosmolar dehydration

~Hypoosmolar dehydration

~Hypoosmolar hyperhydration

~lsoosmolar hypohydration}

A hyperketonemic coma developed in a patient with diabetes mellitus. What type violation of acid-basic
balance he has? {=Metabolic acidosis

~Exogenous acidosis

“Respiratory acidosis

“Respiratory alkalosis
~Not respiratory alkalosis}

After an insulin injection a 45-yearold woman with a long history of diabetes mellitus has developed
weakness, paleness, palpitation, anxiety, double vision, numbness of lips and the tip of tongue. Blood
glucose is at the rate of 2,5 mmol/I. What complication has developed in the patient?

{=Hypoglycemic coma
~Hyperosmolar coma

~Hyperglycemic coma

~Hyperketonemic coma

~Uremic coma}

Food rich in carbohydrates at first increases the blood glucose and then decreases its rate due to the
insulin action. What process is activated by this hormone?

{=Synthesis of glycogen

~Gluconeogenesis

~Breakdown of glycogen

~Breakdown of proteins

~Breakdown of lipids}

After an insulin injection a 45-year old female with a long history of diabetes mellitus has developed
weakness, paleness, palpitation, anxiety, double vision, numbness of lips and the tip of tongue. Blood
glucose is at the rate of 2,5 mmol/l. What complication has developed in the patient?

{=Hypoglycemic coma
~Hyperosmolar coma

~Hyperglycemic coma

~Hyperketonemic coma

~Uremic coma}

Food rich in carbohydrates at fi rst increases the blood sugar and then decreases its rate due to the
insulin action. What process is activated by this hormone?

{=Synthesis of glycogen

~Gluconeogenesis
~Breakdown of glycogen

~Breakdown of proteins

~Breakdown of lipids}

Diabetes and starvation cause the excess production of ketone bodies that are used as an energy
source. They are produced from the following compound:

{=Acetyl-CoA

~|socitrate

“Lactate

~Malate

~Ketoglutarate}

The 49-year-old female patient suffering long-term from pancreatic diabetes has developed the
following symptoms after administering insulin: weakness, facial pallor, palpitation, anxiety, double
vision, numbness of lips and tongue apex. Glucose molar concentration in blood was 2,5 mmol/I. What
complication has developed in the patient?

{=Hypoglycemic coma
~Hyperosmolar coma

~Hyperglycemic coma

~Hyperketonemic coma

~Uremic coma}

The 13-year-old female patient having suffered from measles complains of dry mouth, thirst, body
weight loss, polyuria, her glucose concentration in blood is 16 mmol/|. What disease can be suspected?
{=Type | pancreatic diabetes

~Type Il pancreatic diabetes

~Diabetes insipidus

~Steroidogenic diabetes

~Glycogenosis}
The most severe and dangerous complication of diabetes mellitus is hypoglycemic coma that is
characterized by loss of consciousness and is lethal, unless efficient emergency treatment is received by
patient. What is the main pathogenetic component of hypoglycemic coma?
{=Carbohydrate deficiency and low energy of cerebral neurons
~Carbohydrate deficiency and low energy of myocardium cells

~Blood hyperosmia

~Noncompensated ketoacidosis}

Insulin production in B-cells involves many substances. What substance gives the main signal for insulin
synthesis when its concentration changes?

{=Glucose

~Carbon dioxide

~Heparin

~Hemoglobin

~Urea}

After a traffic accident the driver presents with increased blood glucose. What mechanism leads to
hyperglycemia in this case?

{=Sympathoadrenal system activation

~Increased production of somatotropic hormone

~Decreased production of insulin

~Decreased production of glucagon

~Decreased tone of parasympathetic nervous system}

Flatulence, bowel spasms, abdominal pain and diarrhea often develop in some people after taking milk.
These symptoms arise in 1 - 4 hours after intake only one glass of milk. What component of milk these
symptoms develop due to?

{=Galactose

~Lactose

~Maltose

~Saccharose

~Fructose}
Newborn has been refusing food, having vomiting and diarrhea, and some time later its crystalline lens
become opaque. At examination of newborn: glucose in blood — 8.5 mmol/L and in urine — 1%. What is
the most possible diagnosis? {=Galactosemia

~Phenylketonuria

~Tyrosinosis

~Cystinuria

~Alkaptonuria}

In woman, aged 45, without symptoms of diabetes mellitus, content of glucose in blood on an empty
stomach reaches 7.5 mmol/L. What test is necessary to be performed?

{=Determination of residual nitrogen in blood

~Determination of glucose in blood on an empty stomach

~Determination of tolerance to glucose

~Determination of ketone bodies in urine

~Determination of glycosylated hemoglobin}

In man, aged 60, who is 170 cm tall and 110 kg weight, content of glucose in blood is 6.8-7.0 mmol/L.
Content of insulin in his blood is normal. Tolerance to glucose is decreased, as well as number of insulin
receptors, in this patient. What signs allow us to evaluate this diabetes mellitus as non-insulin
dependent? {=Decreased tolerance to carbohydrates

~Recurrent hyperglycemia
~Patient’s age

~Normal content of insulin in blood

~Obesity}

A 24-years-old woman complains of dryness in mouth and loss of weight in spite of good appetite. At
examination of the patient: height — 162 cm, weight — 65 kg, content of glucose in blood — 8.3 mmol/L,
and presence of glucose in urine. What disease does these symptoms characteristic for? {=Diabetes
mellitus

~Steroid diabetes

~Diabetes insipidus

~Alimentary glucosuria
~Renal diabetes}

Content of glucose in patient’s blood is: on an empty stomach — 5.65 mmol/l, in one hour after taking
sugar— 8.55 mmol/l, and in two hours after taking sugar — 4.95 mmol/I. These signs are characteristic
for:

{=Healthy person

~Person suffered from hidden diabetes mellitus

~Person suffered from non-insulin-dependent diabetes mellitus

~Person suffered from insulin-dependent diabetes mellitus

~Person suffered from thyrotoxicosis}

A man, who have been suffering from diabetes mellitus for a long time, was admitted to the hospital
because of rapid worsening of his condition: general malaise, polyuria, polydipsia, nausea and vomiting,
confusion, sleepiness. Kussmaul respiration and scent of acetone from mouth were observed in this
patient. In his urine high contents of glucose and acetone bodies were found. What is the reason for
worsening of patient’s condition?

{=Diabetic ketoacidosis

~Gas acidosis

~Heart failure

~Renal failure

~Hypoglycemic coma}

Patient, aged 26, who suffered from hypoglycemic coma resulted from insulin overdosage, was
intravenously infused with 20% solution of glucose. After this manipulation patient’s condition
improved. What process helps glucose enter the cell? {=Active transport

~Osmotic transport

~Pinocytosis

~Secretion

~Phagocytosis}

A 40-years-old man is suffering from diabetes mellitus. After he has endured tonsillitis, reinforcement of
thirst, nausea, vomiting, abdominal pain, and sleepiness develop in him. Patient’s BP is 80/45 mmHg,
pulse rate 125 bpm, and his skin is dry. Content of glucose in blood is 28 mmol/L. What complication of
diabetes mellitus appears in this patient? {=Hyperosmolar coma
~Diabetic ketoacidosis

~Hepatic coma

~Lactic acidosis

~Hypoglycemic coma}

After the break of diet (taking the easy for assimilation carbohydrates) in a woman, who has been
suffering from diabetes mellitus for a long time, general malaise and increase of blood pressure
gradually develop and hallucinations and cramps appear. Woman has dry skin and distinct signs of
dehydration. What is the reason for worsening of patient’s condition?

{=Hyperosmolar hyperglycemic coma

~Hypoglycemic coma

~Diabetic ketoacidosis

~Heart failure

“Respiratory failure}

During the experiment rat was injected with 5% alloxan solution in dose 200 mg per kg of rat weight.
What kind of pathology arises in this case?

{=Diabetes mellitus

~Acute renal failure

~Diabetes insipidus

~Arterial hypertension

~Hepatic failure}

In patient with constant hypoglycemia blood analysis does not change after injection of adrenalin. A
doctor supposes hepatic disorder. What function disorder it is?

{=Glycogen deposition

~Excretion

~Glycolysis

~Ketogenesis

~Cholesterol formation}
A woman, aged 58, was admitted to the hospital in severe condition. She has confused consciousness;
dry skin, hollow eyes, cyanosis, and scent of rotten apples from her mouth. At laboratory examination of
her: glucose in blood — 15.1 mmol/L, glucose in urine — 3.5%. What is the most possible reason for this
condition? {=Hyperglycemic coma

~Hypoglycemic coma

~Hypovolemic coma

~Uremic coma

~Anaphylactic shock}

In a woman, of 52 years old and of 125 kg weight, diabetes mellitus develops. It happens due to:

{=Decrease of number of insulin receptors

~High-affinity binding insulin to synalbumin

~Reduced cell susceptibility to insulin

~Increase activity of insulinase

~Broken insulin synthesis}

Unconscious patient was admitted to the hospital. He has Kussmaul respiration, blood pressure 80/50
mmHg, and acetone scent from his mouth. What substances accumulation in organism may lead to
these disturbances? {=Ketone bodies complex carbohydrates

~Carbonic acid

~Lactic acid

~Modified lipoproteins

~Complex carbohydrates}

In patient suffered from diabetes mellitus metabolic acidosis develops due to accumulation of ketone
bodies (beta-oxybutyric acid and acetoacetic acid). At this condition pH of arterial blood is: {=7.56 ~7.48

~7.40

~7.32

~7.66}

A patient D., 40-years-old woman, was admitted to the hospital with complaints of weakness, giddiness,
hunger, cold sweat, and cramps. At examination of the patient: distension of pupils of the eyes,
weakening of respiration, and BP is 90/50 mmHg. Biochemical analysis of her blood shows: general
bilirubin is 16.0 mcmol/L, urea is 4.7 mmol/L, creatinine is 98 mcmol/L, and glucose is 2.0 mmol/L. What
kind of coma may develop in this patient?

{=Hypoglycemic

~Hepatic

~Renal

~Hyperglycemic

~At adrenal glands deficiency}

A patient, 56 years old woman, who have been suffering from diabetes mellitus for 6 years, complains of
compressing pain behind her sternum. What mechanism of heart affection is the most possible in this
case?

{=Macroangiopathy of coronary vessels

~Microangiopathy of myocardial vessels

~Myocardial dystrophy

~Myocarditis

~Vegetative neuropathy of heart}

Content of glucose in patient’s blood is: on an empty stomach — 4.52 mmol/l, in one hour after taking
sugar— 6.23 mmol/l, and in two hours after taking sugar — 2.56 mmol/|. These signs are characteristic
for:

{=Person suffered from insulinoma

~Healthy person

~Person suffered from hidden diabetes mellitus

~Person suffered from insulin-dependent diabetes mellitus

~Person suffered from thyrotoxicosis}

A female patient, aged 24, complains of dryness in her mouth, loss of weight despite good appetite. Her
height is 157 cm her weight is 72 kg. What analysis we have to perform at fist in this patient?

{=Determination of glucose content in 24-hour urine quantity

~Urinalysis by Zemnitsky

~General urinalysis
~Determination of protein fractions in blood serum

~Coagulogram}

Diabetes mellitus develops in animals after injection to them some alloxan. What is the main
mechanism of this type of diabetes mellitus?

{=Selective damage of beta-cells of pancreatic islets

~Damage of beta- and lambda-cells of pancreatic islets

~Formation of antibodies to insulin

~Selective damage of alfa-cells of pancreatic islets

~Gluconeogenesis activation}

A patient address to a doctor with complaints of constant thirst. Hyperglycemia, polyuria and increased
content of 17-ketosteroids in urine were revealed. What is the most probable disease in this case?
{=Steroid diabetes

~Myxedema

~Glycogenosis of | type

~Insulin dependent diabetes mellitus


~Addison’s disease}

In 62-years-old woman cataract (crystalline lens dimness) develops at the diabetes mellitus background.
What process enhancement due to diabetes mellitus is the cause of cataract?

{=Glycosilation of proteins

~Lipogenesis

~Proteolysis

~Ketogenesis

~Gluconeogenesis}

A 19-years-old patient has suffered from diabetes mellitus since he was 8. He took cure irregularly. He
was admitted to the hospital in connection to diabetes ketoacidosis development. What kind of
respiration is the most possible in this condition? {=Kussmaul respiration
~Biot’s respiration

~Chane-Stocks respiration
~Inspiratory breathlessness

~Expiratory breathlessness}

A patient aged 60 has been suffering from diabetes mellitus for 18 years. He complains of cold of his
lower extremities and intermittent lameness during last years. What is the mechanism of indicated
symptoms development?

{=Macroangiopathy of lower extremities

~Hyperketonemia

~Neuropathy

~Disturbances of metabolism of muscles

~Hyperglycemia}

An adolescent aged 17 addressed to a physician in connection with enter a college. He has no


complaints. His height is 178 cm; his weight is 96 kg without essential changes during a year. He has
even distribution of subcutaneous adipose tissue. His pulse rate is 82 bpm; his BP is 115/80 mmHg. At
laboratory examination of a youth: content of glucose in blood is 8.2 mmol/L; glucosuria — 4.6 g/L. What
type of diabetes mellitus is the most probable in this patient?

{=Non-insulin-dependent diabetes mellitus with obesity

~Insulin-dependent diabetes mellitus

~Symptomatic diabetes mellitus is due to dyencephalic syndrome


~Steroid diabetes mellitus is due to Cushing’s syndrome

~Symptomatic diabetes mellitus is due to acromegaly}

Signs of fatty dystrophy of liver are revealed in 38-years-old female patient who has been suffering from
diabetes mellitus for a long time. What factor deficiency is leading in this state development? {=Lipocain

“Lecithin

~Insulin

~Glycogen

~Acetyl-CoA}

Hyperglycemia and decreased number of insulin receptors on the lipocytes are established in mice with
hereditary obesity. What is the primary mechanism of lipogenesis intensification in these animals?
{=Hyperinsulinemia
~Hypoinsulinemia

~Hyperfunction of lipocytes

~Decreased tolerance to glucose

~Increased fat deposition}

A patient with diabetes mellitus did not take the dose of insulin in time. Thus gyperglycemic coma
developed {content of glucose in patient’s blood is 50 mmol/L). What is the leading factor for such coma
development? {=Hyperosmolarity of blood plasma

~Hypokalemia

~Hypoxia

~Hyponatremia

~Acidosis}

What complication may develop when treating diabetic ketoacidosis with large doses of insulin?
{=Hypoglycemia

~Leukocytosis

~Arterial hypotension

~Arterial hypertension

~Hyperkalemia}

What is the most important mechanism of hypoglycemic coma development?

{=Carbohydrate starvation of brain

~Reinforcement of glycogenilysis

~Oppression of glycogenesis

~Oppression of gluconeogenesis

~Intensification of ketogenesis}

What is the reason for glucosuria appearance under diabetes mellitus?

{=Hyperglycemia

~Increase of renal threshold

~Rise of hexokinase activity


~Increase of glomerular membrane permeability

~Polyuria}

Ina patient with diabetes mellitus loss of consciousness and cramps were observed after insulin
injection. What is a possible result of blood test for glucose in this case?

{=5.5 mmol/L

~3.3 mmol/L

~8.0 mmol/L

~10 mmol/L

~2.5 mmol/L}

A patient was admitted to the hospital in connection to osteomyelitis of lower jaw. When laboratory
examining of the patient glucose was found in her urine. Content of glucose in her blood is normal.
What is the reason for glucosuria?
{=Insufficiency of tubular enzyme systems

~Increase of glomerular filtration

~Enhancement of tubular secretion

~Decrease of renal concentration function

~Rise of osmotic pressure in tubules}

Ina patient, who has poisoning with flodzine, glucosuria was revealed; concentration of glucose in
patient’s blood is 5.6 mmol/L. What is the mechanism of glucosuria in this case?

{=Disturbances of tubular reabsorption

~Reduction of glomerular filtration

~Increase of glomerular filtration

~Disorders of tubular secretion

~Rise of oncotic pressure of blood}

At the diabetes mellitus is developed:

{=Hyperosmolar dehydration

~lsoosmolar dehydration

~Hypoosmolar dehydration
~Hypoosmolar hyperhydration

~lsoosmolar hypohydration}

A hyperketonemic coma developed in a patient with diabetes mellitus. What type violation of acid-basic
balance he has? {=Metabolic acidosis

~Exogenous acidosis

“Respiratory acidosis

“Respiratory alkalosis

~Not respiratory alkalosis}

After an insulin injection a 45-yearold woman with a long history of diabetes mellitus has developed
weakness, paleness, palpitation, anxiety, double vision, numbness of lips and the tip of tongue. Blood
glucose is at the rate of 2,5 mmol/I. What complication has developed in the patient?

{=Hypoglycemic coma
~Hyperosmolar coma

~Hyperglycemic coma

~Hyperketonemic coma

~Uremic coma}

Food rich in carbohydrates at first increases the blood glucose and then decreases its rate due to the
insulin action. What process is activated by this hormone?

{=Synthesis of glycogen

~Gluconeogenesis

~Breakdown of glycogen

~Breakdown of proteins

~Breakdown of lipids}

After an insulin injection a 45-year old female with a long history of diabetes mellitus has developed
weakness, paleness, palpitation, anxiety, double vision, numbness of lips and the tip of tongue. Blood
glucose is at the rate of 2,5 mmol/l. What complication has developed in the patient?

{=Hypoglycemic coma
~Hyperosmolar coma
~Hyperglycemic coma

~Hyperketonemic coma

~Uremic coma}

Food rich in carbohydrates at fi rst increases the blood sugar and then decreases its rate due to the
insulin action. What process is activated by this hormone?

{=Synthesis of glycogen

~Gluconeogenesis

~Breakdown of glycogen

~Breakdown of proteins

~Breakdown of lipids}

Diabetes and starvation cause the excess production of ketone bodies that are used as an energy
source. They are produced from the following compound:

{=Acetyl-CoA

~|socitrate

“Lactate

~Malate

~Ketoglutarate}

The 49-year-old female patient suffering long-term from pancreatic diabetes has developed the
following symptoms after administering insulin: weakness, facial pallor, palpitation, anxiety, double
vision, numbness of lips and tongue apex. Glucose molar concentration in blood was 2,5 mmol/I. What
complication has developed in the patient?

{=Hypoglycemic coma
~Hyperosmolar coma

~Hyperglycemic coma

~Hyperketonemic coma

~Uremic coma}
The 13-year-old female patient having suffered from measles complains of dry mouth, thirst, body
weight loss, polyuria, her glucose concentration in blood is 16 mmol/|. What disease can be suspected?
{=Type | pancreatic diabetes

~Type Il pancreatic diabetes

~Diabetes insipidus

~Steroidogenic diabetes

~Glycogenosis}

The most severe and dangerous complication of diabetes mellitus is hypoglycemic coma that is
characterized by loss of consciousness and is lethal, unless efficient emergency treatment is received by
patient. What is the main pathogenetic component of hypoglycemic coma?
{=Carbohydrate deficiency and low energy of cerebral neurons
~Carbohydrate deficiency and low energy of myocardium cells

~Blood hyperosmia

~Noncompensated ketoacidosis}

Insulin production in B-cells involves many substances. What substance gives the main signal for insulin
synthesis when its concentration changes?

{=Glucose

~Carbon dioxide

~Heparin

~Hemoglobin

~Urea}

After a traffic accident the driver presents with increased blood glucose. What mechanism leads to
hyperglycemia in this case?

{=Sympathoadrenal system activation

~Increased production of somatotropic hormone

~Decreased production of insulin

~Decreased production of glucagon

~Decreased tone of parasympathetic nervous system}


Flatulence, bowel spasms, abdominal pain and diarrhea often develop in some people after taking milk.
These symptoms arise in 1 - 4 hours after intake only one glass of milk. What component of milk these
symptoms develop due to?

{=Galactose

~Lactose

~Maltose

~Saccharose

~Fructose}

Newborn has been refusing food, having vomiting and diarrhea, and some time later its crystalline lens
become opaque. At examination of newborn: glucose in blood — 8.5 mmol/L and in urine — 1%. What is
the most possible diagnosis? {=Galactosemia

~Phenylketonuria

~Tyrosinosis

~Cystinuria

~Alkaptonuria}

In woman, aged 45, without symptoms of diabetes mellitus, content of glucose in blood on an empty
stomach reaches 7.5 mmol/L. What test is necessary to be performed?

{=Determination of residual nitrogen in blood

~Determination of glucose in blood on an empty stomach

~Determination of tolerance to glucose

~Determination of ketone bodies in urine

~Determination of glycosylated hemoglobin}

In man, aged 60, who is 170 cm tall and 110 kg weight, content of glucose in blood is 6.8-7.0 mmol/L.
Content of insulin in his blood is normal. Tolerance to glucose is decreased, as well as number of insulin
receptors, in this patient. What signs allow us to evaluate this diabetes mellitus as non-insulin
dependent? {=Decreased tolerance to carbohydrates

~Recurrent hyperglycemia
~Patient’s age

~Normal content of insulin in blood


~Obesity}

A 24-years-old woman complains of dryness in mouth and loss of weight in spite of good appetite. At
examination of the patient: height — 162 cm, weight — 65 kg, content of glucose in blood — 8.3 mmol/L,
and presence of glucose in urine. What disease does these symptoms characteristic for? {=Diabetes
mellitus

~Steroid diabetes

~Diabetes insipidus

~Alimentary glucosuria

~Renal diabetes}

Content of glucose in patient’s blood is: on an empty stomach — 5.65 mmol/l, in one hour after taking
sugar— 8.55 mmol/l, and in two hours after taking sugar — 4.95 mmol/I. These signs are characteristic
for:

{=Healthy person

~Person suffered from hidden diabetes mellitus

~Person suffered from non-insulin-dependent diabetes mellitus

~Person suffered from insulin-dependent diabetes mellitus

~Person suffered from thyrotoxicosis}

A man, who have been suffering from diabetes mellitus for a long time, was admitted to the hospital
because of rapid worsening of his condition: general malaise, polyuria, polydipsia, nausea and vomiting,
confusion, sleepiness. Kussmaul respiration and scent of acetone from mouth were observed in this
patient. In his urine high contents of glucose and acetone bodies were found. What is the reason for
worsening of patient’s condition?

{=Diabetic ketoacidosis

~Gas acidosis

~Heart failure

~Renal failure

~Hypoglycemic coma}

Patient, aged 26, who suffered from hypoglycemic coma resulted from insulin overdosage, was
intravenously infused with 20% solution of glucose. After this manipulation patient’s condition
improved. What process helps glucose enter the cell? {=Active transport
~Osmotic transport

~Pinocytosis

~Secretion

~Phagocytosis}

A 40-years-old man is suffering from diabetes mellitus. After he has endured tonsillitis, reinforcement of
thirst, nausea, vomiting, abdominal pain, and sleepiness develop in him. Patient’s BP is 80/45 mmHg,
pulse rate 125 bpm, and his skin is dry. Content of glucose in blood is 28 mmol/L. What complication of
diabetes mellitus appears in this patient? {=Hyperosmolar coma

~Diabetic ketoacidosis

~Hepatic coma

~Lactic acidosis

~Hypoglycemic coma}

After the break of diet (taking the easy for assimilation carbohydrates) in a woman, who has been
suffering from diabetes mellitus for a long time, general malaise and increase of blood pressure
gradually develop and hallucinations and cramps appear. Woman has dry skin and distinct signs of
dehydration. What is the reason for worsening of patient’s condition?

{=Hyperosmolar hyperglycemic coma

~Hypoglycemic coma

~Diabetic ketoacidosis

~Heart failure

“Respiratory failure}

During the experiment rat was injected with 5% alloxan solution in dose 200 mg per kg of rat weight.
What kind of pathology arises in this case?

{=Diabetes mellitus

~Acute renal failure

~Diabetes insipidus

~Arterial hypertension

~Hepatic failure}
In patient with constant hypoglycemia blood analysis does not change after injection of adrenalin. A
doctor supposes hepatic disorder. What function disorder it is?

{=Glycogen deposition

~Excretion

~Glycolysis

~Ketogenesis

~Cholesterol formation}

A woman, aged 58, was admitted to the hospital in severe condition. She has confused consciousness;
dry skin, hollow eyes, cyanosis, and scent of rotten apples from her mouth. At laboratory examination of
her: glucose in blood — 15.1 mmol/L, glucose in urine — 3.5%. What is the most possible reason for this
condition? {=Hyperglycemic coma

~Hypoglycemic coma

~Hypovolemic coma

~Uremic coma

~Anaphylactic shock}

In a woman, of 52 years old and of 125 kg weight, diabetes mellitus develops. It happens due to:

{=Decrease of number of insulin receptors

~High-affinity binding insulin to synalbumin

~Reduced cell susceptibility to insulin

~Increase activity of insulinase

~Broken insulin synthesis}

Unconscious patient was admitted to the hospital. He has Kussmaul respiration, blood pressure 80/50
mmHg, and acetone scent from his mouth. What substances accumulation in organism may lead to
these disturbances? {=Ketone bodies complex carbohydrates

~Carbonic acid

~Lactic acid

~Modified lipoproteins

~Complex carbohydrates}
In patient suffered from diabetes mellitus metabolic acidosis develops due to accumulation of ketone
bodies (beta-oxybutyric acid and acetoacetic acid). At this condition pH of arterial blood is: {=7.56 ~7.48

~7.40

~7.32

~7.66}

A patient D., 40-years-old woman, was admitted to the hospital with complaints of weakness, giddiness,
hunger, cold sweat, and cramps. At examination of the patient: distension of pupils of the eyes,
weakening of respiration, and BP is 90/50 mmHg. Biochemical analysis of her blood shows: general
bilirubin is 16.0 mcmol/L, urea is 4.7 mmol/L, creatinine is 98 mcmol/L, and glucose is 2.0 mmol/L. What
kind of coma may develop in this patient?

{=Hypoglycemic

~Hepatic

~Renal

~Hyperglycemic

~At adrenal glands deficiency}

A patient, 56 years old woman, who have been suffering from diabetes mellitus for 6 years, complains of
compressing pain behind her sternum. What mechanism of heart affection is the most possible in this
case?

{=Macroangiopathy of coronary vessels

~Microangiopathy of myocardial vessels

~Myocardial dystrophy

~Myocarditis

~Vegetative neuropathy of heart}

Content of glucose in patient’s blood is: on an empty stomach — 4.52 mmol/l, in one hour after taking
sugar— 6.23 mmol/l, and in two hours after taking sugar — 2.56 mmol/|. These signs are characteristic
for:

{=Person suffered from insulinoma

~Healthy person

~Person suffered from hidden diabetes mellitus


~Person suffered from insulin-dependent diabetes mellitus

~Person suffered from thyrotoxicosis}

A female patient, aged 24, complains of dryness in her mouth, loss of weight despite good appetite. Her
height is 157 cm her weight is 72 kg. What analysis we have to perform at fist in this patient?

{=Determination of glucose content in 24-hour urine quantity

~Urinalysis by Zemnitsky

~General urinalysis

~Determination of protein fractions in blood serum

~Coagulogram}

Diabetes mellitus develops in animals after injection to them some alloxan. What is the main
mechanism of this type of diabetes mellitus?

{=Selective damage of beta-cells of pancreatic islets

~Damage of beta- and lambda-cells of pancreatic islets

~Formation of antibodies to insulin

~Selective damage of alfa-cells of pancreatic islets

~Gluconeogenesis activation}

A patient address to a doctor with complaints of constant thirst. Hyperglycemia, polyuria and increased
content of 17-ketosteroids in urine were revealed. What is the most probable disease in this case?
{=Steroid diabetes

~Myxedema

~Glycogenosis of | type

~Insulin dependent diabetes mellitus


~Addison’s disease}

In 62-years-old woman cataract (crystalline lens dimness) develops at the diabetes mellitus background.
What process enhancement due to diabetes mellitus is the cause of cataract?

{=Glycosilation of proteins

~Lipogenesis

~Proteolysis
~Ketogenesis

~Gluconeogenesis}

A 19-years-old patient has suffered from diabetes mellitus since he was 8. He took cure irregularly. He
was admitted to the hospital in connection to diabetes ketoacidosis development. What kind of
respiration is the most possible in this condition? {=Kussmaul respiration
~Biot’s respiration

~Chane-Stocks respiration

~Inspiratory breathlessness

~Expiratory breathlessness}

A patient aged 60 has been suffering from diabetes mellitus for 18 years. He complains of cold of his
lower extremities and intermittent lameness during last years. What is the mechanism of indicated
symptoms development?

{=Macroangiopathy of lower extremities

~Hyperketonemia

~Neuropathy

~Disturbances of metabolism of muscles

~Hyperglycemia}

An adolescent aged 17 addressed to a physician in connection with enter a college. He has no


complaints. His height is 178 cm; his weight is 96 kg without essential changes during a year. He has
even distribution of subcutaneous adipose tissue. His pulse rate is 82 bpm; his BP is 115/80 mmHg. At
laboratory examination of a youth: content of glucose in blood is 8.2 mmol/L; glucosuria — 4.6 g/L. What
type of diabetes mellitus is the most probable in this patient?

{=Non-insulin-dependent diabetes mellitus with obesity

~Insulin-dependent diabetes mellitus

~Symptomatic diabetes mellitus is due to dyencephalic syndrome


~Steroid diabetes mellitus is due to Cushing’s syndrome

~Symptomatic diabetes mellitus is due to acromegaly}

Signs of fatty dystrophy of liver are revealed in 38-years-old female patient who has been suffering from
diabetes mellitus for a long time. What factor deficiency is leading in this state development? {=Lipocain
“Lecithin

~Insulin

~Glycogen

~Acetyl-CoA}

Hyperglycemia and decreased number of insulin receptors on the lipocytes are established in mice with
hereditary obesity. What is the primary mechanism of lipogenesis intensification in these animals?
{=Hyperinsulinemia

~Hypoinsulinemia

~Hyperfunction of lipocytes

~Decreased tolerance to glucose

~Increased fat deposition}

A patient with diabetes mellitus did not take the dose of insulin in time. Thus gyperglycemic coma
developed {content of glucose in patient’s blood is 50 mmol/L). What is the leading factor for such coma
development? {=Hyperosmolarity of blood plasma

~Hypokalemia

~Hypoxia

~Hyponatremia

~Acidosis}

What complication may develop when treating diabetic ketoacidosis with large doses of insulin?
{=Hypoglycemia

~Leukocytosis

~Arterial hypotension

~Arterial hypertension

~Hyperkalemia}

What is the most important mechanism of hypoglycemic coma development?

{=Carbohydrate starvation of brain

~Reinforcement of glycogenilysis

~Oppression of glycogenesis
~Oppression of gluconeogenesis

~Intensification of ketogenesis}

What is the reason for glucosuria appearance under diabetes mellitus?

{=Hyperglycemia

~Increase of renal threshold

~Rise of hexokinase activity

~Increase of glomerular membrane permeability

~Polyuria}

Ina patient with diabetes mellitus loss of consciousness and cramps were observed after insulin
injection. What is a possible result of blood test for glucose in this case?

{=5.5 mmol/L

~3.3 mmol/L

~8.0 mmol/L

~10 mmol/L

~2.5 mmol/L}

A patient was admitted to the hospital in connection to osteomyelitis of lower jaw. When laboratory
examining of the patient glucose was found in her urine. Content of glucose in her blood is normal.
What is the reason for glucosuria?
{=Insufficiency of tubular enzyme systems

~Increase of glomerular filtration

~Enhancement of tubular secretion

~Decrease of renal concentration function

~Rise of osmotic pressure in tubules}

Ina patient, who has poisoning with flodzine, glucosuria was revealed; concentration of glucose in
patient’s blood is 5.6 mmol/L. What is the mechanism of glucosuria in this case?

{=Disturbances of tubular reabsorption

~Reduction of glomerular filtration

~Increase of glomerular filtration


~Disorders of tubular secretion

~Rise of oncotic pressure of blood}

At the diabetes mellitus is developed:

{=Hyperosmolar dehydration

~lsoosmolar dehydration

~Hypoosmolar dehydration

~Hypoosmolar hyperhydration

~lsoosmolar hypohydration}

A hyperketonemic coma developed in a patient with diabetes mellitus. What type violation of acid-basic
balance he has? {=Metabolic acidosis

~Exogenous acidosis

“Respiratory acidosis

“Respiratory alkalosis

~Not respiratory alkalosis}

After an insulin injection a 45-yearold woman with a long history of diabetes mellitus has developed
weakness, paleness, palpitation, anxiety, double vision, numbness of lips and the tip of tongue. Blood
glucose is at the rate of 2,5 mmol/l. What complication has developed in the patient?

{=Hypoglycemic coma
~Hyperosmolar coma

~Hyperglycemic coma

~Hyperketonemic coma

~Uremic coma}

Food rich in carbohydrates at first increases the blood glucose and then decreases its rate due to the
insulin action. What process is activated by this hormone?

{=Synthesis of glycogen

~Gluconeogenesis

~Breakdown of glycogen

~Breakdown of proteins
~Breakdown of lipids}

After an insulin injection a 45-year old female with a long history of diabetes mellitus has developed
weakness, paleness, palpitation, anxiety, double vision, numbness of lips and the tip of tongue. Blood
glucose is at the rate of 2,5 mmol/l. What complication has developed in the patient?

{=Hypoglycemic coma
~Hyperosmolar coma

~Hyperglycemic coma

~Hyperketonemic coma

~Uremic coma}

Food rich in carbohydrates at fi rst increases the blood sugar and then decreases its rate due to the
insulin action. What process is activated by this hormone?

{=Synthesis of glycogen

~Gluconeogenesis

~Breakdown of glycogen

~Breakdown of proteins

~Breakdown of lipids}

Diabetes and starvation cause the excess production of ketone bodies that are used as an energy
source. They are produced from the following compound:

{=Acetyl-CoA

~|socitrate

“Lactate

~Malate

~Ketoglutarate}

The 49-year-old female patient suffering long-term from pancreatic diabetes has developed the
following symptoms after administering insulin: weakness, facial pallor, palpitation, anxiety, double
vision, numbness of lips and tongue apex. Glucose molar concentration in blood was 2,5 mmol/I. What
complication has developed in the patient?

{=Hypoglycemic coma
~Hyperosmolar coma

~Hyperglycemic coma

~Hyperketonemic coma

~Uremic coma}

The 13-year-old female patient having suffered from measles complains of dry mouth, thirst, body
weight loss, polyuria, her glucose concentration in blood is 16 mmol/|. What disease can be suspected?
{=Type | pancreatic diabetes

~Type Il pancreatic diabetes

~Diabetes insipidus

~Steroidogenic diabetes

~Glycogenosis}

The most severe and dangerous complication of diabetes mellitus is hypoglycemic coma that is
characterized by loss of consciousness and is lethal, unless efficient emergency treatment is received by
patient. What is the main pathogenetic component of hypoglycemic coma?
{=Carbohydrate deficiency and low energy of cerebral neurons
~Carbohydrate deficiency and low energy of myocardium cells

~Blood hyperosmia

~Noncompensated ketoacidosis}

Insulin production in B-cells involves many substances. What substance gives the main signal for insulin
synthesis when its concentration changes?

{=Glucose

~Carbon dioxide

~Heparin

~Hemoglobin

~Urea}

After a traffic accident the driver presents with increased blood glucose. What mechanism leads to
hyperglycemia in this case?

{=Sympathoadrenal system activation

~Increased production of somatotropic hormone


~Decreased production of insulin

~Decreased production of glucagon

~Decreased tone of parasympathetic nervous system}

In crisis period a 14 year old child ill with diphtheria has AP- 70/50 mm Hg accompanied by abrupt fall in
temperature and tachycardia. What form of vascular tone disturbance is it?

{=Acute hypotension

~Chronic hypotension

~Vegetovascular dystonia

~Essential arterial hypotension

~-}

A patient has the following diagnosis: renal hypertension. What is the initial pathogenetic factor of
arterial hypertension development in this case?

{=Renal ischemia

~Hypernatremia

~Intensified renin synthesis

~Intensified angiotensin synthesis

~Hyperaldosteronism}

Rabbits lived on food with addition of cholesterol. Five months later the atherosclerotic aorta changes
were revealed. Name the main cause of atherogenesis in this case:

{=Exogenous hypercholesterolemia

~Overeating

~Endogenous hypercholesterolemia
~

~Hypodynamia}
18-year-old patient complains of general weakness, fatigue, low spirits. The patient is of the asthenic
constitution type. Ps- 68/min., AP- 90/60 mm Hg. She has been found to have primary neurocirculatory
hypotension. What is the leading factor of the arterial pressure drop in this patient?

{=Decreased cardiac output

~Decreased tonus of resistive vessels

~Hypovolemia

~Decreased minute blood volume

~Deposition of blood in the veins of the systemic circulation}

18-year-old patient complains of general weakness, fatigue, low spirits. The patient is of the asthenic
constitution type. Ps - 68/min., AP - 90/60 mm Hg. She has been found to have primary neurocirculatory
hypotension. What is the leading factor of the arterial pressure drop in this patient?

{=Decreased tonus of resistive vessels

~Decreased cardiac output

~Hypovolemia

~Decreased minute blood volume

~Deposition of blood in the veins of the systemic circulation}

A 67-year-old patient has atherosclerosis of cardiac and cerebral vessels. Examination revealed
hyperlipidemia. What class of blood plasma lipoproteids is most important in atherosclerosis
pathogenesis? {=Low-density lipoproteids
~Chylomicrons

~a-lipoproteids

~High-density lipoproteids

~-}

A 5-year-old boy with congenital pulmonary artery stenosis while walking developed dyspnea, cyanosis
appeared on his face and finally he lost consciousness. What is the primary mechanism of development
of this condition?

{=Acute hypoxia of the brain

~Expansion of peripheral vessels

~Decrease of blood pressure

~Disorder of pulmonary ventilation


~Disorder of gases diffusion in the lungs}

A 55-year-old woman suffers from renal failure and has blood pressure 170/100 mm Hg. Excessive
activation of which of the following systems determines constant increase of blood pressure in the
woman? {=Renin-angiotensin-aldosterone

~Simpatico-adrenal

~Hypothalamus-pituitary

~CNS

~Kalikrein-kinin system}

A 58-year-old patient appealed to a doctor with complaints of the steady increase of arterial blood
pressure. On clinical examination of the patient chronic kidney disease with impairment of renal blood
circulation was found. Activation of which of the mechanisms led to development of increased blood
pressure in the man? {=Renin-angiotensin-aldosterone

~Parasympathetic nervous system

~Sympathetic nervous system

~Cardiovascular

~Metasympathetic nervous system}

What mechanism is responsible for the development of hypertension in a patient with renal
insufficiency? {=Increase the renin secretion

~Reduction of water reabsorption

~Reduction of electrolytes reabsorption

~Increase of glucose reabsorption

~Proteinuria}

According to Pauzel’s formula: P = QR, where P it’s a fluid pressure in the pipe, Q - value of their filling
and R - resistance of fluid flow. What hemodynamic type of arterial hypertension the increase of as Qso
R is characteristic of?

{=Eukinetic

~Hyperkinetic

~Hipokinetic

~Mixed
~For all of these types}

A 24-year-old female was admitted to the hospital with complaints of headaches, pain in lower back and
general weakness. A month before the patient sustained tonsillitis. Objectively: blood pressure180/110
mmHg, peripheral blood analysis: Er - 3.1x10"/ |, Le - 12,6x10°/l, ESR-28 mm/hr. Urine analysis
demonstrates pronounced proteinuria, hematuria, leukocyteuria. What is the mechanism of
development of hypertension in this case? {=Renal

~Mineralocorticoid

~Renovascular

~Reflexogenous

~Aldosterone}

Ina patient with hypertension an increased concentration of angiotensin Il in the blood has been
detected. What is the hypertensive effect of angiotensin associated with?

{=Muscle arterioles contraction

~Activation of biogenic amines synthesis

~Excessive prostaglandins synthesis

~Stimulation of vasopressin production

~Activation of kalikrein-kinin system}

Ina patient with hypertension an increased concentration of vasopressin in the blood has been
detected. Which organ is the target of this hormone?

{=Kidneys

~Liver

~Heart

~Lungs

~Adrenals}

Arterial hypertension due to artificial narrowing of the renal arteries has been made in a dog. The
increase of reninangiotensin-aldosterone system activity has been observed. What component of this
system has most powerful hypertensive effect?

{=Angiotensin II

~Renin
~Angiotensin |

~Vasopressin

~Oxytocin}

A 65-year-old man has been suffering from marked arterial hypertension for 15 years. Recently his
systolic pressure began to decrease but diastolic remained unchanged. What is haemodynamic type of
hypertension in the patient?

{=Hypokinetic

~Normokinetic

~Hyperkinetic

~Eukinetic

~Akinetic}

A 47-year-old man has high cholesterol levels in the blood. Which vessels are the primary targets for
cholesterol deposition? {=Arteries

~Arteriols

~Capillaries

~Venules

~Veins}

A patient is experiencing high blood pressure, headaches. It was established that this pathological state
is related to a kidney disease. Which substance can cause this?

{=Renin

~Creatine

~NHat

~Urea

~Histamine}

A patient complains of headache. He has high blood pressure and angiotensin levels in the blood. What
exactly causes this?

{=Renin
~Creatine

~NH4+

~Urea

~Histamine}

Routine medical examinations of a 36-year-old man, who is a driver by profession, determined blood
pressure 150/90 mm Hg. The patient complains of tinnitus and weakness at the end of his working day,
the symptoms disappears after rest. Hypertension was diagnosed. What is the leading pathogenetic
mechanism in this disease? {=Nervous

~Renal

~Humoral

~Endocrine

~Reflexogenous}

Ina patient, who suffers from hypertension, daily fluctuations in peripheral vascular resistance to
blood flow were determined. What type of vessels are responsible for the vascular resistance?
{=Arterioles

~Aorta

~Capillaries

~Arteriolar bypass

~Veins}

A 82-year-old patient has been admitted to a hospital with systolic and diastolic blood pressure 195
mm Hg and 115 mm Hg respectively. What is the most important index for determining
atherosclerosis? {=Pulse wave velocity

~Arterial pressure

~Cardiac output

~Blood viscosity

“Volume of circulating blood}

A 39-year-old woman with a poor health condition preceding comma due to chronic renal failure was
admitted to the kidney center. Objective findings: edema of the legs and face; congestive and enlarged
liver. Blood pressure 190/120 mm Hg. What is the primary pathogenic mechanisms that most likely led
to the development of hypertension in this patient?

{=Increased renin secretion

~Increased glucocorticoids
secretion ~Increased
aldosterone secretion

~Increased secretion catecholamines

~Increased concentration of angiotensin in the blood serum}

A 54-year-old man suffering from atherosclerosis, ischemic heart disease, and angina has been
hospitalized to the cardiological department. Increased lipids levels were found on laboratory
examination of the patient’s blood serum. What class of plasma lipids performs a central role in the
pathogenesis of atherosclerosis? {=Low density lipoproteins

~Hilomicrons

~Alpha-lipoproteins

~High density lipoproteins

~Complexes of fatty acids with albumin}

In an experimental animal one kidney was removed, and a ligature narrowing conralateral kidney artery
was put. What type of secondary hypertension appeared?

{=Renovascular

~Metabolic

~Endocrine

~Neurogenic

~Anhiogenic}

A patient was diagnosed with renal hypertension. Name the initial pathogenetic factor in the
development of hypertension in this case?

{=Renal ischemia

~Hypernatriemia

~Hyperaldosteronism

~Increased renin synthesis


~Increased angiotensin synthesis}

A 65-year-old female suffers from atherosclerosis of brain vessels. Laboratory examination discovered
hyperlipidemia. Increase of what class of plasma lipoproteins has been detected in this biochemical
study? {=Low density lipoproteins

~Alpha-lipoproteins

~Hilomicrons

~High density lipoproteins

~Complexes of fatty acids with albumin}

Nowadays it has been proved that not native but modified lipoproteins play a central role in the
development of atherosclerosis. Capture and accumulation of modified complexes in the vascular wall is
accomplished by: {=Monocytes

~Fibroblast

~Endothelial cells

~Myocytes

~Neutrophils}

Rabbits were fed with foods supplemented with cholesterol. Atherosclerotic changes in aorta were
revealed after 3 months. What is the main cause of atherosclerosis in this case?

{=Exogenous hypercholesterolemia

~Overeating

~Stress

~Hypodynamia

~Endogenous hypercholesterolemia}

A 35-year-old patient has been diagnosed with arterial hypertension. What is characteristic of primary
pulmonary hypertension?

{=Dyspnea and cyanosis

~Edema of lower extremities

~Paroxysmal tachycardia

~Cough
~Bradycardia}

A 47-year-old patient has been diagnosed with atherosclerosis. What is the initial moment in the
pathogenesis of atherosclerosis?
{=Excessive deposition of plasma lipoproteins in the artery’s intima

~Slowdown of blood flow

~Proliferative-degenerative changes of endarterium

~Formation on the inner shell artery fibrous plaques

~Impairment of the arterial walls integrity}

A 62-year-old patient has been treated for arterial hypertension for 20 years. Recently he began to feel
dizziness and twinkling in the eyes. What is the most likely complication of hypertensive disease one
should beware of? {=Stroke

~Myocardial infarction

~Renal insufficiency

~Pulmonary hypertension

~Shock}

An autopsy revealed atherosclerotic changes in aorta and arteries of flexible and muscular-elastic types.
Which functional type do these vessels belong to?

{=Compensatory vessels

~Resistant vessels

~Shunt vessels

~Vessels of exchange

~Reservoir vessels}

Following a 38-year-old woman’s family history it has been revealed that the patient has an inherent
predilection to atherosclerosis due to high incidence of the disease in her family members.
Atherosclerotic changes in the woman began to appear 2 years ago when a routine hysterectomy and
removal of ovaries were performed. Which of the following etiological factors is responsible in the
development of atherosclerosis in this woman? {=Insufficiency of sex glands function

~Hereditary factor

~Female gender
~Age of the patient

~High content of fats on her diet}

A 54-year-old man has been diagnosed with hypertension. Which type of hypertension can be this
nosological unit attributed to?

{=Essential

~Due to renovascular disorders

~Due to disorders in mineralocorticoids metabolism

~Due to salt disorders}

A 56-year-old man has been affected by atherosclerosis of aorta, coronary arteries, cerebral arteries,
and arteries of lower limbs. Due to decreased elasticity these vessels:
{=Can not adequately alter their diameter according to tissues’ needs

~Can be obstructed by an exogenous embolus

~Can be obstructed by an endogenous embolus

~Can undergo spasm

~May be replaced with connective tissue}

A 55-year-old man has developed impairment of functional state of cortical and subcortical
centers regulating vascular tone. Hypertension, resulting from bilateral cerebral ischemia can be
attributed to: {=Central ischemic

~Reflexogenic

~Renovascular

~Due to disorders in glucocorticoids metabolism

~Due to disorders in mineralocorticoids metabolism}

A patient suffers from hypertensive disease. What hemodynamic type of hypertension does essential
hypertension belong to? {=Hypokinetic

~Hyperkinetic

~Eukinetic

~Not to any of these types

~Not any of these types}


A patient due to impairment of renal circulation developed arterial hypertension. What has a direct
impact on the precapillary vascular wall?

{=Angiotensin II

~Angiotensin

“Renin

~Aldosterone

~Angiotensynase}

A 45-year-old patient developed circulatory failure by right heart type. What type of hypertension
developed in this case?

{=Pulmonary hypertension

~Central ischemic hypertension

~Essential hypertension

~Reflexogenic hypertension

~Salt hypertension}

A 56-year-old woman weighing 120 kg developed constant pain in the heat region, that is a
consequence of atherosclerotic changes in coronary vessels. Increased levels of which substances in the
blood is important in the pathogenesis of atherosclerosis?

{=Low density lipoproteins

~Alpha- lipoproteins

~High density lipoproteins

~Hilomicrons

~Fatty acids}

A 56-year-old patient suddenly felt a sever retrosternal pain irradiating to the left hand, accompanied
with a fear of death. Face became pale coated with cold sweat. Nitroglycerin did not alleviate the pain.
The diagnosis of myocardium infarction was established. What is the most frequent cause of myocardial
ischemia? {=Atherosclerosis of coronary arteries

~Spasm of coronary arteries

~Thrombosis of heart vessels


~Coronary embolism

~Aggregation of erythrocytes in coronary vessels}

A 42-year-old man with chronic heart failure developed edema of the lower extremities. The
main role in the development of edema is performed by: {=Increased hydrostatic pressure

~Increased plasma oncotic pressure

~Increased plasma osmotic pressure

~Reduction of hydrostatic pressure

~Urorrhagia (polyuria)}

A 52-year-old patient died of myocardial infarction, that was confirmed by clinical and
electrocardiographic examinations; nevertheless autopsy did not found any evidence of sclerotic
changes in coronary vessels. What was the probable cause of myocardial infarction in this patient?

{=Increased secretion of catecholamines

~Increased sympathetic tone of the autonomic nervous system

~Obturation of coronary vessels by an embolus

~Coronary thrombosis

~Rheumatoid coronaritis}

A 46-year-old patient with ischemic heart disease on the basis of arteriosclerotic lesions of coronary
arteries after considerable physical exertion developed acute myocardial infarction. What is the most
likely cause of coronary circulation failure in this case? {=Coronary artery stenosis

~Embolism of coronary arteries

~Coronary artery thromboembolism

“Rupture of coronary artery

~The blood redistribution}

A 54-year-old female experiences cardiac arrhythmia with generation of cardiac contractions up to


400 b/min. Pulse rate is lower than the heart rate, suggesting disorders in excitability and
conduction. What pathology may cause such changes? {=Cardiosclerosis

~Coronarosclerosis

~Coronary embolism
“Coronary stenosis

~Coronary thrombosis}

The first stage in the pathogenesis of atherosclerosis is:

{=Excessive deposition of plasma lipoproteins in the inner membrane of arteries

~Slowdown of blood flow

~Proliferative degenerative changes of the


intima ~Formation of fibrous plaques on the
inner artery’s wall ~Disturbance of the
arterial wall integrity}

A hospital admitted a patient with arterial hypertension induced by renal artery stenosis. The patient
complains of persistent nausea and headache. The main element in the pathogenesis of hypertension
is the activation of the following system: {=Renin-angiotensin

~Hypothalamic-pituitary

~Kallikrein-kinin

~Sympathoadrenal

~Parasympathetic}

What is the main cause of hypertension in diabetes mellitus?

{=Atherosclerosis

~Renal failure

~Heart failure

~Lack of insulin

~Hyperglycemia}

A 43-year-old patient with pheochromocytoma (adrenal medullar tumor producing epinephrine)


developed elevated blood pressure. What type of hypertension can this blood pressure change be
attributed to? {=Secondary

~Primary

~Essential

~Compensated
~Tertiary}

A 61-year-old female developed general adynamy, fatigue, shortness of breath, dizziness, and
headaches. Arterial pressure is decreased. What is the cause of the symptoms described?

{=Hypoxemia

~Heart failure

“Vascular insufficiency

“Respiratory failure

~Intoxication}

After acute blood loss a patient developed hypotension. What is the cause of lowering blood
pressure? {=Anemia

~Vasodilation

~Increased tone of parasympathetic nervous system

~Decreased tone of parasympathetic nervous system

~Loss of blood proteins}

A 34-year-old patients has blood pressure 140/100 mm Hg. What is the pulse pressure in the patient?

{=40
~100

~140

~240

~70}

Examination of a patient revealed an increase in low-density lipoprotein concentration in blood serum.


The patient can be expected to have the following disease:

{=Atherosclerosis

~Pneumonia

~Glomerulonephritis

~Acute pancreatitis

~Gastritis}
A hospital admitted a patient with arterial hypertension induced by renal artery stenosis, complaints of
persistent nausea and headache. The main element in the pathogenesis of hypertension is the
activation of the following system:

{=Renin-angiotensin

~Hypothalamic-pituitary

~Kallikrein-kinin

~Sympathoadrenal

~Parasympathetic}

A patient was found to have an increased blood serum LDH-1 activity. In which organ is the
pathological process localized? {=Heart

~Liver

~Kidneys

~Stomach

~Muscles}

A 70-year-old patient has been found to have atherosclerosis of heart and brain vessels. Examination
revealed the changes in the lipid profile. Pathogenesis of atherosclerosis is greatly influenced by an
increase in the following lipoproteins rate:

{=Low-density lipoprotein

~Very-low-density lipoproteins

~Intermediate-density lipoproteins

~High-density lipoprotein

~Chylomicrons}

During ultrasound investigation a patient was diagnosed with bilateral renal artery stenosis of
atherosclerotic genesis. Specify the bioactive substance that due to its excessive secretion is the key
component of arterial hypertension pathogenesis in the given case:

{=Renin

~Cortisol

~Vasopressin

~Noradrenaline
~Thyroxin}

A 70-year-old patient presents with cardiac and cerebral atherosclerosis. Examination revealed
changes of blood lipid spectre. Increase of the following lipoproteins plays a significant role in
atherosclerosis pathogenesis: {=Low-density lipoproteins
~Very low-density lipoproteins

~Intermediate density lipoproteins

~High-density lipoproteins

~Chylomicrons}

During ultrasound investigation a patient has been diagnosed with bilateral stenosis of renal artery with
atherosclerotic genesis. Specify the bioactive substance that due to its excessive secretion is the key
component of arterial hypertension pathogenesis in the given case:

{=Renin

~Cortisol

~Vasopressin

~Noradrenaline

~Thyroxin}

A 54-year-old man requested a pharmacist’s advice on drug prescription. The patient has 4-year-long
history of chronic glomerulonephritis and 2-yearlong history of persistent hypertension. What
substance synthesized in the kidneys has important role in development of arterial hypertension?
{=Renin

~Nitric oxide

~Aldosterone

~Histamine

~Catecholamines}

A young man with suspicion on narcotic poisoning was admitted into neurological department. Which of
the disorders of external respiration can be expected?

{=Alveolar hypoventilation

~Alveolar hyperventilation
~Kussmal respiration

~Biot’s Respiration

~Asphyxia}

In pathogenesis of which types of respiration the main link is the fall of excitability of respiratory center
to carbon dioxide due to oxygen starvation of this center?

{=Accelerated and deep breathing

~Expiratory dyspnoea

~Periodic respiration

“Inspiratory dyspnoea

~Combined dyspnoea}

As a result of casualty the obturation of the lung trachea occurred. Which stage of respiration will be
impaired first?

{=Lung ventilation

“Tissue respiration

~Exchange of gases in the lungs

~Exchange of gases in tissues

~Transport of oxygen and carbon dioxide}

The functional state of respiratory system was examined ina patient with emphysema. What is more
characteristic for this state?

{=Increase of tidal volume

~Increase of vital volume of the lungs

“Increase of inspiratory reserve volume

~Decrease of total volume of the lungs

~Decrease of inspiratory reserve volume}

The syndrome of respiratory insufficiency is often observed in premature born children. What is the
main cause of this?

{=Immaturity of lung alveoli due to deficiency of surfactant


~Swallowing of amniotic water

~Intrauterine hypercapnia

~Imperfection of the nervous regulation of the respiratory act


~Intrauterine asphyxia}

Paleness of the skin, accelerated superficial respiration is observed in a newborn. Numerous diffused
atelectasis are revealed at X-ray examination. What is the most possible cause of this condition?
{=Surfactant deficiency
~Hydrothorax

~Tuberculosis

~Bronchial asthma

~Pneumothorax}

A patient who was at a resuscitation department with skull injury suddenly developed convulsions on
the background of regaining consciousness, and short arrest of breathing was changed by solitary sighs
with calming down character. What types of respiration appeared in the patient?

{=Gasping respiration

~Cheyne-Stokes respiration

~Biot’s respiration

~Kussmaul respiration

~Apnoeystic respiration}

During meal a child breathe in a seed. What respiratory changes will develop in the child?

{=At first inspiratory dyspnoea, then expiratory one

~First expiratory dyspnoea, then inspiratory one

~At first arrest of breathing, then expiratory dyspnoea

~Expiratory dyspnoea, then Cheyne-Stokes respiration

~Inspiratory dyspnoea, then Biot’s respiration}

A partial respiratory insufficiency developed in a patient due to a chronic impairment of the organs of
respiratory system. What is a characteristic sign of partial respiratory insufficiency?

{=Hypoxemia without hyperapnia


~Hypoxemia and decrease of alveolar ventilation

~Decrease of alveolar ventilation and hypercapnia

~Hypoxemia and gas acidosis

~Hypoxemia and hypercapnia}

When developing of pulmonary emphysema in a patient the following is notified:

{=Increase of functional “dead space”.

“Inspiratory dyspnoea

~Cardiac insufficiency (left ventricular type)

~Decrease of resistance to the airflow in the respiratory tract

~Decrease of functional “dead space”}

Decrease of the passage at the level of middle and small bronchi is observed in a patient. What process
will be the leading in the development of respiratory insufficiency?

{=Hypoventilation

~Hyperperfusion

~Disturbance of diffusion

~Hypoperfusion

~Hyperventilation}

In the decrease of the middle and small bronchial passages in a patient the following is observed:

{=Decrease of pO2 and increase of pCOz in the alveolar air

~Decrease of pressure in pulmonary circulation

~Development of inspiratory dyspnoea

~Hypocapnia

~Development of gas alkalosis}

In obstructive type of respiratory disturbances in a patient it will be determined


{=Expiratory dyspnoea

~Increase in forced vital volume of the lungs


~Increase in vital volume of the lungs

~Decrease of the tidal volume

~Decrease of the total volume of the lungs}

In restrictive type of respiratory disturbances in a patient the following is revealed


{=Decrease of tidal volume of the lungs

~Expiratory dyspnoea

~Increase of vital capacity of the lungs

~Increase of tidal volume of the lungs

~Decrease of rate and increase of depth of respiration}

Gas alkolosis is revealed in the patient is blood. What process impairment is connected with this
development?

{=Hyperventilation

~Impairment of diffusion

~Hyperperfusion

~Hypoventilation

~Hypoperfusion}

The application of oxygen did not eliminate hypoxemia in a patient with partial respiratory insufficiency.
What is the mechanism of respiratory insufficiency?

{=Hyperperfusion (anatomical shunt)

~Hyperperfusion (functional shunt)

~Hypoventilation

~Disturbance of diffusion

~Hyperventilation}

Patient aged 62 was admitted into neurological department due to cerebral haemorrhage. Grave
condition. Increase of depth and rate of respiration and then its decrease and apnoea is observed. After
that the cycle of respiratory movements start again. What type of respiration appeared in the patient?
{=Cheyne-Stokes respiration

~Kussmaul respiration
~Gasping respiration

~Apneustic respiration

~Biot’s respiration}

After Typhno’s test index decreased to 30% in the patient. The development of what pathological
process in the organs of respiratory systems does it indicate?

{=Obstructive bronchitis

~Pneumothorax

~Lobular pneumonia

~Tuberculous pleurisy

~Pneumosclerosis}

A patient has developed athelectasis, which was accompanied by alveolar collapse. What contributes to
this? {=Surfactant deficiency

~Spasm of lung vessels

~Arterial hypertension

~Hyperventilation

“Respiratory acidosis}

A patient has the pathology of the lung with the disturbance of external respiration and development of
hypercapnia and hypoxia. How many times is the ability of CO2 to diffusion through alveolar-capillary
membrane higher than the same one of oxygen?

{=25
~10
~15
~20
~5}

Patient’s respiratory rate is 10 per min, respiratory volume - 1.1 | whistling rals are heard over the lungs
on auscultation. Which of pathogenetic forms of external respiratory disturbance has this patient:
{=Obstructive

~Diffuse-restrictive
~Ventilative-restrictive

~Primary-dyskinetic

~Diffuse-pneumonia}

Which of the pathogenetic forms of external respiratory disturbance may develop in traumatic costal
fracture? {=Ventilative-restrictive.

~Primary-dyskinetic

~Obstructive

~Diffuse-restrictive

~Diffuse pneumonia}

Which of pathogenetic forms of external respiratory disturbance may appear in cerebrospinal trauma of
the level of cervico-thoracic part of the spine?

{=Primary-dyskinetic

~Obstructive

~Ventilative-restrictive

~Diffuse-restrictive

~Diffuse pneumonic}

In which of pathologic processes does obstructive form of disturbance of external respiration appear?
{=Bronchial asthma

~Pneumonia

~Pleurisy

~Pulmonary edema

~Open pneumothorax}

In which of pathologic processes does ventilative-restrictive form of external respiratory disturbance


develop? {=Pulmonary edema

~Bronchial asthma

~Poliomyelitis

~Syringomyelitis
~Bronchitis}

In what form of dyspnea develops in insufficiency of surfactant system of the lungs?

{=Fast shallow respiration

~Fast deep respiration

~Rare shallow respiration

~Expiratory dyspnea

~Rare deep respiration}

Which of pathogenetic forms of external respiratory disturbance may develop in craniocerebral trauma?
{=Primary dyskinetic

~Diffuse-restrictive

~Ventilative-restrictive

~Diffuse-pneumonic

~Obstructive}

A patient aged 23 was admitted to the hospital with a craniocerebral injury in a poor condition.
Respiration is characterised by spasmatic effort of inspiration which does not stop and sometimes is
broken by expiration. What type of respiration is this typical for?

{=Apneustic
respiration ~Gasping
respiration

~Kussmal’s respiration

~Cheyne-Stokes’ respiration

~Biot’s respiration}

A patient aged 64 was admitted to the hospital with complaints of cough with sputum, marred dyspnea.
Objectively forced position, respiratory rate - 32 per min: additional muscles take part in the act of
respiration. X-ray examination: the lungs have increased transparency. What is the main link in the
pathogenesis of respiratory insufficiency of this patient? {=Decrease of elastic abilities of the lungs

~Accumulation of sputum in the lungs

~Thin mucus membranes of bronchi

~Impairment of surfactant system of the lung


~Pneumosclerosis}

Both vagus nerves were cut during experiment on animal. How may vagus respiration be characterized?
{=Very rare and deep

~Frequent and deep

~Rare and shallow

~Frequent and shallow

~Periodic}

A child with diphtheria developed larynx edema. What type of respiration is observed?

{=Dyspnea

~Apneustic respiration

~Kussmal’s
respiration
~Gasping
respiration

~Biot’s respiration}

The deficiency of alpha-1-antitrypsine was determined in a patient with pulmonary emphysema. What is
the leading mechanism of the development of emphysema in this patient?

{=Activation of elastolysis

~Increase of histamine discharge

~Dilation of bronchioles

~Excessive discharge of elastase by lymphocytes

~Decrease of elastase discharge by neutrophilic granulocytes}

A patient with bronchial asthma has developed acute respiratory insufficiency. What type of respiratory
insufficiency appears in this case?

{=Obstructive disturbance of alveolar ventilation

~Diffuse

~Perfuse

“Restrictive disturbance of alveolar ventilation


~Disregulative disorder of alveolar ventilation}

0.5 ml of air was introduced into a rat’s pleural cavity. What type of respiratory insufficiency appears in
this case?

{=Restrictive disturbance of alveolar respiration

~Obstructive disturbance of alveolar ventilation

~Perfuse

~Diffuse

~Disregulative disturbance of alveolar ventilation}

Acute respiratory insufficiency appeared in a patient with tetanus. What type of respiratory insufficiency
develops in this case?

{=Disregulative disturbance of alveolar ventilation

“Restrictive disturbance of alveolar respiration

~Obstructive disturbance of alveolar ventilation

~Perfuse

~Diffuse}

A female patient was admitted to the hospital with exertion of chronic bronchitis. Antibiotic treatment
was administered. On the fourth day general condition became worse: the temperature persisted,
dyspnea and cough increased, itching eruptions appeared on the skin, diffused dry whistles were heard
on auscultation. There is eosinophilia (10%) in the blood. Aggregation of symptoms of the patient is
caused by the development of: {=Drug allergic reaction

~Bronchial asthma

~Bronchopneumonia

~Allergic reaction

~Asthmatic bronchitis}

Increased content of carbon dioxide is in a room. How does respiration change (its depth and rate) ina
man entered this room?

{=Depth and rate of respiration increase

~Rate of respiration increases


~Depth of respiration increases

~Depth of respiration decreases

~Rate of respiration decreases}

When eating a child aspired some food. Strong cough, cyanosis of skin and mucous membranes,
increase in pulse rate, rare respiration, and lengthening of inspiration developed in him. What kind of
disorder of external respiration developed in this child? {=Expiratory dyspnea stage in asphyxia

~Stenotic respiration

~Inpiratory dyspnea stage in asphyxia

~Biot’s respiration

~Alternative respiration}

A 56-year-old patient, who is fitter by occupation, has been suffering form fibrous-cavernous
tuberculosis of lungs for 10 years. During later 3 weeks cough and weakness intensify, amount of
purulent-mucous phlegm with blood increases in him. What is the reason for developed ventilation
insufficiency in this case? {=Decrease in number of functioning alveoli

~Disturbances of functions of respiratory center

~Disturbances of functions of neuro-muscular apparatus

~Disturbances of agility of the thorax

~Disturbances of airways passage}

A 50-year-old unconscious patient was admitted to the endocrinology department. It is known that
patient has been suffering from diabetes mellitus for many years. What kind of disorders of external
respiration will be present in this patient?

{=Kussmaul’s respiration

~Asphyxia

~Biot’s respiration

~Stenotic respiration

~Chain-Stocks respiration}

Attack of suffocation developed in a 45-year-old woman suffered from bronchial asthma for a long time.
What pathogenetic mechanism does this phenomenon have?

{=Expiratory narrowing of small bronchi


~Loose of elasticity of lung tissue

~Decrease in sensitivity of respiratory center

~Disorders of agility of the thorax

~Impairment of perfusion of lung tissue}

Edema of larynx developed in a child on the background of allergic reaction. What kind of respiratory
failure has developed in this case?

{=Obstructive

~Restrictive

~Disregulative

~Parenchimatous

~-}

A patient with both-sided lung edema was in the emergency department. What kind of dyspnea is
observed in this case?

{=Shallow frequent respiration

~Deep frequent respiration

~Shallow rare respiration

~Shallow respiration

~Asphyxic respiration}

Lung fibrosis with disorders of alveolar ventilation developed in a miner. What is the leading mechanism
in this disorder development?

{=Restriction of respiratory surface of lungs

~Narrowing of upper airways

~Disorders of neural regulation of respiration

~Restriction of agility of the thorax

~Spasm of bronchi}

A 60-year-old male patient in severe condition was admitted to the hospital. The patient has been
suffering from diabetes mellitus for 10 years. At second day of his stay at the in-patient department his
condition sharply worsened: coma developed, noisy respiration appeared, which was accompanied by
deep inspirations with forced expirations and participation of expiratory muscles. What kind of disorders
of respiration is observed in the patient?

{=Kussmaul’s respiration

~Stenotic
respiration
~Tachipnea

~Chain-Stocks’ respiration

~Biot’s respiration}

A patient with craniocerebral trauma has respiration, which is characterized by respiratory movements
becoming deeper and then becoming shallower and after that short pause in respiration occurs. What
periodical respiration is this type characteristic for?

{=Chain-Stocks’ respiration

~Biot’s respiration

~Kussmaul’s respiration

~Gasping respiration

~Apneastic respiration}

A female patient N., aged 16, with asphyxia was admitted to the emergency department. She has
expiratory dyspnea with respiration rate of 8 per minute, BP of 80/50 mmbg, heartbeat rate of 40 bpm,
and narrowed pupils. What phase of asphyxia does this patient have?

{=ll phase

~| phase

~II phase

~IV phase

~-}

A boy was admitted to resuscitation department. He has inspiratory dyspnea, general excitation,
widened pupils, tachycardia, and BP of 160/100 mmHg. What phase of asphyxia does this patient have?
{=I phase

~II phase

~IIl phase
~IV phase

~-}

A 15-year-old girl has been suffering from bronchial asthma for 5 years. What kind of hypoxia does this
patient have? {=Respiratory
~Circulatory

~Hemic

~Tissue

~Mixed}

A 37-year-old man was admitted to a hospital with an attack of bronchial asthma. What respiration
type will be observed in this patient? {=Expiratory dyspnea

~Inspiratory dyspnea

~Apnoea

~Gasping respiration

~Hyperpnoea}

A patient was diagnosed with right lung cancer and doctors administered him surgical treatment. After
rightsided pulmonectomy the patient began to suffer from evident dyspnea. What form of respiratory
failure is it?

{=Pulmonary restrictive

~Central

~Peripheral

~Pulmonary obstructive

~Thoracodiaphragmal}

A patient had been diagnosed with right lung cancer and administered surgical treatment. After right-
sided pulmonectomy the patient presented with evident dyspnea. What form of respiratory failure has
developed in this patient?
{=Pulmonary restrictive

~Central

~Peripheral

~Pulmonary obstructive
~Thoracodiaphragmal}

The patient has been hospitalized with pneumonia. What kind of respiratory failure does the patient
have? {=Restrictive

~Obstructive

~Central

~Peripheral

“Thoracic diaphragm}

The patient has been hospitalised with pneumonia. What kind of respiratory failure does the patient
have? {=Restrictive

~Obstructive

~Central

~Peripheral

“Thoracic diaphragm}

A patient has been hospitalised with pneumonia. What kind of respiratory failure does the patient have?
{=Restrictive

~Obstructive

~Central

~Peripheral

“Thoracic diaphragm}

A patient in the state of ketoacidotic coma presents with loud rapid respiration: labored expiration with
tension of expiratory muscles occurs after deep inspiration. Name the type of pathologic respiration:

{=Kussmaul’s

~Cheyne-Stokes’

~Gas
ping
~Sten
otic

~Biot’s}
A patient with brain edema presents with respiration that is characterized by periods of several
respiratory movements of equal amplitude alternating with periods of apnea. What pathologic
respiration is it characteristic of?
{=Biot’s respiration

~Gasping respiration

~Apneustic respiration

~Cheyne-Stokes’ respiration

~Kussmaul’s respiration}

A patient in the state of ketoacidotic coma presents with loud rapid respiration: labored expiration with
tension of expiratory muscles occurs after deep inspiration. Name the type of pathologic respiration:

{=Kussmaul’s

~Cheyne-Stokes’

~Gasping

~Stenotic

~Biot’s}

A patient in the state of ketoacidotic coma presents with loud rapid respiration: labored expiration with
tension of expiratory muscles occurs after deep inspiration. Name this type of pathologic respiration

{=Kussmaul’s

~Cheyne-Stokes’

~Gas
ping
~Sten
otic

~Biot’s}

EXAM BASE 1

1. Depressurization of the cabin at an altitude of 19 km led to instantaneous death of pilots. What is


its cause?
What disorder of local circulation is characterized by pallor, local temperature drop, pain, local
sensitivity disorder, reduction in the volume of the organ?

Cellular and plasma mediators play an important part in the pathogenesis of secondary alteration
during inflammation. What mediators are produced in the blood plasma?

A 73-year-old patient had been admitted to a hospital with closed fracture of his right femur.
Suddenly his condition deteriorated, the patient was diagnosed with vascular embolism. What type
of embolism is observed most often in patients with the fractures of tubular bones?

Stable contraction of myofibrilla of muscle fibers takes place due to accumulation of the following
ions in the cytoplasm:

After a 5-year-old child has been brought home from the kindergarten he presented with weakness,
headache, body temperature rise up to 37,50C. What period of disease develompent is the case?

Every year during the plant blossoming a female patient develops acute catarrhal inflammation of
conjunctiva and nasal mucosa that is the clinical presentation of an allergy. These symptoms relate
to the following type of allergic reactions:

A female patient consulted a doctor about leg pain that arises usually toward the evening; feet and
shins edemata. Objectively: leg skin is cyanotic, cold to the touch. What type of peripheral
circulation disorder does the patient present with?

It is required to measure the nitrogen metabolism in a person under observation who is recovering
from continuous starvation. What result is most likely to be expected?

10. Humoral immune response to an antigen results in generation of antibodies produced by


plasmacytes. Plasmacytes arise as a result of immunostimulated division from the following cells of
immune system:

11. Examination of the lower limbs of a40-year-old patient with coronary artery disease and vascular
disease of the lower limbs (obliterating endarteritis)revealed skin pallor and dystrophy, local
temperature decrease, sense shock, pain. The patient is likely to have the following disorder of the
peripheral blood circulation:

12. In an emergency situation a scuba diver has quickly risen from the depths to the surface, which is
against the rule. He is unconscious, presents with respiratory failure and cardiac activity disorder
as a result of decompression sickness. What complication may develop in the scuba diver?

13. A patient with pneumosclerosis has blood pH at the rate of 7,34. Analysis of gas formula of blood
showed hypercapnia. Urine analysis revealed an acidity increase. What form of acidbasedisbalance
is the case?
14. After eating strawberries a child presented with itchy red spots on the skin (hives). According to the
classification of Coombs and Jell this reaction relates to the following type of allergic reactions:
15. After an insulin injection a 45-yearold woman with a long history of diabetes mellitus has developed
weakness, paleness, palpitation, anxiety, double vision, numbness of lips and the tip of tongue.
Blood glucose is at the rate of 2,5 mmol/I. What complication has developed in the patient?

16. Food rich in carbohydrates at first increases the blood glucose and then decreases its rate due to
the insulin action. What process is activated by this hormone?

17. Laboratory analysis revealed protein in the urine of a young man. In what case a healthy person
may present a slight proteinuria?

18. A male received a radiation dose of 30 Gy. He presents with necrotic angina, disorders of the
gastrointestinal tract. Blood tests revealed anemia, leucopenia and thrombocytopenia. What
period of acute radiation sickness is observed in the patient?

19. In an emergency situation a scuba diver has quickly risen from the depths to the surface, which is
against the rule. He is unconscious, exhibits respiratory failure and cardiac activity disorder as a
result of decompression sickness. What complication may develop in the scuba diver?

20. An injured person exhibits the following signs at the site of trauma: skin redness, throbbing small
arteries, elevated local temperature, increased tissue turgor. What local blood circulation disorder
are these presentations typical for?

21. On the 2nd day after developing acute inflammation of the knee joint, the patient exhibits the joint
enlargement, swelling of the skin. At what stage of inflammation are these signs typically observed?

22. After an insulin injection a 45-year old female with a long history of diabetes mellitus has developed
weakness, paleness, palpitation, anxiety, double vision, numbness of lips and the tip of tongue.
Blood glucose is at the rate of 2,5 mmol/I. What complication has developed in the patient?

23. A 45-year-old male patient was diagnosed with stomach ulcer. After the conservative treatment
the pain and heartburn disappeared, the function of the gastrointestinal tract was normalized.
Endoscopic examination of stomach revealed cicatrization of the ulcer. Qualify this course of the
disease:

24. Food rich in carbohydrates at fi rst increases the blood sugar and then decreases its rate due to the
insulin action. What process is activated by this hormone?

25. After drinking milk a 1-year-old child developed diarrhea, flatulence. The baby is likely to have the
deficiency of the following enzyme:
26. In response to the administration of protein drugs, a patient developed an allergic reaction. The
development of the allergic reaction is caused by the increased synthesis of the following
compound:

27. After a contact with a person having an infectious diseases, the disease pathogens entered the
patient’s body and started to multiply, but the symptoms of the disease were not yet observable.
What period of the disease is this typical for?
28. A male patient developed fever up to 400C, there are vomiting, diarrhea, the patient is in grave
condition. Blood osmolality is 270 mOsm/I. What disorder of water-salt metabolism is observed in
the patient?

29. Diabetes and starvation cause the excess production of ketone bodies that are used as an energy
source. They are produced from the following compound:

30. A male received a radiation dose of 30 Gy. He presents with necrotic angina, disorders of the
gastrointestinal tract. Blood tests revealed anemia, leucopenia and thrombocytopenia. What phase
of acute radiation syndrome is observed in the patient?

31. As a result of hypothermia a male patient developed acute diffuse glomerulonephritis. What type
of allergic reaction caused damage to the glomerular capillaries in the patient?

32. As a result of an emergency situation (shipwreck) a man had to drink sea (salty) water. What form
of water-salt imbalance may occur in this case?

33. For the specific prevention of influenza, the employees of an enterprise were vaccinated with
"Influvac". What type of immunity will develop in the body of the vaccinated?

34. A patient exhibits small (petechial) hemorrhages under the skin and mucous membranes, bleeding
gums, tooth decay, general weakness, edemata of the lower extremities. What vitamin deficiency
can be suspected?

35. 1 minute after the patient had been administered penicillin the patient’s arterial pressure sharply
dropped, pulse became thready, cold sweating and clonic convulsions began. Name this condition.

36. The patient with acute left ventricular failure has developed edema of lungs. What peripheral
circulation disorder taking place in the lungs has caused this complication?

37. Knee joint enlargement and cutaneous edema has developed in the 46-year-old patient with acute
knee joint inflammation on the second day. What stage of inflammation progressing are these
symptoms usually observed at?

38. In the process of chemical solution preparation laboratory assistant’s forearm was exposed to
concentrated hydrochloric acid. There are burning pain, hyperemia and swelling of the damaged
area. What pathologic process are these symptoms evidential of?
39, In 1915 Japanese scientists Katsusaburo Yamagiwa and Koichi Ichikawa became the first, who
induced experimental tumors, by painting ears of rabbits with coal tar. What method of
experimental tumor inducing did they use?

40. The 49-year-old female patient suffering long-term from pancreatic diabetes has developed the
following symptoms after administering insulin: weakness, facial pallor, palpitation, anxiety, double
vision, numbness of lips and tongue apex. Glucose molar concentration in blood was 2,5 mmol/l.
What complication has developed in the patient?

41. The 40-year-old patient has been diagnosed with gastric ulcer, disease symptoms making
reappearance after prolonged period of dormancy. How can this kind of disease progression be
qualified?
42. Milk intake has resulted in the one-year-old child having diarrhea and abdominal distension. What
enzyme deficiency does the child have?

43. The 13-year-old female patient having suffered from measles complains of dry mouth, thirst, body
weight loss, polyuria, her glucose concentration in blood is 16 mmol/l. What disease can be
suspected?

44, The patient has been admitted to the hospital with complaints of general fatigue, headache,
lumbago, edema of face and extremities. Urine analysis revealed proteinuria, hematuria and
cylindruria. What is the main pathogenetic mechanism of edema formation during
glomerulonephritis?

45. Fluorography examination of the 59-year-old patient has revealed well defined shadow, which is
characteristic to tumor, in the lower part of the left lung. What trait is characteristic of benign
tumor?

46. In an emergency situation a scuba diver has quickly risen from the depths to the surface, thus
breaking safety rules. He is unconscious, presents with respiratory failure and cardiac activity
disorder as the result of decompression sickness. What complication may develop in the scuba
diver?

47. What disorder of local circulation is characterized by pallor, local temperature drop, pain, local
sensitivity disorder,reduction of the organ volume?

48. Diet of a human must contain vitamins. What vitamin is usually prescribed for treatment and
prevention of pellagra?

49. A 22-year-old man was stung by bees; the affected area became hyperemic and edematous. What
is the leading mechanism of edema development in this patient?
50. A 40-year-old man diagnosed with gastric ulcer has developed the symptoms anew after a long
period of dormancy. Such disease course can be characterized as a:

51. After drinking milk a 1-year-old child has developed diarrhea and flatulence. The baby is likely to
have the deficiency of the following enzyme:

52. In response to the administration of protein drugs, a patient developed an allergic reaction. The
development of the allergic reaction is caused by the increased synthesis of the following
compound:

53. A 25-year-old-patient with the Il degree thermal burns addressed a doctor. Objectively: there are
large blisters on the upper limbs; the blisters are filled with clear exudate containing mostly water
and albumines with isolated leukocytes. Name the type of the exudate:

54. In an emergency situation a scuba diver has quickly risen from the depth to the surface in violation
of safety regulations. He is unconscious, presents with respiratory failure and cardiac activity
disorder as the result of decompression sickness. What complication can develop in the scuba
diver?

55. In the state of fright the following signs can be observed: acute pallor of face, tremor of extremities.
What kind of ischemia can be observed in such a condition?
56. An injured person exhibits the following signs at the site of trauma: skin redness, throbbing small
arteries, elevated local temperature, increased tissue turgor. What local blood circulation disorder
are these presentations typical of?

57. Diet of an individual must contain vitamins. What vitamin is usually prescribed for treatment and
prevention of pellagra?
58. If the amount of a high-molecular substance added to the sol is very small, it can not increase but
decrease its stability. This phenomenon is called:

59. An ophthalmologist has detected increased time of dark adaptation in a patient. What vitamin
deficiency can result in such symptom?

60. During containment measures following Chornobyl Nuclear Power Plant disaster a worker has been
exposed to a dose of ionizing emission of 6 Gy (600 R). The worker complains of general fatigue,
nausea, dizziness, labile blood pressure and heart rate, short-term leukocytosis with lymphopenia.
What stage of acute radiation sickness can be characterized by such presentations?

ol. A woman noticed that a cut on her skin was still bleeding even after 20 minutes had passed. What
vitamin deficiency causes such condition?

62. A man received a radiation dose of 30 Gy. He presents with necrotic angina, disorders of the
gastrointestinal tract. Blood tests revealed anemia, leukopenia and thrombocytopenia. What stage
of acute radiation sickness is observed in the patient?
63. A patient with croupous pneumonia presents with sharp increase of body temperature up to 390
C, which persisted for 9 days with daily amplitude of 1 degree. What temperature curve could be
observed?

64. The most severe and dangerous complication of diabetes mellitus is hypoglycemic coma that is
characterized by loss of consciousness and is lethal, unless efficient emergency treatment is
received by patient. What is the main pathogenetic component of hypoglycemic coma?

65. An injured person exhibits the following signs at the site of trauma: skin redness, throbbing small
arteries, elevated local temperature, increased tissue turgor. What local blood circulation disorder
are these presentations typical of?

66. An ophthalmologist has detected increased time of dark adaptation in a patient. What vitamin
deficiency can result in such symptom?

67. A patient with type | diabetes mellitus developed hyperketonemic coma. What acid-base imbalance
will be observed in the patient?

68. The patient presents with rapid growth of a tumor node and its progressing malignization. What
stage of tumor growth can be characterized by these presentations?

69. A 25-year-old-patient with the Il degree thermal burns came to a doctor. Objectively: there are
large blisters on the upper limbs; the blisters are filled with clear exudate containing mostly water
and albumines with isolated leukocytes. Name the type of the exudate:
70. In 9 days after administration of a therapeutic serum the patient developed urticaria, itching,
edemas, and lymph nodes enlargement. What type of allergic reaction has occurred in the patient?

71. A patient suffers from Down’s disease that manifests as mental retardation, shortness of stature,
pathologically short fi- ngers and toes, and eyes with mongoloid slant. Karyotype analysis revealed
trisomy 21. What group of diseases does this pathology belong to?

72. HIV-infection occupational risk groups include people of various professions, healthcare workers
included. Specify the mosl likely route of infection transmission for healthcare workers:

73. A patient with high risk of hemorrhages is recommended to take vicasol (menadione) by his
physician. This drug is the structural analog of:

74. A 25-year-old man has an appointment with the dentist. Several minutes after his oral cavity was
lavaged with furacilin (nitrofurazone) the patient developed significant labial edema. What type of
allergic reaction is observed in this case?

75. A patient with gastric carcinoma has undergone several courses of radiation therapy. What system
is the first to become functionally disturbed after the body was exposed to ionizing radiation?
76. Insulin production in B-cells involves many substances. What substance gives the main signal for
insulin synthesis when its concentration changes?

77. When working in the garden, a man accidentally cut his hand. The wound remained untreated.
Shortly after that the wounded area developed inflammation with accumulation of exudate that
contained numerous viable and degenerate neutrophils. What type of exudate is it?\

78. A patient presents with persistent fever, with the difference between evening and morning
temperature not exceeding 10C. What type of fever curve is present in this patient?

79. Ultraviolet irradiation is used in medicine in various physiotherapeutic procedures. What


mechanism of medicinal action is characteristic of ultraviolet rays?

80. Aman received a radiation dose of 30 Gy. He presents with necrotic angina and disorders of the
gastrointestinal tract. Blood tests revealed anemia, leukopenia and thrombocytopenia. What stage
of acute radiation sickness is observed in the patient?

81. A patient suffers from hyperchromic B12-deficiency anemia. What vitamin preparation should be
prescribed in this case?

82. Preliminary disinfection of air and working surfaces of the equipment was conducted in the
operating room of the surgical inpatient unit. What method of sterilization would be the most
advisable in this case?

83. At the end of his shift a worker of the steel foundry felt dizziness, his body temperature increased
to 38.50C. What condition does he present with?

84. A woman complains of itching lips; they are reddened and covered in scabs and scales after she had
been using new lipstick for two weeks. What allergic reactions result in this kind of disorders?

85. It can be safely assumed that the infants born from the mothers with the history of measles will
not be affected by the measles outbreak during their stay in the maternity ward. What classes of
antibodies provide the infants with the resistance to this disease?

86. A child with evident hypotrophy got edemata on his lower extremities, ascites. What is the main
mechanism of pathogenesis of cachectic edema?

87. Examination of a patient revealed an increase in low-density lipoprotein concentration in blood


serum. The patient can be expected to have the following disease:

88. A patient had cerebral haemorrhage that made impossible active motions of left arm and leg.
Muscle tone of these limbs is increased, their spinal reflexes are intensified, reflex zones are
increased. What type of CNS disorder is it?
89. After a girl had accidentally eaten inedible mushrooms she was admitted to the resuscitation unit
with symptoms of impaired consciousness, arterial hypotension, anuria, hyperazotemia. What kind
of renal dysfunction is it?

90. Gastric juice of a patient has decreased concentration of enzymes. What secretory cells of stomach
display disfunction?

91. A 45-year-old woman has frequent uterine haemorrhages, she presents with general weakness,
dyspnea, tachycardia, cardiac pain. In blood: erythrocytes - 3 - 109/I, Hb- 70 g/I, colour index - 0,7.
The smear contains mostly hypochromic erythrocytes, microcytes. Specify the type of anaemia
accrding to its mechanism of development:

92. A 37-year-old man was admitted to a hospital with an attack of bronchial asthma. What respiration
type will be observed in this patient?

93. A patient was diagnosed with right lung cancer and doctors administered him surgical treatment.
After right-sided pulmonectomy the patient began to suffer from evident dyspnea. What form of
respiratory failure is it?

94. A patient complains about an increase in heart rate, hyperperspiration, irritability, sleeplessness.
He has been presenting with these symptoms for the latest six months. They indicate the
hyperfunction of the following endocrine gland:

95. A 50-year-old patient complains of having dyspnea under a considerable physical stress, leg
edemata. Examination reveals chronic myocarditis and circulatory failure. What is the evidence of
cardiac decompensation in the patient?

96. Parents of a 11-year-old boy noticed that he is far behind his peers in the physical development.
After the Xray an endocrinologist revealed that the growth zones of tubular bones had already
closed. Under these conditions, the intake of growth hormone can result in the development of:

97. A patient with obstructive jaundice presents with bradycardia, low arterial pressure, itching,
irritability, asthenia. What is the cause of these presentations?

98. A patient has been found to have sugari the urine. Blood glucose is normal. Arterial pressure is
normal. What is the mechanism of glycosuria development in this case?

99, A newborn born to an Rh-negative mother (3rd pregnancy) presents withprogressing jaundice,
symptoms of CNS excitation, anemia. What type of jaundice is it?

100. A patient has been hospitalized for chronic heart failure. Objectively: skin and mucous membranes
are cyanotic, the patient has tachycardia, tachypnea. What type of hypoxia has developed in the
patient?
101. Urine analysis revealed a decrease in sodium ion concentration. Which hormone provides an
enhanced reabsorption of sodium ions in the convoluted nephron tubules?

102. A 40-year-old patient has developed polyuria (10-12 liters per day) and polydipsia induced by
damage to the hypothalamo-hypophyseal tract. What hormone deficiency causes such disorders?

103. A group of alpinists climbing to the top had their blood tested. The test revealed erythrocytosis and
an increase in hemoglobin rate. What type of hypoxia caused the stimulation of erythropoiesis in
the bone marrow?

104. A patient had been diagnosed with right lung cancer and administered surgical treatment. After
right-sided pulmonectomy the patient presented with evident dyspnea. What form of respiratory
failure has developed in this patient?

105. A patient has obstruction of the common bile duct. Which of these substances is usually found in
urine in such cases?

106. A continuous stay in the mountains causes an increase of blood oxygen capacity. What is the
possible reason for this phenomenon?

107. A patient with systemic lupuserythematosus has developed a diffuse renal affection accompanied
by proteinuria, hypoproteinemia, massive edema. What is the mechanism of proteinuria
development in this case?

108. A hospital admitted a patient with arterial hypertension induced by renal artery stenosis,
complaints of persistent nausea and headache. The main element in the pathogenesis of
hypertension is the activation of the following system:

109. A 70-year-old patient has been found to have atherosclerosis of heart and brain vessels.
Examination revealed the changes in the lipid profile. Pathogenesis of atherosclerosis is greatly
influenced by an increase in the following lipoproteins rate:

110. A patient has developed megaloblastic anemia on a background of alcoholic cirrhosis. The main
cause of anemia in this patient is the following vitamin deficiency:

111. A patient was found to have an increased blood serum LDH-1 activity. In which organ is the
pathological process localized?

112. A patient with alcoholic cirrhosis complains of general weakness, dyspnea. He has been found to
have decreased blood pressure, ascites, enlargement of superficial veins of the anterior abdominal
wall, esophageal varices, splenomegaly. What hemodynamic disorder is observed
in the patient?
113. Seroprophylaxis and serotherapy of infectious diseases involves using immune sera. What type of
immunity is thus acquired?

114. A patient has been hospitalized for chronic heart failure. Objectively: skin and mucous membranes
are cyanotic, the patient has tachycardia, tachypnea. What type of hypoxia has developed in the
patient?

115. As a result of an accident (snakebite) a male patient has the following blood values: Hb- 80 g/I, RBC-
3, 0-1012/l; WBC- 5, 5-109/I. What type of anemia is observed in this case?

116. A patient had been diagnosed with right lung cancer and administered surgical treatment. After
right-sided pulmonectomy the patient developed evident dyspnea. What form of respiratory failure
developed in this patient?

117. A 22-year-old male was stung by bees, the affected region became hyperemic and edematous.
What is the leading mechanism of edema development in this patient?

118. A patient has obstruction of the common bile duct. Which of these substances is usually found in
urine in such cases?

119. A patient with chronic renal failure exhibits azotemia, hypo- and isosthenuria. What is the main
factor in the pathogenesis of these symptoms in the patient?

120. A hospital admitted a patient with arterial hypertension induced by renal artery stenosis. The
patient complains of persistent nausea and headache. The main element in the pathogenesis of
hypertension is the activation of the following system:

121. Addison’s (bronze) disease is treated with glucocorticoids. Their effect is provided by the
potentiation of the following process:

122. An older patient exhibits low levels of red blood cells and hemoglobin in blood, but the color index
is 1,3. Blood smear analysis revealed megaloblasts. What type of anemia is observed in this case?

123. A patient has developed megaloblastic anemia on a background of alcoholic cirrhosis. The main
cause of anemia in this patient is the following vitamin deficiency:

124. A patient who had been continuously treated with glucocorticoids was found to have a duodenal
ulcer. What mechanism plays a major part in its development?

125. A patient with acute myocarditis exhibits rapid fatigability, shortness of breath, edemata of legs,
hepatomegaly. Classify the type of heart failure by the mechanism of its development:
126. A child with PKU has an unpleasant mouse-like odor, growth retardation, mental retardation. These
symptoms are associated with the high concentration of the following substance in blood:

127. A 40-year-old male presented to the endocrinology department with disproprortionate


enlargement of limbs, mandible and nose. These manifestations are caused by the overproduction
of the following hormone:

128. A parturient woman diagnosed withuterine inertia has been delivered to the maternity ward. The
doctor gave her an injection of the drug that activates the contraction of smooth muscles of the
uterus. What hormone is a component of this drug?
129. During calculous cholecystitis attack the patient has developed the following symptoms:
saponated feces and steatorrhea. What stage of fats metabolism is disrupted according to those
symptoms?

130. Parents of the 10-year-old child have made an appointment with endocrinologist due to complaints
of child’s low height. The child’s appearance is corresponding with that of 5-yearold child. What
hormon secretion disorder causes such physical development changes?

131. The 55-year-old patient has been hospitalised due to chronic cardiac failure. Objectively: skin and
mucosa are cyanotic, tachycardia, tachypnea. What kind of hypoxia does the patient have?

132. The patient has been hospitalisedwith pneumonia. What kind of respiratory failure does the patient
have?

133. During gastric secretory function research decrease of hydrochloric acid concentration in gastric
juice was detected. What enzyme will be less active in such a condition?

134. The patient with acute cardiac failure has developed dyspnea, tachycardia and cyanosis during
physical exertion. Name the type of hypoxia.

135. At the sixth month of pregnancy the female patient has been diagnosed with severe irondeficiency
anemia. Diagnostic character was the appearance of the following in blood:

136. The alleged diagnosis of the newlyhospitalised in-patient is leukemia. What symptom among those
given below is diagnostic character differentiating acute leukemia from chronic leukemia?

137. Tetanic spasms of skeletal muscles occur under low calcium concentration in blood. What
endocrine disorder can this condition be associated with?

138. A newborn infant has hemolytic jaundice caused by rhesus incompatibility. What bile pigment will
be concentrated highest in the blood of this infant?

139. The 55-year-old female patient has developed a case of acute pancreatitis caused by greasy food.
What is the main pathogenesis step of this disorder?

140. As the result of taking herbal medicine the 30-year-old patient has developed anaphylactic allergic
reaction and blood leukocytosis. What kind of leukocytosis is characteristic of this case?

141. The 56-year-old patient has developed megaloblastic anemia in the course of alcoholic cirrhosis.
What vitamin deficiency is the main cause of anemia in this patient?

142. The patient with alcoholic cirrhosis complains of general weakness and dyspnea. The following is
revealed: decrease of arterial pressure, ascites, dilation of stomach anterior wall superficial veins,
esophageal varicose veins dilatation, splenomegaly. What haemodynamics disorder does the
patient uffer from?

143. The patient with mushroom poisoning has developed the following symptoms: yellow coloring of
skin and sclera, dark-colored urine. Hemolytic jaundice was diagnosed. What pigment causes such
coloring of the patient’s urine?

144. The patient has icteritous skin; unconjugated bilirubin content in blood is high; conjugated bilirubin
in urine is not detected. There is significant amount of urobilin in urine and stercobilin in feces.
Name the pathology characterized by given symptoms.
145. A newborn child born from Rhnegative mother in the result of her third pregnancy presents with
gradually worsening jaundice, irritated central nervous system, anemia. What type of jaundice does
the infant suffer from?

146. The patient has been hospitalised with pneumonia. What kind of respiratory failure does the
patient have?

147. Heart rate of a person at rest is 40/min. What structure is the pacemaker of heart in this man?

148. At the sixth month of pregnancy a woman has been diagnosed with severe iron-deficiency anemia.
Diagnostic character was the appearance of the following in blood:

149. A patient has obstruction of the common bile duct. Which of these substances is usually found in
urine in such cases?

150. A patient with systemic lupus erythematosus has developed diffuse affection of kidneys followed
by proteinuria, hypoproteinemia, extensive swelling. What mechanism of proteinuria development
is the most likely in this case?

151. An elderly patient exhibits low levels of red blood cells and hemoglobin in blood, but the color index
is 1,3. Blood smear analysis revealed megaloblasts. What type of anemia is observed in this case?

152. The patient with alcoholic cirrhosis complains of general weakness and dyspnea. The following is
revealed: decrease of arterial pressure, ascites, dilation of the superficial veins of the stomach
anterior wall, esophageal varicose veins dilatation, splenomegaly. What haemodynamics disorder
does the patient suffer from?

153. Inhibition of the synthesis of bile acids from cholesterol in liver of an experimental animals has
caused maldigestion of lipids. What is the role of these acids in the enteral lipidic metabolism?

154. The patient has icteric skin; unconjugated bilirubin content in blood is high; conjugated bilirubin in
urine is not detected. There is significant amount of urobilin in urine and stercobilin in feces. Name
the pathology characterized by the given symptoms:
155. A patient complains of pain in the small joints. High concentration of uric acid is detected in his
blood plasma. What pathology causes such changes?

156. A patient suffering from coronary heart disease, who had had two myocardial infarctions of left
ventricular wall, presents with bubbling breathing and dyspnea. Pulmonary auscultation reveals
numerous moist crackles. What kind of heart failure is it?

157. A man received a radiation dose of 30 Gy. He presents with necrotic angina, disorders of the
gastrointestinal tract. Blood tests revealed anemia, leucopenia and thrombocytopenia. What
period of acute radiation sickness is observed in the patient?

158. For the specific prevention of influenza the employees of an enterprise were vaccinated with
"Influvac”. What type of immunity will develop in the body of those vaccinated?

159. A 5-year-old child presents with abdominal distension, abdominal cramps,and diarrhea occurring
1-4 hours after drinking milk. Described symptoms are caused by the lack of enzymes that break
up:

160. During ultrasound investigation a patient was diagnosed with bilateral renal artery stenosis of
atherosclerotic genesis. Specify the bioactive substance that due to its excessive secretion is the
key component of arterial hypertension pathogenesis in the given case:

161. After ishemic stroke a 67-year-old patient developed reduced mobility of the left leg. Name this
condition:

162. A 32-year-old patient with cerebellar tumor was delivered to an admission room of a hospital. The
patient presents with ataxia that can be characterized by:

163. A 40-year-old man presents with rapid weight gain after he had suffered a severe craniocerebral
trauma. At doctor’s examination the patient’s weight was 125 kg, with his weight being 175 cm.
What mechanism of obesity development is the most likely in this case?

164. A patient has developed anuria. Blood pressure is 50/20 mm Hg. What process of uropoiesis caused
acute decrease of urination?

165. Parents ofa 10-year-old child have made an appointment with endocrinologist due to complaints
of the child’s low height. The child’s appearance is corresponding with that of a Syear-old. What
hormone causes such changes in physical development, if its secretion is disrupted?

166. A patient complains of tachycardia, insomnia, weight loss, irritability, sweating. Objectively:
the patient has goiter and slight exophthalmos. What gland is affected, and what functional
disorder is it?
167. A patient has been hospitalised with pneumonia. What kind of respiratory failure does the patient
have?

168. A 70-year-old patient presents with cardiac and cerebral atherosclerosis. Examination revealed
changes of blood lipid spectre. Increase of the following lipoproteins plays a significant role in
atherosclerosis pathogenesis:

169. An elderly man exhibits low levels of red blood cells and hemoglobin in blood; however, his color
index is 1,3. Blood smear analysis revealed megaloblasts. What type of anemia is observed in this
case?

170. A patient with alcoholic cirrhosis complains of general weakness and dyspnea. The following is
revealed: decrease of blood pressure, ascites, dilation of superficial veins of the stomach anterior
wall, esophageal varicose veins dilatation, splenomegaly. What hemodynamics disorder does the
patient suffer from?

171. A patient has icteric skin; unconjugated bilirubin content in blood is high; conjugated bilirubin in
urine is not detected. There is significant amount of urobilin in urine and stercobilin in feces. Name
the pathology characterized by given symptoms:

172. A 46-year-old patient was found to have hyperactivity of creatine kinase in the blood serum. What
pathology can be suspected?

173. A patient with pulmonary carcinoma has developed a case of pleurisy. Large amount of
hemorrhagic exudate was obtained for analysis. What component is specific for hemorrhagic
exudate?

174. A patient of a neurology unit suffers from paralysis of all limbs. Name this condition:

175. A patient demonstrates symmetrical dermatitis on the palms. A doctor made a diagnosis of
pellagra. What vitamin deficiency can result in such symptoms?

176. A woman complains of nausea, vomiting, skin itch. She was diagnosed with mechanical jaundice.
What is the possible cause of skin itch in such a condition?

177. During ultrasound investigation a patient has been diagnosed with bilateral stenosis of renal artery
with atherosclerotic genesis. Specify the bioactive substance that due to its excessive secretion is
the key component of arterial hypertension pathogenesis in the given case:

178. A patient in the state of ketoacidotic coma presents with loud rapid respiration: labored expiration
with tension of expiratory muscles occurs after deep inspiration. Name the type of pathologic
respiration:
179. Parents of a 10-year-old child have made an appointment with an endocrinologist due to
complaints of the child’s low height. The child’s appearance is corresponding with that of a
5yearold. What hormone causes such changes in physical development, if its secretion is disturbed?

180. Prolonged taking of cytostatic agents resulted in development of necrotic tonsillitis in the patient.
It can be associated with the following changes in the leukocyte content:

181. A patient presents with icteric sclera and mucous tunics; urine is dark; feces are light-colored. Blood
content of direct and indirect bilirubin is increased, urine content of direct bilirubin is increased.
What pathology can be characterized by these signs?

182. A 55-year-old man, who had been suffering from mitral insufficiency for many years, developed
acute heart failure. What pathophysiological type of heart failure can be observed in this case?

183. A 54-year-old man requested a pharmacist’s advice on drug prescription. The patient has 4year-
long history of chronic glomerulonephritis and 2-yearlong history of persistent hypertension. What
substance synthesized in the kidneys has important role in development of arterial hypertension?

184. A patient complains of general weakness, muscle weakness in the extremities (if the patient is asked
to make a fist several times in a row, for example, the patient is capable of doing it only once), facial
muscles are weak, swallowing is disturbed. Administration of acetylcholinesterase drugs removes
these disturbances to a certain degree. Determine the pathology:

185. A 71-year-old woman developed mechanical jaundice due to obstruction of the bile duct with a
chololith. Decrease of blood pressure and bradycardia are detected. These changes in functioning
of the patient’s cardiovascular system are caused by increased blood content of the following
substance:

186. A 5-year-old child presents with abdominal distension, abdominal cramps, and diarrhea occurring
1-4 hours after drinking milk. Described symptoms are caused by the lack of enzymes that break
up:

187. A patient presents with hypoxia. What metabolic process activates when oxygen supply is
insufficient?

188. A child had been administered antidiphtheric serum. What resistance was formed in the child?

189. A patient with brain edema presents with respiration that is characterized by periods of several
respiratory movements of equal amplitude alternating with periods of apnea. What pathologic
respiration is it characteristic of?

190. A patient in the state of ketoacidotic coma presents with loud rapid respiration: labored expiration
with tension of expiratory muscles occurs after deep inspiration. Name the type of pathologic
respiration:
191. The patient’s large-focal myocardial infarction is complicated with pulmonary edema. What
disturbance of cardiohemodynamics contributed to the pulmonary edema development?

192. Hormones regulate numerous metabolic processes. What hormone activates glycogen synthesis?

193, A patient has developed anuria. Blood pressure is 50/20 mm Hg. What process of uropoiesis was
disturbed resulting in acute decrease of urine output?

194. A 12-year-old boy is of short stature, but his mental development corresponds with that of his age
group. What hormone deficiency is the most likely to cause this pathology?

195. A 10-year-old child has height of 178 cm and body mass of 67 kg. These presentations are caused
by the functional disturbance of the:

196. Coumarins, vitamin K antagonists, suppress the processes of blood coagulation. What protein
synthesis is blocked by coumarins?

197. Any damage to the patient’s vessels results in persistent hemorrhage. Blood clotting factor VIII is
deficient in the patient’s blood. What disease does this patient suffer from?

198. A patient presents with persistent tachycardia, exophthalmos, high excitability, increased basal
metabolic rate. What disorder can lead to the development of this syndrome?

199. On examination the patient’s sclera and oral mucosa are icteric. What biochemical blood value can
be expected to be increased?

200. A patient with acute myocardial infarction received anticoagulation therapy. What compound will
have anticoagulation effect?

201, A patient has a gallstone lodged in the common bile duct, which blocks bile supply to the intestine.
What digestive process will be disturbed in this case?

202. Ascarids were detected in a sick child. What changes in leukogram will be the most characteristic
of helminthiasis?

203. To stimulate birth activity, a certain neurohypophyseal hormone is used. Name this hormone:

204. After a traffic accident the driver presents with increased blood glucose. What mechanism leads to
hyperglycemia in this case?

205. After severe emotional strain a 53- year-old man suddenly developed acute pain in the heart area,
which irradiates to the left hand, to the neck, and under the left scapula. He noted numbness of his
left hand. His face is pale and covered in cold sweat. Nitroglycerine administration stopped the pain
attack after 10 minutes had passed. What is the most likely disease in this case?

206. During ultrasound investigation a patient was diagnosed with bilateral renal artery stenosis of
atherosclerotic genesis. Specify the bioactive substance that due to its excessive secretion is the
key component of arterial hypertension pathogenesis in the given case:

207. A patient in the state of ketoacidotic coma presents with loud rapid respiration: labored expiration
with tension of expiratory muscles occurs after deep inspiration. Name this type of pathologic
respiration

EXAM BASE 2

1. What diseases of sex linked type of inheritance?

2. What diseases of autosomal-dominant type of inheritance?

3. What diseases of autosomal-recessive type of inheritance?

4. What diseases of congenital (teratopaty) type of inheritance?

5. What forms of endogenous mutagens do you know?

6. What diseases caused by chromosomal aberrations?

7. The main methods of diagnosis of hereditary diseases?

8. The main methods of treatment of hereditary diseases?

9. The main methods of prevention of hereditary diseases?

10. Levels of lg was changed during examination of immune status in 5-year-old boy. Reactions of
T-lymphocytes are normal. This is inherited sex-linked disease. Was diagnosed. Bruton’s
disease. What manifestations of this pathology do you know?
11. A 9-year-old girl was getting breast feeding during the first year of life. She suffered prolonged
severe pneumonia in the end of the first year. She started walking late. Was diagnosed Luis-
Barr syndrome. Which changes are defining this syndrome?

12. What symptoms are characterased for the Wiscott-Aldrich’s syndrome?

13. What symptoms are characterized for the DiGorge syndrome?

14. What can increase permeability of hematoencephalic barriers?

15. Which factors can stimulate phagocyte activity?

16. Which factors decreases phagocyte activity?

17. Which clinical forms of allergic reactions belong to the | type (anaphylactic) by Gell and Cumbs?

18. Which clinical forms of allergic reactions belong to the Ill type — immunocomplex reactions by
Gell and Cumbs?

19. What are the diseases developing by the immunocomplex type:

20. What are the delayed-type hypersensitivity mechanisms:

21. List the mechanisms of action of eicosanoids:

22. Ig E are involved in:

23. What mediators are associated with an increase in vascular permeability:

24. What are some examples of delayed-type hypersensitivity:

25. What manifestations of venous hyperemia?

20. What is microscopic picture characterized by arterial hyperemia?

27. Discribe Virchow’s triad.

28. What manifestations of ischemia....


29. What significances of arterial hyperemia....

30. What manifestations of thrombosis do you know?

31. What manifestations of embolism do you know?

32. Check reasons spastic type of ischemia

33. What reasons of fat embolisms do you know?

34. What reasons of gas embolisms do you know?

35. What reasons of air embolisms do you know?

36. In the development of pain with inflammation involved:

37. The products of arachidonic acid metabolism via the cyclooxygenase pathway are:

38. An increase in vascular permeability during inflammation causes:

39. From the phospholipids of cell membranes are formed:

40. Statements characterizing histamine:

Al. Following emigration from blood vessel, leucocyte migration to the site of injury is mediated
by

42. Which humoral mediators do you know?

43. Local manifestations of inflammation?

44. Systemic manifestations of inflammation?

45. What components of exudate do you know?

46. What components of transudate do you know?

47. Names factors which suppress prolipheration?


48. Acute increase of body temperature, dyspnea, tachycardia, nausea, convulsions, and loss of
consciousness developed in a worker, working in the thick uniform in summer. In a worker
hyperthermia develops. What reasons of hyperthermia do you know?

49. A patient has flu. Which manifestations will characterize 1 stage of fever?

50. Manifestations second stage of fever....

51. A sick man felt on work general weakness, temperature increased to 38 C°. In evening the sick
felt sever fever, temperature reached 38.8 C°. Family doctor diagnosed influenza. What
manifestations third stage of fever do you know?

52. Positive effects of fever?

53, Negative effects of fever?

54, What breathing is observed at the temperature of body 40°C?

55, Which primary Pyrogens do you know?

56. What substances will play role secondary Pyrogens?

57, Cardinal signs of benign tumor are:

58. Name endogenous cancerogens:

59. Check of the systemic effects of tumors:

60. What cardinal signs of malignant tumor do you know?

61. Manifestations of biochemical anaplasia:

62. Manifestations of immunological anaplasia:

63. What are the of immune protective mechanisms against neoplasia:

64. Which mechanisms of cancer cachexia do you know?

65. What periods in pathogenesis of cancer do you know?


66. A woman 45 years old has the state of excitation with the feeling of starvation is the 1 period
of starvation. Which manifestation 1 period of starvation do you know?

67. Peculiarities of absolute starvation?

68. A man_55 years old has the state of excitation with the feeling of starvation is the 1 period of
starvation. Which manifestation 2 period of starvation do you know?

69. A woman 35 years old has the state of excitation with the feeling of starvation is the 1 period
of starvation. Which manifestation 3 period of starvation do you know?

70. Aman 48 years old with use fasting with therapeutic aim. What diseases are treated by medical
fasting nowadays?

71. Peculiarities of qualitative starvation of proteins?

72. Peculiarities of quantitave incomplete starvation of proteins?

73. Peculiarities of qualitative incomplete starvation of vitamin PP:

74. Peculiarities of qualitative incomplete starvation of vitamin B2

75. Peculiarities of qualitative incomplete starvation of vitamin B6

70. Peculiarities of qualitative incomplete starvation of vitamin C

77. Peculiarities of qualitative incomplete starvation of vitamin D

78. What hormones elevate the blood glucose level?

79. Describe changes in protein metabolism in Diabetes 1 type?

80. Describe peculiarities in Diabetes 1 type?

81. Describe peculiarities in Diabetes 2 type?

82. Describe metabolic disorders in diabetes ketoacidosis development:

83. In what cases does relative insulin insufficiency is occurred?


84. In what cases does absolute insulin insufficiency is occurred?

85, What causes can lead to hypoglycemia?

86. In what cases does absolute insulin insufficiency is occurred?

87, What manifestation of hypoglycemic coma _ do you know?

88. What other pathological processes can be accompanied by the hypoosmolar hyperhydration?

89. What other pathological processes can be accompanied by the hyperosmolar hyperhydration?

90. Myxedema is characterised by....?

91. Lung edema was modeled in rat using solution of ammonium chloride. This type of
edema is characterised by...

92. Reasons of hypernatriemia are:

93. Etiological factors of hyperkaliemia are :

94. For hypocalciemia all symptoms are thypical except:

95. Which clinical symptoms don't correspond to hyponatriemia ?

96. Etiological factors of hyperkaliemia don't include:

97. Clinical manifestation of hypercalcemia don't include some of next:

98. Metabolic acidosis develops with:

99. Severe hypercapnia with gas acidosis leads to:

100. What are the mechanisms of compensation for metabolic alkalosis?

101. List the pathological processes leading to the development of excretory acidosis:

102. The causes of metabolic acidosis are:


103. The causes of respiratory alkalosis are:

104. What conditions lead to the development of excretory acidosis:

105. What reasons can lead to the non-gas excretory acidosis?

106. What reasons can lead to the metabolic acidosis?

107. What characteristic parameters of acid-base imbalance is observed in sailors who continue to
work during submarine catastrophe?

108. What reasons can lead to the respiratory excretory alkalosis?


109. What reasons can lead to the non-gas excretory acidosis?

110. What forms of buffer systems do you know?

111. A patient has chronic posthemorrhagic anemia which is accompanied with reduce of
concentration of serum iron. What changes are the most likely in this case?

112. What conditions lead to polycythemic hypovolemia?

113. What conditions lead to oligocythemic hypovolemia?

114. What conditions lead to oligocythemic hypervolemia?

115. What are the causes of olygocythemic normovolemia ?

116. What are the mechanisms of blood volume restoration after hemorrhage?

117. What are the reasons of normalization of blood pressure at hydrous phase after haemorrhage?

118. Which reasons of Erythrocytosis do you know?

119. Severe intravascular hemolysis is manifested by...

120. Beta-thalassemia is characterised by:

121. Sykcle cell anemia is characterised by :

122. What factors may cause iron deficiency anemia?


123. Which changes in blood are characterized lron deficiency anemia?

124. Which changes in blood are characterized Addisone-Birmer anemia?

125. A patient has stomach resection. Later vit B12 deficiency anemia develops. What
manifestations are characterized this pathology?

126. Megaloblastic anemia was characterised by...

127. A patient with lympholeucosis complains of the frequent bleeding from nose. Was diagnosted
Insufficiency of vascular-thrombocytic hemostasis What change of the blood will be most
characterized this pathology?

128. Mechanisms and reasons of leucopenia...

129. Leukopenia may be a result of...


130. During medical examination of boy 4 years old with weight 18 kg the considerable increase of
number of eosinophils in unit of blood volume was revealed. What can be the reason of
eosinophilia?

131. In a child 13 years old leukocytosis developed after the protracted training (12,0¢109/l).
Development redistributive leukoytosis. Check all possible mechanisms of redistributive
leukocytosis...

132. The nuclear regenerative shift to the left is...

133. The nuclear shift to the write is....

134. What changes occur in peripheral blood at chronic lymphoid leukemia?

135. Which cells are increase in peripheral blood at chronic myeloid leukemia?

136. Chronic myeloid leukemia is associated with...

137. Violation thrombocytovascular hemostasis.

138. Lack of coagulative hemostasis.

139. Causes and mechanisms for violations of certain stages of blood clotting
140. Thrombotic resistance of the vascular wall due to:

141. Activation of platelet adhesion contribute to:

142. Immune thrombocytopenic purpura is characterized by:

143. Hemophilia A is characterized by:

144. Primary anticoagulants:

145. Thrombocytopenia due to a violation of platelet production in the bone marrow is observed:

146. Hemorrhagic syndrome in violation of vascular-platelet hemostasis may be due to:

147. The condition of coagulation hemostasis can be estimated:

148. The state of vascular platelet hemostasis can be assessed

149. Disseminated intravascular coagulation syndrome of blood pathogenesis.

150. Disseminated intravascular coagulation syndrome of blood etiology


151. Mechanisms of long-term adaptation to cardiac overload

152. Mechanisms of immediate adaptation to cardiac overload

153. Decompensation of heart insufficiency is characterised by ...

154. Heart insufficiency because myocardium damage can arise at ?

155. Nomotopic violations of heart automatism include such arrhythmias...

156. Which are the main risk factors of myocardium infarction development?

157. The out of heart mechanisms of cardiac insufficiency compensation include ?

158. What arrhythmias do arise as a result of conduction violation?

159. Carcinogenic shock. Pathogenesis of its individual species.


160. Which are the basic indexes of the first stage of myocardium hypertrophy ?

161. Atherosclerosis (according to HWO’s determination) is the result of accumulation in arteries of


va?

162. Arterial hypertensions. Etiology and pathogenesis and specific manifestations

163. What are causes of asphyxia?

164. What kind of impaired content of gases in the alveolar air and blood occurs in ventilatory
respiratory insufficiency?

165. What kind of impaired content of gases in the alveolar air and blood occurs in parenchymal
diffusive respiratory insufficiency?

166. Patient was diagnosed a considerable reduction of the perfusive ability of the lungs. What are
possible causes of this disorder?

167. Patient was diagnosed a considerable reduction of the diffusive ability of the lungs. What are
possible causes of this disorder?

168. What else types of respiration belongs to terminal (agonal) respiration?

169. Periods of respiration are substituted by periods of apnea. What types of respiration belongs
to this group?

170. What causes lead to restrictive respiratory insufficiency?

171. What reasons of tachypnea do you know?

172. What reasons of hyperventilation do you know?

173. What reasons of terminal respirations do you know?

174. Causes and mechanisms of pulmonary edema?

175. What nonrespiratory functions of the lungs do you know?

176. What are protective factors, which act in the stomach?


177. What are simulative factors, which act in the stomach?

178. Choose factors, which determine genetic predisposition in ulcer disease.

179. A 67-year-old woman, who has cholecystitis for a long time, suddenly developed sharp pain in
the upper part of abdomen, nausea, and vomiting after food intake. Acute pancreatitis was
diagnosed in this patient. What are the peculiarities of acute pancreas inflammation?

180. Experimental animal was fixed on the fixing tool and left without nutrition for day. After
dissecting in stomach the ulcers are found. What mechanisms of it ulcerogenic acting?

181. Which negative complications of hyposalivation do you know?

182. Which negative complications of Hypersalivation do you know?

183. Etiology and pathogenesis of gastric ulcer and duodenal ulcer.

184. Violation of cavernous digestion in the intestine, causes, mechanisms and manifestations.

185. Disorders associated with secretary pancreatic insufficiency.

186. Pancreatitis: types, causes, pathogenesis of acute pancreatitis.

187. Pancreatic shock.

188. Malabsorption. Causes and mechanisms of malabsorption, the basic pathogenesis of clinical
manifestations

189. Which mechanisms belong to the pathogenesis of hepatic coma.

190. What manifestations of acholic syndrome?

191. What manifestations of cholemic syndrome?

192. Specific manifestations of portal hypertension do you now?

193. Lack of hepatic excretory function: causes, mechanisms and clinical implications

194. Etiological factors contributing to the development of polyuria are:


195. Etiological factors contributing to the development of oliguria are:

196. Pathogenetically, the development of anemia in kidney pathology is associated with:

197. Pathogenetically, "urinary syndrome" is due to the development of the patient:

198. The reasons for the development of prerenal acute renal failure may be:

199. The most characteristic manifestations of urolithiasis are:

200. The development mechanism of acute renal failure includes:

201. In the pathogenesis of renal edema, the leading role is played by:

202. In typical cases of pyelonephritis, the following clinical syndromes are distinguished:

203. The mechanism for the development of nephrotic syndrome is:

204. The renal symptoms of acute glomerulonephritis include:

205. Uremic cardiopathy is characterized by:

206. Violation of acid-base balance with uremia manifests itself:

207. The renal factors of kidney stone formation are:

208. Which of the following indicators characterize the renal tubule dysfunction?

209. Indicate the main mechanisms contributing to the development of renal hypertension:

210. The urinary syndrome includes:

211. Which complications are characterised nephrotic syndrome?

212. Which complications are characterised nephritic syndrome

213. A patient aged 41 complains of weakness, sweating, fever, tremor of hands, BP160/90 mm Hg.
Diffuse toxic goiter was diagnosed (Basedow’s disease). What are the mechanisms of
impairment of the functions in this disease?
214. Enumerate complex of non-specific signs of damage which are caused by prolonged
nonspecific adaptive reactions...

215. A patient aged 23 complains of headache, changes of his appearance (increase of size of feet,
fingers, features of the face), hoarseness of the voice, worsening of the memory. The disease
has begun 3 years ago without any causes. Increase of superciliarry arches, nose, and tongue
were found during physical examination of this patient; the blood sugar was normal; the
urinalysis is out of changes. Pituitary gigantism was diagnosed. What are the pathogenesis in
this disease?

216. Parathyreoid gland are removed during the operation on thyroid gland. What pathological
changes will be observing after this operation?

217. A woman with primary hyperparathyroidism has periodically repeated attacks of renal colic.
Ultrasound examination shows presence of small stones in her kidneys. What pathological
changes will be observing in this disease?

218. Persistent hyperglycemia developed in a patient with Cushing’s syndrome under the influence
of excessive amount of glucocorticoids. What are manifestations of this syndrome?

219. Which of the following characterizes Cohn's syndrome?

220. The development of Addison's disease causes a decrease in the number of the following
hormones

221, With pituitary nanism, it is noted

222. The characteristic changes that occur in the blood system under the influence of glucocorticoids
include:

223. Indicate the mechanism of change in blood glucose in Addison's disease:

224. Transhypophysial regulation is the basis for:

225. The production of which hormones decreases with total hypofunction of the anterior pituitary
gland:

220. Which of the following changes in women testify in favor of congenital adrenogenital
syndrome:
227. What are the characteristic manifestations of central paralysis in the affected limbs? 228. What
signs characterize peripheral paralysis?

229. List the exogenous causes of damage to the nervous system:

230. List the conditions that determine the strength of damage to the nervous system:

231, Damage to the cerebellum may be accompanied by:

232. What signs characterize peripheral paralysis?

233. In the pathogenesis of neurogenic degeneration of cells of a denervated organ, the following
processes are important:

234, List the specific mechanisms of damage to neurons:

235. What is characteristic of a neurodystrophic process

236, Life-threatening complications in shock are due to hypoxic cell injury resulting in immuno-
inflammatory responses and activation of various cascades (clotting, complement, kinin). These
include the following...

237. The pathogenic factors of hemorrhagic shock include...

238. Torpid phase of hemorrhagic shock is characterized by...

239. The pathogenic factors of anaphylactic shock include...

240. The pathogenic factors of paine shock include...

241, The pathogenic factors of pancreatic shock include...

242. The pathogenic factors of hepatic coma include...

243, The pathogenic factors of cardiogenic shock include...

244, What manifestations of ketonemic hyperglycemic coma do you know?

245. What specific manifestations of lactacidemic hyperglycemic coma do you know?


246. What manifestations of hyperosmolar hyperglycemic coma do you know?

247. The pathogenic factors of pancreatic shock include...

248. The pathogenic factors of collapse include...

249. Collapse: general manifestations

153.Whatarethereasonsofnormalizationofbloodpressureathydrousphaseafterhaemorrhage?

veetoniectcumtchoresans nT nmelnrecaueanepraneretertarg

154.WhichreasonsofErythrocytosisdoyouknow?

chroniccommonhypoxia,ischemiaofthekidneys

155.Apatientcomplainsofdyspneafortherapidwalking.Askinispale,cold.Resultsofanalysis: erythrocytes—
3,2°1012/|,hemoglobin—90g/I, colourindex—0,6,contentsofproteins—72g/l.Inthe
smearofbloodtherearemuchanulocytesandmicrocytes,singleoxyphilicnormocytes.Whatmayleads
tosuchdisorders?

156.Severeintravascularhemolysisismanifestedby...

methemalbumine
mia
hemoglobinuria
b)jaundice
157.Beta-thalassemiaischaracterisedby:
familialoccurrence,increaseddestructionofmaturingerythroblastsinthe
bonemarrow,anemiadevelopment,reductioninthesynthesisofbeta-
globin chains

158.Sykclecellanemiaischaracterisedby:

itisaccompaniedbysplenomegaly,

itresultsfromabnormalhemoglobinsynthesis
159.Whatfactorsmaycauseirondeficiencyanemia?

depletionofintrinsicironstores,impairmentoffolicacidactivation,
adecreasedproductionofhydrochloricacidbygastricmucosa,an
increasedirondemands
160.Whichchangesinbloodarecharacterizedlrondeficiencyanemia?
colourindex<0.7,reticulocytes1%;,anisocytosiswithapredominanceofmicrocytes

161.WhichchangesinbloodarecharacterizedAddisone-Birmeranemia?

neutropeniavaryingdegrees;
b)hyperchromiaoferythrocytes;
c)leukopenia, thrombocytopenia;
d)atrophicglossitis;
esymptomsoffunicularmyelosis
Itisdyserythropoieticanemia,characterizedbyshiftofPrice-Jonescurvetothe

Right,|Increasedcolorindex

162.Apatienthasstomachresection.LatervitB12deficiencyanemiadevelops.Whatmanifestationsare
characterizedthispathology?

increaseinserumiron
megaloblastictypeofhemopoiesis;
d)macracytosis;
e)presenceofRBCswithJollybodiesandCabotrings;
f)anisocytosiswithapredominanceofmacracytes

decreaseofsyntesisofnucleicacids,
163.Inaninfant,whoisunderanartificialnutritionwithcowmilk,severeanemiahasdeveloped.Atthe
bloodcountoftheinfant:numberoferythrocytesis4x1012/L,contentofhemoglobinis68g/L,
reticulocytesof0%. Whatkindofanemiadevelopedintheinfant?lrondeficiencyanemia

164.Megaloblasticanemiawascharacterisedby...

folicaciddeficiencyanemia,intrinsicfactordeficiency,vitaminB12-
deficiency anemia,
increaseinserumiron
macrocytosis;
presenceofRBCswithJollybodiesandCabotrings;
anisocytosiswithapredominanceofmacrocytes

165.PatientS36yearsoldhasaplasticanemia. Whatchangesinbloodwillbe?

peripheralcytopenias
166.Apatientwithlympholeucosiscomplainsofthefrequentbleedingfromnose.Wasdiagnosted
Insufficiencyofvascular-thrombocytichemostasisWhatchangeofthebloodwillbemostcharacterized
thispathology?

167.Resultsofbloodtestsofthepatient:erythrocytes—1,2°1012/I,concentrationofhemoglobin—48g/I,
colourindex—1,2.Reticulocytesisabsent,anisocytosis, poikilocytosis,singlemegalocytes,.Itwas
diagnosed:B12-(folate)deficiencyanemia.Namethetypeofanemiaaccordingtoregeneratory
possibilities,andwhatchangeofthebloodwillbemostcharacterizedthispathology?

168.Amanappealedtothedoctorwithcomplaintsofappearanceofhypodermichematomas.Intheblood
test:erythrocytes—2,5¢1012/I;hemoglobin—80,0g/|;colourindex—0,9.Leucocytes—5,8°109/I.
Leucocyticformula:myeloblasts—6%,myelocytes—10%,metamyelocytes—18%,stabneutrophils—26

%,segmentonucleonicneutrophils—10%,limphocytes—10%,monocytes—2%.WasdiagnostChronic
myelocyticleucosisWhatchangeofthebloodwillcharacterizedthispathology?

169.A20years-oldpatienthastransfusionreactions.Itscytotoxictypeofallergy.Checkpathologyof
CytotoxiccomponentsofCytotoxicantibodiestobloodcellse.g.
TCRshavetwo parts,usuallyanalphaandabetachain.(SomeT CRshaveagammaandadeltachain.)
Hematopoieticstemcellsinthebonemarrowmigrateintothethy mus, wheretheyundergoV(D)Jrecombinati
onof theirbeta-chainT CRDNAtoformadevelopmentalformoftheT CRprotein,knownaspre-TCR
170.Mechanismsandreasonsofleucopenia...

suppressionofleucopoiesis, increasedleukocytedestructioninthevascular
bed.
increasedmarginationofleukocytesinthevascularbed

171.Duringmedicalexaminationofboy4yearsoldwithweight18kgtheconsiderableincreaseofnumber
ofeosinophilsinunitofbloodvolumewasrevealed.Whatcanbethereasonofeosinophilia?

172.Leukopeniamaybearesultof...

insufficiencyofvitaminB12

b)overwhelmingbacterial,fungalorrickettsialinfections
c)antimetabolitesusedincancertreatment

173.Inachild13yearsoldleukocytosisdevelopedaftertheprotractedtraining(12,0°109/l).Development
redistributiveleukoytosis.Checkallpossiblemechanismsofredistributiveleukocytosis...

174.Thenuclearregenerativeshifttotheleftis...

Shiftthatoccursafter6—8daysafteracutebloodlossofmoderateseverity:

175.Thenuclearshifttothewriteis....

Increaseinhypersegmentationofneutrophilsnuclei;
c)cytoplasmicvacuolation

176.Whatchangesoccurinperipheralbloodatchroniclymphoidleukemia?
smallcountoflymphoblastes thepresenceofBotkin-
Gumbrechtbodies

177.Whichcellsareincreaseinperipheralbloodatchronicmyeloidleukemia? myeloblastes

b)matureneutrophiles(segmentes
178.Chronicmyeloidleukemiaisassociatedwith...

asmallamountofmyeloblasts
c)increasedamountofbasophilesandeosinophiles
d)absenthiatusleukemicus
179.Violationthrombocytovascularhemostasis.

180.Lackofcoagulativehemostasis

- 61 Following emigration from blood vessel, leucocyte migration to the site of injury is
mediated by:

the capillary wall between endothelial cells


the basal membrane

- 62 Which humoral mediators do you know?

- complement system
- kinins ( bradykinin,kallidin)
- Hageman factor

63 Local manifestations of inflammation?

- tumor ( swelling )
- rubor (redness )
- calor (heat )
- dolor ( pain )
- funcito leasa ( functional disorder )

- 64 Systemic manifestations of inflammation?


- Increase in body temperature (fever) is one of the most prominent systemic manifestations,
especially when inflammation is associated with bacteremia.

- Endocrine system activation.

- Hypothalamus involvement in fever development, adenopituitary and adrenal cortex


activation.

- Mineralocorticoids stimulate inflammation, and glucocorticoids inhibit it.

- Hematological syndrome is characterized with neuptrophilic leukocytosis,which 1s a


common feature of inflammatory reactions, especially those induced by bacterial infection.
The leukocyte count usually climbs to 10 G/1 and more. Leukoformula shows a regenerative
left-side nuclear shift.

- Increased number of neutrophils in leukoformula and appearance of their 1m- mature forms
are an evidence of hemopoiesis activation in the bone marrow.

- Blood sedimentation rate (BSR) is accelerated.

- Immune system activation.

- Nervous disturbances, insomnia.

- So, inflammation, immunity and fever are tightly connected. INFLAMMATION

- 65 What components of exudate do you know?

- plasma proteins (albumins,globumins, fibrinogen)

- serum,fibrin, woc

- 66 What components of transudate do you know?

plasma,glucose,mesathelial cell and cellular debris

- 67 Names factors which suppress prolipheration?

- inhibition of FOXM1 transcription factor

Tumor suppressor genes


- chalones

- 68 Acute increase of body temperature, dyspnea, tachycardia, nausea, convulsions, and


loss of consciousness developed in a worker, working in the thick uniform in summer.
In a worker hyperthermia develops. What reasons of hyperthermia do you know?

limitation of heat emmision

activation of heat production

- disconnection (uncoupling of oxidation and phasphospharylation)

- 69 A patient has flu. Which manifestations will characterize 1 stage of fever?

- hp>he (p production ,e emission)

- body temperature increase

- Hairs become elevated

chills develop

- Diuresis is increased

- artirial BP rise

- Vasoconstriction

- 70 Manifestations second stage of fever....

-maintaince of high body temperature with fluctuation in morning and evening

Hp=He

- dures is decrease

Retention of Na, cl and water is detect

- vasodilation

- 71 Extensive myocardium infarction was diagnosed in a patient. By the end of day


body temperature increased to 38 Co. The number of leucocytes increased to 11.6*109.
Which internal Pyrogens do you know?

- Interleukien 1-6

- production price f 1°2° atteration


Antigen antibody complex

PGE1,2

tumor necrosis factor

Mitabolits of steroid hormones

Leukocyte pyrogen that are prduct of neutrophils and macrophages

Interferons -alpha

gp130
72 A sick man felt on work general weakness, temperature increased to 38 Co. In
evening the sick felt sever fever, temperature reached 38.8 Co. Family doctor diagnosed
influenza. What manifestations third stage of fiver do you know?

He>Hp

Loss of water

Na and cl excretion increase

Vasodilation

Intense sweating

73 Positive effects of fever?

multiplications of many virus is inhibited

phagocytic activity of macrophages and neutophiles increase


Antibody synthesis become more Mitabolits of steroid hormones
intense
Leukocyte pyrogen that are prduct of
neutrophils and macrophages

74 Negative effects of fever? Interferons -alpha

Disturbance of patients general gp130

condition - malaise , headache

metabolic disorders
78 Both benign and malignant
Increase heat load or decrease in bp tumours cause local effects on the
host due to their size or location.
Disorder of CNS Malignant tumours due to rapid and
invasive growth potential have more
Fetal development disorder serious
effects. Check of the local effects of tumors

75 What breathing is observed at the


temperature of body 40.C? Compression

tachypnea ( polypnea ) Tissue distruction

rapid or quick breathing Hemorrhage

mechanical
hyperventilation
Infraction
76 Which primary Pyrogens do you
know? Bleeding

Lipo polysaccharides obstruction

Virus and other microbes Ularation

Exerotoxin , and erythrogenia toxin Abnormal biochemistry

Exogenous protein Distruction of adernal cortex by tumor

Antigen breakdawn products ( leads to increase aldosterone and


eticocholanolone)
cortisol sectetion - Cause lactic acidosis
77 What substances will play role
Serum lactate DH increase
secondary Pyrogens?

Like increase serum rate concentration


Interleukien 1-6
Renal failure may occur
production price f 1°2° atteration
79 A 56-years-old patient, who had
Antigen antibody complex
contact with diethyInitrozamine at
PGE1,2
his work place, complains of pain in
right subcostal area, weakness, loss
tumor necrosis factor of appetite, and decreased
workability. At examination of this
patient: surface of his liver is rough, mucopolysacccharid
splenomegaly and ascites are present e which resembles
in him; his body temperature is heparin
37.20C; in his hypochromic anemia plasminogen
were found. Blood analysis ESR is 25 activation
muiv/hour, besides neutrophilic
leukocytosis. Was diagnosed Cancer
2. Activation of platelet adhesion
of liver, because this cancerogen has
contribute to:
organotropic effect irrespective of
Slowing of blood flow
the way of entry. Which
The reversalof electrical
manifestations are characterized
malignant cancer?
charge in the intima from
—tot+
- Monoclonality of origin Contact of platelet
processes with collagen
- 80 Cardinal signs of benign tumor
are: 3. Immune thrombocytopenic
purpura is characterized by:
- Well different cell purple rash red or purple dots
called petiache reduction in
- Retain typical structure of tissue circulating thrombocyte
because of autoimmune
- Grow slowly may stop and sometimes
agression
reverse development
4. Hemophilia A is characterized by:
- Have a capsule and grow expansily
Deficiency of factor 8
- Aere not lethal Is x linked
eg
Ee
TMP

Causes and mechanisms for


violations of certain stages of
blood clotting
1* phase Vii defieciency, ix
tae

deficiency,xiii deficiency
palte;et factor 3
toom

2-4 phase deficiency of


factor ii, factor v
3" phase afibinogenemia,
hypofibrinogenemia
Be

1. Thrombotic resistance of the


vascular wall due to:
mvome

produvtion of
prostsglandin,
destruction of the
agent which
SP

stimulate
thrombocyte
oc

aggregation such as
ADP production of
sce
Abnormalities of normal
oceoemmraacnrms
oS cells
7. Hemorrhagic syndrome in
violation of vascular-platelet
hemostasis may be due to:
Lack of plasma
procoagulants
Surplus of anticoagulants
Activation of fibrinolysis
Deficit of platelets
Pathology
8. The condition of coagulation
Ba

hemostasis can be estimated:

9. The state of vascular platelet


wMne

hemostasis can be assessed

10. Disseminated intravascular


coagulation syndrome of blood
pathogenesis.
Stage of hypercoagulation
Be

Stage of hypocoagulation
eee

11. Disseminated intravascular


coagulation syndrome of blood
etiology
Massive burn
FOR

Obstetrical pathology,
premature placental
POOF

removal manual placenta


removal
Acute intravascular
erythrocyte hemolysis
Leukemia
HS

True erythrocytosis
peP

Uremia
Generalized infecyions
"AO

sepsis
Acute pacreanecrosis
Heparin Peritonitis
Antithrombin 3 Side effects of the
Alpha 1 antitrypsin, alpha medicicne
2 macroglobulin 12. Study findings showed that at 30
6. Thrombocytopenia due to a days after surgery, patients with
violation of platelet production in obstructive sleep apnea had a 49%
the bone marrow is observed: higher risk of...
Anemia
Leukopenia 13. Mechanisms of long-term
Damage to stem cells adaptation to cardiac overload (pg
343) heart enlargement
stage of emergency
hyperfunction
stage of
complete
hypertrophy
and
relatively
stable
hyperfunctio
n stage of
decompensat
ion
14. Mechanisms of immediate
adaptation to cardiac
overload(pg303 new book)
Heterometric chronoionotropic
inotropic homotropic
15. Decompensation of heart
insufficiency is characterised by
...(pg344)
Impaired
myocardial
trophicity
Hypoxia
Cardiosclerosis
Accumulation of
ca2/

16. Heart insufficiency because


myocardium damage can arise at
? (pg346)
Stenosis
Occlusion of vessel
Physical exertion or
emotional stress
17. Manifestations of 4-th stages of 23. What clinical triad does
atherosclerosis are ? characterise Dresler’s syndrome,
which arises after carried
myocardium infarction?
18. Nomotopic violations of heart pericarditis,
automatism include such pleurisy,
arrhythmias. ..(pg 348) pneumonia
Bradycardia
Tachycardia
Respiratory 24. Which manifestations of Left side
cardiac insufficiency do you
19. Which are the main risk factors of know? Pg 353
myocardium infarction Blood congestion in veins of pulmonary
development?(Wikipedia) High circulation lung edema
blood pressure, smoking, Cardiac asthma syndrome
diabetes, lack of exercise, 25. Which manifestations of Right
obesity, side cardiac insufficiency do you
high blood know? Pg 354
cholesterol Veins of systemic liver
enlargement edema on legs
ascites anarsaca
20. The out of heart mechanisms of 26. Carcinogenic shock. Pathogenesis
cardiac insufficiency of its individual species. Pg 334
compensation include ? new book
Caval and
pulmonaryveins Reflex form:
increase pressure on prolonged pain
inflow pathways activation of
Aorta and carotid sinus decrease sympatho adrenal
pressure on the outflow pathways system
and subsequent decrease in the Hypokinetic form:
reflux depressor effect Pericardium sharp increase in
irritated cardiac function
21. What are the components of so- Dyskinetic
called ,,lipid triad”, which is the form:mismatch of
one group of mechanisms of cardiac contractions
myocardium necrosis? Arrhythmic forms:
Ldl it is a consequence
Hdl of severe
Triacylglycerides arrhythmias

27. Hypertrophied heart of sick man


22. What arrhythmias do arise as a is difference to hypertrophied
result of conduction violation? heart of sportsman by...
Idioventricular rhythm Sportsmans has
O Due to blocks: Balanced hypertrophy in
intratrial which all components of
atrioventricular the heart are uniformly
intraventricular increased Functional
reserves of the heart are
significantly increased
Adrenalglanddisease

130.Thecausesofmetabolicacidosisare:

Completestarvation,shock,heartblock,res
125.Clinicalmanifestationofhypercalciemiado piratoryfailure, DM,Accumulationofacid
n'tincludesomeofnext: metabolites, pHco2HCO3areallreduced

¢ Excessivethirstandfrequenturination....
* Stomachpainanddigestiveproblems.... 131.Thecausesofrespiratoryalkalosisare:
¢ Bonepainandmuscleweakness....
¢ Confusion,lethargy,andfatigue.... Hyperventilation,cnsdamage
¢ Anxietyanddepression.... 132.Whatconditionsleadtothedevelopmentof
¢ Highbloodpressureandabnormalheartrhyt excretoryacidosis:
hms.
* Muscleacheandtwitches(tone) Kidneyorlunginsufficiency

Readthequestionverywellsotheanswertot 133.Whatreasonscanleadtothenon-

hequestionareoptionsthatare gasexcretoryacidosis ?Questionproblem


nothere 134.Whatreasonscanleadtothemetabolicacid
126.Metabolicacidosisdevelops
osis?
with
Completestarvation,shock,heartblock, respira
whenthebodyproducestoomu toryfailure,DM,Accumulationofacid
metabolites, pHco2HCO3areallreduced
chacid.
135.Whatcharacteristicparametersofacid-
whenthekidneysarenotremovingenougha baseimbalanceisobservedinsailorswhocontin
cidfromthebody ue toworkduringsubmarinecatastrophe?
lossofbicarbonate Increasedco2

127 .Severehypercapniawithgasacidosisleadst DecreasedpH


O:
Respiratoryacidosis
Respiratoryacidosis

128.Whatarethemechanismsofcompensation
formetabolicalkalosis: 136.AriseinthebloodpHoccurringduetolowere
dPco2consequenttohyperventilationofthe
Hypoventilation, lungs(excessremovalofCO2)iscalledrespirator
metabolicalkalosis yalkalosis. Thisoccursinthefollowing
ishypoventilationleadingtoapHcorrectiontowa conditions...
rdsnormal.Increasesin Hyperventilation,cnsdamage
arterialbloodpHdepressrespiratorycenters.Th
eresultingalveolarhypoventilationtendsto 137.AfallinthebloodpHoccurringduetoraisedP
elevatePaCO2andrestorearterialpHtowardnor co2consequenttohypoventilationoflungs
mal. (CO2retention)causesrespiratoryacidosis.This
canoccurinthefollowingcircumstances...
129.Listthepathologicalprocessesleadingtoth
edevelopmentofexcretoryacidosis: Chronicobstructivediseaseoflungs, respiratory
insufficiency,cardiopulmonaryinsufficiency,
Diarrhea
drugabuse,submarineaccident,asphyxia,cnstr
Renaldysfunction
aumaordamage,accidentinclosedspace, Thrombosisbloodcoagulation
traumaofbreast
Redistributionofblood
138.Whatreasonscanleadtotherespiratoryexc
Restorationofcirculatoryvolumeanderythrocy
retoryalkalosis ?Questionproblembutmaybe
answerishyperadrenocorticism(cushingsyndr tecontentduetobloodreserve
ome),aldosteronism,wastingHCLunder
143.Whatconditionsleadtopolycythemichypo
vomiting
volemia?
139.Whatreasonscanleadtothenon-
gasexcretoryacidosis?Questionproblembutm Dehydrationoforganism,diarrhea,vomiting,hy
aybe perventilation,increasedperspiration
answerispulmonaryhyperventilationincrease
144.Whatconditionsleadtopolycythemichype
dco2excretion
rvolemia?
140.Whatformsofbuffersystemsdoyouknow?
Hypobaria,Reductionofatmosphericpressure,
Proteinbuffersystem
heartfailure,emphysema,oxygendeficiency
Phosphatebuffersystem
145.Whatconditionsleadtooligocythemichyp
Carbonicacidbicarbonatebuffersystem
ovolemia?
Hemoglobinbuffersystem
Acutebloodloss,
141.Apatienthaschronicposthemorrhagicane
miawhichisaccompaniedwithreduceof 146.Whatconditionsleadtooligocythemichyp
concentrationofserumiron.Whatchangesaret ervolemia?
hemostlikelyinthiscase?
Waterretention,renaldisorders,rejectedblood
lrondeficiencyanemia substitutes

Lowhemoglobincount 147.Whatarethecausesofolygocythemicnorm
ovolemia?
Tissuestarvationofoxygen
Anemiasofdifferenttypes
Erythrocytescanbedecreasedinsizeandamoun
t 148.Whatarethefactorsofthenegativeoutcom
eofthebleeding?
Hyporegenerativeanemia
Sizeofdamagedvessel
142.Apatientwasbroughtwithmassivebloodlo
ssduetoofroad-transporttrauma.Pulse— Volumeofbloodloss
110/min, breathingrate—
Rateofbloodescape
11/min,arterialpressureis90/SOmmHg.Whatc
hangeoftheblood Localizationofhemorrhage
willbemostcharacterizedthrough0O, Shafterblo
odloss? Abilityoforganismtoclot(coagulativesystem,q
uantity
Decreaseoferythrocyteandhemoglobinconte 149.Thereflexphaseofcompensationafteracut
nt ehemorrhageischaracterizedby...

Hypovolemia{lowcirculatingbloodvolume) Spasmofvessels

Non- Decreaseddiuresisandthirst
respiratorymetabolicacidosis(qcidbaseimbala
Reflexaccelerationandintensificationofcardia
nce)
ccontractions
Reflexventilationincreaseduetoacceler Increasereninandangiotensin2

ationanddeepeningtocompensateforo Increased aldosterone


xygen deficiencyoftissues
Vasoconstriction
Increasedhemopoiesis
Stimulation of volume receptors of aortic arc
Increasedproteinsynthesisinliver hand carotid artery
Increasederythropoietininkidneyandgastrici

nternalcastlefactor Expenseofbloodreserve

98.Peculiaritiesofqualitativeincomplet

estarvationofvitaminB6? Peculiaritiescanbe
150.Whatarethemechanismsofcompensation
duringreflexphaseafterhemorrhage? *exogenous(asaresultoftheabsenceoralowc
ontentofvitamininfood)orendogenous
Increased hemopoiesis

Increased proteinsynthesis in liver *nervoussystem(neurological)problemssinc


eVit.B6isimportantforit,so
Increased erythropoietin in kidney andgastric absence(Avitaminosis)orinsufficient(h
internal castle factor
ypovitaminosis)amountcouldcause
Expense of blood reserve neurologicalproblems..

Spasm of vessels *Endocrinesystempathologiesordiaea


sescouldbeobservedifthere's
Decreased diuresis and thirst
insufficiency({hypovitaminosis)orAbse
Reflex acceleration and intensification of nce{Avitaminosis)ofVit.B6
cardiac contractions
*seborrheicdermatitis, *Cheilosis
Reflex ventilation increase due to
acceleration and deepening to compensate *Confusionanddepression.
for oxygen deficiency Of tissues *seizure,glossitisandperipheralneurop
athy.

151.Whatarethemechanismsofbloodvolumer
estorationafterhemorrhage?

Increased hemopoiesis Intypicalcasesofpyelonephritis,thefollowin


gclinicalsyndromesare
Increased protein synthesis in liver distinguished:INTOXICATION,ARTERIALHY
PERTENSION,MPDERATE
Increased erythropoietin in kidney and
EDEMA,ANEMIA,POLYURIA,OLIGURIA,HYP
gastric internal castle factor
OSTENURIA,LEUKOCYTOURIA,HEMATURI
Expense of blood reserve A(NOT SURE)
152.What are the mechanisms of blood Themechanismforthedevelopmentofneph
pressure normalization in reflex phase after roticsyndromeis:|MMUNEMECHANISM
hemorrhage? (DELAYEDTYPEHYPERSENSITY)}BASEDONIMMUN
OCOMPLEXREACTIONSTYPEIII
Reflex acceleration and intensification of
REACTIONACCORDINGTOCOOMBSCELLS,-
cardiac contractions MATABOLICMECHANISMANDASSOCIATED

Tachycardia PHYSICO-CHEMICALCHANGES
Therenalsymptomsofacuteglomeruloneph Hg. Diffusetoxicgoiterwasdiagnosed(Based
ritis ow’ sdisease).Whatarethemechanismsof
include:OLIGURIA, PROTEINURIA,AZOTEMI impairmentofthefunctionsinthisdisease?A
A,HYPERTENTION,EDEMA,HEMATURIA,DI
NTIBODIESTOWARDSTHETHYROIDGLAND,
SORDE ROFCENTRALNERVOUSSYSTEM

EXCESSIVERELEASEOFTHYROIDHORMONE
Uremiccardiopathyischaracterizedby:CHA
RACTERIZEDBYDIASTOLICDYSFUNCTION S

ANDMARKEDLEFTVENTRICULARHYPERTR 13.Enumeratecomplexofnon-
OPHYWITHPROFOUNDVENTRICULAR specificsignsofdamagewhicharecausedbyp
FIBROSIS(NOTSURE) rolongednon
Violationofacid-
specificadaptivereactions...FOREXAMPLEI
basebalancewithuremiamanifestsitself:V| NCASESOFCHRONICHYPOXIAADAPTIVE
OLATIONOF REACTIONINCLUDEHYPERTROPHYANDHY
AMMONIOGENESIS, IMPAIREDACIDOGENE PERPLASIA,(NOTSURE)
SIS

HEARTENLARGEMENTINTERMSOFHYPERTENTIO
Therenalfactorsofkidneystoneformationar
N({NOTSURE)
e:CALCIUM, OXALATE, URICACID
14.Apatientaged23complainsofheadache,
Whichofthefollowingindicatorscharacteriz changesofhisappearance(increaseofsizeof
etherenaltubuledysfunction?>CHRONICRE feet,fingers, featuresoftheface),hoarsenes
NAL softhevoice,worseningofthememory.The
INSUFFICIENCYWITHSYPMTOMSASPROTE diseasehasbegun3yearsagowithoutanycau
NNURIA(ALBUMINURIA),OLIGURIA(REDUC ses.Increaseofsuperciliarryarches,nose,
TIONINNORMALQUANTITYOFURINEPERD
andtonguewerefoundduringphysicalexami
AY.
nationofthispatient;thebloodsugarwas
Indicatethemainmechanismscontributingt normal;theurinalysisisoutofchanges.Pituit
othedevelopmentofrenal arygigantismwasdiagnosed.Whatarethe
hypertension: NARROWEDRENALARTERYS pathogenesisinthisdisease >PTUMOROFTHE
UPPLYTHEKIDNRY,-CHRONICHEART PITUITARYGLAND,-EXCESSIVEGROWTH
FAILURE HORMONE

Theurinarysyndromeincludes:INTERSTITIA 15.Parathyreoidglandareremovedduringth
LCYSTITIS,-OVERACTIVEBLADDEROR eoperationonthyroidgland.Whatpathologi
PAINFULBLADDERSYNDROME,- cal
URETHRALSYNDROME changeswillbeobserveafterthisoperation?
OSTEOPOROSIS,-RHABDOMYOLYSIS
10.Whichcomplicationsarecharacterisednephrot
icsyndrome?BLOODCLOTS,-HIGHBLOOD 16.Awomanwithprimaryhyperparathyroid
CHOLESTEROLANDELEVATEDBLOODSYNDROM ismhasperiodicallyrepeatedattacksofrenal
,-POORNUTRITION,-HIGHBLOOD colic.
PRESSURE,-ACUTEKIDNEYINJURY,- Ultrasoundexaminationshowspresenceofs
CHRONICKIDNEYDISEASE,-INFECTION mallstonesinherkidneys.Whatpathological
changeswillbeobserveinthisdisease PNEPH
11.Whichcomplicationsarecharacterisedn ROLITHIASIS,-HYPERCALCEMIA
ephriticsyndrome-HEMATURIA,-PYURIA,-
WATER RENTENTION,- 17.Persistenthyperglycemiadevelopedina
SUBSEQUENTHYERTENSION,-EDEA patientwithCushing’ ssyndromeunderthe
influenceofexcessiveamountofglucocortic
12.Apatientaged41complainsofweakness, oids.Whataremanifestationsofthis
sweating,fever,tremorofhands,BP- syndrome ?DIABETESMELLITUSTYPE2,-
160/90mm HIGHBLOODPRESSURE,-MOONFACE
18.Afterphysicexertionapatientwithwithp OUTPUT,-
heochromocytomacomplainsoftachycardi ALTEREDBAROREFLEXANDCARDIOVASCULARRES
a, POMSESTOACUTESTRESS
increasedarterialpressure,andsharppainin 25.Indicatethemechanismofchangeinbloodgluco
epigastricarea.Whatchangesare seinAddison'sdisease:
characterizedthistumor?INCREASEDRELEA THEREISDISTURBEDFEEDBACKREGULATIONOFGL
SEOFEPINEPHRINE,-INCREASEDRELEASE UCOSE,,EXCESSIVEPRODUCTIONOF
OFNOREPINEPHRINE
CORTISOLWHICHINCREASETHEINSULINPRODUCTI
19.Thedevelopmentofpolydipsiaindiabete ONANDFUNCTION
sinsipiduscanleadto:NEPHROTICSYNDRO 26.Transhypophysialregulationisthebasisf
ME,ENURESIS,-DILATIONOFBLADDER,- or:REGULATIONOFTHEBODYFUNCTIONBO
HYDRONEPHROSIS DYS
FUNCTION,REGULATIONOFGROWTHOFTH
20.Awomenaged44complainsofgeneralma
EBODY
laise, painintheheartarea,andconsiderable
increaseofbodyweight.Atexaminationofthi 27.Theproductionofwhichhormonesdecre
spatientfollowingsymptomsarerevealed: aseswithtotalhypofunctionoftheanterior
lunarface,hirsutism,BP165/100mmHg,
hei pituitarygland: PROLACTIN,ESTROGEN,PRO
ghtis164cm,weightis103kg,theadipose GESTERONE,CORTISONE,ALDOSTERONE,T
depositsmainlyontheneck,shoulders,abdo 3 TRIIDODOTHYRONINE, T€ATHYROXIN
men, persistenthyperglycemia.
Whatmore
28.Whichofthefollowingchangesinwomen
manifestationsofthisendocrinedisorder?
testifyinfavorofcongenitaladrenogenital
21.WhichofthefollowingcharacterizesCohn'ssyn syndrome:
drome?
DEVELOPMENTALDELAYED,-
INTELLECTUALDISABILITY,-
SMALLHEADSIZE(MICROCEPHALY), WEAKMUSCLE
TONE(TYPOTONIA), BODDYSTOOL,- BASE 41
ABDOMINALPAINANDCRAMPING,-
REDUCEDAPPETITEANDWEIGHTLOSS
22.ThedevelopmentofAddison'sdiseasecausesad
ecreaseinthenumberofthefollowing
hormones.ADRENALINSUFFICIENCY,-
INTENSIFICATIONMELANO,-STIMULATION
intosuckedbemayairemboli
AVTIVITYOFPITUITARYGLANDS(MSH},-
smairvenoustoDue41.
INCREASEOFCORTICOTROPHINSECRETION
(DURINGHYPOADRENO) ?circumstancesfollowingth
23.Withpituitarynanism,itisnoted;DECREA
SEDGROWTHHORMONE,-
eunderveinssystemic
UNUSUALSTRUCTURE, -SLOWGROWTH wounded.areneckandheadtheofveinslargewheno
ccursembolismAir
24.Thecharacteristicchangesthatoccurinth
openbeingnegative,orzerotocloseiswhichinpressu
ebloodsystemundertheinfluenceof
reveins, These
glucocorticoidsinclude:
ofvesselstheinembolismresult,aAsoutside.thefro
-INCREASEDBLOODPRESSURE,-
mairsuck developscirculationpulmonary
INCREASEDCARDIACCONTRACTILITY,-
INCREASEDCARDIAC
;capillariesinactiveofopening
Phyperemiavenousofmanif
andcellsbloodtheofflowcentralparts.twointoflow
estationsWhat42. bloodofdivision
;({tumorswellingtoduetissuesororgansofenlarge
ment
plasmatheofflowperipheral
non-
oxygenatedofaccumulationandhypoxiaofresultaa
scyanosis
triad.Virchow’ sDiscri
shemoglobin

permeabilityincreasingandpressurehydrostatichig
hbycausededema ;conditionnon- be44.
oxygenatedtoduewallsvesselof

;hypothermialocal
.((endotheliumwallvesseltheof1.Injury
pain.andswellingofsensationsunpleasantsubjectiv
bloodfibrinolyticandanticoagulativecoagulative,b
e

etween|Imbalance2.
,arteriessmallofdilatationby:iScharacterizedpict
.((hypercoagulabilitytem
uremicroscopicThe
ssysdeceleration.flowBlo
;capillariesveins,arterioles,
od3.
;viscositybloodincreasedtoduedeceferationflowbl
ood
ofthatthanoftenmoreoccursveinsofthrombosisw
sthrombosisformation;transudate hyexplainsit
offactorsetiologicalareconditionsthesecreatingfa
yminooflanonsqyrdtissue;theinchangesdystrop
ctorstheAllarteries.
hicandatrophic onesexogenousTheendogenous.andexogenousar
tissueconnectivetheofgrowthexcessive eTheythrombosis.
sidesmoking,(tobaccochemicalcurrent),electric(tr
sclerosis
auma,physicalare

arterialbycharacterizedpi ,agentsimmune){infectious,logicalbio-
anddrugs)someofeffects

cturemicroscopicisWhat4 ofpropertiesrheologicalthechangeorvesselsblood
thedamagewhich causemaylergyal-
andhypertensionarterialAtherosclerosis,blood.th
3. Phyperemia e
Diabetesthrombosis.topredisposeandwallvesselt
;capillariesveins,arterioles,arteries,smallofdilatati
heofinjuryan
on
thrombosistopredisposeandcoagulationchangeob
esityandmellitus
¢;vesselsfunctioningofnumberincreased

e;accelerationflowblood
...ischemiaofmanifestation ?knowyoudothrombosisof
sWhat45. manifestationsWhat47.
ORStheofchangessegment,S-
reactiondefenseaasevolutionofprocesstheincreat
TtheofdeviationsassuchchangesECG
edisThrombosis
wave. Tandcomplex
vesselandtraumasmechanicalafterlossbloodandbl
bysuppliediswhichmyocardium,theofareatheinde
eedingarresting ruptures.
velopsinfarction
vessels.damagedthroughbloodthe ischemia.causesitifrolenegativeaplaysthrombosis
noted.ismitochondriaofimpairmentstructuralfirst gravesttheisandnecrosistissueischemicofareaanis
atMorphologically,
Infarction thrombosis.ofoutcome
occurs.pyknosisorswellingnucleiThen
moreforaccountcollectivelybrainandlungheart,th
Cardiomyocytesappears.fibersmuscletheofstriati
eofinfarction
ontransverseThe potassium.andglycogenlose
together.takendiseasesinfectionsandcanceroffor
localisinfarctionofconsequencemainTheaccumula
msallthandeaths
ted.areionsH*
ofocclusionthromboembolic)(orthromboticbycau
causesinfarctionMyocardiallysis..myocyteandnec
sedusuallyisit vessels.
rosiscoagulative
congestionvenouscausingcoursechronicahasveins
ofdysfunctionacidosis,hypoxia,toleadsthatinsuffic
ofThrombosis ((thrombophlebitisflammationin-
iencycardiacacute
and
death.tofinallyandorgans,otherandbrainthe
?knowyoudoembolismofm
breastbonethebehindbodytheofpartleftupperthe
onlocalizationPain anifestationsWhat48.
theofTfhromboembolismeffect.negativeahasalwa
ysEmbolism

arterialofsignificancesWha ,chesttheinpainacutedyspnea,picture:clinical(the
arterypulmonary
t 46. indeathofcausefrequentaisveins)cervicaltheofsw
ellingcyanosis, patients.hospitalized
... hyperemia
,diseaseheartrheumaticinfarction,(myocardialdis
byaccompaniedishyperemiaarterialcasesofmajori easeHeart
tytheln embolismofrisktheincreasearrhythmia)

,Howeverfunctioning.organandmetabolismofint
ensification

dilatationvesselscleroticThus,possible.alsoisoutco
meunfavorable
ischemiaoftypespasticreas
theindangerousespeciallyislthemorrhage.andrapt onsCheck49.
ureinresultmay brain
vasodilativeandvasoconstrictivetheofdisturbance
sFunctional vesselscoronarytheofapparatus
.inflammationofcoursetheinsignificancegreatofis
hyperemiaArterial
reveins, These
?knowyoudoembolismsfat ofvesselstheinembolismresult,aAsoutside.thefro
mairsuck developscirculationpulmonary
ofreasonsWhat50O.
whenbone,longbrokenaisembolismfataofcauseco
mmonmostThe fromreleasedaremet
theintooutseepcanmarrowbonethefromfatbreak
s,bonea abolitesOxygen-
.bloodstream
derived53.
{replacementskneeandhip(likesurgerymajorincl
includeandmacroph
udecausesOther

,damagetissuesoftbiopsies, marrowbonebirth,chi
agesandneutrophils
Idburns, pancreatitis,
orliver,fattyliposuction,likeconditionsorprocedur
activated
esnon-traumaticand NOtoxicandOQH’H20
canbloodstreamtheintofatintroducesthatAnythin
gdisease.cellsickle 2,(O’2),oxygensuper
fitcan'titvesselbloodareachesfattheifembolismfat
ainresult through oxide
radicalsfreeoxygen-
?knowyoudoembolismsgas derivedTheseproduc
ofreasonsWhat51. ts.

from(a)pressure:atmosphericindropsharpatoduei ..inflammationinactionsfol
sembolismGas
ordivers)andcaissonsofworkers{innormaltoelevat
lowingthehave
ed permeability. vascularincreasedtherebyanddamag
ecellEndothelial
duringorheightatoriserapidextremely(withlowton
tissuecausingantiproteasesofinactivationandprot
ormalfromb))
easeofActivation damage.matrix
{aircraftaltitudehighincabintheofdepressurizatio
n likecellsotherto Damage
anddecreasesbloodandtissuesingasesofsolubilityt
hecases,theseln «RBC
dissolvedthefromgonitrogen){ primarilygasesTheo
ccurs.desaturation
theofcapillariestheoccludewhichemboli,becomea
ndgaseoustostate .circulationsystemic
inrolesfollowingtheplaysN
?knowyoudoembolismsair
O(NO)oxideNitric54.
ofreasonsWhat52.
..Inflammationofmediator
wounded.areneckandheadtheofveinslargewheno
ccursembolismAir S
openbeingnegative,orzerotocloseiswhichinpressu
musclesmoothvascularof(relaxation1.Vasodilatati
duringpermeabilityvascula
on
rinincreaseAn5S/.
aggregation(adhesion,activatioplateletofstagesall

ofAntagonism2. :causesinflammation
theofpermeabilityinincreasestrikingisthereinflam
(degranulationand mationacuteln proteinstovessels

phagesmacroactivatedinactioncytotoxicorMicrob osmoticintravascularthereducesplasmafromprot
icidal3. einoflossThe

inflammatoatrecruitmentleukocyteofReduction fluidinterstitialofpressureosmoticincreaseand
pressure
4.
theinaccumulationitsandfluidofflowoutmarkedca
sites usefurtherThis tissueextravascular/interstitial

oedemainflammatorycalledisfluidextravascularof

inflammationwithpainofde increasenetThe

venules:ingapsendothelialofFormation
velopmenttheln55.
tightbyfusedarevesselsbloodofcellsendothelialth
‘involved eNormally, junctionsintercellular

andcytokinesincludingmediatorsfurtherreleasece proteinandfluidofoutflowpermittoloosenedareth
llsimmuneRecruited factorsgrowth eseinflammation,In

,{ACTHhormoneadrenocorticotropicincluded:m
ediatorsPain
vascularinincreaseancauseinflammationofmediat
,opiodsbraincatecholamines,oxitocin,vasopressi orschemicalMost
n,glucocorticoids,
junctionsinter-
{(VIPpeptideintestinalvasoactiveencephalin,/en endothelialopeningbypermeability
dorphinll,angiotensin
endothelialofcontractionbyproducedaregapsinter
tissueleukotrienes), prostaglandins, (e.g.,eicosanoi
-endothelialThe
dsP,substance histamine(bradykinin),kininogens

.junctiontheofwideninginresultswhichcells,

theviametabolismacidarac
hidonicofproductsThe56.
aremembranescellofphosp
:arepathwaycyclooxygenas
holipidstheFrom58.
e
;(PGF2aPGE2,(PGD2, prostaglandinsclassical""
‘formed
?;(, TXB2(TXA2,,thromboxanes

{12(PGprostacyclin
‘formedaremembranes

cellofphospholipidsFro

m59 « acidArchidonic Leukotrienes

Prostaglandins thromboxanes

shistaminecharacterizingSt
atements 60.
Vasodilation

Local

permeabilityv

ascularlncrea

sed

musclesmoothofCo

ntraction

vesselsbloodofDilati

on

processinflammatio

ninMediator
293.Whatarethecharacteri Absenceofactivemovemen
sticmanifestations ts
ofcentralparalysisintheaffe Transientpain
ctedlimbs? Parasthesia-thoracicspine
Muscletoneofextremitiesin Hypokinesia
creased MuscularAtrophyofuppere
Spinalreflexesaregreatlystr xtremity
engthened 296.Listtheexogenouscaus
Reflexzonesareexpanded esofdamageto
Hyperreflexia thenervoussystem:physical
294.|Indicateneu factors: mechanicaltrauma
rotropicviruses( Barometricpressure
TORCH lonizingradiation
infection) Chemicalfactors:ne
Herpexvirus urotropic(cerebroto
Rubella xic) poisons
HIV Industrialpoisons
Meningococcus Alcohol
Poliovirus Narcotics
Measles Pschotropics
Cytomegalovirus Biology:-
Encephalitisvirus neurotropicviralinfection
Meningococcus
295.Whatsignscharacterize Turbclebacillus
peripheral paralysis? HIV
Muscularhypotonia/atroph TORCHinfections
ia(lossofreflexes) Immunefactors
Decreasedreflexes
297.|Indicateneurotropicmi Ataxicdysarthia
crobialtoxins: Nystagmus
Cerebrotoxin(boutoulintox
in) 300.Whatsignscharacteriz
Industrialpoison eperipheral paralysis?
Alcoholpoison Muscularhypotonia/atroph
Narcotics ia(lossofreflexes)
Psychotropics Decreasedreflexes
298.Listtheconditionsthat Absenceofactivemovemen
determinethe ts
strengthofdamagetothene Transientpain
rvoussystem: Parasthesia-thoracicspine
localizationofthedamage(n Hypokinesia
ervous MuscularAtrophyofuppere
centre,conductor,synapses xtremity
,receptor)
299.Damagetothecerebell 301.Inthepathogenesisofn
ummaybe eurogenic
accompaniedby:movemen degenerationofcellsofaden
tdisorder(asynergia) ervatedorgan,
&tremor thefollowingprocessesarel
Dysmetria mportant:Nervous
Inabilitytoperformrapidalt trophism-
ernating mediatormechanism
movements(adiadochokin Non-mediatedmechanism
esia) Vascularmechanism
Hypotonia 302.Listthespecificmechani
Ataxicgait smsofdamage toneurons:
Exogenousandendogenou Metabolicdisorders-
S inhibitionofcertain
Exogenous(physical- enzymes
ionisationradiation,trauma 304.Life-
) Chemical- threateningcomplicationsi
cerebrotoxic,industrial nshock
poison,alcohol,narcotics areduetohypoxiccellinjuryr
Biologicalfactors- esultingin
neurotropicviralinfection, immunoinflammatoryresp
TORCH,HIV onsesandactivation
Disturbancesofneurochem ofvariouscascades(clotting,
icalprocess complement,
Disturbancesofmotorfunct kinin). Theseincludethefollo
ion, vegetative function wing....acute
respiratorydistresssyndro
me
303.Whatischaracteristicof Disseminatedintravascular
a neurodystrophicprocess- coagulation
severedisorders Acuterenalfailure
developwhenafferentfiber Multipleorgandysfunctions
sandnervesare damaged yndrome
Structuraldisorders-
305.Thepathogenicfactors
muscleatrophy,dystrophic
ofhemorrhagic
changes
shockinclude...b!oodloss
Functionalchanges-
Lowcardiacoutput
anincreaseinthesensitivity
Hightotalperipheralvasc
ofdenervatedstructures
ularresistance
Bloodplasmalossinburns Expandingmicrocirulatoryv
,lesions essels&increasesits
Lossoffluids(deficitofext permeability
racellularfluid) Hemodynamicoption
Spasmofvesselsofmicroc Reductionofvenousreturnt
irculatorybed oheart
Centeralizationofbloodc Dilationofcapillaryandcapa
irculationcause citorvessels
damagetoorgans
Hypovolemicshockdevel 308.Thepathogenicfactors
ops ofpaineshock
306.Torpidphaseofhemorr include...fallofneurogenict
hagicshockis oneofarterioles
characterizedby... Decreaseinbloodpressure
Weakeningeffectsofsympa Bradycardia
thoadrenaland pituitary— Decreasedcardiacoutput
adrenalsystem Peripheralvasodilation&ve
Reductionofcardiacoutput nouspooling
Depositingofblood 309.Thepathogenicfactors
Arterialhypoxemia ofpancreatic
307.Thepathogenicfactors shockinclude...reductionof
ofanaphylactic generalperipheral
shockinclude...anexitofflui resistanceinresultofgener
dfromblood alizeddilationof vessels
vessels syndromeofintravasculardi
Vasculartonusisreduced sseminated
coagulationofblood
Reductionofbasaltoneofve Thereductionofvenousretu
sselsunderaction rnofbloodtoheart
ofbiologicalyactivesubstan Disordersofintracardiache
ces modynamics
Thecardiactamponade,mas
310.Thepathogenicfactors sive
ofhepaticcoma thromboembolismofpulmo
include...hypoglycemia naryartery(increase
DisordersofWaterelectroly ofheartfillingpressure)
tesbalance
Hyponatremia 312.Whatmanifestationso
Levelofaldosteroneincreas fketonemic
es hyperglycemiccomadoyou
Increasesensitivitytogluta know?
mine Respiratoryfailure
Lossofpotassium Vascularcollapse
Lowbloodpressure
311.Thepathogenicfactors Decreasedmuscletonus
ofcardiogenic Oliguria/anuria
shockinclude... LowbloodpH(6.8-7.3)
Increaseofperipheralresist Ecgsignsofhyperkalmiafirst
anceofvessel andhypokalemia
Tachycardia
decreaseofcardiacoutput
Decreasedcontractilityofth
eheart 313.Whatspecificmanifesta
Severedisorderofheartrhyt tionsof
hm lactacidemichyperglycemic
comadoyouknow?
Hypoxia- Hyperchloremia
excessoflactatemorethan2 Bloodpressuredecreases
mM/1consciousnesssuppr Hemodynamicinstabilitylea
ession dtothrombosis
Heartfailure(tachycardia,h
ypotension)
Anemia 315.Thepathogenicfactors
Disordersoflung,kidneylive ofpancreatic
randinfectious diseases shockinclude...
Paininthemuscles,heartac reductionofgeneralperiph
he eralresistancein
dyspepsia,stomachache resultofgeneralizeddilatio
Kussumalsrespirationdeve nofvessels
lops syndromeofintravasculard
isseminated
coagulationofblood
314.Whatmanifestationso Reductionofbasaltoneofve
fhyperosmolar sselsunderaction
hyperglycemiccomadoyou ofbiologicalyactivesubstan
know? ces
Secretionofconsularhormo
nes(ofthestress, 316.Thepathogenicfactor
trauma,orinfection) sofcollapse include...
Dehydration Cardiovascularfailure
Polyuria(osmoticdiuresis) Rapidandsignificantdecre
Glucoselevelincreasesinblo asebloodpressure
od Fallofvasculartone
Hypernatremia Reductionofvenousbloodi
nflowtoheart
A patient suffers from Down’s disease that
Decreaseofheartoutput manifests as mental retardation, shortness of

Decreasevenouspressure stature, pathologically short fi- ngers and toes,


and eyes with mongoloid slant. Karyotype
Infringementoftissuesper analysis revealed trisomy 21. What group of
diseases does this pathology belong to? (2018, N
fusionand metabolism 3)

317.Collapse:generalman +Chromosomal disorders

ifestations Molecular genetic disease

Gametopathy
Coronayinsufficiency
Fetopathy
Decreasecardiacoutput
Blastopathy
Hypoperfusionoftissue
Hormones regulate numerous metabolic
Venouscongestion processes. What hormone activates glycogen
Redistributionofcirculation synthesis? (2018, N 4)

S +Insulin

Adrenaline
Capillarotrophicinsufficien
Vasopressin
cy Thyroxine
Tremorsoffingers Oxytocin

Inhibitionness
A patient has developed anuria. Blood pressure is
Apathy 50/20 mm Hg. What process of uropoiesis was
disturbed resulting in acute decrease of urine
Decreaseofnervo- output? (2018, N 21)

muscleexcitation +Glomerular filtration

Oliguria Obligate reabsorption

Increaseofbloodviscosity Facultative reabsorption

(Irregularheartbeat,lowblo Tubular secretion

odpressure, seizures)
A 12-year-old boy is of short stature, but his
mental development corresponds with that of his
age group. What hormone deficiency is the most
likely to cause this pathology? (2018, N 22)

+Somatotropin
THE FIRST ANSWER IS CORRECT IN EACH
Insulin
QUESTION
Oxytocin
Vasopressin Hypoparathyroidism

Adrenaline Hypothyroidism

Hyperparathyroidism
A 10-year-old child has height of 178 cm and
Adrenal hypofunction
body mass of 67 kg. These presentations are
caused by the functional disturbance of the:
(2018, N 23) A patient, who lives in the area with specific
geochemical conditions, was diagnosed with
+Pituitary gland
endemic goiter. What microelement deficiency
Thyroid gland results in development of this pathology? (2018,
N53)
Gonads +1

Adrenal glands Cl

Parathyroid glands Br

Any damage to the patient’s vessels results in


Na
persistent hemorrhage. Blood clotting factor VIII
is deficient in the patient’s blood. What disease
does this patient suffer from? (2018, N 33) On examination the patient’s sclera and oral
mucosa are icteric. What biochemical blood
+Hemophilia
value can be expected to be increased? (2018, N
Acute vascular purpura 59)
Thrombocytopenic purpura +Bilirubin

Anemia Amylase

Radiation sickness Glucose

Albumin
To induce diabetes mellitus in a rabbit, B-cells of
Cholesterol
pancreatic islets {islets of Langerhans) were
selectively damaged with alloxan. What method
of diabetes induction was used in this A 25-year-old man has an appointment with the
experiment? (2018, N 36) dentist. Several minutes after his oral cavity was
lavaged with furacilin (nitrofurazone) the patient
+Shutdown
developed significant labial edema. What type of
Irritation allergic reaction is observed in this case?

Introduction of enzymes, hormones (2018, N 71)


Isolated organs +Anaphylactic

Stimulation Delayed-type hypersensitivity

Cytolytic
A patient presents with persistent tachycardia,
Stimulated
exophthalmos, high excitability, increased basal
metabolic rate. What disorder can lead to the Immune complex
development of this syndrome? (2018, N 39)
+Hyperthyroidism
A patient with gastric carcinoma has undergone B6
several courses of radiation therapy. What
system is the first to become functionally
When working in the garden, a man accidentally
disturbed after the body was exposed to ionizing
cut his hand. The wound remained untreated.
radiation? (2018, N 72)
Shortly after that the wounded area developed
+Blood inflammation with accumulation of exudate that
contained numerous viable and degenerate
Nervous
neutrophils. What type of exudate is it? (2018, N
Digestive 39)

Urinary +Purulent

Respiratory Serous

Fibrinous
During regular check-up a patient presents with
Hemorrhagic
enlarged thyroid gland, exophthalmos, increased
body temperature, heart rate up to 110/min. Catarrhal
What hormone should be measured in the
patient’s blood in this case? (2018, N 76)
A patient presents with persistent fever, with the
+Thyroxin
difference between evening and morning
Testosterone temperature not exceeding 10C. What type of
fever curve is present in this patient? (2018, N
Glucagon
90)
Insulin
+Continuous
Cortisol
Remittent

Hectic
Insulin production in B-cells involves many
substances. What substance gives the main Recurrent
signal for insulin synthesis when its
Intermittent
concentration changes? (2018, N 80) +Glucose

Carbon dioxide
Ultraviolet irradiation is used in medicine in
Heparin various physiotherapeutic procedures.

Hemoglobin What mechanism of medicinal action is


characteristic of ultraviolet rays? (2018, N 99)
Urea
+Activation of vitamin D synthesis

An ophthalmologist has detected increased time Activation of drug action


of dark adaptation in a patient. What vitamin
Decrease of melanin synthesis in the skin
deficiency can result in such symptom? (2018, N
32) Intensification of cell division

+A Activation of lipid peroxidation

Cc
A patient has a gallstone lodged in the common
K
bile duct, which blocks bile supply to the
B1 intestine. What digestive process will be
disturbed in this case? (2018, N 101)
+Fat digestion observed in the patient? (2018, N 108) +Manifest
illness stage
Protein absorption
Prodromal stage
Carbohydrate digestion
Latent stage
Carbohydrate absorption
Recover—
Protein digestion

A patient suffers from hyperchromic B12-


A 46-year-old patient presents with hyperactivity
deficiency anemia. What vitamin preparation
of creatine kinase in his blood serum. What
should be prescribed in this case? (2018, N 122)
pathology can be suspected? (2018, N 103)
+Cyanocobalamin
+Myocardial infarction
Riboflavin
Acute pancreatitis
Vicasol (Menadione)
Chronic hepatitis
Thiamine chloride
Hemolytic anemia
Retinol acetate
Renal failure

After a traffic accident the driver presents with


A patient is pale, has goose bumps and chills.
increased blood glucose. What mechanism leads
What stage of fever is it characteristic of? (2018,
to hyperglycemia in this case? (2018, N 125)
N 106)
+Sympathoadrenal system activation
+Temperature increase
Increased production of somatotropic hormone
Temperature decrease
Decreased production of insulin
Continuous fever
Decreased production of glucagon
Compensation
Decreased tone of parasympathetic nervous
Latent stage
system

Ascarids were detected ina sick child. What


Examination of a 45-year-old man, who for a long
changes in leukogram will be the most
time kept to a vegetarian plant based diet,
characteristic of helminthiasis? (2018, N 107)
revealed him to have negative nitrogen balance.
+Eosinophilia What peculiarity of his diet has caused such
developments? (2018, N 132)
Basophilia
+Insufficient protein content
Neutrophilia
Insufficient fat content
Lymphocytosis
Insufficient vitamin content
Monocytosis
Excessive water content

Aman received a radiation dose of 30 Gy. He Excessive carbohydrate content


presents with necrotic angina and disorders of
the gastrointestinal tract. Blood tests revealed
After severe emotional strain a 53-year-old man
anemia, leukopenia and thrombocytopenia.
suddenly developed acute pain in the heart area,
What stage of acute radiation sickness is
which irradiates to the left hand, to the neck, and Cheyne-Stokes’
under the left scapula. He noted numbness of his
left hand. His face is pale and covered in cold

->DmHoag
sweat. Nitroglycerine administration stopped the
pain attack after 10 minutes had passed. What is
the most likely disease in this case? (2018, N 133)

+Angina pectoris

FMNM}WSS
Stroke

Myocardial infarction

09D
Pulmonary embolism

73>.
Somatoform autonomic dysfunction

ro
During ultrasound investigation a patient was c
diagnosed with bilateral renal artery stenosis of
atherosclerotic genesis. Specify the bioactive Biot’s
substance that due to its excessive secretion is
the key component of arterial hypertension
A woman complains of itching lips; they are
pathogenesis in the given case: (2018, N 138) reddened and covered in scabs and scales after
+Renin she had been using new lipstick for two weeks.
What allergic reactions result in this kind of
Cortisol disorders? (2018, N 169)

Vasopressin +Delayed
Noradrenaline Cytotoxic

Thyroxin Immune complex

Anaphylactic
At the end of his shift a worker of the steel
foundry felt dizziness, his body temperature Stimulating
increased to 38.50C. What condition does he
present with? (2018, N 143)
What ion increases osmotic pressure in the focus
+Hyperthermia of inflammation? (2018, N 188) +Potassium

Decompression Calcium

Fever Fluorine

Hypothermia Magnesium

Hypertension Chlorine

A patient in the state of ketoacidotic coma A 55-year-old man came to a doctor with
presents with loud rapid respiration: labored complaints of acute pain in his big toes. Meat
expiration with tension of expiratory muscles and wine are a permanent fixture in his diet. The
occurs after deep inspiration. Name this type of doctor suspects gout. What substance must be
pathologic respiration (2018, N 157) measured in the patient’s blood to confirm this
diagnosis? (2018, N 189)
+Kussmaul’s
+Uric acid Embolism

Urea Ischemia

Lactate
A person has been stung by a bee. The stung
Bilirubin
area developed redness and edema.
Ketone bodies
What is the main mechanism of edema
development? (2017)
Upon examination the ophthalmologist
+Increased permeability of the capillaries
diagnosed a 21-year-old woman with visual
impairment - hemeralopia (“night blindness”). Decreased oncotic blood pressure
What drug should this patient take to restore her
Increased hydrostatic blood pressure
vision? (2018, N 193)
Decreased osmotic blood pressure
+Retinol acetate
Disturbed lymphatic efflux
Ergocalciferol

Suprastin (Chloropyramine)
Nicotinic acid amide fulfills important metabolic
Cholecalciferol function. What disorder develops, when it is
deficient in the organism? (2017)
Sustac forte (Nitroglycerin)
+Pellagra

Parents of a 10-year-old child have made an Rickets


appointment with an endocrinologist due to
Anemia
complaints of the child’s low height. The child’s
appearance is corresponding with that of a 5- Xerophthalmia
yearold. What hormone causes such changes in
Beriberi
physical development, if its secretion is
disturbed? (2017; 2016, N 16, TB; 2014, N 7, TB)

+Somatotropic hormone A patient was delivered into a resuscitation unit


with signs of alcohol poisoning. The patient
Adrenocorticotropic hormone developed hypoxia of the following
pathogenesis: (2017) +Tissue
Thyroxin
Hypoxic
Testosterone
Hemic
Insulin
Circulatory

An injured person exhibits the following signs at Mixed


the site of trauma: skin redness, throbbing small
arteries, elevated local temperature, increased
tissue turgor. What local blood circulation Prolonged taking of cytostatic agents resulted in
development of necrotic tonsillitis in the patient.
disorder are these presentations typical of?
(2017; 2016, N 26, TB; 2013)
It can be associated with the following changes in
the leukocyte content: (2017) +Agranulocytosis
+Arterial hyperemia
Neutrophilic leukocytosis
Venous hyperemia
Lymphopenia
Thrombosis
Eosinopenia
Lymphocytosis Exudation

Inactivation
A patient with type | diabetes mellitus developed
hyperketonemic coma. What acid-base
A 55-year-old man, who had been suffering from
imbalance will be observed in the patient? (2017)
mitral insufficiency for many years, developed
+Nongaseous acidosis acute heart failure. What pathophysiological type
of heart failure can be observed in this case?
Gaseous acidosis
(2017) +Due to cardiac overload
Nongaseous alkalosis
Due to hypoxic damage to the heart
Gaseous alkalosis
Due to coronarogenic damage to the heart
There will be no acid-base imbalances
Due to neurogenic damage to the heart

Due to acute cardiac tamponade


Hyperlipemia can be observed in 2-3 hours after
eating fatty food. 9 hours later lipid content
normalizes again. How can this condition be Cataract (lenticular opacity) has developed ina
characterized? (2017) 52-year-old woman with diabetes mellitus.
Lenticular opacity was caused by intensification
+Alimentary hyperlipemia
of the following processes: (2017; 2016, N 125,
Transport hyperlipemia TB; 2015, N 128, TB)

Hyperplastic obesity +Protein glycosylation

Retention hyperlipemia Lipolysis

Hypertrophic obesity Ketogenesis

Protein proteolysis
A patient presents with icteric sclera and mucous
Gluconeogenesis
tunics; urine is dark; feces are light-colored.
Blood content of direct and indirect bilirubin is
increased, urine content of direct bilirubin is A patient suffers from hyperchromic B12-
increased. What pathology can be characterized deficiency anemia. What vitamin preparation
by these signs? (2017) +Obstructive jaundice should be prescribed in this case? (2017)

Hemolytic jaundice +Cyanocobalamin

Hepatocellular jaundice Riboflavin

Jaundice of the newborn Vicasol (Menadione)

Atherosclerosis Thiamine chloride

Retinol acetate
The patient presents with rapid growth of a
tumor node and its progressing malignization.
A 54-year-old man requested a pharmacist’s
What stage of tumor growth can be
advice on drug prescription. The patient has 4-
characterized by these presentations? (2017)
year-long history of chronic glomerulonephritis
+Progression
and 2-yearlong history of persistent
Promotion (activation) hypertension. What substance synthesized in the
kidneys has important role in development of
Transformation
arterial hypertension? (2017)
+Renin Maltose

Nitric oxide Saccharose

Aldosterone Fructose

Histamine
During routine preventive examination the local
Catecholamines
pediatrician noticed a boy of short stature.
Mental development of the child corresponds
with his age. What endocrine disorder is it?
A patient complains of general weakness, muscle
(2017)
weakness in the extremities (if the patient is
+Pituitary nanism
asked to make a fist several times in a row, for
example, the patient is capable of doing it only Cretinism
once), facial muscles are weak, swallowing is
Acromegalia
disturbed. Administration of acetylcholinesterase
drugs removes these disturbances to a certain Gigantism
degree. Determine the pathology: (2017)
Rickets
+Myasthenia

Paralysis A 25-year-old-patient with the Il degree thermal


burns came to a doctor. Objectively: there are
Paresis
large blisters on the upper limbs; the blisters are
Hemiplegia filled with clear exudate containing mostly water
and albumines with isolated leukocytes. Name
Monoplegia
the type of the exudate: (2017)

+Serous
A 71-year-old woman developed mechanical
jaundice due to obstruction of the bile duct with Catarrhal (mucous)
a chololith. Decrease of blood pressure and
Fibrinous
bradycardia are detected.
Purulent
These changes in functioning of the patient’s
cardiovascular system are caused by increased Hemorrhagic
blood content of the following substance: (2017)

+Bile acids A patient with brain edema presents with


respiration that is characterized by periods of
Direct bilirubin
several respiratory movements of equal
Indirect bilirubin amplitude alternating with periods of apnea.
What pathologic respiration is it characteristic
Urobilin
of? (2017)
Stercobilin
+Biot’s respiration

Gasping respiration
A 5-year-old child presents with abdominal
distension, abdominal cramps, and diarrhea Apneustic respiration
occurring 1-4 hours after drinking milk. Described
Cheyne-Stokes’ respiration
symptoms are caused by the lack of enzymes
that break up: (2017; 2015, N 154, TB) Kussmaul’s respiration

+Lactose

Glucose
A patient in the state of ketoacidotic coma Decreased permeability of vessel wall
presents with loud rapid respiration: labored
expiration with tension of expiratory muscles
A 55-year-old man came to a doctor with
occurs after deep inspiration. Name the type of
complaints of acute pain in his big toes. Meat
pathologic respiration: (2017; 2016, N 192, TB)
and wine remain permanently in his diet. The
+Kussmaul’s doctor suspects gout. What substance must be
measured in the patient’s blood to confirm this
Cheyne-Stokes’
diagnosis? (2017)

+Uric acid
—->DmDHag

Urea

Lactate

Bilirubin
SD

Ketone bodies
FMM”

The patient’s large-focal myocardial infarction is


DO

complicated with pulmonary edema. What


disturbance of cardiohemodynamics contributed
ro

to the pulmonary edema development? (2017)


+Acute left ventricular failure

Acute right ventricular failure


Biot’s
Autoimmune myocarditis

In 9 days after administration of a therapeutic Cardiogenic shock


serum the patient developed urticaria, itching, Reperfusion syndrome
edemas, and lymph nodes enlargement. What
type of allergic reaction has occurred in the
patient? (2017) A patient has developed anuria. Blood pressure is
50/20 mm Hg. What process of uropoiesis
+lmmune complex caused acute decrease of urination? (2016, N 5,

Cytotoxic TB)

Anaphylactic +Glomerular filtration

Stimulating Obligate reabsorption

Cellular Facultative reabsorption

Tubular secretion

Exudation is characteristic of inflammation. What


factors cause exudation and local edema of the In an emergency situation a scuba diver has
inflamed area? (2017) quickly risen from the depth to the surface in
+Increased permeability of vessel wall violation of safety regulations. He is unconscious,
presents with respiratory failure and cardiac
Hyperglycemia activity disorder as the result of decompression
sickness. What complication can develop in the
Ischemia
scuba diver? (2016, N 15, TB)
Leukocyte adhesion to endothelial cells
(2015, N 12, TB)(2013, 2012)
+Gas embolism anemia. Appearance of the following elements in
her blood became the diagnostic character:
Fat embolism
(2016, N 35, TB) (2015, N 37, TB)
Air embolism
+Hypochromic erythrocytes
Cellular embolism
Macrocytes
Thromboembolism
Megalocytes

Reticulocytes
A patient complains of tachycardia, insomnia,
weight loss, irritability, sweating. Objectively: the Erythroblasts
patient has goiter and slight exophthalmos. What
gland is affected, and what functional disorder is
An ophthalmologist has detected increased time
it? (2016, N17, TB)
of dark adaptation in a patient. What vitamin
+Hyperthyroidism deficiency can result in such symptom? (2016, N
31, TB) +A
Hypothyroidism
Cc
Hyperparathyroidism
K
Hypoparathyroidism
B1
Adrenomedullary hyperfunction
B6

A patient has been hospitalised with pneumonia.


What kind of respiratory failure does the patient A 70-year-old patient presents with cardiac and
have? (2016, N 19, TB) cerebral atherosclerosis. Examination revealed
changes of blood lipid spectre. Increase of the
+Restrictive
following lipoproteins plays a significant role in
Obstructive atherosclerosis pathogenesis: (2016, N 82,

Central TB)
Peripheral +Low-density lipoproteins

Thoracic diaphragm Very low-density lipoproteins

Intermediate density lipoproteins


In the state of fright the following signs can be
High-density lipoproteins
observed: acute pallor of face, tremor of
extremities. What kind of ischemia can be Chylomicrons
observed in such a condition? (2016, N 20, TB)

+Angiospastic During containment measures following


Chornobyl Nuclear Power Plant disaster a worker
Compression
has been exposed to a dose of ionizing emission
Obstructive (thrombus) of 6 Gy (600 R). The worker complains of general
fatigue, nausea, dizziness, labile blood pressure
Metabolic and heart rate, short-term leukocytosis with
Obstructive (vascular wall thickening lymphopenia. What stage of acute radiation
sickness can be characterized by such
presentations? (2016, N 83, TB)
At the sixth month of pregnancy a woman has
been diagnosed with severe irondeficiency +Prodromal
Manifest A patient with alcoholic cirrhosis complains of
general weakness and dyspnea. The following is
Latent
revealed: decrease of blood pressure, ascites,
Recovery dilation of superficial veins of the stomach
anterior wall, esophageal varicose veins
Long-term consequences
dilatation, splenomegaly. What hemodynamics
disorder does the patient suffer from? (2016,
A woman noticed that a cut on her skin was still N 111, TB) (2015, N 104, TB) (2014, N 134,
bleeding even after 20 minutes had passed. TB)(2012)
What vitamin deficiency causes such condition?
(2016, N 88, TB) (2015, N 83, TB) +Portal hypertension

+ Vitamin K Left ventricular failure

Vitamin A Right ventricular failure

Vitamin D Cardiac insufficiency

Vitamin E Collapse

Vitamin B12
A patient has icteric skin; unconjugated bilirubin
content in blood is high; conjugated bilirubin in
An elderly man exhibits low levels of red blood urine is not detected. There is significant amount
cells and hemoglobin in blood; however, his color
of urobilin in urine and stercobilin in feces. Name
index is 1,3. Blood smear analysis revealed
the pathology characterized by given symptoms:
megaloblasts. What type of anemia is observed
(2016, N118, TB) (2015, N 117, TB) (2014, N 155,
in this case? (2016, N 98, TB)
TB)
+B12-folic acid deficiency +Hemolytic jaundice
lron-deficiency
Obstructive jaundice
Acquired hemolytic
Jaundice of the newborn
Hereditary hemolytic
Hepatocellular jaundice
Chronic posthemorrhagic
Atherosclerosis

After drinking milk a 1-year-old child developed A 46-year-old patient was found to have
diarrhea, flatulence. The baby is likely to have
hyperactivity of creatine kinase in the blood
deficiency of the following enzyme: (2016, N 100, serum. What pathology can be suspected? (2016,
TB) (2015, N 94, N 121, TB) (2015, N 124,
TB) TB)
+Lactase
+Myocardial infarction
Maltase
Acute pancreatitis
Aldolase
Chronic hepatitis
Hexokinase
Hemolytic anemia
Glycosidase
Renal failure
A patient with pulmonary carcinoma has Paresis
developed a case of pleurisy. Large amount of
Hypodynamia
hemorrhagic exudate was obtained for analysis.
What component is specific for hemorrhagic
exudate? (2016, N 123, TB) The most severe and dangerous complication of
diabetes mellitus is hypoglycemic coma that is
+Erythrocytes
characterized by loss of consciousness and is
Leukocytes lethal, unless efficient emergency treatment is
received by patient. What is the main
Platelets
pathogenetic component of hypoglycemic coma?
Fibrin (2016, N 159, TB) +Carbohydrate deficiency and
low energy of cerebral neurons
Pus
Carbohydrate deficiency and low energy of
myocardium cells
Aman received a radiation dose of 30 Gy. He
presents with necrotic angina, disorders of the Blood hyperosmia
gastrointestinal tract. Blood tests revealed
Noncompensated ketoacidosis
anemia, leukopenia and thrombocytopenia.
What stage of acute radiation sickness is Respiratory alkalosis
observed in the patient? (2016, N 127, TB)

+Manifest illness stage A woman complains of nausea, vomiting, skin


itch. She was diagnosed with mechanical
Prodromal stage
jaundice. What is the possible cause of skin itch
Latent stage in such a condition? (2016, N 165, TB)

Recovery +Bile acids accumulating in the blood

Increased blood content of indirect bilirubin

Cholesterol accumulating in the blood


A patient with croupous pneumonia presents
Direct bilirubin appearing in the blood
with sharp increase of body temperature up to
39°C, which persisted for 9 days with daily Erythrocyte disintegration products accumulating
amplitude of 1 degree. What temperature curve in the blood
could be observed? (2016, N 129, TB) +Stable

Hectic During ultrasound investigation a patient has


been diagnosed with bilateral stenosis of renal
Septic
artery with atherosclerotic genesis. Specify the
Recurrent bioactive substance that due to its excessive
secretion is the key component of arterial
Atypical hypertension pathogenesis in the given case:
(2016, N 168, TB) (2015, N 159, TB)
A patient of a neurology unit suffers from +Renin
paralysis of all limbs. Name this condition: (2016,
N 158, TB) Cortisol

+ Tetraplegia Vasopressin

Paraplegia Noradrenaline

Hemiplegia Thyroxin
Modelling of immobilization stress is performed r
ona test animal — guinea pig — that starved for a i
day. Dissection revealed hyperemic gastric a
mucosa with multiple erosions. What theory of
Polyuria
ulcer formation is confirmed by this test? (2016,
N 188, TB) Pollakiuria

+Corticovisceral (stress) Nocturia

Vascular
A newborn child born from Rhnegative mother in
Inflammatory
the result of her third pregnancy presents with
Mechanical gradually worsening jaundice, irritated central
nervous system, anemia. What type of jaundice
Peptic
does the infant suffer from? (2015, N 10, TB)

+Hemolytic
A 40-year-old man presents with rapid weight
gain after he had suffered a severe Hepatocellular
craniocerebral trauma. On examination the
Obstructive
patient’s weight was 125 kg, with his height
being 175 cm. What mechanism of obesity Parasitic
development is the most likely in this case?
Toxic
(2016, N 195, TB) (2015, N 195, TB)
+Hypothalamic
The patient has been hospitalised with
Alimentary pneumonia. What kind of respiratory failure does
the patient have? (2015, N 13, TB) (2014, N 9, TB)
Hormonal
+Restrictive
Hereditary
Obstructive

Central

A victim of a traffic accident is hospitalized at a Peripheral


resuscitation unit. Objectively: the patient is
Thoracic diaphragm
unconscious, BP is 90/60 mm Hg, high blood
content of creatinine and urea is observed,
diurnal diuresis is 80 ml. Characterize the What disorder of local circulation is characterized
patient’s diurnal diuresis: (2015, N 2, TB) by pallor, local temperature drop, pain, local
sensitivity disorder, reduction of the organ
3 > +

volume? (2015, N 21,


sc

TB)
+Ischemia

Venostasis

Thrombosis

Embolism

Arterial hyperemia
Heart rate of a person at rest is 40/min. What A 22-year-old man was stung by bees; the
structure is the pacemaker of heart in this man? affected area became hyperemic and
(2015, N 23, TB) edematous. What is the leading mechanism of
edema development in this patient? (2015, N 59,
+Atrioventricular node
TB)
Sinoatrial node
+Increased permeability of the capillaries
His’ bundle
Decreased hydrostatic blood pressure in the
His’ bundle branches capillaries

Purkinje fibers Increased oncotic pressure of tissue fluid

Impaired lymphatic efflux


The volume of air exhaled by a healthy person
Reduced oncotic pressure of blood
during quiet breathing was measured with a
spirometer and determined to be 0,5 liter. What
is this volume called? (2015, N 24, TB) A patient has obstruction of the common bile
duct. Which of these substances is usually found
+Tidal volume
in urine in such cases? (2015, N 60, TB) (2013,
Inspiratory reserve volume 2012)

Expiratory reserve volume +Bilirubin

Vital capacity of lungs Ketone bodies

Residual volume Uric acid

Creatinine
A patient has been taking diclofenac sodium for a
Glucose
long time. A family physician withdrew this drug
and prescribed celecoxib. What disease was the
cause of drug substitution? (2015, N 25, TB) A patient with systemic lupus erythematosus has
developed diffuse affection of kidneys followed
+Peptic ulcer
by proteinuria, hypoproteinemia, extensive
Bronchial asthma swelling. What mechanism of proteinuria
development is the most likely in this case?
Urolithiasis
(2015, N 63, TB)
Arterial hypertension +Autoimmune disorder of the nephron
Chronic hepatitis glomerulus

Inflammatory disorder of the nephron tubule


Diet of a human must contain vitamins. What
Ishemic disorder of the nephron tubule
vitamin is usually prescribed for treatment and
prevention of pellagra? (2015, N 53, TB) Increased concentration of blood proteins

+Vitamin PP Disorder of the urinary tracts

Vitamin C
A 40-year-old man diagnosed with gastric ulcer
Vitamin A
has developed the symptoms anew after a long
Vitamin B1 period of dormancy. Such disease course can be
characterized as a:
Vitamin D
(2015, N 92, TB)
A patient suffering from coronary heart disease,
+Recurrence
who had had two myocardial infarctions of left
Remission ventricular wall, presents with bubbling
breathing and dyspnea. Pulmonary auscultation
Recovery
reveals numerous moist crackles. What kind of
Latency heart failure is it? (2015, N 122, TB)

Prodromal phase +Left ventricular

R
An elderly patient exhibits low levels of red blood
cells and hemoglobin in blood, but the color

—-3st#5
09 fC eam
index is 1,3. Blood smear analysis revealed
megaloblasts. What type of anemia is observed
in this case? (2015, N 93, TB)

+B12-folic acid deficiency

lron-deficiency

Acquired hemolytic

Hereditary hemolytic
07
oOo

Chronic posthemorrhagic
—~
9

Aman presents with signs of albinism: blonde


3007

hair, extreme photosensitivity, impaired vision.


What amino acid metabolism is disrupted in the
patient? (2015, N 95, TB)
ngono0

+Tyrosine

Methionine
va

Proline
qaorto

Histidine

Valine

Subcompensated
A patient complains of pain in the small joints. Combined
High concentration of uric acid is detected in his
blood plasma. What pathology causes such
changes? (2015, N 118, TB) A man received a radiation dose of 30 Gy. He
presents with necrotic angina, disorders of the
+Gout gastrointestinal tract. Blood tests revealed
Diabetes mellitus anemia, leukopenia and thrombocytopenia.
What period of acute radiation sickness is
Phenylketonuria observed in the patient? (2015, N 130, TB)

Lesch-Nyhan syndrome +Height of disease


Diabetes insipidus Primary reactions
Imaginary wellbeing Peptic gastric ulcer disease

End of disease Dysphagia

Pneumonia
In the course of an experiment in the mesenteric
Somatoform autonomic dysfunction
vein of a toad a trombus was created with a
crystal of common salt. What processes occurred
during the first stage of trombus formation? A 25-year-old-patient with the Il degree thermal
(2015, N 131, TB) + Adhesion, aggregation, burns addressed a doctor. Objectively: there are
agglutination of platelets large blisters on the upper limbs; the blisters are
filled with clear exudate containing mostly water
Production of active thromboplastin
and albumines with isolated leukocytes. Name
Production of thrombin the type of the exudate: (2015, N 181, TB)

Production of fibrin monomer +Serous

Production of fibrin polymer Catarrhal (mucous)

Fibrinous
A patient has a mental disorder due to the
Purulent
insufficient synthesis of gammaaminobutyric acid
in the brain. Such pathological changes might be Hemorrhagic
caused by the deficiency of the following vitamin:
(2015, N 135, TB)
+Pyridoxine After ishemic stroke a 67-year-old patient
developed reduced mobility of the left leg. Name
Tocopherol
this condition: (2015, N 192, TB)
Cyanocobalamin
+Paresis
Folic acid
Paralysis
Riboflavin
Myasthenia

Hyperkinesia
A patient suffers from mucosal dryness and
mesopic vision disorder. What vitamin deficiency Tremor
causes these symptons? (2015, N 149, TB)

+A A 32-year-old patient with cerebellar tumor was


P delivered to an admission room of a hospital. The
patient presents with ataxia that can be
characterized by: (2015, N 193, TB)

+Disrupted coordination of movements

Involuntary contraction of skeletal muscles

Increased muscle tone


A patient was visiting a pharmacy, when he
Pathological reflexes
suddenly felt unwell. He developed palpitations,
rapid heart rate, pain in the chest that after Irregular force and direction of movements
several minutes spread to the left scapula and
left side of the head. What condition should be
During calculous cholecystitis attack the patient
considered first? (2015, N 168, TB) +lschemic
has developed the following symptoms:
heart disease
saponated feces and steatorrhea. What stage of
fats metabolism is disrupted according to those 1 minute after the patient had been
symptoms? (2014, N 3, TB) administered penicillin the patient’s arterial
pressure sharply dropped, pulse became
+Fat digestion, absorption and secretion
thready, cold sweating and clonic convulsions
Fat absorption began. Name this condition. (2014, N 41, TB)

Intermediary metabolism of fats +Anaphylactic shock

Fats metabolism in adipose tissue Traumatic shock

Depositing disruption Cardiogenic shock

Septic shock
The 55-year-old patient has been hospitalised
Burn shock
due to chronic cardiac failure. Objectively: skin
and mucosa are cyanotic, tachycardia,
tachypnea. What kind of hypoxia does the The patient with acute left ventricular failure has
patient have? (2014, N 8, TB) (2012) developed edema of lungs. What peripheral
circulation disorder taking place in the lungs has
+Circulatory caused this complication? (2014, N 56, TB)
+Venous hyperemia
Anemic
Arterial hyperemia
Hemic
Neuroparalytic arterial hyperemia
Tissue
Pulmonary artery thrombosis
Hypoxic
Ischemia

The patient with acute cardiac failure has


developed dyspnea, tachycardia and cyanosis Knee joint enlargement and cutaneous edema
during physical exertion. Name the type of has developed in the 46-year-old patient with
hypoxia. (2014, N 23, TB) +Circulatory acute knee joint inflammation on the second
day. What stage of inflammation progressing are
Respiratory
these symptoms usually observed at? (2014, N
Hemic 57, TB)
Hypoxic +Exudation

Tissue Alteration

Proliferation
At the sixth month of pregnancy the female
Regeneration
patient has been diagnosed with severe iron-
deficiency anemia. Diagnostic character was the Sclerosis
appearance of the following in blood: (2014, N
36, TB) +Hypochromic erythrocytes
In the process of chemical solution preparation
Macrocytes laboratory assistant’s forearm was exposed to
concentrated hydrochloric acid. There are
Megalocytes
burning pain, hyperemia and swelling of the
Reticulocytes damaged area. What pathologic process are
these symptoms evidential of? (2014, N 59, TB)
Erythroblasts
+Inflammation
Tumor
Tetanic spasms of skeletal muscles occur under
Embolism
low calcium concentration in blood. What
Thrombosis endocrine disorder can this condition be
associated with? (2014, N 98, TB)
Lymphostasis
+Hypofunction of parathyroid glands

In 1915 Japanese scientists Katsusaburo Hyperfunction of adrenal cortex


Yamagiwa and Koichi Ichikawa became the first,
Hypofunction of adrenal cortex
who induced experimental tumors, by painting
ears of rabbits with coal tar. What method of Hyperthyroidism
experimental tumor inducing did they use?
Hypothyroidism
(2014, N 65, TB)
+Chemical induction
A newborn infant has hemolytic jaundice caused
Transplantation by rhesus incompatibility. What bile pigment will
be concentrated highest in the blood of this
Explantation
infant? (2014, N 107, TB)
Cell-free filtrate induction
+Unconjugated bilirubin
Radioisotope induction
Conjugated bilirubin

Urobilinogen
The alleged diagnosis of the newlyhospitalised in-
patient is leukemia. What symptom among those Stercobilinogen
given below is diagnostic character
Bile acids
differentiating acute leukemia from chronic
leukemia? (2014, N 66, TB)
The patient with acute cardiac insufficiency has
+Leukemic hiatus
decreased urine excretion caused by reduction of
Significant increase of leucocytes number filtering taking place in glomerules. What causes
this drop in filtration? (2014, N 108, TB)
Leukosis rate
+Decrease of arterial pressure
Eosinophil and basophil levels
Increase of hepatic blood flow
Gumprecht’s shadows (smudge cells)
Exsiccosis

Duct lumen obstruction


Catabolism of body’s own tissue proteins is
intensified during such diseases as thyrotoxicosis Decrease in number of functioning glomerules
and tuberculosis. This process is attended by
intensive synthesis in liver and subsequent
The 49-year-old female patient suffering long-
excretion with urine of the following: (2014, N
term from pancreatic diabetes has developed the
88, TB)
following symptoms after administering insulin:
+ Urea weakness, facial pallor, palpitation, anxiety,
double vision, numbness of lips and tongue apex.
Glucose
Glucose molar concentration in blood was 2,5
Acetone bodies mmol/I. What complication has developed in the
patient? (2014, N 109, TB)
Fatty acids
+Hypoglycemic coma
Nucleotides
Hyperosmolar coma Milk intake has resulted in the one-year-old child
having diarrhea and abdominal distension. What
Hyperglycemic coma
enzyme deficiency does the child have? (2014, N
Hyperketonemic coma 115, TB) +Lactase

Uremic coma Maltase

Aldolase
The 40-year-old patient has been diagnosed with
Hexokinase
gastric ulcer, disease symptoms making
reappearance after prolonged period of Glycosidase
dormancy. How can this kind of disease
progression be qualified? (2014, N 110, TB)
The 56-year-old patient has developed
+Relapse megaloblastic anemia in the course of alcoholic
cirrhosis. What vitamin deficiency is the main
Remission
cause of anemia in this patient? (2014, N 116,
Recovery TB)
+
Latent period
F
Prodromal stage 0

|
The 55-year-old female patient has developed a
-ovbpr-ran-ranawaononnr

case of acute pancreatitis caused by greasy food.


What is the main pathogenesis step of this
disorder? (2014, N 111, TB)

+Premature activation af enzymes in gland ducts


and cells

Pancreatic juice deficiency

Low bile production in liver

Fats digestion disruption

Acute bowel obstruction


a Oo

i
As the result of taking herbal medicine the 30-
d
year-old patient has developed anaphylactic
allergic reaction and blood leukocytosis. What Biotin
kind of leukocytosis is characteristic of this case?
Thiamine
(2014, N 112, TB)
Pantothenic acid
+Eosinophilia

Monocytosis
The dispensing chemist’s arterial pressure has
Lymphocytosis increased {160/110 mm Hg) due to his
conducting long-term analytical analysis
Basophilia
(neurosis). What neurohumoral regulation
Heutrophilia changes can cause increased arterial pressure in
the given case? (2014, N 133, TB)
+Sympathoadrenal system activation Increase of hydrodynamic blood pressure

Activation of aldosterone producing and Hormonal disbalance


secretion
Lymph flow disruption
Renin-angiotensin system activation

Kallikrein-kinin system activation Fluorography examination of the 59-year-old


patient has revealed welldefined shadow, which
Sympathoadrenal system inhibition
is characteristic to tumor, in the lower part of the
left lung. What trait is characteristic of benign
The 13-year-old female patient having suffered tumor? (2014, N 171, TB)
from measles complains of dry mouth, thirst,
+Expansive growth
body weight loss, polyuria, her glucose
concentration in blood is 16 mmol/l. What Metastasis
disease can be suspected? (2014, N 141, TB)
Cancer cachexia
+Type | pancreatic diabetes
Invasion in surrounding tissues
Type II pancreatic diabetes
Infiltrating growth
Diabetes insipidus

Steroidogenic diabetes

Glycogenosis
1. A 25-year-old woman has been suffering
from diabetes mellitus since she was 9. She was
The patient with mushroom poisoning has admitted into the nephrology unit with significant
developed the following symptoms: yellow edemas of the face, arms, and legs. Blood pressure
coloring of skin and sclera, dark-colored urine. - 200/110 mm Hg, Hb- 90 g/L, blood creatinine -
Hemolytic jaundice was diagnosed. What 850 memol/L, urine proteims - 1.0 g/L, leukocytes
pigment causes such coloring of the patient’s - 10-15 in the vision field. Glomerular filtration
rate 10 mL/min. What tactics should the doctor
urine? (2014, N
choose?
142, TB)
A. Transfer into the hemodialysis unit B. Active
+Stercobilin conservative therapy for diabetic nephropathy C.
Dietotherapy
Conjugated bilirubin D. Transfer into the endocrinology clinic
E. Renal transplantation
Biliverdin
2. A 59-year-old woman was brought into the
Unconjugated bilirubin theumatology unit. Extremely severe case of
scleroderma is suspected. Objectively she
Verdohemoglobin
presents with malnourishment, ’*mask-like” face,
and acro-osteolysis. Blood: erythrocytes - 2.2 -
The patient has been admitted to the hospital 10°/L, erythrocyte sedimentation rate 40
mivhour. Urine: elevated levels of free
with complaints of general fatigue, headache,
oxyproline. Name one of the most likely
lumbago, edema of face and extremities. Urine
pathogenetic links in this case:
analysis revealed proteinuria, hematuria and
cylindruria. What is the main pathogenetic A. Formation of antibodies to collagen
mechanism of edema formation during B. Formation of antibodies to native DNA C.
Formation of antibodies to blood
glomerulonephritis? (2014, N 168, TB)
corpuscles
+Decrease of oncotic blood pressure D. Formation of antibodies to transversely striated
muscles
Increase of vascular permeability
E. Formation of antibodies to vessel wall 1.2-109/L, platelets - 42-109/L. What analysis
would be the most advisable for diagnosismaking
3. A 34-year-old man on the 3rd day of
in this case?
ceftriaxone treatment for acute otitis (daily
dosage - 2 grams) developed diarrhea occurring A. Sternal puncture (bone marrow biopsy)
5-6 times per day. Feces are without mucus or B. Splenic biopsy
blood admixtures. Temperature is 36.6°C. C. Liver biopsy
Gregersen reaction (occult blood in feces) is D. Coagulation studies
negative. Stool culture detected no pathogenic E. US of the gastrointestinal tract
germs. What 1s the most likely cause of diarrhea
in this case? 7. A 35-year-old man complains of rapidly
incresing fatigue, palpitations, ’visual snow”, and
A. Antibiotic-associated diarrhea dizziness. He has a history of peptic ulcer of the
B. Intestinal dysbiosis stomach. Objectively the skin is pale. Vesicular
C. Bacterial overgrowth syndrome respiration is observed in the lungs. Systolic
D. Ulcerative colitis murmur is detected over the cardiac apex, heart
E. Crohn’s disease (regional enteritis) rate is 100/min., BP is 100/70 mm Hg. The
epigastrium is slightly tender on palpation. Blood
4. A chronic alcoholic was hospitalized into the
test: erythrocytes - 3.2 - 1012/L, Hb- 100 g/L,
therapeutic inpatient unit due to pneumonia. On
color index 0.95. What type of anemia is the most
the day 5 of his hospitalization he became
likely present in this case?
disoriented in time and space, developed fear-
inducing visual hallucinations and motor A. Posthemorrhagic anemia
agitation. Full body tremor and tremor of the B. Sideroblastic anemia
limbs are observed. X-ray and _ physical C. Chronic iron-deficiency anemia
examinations detect the signs of his D. Hemolytic anemia
convalescence from pneumonia. What tactics E. Hypoplastic anemia
should be chosen regarding this patient?
A. Transfer into the inpatient 8. A 62-year-old patient has been hospitalized
narcology department with complaints of pain in the thorax on the right
B. Continue the treatment in the therapeutic during breathing, dyspnea, and dry cough. Ten
department days ago he slipped and fell hitting his right side.
C. Transfer into the On examintaion: the patient lies on the left side.
neuroresuscitation department The right side of the thorax lags during breathing.
D. Compulsory medical treatment On the right there are crepitation and pain in the
for I-IV ribs. Dullness of percussion sound and
alcoholism E. Discharge from the sharply dimimished breath sounds can _ be
hospital observed. On X-ray: signs of exudate, fracture of
the II-IV ribs. On pleurocentesis: blood is
5. After eating shrimps, a 25-year-old man detected. Choose the further tactics:
suddenly developed skin itching, some areas of
his skin became hyperemic or erupted into A. Transfer to a thoracic surgery department
vesicles. Make the diagnosis: B. Prescribe conservative therapy
C. Perform repeated pleural taps
A. Acute urticaria D. Apply a fixation bandage to the rib cage
B. Hemorrhagic vasculitis (Henoch-Schonlein E. Refer to a traumatologist
purpura) C. Urticaria
pigmentosa 9. A 51-year-old man complains of vomiting with
D. Psoriasis blood. He has been drinking alcohol excessively.
Health disorder has been observed since he was 40,
E. Scabies
when he first developed jaundice. On examination
6. A 25-year-old woman complains of fatigue, the skin and visible mucosa are icteric, with a
dizziness, hemorrhagic rashes on the skin. She stellate vascular pattern. The patient is
has been presenting with these signs for a month. malnourished and presents with abdominal
Blood test: erythrocytes - 1.0 - 1012/L, Hb- 37 distension, umbilical hernia, and ascites. The edge
g/L, color index - 1.1, leukocytes - of the liver is tapered and painless, +3 cm, the
spleen is +2 cm. Blood test: Hb- 80 g/L, leukocytes
- 3-10°/L, platelets - 85-109/L. What is the cause
of portal hypertension in this patient?
A. Hepatic cirrhosis 13. A woman came to the doctor with complaints
B. Thrombosis of the splenic vein of increased body temperature up to 37.8°C and
C. Hemochromatosis moderately sore throat for the last 3 days.
D. Constrictive pericarditis Objectively: mandibular lymph nodes are
E. Budd-Chiari syndrome enlarged up to 3 cm. Palatine tonsils are
hypertrophied and covered with gray coating that
10. A young man has made an appointment with spreads to the uvula and anterior pillars of the
the dermatologist. He complains of a painful facial fauces. What is the most likely diagnosis?
rash in the beard and mustache area. This condition
has been persisting for several weaks already. A. Oropharyngeal diphtheria
After shaving, the patient’s condition aggravates. B. Infectious mononucleosis
The diagnosis of sycosis is made. What primary C. Pseudomembranous (Vincent’s) tonsillitis
morphological elements can be observed in the D. Agranulocytosis
rash in this case? E. Oropharyngeal candidiasis

A. Pustules, papulae 14. A 42-year-old man, a dispatcher, suffes from


B. Nodes, nodules peptic ulcer disease of the duodenum. The disease
C. Pustules, bumps is of moderate severity. He wants to be assigned
D. Phlyctenae, maculae a disability group. Make the conclusion regarding
E. Maculae, nodes his working ability:

11. At night a 63-year-old woman suddenly A. Capable of working, employable


developed an asphyxia attack. She has a 15year- B. Capable of working, non-employable
long history of essential hypertension and had a C. First group of disability
myocardial infarction 2 years ago. Objectively her D. Second group of disability
position in bed is orthopneic, the skin is pale, the E. Third group of disability
patient is covered with cold sweat, acrocyanosis is
observed. Pulse - 104/min. Blood pressure - 15. A 58-year-old man complains of weakness
210/130 mm Hg, respiration rate - 38/min. and tumor-like formations that appeared on the
Pulmonary percussion sound is clear, with slight anterior surface of his neck and in the inguinal
dullness in the lower segments, throughout the region. Palpation detects soft painless mobile
lungs single dry crackles can be heard that become cervical and inguinal lymph nodes up to 2 cm in
bubbling and non-resonant in the lower segments. diameter. The liver protrudes by 2 cm from the
What is the most likely complication im this edge of the costal margin, the lower splenic pole
patient? is at the umbilical level. In blood: erythrocytes -
A. Acute left ventricular failure 3.5 + 1012/L, Hb- 88 g/L, leukocytes -
B. Paroxysmal tachycardia 86 - 10°/L, band neutrophils - 1%, segmented
C. Bronchial asthma attack neutrophils - 10%, lymphocytes - 85%,
D. Pulmonary embolism eosinophils - 2%, basocytes - 0%, monocytes -
E. Acute right ventricular failure 2%, erythrocyte sedimentation rate - 15 mm/hour,
Gumprecht shadows. What is the most likely
12. A 26-year-old woman is suspected to suffer
diagnosis?
from systemic lupus erythematosus due to
systemic lesions of skin, vessels, joints, serous A. Chronic lymphatic leukemia
tunics, and heart that developed after B. Lymphocytic leukemoid reaction
photosensitization. The following is detected: LE
C. Acute leukemia
cells, antibodies to native DNA, isolated anti-
D. Chronic myeloleukemia
centromere antibodies, rheumatoid factor is
E. Lymphogranulomatosis
1:100, Wassermann reaction is _ positive,
circulating immune complex is 120 units. What 16. A 23-year-old man complains of facial edemas,
immunological indicators are considered to be headache, dizziness, low urinary output, and urine
specific to this disease? discoloration (dark red). These complaints arose
after a case of acute tonsillitis. On examination
A. DNA antibodies
there are facial edemas, the skin is_ pale,
B. Rheumatoid factor
temperature is 37.4°C; heart rate is 86/min., blood
C. Anti-centromere antibodies
pressure is 170/110 mm Hg. Heart sounds are
D. Immunoglobulin A
muffled, the II heart sound is accentuated over the
E. Increased circulating immune complex
aorta. What etiological factor 1s the most likely in 20. A 63-year-old man complains of unmotivated
this case? weakness and pressing and bursting sensation in
the left subcostal area. According to him, these
A. Beta-hemolytic streptococcus signs have been present for a year already.
B. Staphylococcus aureus Previously he was healthy. He took part in
C. Streptococcus viridans containment measures during the accident at the
D. Streptococcus pyogenes Chornobyl Nuclear Power Plant. Objectively: the
E. Staphylococcus saprophyticus skin is pale, peripheral lymph nodes are not
enlarged, the liver is +3 cm, the spleen is +10 cm.
17. An 18-year-old young man complains of pain
Complete blood count: erythrocytes - 3.1 -
in his knee and elbow jomts and body temperature
1012/L, Hb- 100 g/L, leukocytes - 46 - 109/L,
up to 39.5°C. One week and a half earlier he
blasts - 2%, promyelocytes - 10%, myelocytes
developed sore throat. On examination his body
18%, band neutrophils - 27%, segmented
temperature is 38.5°C. Swelling of the knee and
neutrophils - 10%, lymphocytes - 12%,
elbow jomts is observed. Pulse is 106/min.,
eosinophils - 6%, basocytes - 3%, monocytes -
rhythmic. Blood pressure is 90/60 mm Hg. Cardiac
2%, erythrocyte sedimentation rate - 20 mm/hour.
borders are unchanged, heart sounds are
What is the most likely diagnosis?
weakened, at the cardiac apex there is a soft
systolic murmur. What factor would be the most A. Chronic myeloleukemia
indicative of the likely disease etiology? B. Hepatic cirrhosis
A. Anti-streptolysin O C. Acute leukemia
B. C-reactive protein D. Hemolytic anemia
C. Creatine kinase
E. Chronic lymphatic leukemia
D. Rheumatoid factor 21. For three years a 31-year-old woman has been
E. Seromucoid complaining of pam and swelling of her
radiocarpal and metacarpophalangeal
18. A woman has been provisionally diagnosed
articulations and their reduced mobility in the
with pheochromocytoma. At the stage of
morning, which persisted up to 1.5 hours. Two
intermission her BP is within norm; there is a
weeks ago she developed pain, swelling, and
tendency towards tachycardia. No urine
reddening of her knee joints, her body
pathologies. The decision has been made to
temperature increased up to 37.5°C. The treatment
perform a provocative test with histamine. What
was untimely. Examination of the internal organs
drug should be kept close at hand for emergency
revealed no pathologic alterations. Diagnosis of
aid in case of positive test result?
rheumatoid arthritis was made. What changes are
A. Phentolamine most likely to be visible on the arthrogram?
B. Pipolphen (Promethazine)
A. Joint space narrowing, usuration
C. Nifedipine
B. Jomt space narrowing,
D. Mesaton (Phenylephrine)
subchondral osteosclerosis
E. Prednisolone
C. Cysts in the subchondral bone
19. A 40-year-old man with Bekhterev disease D. Numerous marginal osteophytes
(ankylosmg spondylitis) complains of elevated E. Epiphyseal osteolysis
body temperature up to 37.8°C, back pain and
22. A 52-year-old woman has been suffering for
stiffness, especially observed during the second
2 years from dull, occasionally exacerbating pain
half of the night. This condition has been lasting
in her right subcostal area, occurring after eating
for 2 years. Objectively: reduced spinal mobility,
high-fat foods, bitter taste in her mouth in the
painful sacroiliac joint, erythrocyte sedimentation
morning, constipations, and flatulence.
rate - 45 mm/hour. X-ray shows narrowing of the
Objectively she has excess weight, her body
intervertebral disc space and of the sacroiliac joint.
temperature is 36.9°C; there is a coating on the
What eye pathology is often associated with this
root of her tongue; the abdomen is moderately
type of disease progression?
distended and painful in the area of gallbladder
A. Iridocyclitis projection. What examination would be the most
B. Retinal detachment helpful for diagnosismaking?
C. Cataract
A. Ultrasound
D. Optic nerve atrophy
B. Duodenal intubation
E. Blepharitis
C. Cholecystography 26. A 28-year-old man complains of skin rash and
D. Duodenoscopy itching on the both of his hands. The condition
E. Liver scanning persists for 1.5 years. The exacerbation of his
condition he ascribes to the occupational contact
23. A 57-year-old woman complains of weakness, with formaldehyde resins. Objectively the lesion
dyspnea, loss of appetite, and liquid feces. She has foci are symmetrically localized on both hands.
been suffering from this condition for 2 years. Against the background of erythema with blurred
Objectively she presents with pale skin, subicteric margins there are papulae, vesicles, erosions,
sclerae, and bright-red fissured tongue. Lymph crusts, and scales. What is the most likely
nodes are not enlarged. Pulse - 100/min. BP- pathology?
105/70 mm Hg. Liver +3 cm, the spleen cannot be
palpated. A. Occupational eczema
Blood test: erythrocytes - 1.2 - 1012/L, Hb- 56 g/L, B. Idiopathic eczema
color index - 1.4, macrocytes, leukocytes - 2,5 - C. Allergic dermatitis
109/L, eosinophils - 1%, juvenile - 1%, D. Simple contact dermatitis
metamyelocytes - 1%, band neutrophils - 8%, E. Erythema multiforme
segmented neutrophils 47%, lymphocytes - 38%,
27. A 20-year-old student after failing an exam
monocytes - 4%, reticulocytes - 0.1%, platelets -
developed complaints of a sensation of a round
100 - 109/L, ESR- 30 mm/hour, indirect bilirubin
foreign body in her throat, difficult swallowing.
- 26 mmol/L. What changes can be expected in the
bone marrow puncture material? She fixates on her condition, limits her diet, often
cries, seeks attention, exhibits demonstrative
A. Prevalence of megaloblasts attitude. She is highly susceptible to
B. Increased number of sideroblasts psychotherapeutic suggestion. What psychiatric
C. Erythroid hyperplasia diagnosis can be made in this case?
D. Presence of blast cells
A. Hysterical neurosis
E. Prevalence of lymphoid tissue
B. Hypochondriacal neurosis
24. A 35-year-old man _ suffers from C. Depressive neurosis
insulindependent diabetes mellitus and chronic D. Obsessive neurosis
cholecystitis. He takes NPH insulin: 20 units in the E. Paranoid personality disorder
morning and 12 units in the evening. After a meal
he developed pain in the right subcostal area, 28. A woman with atopic bronchial asthma was
nausea, vomiting, sleepiness, and increased found to have one allergen to dog hair +++.
polyuria. What prehospital measures will be the Carpets were removed from the apartment, the
most effective for prevention of crisis within the apartment was renovated, and air conditioner was
next several hours? installed. However, recurrent asphyxia attacks
still occur every night, despite the patient
A. Change insulin regimen undergoing pathogenetic therapy. What long-
B. Take analgesics term treatment tactics can help this patient to
C. Take cholagogues decrease her sensitivity to the allergen?
D. Exclude fats from the diet
A. Specific hyposensitization
E. Decrease carbohydrates in the diet
B. Continuation of prior treatment
25. A 45-year-old woman complains of intolerable C. Antihistamine therapy
paroxysmal facial pain on the left with attacks that D. Buteyko breathing technique
last for 1-2 minutes. Attacks are provoked by E. Referral for speleotherapy
chewing. The disease onset was two months ago
after the overexposure to cold. Objectively: pain at 29. A 20-year-old man was hospitalized on the 9th
the exit points of the trigeminal nerve on the left. day of the disease. He attributes his disease to
Touching near the wing of the nose on the left eating of insufficiently thermally processed pork.
induces a pain attack with tonic spasm of the facial At its onset this condition manifested as
muscles. What is the most likely diagnosis? periorbital edemas and fever. Objectively his
body temperature is 38.5 C. The face is puffy and
A. Trigeminal neuralgia the eyelids are markedly swollen. Palpation of
B. Glossopharyngeal neuralgia gastrocnemius muscles is sharply painful. Blood
C. Temporomandibular joint arthritis test shows hypereosinophilia. What is the
D. Facial migraine etiology of this disease?
E. Maxillary sinusitis
A. Trichinella depression of S-T segment by 3 mm below the
B. Trichuris isoline in V3-V4. What is the provisional
C. Ascarididae diagnosis?
D. Echinococci
A. Exertional angina pectoris, functional
E. Leptospira
class III
30. A 40-year-old man claims that his wife is B. Exertional angina pectoris, functional
cheating on him and presents a ’proof” of her class IV
infidelity. He repeatedly initiated scandals with his C. Exertional angina pectoris, functional
wife at home and at work, demanding that she class II
confess her infidelity, insulted her, and threatened D. Somatoform autonomic dysfunction,
to kill her. What preventive measures should be hypertension type E. Alcoholic
taken against socially dangerous actions on his myocardiodystrophy
part?
34. A 45-year-old man, a farmer, presents with
A. Consultation with the psychiatrist acute onset of a disease. He complains of
B. Outpatient treatment headache, high temperature, pain im _ the
C. Consultation with the general practitioner gastrocnemius muscles, icteric face, and dark
D. Consultation with the psychologist urine. Objectively: body temperature - 38°C,
E. Family counseling blood pressure - 100/70 mm Hg, conjunctival
hemorrhages, hepatosplenomegaly, and oliguria.
31. A 55-year-old woman complains of pain and What is the most likely provisional diagnosis?
popping sounds in her left knee joint, which occur
when she climbs the stairs. Occasionally during A. Leptospirosis
movements her joint becomes ”’stuck’’. 5 years ago B. Brucellosis
she suffered a trauma of her left knee. Complete C. Viral hepatitis
blood count and biochemical blood analysis show D. Pseudotuberculosis
normal results. X-ray shows marked osteosclerosis E. Trichinosis
and osteophytes. The joint space is narrowed.
Make the provisional diagnosis: 35. A woman has been working as a polisher for
a year and a half. Her workstation is equipped
A. Osteoarthritis with a grinding machine (grinding wheels). She
B. Rheumatoid arthritis complains of white discoloration of her fingers
C. Gouty arthritis D. Psoriatic arthritis and toes that appears when she is nervous.
E. Reactive arthritis Objectively there are no changes in the coloration
of the distal segments of her limbs. Grip strength
32. A 40-year-old man, a welder, uses manganese measured with a dynamometer is 25 kg,
electrodes in his line of work (18 years of algesimetry fmdings are 0.1; 0.3; 0.5. Cold
experience). He complains of difficulties with stimulus is extremely positive on the upper and
writing, bad mood, inertness, gait abnormalities, lower limbs. Internal organs are without
problems with speech, and hand tremors. pathologies. Make the diagnosis:
Objectively the following is observed in the
patient: hypomimia, mcreased muscle tone of A. Vibration disease
plastic type, and quiet monotonous speech, tremor B. Raynaud disease
of the tongue, pill-rolling tremor of the fingers, and C. Syringomyelia
retropulsion. What syndrome developed in this D. Raynaud syndrome
patient due to manganese poisoning? E. Polyneuritis

A. Parkinsonism 36. A 37-year-old man suddenly developed acute


B. Meningism headache accompanied by nausea, vomiting, and
C. Hypothalamic syndrome impaired consciousness. Objectively blood
D. Polyneuritic syndrome pressure is 190/120 mm Hg, the face is
E. Vestibular syndrome hyperemic. Patient’s consciousness is clouded,
his answers to the questions are short,
33. A 45-year-old man developed constricting monosyllabic. Movement and — sensory
retrosternal pain that occurs during walks at the disturbances are absent. Meningeal signs are
distance of 200 m. Objectively: heart rate is positive. Cerebrospinal fluid contains blood.
80/min., BP is 160/00 mm Hg. During What provisional diagnosis can be made?
cardiopulmonary exercise test at 50 W there is a A. Subarachnoid hemorrhage
B. Meningitis analysis would be the most indicative of such
C. Ischemic stroke infection?
D. Encephalitis A. Leukocyturia, gross hematuria
E. Cerebral vascular embolism B. Gross hematuria
C. Increased blood creatinine and blood urea
37. A woman undergoing in-patient treatment for
D. Daily proteinuria under 3.0
viral hepatitis type B developed headache, nausea,
E. Daily proteinuria over 3.0
recurrent vomiting, memory lapses, flapping
tremor of her hands, and rapid pulse. Sweet smell 41. A 62-year-old woman was brought into the
from her mouth is detected. Body temperature is admission room with complaints of severe
37.6°C, heart rate is 89/min. What complication burning retrosternal pain and asphyxia. She has a
developed in the patient? 10-year-long history of essential hypertension.
Objectively her condition is moderately severe.
A. Acute liver failure
She presents with skin pallor, cyanotic lips, and
B. Ischemic stroke
vesicular respiration over her lungs. The II heart
C. Gastrointestinal hemorrhage sound is accentuated over the aorta. Blood
D. Hypoglycemic shock pressure - 210/120 mm Hg, heart rate (pulse) -
E. Meningoencephalitis 76/min. ECG shows elevation of ST segment in
38. A 43-year-old man, a coal-face worker with 15- the leads I, AVL, and V5-V6. What is the most
year-long record of work, complains of cough, likely diagnosis?
thoracic pain, and dyspnea. The cough is mild, A. Hypertensive crisis complicated with acute
usually dry, occurs mostly in the morning. The myocardial infarction B. Uncomplicated
pain is localized in the interscapular region and hypertensive crisis
aggravates during a deep intake of breath. Dyspnea C. Hypertensive crisis complicated with imstable
occurs during physical exertion. Vesicular angina pectoris
respiration in the lungs is weakened. Heart sounds D. Hypertensive crisis complicated with acute left
are rhythmic, heart rate 1s 86/min., blood pressure ventricular failure
is 135/80 mm Hg. The abdomen is soft and
E. Pulmonary embolism
painless. X-ray shows micronodular pulmonary
fibrosis. Make the provisional diagnosis: 42. A 35-year-old patient developed an epileptic
attack with tonoclonic spasms that lasted for 3
A. Carboconiosis minutes. After the attack the patient fell asleep but
B. Byssinosis in 5 minutes the second attack occurred. The first
C. Siderosis step of emergency aid would be to:
D. Berylliosis
E. Metal pneumoconiosis A. Ensure patency of airways
B. Take blood from the vein for analysis
39. Having examined a 52-year-old patient, the C. Introduce diazepam intravenously
doctor diagnosed him with obesity (body mass D. Prescribe antiepileptic drugs
index - 34 kg/m2, waist circumference - 112 cm)
E. Administer chloral hydrate via an enema
and arterial hypertension (170/105 mm Hg). 2-
hour postprandial blood sugar is 10.8 mmol/L. 43. A 27-year-old woman, a teacher in the
What biochemical blood analysis needs to be elementary school, complains of frequent stools,
conducted to diagnose the patient with metabolic up to 3 times per day, with lumpy feces and large
syndrome X? amount of mucus, abdominal pain that gradually
abates after a defecation, irritability. Her skin is
A. Lipid profile pale and icteric. Pulse is 74/min., rhythmic, can
B. Bilirubin be characterized as satisfactory. Blood pressure is
C. Calcrum and phosphorus 115/70 mm Hg. The abdomen is soft, moderately
D. Creatinine and urea tender along the colon on palpation. Fiberoptic
E. Electrolytes colonoscopy detects no changes. What disease
can be suspected?
40. After overexposure to cold a 45year-old
woman developed acute pain in her suprapubic and A. Irritable bowel syndrome
lumbar areas during urination, sharp pains at the B. Chronic non-ulcerative colitis
end of urmation, false urges to urinate. Urine is C. Chronic enteritis
turbid with blood streaks. The doctor suspects D. Crohn disease (regional enteritis)
urinary tract infection. What results of laboratory
E. Whipple disease 47. A 36-year-old man complains of marked
dyspnea and cardiac pain. He ascribes his disease
44. A 72-year-old man complains of lower to the case of influenza that he had 2 weeks ago.
extremity edema, sensation of heaviness in the Objectively he leans forward when sitting. The
right subcostal area, dyspnea at rest. For over 25
face is swollen, cyanotic, cervical veins are
years he has been suffering from COPD.
distended. Heart borders are extended on the both
Objectively: orthopnea, jugular venous sides, heart sounds are muffled, heart rate = Ps =
distention, diffuse cyanosis, acrocyanosis. Barrel
118/min., BP is 90/60 mm Hg. Blood test: ESR is
chest is observed, on percussion there is a
46 mm/hour. ECG shows low voltage. Xray
vesiculotympanitic
shows trapezoidal cardiac silhouette and signs of
(bandbox) resonance, sharply weakened vesicular pulmonary congestion. Choose the treatment
respiration on both sides, moist crepitant crackles tactics:
in the lower segments of the lungs. Heart sounds
are weakened, the II heart sound is accentuated A. Pericardial puncture (pericardiocenthesis)
over the pulmonary artery. The liver is +3 cm. B. Diuretics
What complicated the clinical course of COPD in C. Antibiotics
this patient? D. Pericardectomy
E. Glucocorticosteroids
A. Chronic pulmonary heart
B. Pulmonary embolism 48. A 39-year-old man suffers from chronic
C. Acute left ventricular failure rheumatic heart disease. He complains of dyspnea
D. Diffuse pneumosclerosis during physical exertion, cough with
E. Community-acquired pneumonia expectoration, and palpitations. Ausculation
detects intensified I heart sound and diastolic
45. A 72-year-old man with pneumonia murmur; the sound of opening mitral valve can be
complains of marked dyspnea, chest pain, severe auscultated at the cardiac apex. The IT heart sound
cough with expectoration, ft is 39.5-40°C, no is accentuated over the pulmonary artery. The
urination for a whole day. Objectively the patient patient is cyanotic. X-ray shows dilated
is conscious. Respiratory rate is 36/min. Over the pulmonary root and enlargement of the nght
right lower pulmonary lobe percussion sound is ventricle and left atrium. What is the most likely
dull; on auscultation there is bronchial respiration diagnosis?
and numerous moist crackles. Blood pressure is
80/60 mm Hg. Heart rate is 120/min. Heart A. Mitral stenosis
sounds are muffled, there is tachycardia. What B. Aortic stenosis
tactics should the family doctor choose in the C. Pulmonary artery stenosis
management of this patient? D. Coarctation of the aorta
E. Patent ductus arteriosus
A. Hospitalization into the intensive care unit
B. Outpatient treatment 49. A 23-year-old man complains of severe pain
C. Treatment in the day patient facility in his left knee joint. Objectively the left knee
D. Hospitalization into the pulmonology unit joint is enlarged, with hyperemic skin, painful on
E. Hospitalization into the neurology unit palpation. Complete blood count: erythrocytes -
3.8 - 1012/L, Hb- 122 g/L, leukocytes - 7.4 -
46. 2 hours after eating unknown mushrooms, a 10°/L, platelets 183 109/L. Erythrocyte
28-year-old man sensed a decrease in his mobility sedimentation rate
and deterioration of his ability to focus. This
condition was then followed by a state of - 10 mm/hour. Bleeding time (Duke method) - 4
agitation and agression. On examiantion he is min., Lee-White coagulation time - 24 mun.
disoriented and his speech is illegible. 4 hours Partial thromboplastin time (activated) - 89
later he developed fetor hepaticus and lost his seconds. Rheumatoid factor - negative. What is
consciousness. What syndrome can be observed the most likely diagnosis?
in this patient?
A. Hemophilia, hemarthrosis
A. Acute hepatic failure B. Werlhof disease (immune
B. Hepatolienal syndrome thrombocytopenia) C.
C. Portal hypertension Rheumatoid arthritis
D. Cholestatic syndrome D. Thrombocytopathy
E. Cytolytic syndrome E. Hemorrhagic vasculitis (Henoch-Schonlein
purpura), articular form
50. A 24-year-old woman, a kindergarten teacher, C. Esophageal achalasia
has been sick for 2 days already. Disease onset D. Reflux esophagitis
was acute. She presents with elevated body E. Esophageal diverticulum
temperature up to 38.0°C, pain attacks in her
lower left abdomen, liquid stool in small amounts 54. A 53-year-old man complains of general
with blood and mucus admixtures 10 times a day. weakness, loss of appetite, and painful vesicles
Pulse - 98/min., blood pressure - 110/70 mm Hg. appearing on his skin. The disease onset occurred
Her tongue is moist and coated with white suddenly, after hyperinsolation one week ago.
deposits. The abdomen is soft, the sigmoid colon Examination detects isolated vesicles with
is painful and spastic. Make the provisional wrinkled opercula and occasional painful
diagnosis: erosions on the skin of the patient’s torso and
limbs. Nikolsky sign is positive. What is the most
A. Shigellosis likely diagnosis?
B. Escherichiosis
A. Acantholytic pemphigus
C. Salmonellosis
B. Nonacantholytic pemphigus
D. Yersiniosis
C. Duhring’s disease (dermatitis
E. Rotavirus infection
51. A 38-year-old woman complains of weakness, herpetiformis)
sleepiness, pain in the joints, weight gain despite D. Herpes
low appetite, and constipations. She presents with E. Toxicodermia
dry and thickened skin, puffy and amimic face, 55. A patient is being treated
narrowed palpebral fissures, thick tongue, and in the tuberculosis clinic. Throughout the
deep hoarse voice. Her heart sounds are weak, last 3 weeks he has been suffering from
pulse is 56/min. Low levels of free T4 are headaches of increasing intensity. Neurological
observed. This patient needs to take the following examination detects nuchal rigidity without focal
on a regular basis: signs. Make the provisional diagnosis:
A. Thyroxine A. Tuberculous meningitis
B. Mercazolil (Thiamazole) B. Chorea minor
C. Lithium carbonate D. Furosemide C. Brain tumor
E. Calcium gluconate D. Myelitis
52. A 23-year-old man has accidentally E. Convexital arachnoiditis
swallowed brake fluid. After that he has been 56. A patient has gradually lost consciousness.
presenting with anuria for 5 days already; his The skin is pale and dry. There is a smell of
creatinine levels elevated up to 0.569 mmol/L.
ammonia from the mouth. Respirations are deep
What treatment tactics should be chosen in this
and noisy. Heart sounds are muffled, pericardial
case? friction rub is present. Blood pressure is 180/130
A. Hemodialysis mm Hg. Blood test: Hb- 80 g/L, leukocytes 12 -
B. Detoxication therapy 10°/L, blood glucose - 6.4 mmol/L, urea 50
C. Antidotal therapy mmol/L, creatinine - 1200 mcemol/L, blood
D. Diuretics osmolarity - 350 mOsmol/L. No urinary
excretion. Make the diagnosis:
E. Plasmapheresis
A. Uremic coma
53. A 52-year-old man for the last 3 years has
been suffering from difficult swallowing of solid B. Hyperglycemic coma
food, burning retrosternal pain that aggravated C. Acute renal failure
during eating, loss of body mass, and occasional D. Acute disturbance of cerebral circulation
vomiting with undigested food. Esophageal X-ray E. Hyperosmolar coma
shows S-shaped deformation of the esophagus
57. A 72-year-old man diagnosed with ischemic
and its dilation, at the cardiac orifice the
heart disease presents with diffuse
esophagus is constricted; esophageal mucosa is
cardiosclerosis, permanent tachysystolic atrial
smooth, without signs of peristalsis. Make the
fibrillation, heart failure Ha, FC II. Objective
provisional diagnosis:
examination of vital signs: blood pressure is
A. Esophageal carcinoma 135/80 mm Hg, heart rate is 160/min., pulse is
B. Diaphragmatic hernia 125/min. Left ventricular ejection fraction is
32%. What drug is indicated in this case and A. Total proteim content in the pleural fluid below
should be presribed to the patient? 25 g/L
B. Presence of atypical cells
A. Digoxin
C. Total protein content in the pleural fluid
B. Procainamide (Novocainamide)
exceeding 30 g/L D. Specific gravity exceeding
C. Isadrine (Isoprenaline) 1015
D. Verapamil E. Positive Rivalta’s test
E. Ivabradine
61. A 33-year-old man developed multiple rashes
58. A 34-year-old man complains of pale edema on the skin of his torso and extensor surfaces of
of the face, feet, shins, and lumbar area, elevated his upper and lower limbs. The rashes itch and
blood pressure up to 160/100 mm Hg, and general occasionally fuse together and form plaques. The
weakness. He has a clinical history of nonspecific elements of rash are covered with silver-white
ulcerative colitis. Objectively: pulse - 84/min., fine scales that easily flake off when scratched.
rhythmic, blood pressure - 165/100 mm Hg; Grattage test results in three sequential
edemas all over the body; the skin is pale and dry, phenomena: stearin spot, terminal film, and
with low turgor. The kidneys cannot be palpated, punctate hemorrhage. What diagnosis can be
on an attempt to palpate them they are painless. suspected?
Blood test: erythrocytes - 3.0-1012/L, Hb- 100
g/L, erythrocyte sedimentation rate - 50 mm/hour. A. Psoriasis
Urinalysis: proteins 3.5 g/L, erythrocytes - 7-10 B. Parapsoriasis
in the vision field, leukocytes - 5-6 in the vision C. Pyoderma
field. Daily protemuria - 6 grams. What analysis D. Lichen ruber planus
should be conducted additionally to verify the E. Secondary papular syphilid
diagnosis?
A. Gingival biopsy for the diagnosis of amyloid 62. A 38-year-old woman after physical
disease overexertion suddenly developed palpitations,
B. Radioisotopic examination of kidneys dyspnea, and a dull pain in the cardiac area. For 10
C. Urinalysis for Bence-Jones protein years she has been registered for regular check-ups
due to rheumatism and mitral valve disease with
D. Renal ultrasound
non-disturbed blood circulation. Oblectively her
E. Survey and excretory urography
pulse is 96/min., of unequal strength. Blood
59. A 42-year-old man, a worker at the meat pressure is 110/70 mm Hg, heart rate 1s 120/min.
processing factory, developed an itching spot on ECG registers small unevenly-sized waves in place
his lower jaw, which gradually transformed into a of P-waves, R-R intervals are of unequal length.
slightly painful carbuncle 3 cm im diameter, What 1s the most likely diagnosis?
surrounded by a painless swelling that reaches the
A. Atnial fibrillation
clavicle. Temperature is subfebrile, under 37.8°C.
B. Paroxysmal supraventricular tachycardia
The doctor suspects anthrax. What drug should
this man be prescribed for treatment? C. Atrial flutter
D. Paroxysmal ventricular tachycardia
A. Penicillin E. Respiratory arrhythmia
B. Levomycetin (Chloramphenicol)
C. Biseptol (Co-trimoxazole) 63. An 18-year-old patient always obeys others
and adapts his needs to the demands of the people
D. Interferon alpha
on whom he depends. He excessively defers to
E. Azidothymidin (Zidovudine)
their wishes and makes them responsible for his
60. A 57-year-old patient complains of dyspnea at wellbeing, cannot defend his interests and needs
rest. The patient presents with orthopnea, support from other people. Such psychic profile
acrocyanosis, bulging cervical veins. On has been formed in the childhood, remains
percussion: dull sound over the lower lung unchanged, and hinders adaptation. What psychic
segments. On auscultation: no respiratory sounds. disorder is observed in this patient?
Heart rate is 92/min. Right-sided cardiac
A. Dependent personality disorder
dilatation is observed. The liver is +7 cm. Shins
B. Anxiety (avoidant) personality disorder
are swollen. Pleural effusion is suspected. What
indicator would confirm the presence of C. Anankastic personality disorder
transudate in this case? D. Markedly accentuated personality
E. Psychopathy-like state
64. A 45-year-old man with thrombophlebitis of 68. A 39-year-old man suffers from chronic
the deep veins in his legs suddenly after physical adrenal insufficiency and receives replacement
exertion developed sharp pain in his thorax on the glucocorticoid therapy
right, dyspnea, and hemoptysis. Objectively his (hydrocortisone - 15 mg/day). He is to undergo
condition is severe; he presents with elective surgery for calculous cholecystitis. What
acrocyanosis, shortening of pulmonary medication adjustment should be made on the day
percussion sound on the right, and weakened of the surgery to prevent the development of acute
respiration. Respiration is 30/min., blood pressure adrenal insufficiency?
is 110/80 mm Hg. ECG shows sinus tachycardia,
heart rate is 120/muin., electrical axis of the heart . Increase the dosage by 2-3 times

SOAR >
deviates to the right, S;Qy;. What is the most . Cancel the drug for the day of the surgery
likely diagnosis? . Add a mineralocorticoid
. Add an antibiotic
A. Pulmonary embolism B. Community- . Prescribe a large volume intravenous fluid
acquired right-sided pneumonia C. Cancer
infusion
of the right lung
D. Right-sided exudative pleurisy 69. After a long drive with the window open a
E. Spontaneous pneumothorax man developed facial asymmetry; he cannot close
his right eye, his right nasolabial fold is smoothed
65. A 48-year-old woman has been hospitalized
out, movements of expression are absent on the
due to development of tachysystolic atrial
right, there is a disturbance of gustatory sensation
fibrillation. She has lost
in the tongue on the right. No other neurological
5 kg of body weight within 2 months. On
pathologies were detected. What disease can be
palpation there is a node in the left lobe of the
provisionally diagnosed in this patient?
thyroid gland. What pathology resulted in the
development of this condition? A. Neuropathy of the facial nerve
B. Neuropathy of the trigeminal nerve
A. Toxic nodular goiter
C. Trigeminal ganglionitis
B. Aterosclerotic cardiosclerosis
D. Neuropathy of the oculomotor nerve
C. Chronic thyroiditis
E. Ischemic stroke
D. Nontoxic nodular goiter
E. Autoimmune thyroiditis 70. A 56-year-old woman was diagnosed with
stage 2 hypertension of the 2nd degree. She
66. A 48-year-old woman developed insomnia,
belongs to the group of moderate risk and has
depressive mood, anxiety, fears and suicidal
bronchial asthma. What group of drugs is
thoughts after the death of her husband that
CONTRAINDICATED to this patient?
occurred one month ago. During her stay in the
A. f-blockers
hospital she speaks in a low voice, is depressed,
B. Angiotensin-converting enzyme inhibitors
anxious, avoids sleeping, refuses to eat. What
medications should be prescribed in this case? C. Diuretics
D. Calcium antagonists
A. Antidepressants E. Imidazoline receptor antagonists
B. Antipsychotics
71. A 45-year-old woman 1s registered for regular
C. Group B vitamins
check-ups due to Werlhof disease (immune
D. Nootropics
thrombocytopenia). Complete blood count: Hb-
E. Anticonvulsants
100 g/L, erythrocytes 2.8-1012/L, platelets -
67. A 32-year-old woman complains of episodes 90.0-10°/L, leukocytes - 8.4 - 109/L, erythrocyte
of intense fear that occur without visible cause sedimentation rate 13 mm/hour. Examination
and last for 10-20 minutes, the episodes are detects a single small hematoma on the anterior
characterized by rapid pulse, sweating, labored surface of the thigh, developed after the patient
breathing, and vertigo. Specify the likely accidentally stumbled on a table. What treatment
diagnosis: tactics should be chosen in this case?

A. Panic disorder A. Continue the supervision by the hospital


B. Paranoid syndrome hematologist
C. Manic syndrome B. Urgent hospitalization imto the
D. Simple schizophrenia hematology unit
E. Claustrophobia
C. Urgently start a hemostatic therapy, 75. A 27-year-old man complains of pain in his leg
followed by a planned hospitalization into the joints, purulent discharge from the eyes, and
hematology unit painful burning sensations during urination.
D. Urgent hospitalization into the general Disease onset was acute. He has a history of
care unit influenza. The patient smokes and drinks alcohol
E. Administer thrombocytic mass, continue in excess. In his line of work he is often away on
the treatment in the hematology unit business trips. What is the most likely etiological
factor of this disease?
72. The dermatologist has an appointment with a
30-year-old man that complains of severely A. Chlamydia
itching rashes that especially disturb him at night. B. Adenovirus
The rashes developed 2 weeks ago, after he had C. Streptococci D. Staphylococci
returned from a travel. Objectively on the lateral E. Candida
surfaces of his fingers, hands, wrists, elbows,
lower abdomen, genitals, and thighs there are 76. A 46-year-old woman has diarrhea with
paired papulovesicles, single pustules, and abdominal distension, loss of body mass, and large
scratch marks. What disease can be suspected? amounts of porridge-like foulsmelling stool
without blood streaks or tenesmus. Objective
A. Scabies examination detects moderate tenderness in the
B. Pyoderma mesogastrium and left abdominal flank. Feces
C. Dermatitis analysis detects steatorrhea with neutral fat and
D. Eczema creatorrhea. What prescription would be the most
E. Shingles advisable in this case?

73. A 38-year-old woman developed a medical . Multi-enzyme preparations


. Cholinergic antagonists
Hoan

condition 7 days after her return from


Bangladesh. Periodical elevation of temperature . Metronidazole and loperamide
was accompanied by chills and excessive . Antacids and antispasmodics
sweating. She was diagnosed with tropical . Cholinergic antagonists and antibacterial
malaria. Next day her condition further agents
deteriorated: body temperature - 38°C, inertness,
periodical loss of consciousness, generalized 77. A man was brought into the admission room
seizures, tachycardia, hypotension, and icteric after an overexposure to cold. He complains of
skin. What complication can be suspected in this sharp pain in the small of his back and elevated
case? body temperature up to 38°C. He took some
A. Cerebral coma aspirin. Blood test: leukocytes - 10.5 - 1012/L,
B. Serous meningitis eosinophils - 5%, band neutrophils - 8%,
segmented neutrophils - 51%, lymphocytes -
C. Purulent meningitis
32%, monocytes - 4%, erythrocyte sedimentation
D. Acute hepatic failure
rate - 28 mm/hour. Urinalysis: protein - 0.6 g/L,
E. Acute heart failure
leukocytes - cover the whole vision field, large
74. A 73-year-old woman came to the family amount of mucus. What is the most likely
physician for one of her regular follow-up diagnosis?
examinations. Three months ago she was found to
A. Acute pyelonephritis
have type 2 diabetes mellitus. She was keeping to
B. Chronic pyelonephritis
her diet and exercise plan and _ taking
C. Acute glomerulonephritis
phytopreparations. On examination her fasting
glucose was within the range of 7.8-8.6 mmol/L, D. Tubulointerstitial nephritis
HbAlc - 7.9%. Height - 164 cm, weight - 83 kg. E. Subacute malignant glomerulonephritis
What blood sugar-controlling medicine should she 78. A 26-year-old man complains of chills,
be prescribed first in the course of her rhinitis, dry cough, and fever up to 38°C.
pharmacological therapy? Examination shows him to be in a moderately
A. Metformin severe condition; there are small pale pink non-
B. Glibenclamide
merging spots on the skin of his back, abdomen,
and extremities. Palpation reveals enlarged
C. Glimepiride
occipital and axillary lymph nodes. No
D. Gliclazide
information about vaccination history could be
E. Insulin
obtained. What is the likely etiology of this
disease?

A. Rubella virus
B. Epstein-Barr virus
C. Streptococcus
D. Mumps virus
E. Neisseria meningitis
1. During medical examination a cadet in the fingers appeared. Determine the frostbite degree
naval college was detected to have a painless in this child:
dense ulcer 1.5x0.5 in size in his perianal area at A. Frostbite of the I degree
the 2 o’clock position. The ulcer floor resembles B. Perniosis
“old fat”. What is the provisional diagnosis? C. Frostbite of the IT degree
D. Frostbite of the ITI degree
A. Hard syphilitic chancre of the
E. Frostbite of the IV degree
rectum
B. Rectal fissure C. Rectal fistula 5. A 16-year-old patient has made an
D. Anal cancer appointment with an otolaryngologist. He
E. Anal crypt suppuration complains of elevated body temperature and sore
throat. Disease onset was 2 days ago, after the
2. A 32-year-old woman complains of tumorlike
patient ate two portions of ice-cream.
formation on the anterior surface of her neck that
Pharyngoscopy shows hyperemic mucosa of the
appeared 2 years ago. Within the last 3 months the
palatine tonsils, with purulent exudate in the
tumor has been rapidly growing. It hinders
lacunae. Make the provisional diagnosis:
swallowing and impairs speech; the tumor causes
a sensation of pressure. Objectively the skin A. Lacunar tonsillitis
moisture is normal, pulse is 80/min., rhythmic, B. Follicular tonsillitis
blood pressure is 130/80 mm Hg. In the night lobe C. Diphtheria
of the thyroid gland there is a dense lumpy node D. Acute pharyngitis
3.0x3.5 cm that moves during swallowing. E. Pseudomembranous (Vincent’s) tonsillitis
Scanning image shows a “cold nodule” in the
6. A 35-year-old woman complains of high body
thyroid gland. Make the provisional diagnosis:
temperature and pain in the upper outer quadrant
A. Thyroid cancer of her right buttock, which developed after an
B. Thyroid adenoma injection. She has been presenting with this
C. Thyroid cyst condition for 3 days. At the site of injection the
D. Nodular goiter skin is hyperemic; there is a painful infiltrate
E. Autoimmune thyroiditis with an area of softening m its center. The
woman is diagnosed with a postinjection abscess
3. After a surgery for a left thigh phlegmon the
of the right buttock. What tactics should the
disease progression was complicated by sepsis.
surgeon choose in this case?
On the 7th day after the surgery there are marked
signs of a generalized inflammatory reaction, in A. Abscess incision, sanation and drainage of
blood there are signs of toxic anemia and the cavity
progressing hypoproteinemia, bilirubin levels are B. Hospitalization, prescription of antibiotics,
40 memol/L, AST and ALT exceed the norm by UHF
2.5 times. Oliguria persists (700 mL of urine per C. Abscess puncture, pus removal followed by
day). Name the phase of sepsis progression: application of antiseptics
D. 10-15 minutes of low-intensity
A. Catabolic phase
laser radiation directed at the right
B. Stress phase
buttock
C. Anabolic phase D. Recovery phase
E. Antipyretic agents, massage,
E. Mixed phase
and
4. A 10-year-old boy, who was outdoors in windy application of dry heat to the right buttock
and cold weather, developed moderate pain and
7. A 65-year-old woman on abdominal palpation
tingling in his fingers and toes. When he returned
presents with a tumor in the umbilical region and
home, his parents noticed that the tips of his
above it; the tumor is 13x8 cm in size,
fingers and toes were white and their sensitivity
moderately painful, nonmobile, pulsing. On
was lost. As the affected areas were warming up,
auscultation systolic murmur can be observed.
the fingers and toes developed tingling and painful
What is the most likely diagnosis?
sensations. Skin pallor changed into redness,
tingling stopped, mild itching and swelling of the A. Abdominal aortic aneurysm
B. Gastric tumor
C. Arteriovenous aneurysm A. Acute attack of glaucoma of the left eye
D. Tricuspid insufficiency B. Acute iridocyclitis of the left eye
E. Bicuspid insufficiency C. Stage II intraocular tumor of the left eye
D. Endophthalmitis of the left eye
8. A 32-year-old man complains of pain in his
E. Panophthalmitis of the left eye
legs that intensifies during walking, intermittent
claudication, numbness of his toes, extremity 11. On the 15th day after a small trauma of the
coldness, and inability to walk more that 100 right foot, the patient developed indisposition,
fatigability, irritability, headache,
meters. When he sleeps, his leg usually hangs
elevated body temperature, and sensation
down. The patient has been smoking since he of constriction, tension, and
was 16. He drinks alcohol in excess. The left leg twitching in the muscles of the right shin. What
is colder than the right one; the skin of the disease can be suspected?
extremities is dry. No pulse can be detected on
A. Tetanus
the pedal arteries, while pulsation of the femoral
B. Anaerobic gas gangrene C.
arteries is retammed. What is the most likely
Erysipelas
diagnosis?
D. Acute thrombophlebitis
A. Obliterating endarteritis E. Thrombophlebitis of the popliteal artery
B. Diabetic angiopathy
12. A patient has the second and third degree burns
C. Leriche syndrome (aortoiliac occlusive
of the 15% of the body surface. On the 20th day
disease)
after the trauma the patient presents with sharp
D. Raynaud disease
increase of body temperature, general weakness,
E. Deep thrombophlebitis
rapid vesicular respiration, facial features are
9. A 50-year-old patient was brought to a hospital sharpened, BP is 90/50 mm Hg, heart rate is
with complaints of blood in urme. Urination is 112/min. What complication is 1t?
painless and undisturbed. Macrohematuria had
A. Sepsis
been observed for 3 days. Objectively: kidneys
B. Pneumonia
cannot be palpated, suprapubic area is without
C. Acute intoxication
alterations, external genitalia are nonpathologic.
D. Purulent bronchitis
On rectal investigation: prostate is not enlarged,
E. Anaerobic infection
painless, has normal structure. Cystoscopy
revealed no changes. What is the most likely 13. 2 hours after a traffic accident a 28-yearold
diagnosis? man in a grave condition was brought to a
hospital. The patient complains of abdominal
A. Renal carcinoma
pain. He received a blow to the abdomen with the
B. Bladder tuberculosis
steering wheel. Objective examination revealed
C. Varicocele
the following: the abdomen does not participate
D. Dystopic kidney
in respiration, is tense and acutely painful on
E. Necrotic papillitis
palpation; the abdominal muscles are defensively
10. A 59-year-old man complains of pain in his tense, peritoneal irritation signs are positive,
left eye and left side of his head, significant hepatic dullness is absent. BP is 90/60 mm Hg,
vision impairment of the left eye, nausea, and heart rate is 120/min. What further treatment
vomiting. Visual acuity of the right eye is 1.0. tactics should be chosen?
Visual acuity of the left eye is 0.03, attempts at
A. Laparotomy
correction bring no improvement. Right eye
B. Laparoscopy
intraocular pressure - 21 mm Hg, left eye
C. Cold to the abdomen
intraocular pressure 65 mm Hg. Congestive
D. Ultrasound investigation
injection is observed on the sclera of the left eye.
E. Laparocentesis
The cornea is thick and swollen. The anterior
chamber is shallow, moist, and clear. The pupil 14. A 48-year-old woman has arrived to the
is dilated and unresponsive to the light, the surgical unit with wounds in her thigh. On
fundus of the eye is not visible. What is the most examination the wound surface has dirty-gray
likely diagnosis? coating with unpleasant sweet smell. Wound
content resembles raspberry jelly. Skin tissues
around the wound are glossy and_ turgid. A. Right-sided reducible inguinal hernia
Palpation reveals moderate crepitation in the B. Right-sided reducible femoral hernia
tissues. What microflora is the most likely to C. Cyst of the right spermatic cord
cause such inflammation? D. Right-sided inguinal lymphadenitis
E. Right-sided reducible arcuate line hernia
A. Anaerobic clostridial
B. Anaerobic non-clostridial 18. A 78-year-old man with a prostate adenoma
C. Streptococci underwent a herniotomy for a direct inguinal
D. Staphylococei hernia. After the surgery he presents with absent
E. Blue pus bacillus urination. Enlarged urmary bladder is detectable
above the patient’s pubis. What measures should
15. After a pain attack in the right subcostal area,
be taken in this case?
a 58-year-old woman with overnutrition
developed icteric skin and sclera, lightcolored A. Bladder catheterization
feces, and dark urine. Her abdomen is distended B. Apply cold to the urinary bladder area C.
and painful on palpation in the right subcostal Prescribe processing of the postoperative
area. Palpation detects liver enlargement by 2-3 wound with UHF field
cm. Blood test: total bilirubin - 90 memol/L, D. Prescribe —_ proserin (neostigmine)
conjugated bilirubin - 60 mcmol/L. What method intramuscularly
of examination will be the most informative for E. Prescribe antispasmodics subcutaneously
diagnosis clarification?
19. A 38-year-old patient has been brought by an
A. Retrograde cholangiopancreatography
ambulance to the surgical department with
B. Intravenous cholegraphy
complaints of general weakness, indisposition,
C. Infusion cholegraphy
black stool. On examination the patient is pale,
D. Percutaneous transhepatic cholegraphy
there are dotted hemorrhages on the skin of his
E. US of the hepatopancreatobiliary zone
torso and extremities. On digital investigation
16. An 11-year-old boy for a month has been there are black feces on the glove. Blood test:
presenting with increasing pain in the right Hb- 108 g/L, thrombocytopenia. Anamnesis
femur. In the painful area there is a nonmobile states that a similar condition was observed 1
painful tumor with unclear margins. The child year ago. Make the diagnosis:
complains of general indisposition, weakness,
A. Thrombocytopenic purpura
increased body temperature up to 39°C. X-ray
B. Hemophilia
shows widened medullary cavity, small foci of
C. Ulcerative bleeding
cancellous bone destruction, and onion-like
D. Rectal tumor
lamellar exfoliation of the cortical layer. What is
E. Nonspecific ulcerative colitis
the most likely pathology resultmg in such
clinical presentation? 20. A 30-year-old man came to the family
physician. 2 months ago he underwent a surgery
A. Ewing sarcoma
for open fracture of the humerus. On examination
B. Osteogenic sarcoma
the patient’s condition is satisfactory; in the area
C. Fibrosarcoma
of the postoperative wound there is a fistula that
D. Chondrosarcoma
discharges a small amount of pus; the area itself
E. Juxtacortical sarcoma
is red; fluctuation is detected. X-ray shows
17. A 43-year-old man complains of a protrusion destruction of the humerus with sequestra. What
in the right inguinal region, that enlarges due to complication did the patient develop during the
strain. He has been presenting with this condition postoperative period?
for 6 months. Within this period the protrusion
A. Posttraumatic osteomyelitis
has grown. Objectively in the might inguinal
B. Hematogenous osteomyelitis
region an elastic protrusion 8x5 cm is visible. On
C. Wound suppuration
palpation it disappears, leaving an empty space
D. Posttraumatic phlegmon
4x4 cm between the pedicles of the Poupart
E. Suture sinus
ligament. *Cough push” sign 1s positive over this
opening. Make the diagnosis: 21. 3 hours after a trauma, a young man
developed bradycardia of 46/min., anisocoria
D>S8, hemi-hyperreflexia S>D, hemihypesthesia position, cyanotic skin covered in cold sweat,
on the left, and a convulsive disorder. The tachycardia, deficient pulse, and low blood
character of this process needs to be clarified. pressure. What urgent treatment tactics should be
What method of examination will be the most chosen?
accurate for this purpose?
A. Tracheostomy
A. Brain CT B. Oral administration of hyposensitization
B. Skull X-ray substances and broncholytics
C. Electroencephalography C. Intravenous admuistration of dehydrating
D. Echoencephalography agents
E. Lumbar puncture D. Administration of glucocorticoid hormones
E. Oxygen therapy
22. The body of a 24-year-old woman with
suspected poisoning has been found on the street. 26. Heart X-ray of a 31-year-old man has
Forensic medical examination was requested by revealed the following: with tightly filled
an investigator during examination of the site and opacified esophagus there is a marginal filling
the body. According to the Criminal Procedure defect in its middle third on the posterior wall;
Code currently in force in Ukraine, forensic the defect is 1.8x1.3 cm in size with clear oval
medical examination is required when it is border. Mucosal folds are retained and envelop
necessary to determine the: the defect; wall peristalsis and elasticity are not
affected. There are no complaints regarding the
A. Cause of death
condition of the patient’s alimentary canal. Make
B. Manner of death
the provisional diagnosis:
C. Time of death
D. Mode of death A. Esophageal tumor
E. Mechanism of death B. Achalasia cardiae
C. Esophageal burns
23. A 37-year-old patient complains of pain in
D. Diverticulum
the spimal column, reduced mobility. The
E. Barrett esophagus
condition persists for 7 years. *Sway back” is
observed, there is no movement in all spinal 27. A 25-year-old man was hospitalized with
regions. X-ray shows “bamboo spine” vertebral complaints of pain in his lower abdomen and
column. What is the most likely diagnosis? right lumbar area that appeared one hour ago.
A. Ankylosing spondyloarthritis Patient’s general state is moderately severe.
B. Osteochondrosis Body temperature - 38.2°C, heart rate - 102/min.
C. Spondylitis deformans The tongue is dry. The abdomen is painful on
D. Tuberculous spondylitis deep palpation in the right ilac area and in the
E. Spondylolisthesis Petit triangle. AureRozanov and Gabay signs are
positive. Make the provisional diagnosis:
24. A surgery unit received a person with an
A. Acute appendicitis
incised stab wound on the upper third of the nght
B. Right-sided renal colic
thigh. Examination detects an incised stab wound
C. Cecal tumor
3.0x0.5x2.0 cm in size on the inner surface of the
D. Intestinal obstruction
upper third of the night thigh. Bright-red blood
E. Acute cholecystitis
flows from deep within the wound in a pulsing
stream. Characterize this type of bleeding: 28. A 45-year-old man diagnosed with acute
pulmonary abscess suddenly developed sharp
A. Arterial
pain in his chest on the right and dyspnea up to
B. Venous
30/min. Examination detects facial cyanosis and
C. Parenchimatous
shallow rapid respirations. Auscultation reveals
D. Capillary
acutely weakened respiration throughout the
E. Mixed
whole right lung; percussion reveals a
25. A 47-year-old man developed the signs of vesiculotympanitic (bandbox) resonance at the
decompensated laryngeal stenosis against the lung apex and dullness in the lower lobe. What
background of acute flegmonous laryngitis. He complication developed in this patient?
presents with inspiratory dyspnea at rest, forced
A. Pyopneumothorax patient’s body there is graybrown area of necrosis
B. Pleuropneumonia that covers 3/4 of the body perimeter.
C. Pneumothorax Occasionally there are small blisters with
D. Acute mediastinitis hemorrhagic contents and patches of shredded
E. Esophageal perforation epidermis. What local therapy is necessary in this
case?
29. A 5-year-old child was brought to the ENT
department by an ambulance. The child presents A. Decompression necrectomy
with cough and difficult respiration. From the B. Chemical necrolysis
patient’s history it is known that the child was C. Blister puncture
playing with a toy construction set, when D. Necrectomy with xenotransplantation
suddenly started coughing and developed E. Necrectomy with dermal autograft
labored breathing. Examination detects
32. A woman in her early- to mid-thirties has lost
periodical cough, labored expiration, and
her consciousness 3-5 minutes ago. On
respiratory lag in the left side of the child’s
examination: the skin is pale, no pulse over the
thorax. Auscultation: diminished respiration on
carotid arteries, no spontaneous respiration, pupils
the left. Percussion: tympanitis. X-ray shows a
are dilated; the patient is nonresponsive, presents
displacement of the mediastinal organs to the
with atony. The patient’s condition can be
right. Make the diagnosis:
determined as:
A. A foreign body in the left bronchus,
A. Clinical death
valvular bronchostenosis
B. Natural death
B. A foreign body in the right bronchus,
C. Syncope
valvular bronchostenosis
D. Brain death
C. A foreign body in the trachea
E. Comatose state
D. A foreign body in the left bronchus, complete
bronchostenosis 33. A boy had a foreign body removed from under
E. A foreign body in the right bronchus, partial his nail plate. 3 days later he developed a sharp
bronchostenosis throbbing pain at the end of his distal phalanx,
which intensifies when the phalanx is pressed,
30. A 30-year-old man was brought to the
hyperemia of the nail fold, elevated body
neurosurgical department with complaints of
temperature up to 38.5°C, and nail plate
constant headaches, nausea, vomiting, fever, and
discoloration. Make the diagnosis:
weakness of the right-side limbs. Anamnesis
states that one month ago the patient had a A. Subungual panaritium
surgery for left-sided suppurative otitis and B. Erysipelas
mastoiditis. He has been undergoing treatment in C. Paronychia
an ENT department. Approximately 2 weeks ago D. Erysipeloid
the temperature increased, and the patient E. Abscess
developed headaches. Objectively: heart rate -
98/min., BP- 140/90 mm Hg, temperature 34. A 32-year-old woman complains of body
38.3°C. Neurologically manifested stiff neck: weight loss despite her imcreased appetite,
bilateral Kernig’s symptom, unsteadiness during nervousness, and tremor of the extremities.
Objectively: the skin is moist; the thyroid gland is
the Romberg’s maneuver. Computer tomography
of the brain revealed a threedimensional growth diffusely enlarged, painless, soft, and mobile.
with a capsule in the left hemisphere. Make the Blood test: increased level of T3, T4, and thyroid-
diagnosis: stmulating hormone (THS). What is the most
likely diagnosis?
A. Cerebral abscess
B. Echinococcus A. Diffuse toxic goiter
C. Hemorrhage B. Thyroid carcinoma
D. Hydrocephalus C. Autoimmune (Hashimoto’s) thyroiditis
E. Amold-Chiari malformation D. Thyroid adenoma
31. The burns unit received a patient, who 6 hours E. Diffuse nontoxic goiter
ago during a fire received flame burns. On the
35. A 19-year-old young man complains of cough A. In medico-prophylactic institutions;
with expectoration of purulent sputum in the general physicians and surgeons
amount of 100 mL per day, hemoptysis, dyspnea, B. At the site of the accident; first-response
increased body temperature up to 37.8°C, general emergency teams
weakness, weight loss. The patient’s condition C. At the site of the accident; specialized
lasts for 4 years. Exacerbations occur 2-3 times a second-response emergency teams D. In medico-
year. The patient presents with malnutrition, pale prophylactic institutions; specialized second-
skin, cyanosis of the lips, drumstick response emergency teams E. In medical
(clubbed) fingers. Tympanic percussion sound in institutions; all listed types of healthcare workers
the lungs, weakened respiration, numerous 39. A 45-year-old man underwent a cardiac
various moist crackles in the lower pulmonary surgery one week ago. His general state has been
segments on the left can be observed. In blood: deteriorating since then: dyspnea at rest,
erythrocytes - 3.2-1012/L, leukocytes - retrosternal pain that irradiates to the neck,
8.4-10°/L, ESR- 56 mm/hour. On X-ray: lung marked weakness. Objectively his body
fields are emphysematous, the left pulmonary temperature is hectic. His cardiac borders are
root is deformed and dilated. What is the most expanded, apical beat is weakened. Auscultation
likely diagnosis? detects pericardial friction rub. What is the most
likely diagnosis?
A. Multiple bronchiectasis of the left lung
B. Chronic left-sided pneumonia A. Acute pericarditis
C. Chronic abscess of the left lung B. Acute cardiac aneurysm
D. Left-sided pulmonary cystic dysplasia C. Myocardial infarction
E. Suppuration of the cyst in the left lung D. Acute myogenic dilatation of the heart
E. Pulmonary embolism
36. A 57-year-old woman during a regular
ultrasound examination presented with a space- 40. A 45-year-old man was brought by an
occupying heterogeneous lesion in the right ambulance into the emergency hospital. He
kidney. What is the most informative method of complains of sudden pai in the lumbar area,
renal tumor diagnostics? frequent painful urination, and vomiting.
Examination detects pain in the lumbar area,
A. Spiral computed tomography
costovertebral angle tenderness, pain on palpation
B. Excretory urography
of kidneys and along the ureter on the right. Urine
C. Retrograde pyelography
test: proteins, fresh erythrocytes, leukocytes.
D. Radioisotope renography
Make the provisional diagnosis:
E. Three glass urine test
A. Urolithiasis, renal colic
37. A 40-year-old victim of a traffic accident
B. Acute pyelonephritis
sustained the following injuries: closed
C. Acute glomerulonephritis
diaphyseal femur fracture, brain concussion,
D. Acute renal failure
multiple mb fractures, hemopneumothorax,
E. Polycystic kidney disease
degloving shin injuries. What injuries require the
most urgent attention?

A. Multiple rib fractures, hemopneumothorax


B. Closed diaphyseal femur fracture
C. Brain concussion
D. Degloving shin injuries
E. All injuries are equivalent

38. At the railroad crossing a passenger train


collided with a bus. In this collision 26 bus
passenges died, another 18 passengers received
mechanical injuries of varying severity. Where
will be professional medical aid provided for the
victims of this accident? Who will provide this
aid?
1. A newborn girl has Apgar score of 7-8 disease is made. What examination would be the
points at the 1-5 minutes after birth. During the most informative for diagnosis confirmation?
labor there was a brief difficulty with extraction A. Doppler echocardiography
of the shoulder girdle. After birth the baby B. Electrocardiography
presents with disturbed function of the proximal C. Chest X-ray
segment and forced position of the right arm. The D. Rheography of the pulmonary artery
shoulder is rotated inwards, the elbow is E. Ultrasound of the liver
extended, the forearm is pronated, and the whole
upper limb resembles an arm of a doll. What is 5. Mother of a 5-year-old child noticed on the the
the most likely clinical diagnosis in this case? head of her child a round “bald” spot 3 cm in
diameter. All the hairs in the focus are broken off
A. Erb-Duchenne palsy at the length of 5-6 mm. The day before the child
B. Thoracic spine trauma was petting a stray cat. Make the diagnosis:
C. Osteomyelitis of the nght arm
D. Intracranial hemorrhage A. Microsporia
E. Soft tissue injury of the nght arm B. Superficial trichophytosis
C. Deep trichophytosis
2. Disease onset was acute. A child developed D. Psoriasis
general weakness, pain in the joints, and elevated E. Alopecia areata
temperature. Later these signs became
accompanied by itching skin rash manifested as 6. A 2-year-old child with persisting cough and
erythematous spots 25 mm in size. The rash subfebrile body temperature after a case of URTI
gradually turned hemorrhagic. Large jomts are developed dyspnea, cyanosis of the nasolabial
painful and swollen; pain attacks periodically triangle, percussion dullness and weakened
occur in the paraumbilical area; there are signs of respiration in the lower lobe of the right lung, and
intestinal hemorrhage. What is the most likely a slight mediastinal displacement to the left. What
diagnosis? pulmonary pathology is likely to cause this
clinical presentation?
A. Hemorrhagic vasculitis (Henoch-Schonlein
purpura) B. A. Pleurisy
Scarlet fever B. Emphysema
C. Hemorrhagic meningoencephalitis C. Pneumonia
D. Streptococcal impetigo D. Atelectasis
E. Rheumatism E. Bronchitis

3. A 13-year-old girl for the last two weeks has 7. During examination a 4-month-old child with
been complaining of dyspnea and shin and foot meningococcemia presents with acrocyanosis,
edemas that appear after a physical exertion. In cold extremities, tachypnea, and thready pulse,
the morning the edemas significantly decrease. blood pressure of 30/0 mm Hg, anuria, and sopor.
Clinical examination revealed enlarged liver and What clinical syndrome 1s it?
coarse systolic murmur over the heart area.
. Toxic shock syndrome
Blood test and urinalysis are without changes.
SaoaAn>

. Neurotoxicosis
What is the most likely cause of edemas in this
. Exicosis
child?
. Encephalic syndrome
A. Heart failure . Acute renal failure
B. Nephrotic syndrome
8. At night a 2-year-old child with upper
C. Acute pyelonephritis D. Angioneurotic edema
respiratory tract infection suddenly developed
E. Hepatic cirrhosis
dyspnea with labored inspiration. Objectively the
4. A 7-year-old boy has severe pulmonary skin is pale, perioral cyanosis and_ slight
mucoviscidosis (cystic fibrosis). He complains of acrocyanosis are observed. Breathing is loud,
dyspnea and blood expectoration. Objectively he respiration rate is 32/min. Jugular, supra- and
presents with lagging physical development, infraclavicular fossae retract during breathing.
acrocyanosis, hepatomegaly, drumstick fingers, Respiration is coarse on auscultation. Heart
and nail plates resembling a “clock face”. sounds are clear and sonorous, heart rate is
Provisional diagnosis of chronic pulmonary heart 120/min. What condition was complicated by the
development of the upper respiratory tract What should be excluded from the diet in this
infection? case?

A. Stenosing laryngotracheitis A. Cereals - wheat, oats


B. Airway foreign body B. Milk and dairy products
C. Obstructive bronchitis C. Fruits
D. Bronchiolitis D. Animal protein
E. Bronchial asthma E. Easily digestible carbohydrates
9. A l-year-old child with a case of URTI
13. A 7-year-old boy has been an inpatient for 1.5
suddenly developed noisy respirations with
months. He had been brought to the hospital with
difficult mspiration, intercostal retractions, and
complaints of edemas all over his body, low
barking cough on the 2nd night after the disease
urine output, and headache. Clinical urinalysis:
onset. What is the most likely diagnosis?
proteins 7.1 g/L, leukocytes - 1-2 in the vision
A. Stenosing laryngotracheobronchitis field, erythrocytes - 3-4 m the vision field.
B. Acute pulmonary inflammation During the course of treatment the edemas
C. Bronchial asthma gradually dissipated, headache abated, diuresis
D. Acute bronchitis normalized. Daily urine protems 3 g/L.
E. Acute bronchiolitis Biochemical blood test: total protein 43.2 g/L,
urea - 5.2 mmol/L, cholesterol - 9.2 mmol/L.
10. A 10-year-old boy with symptoms of arthritis What glomerulonephritis syndrome is the most
and myocarditis was brought to a hospital. Based likely to be present in the patient?
on clinical examination the provisional diagnosis
of juvenile rheumatoid arthritis was made. What A. Nephrotic
symptom is the most contributive for the B. Nephritic
diagnostics of this disease? C. Isolated urmary
D. Hematuric
A. Reduced mobility of the joints in the morning E. Mixed
B. Regional hyperemia of the joints
C. Affection of the large joints 14. A 3-month-old child with signs of rickets
D. Enlarged heart presents with positive Chvostek, Trousseau, and
E. Increased heart rate Maslov signs. One day ago the parents witnessed
a cyanotic attack in their child the child broke into
11. A 7-year-old girl has been twice treated with a cold sweat, the eyes bulged, and respiratory
antibacterial agents for urinary tract mfection. arrest occurred. One minute later the child drew in
US shows no severe renal defects. The child a loud breath and the child’s condition normalized
presents with recurrence of leukocyturia and again. What is the cause of the described signs of
bacteriuria, elevated body temperature up to the disease?
38.5°C, and pain in her left lumbar area. What
examination should be conducted first to clarify A. Decrease of blood calcium levels
the cause of urinary infection recurrence? B. Increase of blood calcium levels
C. Decrease of blood phosphorus levels
A. Micturating cystourethrography D. Increase of blood phosphorus levels
B. Excretory urography E. Metabolic acidosis
C. Retrograde pyelography
D. Immunogram 15. A newborn with gestational age of 31 weeks
E. Radioisotope renography presents with hypotonia and _ depressed
consciousness. Hematocrit is 35%, general
12. A child is 1 year old. After solid food was cerebrospinal fluid analysis shows increased
introduced into the diet, within the last several content of erythrocytes and protein, and low
months the child developed loss of appetite, glucose. These data correspond with the clinical
diarrhea with large amount of feces, and presentation of:
occasional vomiting. Body temperature remains
normal. Body weight is 7 kg. The child is very A. Intracranial hemorrhage
pale, has leg edemas and extremely distended B. Meningitis
abdomen. Feces analysis detects high levels of C. Sepsis
fatty acids and soaps. Diagnosis of celiac disease D. Anemia
was made and gluten-free diet was prescribed. E. Intrauterine infection
16. A newborm has Apgar score of 9. When should The cough attacks induce vomiting. X-ray shows
this infant be put to the breast? intensified bronchial pattern. Blood _ test:
leukocytes 16 - 109/L , lymphocytes - 72%,
A. In the delivery room
erythrocyte sedimentation rate - 4 mm/hour.
B. After 12 hours What 1s the most likely diagnosis?
C. After 2 hours
D. On the 2nd day A. Pertussis
E. On the 3rd day B. Obstructive bronchitis
C. Pneumonia
17. A 3-week-old infant developed large, flaccid D. Adenovirus infection
vesicles with purulent contents on the skin of chest
E. Foreign body
and abdomen. The vesicles rupture quickly. Make
the provisional diagnosis: 21. A 3-year-old child presents with dyspnea that
A. Pemphigus neonatorum abates in the sitting position, occasional loss of
B. Vesiculopustulosis consciousness and seizures, delayed physical
C. Toxic erythema development, cyanosis, drumstick fingers.
D. Pemphigus syphiliticus Echocardioscopy detects aortic dextraposition,
E. Pseudofurunculosis ventricular septal defect, pulmonary artery
stenosis, and right ventricular hypertrophy. What
18. 10 hours after birth a child developed is the most likely diagnosis?
jaundice, hypotonia, hyporeflexia, and moderate A. Tetrad of Fallot
hepatosplenomegaly. Feces and urine are of B. Coarctation of the aorta
normal color. Umbilical cord blood bilirubin is C. Transposition of the great vessels
51 memol/L due to unconjugated bilirubin levels. D. Ventricular septal defect
In venous blood: erythrocytes - 3.5-1012/L, Hb- E. Acquired valvular disease
140 g/L, reticulocytes - 1.5%, bilirubin - 111
memol/L, conjugated - 11 memol/L, ALT- 40 22. A 15-year-old girl complains of dizziness and
U/L, AST30 U/L. Mother’s blood group is A(II) sensation of lack of air that she develops in
Rh(- emotionally straining situations. Relief occurs
), child’s blood group is ACI) Rh(+). What after she takes corvalol. Objectively:
laboratory test can confirm the diagnosis? hyperhidrosis and marble-like pattern of the skin
of her palms and feet. Clinical and instrumental
A. Coombs test examination revealed no organic changes in the
B. Viral hepatitis markers analysis central nervous, cardiovascular, and respiratory
C. Measurement of erythrocyte systems. What provisional diagnosis can be
osmotic resistance made?
D. Erythrocytometry
E. Measurement of glucose 6- A. Somatoform autonomic dysfunction
phosphate dehydrogenase levels in B. Obstructive bronchitis
erythrocytes C. Bronchial asthma
D. Stenosing laryngotracheitis
19. A 6-month-old child on breastfeeding is E. Acute epiglottitis
hospitalized in the inpatient department. After
the child recovers, the doctor recommends the 23. A 1.5-month-old child on breasfeeding
mother to start introducing solid food to the presents from birth with daily vomiting, irregular
child’s diet. What products should be introduced liquid foamy feces, and meteorism, which are
to the child’s diet first? resistant to antibacterial and probiotic therapy; no
increase of body mass is observed. The child’s
A. Vegetable puree condition improved, when breastmilk was
B. Fermented dairy products substituted with “NAN low lactose” formula.
C. Grated apple What pathology is it?
D. Semolina porridge
E. Buckwheat porridge A. Lactase deficiency
B. Intestinal lambliasis (Giardiasis)
20. The 5-year-old child has been ill for 2 weeks. C. Infectious enteritis
Cough attacks developed first and were then D. Drug-induced enteritis
followed by reprises. During coughing the E. Functional dyspepsia
child’s face turns red and cervical veins bulge.
24. A 13-year-old girl for a month has been 70%, ESR- 52 mm/hour. Make the provisional
complaining of fatigability, dull pain in her right diagnosis:
subcostal area, abdominal distension, and
constipations. Abdominal palpation reveals . Acute leukemia

maoOOm>
positive Kehr, Murphy, and Ortner signs, while Thrombocytopenic purpura
Desjardins and Mayo-Robson points are painless. . Acute rheumatic fever
Total bilirubin is 14.7 mcemol/L, predominantly . Infectious mononucleosis
indirect, ALT- 20 U/L, AST- 40 U/L, amylase - . Hemorrhagic vasculitis (Henoch-Schonlein
6.3 mmol/L. Echocholecystography shows purpura)
practically no contraction of the gallbladder.
28. During an outdoors school event in hot
Make the provisional diagnosis:
weather, a 10-year-old girl lost her
A. Hypokinetic biliary dyskinesia consciousness. Body temperature - 36.7°C.
B. Hyperkinetic biliary dyskinesia Objectively her skin is pale and cold to touch, her
C. Chronic pancreatitis pupils are dilated. Blood pressure - 90/50 mm Hg.
D. Acute pancreatitis Heart rate - 58/min. What pathology occurred in
E. Chronic hepatitis this case?

25. A 22-day-old infant developed subcutaneous A. Syncope


red nodes from 1.0 to 1.5 cm in size on the scalp; B. Sympathicotonic collapse
later the nodes suppurated. Temperature increased C. Paralytic collapse
up to 37.7°C, intoxication symptoms appeared, D. Sunstroke
regional lymph nodes enlarged. Complete blood E. -
count: anemia, leukocytosis, neutrocytosis,
29. A 13-year-old girl has 30% of excessive body
increased ESR. What diagnosis can be made?
mass, she started to gain weight at the age of 3.
A. Pseudofurunculosis
She has a family history of obesity. Her height and
B. Pemphigus sexual development are normal for her age. The
C. Vesiculopustulosis appetite is excessive. She complains of periodical
D. Scalp phlegmon headaches. Blood pressure - 120/80 mm Hg.
E. - Subcutaneous fat 1s evenly distributed, she has no
26. A 10-year-old boy was brought into the stretch marks. There is juvenile acne on her face.
hospital with complaints of expiratory dyspnea, What type of obesity is it?
respirations are 30/min. He explains his state by A. Alimentary constitutive obesity
a change in the weather conditions. For the last 4 B. Hypothalamic obesity
years the boy has been registered for regular C. Adrenal obesity
check-ups due to his diagnosis of third degree
D. Hypothalamic syndrome of puberty
persistent bronchial asthma. To provide
E. Hypothyroid obesity
emergency aid for this child, first he needs to be
given: 30. An 8-year-old girl complains of frequent
painful urination in small amounts and urinary
A. Salbutamol or short-acting £2-agonists
incontinence. The signs have been present for 2
B. Dexamethasone days already. She explains her disease by
C. Adrenaline overexposure to cold. Costovertebral angle
D. Euphylline (Ammophylline) tenderness is absent. Complete blood count is
E. Claritin (Loratadine) without pathologies. Urine test: leukocytes - 20-
27. A 3-year-old child has been brought to a 30 in the vision field, erythrocytes - 40-50 in the
hospital with complaints of pain in the legs, vision field, unchanged, bacteriuria. What is the
fever, and loss of appetite. Objectively: pale skin most likely diagnosis?
and mucosa, hemorrhagic rash. Lymph nodes are A. Cystitis
enlarged, painless, dense and elastic, not matted B. Vulvitis
together. Bones, joints, and abdomen are painful. C. Pyelonephritis
The liver and spleen are enlarged. Hemogram: D. Glomerulonephritis
Hb- 88 g/L, color index - 1.3, platelets - 80 -
E. Urolithiasis
109/L, leukocytes - 25.8-10°/L, lymphoblasts -
1. A pregnant woman is 28 years old. E. Hormone-producing ovarian tumor
Anamnesis: accelerated labor complicated by the
4. A 48-year-old woman complains of disturbed
II degree cervical rupture. The following two
menstrual cycle: her periods last for 7-9 days and
pregnancies resulted in spontaneous abortions at
are excessively profuse throughout the last half-
the terms of 12 and 14 weeks. On mirror
year. She notes occasional hot flashes in her head,
examination: the uterine cervix is scarred from
insomnia, irritability, and headaches. Her skin is
previous ruptures at 9 and 3 hours, the cervical
of normal color. Blood pressure - 150/90 mm Hg,
canal is gaping. On vaginal examination: the
pulse - 90/min., rhythmic. The abdomen is soft
cervix is 2 cm long, the external orifice is open 1
and painless. Bimanual examination shows no
cm wide, the internal orifice is half-open; the
uterine enlargement, the appendages cannot be
uterus is enlarged to the 12th week of pregnancy,
detected. The vaginal fornices are free. What is
soft, mobile, painless, the appendages are without
the most likely diagnosis?
changes. What diagnosis can be made?
Climacteric syndrome
A. Isthmico-cervical insufficiency, habitual

Sop>
Premenstrual syndrome
noncarrying of pregnancy B. Threatened
Adrenogenital syndrome
spontaneous abortion C. Incipient abortion,
Stem-Leventhal syndrome
habitual noncarrying of pregnancy
(polycystic
D. Cervical hysteromyoma, habitual
ovary syndrome) E.
noncarrying of pregnancy E.
Uterine myoma
Cervical pregnancy, 12 weeks
5. A 30-year-old multigravida has been in labour
2. On the day 4 after the cesarean section a woman
for 18 hours. 2 hours ago the pushing stage began.
developed fever with body temperature up to 39°C
Fetal heart rate is clear, rhythmic, 136/min.
and abdominal pain. Pulse - 104/min. She
Vaginal examination reveals complete cervical
vomited twice. The patient is sluggish, her tongue
dilatation, the fetal head in the pelvic outlet plane.
is dry and has gray coating. The abdomen is
Sagittal suture is in line with obstetric conjugate,
distended. Signs of peritoneal irritation are
the occipital fontanel is near the pubis. The
positive in all segments. Peristalsis cannot be
patient has been diagnosed with primary uterine
auscultated. No passage of gas occurs. Uterine
mertia. What is the further tactics of labor
fundus is located at the level of the navel. The
management?
uterus is pamful on palpation. The discharge is
moderate and contains blood and pus. What is the A. Outlet forceps
most likely diagnosis? B. Labour stimulation
C. Cesarean section
A. Diffuse peritonitis
D. Skin-head Ivanov’s forceps
B. Metroendometritis
E. Vacuum extraction of the fetus
C. Progressive thrombophlebitis
D. Pelvic peritonitis 6. A woman is 40. weeks pregnant.
E. Parametritis The fetus is in the longitudinal lie and cephalic
presentation. Pelvic size: 26-29-3120. Expected
3. A 58-year-old woman came to the
weight of the fetus is 4800 gram. The labor
gynecological clinic. She complains of bloody
contractions has been lasting for 12 hours, within
discharge from her genital tracts. Menopause is 8
the last 2 hours they were extremely painful, the
years. Gynecological examination: the uterus is
parturient woman is anxious. The waters broke 4
slightly enlarged, dense to touch, with limited
hours ago. On external examination the
mobility, the uterine appendages cannot be
contraction ring is located 2 finger widths above
detected, parametrium is free. Fractional
the navel, Henkel-Vasten sign is positive. Fetal
curettage of the uterine cavity yields a significant
heart rate is 160/min., muffled. On internal
amount of medullary substance in the scrape.
examination the uterine cervix is fully open, the
What 1s the most likely diagnosis?
head is engaged and pressed to the entrance into
A. Uterine corpus cancer the lesser pelvis. What is the most likely
B. Adenomyosis diagnosis?
C. Chorioepithelioma
A. Threatened uterine rupture
D. Uterine cervix cancer
B. Complete uterine rupture A. Dysfunctional uterine bleeding
C. Hyperactive uterine contractions B. Adenomyosis
D. Abruption of the normally positioned C. Ectopic pregnancy D. Submucous uterine
placenta myoma
E. Anatomically contracted pelvis E. Cancer of the uterine body

7. A 23-year-old woman came to the 11. A 25-year-old woman was brought into the
gynecological clinic. She complains of pain, gynecological department with profuse bloody
itching, and burning in her vulva, general discharge from her genital tracts. She is 12 weeks
weakness, indisposition, elevated body pregnant, the pregnancy is planned. Within the
temperature up to 37.2°C, and headache. On last 3 days she was experiencing pains in her
examination in the vulva there are multiple lower abdomen that eventually started resembling
vesicles up to 2-3 mm in diameter with clear cramps, she developed bleeding. Her skin is pale,
contents against the background of hyperemia pulse - 88/min., blood pressure - 100/60 mm Hg,
and mucosal edema. Make the provisional body temperature - 36.8°C. Vaginal examination:
diagnosis: the uterus size corresponds with 11 weeks of
pregnancy, the cervical canal allows inserting 1
A. Genital herpes infection
finger and contains fragments of the fertilized
B. Primary syphilis
ovum, the discharge is bloody and profuse. What
C. Papillomavirus infection
is the most likely diagnosis?
D. Vulvar cancer
E. Cytomegalovirus infection A. 12-week pregnancy, spontaneous
abortion in progress
8. A woman with the pregnancy term of 8 weeks
B. 12-week pregnancy,
complains of elevated temperature up to 37.6°C,
threatened
skin rash that can be characterized as macular
spontaneous abortion
exanthema, enlargement of posterior cervical and
C. Disturbed menstrual cycle,
occipital lymph nodes, small amount of bloody
hyperpolymenorrhea D. Disturbed menstrual
discharge from the genital tracts. She was
cycle, amenorrhea
examined by the infectious diseases specialist and
E. Full-term pregnancy, term labor
diagnosed with rubella. What tactics should the
obstetrician-gynecologist choose? 12. A 30-year-old woman came to the
gynecological department. She complains of
A. Abortion
sharp pain in her lower abdomen and temperature
B. Prescription of antibacterial therapy
of 38.8°C. She has a history of extramarital sexual
C. Prescription of antiviral therapy
D. Treatment of incipient abortion activity and 2 = artificial abortions. On
gynecological examination the uterus is
E. Prescription of hemostatic therapy
unchanged. The appendages are bilaterally
9. A 16-year-old girl has primary amenorrhea, no enlarged and painful. Profuse purulent discharge
pubic hair growth, normally developed mammary is being produced from the vagina. What
glands; her genotype is 46 XY; uterus and vagina examination needs to be conducted to clarify the
are absent. What is your diagnosis? diagnosis?
A. Testicular feminization syndrome A. Bacteriological and bacterioscopic analysis
B. Mayer-Rokitansky-Kuster-Hauser syndrome B. Hysteroscopy
C. Cushing syndrome C. Curettage of the uterine cavity
D. Sheehan syndrome
D. Colposcopy
E. Cushing disease
E. Laparoscopy
10. A 46-year-old woman came to the maternity
13. It is the 3rd day after the first normal term
clinic with complaints of moderate blood
labor; the infant 1s rooming-in with the mother
discharge from the vagina, which developed after and is on breastfeeding. Objectively: the mother’s
the menstruation delay of 1.5 months. On vaginal
general condition is satisfactory. Temperature is
examination: the cervix is clean; the uterus is not
36.4°C, heart rate is 80/min.,
enlarged, mobile, painless; appendages without
BP is 120/80 mm Hg. Mammary glands are soft
changes. Make the diagnosis:
and painless; lactation is moderate, unrestricted
milk flow. The uterus is dense, the uterine fundus A. Sheehan syndrome (postpartum pituitary gland
is located 3 finger widths below the navel. Lochia necrosis) B. Physiological amenorrhea
are sanguino-serous, moderate in volume. Assess C. Suspected progressing ectopic pregnancy D.
the dynamics of uterine involution: Stein-Leventhal syndrome (polycystic ovary
syndrome) E. Galactorrhea-amenorrhea syndrome
A. Physiological involution
B. Subinvolution 17. A 45-year-old woman came
C. Lochiometra to the maternity clinic with complaints of
periodical pains in her mammary glands that start
D. Pathologic involution
1 day before menstruation and stop after the
E. Hematometra menstruation begins. Palpation of the mammary
glands detects diffuse nodes predominantly in
14. A 28-year-old woman complaining of
the upper outer quadrants. What is the most likely
irregular menstruations and infertility came to the diagnosis?
gynecological clinic. Menstruations occur since
the age of 15, irregular, with delays up to 2 A. Fibrocystic mastopathy
months. On examination she presents with B. Breast cancer
marked hirsutism and excessive body weight. On C. Mastitis
vaginal examination the uterus is reduced in size D. Hyperprolactinemia
and painless. The ovaries on the both sides are E. Breast cyst
dense and enlarged. Ultasound shows microcystic
18. A 14-year-old girl came to the general
changes in the ovaries, the ovaries are 5x4 cm and
practitioner with complaints of weakness, loss of
4.5x4 cm in size with dense ovarian capsule.
appetite, headache, rapid fatigability. Her last
Basal body temperature is monophasic. What is
menstruation was profuse and lasted for 14 days
the most likely diagnosis?
after the previous delay of 2 months. Objectively:
A. Polycystic ovary syndrome the skin is pale, heart rate is 90/min., BP is 110/70
B. Krukenberg tumor mm Hg, Hb is 88 g/L. Rectal examination: the
C. Endometrioid cysts
uterus and its appendages are without changes, no
D. Bilateral adnexitis
discharge from the genital tracts. What
E. Ovarian carcinoma
complication occurred in the patient?
15. An 18-year-old girl was brought into the A. Posthemorrhagic anemia
gynecology deparment with complaints of B. Somatoform autonomic dysfunction of
elevated body temperature up to 37.8°C, sharp hypotonic type C. Migraine
pain in her lower abdomen, more intense on the D. Gastritis
right, and difficult defecation. Vaginal E. Dysmenorrhea
examination detected a painful dense elastic
formation 5x6 cm in the area of her night ovary. 19. A 22-year-old postparturient woman on the
Pregnancy test is negative. What is the most 12th day after the normal childbirth informs of
likely diagnosis? elevated body temperature up to 39°C for the last
3 days and pain in her right mammary gland. The
A. Torsion of ovarian tumor pedicle right mammary gland is enlarged, hot to touch,
B. Ectopic pregnancy tense, hyperemic, and painful. Palpation reveals
C. Appendicitis there a dense infiltration 8x8 cm with a
D. Ovarian cyst rupture fluctuation in its center. What is the most likely
E. Ovarian apoplexy diagnosis?
16. A 26-year-old woman presents with A. Postpartum period, day 12. Right-sided
amenorrhea. 10 months ago she gave birth for a infiltrative-purulent mastitis
second time. In her early postpartum period she B. Postpartum period, day 12. Right-sided serous
developed a massive hypotonic hemorrhage. No mastitis
breasfeeding. Lately she has been presenting with C. Postpartum period, day 12. Right-sided
loss of weight, loss of hair, and indisposition. gangrenous mastitis
Gynecological examination revealed atrophy of D. Postpartum period, day 12. Right-sided
the external genitals, the uterus is abnormally phlegmonous mastitis
small, no uterine appendages can be detected. E. Postpartum period, day 12. Right-sided
What 1s the most likely diagnosis? lactostasis
20. A 35-year-old pregnant woman with degree 1 sounds detects bradycardia. Evaluation of
essential hypertension, developed edemas and cardiotocogram yielded the following data:
headache at the 33 week of her pregnancy. decrease of basal heart rate down to 90/min.,
Objectively her general condition is satisfactory, variability - monotonous (2 and less); late
blood pressure - 160/100 mm Hg, normal uterine decelerations with amplitude of 50/min. Make the
tone. Fetal heart rate 1s 140/min., rhythmic. She diagnosis and choose the obstetrical tactics
was diagnosed with daily proteinuria - 4 g/L, necessary in this case:
daily diuresis - 1100 mL. Creatinine - 80
A. Fetal distress. Urgent cesarean section
memol/L, urea - 7 mmol/L, platelets - 100 -
delivery
109/L. What complication of pregnancy
B. Fetal distress. Vacuum extraction delivery
occurred?
C. Normal condition of the fetus. Vaginal
A. Moderate preeclampsia birth
B. Severe preeclampsia D. Fetaldistress.Stimulation of uterine
C. Mild preeclampsia contractions
D. Hypertensive crisis E. Fetal distress. Forceps delivery
E. Renal failure 24. A 27-year-old woman complains of foul-
smelling discharge from her genital tracts, pain in
21. A 24-year-old pregnant woman on her 37th
her lower abdomen, and elevated temperature.
week of pregnancy has been brought to the
The complaints arose 2 days ago. She has a
maternity obstetric service with complaints of
history of surgical abortion at the term of 8 weeks
weak fetal movements. Fetal heartbeats are
one week ago. Mirror examination: the uterine
95/min. On vaginal examination the uterine
cervix is clear, external orifice produces
cervix 1s tilted backwards, 2 cm long, external
foulsmelling discharge. Vaginal examination: the
orifice allows inserting a fingertip. Biophysical
uterus lies in anteflexion, is mobile, painful, and
profile of the fetus equals 4 pomts. What tactics
slightly enlarged. The appendages are without
of pregnancy management should be chosen?
changes. Make the provisional diagnosis:
A. Urgent delivery via a cesarean section B.
Treatment of placental dysfunction and repeated A. Postabortal endometritis
analysis of the fetal biophysical profile on the next B. Enterocolitis
day C. Appendicitis
C. Doppler measurement of blood velocity in D. Acute respiratory disease
the umbilical artery E. Salpmgoophoritis
D. Urgent preparation of the uterine cervix
25. A 17-year-old girl has made an appointment
for delivery
with the doctor. She plans to begin her sex life.
E. Treatment of fetal distress; if ineffective,
No signs of gynecological pathology were
then elective cesarean section on the next day
detected. In the family history the patient’s
22. During regular preventive gynecological grandmother had cervical cancer. The patient was
examination a 30-year-old woman was detected consulted about the maintenance of her
to have dark blue punctulated perforations” on reproductive health. What recommendation will
the vaginal portion of the uterme cervix. The be the most helpful for prevention of invasive
doctor suspects endometriosis of the vaginal cervical cancer?
portion of the uterine cervix. What investigation
A. Vaccination against human papillomavirus
method would be most informative for diagnosis
(HPV)
confirmation?
B. Vitamins, calcium, omega-3
A. Colposcopy, target biopsy of the cervix C. Immunomodulators
B. US of the lesser pelvis D. Antiviral and antibacterial drugs E. Timely
C. Hysteroscopy treatment of sexually transmitted
D. Curettage of the uterine cavity diseases
E. Hormone testing

23. A parturient woman is 30 years old, stage I of


the labor is ongoing. The fetus is in the cephalic
presentation. Auscultation of the fetal heart
1. A 38-year-old woman works in flax increased by 10 cm within a year (which is double
processing, she dries flax. She came to the the norm for her age group), the number of
hospital complainng of difficult breathing, permanent teeth corresponds with the age norm
constricting sensation in her chest, and cough (20), the development of her secondary sex
attacks. These signs appear on the first day of her characteristics is three years ahead of her age (Ma,
working week and gradually diminish on the P, Ax, Menarche). Development rate ahead of her
following days. What respiratory disease 1s likely biological age can occur due to:
in this case? A. Endocrine disorders
B. Acceleration
A. Byssinosis
C. Certain components of her diet
B. Silicosis
D. Sports training
C. Allergic rhinopharyngitis
E. Deficient hygienic education
D. Bronchial asthma
E. Asthmatic bronchitis 6. Employees work in conditions of high dust
concentration. Certain chemical (silicon dioxide
2. A district doctor has diagnosed one of his
content) and physical properties of dust aerosols
patients with dysentery. What accounting
contribute to the development of occupational
document reflects this type of morbidity?
dust-induced diseases. What is the main physical
. Urgent report property of dust aerosols?
HOAn >

. Statistical report
A. Dispersion
. Report on a major non-epidemic disease
B. Magnetization
. Certificate of temporary disability
C. Electric charge
. Control card of a patient registered for regular
D. Solubility
check-ups
E. Ionization
3. A 39-year-old man, a battery attendant,
7. In the factory cafeteria
suddenly developed weakness, loss of appetite, there was an outbreak of food poisoning.
nonlocalized colicky abdominal pains, and Clinical presentation —_ indicates
nausea. Objectively his skin is gray; there is a staphylococcal etiology of this disease. 15 people
pink-gray stripe on his gums; the stomach 1s soft are sick. To confirm the diagnosis of food
poisoning, samples need to be sent to the
and sharply painful. Blood test detected
laboratory. What samplesshould be
erythrocytes with basophilic stippling and obtained for analysis?
anemia. The patient has a history of peptic ulcer
disease of the stomach. Constipation tends to A. Vomit masses
occur every 3-4 days. What is the most likely B. Blood for hemoculture
provisional diagnosis? C. Blood (complete blood count)
D. Urine
A. Saturnism (lead poisosning) E. Saliva
B. Acute appendicitis
C. Perforation of gastric ulcer 8. In April during the medical examination of
D. Acute cholecystitis various population groups, 27% of individuals
E. Chronic alcoholism presented with low working ability and rapid
fatigability. The following symptoms were
4. A 9-month-old infant presents with delayed observed in the affected individuals: swollen
tooth eruption and fontanel closure, weakness, friable gingiva that bleeds when pressed,
and excessive sweating. What type of hyperkeratosis follicularis not accompanied by
hypovitaminosis is the most likely in this child? skin dryness. These symptoms most likely result
A. Hypovitaminosis D from the following pathology:
B. Hypovitaminosis C A. C-hypovitaminosis
C. Hypovitaminosis By B. Parodontosis
D. Hypovitaminosis Beg C. A-hypovitaminosis
E. Hypovitaminosis A D. B1-hypovitaminosis
5. A 10-year-old girl exhibits high level of E. Polyhypovitaminosis
physical development ( + 30), her body length
9. Increased general morbidity women living in this locality. What type of
of the local population is observed in the medical examination is it?
area near a factory, where atmosphere is being
intensively polluted with sulfurous gas. What A. Target
effect does polluted air have on human body in B. Preliminary
this case?
C. Regular
A. Chronic nonspecific B. D. Complex
Acute specific E. Screening
C. Acute nonspecific D.
14. In the process of hiring, a prospective
Chronic specific
employee has undergone preventive medical
E. Selective examination and was declared fit to work in this
10. During medical examination of high and manufacturing environment. What type of
middle school students, the doctors vere assessing preventive medical examination was it?
correlation between biological and calendar age
A. Preliminary
of the school students based on the following
B. Scheduled
criteria: height growth rate per year, ossification
C. Periodical
of the carpal bones, the number of permanent
D. Specific
teeth. What additional development criterion
E. Comprehensive
should be assessed at this age?
15. On laboratory investigation of a pork sample
A. Development = of secondary there is 1 dead trichinella detected in 24 sections.
sex
This meat should be:
characteristics
B. Body mass A. Sent for technical disposal
Cc. Chest circumference B. Allowed for sale with no restrictions C.
D. _s*Vital capacity of lungs Processed and sold through public catering
network
E. ‘Hand strength
D. Processed for boiled sausage production E.
11. During analysis of morbidity in the city, it was Frozen until the temperature of -10°C is reached in
determined that age structure of population is the deep layers, with subsequent exposure to cold
different mm each district. What statistical method for 15 days
allows to exclude this factor, so that it would not
16. To assess the effectiveness of medical
skew the morbidity data?
technologies and determine the power and
A. Standardization direction of their effect on the public health
B. Wilcoxon signed-rank test indicators, the research was conducted to study
C. Correlation-regression analysis the immunization rate of children and measles
D. Dynamic time series analysis incidence rate by district. What method of
E. Analysis of average values statistical analysis should be applied in this case?

12. Clinical _ statistical imvestigation was A. Calculation of correlation coefficient B.


performed to determine effectiveness of a new Calculation of morbidity index among the
pharmacological preparation for patients with nonvaccinated C. Calculation of matching factor
ischemic heart disease. What parametric test D. Calculation of standardized ratio E. Calculation
(coefficient) can be used to estimate the reliability of statistical significance of the difference between
of the results? two estimates

A. Student’s t-distribution 17. Having studied the relationship between the


B. Sign test distance from villages to the local outpatient
C. Matching factor clinics and frequency of visits to the clinics
D. Wilcoxon signed-rank test among the rural population of this area, it was
E. Kolmogorov-Smirnov test determined that the rank correlation coefficient in
this case equals -0.9. How can this relationship be
13. In arural health care area there 1s an increasing
characterized?
cervical cancer morbidity observed. The decision
is made to conduct a medical examination of the A. Strong inverse relationship
B. Strong direct relationship 25-year-old woman has been suffermg from
C. Moderate inverse relationship diabetes mellitus since she was 9. She was
D. Moderate direct relationship admitted into the nephrology unit with significant
E. - edemas of the face, arms, and legs. Blood pressure
- 200/110 mm Hg, Hb- 90 g/L, blood creatinine -
18. In the mpatient gynecological unit within a 850 memol/L, urine proteims - 1.0 g/L, leukocytes
year 6500 women underwent treatment. They - 10-15 in the vision field. Glomerular filtration
spent there a total of 102000 bed-days. What rate 10 mL/min. What tactics should the doctor
indicator of the gynecological unit work can be choose?
calculated based on these data?
A. Transfer into the hemodialysis unit B. Active
A. Average length of inpatient stay conservative therapy for diabetic nephropathy C.
B. Average bed occupancy rate per year Dietotherapy
C. Number of beds by hospital department F. Transfer into the endocrinology clinic
D. Bed turnover rate G. Renal transplantation
E. Planned bed occupancy rate per year
2. A 59-year-old woman was brought into the
19. A middle school teacher with 4-yearlong theumatology unit. Extremely severe case of
record of work was issued a medical certificate scleroderma is suspected. Objectively she
for pregnancy and childbirth leave. What amount presents with malnourishment, ’*mask-like” face,
of pay will she receive for the duration of her and acro-osteolysis. Blood: erythrocytes - 2.2 -
10°/L, erythrocyte sedimentation rate 40
leave in this case?
mivhour. Urine: elevated levels of free
A. 100% of average salary oxyproline. Name one of the most likely
B. 50% of average salary pathogenetic links in this case:
C. 70% of average salary
C. Formation of antibodies to collagen
D. 60% of average salary
D. Formation of antibodies to native DNA C.
E. 80% of average salary Formation of antibodies to blood
20. In the air of the feed kitchen at the poultry corpuscles
factory, at the area where formula feed is being F. Formation of antibodies to transversely striated
mixed, the dust concentration reaches 200 mg/m3. muscles
Air microflora is represented predominantly by G. Formation of antibodies to vessel wall
Aspergillus and Mucor fungi. What effect 3. A 34-year-old man on the 3rd day of
determines pathogenic properties of the dust? ceftriaxone treatment for acute otitis (daily
dosage - 2 grams) developed diarrhea occurring
A. Allergenic
5-6 times per day. Feces are without mucus or
B. Teratogenic
blood admixtures. Temperature is 36.6°C.
C. Mutagenic
Gregersen reaction (occult blood in feces) is
D. Fibrogenic negative. Stool culture detected no pathogenic
E. Toxic germs. What is the most likely cause of diarrhea
21. During regular medical examination a lyceum in this case?
student presents with signs of cheilitis that F. Antibiotic-associated diarrhea
manifests as epithelial maceration in the area of lip G. Intestinal dysbiosis
seal. The lips are brightred, with single vertical H. Bacterial overgrowth syndrome
cracks covered with brown-red scabs. These I. Ulcerative colitis
clinical signs are most likely caused by insufficient J. Crohn’s disease (regional enteritis)
content of the following in the diet:
4. A chronic alcoholic was hospitalized into the
A. Riboflavin therapeutic inpatient unit due to pneumonia. On
B. Ascorbic acid the day 5 of his hospitalization he became
C. Retinol disoriented in time and space, developed fear-
D. Thiamine inducing visual hallucinations and motor
E. Calciferol agitation. Full body tremor and tremor of the
limbs are observed. X-ray and _ physical
22. A 30-year-old woman made examinations detect the signs of his
an appointment with the family doctor for 1. A
convalescence from pneumonia. What tactics 8. A 62-year-old patient has been hospitalized
should be chosen regarding this patient? with complaints of pain in the thorax on the right
E. Transfer imto the inpatient during breathing, dyspnea, and dry cough. Ten
narcology department days ago he slipped and fell hitting his right side.
F. Continue the treatment in the therapeutic On examintaion: the patient lies on the left side.
department The right side of the thorax lags during breathing.
G. Transfer into the On the right there are crepitation and pain in the
neuroresuscitation department I-IV ribs. Dullness of percussion sound and
H. Compulsory medical treatment sharply dimimished breath sounds can be
for observed. On X-ray: signs of exudate, fracture of
alcoholism E. Discharge from the the II-IV ribs. On pleurocentesis: blood is
hospital detected. Choose the further tactics:

5. After eating shrimps, a 25-year-old man F. Transfer to a thoracic surgery department


suddenly developed skin itching, some areas of G. Prescribe conservative therapy
his skin became hyperemic or erupted into H. Perform repeated pleural taps
vesicles. Make the diagnosis: I. Apply a fixation bandage to the rib cage
J. Refer to a traumatologist
C. Acute urticaria
D. Hemorrhagic vasculitis (Henoch-Schonlein 9. A 51-year-old man complains of vomiting with
purpura) C. Urticaria blood. He has been drinking alcohol excessively.
pigmentosa Health disorder has been observed since he was 40,
F. Psoriasis when he first developed jaundice. On examination
G. Scabies the skin and visible mucosa are icteric, with a
stellate vascular pattern. The patient is
6. A 25-year-old woman complains of fatigue, malnourished and presents with abdominal
dizziness, hemorrhagic rashes on the skin. She distension, umbilical hernia, and ascites. The edge
has been presenting with these signs for a month. of the liver is tapered and painless, +3 cm, the
Blood test: erythrocytes - 1.0 - 1012/L, Hb- 37 spleen is +2 cm. Blood test: Hb- 80 g/L, leukocytes
g/L, color index - 1.1, leukocytes - - 3-10°/L, platelets - 85-109/L. What is the cause
1.2-109/L, platelets - 42-10°/L. What analysis of portal hypertension in this patient?
would be the most advisable for diagnosismaking
in this case? F. Hepatic cirrhosis
G. Thrombosis of the splenic vein
F. Sternal puncture (bone marrow biopsy) H. Hemochromatosis
G. Splenic biopsy I. Constrictive pericarditis
H. Liver biopsy J. Budd-Chiari syndrome
L Coagulation studies
J. US of the gastrointestinal tract 10. A young man has made an appointment with
the dermatologist. He complams of a painful facial
7. A 35-year-old man complains of rapidly rash in the beard and mustache area. This condition
incresing fatigue, palpitations, ’visual snow”, and has been persisting for several weaks already.
dizziness. He has a history of peptic ulcer of the After shaving, the patient’s condition aggravates.
stomach. Objectively the skin is pale. Vesicular The diagnosis of sycosis is made. What primary
respiration is observed in the lungs. Systolic morphological elements can be observed in the
murmur is detected over the cardiac apex, heart rash in this case?
rate is 100/min., BP is 100/70 mm Hg. The
epigastrium is slightly tender on palpation. Blood F. Pustules, papulae
test: erythrocytes - 3.2 - 1012/L, Hb- 100 g/L, G. Nodes, nodules
color index 0.95. What type of anemia is the most H. Pustules, bumps
likely present in this case? I. Phlyctenae, maculae
J. Maculae, nodes
F. Posthemorrhagic anemia
G. Sideroblastic anemia 11. At night a 63-year-old woman suddenly
EH. Chronic iron-deficiency anemia developed an asphyxia attack. She has a |5year-
L Hemolytic anemia long history of essential hypertension and had a
myocardial infarction 2 years ago. Objectively her
J. Hypoplastic anemia
position in bed is orthopneic, the skin is pale, the
patient is covered with cold sweat, acrocyanosis is
observed. Pulse - 104/min. Blood pressure - 15. A 58-year-old man complains of weakness
210/130 mm Hg, respiration rate - 38/min. and tumor-like formations that appeared on the
Pulmonary percussion sound is clear, with slight anterior surface of his neck and in the inguinal
dullness in the lower segments, throughout the region. Palpation detects soft painless mobile
lungs single dry crackles can be heard that become cervical and inguinal lymph nodes up to 2 cm in
bubbling and non-resonant in the lower segments. diameter. The liver protrudes by 2 cm from the
What is the most likely complication in this edge of the costal margin, the lower splenic pole
patient? is at the umbilical level. In blood: erythrocytes -
F. Acute left ventricular failure 3.5 + 1012/L, Hb- 88 g/L, leukocytes -
G. Paroxysmal tachycardia
86 - 10°/L, band neutrophils - 1%, segmented
H. Bronchial asthma attack
neutrophils - 10%, lymphocytes - 85%,
L Pulmonary embolism
eosinophils - 2%, basocytes - 0%, monocytes -
J. Acute right ventricular failure
2%, erythrocyte sedimentation rate - 15 mm/hour,
12. A 26-year-old woman is suspected to suffer Gumprecht shadows. What is the most likely
from systemic lupus erythematosus due to diagnosis?
systemic lesions of skin, vessels, joints, serous
tunics, and heart that developed after F. Chronic lymphatic leukemia
photosensitization. The following is detected: LE G. Lymphocytic leukemoid reaction
cells, antibodies to native DNA, isolated anti- H. Acute leukemia
centromere antibodies, rheumatoid factor is I. Chronic myeloleukemia
1:100, Wassermann reaction is _ positive, J. Lymphogranulomatosis
circulating immune complex is 120 units. What
16. A 23-year-old man complains of facial edemas,
immunological indicators are considered to be
headache, dizziness, low urinary output, and urine
specific to this disease?
discoloration (dark red). These complaints arose
F. DNA antibodies after a case of acute tonsillitis. On examination
G. Rheumatoid factor there are facial edemas, the skin is pale,
EH. Anti-centromere antibodies temperature is 37.4°C; heart rate is 86/min., blood
L Immunoglobulin A pressure is 170/110 mm Hg. Heart sounds are
J. Increased circulating immune complex muffled, the II heart sound is accentuated over the
aorta. What etiological factor 1s the most likely in
13. A woman came to the doctor with complaints this case?
of mcreased body temperature up to 37.8°C and
moderately sore throat for the last 3 days. F. Beta-hemolytic streptococcus
Objectively: mandibular lymph nodes are G. Staphylococcus aureus
enlarged up to 3 cm. Palatine tonsils are H. Streptococcus viridans
hypertrophied and covered with gray coating that I. Streptococcus pyogenes
spreads to the uvula and anterior pillars of the J. Staphylococcus saprophyticus
fauces. What is the most likely diagnosis?
17. An 18-year-old young man complains of pain
F. Oropharyngeal diphtheria in his knee and elbow joints and body temperature
G. Infectious mononucleosis up to 39.5°C. One week and a half earlier he
H. Pseudomembranous (Vincent’s) tonsillitis developed sore throat. On examination his body
L Agranulocytosis temperature is 38.5°C. Swelling of the knee and
J. Oropharyngeal candidiasis elbow joints is observed. Pulse is 106/min.,
rhythmic. Blood pressure is 90/60 mm Hg. Cardiac
14. A 42-year-old man, a dispatcher, suffes from borders are unchanged, heart sounds are
peptic ulcer disease of the duodenum. The disease weakened, at the cardiac apex there is a soft
is of moderate severity. He wants to be assigned systolic murmur. What factor would be the most
a disability group. Make the conclusion regarding indicative of the likely disease etiology?
his working ability:
F. Anti-streptolysin O
F. Capable of working, employable G. C-reactive protein
G. Capable of working, non-employable H. Creatine kinase
H. First group of disability I. Rheumatoid factor
L Second group of disability J. Seromucoid
J. Third group of disability
18. A woman has been provisionally diagnosed radiocarpal and metacarpophalangeal
with pheochromocytoma. At the stage of articulations and their reduced mobility in the
intermission her BP is within norm; there is a morning, which persisted up to 1.5 hours. Two
tendency towards tachycardia. No urine weeks ago she developed pain, swelling, and
pathologies. The decision has been made to reddening of her knee joints, her body
perform a provocative test with histamine. What temperature increased up to 37.5°C. The treatment
drug should be kept close at hand for emergency was untimely. Examination of the internal organs
aid in case of positive test result? revealed no pathologic alterations. Diagnosis of
rheumatoid arthritis was made. What changes are
F. Phentolamine
most likely to be visible on the arthrogram?
G. Pipolphen (Promethazine)
H. Nifedipine F. Joint space narrowing, usuration
L Mesaton (Phenylephrine) G. Joint space narrowing,
J. Prednisolone subchondral osteosclerosis
H. Cysts in the subchondral bone
19. A 40-year-old man with Bekhterev disease I. Numerous marginal osteophytes
(ankylosmg spondylitis) complains of elevated
J. Epiphyseal osteolysis
body temperature up to 37.8°C, back pain and
stiffness, especially observed during the second 22. A 52-year-old woman has been suffering for
half of the night. This condition has been lasting 2 years from dull, occasionally exacerbating pain
for 2 years. Objectively: reduced spinal mobility, in her right subcostal area, occurring after eating
painful sacroiliac joint, erythrocyte sedimentation high-fat foods, bitter taste in her mouth in the
rate - 45 mm/hour. X-ray shows narrowing of the morning, constipations, and flatulence.
intervertebral disc space and of the sacroiliac joint. Objectively she has excess weight, her body
What eye pathology is often associated with this temperature is 36.9°C; there is a coating on the
type of disease progression? root of her tongue; the abdomen is moderately
distended and painful in the area of gallbladder
F. Iridocyclitis projection. What examination would be the most
G. Retinal detachment helpful for diagnosismaking?
H. Cataract
L Optic nerve atrophy F. Ultrasound
J. Blepharitis G. Duodenal intubation
H. Cholecystography
20. A 63-year-old man complains of unmotivated I. Duodenoscopy
weakness and pressing and bursting sensation in J. Liver scanning
the left subcostal area. According to him, these
signs have been present for a year already. 23. A 57-year-old woman complains of weakness,
Previously he was healthy. He took part in dyspnea, loss of appetite, and liquid feces. She has
containment measures during the accident at the been suffermg from this condition for 2 years.
Chornoby] Nuclear Power Plant. Objectively: the Objectively she presents with pale skin, subicteric
skin is pale, peripheral lymph nodes are not sclerae, and bright-red fissured tongue. Lymph
enlarged, the liver is +3 cm, the spleen is +10 cm. nodes are not enlarged. Pulse - 100/min. BP-
Complete blood count: erythrocytes - 3.1 - 105/70 mm Hg. Liver +3 em, the spleen cannot be
1012/L, Hb- 100 g/L, leukocytes - 46 - 109/L, palpated.
blasts - 2%, promyelocytes - 10%, myelocytes Blood test: erythrocytes - 1.2 - 1012/L, Hb- 56 g/L,
18%, band neutrophils - 27%, segmented color index - 1.4, macrocytes, leukocytes - 2,5 -
neutrophils - 10%, lymphocytes - 12%, 10°/L, eosinophils - 1%, juvenile - 1%,
eosinophils - 6%, basocytes - 3%, monocytes - metamyelocytes - 1%, band neutrophils - 8%,
2%, erythrocyte sedimentation rate - 20 mm/hour. segmented neutrophils 47%, lymphocytes - 38%,
What 1s the most likely diagnosis? monocytes - 4%, reticulocytes - 0.1%, platelets -
100 - 10°/L, ESR- 30 mm/hour, indirect bilirubin
F. Chronic myeloleukemia
- 26 mmol/L. What changes can be expected in the
G. Hepatic cirrhosis
bone marrow puncture material?
HL Acute leukemia
L Hemolytic anemia F. Prevalence of megaloblasts
J. Chronic lymphatic leukemia G. Increased number of sideroblasts
H. Erythroid hyperplasia
21. For three years a 31-year-old woman has been
I. Presence of blast cells
complaining of pain and swelling of her
J. Prevalence of lymphoid tissue F. Hysterical neurosis
G. Hypochondriacal neurosis
24. A 35-year-old man _ suffers from
H. Depressive neurosis
insulindependent diabetes mellitus and chronic
I. Obsessive neurosis
cholecystitis. He takes NPH insulin: 20 units in the
morning and 12 units in the evening. After a meal J. Paranoid personality disorder
he developed pain in the right subcostal area, 28. A woman with atopic bronchial asthma was
nausea, vomiting, sleepiness, and increased found to have one allergen to dog hair +++.
polyuria. What prehospital measures will be the Carpets were removed from the apartment, the
most effective for prevention of crisis within the apartment was renovated, and air conditioner was
next several hours? installed. However, recurrent asphyxia attacks
still occur every night, despite the patient
F. Change insulin regimen
undergoing pathogenetic therapy. What long-
G. Take analgesics
term treatment tactics can help this patient to
H. Take cholagogues
decrease her sensitivity to the allergen?
L_ Exclude fats from the diet
J. Decrease carbohydrates in the diet F. Specific hyposensitization
G. Continuation of prior treatment
25. A 45-year-old woman complains of intolerable
H. Antihistamine therapy
paroxysmal facial pain on the left with attacks that
I. Buteyko breathing technique
last for 1-2 minutes. Attacks are provoked by
J. Referral for speleotherapy
chewing. The disease onset was two months ago
after the overexposure to cold. Objectively: pain at 29. A 20-year-old man was hospitalized on the 9th
the exit points of the trigeminal nerve on the left. day of the disease. He attributes his disease to
Touching near the wing of the nose on the left eating of insufficiently thermally processed pork.
induces a pain attack with tonic spasm of the facial At its onset this condition manifested as
muscles. What is the most likely diagnosis? periorbital edemas and fever. Objectively his
F. Trigeminal neuralgia body temperature is 38.5 C. The face is puffy and
G. Glossopharyngeal neuralgia the eyelids are markedly swollen. Palpation of
H. Temporomandibular joint arthritis gastrocnemius muscles 1s sharply painful. Blood
L_ Facial migraine test shows hypereosinophilia. What is the
etiology of this disease?
J. Maxillary sinusitis
F. Trichinella
26. A 28-year-old man complains of skin rash and
itching on the both of his hands. The condition G. Trichuris
persists for 1.5 years. The exacerbation of his H. Ascarididae
condition he ascribes to the occupational contact I. Echinococci
with formaldehyde resins. Objectively the lesion J. Leptospira
foci are symmetrically localized on both hands.
30. A 40-year-old man claims that his wife is
Against the background of erythema with blurred
cheating on him and presents a ’proof” of her
margins there are papulae, vesicles, erosions,
infidelity. He repeatedly initiated scandals with his
crusts, and scales. What is the most likely
wife at home and at work, demanding that she
pathology?
confess her infidelity, insulted her, and threatened
F. Occupational eczema to kill her. What preventive measures should be
G. Idiopathic eczema taken against socially dangerous actions on his
H. Allergic dermatitis part?
L Simple contact dermatitis F. Consultation with the psychiatrist
J. Erythema multiforme G. Outpatient treatment
27. A 20-year-old student after failing an exam H. Consultation with the general practitioner
developed complaints of a sensation of a round I. Consultation with the psychologist
foreign body in her throat, difficult swallowing. J. Family counseling
She fixates on her condition, limits her diet, often
31. A 55-year-old woman complains of pain and
cries, seeks attention, exhibits demonstrative
popping sounds in her left knee jomt, which occur
attitude. She is highly susceptible to
when she climbs the stairs. Occasionally during
psychotherapeutic suggestion. What psychiatric
movements her joint becomes ”stuck”. 5 years ago
diagnosis can be made in this case?
she suffered a trauma of her left knee. Complete
blood count and biochemical blood analysis show I. Pseudotuberculosis
normal results. X-ray shows marked osteosclerosis J. Trichinosis
and osteophytes. The joint space is narrowed.
Make the provisional diagnosis: 35. A woman has been working as a polisher for
a year and a half. Her workstation is equipped
D. Osteoarthritis with a grinding machine (grinding wheels). She
E. Rheumatoid arthritis complains of white discoloration of her fingers
F. Gouty arthritis D. Psoriatic arthritis and toes that appears when she is nervous.
E. Reactive arthritis Objectively there are no changes in the coloration
of the distal segments of her limbs. Grip strength
32. A 40-year-old man, a welder, uses manganese measured with a dynamometer is 25 kg,
electrodes in his line of work (18 years of algesimetry fmdings are 0.1; 0.3; 0.5. Cold
experience). He complains of difficulties with stimulus is extremely positive on the upper and
writing, bad mood, inertness, gait abnormalities, lower limbs. Internal organs are without
problems with speech, and hand tremors. pathologies. Make the diagnosis:
Objectively the following is observed in the
patient: hypomimia, mcreased muscle tone of F. Vibration disease
plastic type, and quiet monotonous speech, tremor G. Raynaud disease
of the tongue, pill-rolling tremor of the fingers, and H. Syringomyelia
retropulsion. What syndrome developed in this I. Raynaud syndrome
patient due to manganese poisoning? J. Polyneuritis
F. Parkinsonism 36. A 37-year-old man suddenly developed acute
G. Meningism headache accompanied by nausea, vomiting, and
H. Hypothalamic syndrome impaired consciousness. Objectively blood
L Polyneuritic syndrome pressure is 190/120 mm Hg, the face is
J. Vestibular syndrome hyperemic. Patient’s consciousness is clouded,
his answers to the questions are short,
33. A 45-year-old man developed constricting monosyllabic. Movement and — sensory
retrosternal pain that occurs during walks at the disturbances are absent. Meningeal signs are
distance of 200 m. Objectively: heart rate is positive. Cerebrospinal fluid contains blood.
80/min., BP is 160/00 mm Hg. During What provisional diagnosis can be made?
cardiopulmonary exercise test at 50 W there is a F. Subarachnoid hemorrhage
depression of S-T segment by 3 mm below the G. Meningitis
isoline in V3-V4. What is the provisional
H. Ischemic stroke
diagnosis?
I. Encephalitis
E. Exertional angina pectoris, functional J. Cerebral vascular embolism
class II
37. A woman undergoing in-patient treatment for
F. Exertional angina pectoris, functional
viral hepatitis type B developed headache, nausea,
class IV
recurrent vomiting, memory lapses, flapping
G. Exertional angina pectoris, functional
tremor of her hands, and rapid pulse. Sweet smell
class II
from her mouth is detected. Body temperature is
iH. Somatoform autonomic dysfunction, 37.6°C, heart rate is 89/min. What complication
hypertension type E. Alcoholic developed in the patient?
myocardiody strophy
F. Acute liver failure
34. A 45-year-old man, a farmer, presents with
G. Ischemic stroke
acute onset of a disease. He complains of
H. Gastrointestinal hemorrhage
headache, high temperature, pain in the
I. Hypoglycemic shock
gastrocnemius muscles, icteric face, and dark
J. Meningoencephalitis
urine. Objectively: body temperature - 38°C,
blood pressure - 100/70 mm Hg, conjunctival 38. A 43-year-old man, a coal-face worker with 15-
hemorrhages, hepatosplenomegaly, and oliguria. year-long record of work, complains of cough,
What is the most likely provisional diagnosis? thoracic pain, and dyspnea. The cough is mild,
usually dry, occurs mostly in the morning. The
F. Leptospirosis
pain is localized in the interscapular region and
G. Brucellosis
aggravates during a deep intake of breath. Dyspnea
EH. Viral hepatitis
occurs during physical exertion. Vesicular F. Hypertensive crisis complicated with instable
respiration in the lungs is weakened. Heart sounds angina pectoris
are rhythmic, heart rate 1s 86/min., blood pressure G. Hypertensive crisis complicated with acute left
is 135/80 mm Hg. The abdomen is soft and ventricular failure
painless. X-ray shows micronodular pulmonary H. Pulmonary embolism
fibrosis. Make the provisional diagnosis:
42. A 35-year-old patient developed an epileptic
F. Carboconiosis attack with tonoclonic spasms that lasted for 3
G. Byssinosis minutes. After the attack the patient fell asleep but
H. Siderosis in 5 minutes the second attack occurred. The first
L_ Berylliosis step of emergency aid would be to:
J. Metal pneumoconiosis
F. Ensure patency of airways
39. Having examined a 52-year-old patient, the G. Take blood from the vein for analysis
doctor diagnosed him with obesity (body mass H. Introduce diazepam intravenously
index - 34 kg/m?, waist circumference - 112 cm) I. Prescribe antiepileptic drugs
and arterial hypertension (170/105 mm Hg). 2- J. Administer chloral hydrate via an enema
hour postprandial blood sugar is 10.8 mmol/L.
What biochemical blood analysis needs to be 43. A 27-year-old woman, a teacher in the
conducted to diagnose the patient with metabolic elementary school, complains of frequent stools,
syndrome X? up to 3 times per day, with lumpy feces and large
amount of mucus, abdominal pain that gradually
F. Lipid profile abates after a defecation, irritability. Her skin is
G. Bilirubin pale and icteric. Pulse is 74/min., rhythmic, can
H. Calcium and phosphorus be characterized as satisfactory. Blood pressure is
IL Creatinine and urea 115/70 mm Hg. The abdomen is soft, moderately
J. Electrolytes tender along the colon on palpation. Fiberoptic
colonoscopy detects no changes. What disease
40. After overexposure to cold a 45year-old can be suspected?
woman developed acute pain in her suprapubic and
lumbar areas during urination, sharp pains at the F. Irritable bowel syndrome
end of urmation, false urges to urinate. Urine is G. Chronic non-ulcerative colitis
turbid with blood streaks. The doctor suspects H. Chronic enteritis
urinary tract infection. What results of laboratory I. Crohn disease (regional enteritis)
analysis would be the most indicative of such J. Whipple disease
infection?
F. Leukocyturia, gross hematuria 44. A 72-year-old man complains of lower
extremity edema, sensation of heaviness in the
G. Gross hematuria
right subcostal area, dyspnea at rest. For over 25
H. Increased blood creatinine and blood urea
years he has been suffermg from COPD.
L_ Daily proteinuria under 3.0
Objectively: orthopnea, jugular venous
J. Daily proteinuria over 3.0
distention, diffuse cyanosis, acrocyanosis. Barrel
41. A 62-year-old woman was brought into the chest is observed, on percussion there is a
admission room with complaints of severe vesiculotympanitic
burning retrosternal pain and asphyxia. She has a (bandbox) resonance, sharply weakened vesicular
10-year-long history of essential hypertension. respiration on both sides, moist crepitant crackles
Objectively her condition is moderately severe. in the lower segments of the lungs. Heart sounds
She presents with skin pallor, cyanotic lips, and are weakened, the II heart sound is accentuated
vesicular respiration over her lungs. The II heart over the pulmonary artery. The liver is +3 cm.
sound is accentuated over the aorta. Blood What complicated the clinical course of COPD in
pressure - 210/120 mm Hg, heart rate (pulse) - this patient?
76/min. ECG shows elevation of ST segment in
F. Chronic pulmonary heart
the leads I, AVL, and V5-V6. What is the most
G. Pulmonary embolism
likely diagnosis?
H. Acute left ventricular failure
A. Hypertensive crisis complicated with acute I. Diffuse pneumosclerosis
myocardial infarction B. Uncomplicated J. Community-acquired pneumonia
hypertensive crisis
45. A 72-year-old man with pneumonia murmur; the sound of opening mitral valve can be
complains of marked dyspnea, chest pain, severe auscultated at the cardiac apex. The IT heart sound
cough with expectoration, f¢ is 39.5-40°C, no is accentuated over the pulmonary artery. The
urination for a whole day. Objectively the patient patient is cyanotic. X-ray shows dilated
is conscious. Respiratory rate is 36/min. Over the pulmonary root and enlargement of the right
right lower pulmonary lobe percussion sound is ventricle and left atrium. What is the most likely
dull; on auscultation there is bronchial respiration diagnosis?
and numerous moist crackles. Blood pressure is
F. Mitral stenosis
80/60 mm Hg. Heart rate is 120/min. Heart
G. Aortic stenosis
sounds are muffled, there is tachycardia. What
tactics should the family doctor choose in the H. Pulmonary artery stenosis
management of this patient? I. Coarctation of the aorta
J. Patent ductus arteriosus
F. Hospitalization into the intensive care unit
G. Outpatient treatment 49. A 23-year-old man complains of severe pain
H. Treatment in the day patient facility in his left knee joint. Objectively the left knee
L Hospitalization into the pulmonology unit joint is enlarged, with hyperemic skin, painful on
palpation. Complete blood count: erythrocytes -
J. Hospitalization into the neurology unit
3.8 - 1012/L, Hb- 122 g/L, leukocytes - 7.4 -
46. 2 hours after eating unknown mushrooms, a 10°/L, platelets 183 109/L. Erythrocyte
28-year-old man sensed a decrease in his mobility sedimentation rate
and deterioration of his ability to focus. This
condition was then followed by a state of - 10 mm/hour. Bleeding time (Duke method) - 4
agitation and agression. On examiantion he is min., Lee-White coagulation time - 24 mun.
disoriented and his speech is illegible. 4 hours Partial thromboplastin time (activated) - 89
later he developed fetor hepaticus and lost his seconds. Rheumatoid factor - negative. What is
consciousness. What syndrome can be observed the most likely diagnosis?
in this patient? C. Hemophilia, hemarthrosis
F. Acute hepatic failure D. Werlhof disease (immune
G. Hepatolienal syndrome thrombocytopenia) C.
H. Portal hypertension Rheumatoid arthritis
L Cholestatic syndrome F. Thrombocytopathy
J. Cytolytic syndrome G. Hemorrhagic vasculitis (Henoch-Schonlein
purpura), articular form
47. A 36-year-old man complains of marked
dyspnea and cardiac pain. He ascribes his disease 50. A 24-year-old woman, a kindergarten teacher,
to the case of influenza that he had 2 weeks ago. has been sick for 2 days already. Disease onset
Objectively he leans forward when sitting. The was acute. She presents with elevated body
face is swollen, cyanotic, cervical veins are temperature up to 38.0°C, pain attacks in her
distended. Heart borders are extended on the both lower left abdomen, liquid stool in small amounts
sides, heart sounds are muffled, heart rate = Ps = with blood and mucus admixtures 10 times a day.
118/min., BP is 90/60 mm Hg. Blood test: ESR is Pulse - 98/min., blood pressure - 110/70 mm Hg.
46 mm/hour. ECG shows low voltage. Xray Her tongue is moist and coated with white
shows trapezoidal cardiac silhouette and signs of deposits. The abdomen is soft, the sigmoid colon
pulmonary congestion. Choose the treatment is painful and spastic. Make the provisional
tactics: diagnosis:

F. Pericardial puncture (pericardiocenthesis) F. Shigellosis


G. Diuretics G. Escherichiosis
H. Antibiotics H. Salmonellosis
IL Pericardectomy I. Yersiniosis
J. Glucocorticosteroids J. Rotavirus infection
51. A 38-year-old woman complains of weakness,
48. A 39-year-old man suffers from chronic sleepiness, pain in the joints, weight gain despite
rheumatic heart disease. He complains of dyspnea low appetite, and constipations. She presents with
during physical exertion, cough with dry and thickened skin, puffy and amimic face,
expectoration, and palpitations. Ausculation narrowed palpebral fissures, thick tongue, and
detects intensified I heart sound and diastolic
deep hoarse voice. Her heart sounds are weak, last 3 weeks he has been suffering from
pulse is 56/min. Low levels of free T4 are headaches of increasing intensity. Neurological
observed. This patient needs to take the following examination detects nuchal rigidity without focal
on a regular basis: signs. Make the provisional diagnosis:

D. Thyroxine F. Tuberculous meningitis


E. Mercazolil (Thiamazole) G. Chorea minor
F. Lithium carbonate D. Furosemide H. Brain tumor
E. Calcium gluconate I. Myelitis
J. Convexital arachnoiditis
52. A 23-year-old man has accidentally
swallowed brake fluid. After that he has been 56. A patient has gradually lost consciousness.
presenting with anuria for 5 days already; his The skin is pale and dry. There is a smell of
creatinine levels elevated up to 0.569 mmol/L. ammonia from the mouth. Respirations are deep
What treatment tactics should be chosen in this and noisy. Heart sounds are muffled, pericardial
case? friction rub is present. Blood pressure is 180/130
mm Hg. Blood test: Hb- 80 g/L, leukocytes 12 -
F. Hemodialysis
10°9/L, blood glucose - 6.4 mmol/L, urea 50
G. Detoxication therapy
mmol/L, creatinine - 1200 mcemol/L, blood
H. Antidotal therapy osmolarity - 350 mOsmol/L. No urinary
L Diuretics excretion. Make the diagnosis:
J. Plasmapheresis
F. Uremic coma
53. A 52-year-old man for the last 3 years has G. Hyperglycemic coma
been suffering from difficult swallowing of solid H. Acute renal failure
food, burning retrosternal pain that aggravated
I. Acute disturbance of cerebral circulation
during eating, loss of body mass, and occasional
J. Hyperosmolar coma
vomiting with undigested food. Esophageal X-ray
shows S-shaped deformation of the esophagus 57. A 72-year-old man diagnosed with ischemic
and its dilation, at the cardiac orifice the heart disease presents with diffuse
esophagus is constricted; esophageal mucosa is cardiosclerosis, permanent tachysystolic atrial
smooth, without signs of peristalsis. Make the fibrillation, heart failure Ha, FC II. Objective
provisional diagnosis: examination of vital signs: blood pressure is
135/80 mm Hg, heart rate is 160/min., pulse is
F. Esophageal carcinoma
125/min. Left ventricular ejection fraction is
G. Diaphragmatic hernia
32%. What drug is indicated in this case and
H. Esophageal achalasia should be presribed to the patient?
L_ Reflux esophagitis
J. Esophageal diverticulum F. Digoxin
G. Procaiamide (Novocainamide)
54. A 53-year-old man complains of general H. Isadrine (Isoprenaline)
weakness, loss of appetite, and painful vesicles
I. Verapamil
appearing on his skin. The disease onset occurred
J. Ivabradine
suddenly, after hyperinsolation one week ago.
Examination detects isolated vesicles with 58. A 34-year-old man complains of pale edema
wrinkled opercula and occasional painful of the face, feet, shins, and lumbar area, elevated
erosions on the skin of the patient’s torso and blood pressure up to 160/100 mm Hg, and general
limbs. Nikolsky sign is positive. What is the most weakness. He has a clinical history of nonspecific
likely diagnosis? ulcerative colitis. Objectively: pulse - 84/min.,
rhythmic, blood pressure - 165/100 mm Hg;
F. Acantholytic pemphigus
edemas all over the body; the skin is pale and dry,
G. Nonacantholytic pemphigus with low turgor. The kidneys cannot be palpated,
iH. Duhring’s disease (dermatitis on an attempt to palpate them they are painless.
herpetiformis) Blood test: erythrocytes - 3.0-1012/L, Hb- 100
L Herpes g/L, erythrocyte sedimentation rate - 50 mm/hour.
J. Toxicodermia Urinalysis: protems 3.5 g/L, erythrocytes - 7-10
in the vision field, leukocytes - 5-6 in the vision
55. A patient is being treated
field. Daily proteinuria - 6 grams. What analysis
in the tuberculosis clic. Throughout the
should be conducted additionally to verify the J. Secondary papular syphilid
diagnosis?
62. A 38-year-old woman after physical
F. Gingival biopsy for the diagnosis of amyloid
disease overexertion suddenly developed palpitations,
dyspnea, and a dull pain in the cardiac area. For 10
G. Radioisotopic examination of kidneys
years she has been registered for regular check-ups
EH. Urinalysis for Bence-Jones protein
due to rheumatism and mitral valve disease with
L Renal ultrasound
non-disturbed blood circulation. Oblectively her
J. Survey and excretory urography
pulse is 96/min., of unequal strength. Blood
59. A 42-year-old man, a worker at the meat pressure is 110/70 mm Hg, heart rate 1s 120/min.
processing factory, developed an itching spot on ECG registers small unevenly-sized waves in place
his lower jaw, which gradually transformed into a of P-waves, R-R intervals are of unequal length.
slightly painful carbuncle 3 cm im diameter, What is the most likely diagnosis?
surrounded by a painless swelling that reaches the
F. Atrial fibrillation
clavicle. Temperature is subfebrile, under 37.8°C.
G. Paroxysmal supraventricular tachycardia
The doctor suspects anthrax. What drug should
H. Atrial flutter
this man be prescribed for treatment?
I. Paroxysmal ventricular tachycardia
F. Penicillin J. Respiratory arrhythmia
G. Levomycetin (Chloramphenicol)
63. An 18-year-old patient always obeys others
H. Biseptol (Co-trimoxazole)
and adapts his needs to the demands of the people
L Interferon alpha
on whom he depends. He excessively defers to
J. Azidothymidin (Zidovudine) their wishes and makes them responsible for his
60. A 57-year-old patient complains of dyspnea at wellbeing, cannot defend his interests and needs
rest. The patient presents with orthopnea, support from other people. Such psychic profile
acrocyanosis, bulging cervical veins. On has been formed in the childhood, remains
percussion: dull sound over the lower lung unchanged, and hinders adaptation. What psychic
segments. On auscultation: no respiratory sounds. disorder is observed in this patient?
Heart rate is 92/min. Right-sided cardiac F. Dependent personality disorder
dilatation is observed. The liver is +7 cm. Shins
G. Anxiety (avoidant) personality disorder
are swollen. Pleural effusion is suspected. What
H. Anankastic personality disorder
indicator would confirm the presence of
I. Markedly accentuated personality
transudate in this case?
J. Psychopathy-like state
D. Total proteim content in the pleural fluid below
64. A 45-year-old man with thrombophlebitis of
25 g/L
the deep veins in his legs suddenly after physical
E. Presence of atypical cells
exertion developed sharp pain in his thorax on the
F. Total protein content in the pleural fluid
right, dyspnea, and hemoptysis. Objectively his
exceeding 30 g/L D. Specific gravity exceeding
condition is severe; he presents with
1015
acrocyanosis, shortening of pulmonary
E. Positive Rivalta’s test
percussion sound on the mght, and weakened
61. A 33-year-old man developed multiple rashes respiration. Respiration is 30/min., blood pressure
on the skin of his torso and extensor surfaces of is 110/80 mm Hg. ECG shows sinus tachycardia,
his upper and lower limbs. The rashes itch and heart rate is 120/min., electrical axis of the heart
occasionally fuse together and form plaques. The deviates to the right, S+Qy;. What is the most
elements of rash are covered with silver-white likely diagnosis?
fine scales that easily flake off when scratched.
A. Pulmonary embolism B. Community-
Grattage test results im three sequential acquired right-sided pneumonia C. Cancer
phenomena: stearin spot, terminal film, and of the right lung
punctate hemorrhage. What diagnosis can be F. Right-sided exudative pleurisy
suspected? G. Spontaneous pneumothorax
F. Psoriasis 65. A 48-year-old woman has been hospitalized
G. Parapsoriasis due to development of tachysystolic atrial
H. Pyoderma fibrillation. She has lost
L Lichen ruber planus
5 kg of body weight within 2 months. On in the tongue on the right. No other neurological
palpation there is a node in the left lobe of the pathologies were detected. What disease can be
thyroid gland. What pathology resulted in the provisionally diagnosed in this patient?
development of this condition?
F. Neuropathy of the facial nerve
F. Toxic nodular goiter G. Neuropathy of the trigeminal nerve
G. Aterosclerotic cardiosclerosis H. Trigeminal ganglionitis
EH. Chronic thyroiditis I. Neuropathy of the oculomotor nerve
L Nontoxic nodular goiter J. Ischemic stroke
J. Autoimmune thyroiditis
70. A 56-year-old woman was diagnosed with
66. A 48-year-old woman developed insomnia, stage 2 hypertension of the 2nd degree. She
depressive mood, anxiety, fears and suicidal belongs to the group of moderate risk and has
thoughts after the death of her husband that bronchial asthma. What group of drugs is
occurred one month ago. During her stay in the CONTRAINDICATED to this patient?
hospital she speaks in a low voice, is depressed, F. £-blockers
anxious, avoids sleeping, refuses to eat. What G. Angiotensin-converting enzyme inhibitors
medications should be prescribed in this case? H. Diuretics
F. Antidepressants I. Calcium antagonists
G. Antipsychotics J. Imidazoline receptor antagonists
EH. Group B vitamins 71. A 45-year-old woman 1s registered for regular
L Nootropics check-ups due to Werlhof disease (immune
J. Anticonvulsants thrombocytopenia). Complete blood count: Hb-
100 g/L, erythrocytes 2.8-1012/L, platelets -
67. A 32-year-old woman complains of episodes
90.0-10°/L, leukocytes - 8.4 - 109/L, erythrocyte
of intense fear that occur without visible cause
sedimentation rate 13 mm/hour. Examination
and last for 10-20 minutes, the episodes are
detects a single small hematoma on the anterior
characterized by rapid pulse, sweating, labored
surface of the thigh, developed after the patient
breathing, and vertigo. Specify the likely
accidentally stumbled on a table. What treatment
diagnosis:
tactics should be chosen in this case?
F. Panic disorder
F. Continue the supervision by the hospital
G. Paranoid syndrome
hematologist
H. Manic syndrome
G. Urgent hospitalization imto the
L Simple schizophrenia hematology unit
J. Claustrophobia H. Urgently start a hemostatic therapy,
68. A 39-year-old man suffers from chronic followed by a planned hospitalization into the
adrenal insufficiency and receives replacement hematology unit
glucocorticoid therapy L Urgent hospitalization mto the general
(hydrocortisone - 15 mg/day). He is to undergo care unit
elective surgery for calculous cholecystitis. What J. Administer thrombocytic mass, continue
medication adjustment should be made on the day the treatment in the hematology unit
of the surgery to prevent the development of acute
72. The dermatologist has an appointment with a
adrenal insufficiency?
30-year-old man that complains of severely
F. Increase the dosage by 2-3 times itching rashes that especially disturb him at night.
G. Cancel the drug for the day of the surgery The rashes developed 2 weeks ago, after he had
H. Add a mineralocorticoid returned from a travel. Objectively on the lateral
L_ Add an antibiotic surfaces of his fingers, hands, wrists, elbows,
lower abdomen, genitals, and thighs there are
J. Prescribe a large volume intravenous fluid
paired papulovesicles, single pustules, and
infusion
scratch marks. What disease can be suspected?
69. After a long drive with the window open a
F. Scabies
man developed facial asymmetry; he cannot close
his right eye, his right nasolabial fold is smoothed G. Pyoderma
out, movements of expression are absent on the H. Dermatitis
right, there is a disturbance of gustatory sensation I. Eczema
J. Shingles creatorrhea. What prescription would be the most
advisable in this case?
73. A 38-year-old woman developed a medical
condition 7 days after her return from F. Multi-enzyme preparations
Bangladesh. Periodical elevation of temperature G. Cholinergic antagonists
was accompanied by chills and excessive H. Metronidazole and loperamide
sweating. She was diagnosed with tropical I. Antacids and antispasmodics
malaria. Next day her condition further J. Cholinergic antagonists and antibacterial
deteriorated: body temperature - 38°C, inertness, agents
periodical loss of consciousness, generalized
seizures, tachycardia, hypotension, and icteric 77. A man was brought into the admission room
skin. What complication can be suspected in this after an overexposure to cold. He complains of
case? sharp pain in the small of his back and elevated
F. Cerebral coma body temperature up to 38°C. He took some
G. Serous meningitis aspirin. Blood test: leukocytes - 10.5 - 1012/L,
H. Purulent meningitis eosinophils - 5%, band neutrophils - 8%,
L Acute hepatic failure segmented neutrophils - 51%, lymphocytes -
J. Acute heart failure 32%, monocytes - 4%, erythrocyte sedimentation
rate - 28 mm/hour. Urinalysis: protein - 0.6 g/L,
74. A 73-year-old woman came to the family leukocytes - cover the whole vision field, large
physician for one of her regular follow-up amount of mucus. What is the most likely
examinations. Three months ago she was found to diagnosis?
have type 2 diabetes mellitus. She was keeping to
her diet and exercise plan and _ taking F. Acute pyelonephritis
phytopreparations. On examination her fasting G. Chronic pyelonephritis
glucose was within the range of 7.8-8.6 mmol/L, H. Acute glomerulonephritis
HbAlc - 7.9%. Height - 164 cm, weight - 83 kg. I. Tubulointerstitial nephritis
What blood sugar-controlling medicine should she J. Subacute malignant glomerulonephritis
be prescribed first in the course of her
78. A 26-year-old man complains of chills,
pharmacological therapy?
rhinitis, dry cough, and fever up to 38°C.
F. Metformin Examination shows him to be in a moderately
G. Glibenclamide severe condition; there are small pale pink non-
H. Glimepiride merging spots on the skin of his back, abdomen,
I. Gliclazide and extremities. Palpation reveals enlarged
J. Insulin
occipital and axillary lymph nodes. No
information about vaccination history could be
75. A 27-year-old man complains of pain in his leg obtained. What is the likely etiology of this
joints, purulent discharge from the eyes, and disease?
painful burning sensations during urination.
Disease onset was acute. He has a history of F. Rubella virus
influenza. The patient smokes and drinks alcohol G. Epstein-Barr virus
in excess. In his line of work he is often away on H. Streptococcus
business trips. What is the most likely etiological I. Mumps virus
factor of this disease? J. Neisseria meningitis

D. Chlamydia
E. Adenovirus
F. Streptococci D. Staphylococci
E. Candida

76. A 46-year-old woman has diarrhea with


abdominal distension, loss of body mass, and large
amounts of porridge-like foulsmelling stool
without blood streaks or tenesmus. Objective
examination detects moderate tenderness in the
mesogastrium and left abdominal flank. Feces
analysis detects steatorrhea with neutral fat and
1. During medical examination a cadet in the fingers appeared. Determine the frostbite degree
naval college was detected to have a painless in this child:
dense ulcer 1.5x0.5 in size in his perianal area at F. Frostbite of the I degree
the 2 o’clock position. The ulcer floor resembles G. Perniosis
*old fat”. What is the provisional diagnosis? H. Frostbite of the II degree
I. Frostbite of the III degree
C. Hard syphilitic chancre of the
J. Frostbite of the IV degree
rectum
D. Rectal fissure C. Rectal fistula 5. A 16-year-old patient has made an
F. Anal cancer appointment with an otolaryngologist. He
G. Anal crypt suppuration complains of elevated body temperature and sore
throat. Disease onset was 2 days ago, after the
2. A 32-year-old woman complains of tumorlike
patient ate two portions of ice-cream.
formation on the anterior surface of her neck that
Pharyngoscopy shows hyperemic mucosa of the
appeared 2 years ago. Within the last 3 months the
palatine tonsils, with purulent exudate in the
tumor has been rapidly growing. It hinders
lacunae. Make the provisional diagnosis:
swallowing and impairs speech; the tumor causes
a sensation of pressure. Objectively the skin F. Lacunar tonsillitis
moisture is normal, pulse is 80/min., rhythmic, G. Follicular tonsillitis
blood pressure is 130/80 mm Hg. In the night lobe H. Diphtheria
of the thyroid gland there is a dense lumpy node I. Acute pharyngitis
3.0x3.5 cm that moves during swallowing. J. Pseudomembranous (Vincent’s) tonsillitis
Scanning image shows a “cold nodule” in the
6. A 35-year-old woman complains of high body
thyroid gland. Make the provisional diagnosis:
temperature and pain in the upper outer quadrant
F. Thyroid cancer of her right buttock, which developed after an
G. Thyroid adenoma injection. She has been presenting with this
H. Thyroid cyst condition for 3 days. At the site of injection the
L Nodular goiter skin is hyperemic; there is a painful infiltrate
J. Autoimmune thyroiditis with an area of softening m its center. The
woman is diagnosed with a postinjection abscess
3. After a surgery for a left thigh phlegmon the
of the right buttock. What tactics should the
disease progression was complicated by sepsis.
surgeon choose in this case?
On the 7th day after the surgery there are marked
signs of a generalized inflammatory reaction, in F. Abscess incision, sanation and drainage of
blood there are signs of toxic anemia and the cavity
progressing hypoproteinemia, bilirubin levels are G. Hospitalization, prescription of antibiotics,
40 memol/L, AST and ALT exceed the norm by UHF
2.5 times. Oliguria persists (700 mL of urine per H. Abscess puncture, pus removal followed by
day). Name the phase of sepsis progression: application of antiseptics
L 10-15 minutes of low-intensity
D. Catabolic phase
laser radiation directed at the right
E. Stress phase
buttock
F. Anabolic phase D. Recovery phase
J. Antipyretic agents, massage,
E. Mixed phase
and
4. A 10-year-old boy, who was outdoors in windy application of dry heat to the right buttock
and cold weather, developed moderate pain and
7. A 65-year-old woman on abdominal palpation
tingling in his fingers and toes. When he returned
presents with a tumor in the umbilical region and
home, his parents noticed that the tips of his
above it; the tumor is 13x8 cm in size,
fingers and toes were white and their sensitivity
moderately painful, nonmobile, pulsing. On
was lost. As the affected areas were warming up,
auscultation systolic murmur can be observed.
the fingers and toes developed tingling and painful
What is the most likely diagnosis?
sensations. Skin pallor changed into redness,
tingling stopped, mild itching and swelling of the F. Abdominal aortic aneurysm
G. Gastric tumor
EH. Arteriovenous aneurysm F. Acute attack of glaucoma of the left eye
L_ Tricuspid insufficiency G. Acute iridocyclitis of the left eye
J. Bicuspid insufficiency H. Stage II intraocular tumor of the left eye
I. Endophthalmitis of the left eye
8. A 32-year-old man complains of pain in his
J. Panophthalmitis of the left eye
legs that intensifies during walking, intermittent
claudication, numbness of his toes, extremity 11. On the 15th day after a small trauma of the
coldness, and inability to walk more that 100 right foot, the patient developed indisposition,
fatigability, irritability, headache,
meters. When he sleeps, his leg usually hangs
elevated body temperature, and sensation
down. The patient has been smoking since he of constriction, tension, and
was 16. He drinks alcohol in excess. The left leg twitching in the muscles of the right shin. What
is colder than the right one; the skin of the disease can be suspected?
extremities is dry. No pulse can be detected on
C. Tetanus
the pedal arteries, while pulsation of the femoral
D. Anaerobic gas gangrene C.
arteries is retammed. What is the most likely
Erysipelas
diagnosis?
F. Acute thrombophlebitis
F. Obliterating endarteritis G. Thrombophlebitis of the popliteal artery
G. Diabetic angiopathy
12. A patient has the second and third degree burns
H. Leriche syndrome (aortoiliac occlusive
of the 15% of the body surface. On the 20th day
disease)
after the trauma the patient presents with sharp
I Raynaud disease
increase of body temperature, general weakness,
J. Deep thrombophlebitis
rapid vesicular respiration, facial features are
9. A 50-year-old patient was brought to a hospital sharpened, BP is 90/50 mm Hg, heart rate is
with complaints of blood in urme. Urination is 112/min. What complication is 1t?
painless and undisturbed. Macrohematuria had
F. Sepsis
been observed for 3 days. Objectively: kidneys
G. Pneumonia
cannot be palpated, suprapubic area is without
H. Acute intoxication
alterations, external genitalia are nonpathologic.
I. Purulent bronchitis
On rectal investigation: prostate is not enlarged,
J. Anaerobic infection
painless, has normal structure. Cystoscopy
revealed no changes. What is the most likely 13. 2 hours after a traffic accident a 28-yearold
diagnosis? man in a grave condition was brought to a
hospital. The patient complains of abdominal
F. Renal carcinoma
pain. He received a blow to the abdomen with the
G. Bladder tuberculosis
steering wheel. Objective examination revealed
H. Varicocele
the following: the abdomen does not participate
L Dystopic kidney
in respiration, is tense and acutely painful on
J. Necrotic papillitis
palpation; the abdominal muscles are defensively
10. A 59-year-old man complains of pain in his tense, peritoneal irritation signs are positive,
left eye and left side of his head, significant hepatic dullness is absent. BP is 90/60 mm Hg,
vision impairment of the left eye, nausea, and heart rate is 120/min. What further treatment
vomiting. Visual acuity of the right eye is 1.0. tactics should be chosen?
Visual acuity of the left eye is 0.03, attempts at
F. Laparotomy
correction bring no improvement. Right eye
G. Laparoscopy
intraocular pressure - 21 mm Hg, left eye
H. Cold to the abdomen
intraocular pressure 65 mm Hg. Congestive
I. Ultrasound investigation
injection is observed on the sclera of the left eye.
J. Laparocentesis
The cornea is thick and swollen. The anterior
chamber is shallow, moist, and clear. The pupil 14. A 48-year-old woman has arrived to the
is dilated and unresponsive to the light, the surgical unit with wounds in her thigh. On
fundus of the eye is not visible. What is the most examination the wound surface has dirty-gray
likely diagnosis? coating with unpleasant sweet smell. Wound
content resembles raspberry jelly. Skin tissues
around the wound are glossy and_ turgid. F. Right-sided reducible inguinal hernia
Palpation reveals moderate crepitation in the G. Right-sided reducible femoral hernia
tissues. What microflora is the most likely to EH. Cyst of the right spermatic cord
cause such inflammation? I. Right-sided inguinal lymphadenitis
J. Right-sided reducible arcuate line hernia
F. Anaerobic clostridial
G. Anaerobic non-clostridial 18. A 78-year-old man with a prostate adenoma
EH. Streptococci underwent a herniotomy for a direct inguinal
L Staphylococci hernia. After the surgery he presents with absent
J. Blue pus bacillus urination. Enlarged urmary bladder is detectable
above the patient’s pubis. What measures should
15. After a pain attack in the right subcostal area,
be taken in this case?
a 58-year-old woman with overnutrition
developed icteric skin and sclera, lightcolored C. Bladder catheterization
feces, and dark urine. Her abdomen is distended D. Apply cold to the urinary bladder area C.
and painful on palpation in the right subcostal Prescribe processing of the postoperative
area. Palpation detects liver enlargement by 2-3 wound with UHF field
cm. Blood test: total bilirubin - 90 memol/L, F. Prescribe —_ proserin (neostigmine)
conjugated bilirubin - 60 mcmol/L. What method intramuscularly
of examination will be the most informative for G. Prescribe antispasmodics subcutaneously
diagnosis clarification?
19. A 38-year-old patient has been brought by an
F. Retrograde cholangiopancreatography
ambulance to the surgical department with
G. Intravenous cholegraphy
complaints of general weakness, indisposition,
H. Infusion cholegraphy
black stool. On examination the patient is pale,
L Percutaneous transhepatic cholegraphy
there are dotted hemorrhages on the skin of his
J. US of the hepatopancreatobiliary zone
torso and extremities. On digital investigation
16. An 11-year-old boy for a month has been there are black feces on the glove. Blood test:
presenting with increasing pain in the right Hb- 108 g/L, thrombocytopenia. Anamnesis
femur. In the painful area there is a nonmobile states that a similar condition was observed 1
painful tumor with unclear margins. The child year ago. Make the diagnosis:
complains of general indisposition, weakness,
F. Thrombocytopenic purpura
increased body temperature up to 39°C. X-ray
G. Hemophilia
shows widened medullary cavity, small foci of
H. Ulcerative bleeding
cancellous bone destruction, and onion-like
I. Rectal tumor
lamellar exfoliation of the cortical layer. What is
J. Nonspecific ulcerative colitis
the most likely pathology resultmg in such
clinical presentation? 20. A 30-year-old man came to the family
physician. 2 months ago he underwent a surgery
F. Ewing sarcoma
for open fracture of the humerus. On examination
G. Osteogenic sarcoma
the patient’s condition 1s satisfactory; in the area
H. Fibrosarcoma
of the postoperative wound there is a fistula that
I. Chondrosarcoma
discharges a small amount of pus; the area itself
J. Juxtacortical sarcoma
is red; fluctuation is detected. X-ray shows
17. A 43-year-old man complains of a protrusion destruction of the humerus with sequestra. What
in the right inguinal region, that enlarges due to complication did the patient develop during the
strain. He has been presenting with this condition postoperative period?
for 6 months. Within this period the protrusion
F. Posttraumatic osteomyelitis
has grown. Objectively in the might inguinal
G. Hematogenous osteomyelitis
region an elastic protrusion 8x5 cm is visible. On
H. Wound suppuration
palpation it disappears, leaving an empty space
I. Posttraumatic phlegmon
4x4 cm between the pedicles of the Poupart
J. Suture sinus
ligament. *Cough push” sign 1s positive over this
opening. Make the diagnosis: 21. 3 hours after a trauma, a young man
developed bradycardia of 46/min., anisocoria
D>S8, hemi-hyperreflexia S>D, hemihypesthesia position, cyanotic skin covered in cold sweat,
on the left, and a convulsive disorder. The tachycardia, deficient pulse, and low blood
character of this process needs to be clarified. pressure. What urgent treatment tactics should be
What method of examination will be the most chosen?
accurate for this purpose?
F. Tracheostomy
F. Brain CT G. Oral administration of hyposensitization
G. Skull X-ray substances and broncholytics
EH. Electroencephalography EH. Intravenous administration of dehydrating
L_ Echoencephalography agents
J. Lumbar puncture I. Administration of glucocorticoid hormones
J. Oxygen therapy
22. The body of a 24-year-old woman with
suspected poisoning has been found on the street. 26. Heart X-ray of a 31-year-old man has
Forensic medical examination was requested by revealed the following: with tightly filled
an investigator during examination of the site and opacified esophagus there is a marginal filling
the body. According to the Criminal Procedure defect in its middle third on the posterior wall;
Code currently in force in Ukraine, forensic the defect is 1.8x1.3 cm in size with clear oval
medical examination is required when it is border. Mucosal folds are retained and envelop
necessary to determine the: the defect; wall peristalsis and elasticity are not
affected. There are no complaints regarding the
F. Cause of death
condition of the patient’s alimentary canal. Make
G. Manner of death
the provisional diagnosis:
H. Time of death
I. Mode of death F. Esophageal tumor
J. Mechanism of death G. Achalasia cardiae
H. Esophageal burns
23. A 37-year-old patient complains of pain in
IL Diverticulum
the spimal column, reduced mobility. The
J. Barrett esophagus
condition persists for 7 years. *Sway back” is
observed, there is no movement in all spinal 27. A 25-year-old man was hospitalized with
regions. X-ray shows “bamboo spine” vertebral complaints of pain in his lower abdomen and
column. What is the most likely diagnosis? right lumbar area that appeared one hour ago.
F. Ankylosing spondyloarthritis Patient’s general state is moderately severe.
G. Osteochondrosis Body temperature - 38.2°C, heart rate - 102/min.
EH. Spondylitis deformans The tongue is dry. The abdomen is painful on
L Tuberculous spondylitis deep palpation in the right ilac area and in the
J. Spondylolisthesis Petit triangle. AureRozanov and Gabay signs are
positive. Make the provisional diagnosis:
24. A surgery unit received a person with an
F. Acute appendicitis
incised stab wound on the upper third of the nght
G. Right-sided renal colic
thigh. Examination detects an incised stab wound
H. Cecal tumor
3.0x0.5x2.0 cm in size on the inner surface of the
I. Intestinal obstruction
upper third of the night thigh. Bright-red blood
J. Acute cholecystitis
flows from deep within the wound in a pulsing
stream. Characterize this type of bleeding: 28. A 45-year-old man diagnosed with acute
pulmonary abscess suddenly developed sharp
F. Arterial
pain in his chest on the right and dyspnea up to
G. Venous
30/min. Examination detects facial cyanosis and
H. Parenchimatous
shallow rapid respirations. Auscultation reveals
L Capillary
acutely weakened respiration throughout the
J. Mixed
whole right lung; percussion reveals a
25. A 47-year-old man developed the signs of vesiculotympanitic (bandbox) resonance at the
decompensated laryngeal stenosis against the lung apex and dullness in the lower lobe. What
background of acute flegmonous laryngitis. He complication developed in this patient?
presents with inspiratory dyspnea at rest, forced
F. Pyopneumothorax patient’s body there is graybrown area of necrosis
G. Pleuropneumonia that covers 3/4 of the body perimeter.
H. Pneumothorax Occasionally there are small blisters with
L Acute mediastinitis hemorrhagic contents and patches of shredded
J. Esophageal perforation epidermis. What local therapy is necessary in this
case?
29. A 5-year-old child was brought to the ENT
department by an ambulance. The child presents F. Decompression necrectomy
with cough and difficult respiration. From the G. Chemical necrolysis
patient’s history it is known that the child was H. Blister puncture
playmg with a toy construction set, when I. Necrectomy with xenotransplantation
suddenly started coughing and developed J. Necrectomy with dermal autograft
labored breathing. Examination detects
32. A woman in her early- to mid-thirties has lost
periodical cough, labored expiration, and
her consciousness 3-5 minutes ago. On
respiratory lag in the left side of the child’s
examination: the skin is pale, no pulse over the
thorax. Auscultation: diminished respiration on
carotid arteries, no spontaneous respiration, pupils
the left. Percussion: tympanitis. X-ray shows a
are dilated; the patient is nonresponsive, presents
displacement of the mediastinal organs to the
with atony. The patient’s condition can be
right. Make the diagnosis:
determined as:
F. A foreign body in the left bronchus,
F. Clinical death
valvular bronchostenosis
G. Natural death
G. A foreign body in the right bronchus,
H. Syncope
valvular bronchostenosis
I. Brain death
H. A foreign body in the trachea
J. Comatose state
L_ A foreign body in the left bronchus, complete
bronchostenosis 33. A boy had a foreign body removed from under
J. A foreign body in the right bronchus, partial his nail plate. 3 days later he developed a sharp
bronchostenosis throbbing pain at the end of his distal phalanx,
which intensifies when the phalanx is pressed,
30. A 30-year-old man was brought to the
hyperemia of the nail fold, elevated body
neurosurgical department with complaints of
temperature up to 38.5°C, and nail plate
constant headaches, nausea, vomiting, fever, and
discoloration. Make the diagnosis:
weakness of the right-side limbs. Anamnesis
states that one month ago the patient had a F. Subungual panaritium
surgery for left-sided suppurative otitis and G. Erysipelas
mastoiditis. He has been undergoing treatment in H. Paronychia
an ENT department. Approximately 2 weeks ago I. Erysipeloid
the temperature increased, and the patient J. Abscess
developed headaches. Objectively: heart rate -
98/min., BP- 140/90 mm Hg, temperature 34. A 32-year-old woman complains of body
38.3°C. Neurologically manifested stiff neck: weight loss despite her imcreased appetite,
bilateral Kernig’s symptom, unsteadiness during nervousness, and tremor of the extremities.
Objectively: the skin is moist; the thyroid gland is
the Romberg’s maneuver. Computer tomography
of the brain revealed a threedimensional growth diffusely enlarged, painless, soft, and mobile.
with a capsule in the left hemisphere. Make the Blood test: increased level of T3, T4, and thyroid-
diagnosis: stmulating hormone (THS). What is the most
likely diagnosis?
F. Cerebral abscess
G. Echinococcus F. Diffuse toxic goiter
H. Hemorrhage G. Thyroid carcinoma
L Hydrocephalus H. Autoimmune (Hashimoto’s) thyroiditis
J. Armold-Chiari malformation I. Thyroid adenoma
31. The burns unit received a patient, who 6 hours J. Diffuse nontoxic goiter
ago during a fire received flame burns. On the
35. A 19-year-old young man complains of cough D. In medico-prophylactic institutions;
with expectoration of purulent sputum in the general physicians and surgeons
amount of 100 mL per day, hemoptysis, dyspnea, E. At the site of the accident; first-response
increased body temperature up to 37.8°C, general emergency teams
weakness, weight loss. The patient’s condition F. At the site of the accident; specialized
lasts for 4 years. Exacerbations occur 2-3 times a second-response emergency teams D. In medico-
year. The patient presents with malnutrition, pale prophylactic institutions; specialized second-
skin, cyanosis of the lips, drumstick response emergency teams E. In medical
(clubbed) fingers. Tympanic percussion sound in institutions; all listed types of healthcare workers
the lungs, weakened respiration, numerous 39. A 45-year-old man underwent a cardiac
various moist crackles in the lower pulmonary surgery one week ago. His general state has been
segments on the left can be observed. In blood: deteriorating since then: dyspnea at rest,
erythrocytes - 3.2-1012/L, leukocytes - retrosternal pain that irradiates to the neck,
8.4-10°/L, ESR- 56 mm/hour. On X-ray: lung marked weakness. Objectively his body
fields are emphysematous, the left pulmonary temperature is hectic. His cardiac borders are
root is deformed and dilated. What is the most expanded, apical beat is weakened. Auscultation
likely diagnosis? detects pericardial friction rub. What is the most
likely diagnosis?
F. Multiple bronchiectasis of the left lung
G. Chronic left-sided pneumonia F. Acute pericarditis
EH. Chronic abscess of the left lung G. Acute cardiac aneurysm
L_ Left-sided pulmonary cystic dysplasia H. Myocardial infarction
J. Suppuration of the cyst in the left lung I. Acute myogenic dilatation of the heart
J. Pulmonary embolism
36. A 57-year-old woman during a regular
ultrasound examination presented with a space- 40. A 45-year-old man was brought by an
occupying heterogeneous lesion in the right ambulance into the emergency hospital. He
kidney. What is the most informative method of complains of sudden pai in the lumbar area,
renal tumor diagnostics? frequent painful urination, and vomiting.
Examination detects pain mm the lumbar area,
F. Spiral computed tomography
costovertebral angle tenderness, pain on palpation
G. Excretory urography
of kidneys and along the ureter on the right. Urine
H. Retrograde pyelography
test: proteins, fresh erythrocytes, leukocytes.
L Radioisotope renography
Make the provisional diagnosis:
J. Three glass urine test
F. Urolithiasis, renal colic
37. A 40-year-old victim of a traffic accident
G. Acute pyelonephritis
sustained the following injuries: closed
H. Acute glomerulonephritis
diaphyseal femur fracture, brain concussion,
I. Acute renal failure
multiple mb fractures, hemopneumothorax,
J. Polycystic kidney disease
degloving shin injuries. What injuries require the
most urgent attention?

F. Multiple rib fractures, hemopneumothorax


G. Closed diaphyseal femur fracture
H. Brain concussion
L Degloving shin injuries
J. All injuries are equivalent

38. At the railroad crossing a passenger train


collided with a bus. In this collision 26 bus
passenges died, another 18 passengers received
mechanical injuries of varying severity. Where
will be professional medical aid provided for the
victims of this accident? Who will provide this
aid?
1. A newborn girl has Apgar score of 7-8 disease is made. What examination would be the
points at the 1-5 minutes after birth. During the most informative for diagnosis confirmation?
labor there was a brief difficulty with extraction F. Doppler echocardiography
of the shoulder girdle. After birth the baby G. Electrocardiography
presents with disturbed function of the proximal H. Chest X-ray
segment and forced position of the right arm. The I. Rheography of the pulmonary artery
shoulder is rotated inwards, the elbow is J. Ultrasound of the liver
extended, the forearm is pronated, and the whole
upper limb resembles an arm of a doll. What is 5. Mother of a 5-year-old child noticed on the the
the most likely clinical diagnosis in this case? head of her child a round “bald” spot 3 cm in
diameter. All the hairs in the focus are broken off
F. Erb-Duchenne palsy at the length of 5-6 mm. The day before the child
G. Thoracic spine trauma was petting a stray cat. Make the diagnosis:
H. Osteomyelitis of the right arm
L Intracranial hemorrhage F. Microsporia
J. Soft tissue injury of the right arm G. Superficial trichophytosis
H. Deep trichophytosis
2. Disease onset was acute. A child developed I. Psoriasis
general weakness, pain in the joints, and elevated J. Alopecia areata
temperature. Later these signs became
accompanied by itching skin rash manifested as 6. A 2-year-old child with persisting cough and
erythematous spots 25 mm in size. The rash subfebrile body temperature after a case of URTI
gradually turned hemorrhagic. Large joints are developed dyspnea, cyanosis of the nasolabial
painful and swollen; pain attacks periodically triangle, percussion dullness and weakened
occur in the paraumbilical area; there are signs of respiration in the lower lobe of the right lung, and
intestinal hemorrhage. What is the most likely a slight mediastinal displacement to the left. What
diagnosis? pulmonary pathology is likely to cause this
clinical presentation?
A. Hemorrhagic vasculitis (Henoch-Schonlein
purpura) B. F. Pleurisy
Scarlet fever G. Emphysema
F. Hemorrhagic meningoencephalitis H. Pneumonia
G. Streptococcal impetigo I. Atelectasis
H. Rheumatism J. Bronchitis

3. A 13-year-old girl for the last two weeks has 7. During examination a 4-month-old child with
been complaining of dyspnea and shin and foot meningococcemia presents with acrocyanosis,
edemas that appear after a physical exertion. In cold extremities, tachypnea, and thready pulse,
the morning the edemas significantly decrease. blood pressure of 30/0 mm Hg, anuria, and sopor.
Clinical examination revealed enlarged liver and What clinical syndrome 1s it?
coarse systolic murmur over the heart area.
F. Toxic shock syndrome
Blood test and urinalysis are without changes.
G. Neurotoxicosis
What is the most likely cause of edemas in this
H. Exicosis
child?
I. Encephalic syndrome
D. Heart failure J. Acute renal failure
E. Nephrotic syndrome
8. At night a 2-year-old child with upper
F. Acute pyelonephritis D. Angioneurotic edema
respiratory tract infection suddenly developed
E. Hepatic cirrhosis
dyspnea with labored inspiration. Objectively the
4. A 7-year-old boy has severe pulmonary skin is pale, perioral cyanosis and_ slight
mucoviscidosis (cystic fibrosis). He complains of acrocyanosis are observed. Breathing is loud,
dyspnea and blood expectoration. Objectively he respiration rate is 32/min. Jugular, supra- and
presents with lagging physical development, infraclavicular fossae retract during breathing.
acrocyanosis, hepatomegaly, drumstick fingers, Respiration is coarse on auscultation. Heart
and nail plates resembling a “clock face”. sounds are clear and sonorous, heart rate is
Provisional diagnosis of chronic pulmonary heart 120/min. What condition was complicated by the
development of the upper respiratory tract What should be excluded from the diet in this
infection? case?

F. Stenosing laryngotracheitis F. Cereals - wheat, oats


G. Airway foreign body G. Milk and dairy products
H. Obstructive bronchitis H. Fruits
L_ Bronchiolitis I. Animal protein
J. Bronchial asthma J. Easily digestible carbohydrates
9. A l-year-old child with a case of URTI
13. A 7-year-old boy has been an inpatient for 1.5
suddenly developed noisy respirations with
months. He had been brought to the hospital with
difficult mspiration, intercostal retractions, and
complaints of edemas all over his body, low
barking cough on the 2nd night after the disease
urine output, and headache. Clinical urinalysis:
onset. What is the most likely diagnosis?
proteins 7.1 g/L, leukocytes - 1-2 in the vision
F. Stenosing laryngotracheobronchitis field, erythrocytes - 3-4 m the vision field.
G. Acute pulmonary inflammation During the course of treatment the edemas
H. Bronchial asthma gradually dissipated, headache abated, diuresis
L Acute bronchitis normalized. Daily urine protems 3 g/L.
J. Acute bronchiolitis Biochemical blood test: total protein 43.2 g/L,
urea - 5.2 mmol/L, cholesterol - 9.2 mmol/L.
10. A 10-year-old boy with symptoms of arthritis What glomerulonephritis syndrome is the most
and myocarditis was brought to a hospital. Based likely to be present in the patient?
on clinical examination the provisional diagnosis
of juvenile rheumatoid arthritis was made. What F. Nephrotic
symptom is the most contributive for the G. Nephritic
diagnostics of this disease? H. Isolated urinary
I. Hematuric
F. Reduced mobility of the joints in the morning J. Mixed
G. Regional hyperemia of the joints
H. Affection of the large joints 14. A 3-month-old child with signs of rickets
L Enlarged heart presents with positive Chvostek, Trousseau, and
J. Increased heart rate Maslov signs. One day ago the parents witnessed
a cyanotic attack in their child the child broke into
11. A 7-year-old girl has been twice treated with a cold sweat, the eyes bulged, and respiratory
antibacterial agents for urinary tract infection. arrest occurred. One minute later the child drew in
US shows no severe renal defects. The child a loud breath and the child’s condition normalized
presents with recurrence of leukocyturia and again. What is the cause of the described signs of
bacteriuria, elevated body temperature up to the disease?
38.5°C, and pain in her left lumbar area. What
examination should be conducted first to clarify F. Decrease of blood calcium levels
the cause of urinary infection recurrence? G. Increase of blood calcium levels
H. Decrease of blood phosphorus levels
F. Micturating cystourethrography I. Increase of blood phosphorus levels
G. Excretory urography J. Metabolic acidosis
H. Retrograde pyelography
L Immunogram 15. A newborn with gestational age of 31 weeks
J. Radioisotope renography presents with hypotonia and _ depressed
consciousness. Hematocrit is 35%, general
12. A child is 1 year old. After solid food was cerebrospinal fluid analysis shows increased
introduced into the diet, within the last several content of erythrocytes and protein, and low
months the child developed loss of appetite, glucose. These data correspond with the clinical
diarrhea with large amount of feces, and presentation of:
occasional vomiting. Body temperature remains
normal. Body weight is 7 kg. The child is very F. Intracranial hemorrhage
pale, has leg edemas and extremely distended G. Meningitis
abdomen. Feces analysis detects high levels of H. Sepsis
fatty acids and soaps. Diagnosis of celiac disease I. Anemia
was made and gluten-free diet was prescribed. J. Intrauterine infection
16. A newborm has Apgar score of 9. When should The cough attacks induce vomiting. X-ray shows
this infant be put to the breast? intensified bronchial pattern. Blood _ test:
leukocytes 16 - 109/L , lymphocytes - 72%,
F. In the delivery room
erythrocyte sedimentation rate - 4 mm/hour.
G. After 12 hours What 1s the most likely diagnosis?
H. After 2 hours
L On the 2nd day F. Pertussis
J. On the 3rd day G. Obstructive bronchitis
H. Pneumonia
17. A 3-week-old infant developed large, flaccid I. Adenovirus infection
vesicles with purulent contents on the skin of chest
J. Foreign body
and abdomen. The vesicles rupture quickly. Make
the provisional diagnosis: 21. A 3-year-old child presents with dyspnea that
F. Pemphigus neonatorum abates in the sitting position, occasional loss of
G. Vesiculopustulosis consciousness and seizures, delayed physical
H. Toxic erythema development, cyanosis, drumstick fingers.
L Pemphigus syphiliticus Echocardioscopy detects aortic dextraposition,
J. Pseudofurunculosis ventricular septal defect, pulmonary artery
stenosis, and right ventricular hypertrophy. What
18. 10 hours after birth a child developed is the most likely diagnosis?
jaundice, hypotonia, hyporeflexia, and moderate F. Tetrad of Fallot
hepatosplenomegaly. Feces and urine are of G. Coarctation of the aorta
normal color. Umbilical cord blood bilirubin is H. Transposition of the great vessels
51 memol/L due to unconjugated bilirubin levels. I. Ventricular septal defect
In venous blood: erythrocytes - 3.5-1012/L, Hb- J. Acquired valvular disease
140 g/L, reticulocytes - 1.5%, bilirubin - 111
memol/L, conjugated - 11 memol/L, ALT- 40 22. A 15-year-old girl complains of dizziness and
U/L, AST30 U/L. Mother’s blood group is A(II) sensation of lack of air that she develops in
Rh(- emotionally straining situations. Relief occurs
), child’s blood group is ACI) Rh(+). What after she takes corvalol. Objectively:
laboratory test can confirm the diagnosis? hyperhidrosis and marble-like pattern of the skin
of her palms and feet. Clinical and instrumental
F. Coombs test examination revealed no organic changes in the
G. Viral hepatitis markers analysis central nervous, cardiovascular, and respiratory
H. Measurement of erythrocyte systems. What provisional diagnosis can be
osmotic resistance made?
L_ Erythrocytometry
J. Measurement of glucose 6- F. Somatoform autonomic dysfunction
phosphate dehydrogenase levels in G. Obstructive bronchitis
erythrocytes H. Bronchial asthma
I. Stenosing laryngotracheitis
19. A 6-month-old child on breastfeeding is J. Acute epiglottitis
hospitalized in the inpatient department. After
the child recovers, the doctor recommends the 23. A 1.5-month-old child on breasfeeding
mother to start introducing solid food to the presents from birth with daily vomiting, irregular
child’s diet. What products should be introduced liquid foamy feces, and meteorism, which are
to the child’s diet first? resistant to antibacterial and probiotic therapy; no
increase of body mass is observed. The child’s
F. Vegetable puree condition improved, when breastmilk was
G. Fermented dairy products substituted with “NAN low lactose” formula.
EH. Grated apple What pathology is it?
L Semolina porridge
J. Buckwheat porridge F. Lactase deficiency
G. Intestinal lambliasis (Giardiasis)
20. The 5-year-old child has been ill for 2 weeks. H. Infectious enteritis
Cough attacks developed first and were then I. Drug-induced enteritis
followed by reprises. During coughing the J. Functional dyspepsia
child’s face turns red and cervical veins bulge.
24. A 13-year-old girl for a month has been 70%, ESR- 52 mm/hour. Make the provisional
complaining of fatigability, dull pain in her right diagnosis:
subcostal area, abdominal distension, and
F. Acute leukemia
constipations. Abdominal palpation reveals
positive Kehr, Murphy, and Ortner signs, while G. Thrombocytopenic purpura
Desjardins and Mayo-Robson points are painless. H. Acute rheumatic fever
Total bilirubin is 14.7 mcmol/L, predominantly I. Infectious mononucleosis
indirect, ALT- 20 U/L, AST- 40 U/L, amylase - J. Hemorrhagic vasculitis (Henoch-Schonlein
6.3 mmol/L. Echocholecystography shows purpura)
practically no contraction of the gallbladder.
28. During an outdoors school event in hot
Make the provisional diagnosis:
weather, a 10-year-old girl lost her
F. Hypokinetic biliary dyskinesia consciousness. Body temperature - 36.7°C.
G. Hyperkinetic biliary dyskinesia Objectively her skin is pale and cold to touch, her
EH. Chronic pancreatitis pupils are dilated. Blood pressure - 90/50 mm Hg.
L Acute pancreatitis Heart rate - 58/min. What pathology occurred in
J. Chronic hepatitis this case?

25. A 22-day-old infant developed subcutaneous F. Syncope


red nodes from 1.0 to 1.5 cm in size on the scalp; G. Sympathicotonic collapse
later the nodes suppurated. Temperature increased H. Paralytic collapse
up to 37.7°C, intoxication symptoms appeared, I. Sunstroke
regional lymph nodes enlarged. Complete blood J. -
count: anemia, leukocytosis, neutrocytosis,
29. A 13-year-old girl has 30% of excessive body
increased ESR. What diagnosis can be made?
mass, she started to gain weight at the age of 3.
F. Pseudofurunculosis
She has a family history of obesity. Her height and
G. Pemphigus sexual development are normal for her age. The
H. Vesiculopustulosis appetite is excessive. She complains of periodical
L Scalp phlegmon headaches. Blood pressure - 120/80 mm Hg.
J. - Subcutaneous fat 1s evenly distributed, she has no
26. A 10-year-old boy was brought into the stretch marks. There is juvenile acne on her face.
hospital with complaints of expiratory dyspnea, What type of obesity is it?
respirations are 30/min. He explains his state by F. Alimentary constitutive obesity
a change in the weather conditions. For the last 4 G. Hypothalamic obesity
years the boy has been registered for regular H. Adrenal obesity
check-ups due to his diagnosis of third degree
I. Hypothalamic syndrome of puberty
persistent bronchial asthma. To provide
J. Hypothyroid obesity
emergency aid for this child, first he needs to be
given: 30. An 8-year-old girl complains of frequent
painful urination in small amounts and urinary
F. Salbutamol or short-acting £2-agonists
incontinence. The signs have been present for 2
G. Dexamethasone days already. She explains her disease by
H. Adrenaline overexposure to cold. Costovertebral angle
L_ Euphylline (Aminophylline) tenderness is absent. Complete blood count is
J. Claritin (Loratadine) without pathologies. Urine test: leukocytes - 20-
27. A 3-year-old child has been brought to a 30 in the vision field, erythrocytes - 40-50 in the
hospital with complaints of pain in the legs, vision field, unchanged, bacteriuria. What is the
fever, and loss of appetite. Objectively: pale skin most likely diagnosis?
and mucosa, hemorrhagic rash. Lymph nodes are F. Cystitis
enlarged, painless, dense and elastic, not matted G. Vulvitis
together. Bones, joints, and abdomen are painful. H. Pyelonephritis
The liver and spleen are enlarged. Hemogram: I. Glomerulonephritis
Hb- 88 g/L, color index - 1.3, platelets - 80 -
J. Urolithiasis
109/L, leukocytes - 25.8-10°/L, lymphoblasts -
1. A pregnant woman is 28 years old. J. Hormone-producing ovarian tumor
Anamnesis: accelerated labor complicated by the
4. A 48-year-old woman complains of disturbed
II degree cervical rupture. The following two
menstrual cycle: her periods last for 7-9 days and
pregnancies resulted in spontaneous abortions at
are excessively profuse throughout the last half-
the terms of 12 and 14 weeks. On mirror
year. She notes occasional hot flashes in her head,
examination: the uterine cervix is scarred from
insomnia, irritability, and headaches. Her skin is
previous ruptures at 9 and 3 hours, the cervical
of normal color. Blood pressure - 150/90 mm Hg,
canal is gaping. On vaginal examination: the
pulse - 90/min., rhythmic. The abdomen is soft
cervix is 2 cm long, the external orifice is open 1
and painless. Bimanual examination shows no
cm wide, the internal orifice is half-open; the
uterine enlargement, the appendages cannot be
uterus is enlarged to the 12th week of pregnancy,
detected. The vaginal fornices are free. What is
soft, mobile, painless, the appendages are without
the most likely diagnosis?
changes. What diagnosis can be made?
Climacteric syndrome
A. Isthmico-cervical insufficiency, habitual
Premenstrual syndrome
noncarrying of pregnancy B. Threatened
Adrenogenital syndrome
spontaneous abortion C. Incipient abortion,
Stem-Leventhal syndrome
habitual noncarrying of pregnancy
(polycystic
D. Cervical hysteromyoma, habitual
ovary syndrome) E.
noncarrying of pregnancy E.
Uterine myoma
Cervical pregnancy, 12 weeks
5. A 30-year-old multigravida has been in labour
2. On the day 4 after the cesarean section a woman
for 18 hours. 2 hours ago the pushing stage began.
developed fever with body temperature up to 39°C
Fetal heart rate is clear, rhythmic, 136/min.
and abdominal pain. Pulse - 104/min. She
Vaginal examination reveals complete cervical
vomited twice. The patient is sluggish, her tongue
dilatation, the fetal head in the pelvic outlet plane.
is dry and has gray coating. The abdomen is
Sagittal suture is in line with obstetric conjugate,
distended. Signs of peritoneal irritation are
the occipital fontanel is near the pubis. The
positive in all segments. Peristalsis cannot be
patient has been diagnosed with primary uterine
auscultated. No passage of gas occurs. Uterine
mertia. What is the further tactics of labor
fundus is located at the level of the navel. The
management?
uterus is pamful on palpation. The discharge is
moderate and contains blood and pus. What is the F. Outlet forceps
most likely diagnosis? G. Labour stimulation
H. Cesarean section
F. Diffuse peritonitis
I. Skin-head Ivanov’s forceps
G. Metroendometritis
J. Vacuum extraction of the fetus
H. Progressive thrombophlebitis
L Pelvic peritonitis 6. A woman is 40. weeks pregnant.
J. Parametritis The fetus is in the longitudinal lie and cephalic
presentation. Pelvic size: 26-29-3120. Expected
3. A 58-year-old woman came to the
weight of the fetus is 4800 gram. The labor
gynecological clinic. She complains of bloody
contractions has been lasting for 12 hours, within
discharge from her genital tracts. Menopause 1s 8
the last 2 hours they were extremely painful, the
years. Gynecological examination: the uterus is
parturient woman is anxious. The waters broke 4
slightly enlarged, dense to touch, with limited
hours ago. On external examination the
mobility, the uterine appendages cannot be
contraction ring is located 2 finger widths above
detected, parametrium is free. Fractional
the navel, Henkel-Vasten sign is positive. Fetal
curettage of the uterine cavity yields a significant
heart rate is 160/min., muffled. On internal
amount of medullary substance in the scrape.
examination the uterine cervix is fully open, the
What 1s the most likely diagnosis?
head is engaged and pressed to the entrance into
F. Uterine corpus cancer the lesser pelvis. What is the most likely
G. Adenomyosis diagnosis?
H. Chorioepithelioma
F. Threatened uterine rupture
L_ Uterine cervix cancer
G. Complete uterine rupture D. Dysfunctional uterine bleeding
H. Hyperactive uterine contractions E. Adenomyosis
L Abruption of the normally positioned F. Ectopic pregnancy D. Submucous uterine
placenta myoma
J. Anatomically contracted pelvis E. Cancer of the uterine body

7. A 23-year-old woman came to the 11. A 25-year-old woman was brought into the
gynecological clinic. She complains of pain, gynecological department with profuse bloody
itching, and burning in her vulva, general discharge from her genital tracts. She is 12 weeks
weakness, indisposition, elevated body pregnant, the pregnancy is planned. Within the
temperature up to 37.2°C, and headache. On last 3 days she was experiencing pains in her
examination in the vulva there are multiple lower abdomen that eventually started resembling
vesicles up to 2-3 mm in diameter with clear cramps, she developed bleeding. Her skin is pale,
contents against the background of hyperemia pulse - 88/min., blood pressure - 100/60 mm Hg,
and mucosal edema. Make the provisional body temperature - 36.8°C. Vaginal examination:
diagnosis: the uterus size corresponds with 11 weeks of
pregnancy, the cervical canal allows inserting 1
F. Genital herpes infection
finger and contains fragments of the fertilized
G. Primary syphilis
ovum, the discharge is bloody and profuse. What
EH. Papillomavirus infection
is the most likely diagnosis?
L Vulvar cancer
J. Cytomegalovirus infection D. 12-week pregnancy, spontaneous
abortion in progress
8. A woman with the pregnancy term of 8 weeks
E. 12-week pregnancy,
complains of elevated temperature up to 37.6°C,
threatened
skin rash that can be characterized as macular
spontaneous abortion
exanthema, enlargement of posterior cervical and
F. Disturbed menstrual cycle,
occipital lymph nodes, small amount of bloody
hyperpolymenorrhea D. Disturbed menstrual
discharge from the genital tracts. She was
cycle, amenorrhea
examined by the infectious diseases specialist and
E. Full-term pregnancy, term labor
diagnosed with rubella. What tactics should the
obstetrician-gynecologist choose? 12. A 30-year-old woman came to the
gynecological department. She complains of
F. Abortion
sharp pain in her lower abdomen and temperature
G. Prescription of antibacterial therapy
of 38.8°C. She has a history of extramarital sexual
H. Prescription of antiviral therapy
L Treatment of incipient abortion activity and 2 = artificial abortions. On
gynecological examination the uterus is
J. Prescription of hemostatic therapy
unchanged. The appendages are bilaterally
9. A 16-year-old girl has primary amenorrhea, no enlarged and painful. Profuse purulent discharge
pubic hair growth, normally developed mammary is being produced from the vagina. What
glands; her genotype is 46 XY; uterus and vagina examination needs to be conducted to clarify the
are absent. What is your diagnosis? diagnosis?
C. Testicular feminization syndrome F. Bacteriological and bacterioscopic analysis
D. Mayer-Rokitansky-Kuster-Hauser syndrome G. Hysteroscopy
C. Cushing syndrome H. Curettage of the uterine cavity
F. Sheehan syndrome
I. Colposcopy
G. Cushing disease
J. Laparoscopy
10. A 46-year-old woman came to the maternity
13. It is the 3rd day after the first normal term
clinic with complaints of moderate blood
labor; the infant 1s rooming-in with the mother
discharge from the vagina, which developed after and is on breastfeeding. Objectively: the mother’s
the menstruation delay of 1.5 months. On vaginal
general condition is satisfactory. Temperature is
examination: the cervix is clean; the uterus is not
36.4°C, heart rate is 80/min.,
enlarged, mobile, painless; appendages without
BP is 120/80 mm Hg. Mammary glands are soft
changes. Make the diagnosis:
and painless; lactation is moderate, unrestricted
milk flow. The uterus is dense, the uterine fundus A. Sheehan syndrome (postpartum pituitary gland
is located 3 finger widths below the navel. Lochia necrosis) B. Physiological amenorrhea
are sanguino-serous, moderate in volume. Assess C. Suspected progressing ectopic pregnancy D.
the dynamics of uterine involution: Stein-Leventhal syndrome (polycystic ovary
syndrome) E. Galactorrhea-amenorrhea syndrome
F. Physiological involution
G. Subinvolution 17. A 45-year-old woman came
H. Lochiometra to the maternity clinic with complaints of
periodical pains in her mammary glands that start
L Pathologic involution
1 day before menstruation and stop after the
J. Hematometra menstruation begins. Palpation of the mammary
glands detects diffuse nodes predominantly in
14. A 28-year-old woman complaining of
the upper outer quadrants. What is the most likely
irregular menstruations and infertility came to the diagnosis?
gynecological clinic. Menstruations occur since
the age of 15, irregular, with delays up to 2 F. Fibrocystic mastopathy
months. On examination she presents with G. Breast cancer
marked hirsutism and excessive body weight. On H. Mastitis
vaginal examination the uterus is reduced in size I. Hyperprolactinemia
and painless. The ovaries on the both sides are J. Breast cyst
dense and enlarged. Ultasound shows microcystic
18. A 14-year-old girl came to the general
changes in the ovaries, the ovaries are 5x4 cm and
practitioner with complaints of weakness, loss of
4.5x4 cm in size with dense ovarian capsule.
appetite, headache, rapid fatigability. Her last
Basal body temperature is monophasic. What is
menstruation was profuse and lasted for 14 days
the most likely diagnosis?
after the previous delay of 2 months. Objectively:
F. Polycystic ovary syndrome the skin is pale, heart rate is 90/min., BP is 110/70
G. Krukenberg tumor mm Hg, Hb is 88 g/L. Rectal examination: the
H. Endometrioid cysts
uterus and its appendages are without changes, no
L_ Bilateral adnexitis
discharge from the genital tracts. What
J. Ovarian carcinoma
complication occurred in the patient?
15. An 18-year-old girl was brought into the C. Posthemorrhagic anemia
gynecology deparment with complaints of D. Somatoform autonomic dysfunction of
elevated body temperature up to 37.8°C, sharp hypotonic type C. Migraine
pain in her lower abdomen, more intense on the F. Gastritis
right, and difficult defecation. Vaginal G. Dysmenorrhea
examination detected a painful dense elastic
formation 5x6 cm in the area of her night ovary. 19. A 22-year-old postparturient woman on the
Pregnancy test is negative. What is the most 12th day after the normal childbirth informs of
likely diagnosis? elevated body temperature up to 39°C for the last
3 days and pain in her right mammary gland. The
F. Torsion of ovarian tumor pedicle right mammary gland is enlarged, hot to touch,
G. Ectopic pregnancy tense, hyperemic, and painful. Palpation reveals
H. Appendicitis there a dense infiltration 8x8 cm with a
IL Ovarian cyst rupture fluctuation in its center. What is the most likely
J. Ovarian apoplexy diagnosis?
16. A 26-year-old woman presents with F. Postpartum period, day 12. Right-sided
amenorrhea. 10 months ago she gave birth for a infiltrative-purulent mastitis
second time. In her early postpartum period she G. Postpartum period, day 12. Right-sided serous
developed a massive hypotonic hemorrhage. No mastitis
breasfeeding. Lately she has been presenting with H. Postpartum period, day 12. Right-sided
loss of weight, loss of hair, and indisposition. gangrenous mastitis
Gynecological examination revealed atrophy of I. Postpartum period, day 12. Right-sided
the external genitals, the uterus is abnormally phlegmonous mastitis
small, no uterine appendages can be detected. J. Postpartum period, day 12. Right-sided
What 1s the most likely diagnosis? lactostasis
20. A 35-year-old pregnant woman with degree 1 sounds detects bradycardia. Evaluation of
essential hypertension, developed edemas and cardiotocogram yielded the following data:
headache at the 33 week of her pregnancy. decrease of basal heart rate down to 90/min.,
Objectively her general condition is satisfactory, variability - monotonous (2 and less); late
blood pressure - 160/100 mm Hg, normal uterine decelerations with amplitude of 50/min. Make the
tone. Fetal heart rate 1s 140/min., rhythmic. She diagnosis and choose the obstetrical tactics
was diagnosed with daily proteinuria - 4 g/L, necessary in this case:
daily diuresis - 1100 mL. Creatinine - 80
C. Fetal distress. Urgent cesarean section
memol/L, urea - 7 mmol/L, platelets - 100 -
delivery
109/L. What complication of pregnancy
D. Fetal distress. Vacuum extraction delivery
occurred?
C. Normal condition of the fetus. Vaginal
F. Moderate preeclampsia birth
G. Severe preeclampsia F. Fetaldistress.Stimulation of uterine
H. Mild preeclampsia contractions
L Hypertensive crisis G. Fetal distress. Forceps delivery
J. Renal failure 24. A 27-year-old woman complains of foul-
smelling discharge from her genital tracts, pain in
21. A 24-year-old pregnant woman on her 37th
her lower abdomen, and elevated temperature.
week of pregnancy has been brought to the
The complaints arose 2 days ago. She has a
maternity obstetric service with complaints of
history of surgical abortion at the term of 8 weeks
weak fetal movements. Fetal heartbeats are
one week ago. Mirror examination: the uterine
95/min. On vaginal examination the uterine
cervix is clear, external orifice produces
cervix 1s tilted backwards, 2 cm long, external
foulsmelling discharge. Vaginal examination: the
orifice allows inserting a fingertip. Biophysical
uterus lies in anteflexion, is mobile, painful, and
profile of the fetus equals 4 pomts. What tactics
slightly enlarged. The appendages are without
of pregnancy management should be chosen?
changes. Make the provisional diagnosis:
A. Urgent delivery via a cesarean section B.
Treatment of placental dysfunction and repeated F. Postabortal endometritis
analysis of the fetal biophysical profile on the next G. Enterocolitis
day H. Appendicitis
F. Doppler measurement of blood velocity in I. Acute respiratory disease
the umbilical artery J. Salpingoophoritis
G. Urgent preparation of the uterine cervix
25. A 17-year-old girl has made an appointment
for delivery
with the doctor. She plans to begin her sex life.
H. Treatment of fetal distress; if ineffective,
No signs of gynecological pathology were
then elective cesarean section on the next day
detected. In the family history the patient’s
22. During regular preventive gynecological grandmother had cervical cancer. The patient was
examination a 30-year-old woman was detected consulted about the maintenance of her
to have dark blue punctulated perforations” on reproductive health. What recommendation will
the vaginal portion of the uterme cervix. The be the most helpful for prevention of invasive
doctor suspects endometriosis of the vaginal cervical cancer?
portion of the uterine cervix. What investigation
E. Vaccination against human papillomavirus
method would be most informative for diagnosis
(HPV)
confirmation?
F. Vitamins, calcium, omega-3
F. Colposcopy, target biopsy of the cervix G. Immunomodulators
G. US of the lesser pelvis H. Antiviral and antibacterial drugs E. Timely
H. Hysteroscopy treatment of sexually transmitted
L Curettage of the uterine cavity diseases
J. Hormone testing

23. A parturient woman is 30 years old, stage I of


the labor is ongoing. The fetus is in the cephalic
presentation. Auscultation of the fetal heart
1. A 38-year-old woman works in flax increased by 10 cm within a year (which is double
processing, she dries flax. She came to the the norm for her age group), the number of
hospital complainng of difficult breathing, permanent teeth corresponds with the age norm
constricting sensation in her chest, and cough (20), the development of her secondary sex
attacks. These signs appear on the first day of her characteristics is three years ahead of her age (Ma,
working week and gradually diminish on the P, Ax, Menarche). Development rate ahead of her
following days. What respiratory disease 1s likely biological age can occur due to:
in this case? F. Endocrine disorders
G. Acceleration
F. Byssinosis
H. Certain components of her diet
G. Silicosis
I. Sports training
H. Allergic rhinopharyngitis
J. Deficient hygienic education
I. Bronchial asthma
J. Asthmatic bronchitis 6. Employees work in conditions of high dust
concentration. Certain chemical (silicon dioxide
2. A district doctor has diagnosed one of his
content) and physical properties of dust aerosols
patients with dysentery. What accounting
contribute to the development of occupational
document reflects this type of morbidity?
dust-induced diseases. What is the main physical
F. Urgent report property of dust aerosols?
G. Statistical report
F. Dispersion
H. Report on a major non-epidemic disease
G. Magnetization
L Certificate of temporary disability
H. Electric charge
J. Control card of a patient registered for regular
I. Solubility
check-ups
J. Ionization
3. A 39-year-old man, a battery attendant,
7. In the factory cafeteria
suddenly developed weakness, loss of appetite, there was an outbreak of food poisoning.
nonlocalized colicky abdominal pains, and Clinical presentation —_ indicates
nausea. Objectively his skin is gray; there is a staphylococcal etiology of this disease. 15 people
pink-gray stripe on his gums; the stomach 1s soft are sick. To confirm the diagnosis of food
poisoning, samples need to be sent to the
and sharply painful. Blood test detected
laboratory. What samplesshould be
erythrocytes with basophilic stippling and obtained for analysis?
anemia. The patient has a history of peptic ulcer
disease of the stomach. Constipation tends to F. Vomit masses
occur every 3-4 days. What is the most likely G. Blood for hemoculture
provisional diagnosis? H. Blood (complete blood count)
I. Urine
F. Saturnism (lead poisosning) J. Saliva
G. Acute appendicitis
H. Perforation of gastric ulcer 8. In April during the medical examination of
L Acute cholecystitis various population groups, 27% of individuals
J. Chronic alcoholism presented with low working ability and rapid
fatigability. The following symptoms were
4. A 9-month-old infant presents with delayed observed in the affected individuals: swollen
tooth eruption and fontanel closure, weakness, friable gingiva that bleeds when pressed,
and excessive sweating. What type of hyperkeratosis follicularis not accompanied by
hypovitaminosis is the most likely in this child? skin dryness. These symptoms most likely result
F. Hypovitaminosis D from the following pathology:
G. Hypovitaminosis C F. C-hypovitaminosis
H. Hypovitaminosis B; G. Parodontosis
L Hypovitaminosis Bg H. A-hypovitaminosis
J. HypovitaminosisA I. Bi-hypovitaminosis
5. A 10-year-old girl exhibits high level of J. Polyhypovitaminosis
physical development ( + 30), her body length
9. Increased general morbidity women living in this locality. What type of
of the local population is observed in the medical examination is it?
area near a factory, where atmosphere is being
intensively polluted with sulfurous gas. What F. Target
effect does polluted air have on human body in G. Preliminary
this case?
H. Regular
A. Chronic nonspecific B. I. Complex
Acute specific J. Screening
C. Acute nonspecific D.
14. In the process of hiring, a prospective
Chronic specific
employee has undergone preventive medical
E. Selective examination and was declared fit to work in this
10. During medical examination of high and manufacturing environment. What type of
middle school students, the doctors vere assessing preventive medical examination was it?
correlation between biological and calendar age
F. Preliminary
of the school students based on the following
G. Scheduled
criteria: height growth rate per year, ossification
H. Periodical
of the carpal bones, the number of permanent
I. Specific
teeth. What additional development criterion
J. Comprehensive
should be assessed at this age?
15. On laboratory investigation of a pork sample
F. Development of secondary there is 1 dead trichinella detected in 24 sections.
sex
This meat should be:
characteristics
G. __ Body mass C. Sent for technical disposal
H. Chest circumference D. Allowed for sale with no restrictions C.
L Vital capacity of lungs Processed and sold through public catering
network
J. Hand strength
D. Processed for boiled sausage production E.
11. During analysis of morbidity in the city, it was Frozen until the temperature of -10°C is reached in
determined that age structure of population is the deep layers, with subsequent exposure to cold
different mm each district. What statistical method for 15 days
allows to exclude this factor, so that it would not
16. To assess the effectiveness of medical
skew the morbidity data?
technologies and determine the power and
F. Standardization direction of their effect on the public health
G. Wilcoxon signed-rank test indicators, the research was conducted to study
H. Correlation-regression analysis the immunization rate of children and measles
L Dynamic time series analysis incidence rate by district. What method of
J. Analysis of average values statistical analysis should be applied in this case?

12. Clinical _ statistical imvestigation was A. Calculation of correlation coefficient B.


performed to determine effectiveness of a new Calculation of morbidity index among the
pharmacological preparation for patients with nonvaccinated C. Calculation of matching factor
ischemic heart disease. What parametric test D. Calculation of standardized ratio E. Calculation
(coefficient) can be used to estimate the reliability of statistical significance of the difference between
of the results? two estimates

F. Student’s t-distribution 17. Having studied the relationship between the


G. Sign test distance from villages to the local outpatient
H. Matching factor clinics and frequency of visits to the clinics
L Wilcoxon signed-rank test among the rural population of this area, it was
J. Kolmogorov-Smirnov test determined that the rank correlation coefficient in
this case equals -0.9. How can this relationship be
13. In arural health care area there 1s an increasing
characterized?
cervical cancer morbidity observed. The decision
is made to conduct a medical examination of the F. Strong inverse relationship
G. Strong direct relationship
H. Moderate inverse relationship
1. A 25-year-old woman has been suffering
L Moderate direct relationship
from diabetes mellitus since she was 9. She was
J. -
admitted into the nephrology unit with significant
18. In the mpatient gynecological unit within a edemas of the face, arms, and legs. Blood pressure
year 6500 women underwent treatment. They - 200/110 mm Hg, Hb- 90 g/L, blood creatinine -
spent there a total of 102000 bed-days. What 850 memol/L, urine proteims - 1.0 g/L, leukocytes
indicator of the gynecological unit work can be - 10-15 in the vision field. Glomerular filtration
calculated based on these data? rate 10 mL/min. What tactics should the doctor
choose?
F. Average length of inpatient stay
A. Transfer into the hemodialysis unit B. Active
G. Average bed occupancy rate per year
conservative therapy for diabetic nephropathy C.
H. Number of beds by hospital department
Dietotherapy
L_ Bed turnover rate
H. Transfer into the endocrinology clinic
J. Planned bed occupancy rate per year
I. Renal transplantation
19. A middle school teacher with 4-yearlong
2. A 59-year-old woman was brought into the
record of work was issued a medical certificate
theumatology unit. Extremely severe case of
for pregnancy and childbirth leave. What amount scleroderma is suspected. Objectively she
of pay will she receive for the duration of her presents with malnourishment, ’*mask-like” face,
leave in this case? and acro-osteolysis. Blood: erythrocytes - 2.2 -
F. 100% of average salary 10°/L, erythrocyte sedimentation rate 40
mivhour. Urine: elevated levels of free
G. 50% of average salary
oxyproline. Name one of the most likely
H. 70% of average salary
pathogenetic links in this case:
L 60% of average salary
J. 80% of average salary E. Formation of antibodies to collagen
F. Formation of antibodies to native DNA C.
20. In the air of the feed kitchen at the poultry
Formation of antibodies to blood
factory, at the area where formula feed is being
corpuscles
mixed, the dust concentration reaches 200 mg/m3.
H. Formation of antibodies to transversely striated
Air microflora is represented predominantly by muscles
Aspergillus and Mucor fungi. What effect I. Formation of antibodies to vessel wall
determines pathogenic properties of the dust?
3. A 34-year-old man on the 3rd day of
F. Allergenic ceftriaxone treatment for acute otitis (daily
G. Teratogenic dosage - 2 grams) developed diarrhea occurring
H. Mutagenic 5-6 times per day. Feces are without mucus or
L Fibrogenic blood admixtures. Temperature is 36.6°C.
J. Toxic Gregersen reaction (occult blood in feces) is
negative. Stool culture detected no pathogenic
21. During regular medical examination a lyceum germs. What is the most likely cause of diarrhea
student presents with signs of cheilitis that in this case?
manifests as epithelial maceration in the area of lip
seal. The lips are brightred, with single vertical K. Antibiotic-associated diarrhea
cracks covered with brown-red scabs. These L. Intestinal dysbiosis
clinical signs are most likely caused by insufficient M. Bacterial overgrowth syndrome
content of the following in the diet: N. Ulcerative colitis
O. Crohn’s disease (regional enteritis)
F. Riboflavin
G. Ascorbic acid 4. A chronic alcoholic was hospitalized into the
H. Retinol therapeutic inpatient unit due to pneumonia. On
I. Thiamine
the day 5 of his hospitalization he became
disoriented in time and space, developed fear-
J. Calciferol
inducing visual hallucinations and motor
22. A 30-year-old woman made agitation. Full body tremor and tremor of the
an appointment with the family doctor for limbs are observed. X-ray and _ physical
examinations detect the signs of his 8. A 62-year-old patient has been hospitalized
convalescence from pneumonia. What tactics with complaints of pain in the thorax on the right
should be chosen regarding this patient? during breathing, dyspnea, and dry cough. Ten
I Transfer imto the inpatient days ago he slipped and fell hitting his right side.
narcology department On examuntaion: the patient lies on the left side.
J. Continue the treatment in the therapeutic The right side of the thorax lags during breathing.
department On the right there are crepitation and pain in the
Kk. Transfer into the I-IV ribs. Dullness of percussion sound and
neuroresuscitation department sharply dimimished breath sounds can be
L. Compulsory medical treatment observed. On X-ray: signs of exudate, fracture of
for the II-IV ribs. On pleurocentesis: blood is
alcoholism E. Discharge from the detected. Choose the further tactics:
hospital K. Transfer to a thoracic surgery department
5. After eating shrimps, a 25-year-old man L. Prescribe conservative therapy
suddenly developed skin itching, some areas of M. Perform repeated pleural taps
his skin became hyperemic or erupted into N. Apply a fixation bandage to the rib cage
vesicles. Make the diagnosis: O. Refer to a traumatologist

E. Acute urticaria 9. A 51-year-old man complains of vomiting with


F. Hemorrhagic vasculitis (Henoch-Schonlein blood. He has been drinking alcohol excessively.
purpura) C. Urticaria Health disorder has been observed since he was 40,
pigmentosa when he first developed jaundice. On examination
H. Psoriasis the skin and visible mucosa are icteric, with a
L Scabies stellate vascular pattern. The patient is
malnourished and presents with abdominal
6. A 25-year-old woman complains of fatigue, distension, umbilical hernia, and ascites. The edge
dizziness, hemorrhagic rashes on the skin. She of the liver is tapered and painless, +3 cm, the
has been presenting with these signs for a month. spleen is +2 cm. Blood test: Hb- 80 g/L, leukocytes
Blood test: erythrocytes - 1.0 - 1012/L, Hb- 37 - 3-10°/L, platelets - 85-109/L. What is the cause
g/L, color index - 1.1, leukocytes - of portal hypertension in this patient?
1.2-109/L, platelets - 42-10°/L. What analysis
would be the most advisable for diagnosismaking K. Hepatic cirrhosis
in this case? L. Thrombosis of the splenic vein
M. Hemochromatosis
K. Sternal puncture (bone marrow biopsy) N. Constrictive pericarditis
L. Splenic biopsy O. Budd-Chiari syndrome
M. Liver biopsy
N. Coagulation studies 10. A young man has made an appointment with
O. US of the gastrointestinal tract the dermatologist. He complains of a painful facial
rash in the beard and mustache area. This condition
7. A 35-year-old man complains of rapidly has been persisting for several weaks already.
incresing fatigue, palpitations, ’visual snow”, and After shaving, the patient’s condition aggravates.
dizziness. He has a history of peptic ulcer of the The diagnosis of sycosis is made. What primary
stomach. Objectively the skin is pale. Vesicular morphological elements can be observed in the
respiration is observed in the lungs. Systolic rash in this case?
murmur is detected over the cardiac apex, heart
rate is 100/min., BP is 100/70 mm Hg. The K. Pustules, papulae
epigastrium is slightly tender on palpation. Blood L. Nodes, nodules
test: erythrocytes - 3.2 - 1012/L, Hb- 100 g/L, M. Pustules, bumps
color index 0.95. What type of anemia is the most N. Phlyctenae, maculae
likely present in this case? O. Maculae, nodes

K. Posthemorrhagic anemia 11. At night a 63-year-old woman suddenly


L. Sideroblastic anemia developed an asphyxia attack. She has a 15year-
M. Chronic iron-deficiency anemia long history of essential hypertension and had a
N. Hemolytic anemia myocardial infarction 2 years ago. Objectively her
O. Hypoplastic anemia position in bed is orthopneic, the skin is pale, the
patient is covered with cold sweat, acrocyanosis is
observed. Pulse - 104/min. Blood pressure - 15. A 58-year-old man complains of weakness
210/130 mm Hg, respiration rate - 38/min. and tumor-like formations that appeared on the
Pulmonary percussion sound is clear, with slight anterior surface of his neck and in the inguinal
dullness in the lower segments, throughout the region. Palpation detects soft painless mobile
lungs single dry crackles can be heard that become cervical and inguinal lymph nodes up to 2 cm in
bubbling and non-resonant in the lower segments. diameter. The liver protrudes by 2 cm from the
What is the most likely complication in this edge of the costal margin, the lower splenic pole
patient? is at the umbilical level. In blood: erythrocytes -
K. Acute left ventricular failure 3.5 + 1012/L, Hb- 88 g/L, leukocytes -
L. Paroxysmal tachycardia
86 - 10°/L, band neutrophils - 1%, segmented
M. Bronchial asthma attack
neutrophils - 10%, lymphocytes - 85%,
N. Pulmonary embolism
eosinophils - 2%, basocytes - 0%, monocytes -
O. Acute right ventricular failure
2%, erythrocyte sedimentation rate - 15 mm/hour,
12. A 26-year-old woman is suspected to suffer Gumprecht shadows. What is the most likely
from systemic lupus erythematosus due to diagnosis?
systemic lesions of skin, vessels, joints, serous
tunics, and heart that developed after K. Chronic lymphatic leukemia
photosensitization. The following is detected: LE L. Lymphocytic leukemoid reaction
cells, antibodies to native DNA, isolated anti- M. Acute leukemia
centromere antibodies, rheumatoid factor is N. Chronic myeloleukemia
1:100, Wassermann reaction is _ positive, O. Lymphogranulomatosis
circulating immune complex is 120 units. What
16. A 23-year-old man complains of facial edemas,
immunological indicators are considered to be
headache, dizziness, low urinary output, and urine
specific to this disease?
discoloration (dark red). These complaints arose
K. DNA antibodies after a case of acute tonsillitis. On examination
L. Rheumatoid factor there are facial edemas, the skin is pale,
M. Anti-centromere antibodies temperature is 37.4°C; heart rate is 86/min., blood
N. Immunoglobulin A pressure is 170/110 mm Hg. Heart sounds are
O. Increased circulating immune complex muffled, the II heart sound is accentuated over the
aorta. What etiological factor is the most likely in
13. A woman came to the doctor with complaints this case?
of mcreased body temperature up to 37.8°C and
moderately sore throat for the last 3 days. K. Beta-hemolytic streptococcus
Objectively: mandibular lymph nodes are L. Staphylococcus aureus
enlarged up to 3 cm. Palatine tonsils are M. Streptococcus viridans
hypertrophied and covered with gray coating that N. Streptococcus pyogenes
spreads to the uvula and anterior pillars of the O. Staphylococcus saprophyticus
fauces. What is the most likely diagnosis?
17. An 18-year-old young man complains of pain
K. Oropharyngeal diphtheria in his knee and elbow joints and body temperature
L. Infectious mononucleosis up to 39.5°C. One week and a half earlier he
M. Pseudomembranous (Vincent’s) tonsillitis developed sore throat. On examination his body
N. Agranulocytosis temperature is 38.5°C. Swelling of the knee and
O. Oropharyngeal candidiasis elbow joints is observed. Pulse is 106/min.,
rhythmic. Blood pressure is 90/60 mm Hg. Cardiac
14. A 42-year-old man, a dispatcher, suffes from borders are unchanged, heart sounds are
peptic ulcer disease of the duodenum. The disease weakened, at the cardiac apex there is a soft
is of moderate severity. He wants to be assigned systolic murmur. What factor would be the most
a disability group. Make the conclusion regarding indicative of the likely disease etiology?
his working ability:
K. Anti-streptolysin O
K. Capable of working, employable L. C-reactive protein
L. Capable of working, non-employable M. Creatine kinase
M. First group of disability N. Rheumatoid factor
N. Second group of disability O. Seromucoid
O. Third group of disability
18. A woman has been provisionally diagnosed radiocarpal and metacarpophalangeal
with pheochromocytoma. At the stage of articulations and their reduced mobility in the
intermission her BP is within norm; there is a morning, which persisted up to 1.5 hours. Two
tendency towards tachycardia. No urine weeks ago she developed pain, swelling, and
pathologies. The decision has been made to reddening of her knee joints, her body
perform a provocative test with histamine. What temperature increased up to 37.5°C. The treatment
drug should be kept close at hand for emergency was untimely. Examination of the internal organs
aid in case of positive test result? revealed no pathologic alterations. Diagnosis of
rheumatoid arthritis was made. What changes are
K. Phentolamine
most likely to be visible on the arthrogram?
L. Pipolphen (Promethazine)
M. Nifedipine K. Joint space narrowing, usuration
N. Mesaton (Phenylephrine) L. Jomt space narrowing,
O. Prednisolone subchondral osteosclerosis
M. Cysts in the subchondral bone
19. A 40-year-old man with Bekhterev disease N. Numerous marginal osteophytes
(ankylosmg spondylitis) complains of elevated
O. Epiphyseal osteolysis
body temperature up to 37.8°C, back pain and
stiffness, especially observed during the second 22. A 52-year-old woman has been suffering for
half of the night. This condition has been lasting 2 years from dull, occasionally exacerbating pain
for 2 years. Objectively: reduced spinal mobility, in her right subcostal area, occurring after eating
painful sacroiliac joint, erythrocyte sedimentation high-fat foods, bitter taste in her mouth in the
rate - 45 mm/hour. X-ray shows narrowing of the morning, constipations, and flatulence.
intervertebral disc space and of the sacroiliac joint. Objectively she has excess weight, her body
What eye pathology is often associated with this temperature is 36.9°C; there is a coating on the
type of disease progression? root of her tongue; the abdomen is moderately
distended and painful in the area of gallbladder
K. Iridocyclitis projection. What examination would be the most
L. Retinal detachment helpful for diagnosismaking?
M. Cataract
N. Optic nerve atrophy K. Ultrasound
O. Blepharitis L. Duodenal intubation
M. Cholecystography
20. A 63-year-old man complains of unmotivated N. Duodenoscopy
weakness and pressing and bursting sensation in O. Liver scanning
the left subcostal area. According to him, these
signs have been present for a year already. 23. A 57-year-old woman complains of weakness,
Previously he was healthy. He took part in dyspnea, loss of appetite, and liquid feces. She has
containment measures during the accident at the been suffermg from this condition for 2 years.
Chornoby] Nuclear Power Plant. Objectively: the Objectively she presents with pale skin, subicteric
skin is pale, peripheral lymph nodes are not sclerae, and bright-red fissured tongue. Lymph
enlarged, the liver is +3 cm, the spleen is +10 cm. nodes are not enlarged. Pulse - 100/min. BP-
Complete blood count: erythrocytes - 3.1 - 105/70 mm Hg. Liver +3 em, the spleen cannot be
1012/L, Hb- 100 g/L, leukocytes - 46 - 109/L, palpated.
blasts - 2%, promyelocytes - 10%, myelocytes Blood test: erythrocytes - 1.2 - 1012/L, Hb- 56 g/L,
18%, band neutrophils - 27%, segmented color index - 1.4, macrocytes, leukocytes - 2,5 -
neutrophils - 10%, lymphocytes - 12%, 10°/L, eosinophils - 1%, juvenile - 1%,
eosinophils - 6%, basocytes - 3%, monocytes - metamyelocytes - 1%, band neutrophils - 8%,
2%, erythrocyte sedimentation rate - 20 mm/hour. segmented neutrophils 47%, lymphocytes - 38%,
What 1s the most likely diagnosis? monocytes - 4%, reticulocytes - 0.1%, platelets -
100 - 10°/L, ESR- 30 mm/hour, indirect bilirubin
K. Chronic myeloleukemia
- 26 mmol/L. What changes can be expected in the
L. Hepatic cirrhosis
bone marrow puncture material?
M. Acute leukemia
N. Hemolytic anemia K. Prevalence of megaloblasts
O. Chronic lymphatic leukemia L. Increased number of sideroblasts
M. Erythroid hyperplasia
21. For three years a 31-year-old woman has been
N. Presence of blast cells
complaining of pain and swelling of her
O. Prevalence of lymphoid tissue K. Hysterical neurosis
L. Hypochondriacal neurosis
24. A 35-year-old man _ suffers from
M. Depressive neurosis
insulindependent diabetes mellitus and chronic
N. Obsessive neurosis
cholecystitis. He takes NPH insulin: 20 units in the
morning and 12 units in the evening. After a meal O. Paranoid personality disorder
he developed pain in the right subcostal area, 28. A woman with atopic bronchial asthma was
nausea, vomiting, sleepiness, and increased found to have one allergen to dog hair +++.
polyuria. What prehospital measures will be the Carpets were removed from the apartment, the
most effective for prevention of crisis within the apartment was renovated, and air conditioner was
next several hours? installed. However, recurrent asphyxia attacks
still occur every night, despite the patient
K. Change insulin regimen
undergoing pathogenetic therapy. What long-
L. Take analgesics
term treatment tactics can help this patient to
M. Take cholagogues
decrease her sensitivity to the allergen?
N. Exclude fats from the diet
O. Decrease carbohydrates in the diet K. Specific hyposensitization
L. Continuation of prior treatment
25. A 45-year-old woman complains of intolerable
M. Antihistamine therapy
paroxysmal facial pain on the left with attacks that
N. Buteyko breathing technique
last for 1-2 minutes. Attacks are provoked by
O. Referral for speleotherapy
chewing. The disease onset was two months ago
after the overexposure to cold. Objectively: pain at 29. A 20-year-old man was hospitalized on the 9th
the exit points of the trigeminal nerve on the left. day of the disease. He attributes his disease to
Touching near the wing of the nose on the left eating of insufficiently thermally processed pork.
induces a pain attack with tonic spasm of the facial At its onset this condition manifested as
muscles. What is the most likely diagnosis? periorbital edemas and fever. Objectively his
K. Trigeminal neuralgia body temperature is 38.5 C. The face is puffy and
L. Glossopharyngeal neuralgia the eyelids are markedly swollen. Palpation of
M. Temporomandibular joint arthritis gastrocnemius muscles 1s sharply painful. Blood
N. Facial migraine test shows hypereosinophilia. What is the
etiology of this disease?
O. Maxillary sinusitis
K. Trichinella
26. A 28-year-old man complains of skin rash and
itching on the both of his hands. The condition L. Trichuris
persists for 1.5 years. The exacerbation of his M. Ascarididae
condition he ascribes to the occupational contact N. Echinococci
with formaldehyde resins. Objectively the lesion O. Leptospira
foci are symmetrically localized on both hands.
30. A 40-year-old man claims that his wife is
Against the background of erythema with blurred
cheating on him and presents a ’proof” of her
margins there are papulae, vesicles, erosions,
infidelity. He repeatedly initiated scandals with his
crusts, and scales. What is the most likely
wife at home and at work, demanding that she
pathology?
confess her infidelity, insulted her, and threatened
K. Occupational eczema to kill her. What preventive measures should be
L. Idiopathic eczema taken against socially dangerous actions on his
M. Allergic dermatitis part?
N. Simple contact dermatitis K. Consultation with the psychiatrist
O. Erythema multiforme L. Outpatient treatment
27. A 20-year-old student after failing an exam M. Consultation with the general practitioner
developed complaints of a sensation of a round N. Consultation with the psychologist
foreign body in her throat, difficult swallowing. O. Family counseling
She fixates on her condition, limits her diet, often
31. A 55-year-old woman complains of pain and
cries, seeks attention, exhibits demonstrative
popping sounds in her left knee jomt, which occur
attitude. She is highly susceptible to
when she climbs the stairs. Occasionally during
psychotherapeutic suggestion. What psychiatric
movements her joint becomes ”stuck”. 5 years ago
diagnosis can be made in this case?
she suffered a trauma of her left knee. Complete
blood count and biochemical blood analysis show N. Pseudotuberculosis
normal results. X-ray shows marked osteosclerosis O. Trichinosis
and osteophytes. The joint space is narrowed.
Make the provisional diagnosis: 35. A woman has been working as a polisher for
a year and a half. Her workstation is equipped
G. Osteoarthritis with a grinding machine (grinding wheels). She
H. Rheumatoid arthritis complains of white discoloration of her fingers
IL Gouty arthritis D. Psoriatic arthritis and toes that appears when she is nervous.
E. Reactive arthritis Objectively there are no changes in the coloration
of the distal segments of her limbs. Grip strength
32. A 40-year-old man, a welder, uses manganese measured with a dynamometer is 25 kg,
electrodes in his line of work (18 years of algesimetry fmdings are 0.1; 0.3; 0.5. Cold
experience). He complains of difficulties with stimulus is extremely positive on the upper and
writing, bad mood, inertness, gait abnormalities, lower limbs. Internal organs are without
problems with speech, and hand tremors. pathologies. Make the diagnosis:
Objectively the following is observed in the
patient: hypomimia, mcreased muscle tone of K. Vibration disease
plastic type, and quiet monotonous speech, tremor L. Raynaud disease
of the tongue, pill-rolling tremor of the fingers, and M. Syringomyelia
retropulsion. What syndrome developed in this N. Raynaud syndrome
patient due to manganese poisoning? O. Polyneuritis
K. Parkinsonism 36. A 37-year-old man suddenly developed acute
L. Meningism headache accompanied by nausea, vomiting, and
M. Hypothalamic syndrome impaired consciousness. Objectively blood
N. Polyneuritic syndrome pressure is 190/120 mm Hg, the face is
O. Vestibular syndrome hyperemic. Patient’s consciousness is clouded,
his answers to the questions are short,
33. A 45-year-old man developed constricting monosyllabic. Movement and — sensory
retrosternal pain that occurs during walks at the disturbances are absent. Meningeal signs are
distance of 200 m. Objectively: heart rate is positive. Cerebrospinal fluid contains blood.
80/min., BP is 160/00 mm Hg. During What provisional diagnosis can be made?
cardiopulmonary exercise test at 50 W there is a K. Subarachnoid hemorrhage
depression of S-T segment by 3 mm below the L. Meningitis
isoline in V3-V4. What is the provisional
M. Ischemic stroke
diagnosis?
N. Encephalitis
L Exertional angina pectoris, functional O. Cerebral vascular embolism
class II
37. A woman undergoing in-patient treatment for
J. Exertional angina pectoris, functional
viral hepatitis type B developed headache, nausea,
class IV
recurrent vomiting, memory lapses, flapping
K Exertional angina pectoris, functional
tremor of her hands, and rapid pulse. Sweet smell
class II
from her mouth is detected. Body temperature is
L. Somatoform autonomic dysfunction, 37.6°C, heart rate is 89/min. What complication
hypertension type E. Alcoholic developed in the patient?
myocardiody strophy
K. Acute liver failure
34. A 45-year-old man, a farmer, presents with
L. Ischemic stroke
acute onset of a disease. He complains of
M. Gastrointestinal hemorrhage
headache, high temperature, pam in_ the
N. Hypoglycemic shock
gastrocnemius muscles, icteric face, and dark
O. Meningoencephalitis
urine. Objectively: body temperature - 38°C,
blood pressure - 100/70 mm Hg, conjunctival 38. A 43-year-old man, a coal-face worker with 15-
hemorrhages, hepatosplenomegaly, and oliguria. year-long record of work, complains of cough,
What is the most likely provisional diagnosis? thoracic pain, and dyspnea. The cough is mild,
usually dry, occurs mostly in the morning. The
K. Leptospirosis
pain is localized in the interscapular region and
L. Brucellosis
aggravates during a deep intake of breath. Dyspnea
M. Viral hepatitis
occurs during physical exertion. Vesicular I. Hypertensive crisis complicated with instable
respiration in the lungs is weakened. Heart sounds angina pectoris
are rhythmic, heart rate 1s 86/min., blood pressure J. Hypertensive crisis complicated with acute left
is 135/80 mm Hg. The abdomen is soft and ventricular failure
painless. X-ray shows micronodular pulmonary K. Pulmonary embolism
fibrosis. Make the provisional diagnosis:
42. A 35-year-old patient developed an epileptic
K. Carboconiosis attack with tonoclonic spasms that lasted for 3
L. Byssinosis minutes. After the attack the patient fell asleep but
M. Siderosis in 5 minutes the second attack occurred. The first
N. Berylliosis step of emergency aid would be to:
O. Metal pneumoconiosis
K. Ensure patency of airways
39. Having examined a 52-year-old patient, the L. Take blood from the vein for analysis
doctor diagnosed him with obesity (body mass M. Introduce diazepam intravenously
index - 34 kg/m?, waist circumference - 112 cm) N. Prescribe antiepileptic drugs
and arterial hypertension (170/105 mm Hg). 2- O. Administer chloral hydrate via an enema
hour postprandial blood sugar is 10.8 mmol/L.
What biochemical blood analysis needs to be 43. A 27-year-old woman, a teacher in the
conducted to diagnose the patient with metabolic elementary school, complains of frequent stools,
syndrome X? up to 3 times per day, with lumpy feces and large
amount of mucus, abdominal pain that gradually
K. Lipid profile abates after a defecation, irritability. Her skin is
L. Bilirubin pale and icteric. Pulse is 74/min., rhythmic, can
M. Calcium and phosphorus be characterized as satisfactory. Blood pressure is
N. Creatinine and urea 115/70 mm Hg. The abdomen is soft, moderately
O. Electrolytes tender along the colon on palpation. Fiberoptic
colonoscopy detects no changes. What disease
40. After overexposure to cold a 45year-old can be suspected?
woman developed acute pain in her suprapubic and
lumbar areas during urination, sharp pains at the K. Irritable bowel syndrome
end of urmation, false urges to urinate. Urine is L. Chronic non-ulcerative colitis
turbid with blood streaks. The doctor suspects M. Chronic enteritis
urinary tract infection. What results of laboratory N. Crohn disease (regional enteritis)
analysis would be the most indicative of such O. Whipple disease
infection?
K. Leukocyturia, gross hematuria 44. A 72-year-old man complains of lower
extremity edema, sensation of heaviness in the
L. Gross hematuria
right subcostal area, dyspnea at rest. For over 25
M. Increased blood creatinine and blood urea
years he has been suffermg from COPD.
N. Daily proteinuria under 3.0
Objectively: orthopnea, jugular venous
O. Daily proteinuria over 3.0
distention, diffuse cyanosis, acrocyanosis. Barrel
41. A 62-year-old woman was brought into the chest is observed, on percussion there is a
admission room with complaints of severe vesiculotympanitic
burning retrosternal pain and asphyxia. She has a (bandbox) resonance, sharply weakened vesicular
10-year-long history of essential hypertension. respiration on both sides, moist crepitant crackles
Objectively her condition is moderately severe. in the lower segments of the lungs. Heart sounds
She presents with skin pallor, cyanotic lips, and are weakened, the II heart sound is accentuated
vesicular respiration over her lungs. The II heart over the pulmonary artery. The liver is +3 cm.
sound is accentuated over the aorta. Blood What complicated the clinical course of COPD in
pressure - 210/120 mm Hg, heart rate (pulse) - this patient?
76/min. ECG shows elevation of ST segment in
K. Chronic pulmonary heart
the leads I, AVL, and V5-V6. What is the most
L. Pulmonary embolism
likely diagnosis?
M. Acute left ventricular failure
A. Hypertensive crisis complicated with acute N. Diffuse pneumosclerosis
myocardial infarction B. Uncomplicated O. Community-acquired pneumonia
hypertensive crisis
45. A 72-year-old man with pneumonia murmur; the sound of opening mitral valve can be
complains of marked dyspnea, chest pain, severe auscultated at the cardiac apex. The IT heart sound
cough with expectoration, f¢ is 39.5-40°C, no is accentuated over the pulmonary artery. The
urination for a whole day. Objectively the patient patient is cyanotic. X-ray shows dilated
is conscious. Respiratory rate is 36/min. Over the pulmonary root and enlargement of the right
right lower pulmonary lobe percussion sound is ventricle and left atrium. What is the most likely
dull; on auscultation there is bronchial respiration diagnosis?
and numerous moist crackles. Blood pressure is
K. Mitral stenosis
80/60 mm Hg. Heart rate is 120/min. Heart
L. Aortic stenosis
sounds are muffled, there is tachycardia. What
tactics should the family doctor choose in the M. Pulmonary artery stenosis
management of this patient? N. Coarctation of the aorta
O. Patent ductus arteriosus
K. Hospitalization into the intensive care unit
L. Outpatient treatment 49. A 23-year-old man complains of severe pain
M. Treatment in the day patient facility in his left knee joint. Objectively the left knee
N. Hospitalization into the pulmonology unit joint is enlarged, with hyperemic skin, painful on
palpation. Complete blood count: erythrocytes -
O. Hospitalization into the neurology unit
3.8 - 1012/L, Hb- 122 g/L, leukocytes - 7.4 -
46. 2 hours after eating unknown mushrooms, a 10°/L, platelets 183 109/L. Erythrocyte
28-year-old man sensed a decrease in his mobility sedimentation rate
and deterioration of his ability to focus. This
condition was then followed by a state of - 10 mm/hour. Bleeding time (Duke method) - 4
agitation and agression. On examiantion he is min., Lee-White coagulation time - 24 mun.
disoriented and his speech is illegible. 4 hours Partial thromboplastin time (activated) - 89
later he developed fetor hepaticus and lost his seconds. Rheumatoid factor - negative. What is
consciousness. What syndrome can be observed the most likely diagnosis?
in this patient? E. Hemophilia, hemarthrosis
K. Acute hepatic failure F. Werlhof disease (immune
L. Hepatolienal syndrome thrombocytopenia) C.
M. Portal hypertension Rheumatoid arthritis
N. Cholestatic syndrome H. Thrombocytopathy
O. Cytolytic syndrome I. Hemorrhagic vasculitis (Henoch-Schonlein
purpura), articular form
47. A 36-year-old man complains of marked
dyspnea and cardiac pain. He ascribes his disease 50. A 24-year-old woman, a kindergarten teacher,
to the case of influenza that he had 2 weeks ago. has been sick for 2 days already. Disease onset
Objectively he leans forward when sitting. The was acute. She presents with elevated body
face is swollen, cyanotic, cervical veins are temperature up to 38.0°C, pain attacks in her
distended. Heart borders are extended on the both lower left abdomen, liquid stool in small amounts
sides, heart sounds are muffled, heart rate = Ps = with blood and mucus admixtures 10 times a day.
118/min., BP is 90/60 mm Hg. Blood test: ESR is Pulse - 98/min., blood pressure - 110/70 mm Hg.
46 mm/hour. ECG shows low voltage. Xray Her tongue is moist and coated with white
shows trapezoidal cardiac silhouette and signs of deposits. The abdomen is soft, the sigmoid colon
pulmonary congestion. Choose the treatment is painful and spastic. Make the provisional
tactics: diagnosis:

K. Pericardial puncture (pericardiocenthesis) K. Shigellosis


L. Diuretics L. Escherichiosis
M. Antibiotics M. Salmonellosis
N. Pericardectomy N. Yersiniosis
O. Glucocorticosteroids O. Rotavirus infection
51. A 38-year-old woman complains of weakness,
48. A 39-year-old man suffers from chronic sleepiness, pain in the joints, weight gain despite
rheumatic heart disease. He complains of dyspnea low appetite, and constipations. She presents with
during physical exertion, cough with dry and thickened skin, puffy and amimic face,
expectoration, and palpitations. Ausculation narrowed palpebral fissures, thick tongue, and
detects intensified I heart sound and diastolic
deep hoarse voice. Her heart sounds are weak, last 3 weeks he has been suffering from
pulse is 56/min. Low levels of free T4 are headaches of increasing intensity. Neurological
observed. This patient needs to take the following examination detects nuchal rigidity without focal
on a regular basis: signs. Make the provisional diagnosis:

G. Thyroxine K. Tuberculous meningitis


H. Mercazolil (Thiamazole) L. Chorea minor
L_ Lithium carbonate D. Furosemide M. Brain tumor
E. Calcium gluconate N. Myelitis
O. Convexital arachnoiditis
52. A 23-year-old man has accidentally
swallowed brake fluid. After that he has been 56. A patient has gradually lost consciousness.
presenting with anuria for 5 days already; his The skin is pale and dry. There is a smell of
creatinine levels elevated up to 0.569 mmol/L. ammonia from the mouth. Respirations are deep
What treatment tactics should be chosen in this and noisy. Heart sounds are muffled, pericardial
case? friction rub is present. Blood pressure is 180/130
mm Hg. Blood test: Hb- 80 g/L, leukocytes 12 -
K. Hemodialysis
10°9/L, blood glucose - 6.4 mmol/L, urea 50
L. Detoxication therapy
mmol/L, creatinine - 1200 mcemol/L, blood
M. Antidotal therapy osmolarity - 350 mOsmol/L. No urinary
N. Diuretics excretion. Make the diagnosis:
O. Plasmapheresis
K. Uremic coma
53. A 52-year-old man for the last 3 years has L. Hyperglycemic coma
been suffering from difficult swallowing of solid M. Acute renal failure
food, burning retrosternal pain that aggravated
N. Acute disturbance of cerebral circulation
during eating, loss of body mass, and occasional
O. Hyperosmolar coma
vomiting with undigested food. Esophageal X-ray
shows S-shaped deformation of the esophagus 57. A 72-year-old man diagnosed with ischemic
and its dilation, at the cardiac orifice the heart disease presents with diffuse
esophagus is constricted; esophageal mucosa is cardiosclerosis, permanent tachysystolic atrial
smooth, without signs of peristalsis. Make the fibrillation, heart failure Ha, FC II. Objective
provisional diagnosis: examination of vital signs: blood pressure is
135/80 mm Hg, heart rate is 160/min., pulse is
K. Esophageal carcinoma
125/min. Left ventricular ejection fraction is
L. Diaphragmatic hernia
32%. What drug is indicated in this case and
M. Esophageal achalasia should be presribed to the patient?
N. Reflux esophagitis
O. Esophageal diverticulum K. Digoxin
L. Procainamide (Novocainamide)
54. A 53-year-old man complains of general M. Isadrine (Isoprenaline)
weakness, loss of appetite, and painful vesicles
N. Verapamil
appearing on his skin. The disease onset occurred
O. Ivabradine
suddenly, after hyperinsolation one week ago.
Examination detects isolated vesicles with 58. A 34-year-old man complains of pale edema
wrinkled opercula and occasional painful of the face, feet, shins, and lumbar area, elevated
erosions on the skin of the patient’s torso and blood pressure up to 160/100 mm Hg, and general
limbs. Nikolsky sign is positive. What is the most weakness. He has a clinical history of nonspecific
likely diagnosis? ulcerative colitis. Objectively: pulse - 84/min.,
rhythmic, blood pressure - 165/100 mm Hg;
K Acantholytic pemphigus
edemas all over the body; the skin is pale and dry,
L. Nonacantholytic pemphigus with low turgor. The kidneys cannot be palpated,
M. Duhring’s disease (dermatitis on an attempt to palpate them they are painless.
herpetiformis) Blood test: erythrocytes - 3.0-1012/L, Hb- 100
N. Herpes g/L, erythrocyte sedimentation rate - 50 mm/hour.
Oo. Toxicodermia Urinalysis: protems 3.5 g/L, erythrocytes - 7-10
in the vision field, leukocytes - 5-6 in the vision
55. A patient is being treated
field. Daily proteinuria - 6 grams. What analysis
in the tuberculosis clic. Throughout the
should be conducted additionally to verify the O. Secondary papular syphilid
diagnosis?
62. A 38-year-old woman after physical
K. Gingival biopsy for the diagnosis of amyloid
disease overexertion suddenly developed palpitations,
dyspnea, and a dull pain in the cardiac area. For 10
L. Radioisotopic examination of kidneys
years she has been registered for regular check-ups
M. Urinalysis for Bence-Jones protem
due to rheumatism and mitral valve disease with
N. Renal ultrasound
non-disturbed blood circulation. Oblectively her
O. Survey and excretory urography
pulse is 96/min., of unequal strength. Blood
59. A 42-year-old man, a worker at the meat pressure is 110/70 mm Hg, heart rate 1s 120/min.
processing factory, developed an itching spot on ECG registers small unevenly-sized waves in place
his lower jaw, which gradually transformed into a of P-waves, R-R intervals are of unequal length.
slightly painful carbuncle 3 cm im diameter, What is the most likely diagnosis?
surrounded by a painless swelling that reaches the
K. Atrial fibrillation
clavicle. Temperature is subfebrile, under 37.8°C.
L. Paroxysmal supraventricular tachycardia
The doctor suspects anthrax. What drug should
M. Atrial flutter
this man be prescribed for treatment?
N. Paroxysmal ventricular tachycardia
K. Penicillin O. Respiratory arrhythmia
L. Levomycetin (Chloramphenicol)
63. An 18-year-old patient always obeys others
M. Biseptol (Co-trimoxazole)
and adapts his needs to the demands of the people
N. Interferon alpha
on whom he depends. He excessively defers to
O. Azidothymidin (Zidovudine) their wishes and makes them responsible for his
60. A 57-year-old patient complains of dyspnea at wellbeing, cannot defend his interests and needs
rest. The patient presents with orthopnea, support from other people. Such psychic profile
acrocyanosis, bulging cervical veins. On has been formed in the childhood, remains
percussion: dull sound over the lower lung unchanged, and hinders adaptation. What psychic
segments. On auscultation: no respiratory sounds. disorder is observed in this patient?
Heart rate is 92/min. Right-sided cardiac K. Dependent personality disorder
dilatation is observed. The liver is +7 cm. Shins
L. Anxiety (avoidant) personality disorder
are swollen. Pleural effusion is suspected. What
M. Anankastic personality disorder
indicator would confirm the presence of
N. Markedly accentuated personality
transudate in this case?
O. Psychopathy-like state
G. Total protein content in the pleural fluid below
64. A 45-year-old man with thrombophlebitis of
25 g/L
the deep veins in his legs suddenly after physical
H. Presence of atypical cells
exertion developed sharp pain in his thorax on the
L Total protein content in the pleural fluid
right, dyspnea, and hemoptysis. Objectively his
exceeding 30 g/L D. Specific gravity exceeding
condition is severe; he presents with
1015
acrocyanosis, shortening of pulmonary
E. Positive Rivalta’s test
percussion sound on the mght, and weakened
61. A 33-year-old man developed multiple rashes respiration. Respiration is 30/min., blood pressure
on the skin of his torso and extensor surfaces of is 110/80 mm Hg. ECG shows sinus tachycardia,
his upper and lower limbs. The rashes itch and heart rate is 120/min., electrical axis of the heart
occasionally fuse together and form plaques. The deviates to the right, S+Qy;. What is the most
elements of rash are covered with silver-white likely diagnosis?
fine scales that easily flake off when scratched.
A. Pulmonary embolism B. Community-
Grattage test results im three sequential acquired right-sided pneumonia C. Cancer
phenomena: stearin spot, terminal film, and of the right lung
punctate hemorrhage. What diagnosis can be H. Right-sided exudative pleurisy
suspected? I. Spontaneous pneumothorax
K. Psoriasis 65. A 48-year-old woman has been hospitalized
L. Parapsoriasis due to development of tachysystolic atrial
M. Pyoderma fibrillation. She has lost
N. Lichen ruber planus
5 kg of body weight within 2 months. On in the tongue on the right. No other neurological
palpation there is a node in the left lobe of the pathologies were detected. What disease can be
thyroid gland. What pathology resulted in the provisionally diagnosed in this patient?
development of this condition?
K. Neuropathy of the facial nerve
K. Toxic nodular goiter L. Neuropathy of the trigeminal nerve
L. Aterosclerotic cardiosclerosis M. Trigeminal ganglionitis
M. Chronic thyroiditis N. Neuropathy of the oculomotor nerve
N. Nontoxic nodular goiter O. Ischemic stroke
O. Autoimmune thyroiditis
70. A 56-year-old woman was diagnosed with
66. A 48-year-old woman developed insomnia, stage 2 hypertension of the 2nd degree. She
depressive mood, anxiety, fears and suicidal belongs to the group of moderate risk and has
thoughts after the death of her husband that bronchial asthma. What group of drugs is
occurred one month ago. During her stay in the CONTRAINDICATED to this patient?
hospital she speaks in a low voice, is depressed, K. B-blockers
anxious, avoids sleeping, refuses to eat. What L. Angiotensin-converting enzyme inhibitors
medications should be prescribed in this case? M. Diuretics
K. Antidepressants N. Calcium antagonists
L. Antipsychotics O. Imidazoline receptor antagonists
M. Group B vitamins 71. A 45-year-old woman 1s registered for regular
N. Nootropics check-ups due to Werlhof disease (immune
O. Anticonvulsants thrombocytopenia). Complete blood count: Hb-
100 g/L, erythrocytes 2.8-1012/L, platelets -
67. A 32-year-old woman complains of episodes
90.0-10°/L, leukocytes - 8.4 - 109/L, erythrocyte
of intense fear that occur without visible cause
sedimentation rate 13 mm/hour. Examination
and last for 10-20 minutes, the episodes are
detects a single small hematoma on the anterior
characterized by rapid pulse, sweating, labored
surface of the thigh, developed after the patient
breathing, and vertigo. Specify the likely
accidentally stumbled on a table. What treatment
diagnosis:
tactics should be chosen in this case?
K. Panic disorder
K. Continue the supervision by the hospital
L. Paranoid syndrome
hematologist
M. Manic syndrome
L. Urgent hospitalization into the
N. Simple schizophrenia hematology unit
O. Claustrophobia M. Urgently start a hemostatic therapy,
68. A 39-year-old man suffers from chronic followed by a planned hospitalization into the
adrenal insufficiency and receives replacement hematology unit
glucocorticoid therapy N. Urgent hospitalization mto the general
(hydrocortisone - 15 mg/day). He is to undergo care unit
elective surgery for calculous cholecystitis. What O. Administer thrombocytic mass, continue
medication adjustment should be made on the day the treatment in the hematology unit
of the surgery to prevent the development of acute
72. The dermatologist has an appointment with a
adrenal insufficiency?
30-year-old man that complains of severely
K. Increase the dosage by 2-3 times itching rashes that especially disturb him at night.
L. Cancel the drug for the day of the surgery The rashes developed 2 weeks ago, after he had
M. Add a mineralocorticoid returned from a travel. Objectively on the lateral
N. Add an antibiotic surfaces of his fingers, hands, wrists, elbows,
lower abdomen, genitals, and thighs there are
O. Prescribe a large volume intravenous fluid
paired papulovesicles, single pustules, and
infusion
scratch marks. What disease can be suspected?
69. After a long drive with the window open a
K. Scabies
man developed facial asymmetry; he cannot close
his right eye, his right nasolabial fold is smoothed L. Pyoderma
out, movements of expression are absent on the M. Dermatitis
right, there is a disturbance of gustatory sensation N. Eczema
O. Shingles creatorrhea. What prescription would be the most
advisable in this case?
73. A 38-year-old woman developed a medical
condition 7 days after her return from K. Multi-enzyme preparations
Bangladesh. Periodical elevation of temperature L. Cholinergic antagonists
was accompanied by chills and excessive M. Metronidazole and loperamide
sweating. She was diagnosed with tropical N. Antacids and antispasmodics
malaria. Next day her condition further O. Cholinergic antagonists and antibacterial
deteriorated: body temperature - 38°C, inertness, agents
periodical loss of consciousness, generalized
seizures, tachycardia, hypotension, and icteric 77. A man was brought into the admission room
skin. What complication can be suspected in this after an overexposure to cold. He complains of
case? sharp pain in the small of his back and elevated
K. Cerebral coma body temperature up to 38°C. He took some
L. Serous meningitis aspirin. Blood test: leukocytes - 10.5 - 1012/L,
M. Purulent meningitis eosinophils - 5%, band neutrophils - 8%,
N. Acute hepatic failure segmented neutrophils - 51%, lymphocytes -
O. Acute heart failure 32%, monocytes - 4%, erythrocyte sedimentation
rate - 28 mm/hour. Urinalysis: protein - 0.6 g/L,
74. A 73-year-old woman came to the family leukocytes - cover the whole vision field, large
physician for one of her regular follow-up amount of mucus. What is the most likely
examinations. Three months ago she was found to diagnosis?
have type 2 diabetes mellitus. She was keeping to
her diet and exercise plan and _ taking K. Acute pyelonephritis
phytopreparations. On examination her fasting L. Chronic pyelonephritis
glucose was within the range of 7.8-8.6 mmol/L, M. Acute glomerulonephritis
HbAlc - 7.9%. Height - 164 cm, weight - 83 kg. N. Tubulointerstitial nephritis
What blood sugar-controlling medicine should she O. Subacute malignant glomerulonephritis
be prescribed first in the course of her
78. A 26-year-old man complains of chills,
pharmacological therapy?
rhinitis, dry cough, and fever up to 38°C.
K. Metformin Examination shows him to be in a moderately
L. Glibenclamide severe condition; there are small pale pink non-
M. Glimepiride merging spots on the skin of his back, abdomen,
N. Gliclazide and extremities. Palpation reveals enlarged
O. Insulin
occipital and axillary lymph nodes. No
information about vaccination history could be
75. A 27-year-old man complains of pain in his leg obtained. What is the likely etiology of this
joints, purulent discharge from the eyes, and disease?
painful burning sensations during urination.
Disease onset was acute. He has a history of K. Rubella virus
influenza. The patient smokes and drinks alcohol L. Epstein-Barr virus
in excess. In his line of work he is often away on M. Streptococcus
business trips. What is the most likely etiological N. Mumps virus
factor of this disease? O. Neisseria meningitis

G. Chlamydia
H. Adenovirus
L Streptococci D. Staphylococci
E. Candida

76. A 46-year-old woman has diarrhea with


abdominal distension, loss of body mass, and large
amounts of porridge-like foulsmelling stool
without blood streaks or tenesmus. Objective
examination detects moderate tenderness in the
mesogastrium and left abdominal flank. Feces
analysis detects steatorrhea with neutral fat and
1. During medical examination a cadet in the fingers appeared. Determine the frostbite degree
naval college was detected to have a painless in this child:
dense ulcer 1.5x0.5 in size in his perianal area at K. Frostbite of the I degree
the 2 o’clock position. The ulcer floor resembles L. Perniosis
*old fat”. What is the provisional diagnosis? M. Frostbite of the IT degree
N. Frostbite of the III degree
E. Hard syphilitic chancre of the
O. Frostbite of the IV degree
rectum
F. Rectal fissure C. Rectal fistula 5. A 16-year-old patient has made an
H. Anal cancer appointment with an otolaryngologist. He
L Anal crypt suppuration complains of elevated body temperature and sore
throat. Disease onset was 2 days ago, after the
2. A 32-year-old woman complains of tumorlike
patient ate two portions of ice-cream.
formation on the anterior surface of her neck that
Pharyngoscopy shows hyperemic mucosa of the
appeared 2 years ago. Within the last 3 months the
palatine tonsils, with purulent exudate in the
tumor has been rapidly growing. It hinders
lacunae. Make the provisional diagnosis:
swallowing and impairs speech; the tumor causes
a sensation of pressure. Objectively the skin K. Lacunar tonsillitis
moisture is normal, pulse is 80/min., rhythmic, L. Follicular tonsillitis
blood pressure is 130/80 mm Hg. In the night lobe M. Diphtheria
of the thyroid gland there is a dense lumpy node N. Acute pharyngitis
3.0x3.5 cm that moves during swallowing. O. Pseudomembranous (Vincent’s) tonsillitis
Scanning image shows a “cold nodule” in the
6. A 35-year-old woman complains of high body
thyroid gland. Make the provisional diagnosis:
temperature and pain in the upper outer quadrant
K. Thyroid cancer of her right buttock, which developed after an
L. Thyroid adenoma injection. She has been presenting with this
M. Thyroid cyst condition for 3 days. At the site of injection the
N. Nodular goiter skin is hyperemic; there is a painful infiltrate
O. Autoimmune thyroiditis with an area of softening m its center. The
woman is diagnosed with a postinjection abscess
3. After a surgery for a left thigh phlegmon the
of the right buttock. What tactics should the
disease progression was complicated by sepsis.
surgeon choose in this case?
On the 7th day after the surgery there are marked
signs of a generalized inflammatory reaction, in K._ Abscess incision, sanation and drainage of
blood there are signs of toxic anemia and the cavity
progressing hypoproteinemia, bilirubin levels are L. Hospitalization, prescription of antibiotics,
40 memol/L, AST and ALT exceed the norm by UHF
2.5 times. Oliguria persists (700 mL of urine per M. Abscess puncture, pus removal followed by
day). Name the phase of sepsis progression: application of antiseptics
N. 10-15 minutes of low-intensity
G. Catabolic phase
laser radiation directed at the right
H. Stress phase
buttock
L Anabolic phase D. Recovery phase
O. Antipyretic agents, massage,
E. Mixed phase
and
4. A 10-year-old boy, who was outdoors in windy application of dry heat to the right buttock
and cold weather, developed moderate pain and
7. A 65-year-old woman on abdominal palpation
tingling in his fingers and toes. When he returned
presents with a tumor in the umbilical region and
home, his parents noticed that the tips of his
above it; the tumor is 13x8 cm in size,
fingers and toes were white and their sensitivity
moderately painful, nonmobile, pulsing. On
was lost. As the affected areas were warming up,
auscultation systolic murmur can be observed.
the fingers and toes developed tingling and painful
What is the most likely diagnosis?
sensations. Skin pallor changed into redness,
tingling stopped, mild itching and swelling of the K. Abdominal aortic aneurysm
L. Gastric tumor
M. Arteriovenous aneurysm K. Acute attack of glaucoma of the left eye
N. Tricuspid insufficiency L. Acute iridocyclitis of the left eye
O. Bicuspid insufficiency M. Stage II intraocular tumor of the left eye
N. Endophthalmitis of the left eye
8. A 32-year-old man complains of pain in his
O. Panophthalmitis of the left eye
legs that intensifies during walking, intermittent
claudication, numbness of his toes, extremity 11. On the 15th day after a small trauma of the
coldness, and inability to walk more that 100 right foot, the patient developed indisposition,
fatigability, irritability, headache,
meters. When he sleeps, his leg usually hangs
elevated body temperature, and sensation
down. The patient has been smoking since he of constriction, tension, and
was 16. He drinks alcohol in excess. The left leg twitching in the muscles of the right shin. What
is colder than the right one; the skin of the disease can be suspected?
extremities is dry. No pulse can be detected on
E. Tetanus
the pedal arteries, while pulsation of the femoral
F. Anaerobic gas gangrene C.
arteries is retammed. What is the most likely
Erysipelas
diagnosis?
EH. Acute thrombophlebitis
K. Obliterating endarteritis I. Thrombophlebitis of the popliteal artery
L. Diabetic angiopathy
12. A patient has the second and third degree burns
M. Leriche syndrome (aortoiliac occlusive
of the 15% of the body surface. On the 20th day
disease)
after the trauma the patient presents with sharp
N. Raynaud disease
increase of body temperature, general weakness,
O. Deep thrombophlebitis
rapid vesicular respiration, facial features are
9. A 50-year-old patient was brought to a hospital sharpened, BP is 90/50 mm Hg, heart rate is
with complaints of blood in urme. Urination is 112/min. What complication is 1t?
painless and undisturbed. Macrohematuria had
K. Sepsis
been observed for 3 days. Objectively: kidneys
L. Pneumonia
cannot be palpated, suprapubic area is without
M. Acute intoxication
alterations, external genitalia are nonpathologic.
N. Purulent bronchitis
On rectal investigation: prostate is not enlarged,
O. Anaerobic infection
painless, has normal structure. Cystoscopy
revealed no changes. What is the most likely 13. 2 hours after a traffic accident a 28-yearold
diagnosis? man in a grave condition was brought to a
hospital. The patient complains of abdominal
K. Renal carcinoma
pain. He received a blow to the abdomen with the
L. Bladder tuberculosis
steering wheel. Objective examination revealed
M. Varicocele
the following: the abdomen does not participate
N. Dystopic kidney
in respiration, is tense and acutely painful on
O. Necrotic papillitis
palpation; the abdominal muscles are defensively
10. A 59-year-old man complains of pain in his tense, peritoneal irritation signs are positive,
left eye and left side of his head, significant hepatic dullness is absent. BP is 90/60 mm Hg,
vision impairment of the left eye, nausea, and heart rate is 120/min. What further treatment
vomiting. Visual acuity of the right eye is 1.0. tactics should be chosen?
Visual acuity of the left eye is 0.03, attempts at
Kk. Laparotomy
correction bring no improvement. Right eye
L. Laparoscopy
intraocular pressure - 21 mm Hg, left eye
M. Cold to the abdomen
intraocular pressure 65 mm Hg. Congestive
N. Ultrasound investigation
injection is observed on the sclera of the left eye.
O. Laparocentesis
The cornea is thick and swollen. The anterior
chamber is shallow, moist, and clear. The pupil 14. A 48-year-old woman has arrived to the
is dilated and unresponsive to the light, the surgical unit with wounds in her thigh. On
fundus of the eye is not visible. What is the most examination the wound surface has dirty-gray
likely diagnosis? coating with unpleasant sweet smell. Wound
content resembles raspberry jelly. Skin tissues
around the wound are glossy and_ turgid. K. Right-sided reducible inguinal hernia
Palpation reveals moderate crepitation in the L. Right-sided reducible femoral hernia
tissues. What microflora is the most likely to M. Cyst of the right spermatic cord
cause such inflammation? N. Right-sided inguinal lymphadenitis
O. Right-sided reducible arcuate line hernia
K. Anaerobic clostridial
L. Anaerobic non-clostridial 18. A 78-year-old man with a prostate adenoma
M. Streptococci underwent a herniotomy for a direct inguinal
N. Staphylococei hernia. After the surgery he presents with absent
O. Blue pus bacillus urination. Enlarged urmary bladder is detectable
above the patient’s pubis. What measures should
15. After a pain attack in the right subcostal area,
be taken in this case?
a 58-year-old woman with overnutrition
developed icteric skin and sclera, lightcolored E. Bladder catheterization
feces, and dark urine. Her abdomen is distended F. Apply cold to the urinary bladder area C.
and painful on palpation in the right subcostal Prescribe processing of the postoperative
area. Palpation detects liver enlargement by 2-3 wound with UHF field
cm. Blood test: total bilirubin - 90 memol/L, H. Prescribe —_— proserin (neostigmine)
conjugated bilirubin - 60 mcmol/L. What method intramuscularly
of examination will be the most informative for I. Prescribe antispasmodics subcutaneously
diagnosis clarification?
19. A 38-year-old patient has been brought by an
K. Retrograde cholangiopancreatography
ambulance to the surgical department with
L. Intravenous cholegraphy
complaints of general weakness, indisposition,
M. Infusion cholegraphy
black stool. On examination the patient is pale,
N. Percutaneous transhepatic cholegraphy
there are dotted hemorrhages on the skin of his
O. US of the hepatopancreatobiliary zone
torso and extremities. On digital investigation
16. An 11-year-old boy for a month has been there are black feces on the glove. Blood test:
presenting with increasing pain in the right Hb- 108 g/L, thrombocytopenia. Anamnesis
femur. In the painful area there is a nonmobile states that a similar condition was observed 1
painful tumor with unclear margins. The child year ago. Make the diagnosis:
complains of general indisposition, weakness,
K. Thrombocytopenic purpura
increased body temperature up to 39°C. X-ray
L. Hemophilia
shows widened medullary cavity, small foci of
M. Ulcerative bleeding
cancellous bone destruction, and onion-like
N. Rectal tumor
lamellar exfoliation of the cortical layer. What is
O. Nonspecific ulcerative colitis
the most likely pathology resultmg in such
clinical presentation? 20. A 30-year-old man came to the family
physician. 2 months ago he underwent a surgery
K. Ewing sarcoma
for open fracture of the humerus. On examination
L. Osteogenic sarcoma
the patient’s condition 1s satisfactory; in the area
M. Fibrosarcoma
of the postoperative wound there is a fistula that
N. Chondrosarcoma
discharges a small amount of pus; the area itself
O. Juxtacortical sarcoma
is red; fluctuation is detected. X-ray shows
17. A 43-year-old man complains of a protrusion destruction of the humerus with sequestra. What
in the right inguinal region, that enlarges due to complication did the patient develop during the
strain. He has been presenting with this condition postoperative period?
for 6 months. Within this period the protrusion
K. Posttraumatic osteomyelitis
has grown. Objectively in the might inguinal
L. Hematogenous osteomyelitis
region an elastic protrusion 8x5 cm is visible. On
M. Wound suppuration
palpation it disappears, leaving an empty space
N. Posttraumatic phlegmon
4x4 cm between the pedicles of the Poupart
O. Suture sinus
ligament. *Cough push” sign 1s positive over this
opening. Make the diagnosis: 21. 3 hours after a trauma, a young man
developed bradycardia of 46/min., anisocoria
D>S8, hemi-hyperreflexia S>D, hemihypesthesia position, cyanotic skin covered in cold sweat,
on the left, and a convulsive disorder. The tachycardia, deficient pulse, and low blood
character of this process needs to be clarified. pressure. What urgent treatment tactics should be
What method of examination will be the most chosen?
accurate for this purpose?
K. Tracheostomy
Kk. Brain CT L. Oral administration of hyposensitization
L. Skull X-ray substances and broncholytics
M. Electroencephalography M. Intravenous administration of dehydrating
N. Echoencephalography agents
O. Lumbar puncture N. Administration of glucocorticoid hormones
O. Oxygen therapy
22. The body of a 24-year-old woman with
suspected poisoning has been found on the street. 26. Heart X-ray of a 31-year-old man has
Forensic medical examination was requested by revealed the following: with tightly filled
an investigator during examination of the site and opacified esophagus there is a marginal filling
the body. According to the Criminal Procedure defect in its middle third on the posterior wall;
Code currently in force in Ukraine, forensic the defect is 1.8x1.3 cm in size with clear oval
medical examination is required when it is border. Mucosal folds are retained and envelop
necessary to determine the: the defect; wall peristalsis and elasticity are not
affected. There are no complaints regarding the
K. Cause of death
condition of the patient’s alimentary canal. Make
L. Manner of death
the provisional diagnosis:
M. Time of death
N. Mode of death K. Esophageal tumor
O. Mechanism of death L. Achalasia cardiae
M. Esophageal burns
23. A 37-year-old patient complains of pain in
N. Diverticulum
the spimal column, reduced mobility. The
O. Barrett esophagus
condition persists for 7 years. *Sway back” is
observed, there is no movement in all spinal 27. A 25-year-old man was hospitalized with
regions. X-ray shows “bamboo spine” vertebral complaints of pain in his lower abdomen and
column. What is the most likely diagnosis? right lumbar area that appeared one hour ago.
K. Ankylosing spondyloarthritis Patient’s general state is moderately severe.
L. Osteochondrosis Body temperature - 38.2°C, heart rate - 102/min.
M. Spondylitis deformans The tongue is dry. The abdomen is painful on
N. Tuberculous spondylitis deep palpation in the right ilac area and in the
O. Spondylolisthesis Petit triangle. AureRozanov and Gabay signs are
positive. Make the provisional diagnosis:
24. A surgery unit received a person with an
K. Acute appendicitis
incised stab wound on the upper third of the nght
L. Right-sided renal colic
thigh. Examination detects an incised stab wound
M. Cecal tumor
3.0x0.5x2.0 cm in size on the inner surface of the
N. Intestinal obstruction
upper third of the night thigh. Bright-red blood
O. Acute cholecystitis
flows from deep within the wound in a pulsing
stream. Characterize this type of bleeding: 28. A 45-year-old man diagnosed with acute
pulmonary abscess suddenly developed sharp
K. Arterial
pain in his chest on the right and dyspnea up to
L. Venous
30/min. Examination detects facial cyanosis and
M. Parenchimatous
shallow rapid respirations. Auscultation reveals
N. Capillary
acutely weakened respiration throughout the
O. Mixed
whole right lung; percussion reveals a
25. A 47-year-old man developed the signs of vesiculotympanitic (bandbox) resonance at the
decompensated laryngeal stenosis against the lung apex and dullness in the lower lobe. What
background of acute flegmonous laryngitis. He complication developed in this patient?
presents with inspiratory dyspnea at rest, forced
K. Pyopneumothorax patient’s body there is graybrown area of necrosis
L. Pleuropneumonia that covers 3/4 of the body perimeter.
M. Pneumothorax Occasionally there are small blisters with
N. Acute mediastinitis hemorrhagic contents and patches of shredded
O. Esophageal perforation epidermis. What local therapy is necessary in this
case?
29. A 5-year-old child was brought to the ENT
department by an ambulance. The child presents K. Decompression necrectomy
with cough and difficult respiration. From the L. Chemical necrolysis
patient’s history it is known that the child was M. Blister puncture
playing with a toy construction set, when N. Necrectomy with xenotransplantation
suddenly started coughing and developed O. Necrectomy with dermal autograft
labored breathing. Examination detects
32. A woman in her early- to mid-thirties has lost
periodical cough, labored expiration, and
her consciousness 3-5 minutes ago. On
respiratory lag in the left side of the child’s
examination: the skin is pale, no pulse over the
thorax. Auscultation: diminished respiration on
carotid arteries, no spontaneous respiration, pupils
the left. Percussion: tympanitis. X-ray shows a
are dilated; the patient is nonresponsive, presents
displacement of the mediastinal organs to the
with atony. The patient’s condition can be
right. Make the diagnosis:
determined as:
K. A foreign body in the left bronchus,
K. Clinical death
valvular bronchostenosis
L. Natural death
L. A foreign body in the right bronchus,
M. Syncope
valvular bronchostenosis
N. Brain death
M. A foreign body in the trachea
O. Comatose state
N. A foreign body in the left bronchus, complete
bronchostenosis 33. A boy had a foreign body removed from under
O. A foreign body in the right bronchus, partial his nail plate. 3 days later he developed a sharp
bronchostenosis throbbing pain at the end of his distal phalanx,
which intensifies when the phalanx is pressed,
30. A 30-year-old man was brought to the
hyperemia of the nail fold, elevated body
neurosurgical department with complaints of
temperature up to 38.5°C, and nail plate
constant headaches, nausea, vomiting, fever, and
discoloration. Make the diagnosis:
weakness of the right-side limbs. Anamnesis
states that one month ago the patient had a K. Subungual panaritium
surgery for left-sided suppurative otitis and L. Erysipelas
mastoiditis. He has been undergoing treatment in M. Paronychia
an ENT department. Approximately 2 weeks ago N. Erysipeloid
the temperature increased, and the patient O. Abscess
developed headaches. Objectively: heart rate -
98/min., BP- 140/90 mm Hg, temperature 34. A 32-year-old woman complains of body
38.3°C. Neurologically manifested stiff neck: weight loss despite her imcreased appetite,
bilateral Kernig’s symptom, unsteadiness during nervousness, and tremor of the extremities.
Objectively: the skin is moist; the thyroid gland is
the Romberg’s maneuver. Computer tomography
of the brain revealed a threedimensional growth diffusely enlarged, painless, soft, and mobile.
with a capsule in the left hemisphere. Make the Blood test: increased level of T3, T4, and thyroid-
diagnosis: stmulating hormone (THS). What is the most
likely diagnosis?
K. Cerebral abscess
L. Echinococcus K. Diffuse toxic goiter
M. Hemorrhage L. Thyroid carcinoma
N. Hydrocephalus M. Autoimmune (Hashimoto’s) thyroiditis
O. Amold-Chiari malformation N. Thyroid adenoma
31. The burns unit received a patient, who 6 hours O. Diffuse nontoxic goiter
ago during a fire received flame burns. On the
35. A 19-year-old young man complains of cough G. In medico-prophylactic institutions;
with expectoration of purulent sputum in the general physicians and surgeons
amount of 100 mL per day, hemoptysis, dyspnea, iH. At the site of the accident; first-response
increased body temperature up to 37.8°C, general emergency teams
weakness, weight loss. The patient’s condition L At the site of the accident; specialized
lasts for 4 years. Exacerbations occur 2-3 times a second-response emergency teams D. In medico-
year. The patient presents with malnutrition, pale prophylactic institutions; specialized second-
skin, cyanosis of the lips, drumstick response emergency teams E. In medical
(clubbed) fingers. Tympanic percussion sound in institutions; all listed types of healthcare workers
the lungs, weakened respiration, numerous 39. A 45-year-old man underwent a cardiac
various moist crackles in the lower pulmonary surgery one week ago. His general state has been
segments on the left can be observed. In blood: deteriorating since then: dyspnea at rest,
erythrocytes - 3.2-1012/L, leukocytes - retrosternal pain that irradiates to the neck,
8.4-10°/L, ESR- 56 mm/hour. On X-ray: lung marked weakness. Objectively his body
fields are emphysematous, the left pulmonary temperature is hectic. His cardiac borders are
root is deformed and dilated. What is the most expanded, apical beat is weakened. Auscultation
likely diagnosis? detects pericardial friction rub. What is the most
likely diagnosis?
K. Multiple bronchiectasis of the left lung
L. Chronic left-sided pneumonia K. Acute pericarditis
M. Chronic abscess of the left lung L. Acute cardiac aneurysm
N. Left-sided pulmonary cystic dysplasia M. Myocardial infarction
O. Suppuration of the cyst in the left lung N. Acute myogenic dilatation of the heart
O. Pulmonary embolism
36. A 57-year-old woman during a regular
ultrasound examination presented with a space- 40. A 45-year-old man was brought by an
occupying heterogeneous lesion in the right ambulance into the emergency hospital. He
kidney. What is the most informative method of complains of sudden pam in the lumbar area,
renal tumor diagnostics? frequent painful urination, and vomiting.
Examination detects pain in the lumbar area,
K. Spiral computed tomography
costovertebral angle tenderness, pain on palpation
L. Excretory urography
of kidneys and along the ureter on the right. Urine
M. Retrograde pyelography
test: proteins, fresh erythrocytes, leukocytes.
N. Radioisotope renography
Make the provisional diagnosis:
O. Three glass urine test
K. Urolithiasis, renal colic
37. A 40-year-old victim of a traffic accident
L. Acute pyelonephritis
sustained the following injuries: closed
M. Acute glomerulonephritis
diaphyseal femur fracture, brain concussion,
N. Acute renal failure
multiple mb fractures, hemopneumothorax,
O. Polycystic kidney disease
degloving shin injuries. What injuries require the
most urgent attention?

K. Multiple rib fractures, hemopneumothorax


L. Closed diaphyseal femur fracture
M. Brain concussion
N. Degloving shin injuries
O. All injuries are equivalent

38. At the railroad crossing a passenger train


collided with a bus. In this collision 26 bus
passenges died, another 18 passengers received
mechanical injuries of varying severity. Where
will be professional medical aid provided for the
victims of this accident? Who will provide this
aid?
1. A newborn girl has Apgar score of 7-8 disease is made. What examination would be the
points at the 1-5 minutes after birth. During the most informative for diagnosis confirmation?
labor there was a brief difficulty with extraction K. Doppler echocardiography
of the shoulder girdle. After birth the baby L. Electrocardiography
presents with disturbed function of the proximal M. Chest X-ray
segment and forced position of the right arm. The N. Rheography of the pulmonary artery
shoulder is rotated inwards, the elbow is O. Ultrasound of the liver
extended, the forearm is pronated, and the whole
upper limb resembles an arm of a doll. What is 5. Mother of a 5-year-old child noticed on the the
the most likely clinical diagnosis in this case? head of her child a round “bald” spot 3 cm in
diameter. All the hairs in the focus are broken off
K. Erb-Duchenne palsy at the length of 5-6 mm. The day before the child
L. Thoracic spine trauma was petting a stray cat. Make the diagnosis:
M. Osteomyelitis of the right arm
N. Intracranial hemorrhage K. Microsporia
O. Soft tissue injury of the nght arm L. Superficial trichophytosis
M. Deep trichophytosis
2. Disease onset was acute. A child developed N. Psoriasis
general weakness, pain in the joints, and elevated O. Alopecia areata
temperature. Later these signs became
accompanied by itching skin rash manifested as 6. A 2-year-old child with persisting cough and
erythematous spots 25 mm in size. The rash subfebrile body temperature after a case of URTI
gradually turned hemorrhagic. Large joints are developed dyspnea, cyanosis of the nasolabial
painful and swollen; pain attacks periodically triangle, percussion dullness and weakened
occur in the paraumbilical area; there are signs of respiration in the lower lobe of the right lung, and
intestinal hemorrhage. What is the most likely a slight mediastinal displacement to the left. What
diagnosis? pulmonary pathology is likely to cause this
clinical presentation?
A. Hemorrhagic vasculitis (Henoch-Schonlein
purpura) B. K. Pleurisy
Scarlet fever L. Emphysema
L Hemorrhagic meningoencephalitis M. Pneumonia
J. Streptococcal impetigo N. Atelectasis
Kk. Rheumatism O. Bronchitis

3. A 13-year-old girl for the last two weeks has 7. During examination a 4-month-old child with
been complaining of dyspnea and shin and foot meningococcemia presents with acrocyanosis,
edemas that appear after a physical exertion. In cold extremities, tachypnea, and thready pulse,
the morning the edemas significantly decrease. blood pressure of 30/0 mm Hg, anuria, and sopor.
Clinical examination revealed enlarged liver and What clinical syndrome 1s it?
coarse systolic murmur over the heart area.
K. Toxic shock syndrome
Blood test and urinalysis are without changes.
L. Neurotoxicosis
What is the most likely cause of edemas in this
M. Exicosis
child?
N. Encephalic syndrome
G. Heart failure O. Acute renal failure
EH. Nephrotic syndrome
8. At night a 2-year-old child with upper
L Acute pyelonephritis D. Angioneurotic edema
respiratory tract infection suddenly developed
E. Hepatic cirrhosis
dyspnea with labored inspiration. Objectively the
4. A 7-year-old boy has severe pulmonary skin is pale, perioral cyanosis and_ slight
mucoviscidosis (cystic fibrosis). He complains of acrocyanosis are observed. Breathing is loud,
dyspnea and blood expectoration. Objectively he respiration rate is 32/min. Jugular, supra- and
presents with lagging physical development, infraclavicular fossae retract during breathing.
acrocyanosis, hepatomegaly, drumstick fingers, Respiration is coarse on auscultation. Heart
and nail plates resembling a “clock face”. sounds are clear and sonorous, heart rate is
Provisional diagnosis of chronic pulmonary heart 120/min. What condition was complicated by the
development of the upper respiratory tract What should be excluded from the diet in this
infection? case?

K. Stenosing laryngotracheitis K. Cereals - wheat, oats


L. Airway foreign body L. Milk and dairy products
M. Obstructive bronchitis M. Fruits
N. Bronchiolitis N. Animal protein
O. Bronchial asthma O. Easily digestible carbohydrates
9. A l-year-old child with a case of URTI
13. A 7-year-old boy has been an inpatient for 1.5
suddenly developed noisy respirations with
months. He had been brought to the hospital with
difficult mspiration, intercostal retractions, and
complaints of edemas all over his body, low
barking cough on the 2nd night after the disease
urine output, and headache. Clinical urinalysis:
onset. What is the most likely diagnosis?
proteins 7.1 g/L, leukocytes - 1-2 in the vision
K. Stenosing laryngotracheobronchitis field, erythrocytes - 3-4 m the vision field.
L. Acute pulmonary inflammation During the course of treatment the edemas
M. Bronchial asthma gradually dissipated, headache abated, diuresis
N. Acute bronchitis normalized. Daily urine protems 3 g/L.
O. Acute bronchiolitis Biochemical blood test: total protem 43.2 g/L,
urea - 5.2 mmol/L, cholesterol - 9.2 mmol/L.
10. A 10-year-old boy with symptoms of arthritis What glomerulonephritis syndrome is the most
and myocarditis was brought to a hospital. Based likely to be present in the patient?
on clinical examination the provisional diagnosis
of juvenile rheumatoid arthritis was made. What K. Nephrotic
symptom is the most contributive for the L. Nephritic
diagnostics of this disease? M. Isolated urmary
N. Hematuric
K. Reduced mobility of the joints in the morning O. Mixed
L. Regional hyperemia of the joints
M. Affection of the large joints 14. A 3-month-old child with signs of rickets
N. Enlarged heart presents with positive Chvostek, Trousseau, and
O. Increased heart rate Maslov signs. One day ago the parents witnessed
a cyanotic attack in their child the child broke into
11. A 7-year-old girl has been twice treated with a cold sweat, the eyes bulged, and respiratory
antibacterial agents for urinary tract infection. arrest occurred. One minute later the child drew in
US shows no severe renal defects. The child a loud breath and the child’s condition normalized
presents with recurrence of leukocyturia and again. What is the cause of the described signs of
bacteriuria, elevated body temperature up to the disease?
38.5°C, and pain in her left lumbar area. What
examination should be conducted first to clarify K. Decrease of blood calcium levels
the cause of urinary infection recurrence? L. Increase of blood calcium levels
M. Decrease of blood phosphorus levels
K. Micturating cystourethrography N. Increase of blood phosphorus levels
L. Excretory urography O. Metabolic acidosis
M. Retrograde pyelography
N. Immunogram 15. A newborn with gestational age of 31 weeks
O. Radioisotope renography presents with hypotonia and _ depressed
consciousness. Hematocrit is 35%, general
12. A child is 1 year old. After solid food was cerebrospinal fluid analysis shows increased
introduced into the diet, within the last several content of erythrocytes and protein, and low
months the child developed loss of appetite, glucose. These data correspond with the clinical
diarrhea with large amount of feces, and presentation of:
occasional vomiting. Body temperature remains
normal. Body weight is 7 kg. The child is very K. Intracranial hemorrhage
pale, has leg edemas and extremely distended L. Meningitis
abdomen. Feces analysis detects high levels of M. Sepsis
fatty acids and soaps. Diagnosis of celiac disease N. Anemia
was made and gluten-free diet was prescribed. O. Intrauterine infection
16. A newborm has Apgar score of 9. When should The cough attacks induce vomiting. X-ray shows
this infant be put to the breast? intensified bronchial pattern. Blood _ test:
leukocytes 16 - 109/L , lymphocytes - 72%,
K. In the delivery room
erythrocyte sedimentation rate - 4 mm/hour.
L. After 12 hours What 1s the most likely diagnosis?
M. After 2 hours
N. On the 2nd day K. Pertussis
O. On the 3rd day L. Obstructive bronchitis
M. Pneumonia
17. A 3-week-old infant developed large, flaccid N. Adenovirus infection
vesicles with purulent contents on the skin of chest
O. Foreign body
and abdomen. The vesicles rupture quickly. Make
the provisional diagnosis: 21. A 3-year-old child presents with dyspnea that
K. Pemphigus neonatorum abates in the sitting position, occasional loss of
L. Vesiculopustulosis consciousness and seizures, delayed physical
M. Toxic erythema development, cyanosis, drumstick fingers.
N. Pemphigus syphiliticus Echocardioscopy detects aortic dextraposition,
O. Pseudofurunculosis ventricular septal defect, pulmonary artery
stenosis, and right ventricular hypertrophy. What
18. 10 hours after birth a child developed is the most likely diagnosis?
jaundice, hypotonia, hyporeflexia, and moderate K. Tetrad of Fallot
hepatosplenomegaly. Feces and urine are of L. Coarctation of the aorta
normal color. Umbilical cord blood bilirubin is M. Transposition of the great vessels
51 memol/L due to unconjugated bilirubin levels. N. Ventricular septal defect
In venous blood: erythrocytes - 3.5-1012/L, Hb- O. Acquired valvular disease
140 g/L, reticulocytes - 1.5%, bilirubin - 111
memol/L, conjugated - 11 memol/L, ALT- 40 22. A 15-year-old girl complains of dizziness and
U/L, AST30 U/L. Mother’s blood group is A(II) sensation of lack of air that she develops in
Rh(- emotionally straining situations. Relief occurs
), child’s blood group is ACI) Rh(+). What after she takes corvalol. Objectively:
laboratory test can confirm the diagnosis? hyperhidrosis and marble-like pattern of the skin
of her palms and feet. Clinical and instrumental
K. Coombs test examination revealed no organic changes in the
L. Viral hepatitis markers analysis central nervous, cardiovascular, and respiratory
M. Measurement of erythrocyte systems. What provisional diagnosis can be
osmotic resistance made?
N. Erythrocytometry
O. Measurement of glucose 6- K. Somatoform autonomic dysfunction
phosphate dehydrogenase levels in L. Obstructive bronchitis
erythrocytes M. Bronchial asthma
N. Stenosing laryngotracheitis
19. A 6-month-old child on breastfeeding is O. Acute epiglottitis
hospitalized in the inpatient department. After
the child recovers, the doctor recommends the 23. A 1.5-month-old child on breasfeeding
mother to start introducing solid food to the presents from birth with daily vomiting, irregular
child’s diet. What products should be introduced liquid foamy feces, and meteorism, which are
to the child’s diet first? resistant to antibacterial and probiotic therapy; no
increase of body mass is observed. The child’s
K. Vegetable puree condition improved, when breastmilk was
L. Fermented dairy products substituted with “NAN low lactose” formula.
M. Grated apple What pathology is it?
N. Semolina porridge
O. Buckwheat porridge K. Lactase deficiency
L. Intestinal lambliasis (Giardiasis)
20. The 5-year-old child has been ill for 2 weeks. M. Infectious enteritis
Cough attacks developed first and were then N. Drug-induced enteritis
followed by reprises. During coughing the O. Functional dyspepsia
child’s face turns red and cervical veins bulge.
24. A 13-year-old girl for a month has been 70%, ESR- 52 mm/hour. Make the provisional
complaining of fatigability, dull pain in her right diagnosis:
subcostal area, abdominal distension, and
K. Acute leukemia
constipations. Abdominal palpation reveals
positive Kehr, Murphy, and Ortner signs, while L. Thrombocytopenic purpura
Desjardins and Mayo-Robson points are painless. M. Acute rheumatic fever
Total bilirubin is 14.7 mcmol/L, predominantly N. Infectious mononucleosis
indirect, ALT- 20 U/L, AST- 40 U/L, amylase - O. Hemorrhagic vasculitis (Henoch-Schonlein
6.3 mmol/L. Echocholecystography shows purpura)
practically no contraction of the gallbladder.
28. During an outdoors school event in hot
Make the provisional diagnosis:
weather, a 10-year-old girl lost her
K. Hypokinetic biliary dyskinesia consciousness. Body temperature - 36.7°C.
L. Hyperkinetic biliary dyskinesia Objectively her skin is pale and cold to touch, her
M. Chronic pancreatitis pupils are dilated. Blood pressure - 90/50 mm Hg.
N. Acute pancreatitis Heart rate - 58/min. What pathology occurred in
O. Chronic hepatitis this case?

25. A 22-day-old infant developed subcutaneous K. Syncope


red nodes from 1.0 to 1.5 cm in size on the scalp; L. Sympathicotonic collapse
later the nodes suppurated. Temperature increased M. Paralytic collapse
up to 37.7°C, intoxication symptoms appeared, N. Sunstroke
regional lymph nodes enlarged. Complete blood O. -
count: anemia, leukocytosis, neutrocytosis,
29. A 13-year-old girl has 30% of excessive body
increased ESR. What diagnosis can be made?
mass, she started to gain weight at the age of 3.
K. Pseudofurunculosis
She has a family history of obesity. Her height and
L. Pemphigus sexual development are normal for her age. The
M. Vesiculopustulosis appetite is excessive. She complains of periodical
N. Scalp phlegmon headaches. Blood pressure - 120/80 mm Hg.
O. - Subcutaneous fat 1s evenly distributed, she has no
26. A 10-year-old boy was brought into the stretch marks. There is juvenile acne on her face.
hospital with complaints of expiratory dyspnea, What type of obesity is it?
respirations are 30/min. He explains his state by K. Alimentary constitutive obesity
a change in the weather conditions. For the last 4 L. Hypothalamic obesity
years the boy has been registered for regular M. Adrenal obesity
check-ups due to his diagnosis of third degree
N. Hypothalamic syndrome of puberty
persistent bronchial asthma. To provide
O. Hypothyroid obesity
emergency aid for this child, first he needs to be
given: 30. An 8-year-old girl complains of frequent
painful urination in small amounts and urinary
K. Salbutamol or short-acting £2-agonists
incontinence. The signs have been present for 2
L. Dexamethasone days already. She explains her disease by
M. Adrenaline overexposure to cold. Costovertebral angle
N. Euphylline (Ammophylline) tenderness is absent. Complete blood count is
O. Claritin (Loratadine) without pathologies. Urine test: leukocytes - 20-
27. A 3-year-old child has been brought to a 30 in the vision field, erythrocytes - 40-50 in the
hospital with complaints of pain in the legs, vision field, unchanged, bacteriuria. What is the
fever, and loss of appetite. Objectively: pale skin most likely diagnosis?
and mucosa, hemorrhagic rash. Lymph nodes are K. Cystitis
enlarged, painless, dense and elastic, not matted L. Vulvitis
together. Bones, joints, and abdomen are painful. M. Pyelonephritis
The liver and spleen are enlarged. Hemogram: N. Glomerulonephritis
Hb- 88 g/L, color index - 1.3, platelets - 80 -
O. Urolithiasis
109/L, leukocytes - 25.8-10°/L, lymphoblasts -
1. A pregnant woman is 28 years old. O. Hormone-producing ovarian tumor
Anamnesis: accelerated labor complicated by the
4. A 48-year-old woman complains of disturbed
II degree cervical rupture. The following two
menstrual cycle: her periods last for 7-9 days and
pregnancies resulted in spontaneous abortions at
are excessively profuse throughout the last half-
the terms of 12 and 14 weeks. On mirror
year. She notes occasional hot flashes in her head,
examination: the uterine cervix is scarred from
insomnia, irritability, and headaches. Her skin is
previous ruptures at 9 and 3 hours, the cervical
of normal color. Blood pressure - 150/90 mm Hg,
canal is gaping. On vaginal examination: the
pulse - 90/min., rhythmic. The abdomen is soft
cervix is 2 cm long, the external orifice is open 1
and painless. Bimanual examination shows no
cm wide, the internal orifice is half-open; the
uterine enlargement, the appendages cannot be
uterus is enlarged to the 12th week of pregnancy,
detected. The vaginal fornices are free. What is
soft, mobile, painless, the appendages are without
the most likely diagnosis?
changes. What diagnosis can be made?
I. Climacteric syndrome
A. Isthmico-cervical insufficiency, habitual
J. Premenstrual syndrome
noncarrying of pregnancy B. Threatened
K. Adrenogenital syndrome
spontaneous abortion C. Incipient abortion,
L. Stein-Leventhal syndrome
habitual noncarrying of pregnancy
(polycystic
D. Cervical hysteromyoma, habitual
ovary syndrome) E.
noncarrying of pregnancy E.
Uterine myoma
Cervical pregnancy, 12 weeks
5. A 30-year-old multigravida has been in labour
2. On the day 4 after the cesarean section a woman
for 18 hours. 2 hours ago the pushing stage began.
developed fever with body temperature up to 39°C
Fetal heart rate is clear, rhythmic, 136/min.
and abdominal pain. Pulse - 104/min. She
Vaginal examination reveals complete cervical
vomited twice. The patient is sluggish, her tongue
dilatation, the fetal head in the pelvic outlet plane.
is dry and has gray coating. The abdomen is
Sagittal suture is in line with obstetric conjugate,
distended. Signs of peritoneal irritation are
the occipital fontanel is near the pubis. The
positive in all segments. Peristalsis cannot be
patient has been diagnosed with primary uterine
auscultated. No passage of gas occurs. Uterine
mertia. What is the further tactics of labor
fundus is located at the level of the navel. The
management?
uterus is pamful on palpation. The discharge is
moderate and contains blood and pus. What is the K. Outlet forceps
most likely diagnosis? L. Labour stimulation
M. Cesarean section
K. Diffuse peritonitis
N. Skin-head Ivanov’s forceps
L. Metroendometritis
O. Vacuum extraction of the fetus
M. Progressive thrombophlebitis
N. Pelvic peritonitis 6. A woman is 40. weeks pregnant.
O. Parametritis The fetus is in the longitudinal lie and cephalic
presentation. Pelvic size: 26-29-3120. Expected
3. A 58-year-old woman came to the
weight of the fetus is 4800 gram. The labor
gynecological clinic. She complains of bloody
contractions has been lasting for 12 hours, within
discharge from her genital tracts. Menopause 1s 8
the last 2 hours they were extremely painful, the
years. Gynecological examination: the uterus is
parturient woman is anxious. The waters broke 4
slightly enlarged, dense to touch, with limited
hours ago. On external examination the
mobility, the uterine appendages cannot be
contraction ring is located 2 finger widths above
detected, parametrium is free. Fractional
the navel, Henkel-Vasten sign is positive. Fetal
curettage of the uterine cavity yields a significant
heart rate is 160/min., muffled. On internal
amount of medullary substance in the scrape.
examination the uterine cervix is fully open, the
What 1s the most likely diagnosis?
head is engaged and pressed to the entrance into
K. Uterine corpus cancer the lesser pelvis. What is the most likely
L. Adenomyosis diagnosis?
M. Chorioepithelioma
K. Threatened uterine rupture
N. Uterine cervix cancer
L. Complete uterine rupture G. Dysfunctional uterine bleeding
M. Hyperactive uterine contractions H. Adenomyosis
N. Abruption of the normally positioned I. Ectopic pregnancy D. Submucous uterine
placenta myoma
O. Anatomically contracted pelvis E. Cancer of the uterine body

7. A 23-year-old woman came to the 11. A 25-year-old woman was brought into the
gynecological clinic. She complains of pain, gynecological department with profuse bloody
itching, and burning in her vulva, general discharge from her genital tracts. She is 12 weeks
weakness, indisposition, elevated body pregnant, the pregnancy is planned. Within the
temperature up to 37.2°C, and headache. On last 3 days she was experiencing pains in her
examination in the vulva there are multiple lower abdomen that eventually started resembling
vesicles up to 2-3 mm in diameter with clear cramps, she developed bleeding. Her skin is pale,
contents against the background of hyperemia pulse - 88/min., blood pressure - 100/60 mm Hg,
and mucosal edema. Make the provisional body temperature - 36.8°C. Vaginal examination:
diagnosis: the uterus size corresponds with 11 weeks of
pregnancy, the cervical canal allows inserting 1
K. Genital herpes infection
finger and contains fragments of the fertilized
L. Primary syphilis
ovum, the discharge is bloody and profuse. What
M. Papillomavirus infection
is the most likely diagnosis?
N. Vulvar cancer
O. Cytomegalovirus infection G. 12-week pregnancy, spontaneous
abortion in progress
8. A woman with the pregnancy term of 8 weeks
iH. 12-week pregnancy,
complains of elevated temperature up to 37.6°C,
threatened
skin rash that can be characterized as macular
spontaneous abortion
exanthema, enlargement of posterior cervical and
L Disturbed menstrual cycle,
occipital lymph nodes, small amount of bloody
hyperpolymenorrhea D. Disturbed menstrual
discharge from the genital tracts. She was
cycle, amenorrhea
examined by the infectious diseases specialist and
E. Full-term pregnancy, term labor
diagnosed with rubella. What tactics should the
obstetrician-gynecologist choose? 12. A 30-year-old woman came to the
gynecological department. She complains of
Kk. Abortion
sharp pain in her lower abdomen and temperature
L. Prescription of antibacterial therapy
of 38.8°C. She has a history of extramarital sexual
M. Prescription of antiviral therapy
N. Treatment of incipient abortion activity and 2 = artificial abortions. On
gynecological examination the uterus is
O. Prescription of hemostatic therapy
unchanged. The appendages are bilaterally
9. A 16-year-old girl has primary amenorrhea, no enlarged and painful. Profuse purulent discharge
pubic hair growth, normally developed mammary is being produced from the vagina. What
glands; her genotype is 46 XY; uterus and vagina examination needs to be conducted to clarify the
are absent. What is your diagnosis? diagnosis?
E. Testicular feminization syndrome K. Bacteriological and bacterioscopic analysis
F. Mayer-Rokitansky-Kuster-Hauser syndrome L. Hysteroscopy
C. Cushing syndrome M. Curettage of the uterine cavity
H. Sheehan syndrome
N. Colposcopy
L Cushing disease
O. Laparoscopy
10. A 46-year-old woman came to the maternity
13. It is the 3rd day after the first normal term
clinic with complaints of moderate blood
labor; the infant 1s rooming-in with the mother
discharge from the vagina, which developed after and is on breastfeeding. Objectively: the mother’s
the menstruation delay of 1.5 months. On vaginal
general condition is satisfactory. Temperature is
examination: the cervix is clean; the uterus is not
36.4°C, heart rate is 80/min.,
enlarged, mobile, painless; appendages without
BP is 120/80 mm Hg. Mammary glands are soft
changes. Make the diagnosis:
and painless; lactation is moderate, unrestricted
milk flow. The uterus is dense, the uterine fundus A. Sheehan syndrome (postpartum pituitary gland
is located 3 finger widths below the navel. Lochia necrosis) B. Physiological amenorrhea
are sanguino-serous, moderate in volume. Assess C. Suspected progressing ectopic pregnancy D.
the dynamics of uterine involution: Stein-Leventhal syndrome (polycystic ovary
syndrome) E. Galactorrhea-amenorrhea syndrome
K. Physiological involution
L. Subinvolution 17. A 45-year-old woman came
M. Lochiometra to the maternity clinic with complaints of
periodical pains in her mammary glands that start
N. Pathologic involution
1 day before menstruation and stop after the
O. Hematometra menstruation begins. Palpation of the mammary
glands detects diffuse nodes predominantly in
14. A 28-year-old woman complaining of
the upper outer quadrants. What is the most likely
irregular menstruations and infertility came to the diagnosis?
gynecological clinic. Menstruations occur since
the age of 15, irregular, with delays up to 2 K. Fibrocystic mastopathy
months. On examination she presents with L. Breast cancer
marked hirsutism and excessive body weight. On M. Mastitis
vaginal examination the uterus is reduced in size N. Hyperprolactinemia
and painless. The ovaries on the both sides are O. Breast cyst
dense and enlarged. Ultasound shows microcystic
18. A 14-year-old girl came to the general
changes in the ovaries, the ovaries are 5x4 cm and
practitioner with complaints of weakness, loss of
4.5x4 cm in size with dense ovarian capsule.
appetite, headache, rapid fatigability. Her last
Basal body temperature is monophasic. What is
menstruation was profuse and lasted for 14 days
the most likely diagnosis?
after the previous delay of 2 months. Objectively:
K. Polycystic ovary syndrome the skin is pale, heart rate is 90/min., BP is 110/70
L. Krukenberg tumor mm Hg, Hb is 88 g/L. Rectal examination: the
M. Endometrioid cysts
uterus and its appendages are without changes, no
N. Bilateral adnexitis
discharge from the genital tracts. What
O. Ovarian carcinoma
complication occurred in the patient?
15. An 18-year-old girl was brought imto the E. Posthemorrhagic anemia
gynecology deparment with complaints of F. Somatoform autonomic dysfunction of
elevated body temperature up to 37.8°C, sharp hypotonic type C. Migraine
pain in her lower abdomen, more intense on the H. Gastritis
right, and difficult defecation. Vaginal I. Dysmenorrhea
examination detected a painful dense elastic
formation 5x6 cm in the area of her right ovary. 19. A 22-year-old postparturient woman on the
Pregnancy test is negative. What is the most 12th day after the normal childbirth informs of
likely diagnosis? elevated body temperature up to 39°C for the last
3 days and pain in her right mammary gland. The
K. Torsion of ovarian tumor pedicle right mammary gland is enlarged, hot to touch,
L. Ectopic pregnancy tense, hyperemic, and painful. Palpation reveals
M. Appendicitis there a dense infiltration 8x8 cm with a
N. Ovarian cyst rupture fluctuation in its center. What is the most likely
O. Ovarian apoplexy diagnosis?
16. A 26-year-old woman presents with K. Postpartum period, day 12. Right-sided
amenorrhea. 10 months ago she gave birth for a infiltrative-purulent mastitis
second time. In her early postpartum period she L. Postpartum period, day 12. Right-sided serous
developed a massive hypotonic hemorrhage. No mastitis
breasfeeding. Lately she has been presenting with M. Postpartum period, day 12. Right-sided
loss of weight, loss of hair, and indisposition. gangrenous mastitis
Gynecological examination revealed atrophy of N. Postpartum period, day 12. Right-sided
the external genitals, the uterus is abnormally phlegmonous mastitis
small, no uterine appendages can be detected. O. Postpartum period, day 12. Right-sided
What 1s the most likely diagnosis? lactostasis
20. A 35-year-old pregnant woman with degree 1 sounds detects bradycardia. Evaluation of
essential hypertension, developed edemas and cardiotocogram yielded the following data:
headache at the 33 week of her pregnancy. decrease of basal heart rate down to 90/min.,
Objectively her general condition is satisfactory, variability - monotonous (2 and less); late
blood pressure - 160/100 mm Hg, normal uterine decelerations with amplitude of 50/min. Make the
tone. Fetal heart rate 1s 140/min., rhythmic. She diagnosis and choose the obstetrical tactics
was diagnosed with daily proteinuria - 4 g/L, necessary in this case:
daily diuresis - 1100 mL. Creatinine - 80
E. Fetal distress. Urgent cesarean section
memol/L, urea - 7 mmol/L, platelets - 100 -
delivery
109/L. What complication of pregnancy
F. Fetal distress. Vacuum extraction delivery
occurred?
C. Normal condition of the fetus. Vaginal
K. Moderate preeclampsia birth
L. Severe preeclampsia H. Fetaldistress.Stimulation of uterine
M. Mild preeclampsia contractions
N. Hypertensive crisis I. Fetal distress. Forceps delivery
O. Renal failure 24. A 27-year-old woman complains of foul-
smelling discharge from her genital tracts, pain in
21. A 24-year-old pregnant woman on her 37th
her lower abdomen, and elevated temperature.
week of pregnancy has been brought to the
The complaints arose 2 days ago. She has a
maternity obstetric service with complaints of
history of surgical abortion at the term of 8 weeks
weak fetal movements. Fetal heartbeats are
one week ago. Mirror examination: the uterine
95/min. On vaginal examination the uterine
cervix is clear, external orifice produces
cervix 1s tilted backwards, 2 cm long, external
foulsmelling discharge. Vaginal examination: the
orifice allows inserting a fingertip. Biophysical
uterus lies in anteflexion, is mobile, painful, and
profile of the fetus equals 4 pomts. What tactics
slightly enlarged. The appendages are without
of pregnancy management should be chosen?
changes. Make the provisional diagnosis:
A. Urgent delivery via a cesarean section B.
Treatment of placental dysfunction and repeated K. Postabortal endometritis
analysis of the fetal biophysical profile on the next L. Enterocolitis
day M. Appendicitis
L Doppler measurement of blood velocity in N. Acute respiratory disease
the umbilical artery O. Salpingoophoritis
J. Urgent preparation of the uterine cervix
25. A 17-year-old girl has made an appointment
for delivery
with the doctor. She plans to begin her sex life.
K. Treatment of fetal distress; if ineffective,
No signs of gynecological pathology were
then elective cesarean section on the next day
detected. In the family history the patient’s
22. During regular preventive gynecological grandmother had cervical cancer. The patient was
examination a 30-year-old woman was detected consulted about the maintenance of her
to have dark blue punctulated perforations” on reproductive health. What recommendation will
the vaginal portion of the uterme cervix. The be the most helpful for prevention of invasive
doctor suspects endometriosis of the vaginal cervical cancer?
portion of the uterine cervix. What investigation
I. Vaccination against human papillomavirus
method would be most informative for diagnosis
(HPV)
confirmation?
J. Vitamins, calctum, omega-3
K. Colposcopy, target biopsy of the cervix Kk. Immunomodulators
L. US of the lesser pelvis L. Antiviral and antibacterial drugs E. Timely
M. Hysteroscopy treatment of sexually transmitted
N. Curettage of the uterine cavity diseases
O. Hormone testing

23. A parturient woman is 30 years old, stage I of


the labor is ongoing. The fetus is in the cephalic
presentation. Auscultation of the fetal heart
1. A 38-year-old woman works in flax processing, she dries flax. She came to the hospital complaining of
difficult breathing, constricting sensation in her chest, and cough attacks. These signs appear on the first day
of her working week and gradually diminish on the following days. What respiratory disease is likely in this
case?

K. Byssinosis
L. Silicosis
M. Allergic rhinopharyngitis
N. Bronchial asthma
O. Asthmatic bronchitis

2. A district doctor has diagnosed one of his patients with dysentery. What accounting document reflects this
type of morbidity?

K. Urgent report
L. Statistical report
M. Report on a major non-epidemic disease
N. Certificate of temporary disability
O. Control card of a patient registered for regular check-ups

3. A 39-year-old man, a battery attendant, suddenly developed weakness, loss of appetite, nonlocalized
colicky abdominal pains, and nausea. Objectively his skin is gray; there is a pmk-gray stripe on his gums; the
stomach is soft and sharply painful. Blood test detected erythrocytes with basophilic stippling and anemia.
The patient has a history of peptic ulcer disease of the stomach. Constipation tends to occur every 3-4 days.
What is the most likely provisional diagnosis?

K. Saturnism (lead poisosning)


L. Acute appendicitis
M. Perforation of gastric ulcer
N. Acute cholecystitis
O. Chronic alcoholism

4. A 9-month-old infant presents with delayed tooth eruption and fontanel closure, weakness, and excessive
sweating. What type of hypovitaminosis is the most likely in this child?

K. Hypovitaminosis D
L. Hypovitaminosis C
M. Hypovitaminosis B,
N. Hypovitamimosis Be
O. Hypovitaminosis A

5. A 10-year-old girl exhibits high level of physical development ( + 3a), her body length increased by 10 cm
within a year (which 1s double the norm for her age group), the number of permanent teeth corresponds with
the age norm (20), the development of her secondary sex characteristics is three years ahead of her age (Ma,
P, Ax, Menarche). Development rate ahead of her biological age can occur due to:
K. Endocrine disorders
L. Acceleration
M. Certain components of her diet
N. Sports training
O. Deficient hygienic education

6. Employees work in conditions of high dust concentration. Certain chemical (silicon dioxide content) and
physical properties of dust aerosols contribute to the development of occupational dust-induced diseases.
What is the main physical property of dust aerosols?

K. Dispersion
L. Magnetization
M. Electric charge
N. Solubility
O. Tonization
7. In the factory cafeteria there was an outbreak of food poisoning. Clinical
presentation indicates staphylococcal etiology of this disease. 15 people are sick. To confirm the
diagnosis of food poisoning, samples need to be sent to the laboratory. What samplesshould be
obtained for analysis?

K. Vomit masses
L. Blood for hemoculture
M. Blood (complete blood count)
N. Urine
O. Saliva

8. In April during the medical examination of various population groups, 27% of individuals presented with
low working ability and rapid fatigability. The following symptoms were observed in the affected mdividuals:
swollen friable gingiva that bleeds when pressed, hyperkeratosis follicularis not accompanied by skin dryness.
These symptoms most likely result from the following pathology:

K. C-hypovitaminosis
L. Parodontosis
M. A-hypovitaminosis
N. B,-hypovitaminosis
O. Polyhypovitaminosis
9. Increased general morbidity of the local population is observed in the area near a
factory, where atmosphere is being intensively polluted with sulfurous gas. What effect does polluted air
have on human body in this case?

A. Chronic nonspecific B. Acute specific


C. Acute nonspecific D. Chronic specific
E. Selective

10. During medical examination of high and middle school students, the doctors vere assessing correlation
between biological and calendar age of the school students based on the following criteria: height growth rate
per year, ossification of the carpal bones, the number of permanent teeth. What additional development
criterion should be assessed at this age?

K. Development = of secondary sex


characteristics
L. Body mass
M. Chest circumference
N. Vital capacity of lungs
O. Hand strength

11. During analysis of morbidity in the city, 1t was determined that age structure of population is different in
each district. What statistical method allows to exclude this factor, so that it would not skew the morbidity
data?

K. Standardization
L. Wilcoxon signed-rank test
M. Correlation-regression analysis
N. Dynamic time series analysis
O. Analysis of average values
12. Clinical statistical investigation was performed to determine effectiveness of a new pharmacological
preparation for patients with ischemic heart disease. What parametric test (coefficient) can be used to estimate
the reliability of the results?

K. Student’s t-distribution
L. Sign test
M. Matching factor
N. Wilcoxon signed-rank test
O. Kolmogorov-Smirmnov test

13. In a rural health care area there is an increasing cervical cancer morbidity observed. The decision is made
to conduct a medical examination of the women living in this locality. What type of medical examination is
it?
K. Target
L. Preliminary
M. Regular
N. Complex
O. Screening

14. In the process of hiring, a prospective employee has undergone preventive medical examination and was
declared fit to work in this manufacturing environment. What type of preventive medical examination was it?

K. Preliminary
L. Scheduled
M. Periodical
N. Specific
O. Comprehensive

15. On laboratory investigation of a pork sample there is 1 dead trichinella detected in 24 sections. This meat
should be:

E. Sent for technical disposal


F. Allowed for sale with no restrictions C. Processed and sold through public catering network
D. Processed for boiled sausage production E. Frozen until the temperature of -10°C 1s reached in the deep
layers, with subsequent exposure to cold for 15 days

16. To assess the effectiveness of medical technologies and determine the power and direction of their effect
on the public health indicators, the research was conducted to study the immunization rate of children and
measles incidence rate by district. What method of statistical analysis should be applied in this case?

A. Calculation of correlation coefficient B. Calculation of morbidity index among the nonvaccinated C.


Calculation of matching factor
D. Calculation of standardized ratio E. Calculation of statistical significance of the difference between two
estimates

17. Having studied the relationship between the distance from villages to the local outpatient clinics and
frequency of visits to the clinics among the rural population of this area, it was determined that the rank
correlation coefficient in this case equals -0.9. How can this relationship be characterized?

K. Strong inverse relationship


L. Strong direct relationship
M. Moderate inverse relationship
N. Moderate direct relationship
O. -
18. In the inpatient gynecological unit within a year 6500 women underwent treatment. They spent there a
total of 102000 bed-days. What indicator of the gynecological unit work can be calculated based on these
data?

K. Average length of inpatient stay


L. Average bed occupancy rate per year
M. Number of beds by hospital department
N. Bed turnover rate
O. Planned bed occupancy rate per year

19. A middle school teacher with 4-yearlong record of work was issued a medical certificate for pregnancy
and childbirth leave. What amount of pay will she receive for the duration of her leave in this case?

K. 100% of average salary


L. 50% of average salary
M. 70% of average salary
N. 60% of average salary
O. 80% of average salary

20. In the air of the feed kitchen at the poultry factory, at the area where formula feed is being mixed, the dust
concentration reaches 200 mg/m3. Air microflora is represented predominantly by Aspergillus and Mucor
fungi. What effect determines pathogenic properties of the dust?

K. Allergenic
L. Teratogenic
M. Mutagenic
N. Fibrogenic
O. Toxic

21. During regular medical examination a lyceum student presents with signs of cheilitis that manifests as
epithelial maceration in the area of lip seal. The lips are brightred, with single vertical cracks covered with
brown-red scabs. These clinical signs are most likely caused by insufficient content of the following in the
diet:

K. Riboflavin
L. Ascorbic acid
M. Retinol
N. Thiamine
O. Calciferol

22. A 30-year-old woman made an appointment with the family doctor for - scheduled
vaccination of her 2-year-old child. What type of healthcare provides such medical services?

A. Primary healthcare
B. Emergency aid
C. Secondary healthcare
D. Tertiary healthcare
E. Palliative care
23. Human body receives from the atmosphere a number of chemicals. What type of action
results in the combined effect that is less than the sum of isolated effects of these chemicals on the body?

A. Antagonism
B. Potentiation
C. Isolated action
D. Synergistic action
E. Complex action
24. Clinicaltrials have proved the *Lipoflavon” drug to be effective for
treatment of unstable angina pectoris in the control group and experimental group of patients. Neither
patients nor researchers knew who belonged to which group. Name this type of study:

A. Double blind study


B. Simple blind study
C. Triple-blind study
D. Total-blind study
E. Multicenter study

You might also like